sesap

March 29, 2018 | Author: Max Sunset | Category: Metastasis, Head And Neck Cancer, Biopsy, Cancer, Neoplasms


Comments



Description

American College of SurgeonsSESAP 13 Category 1 Item 1 A 45-year-old man presents with a tender 3-cm soft midline neck mass. Ultrasound study reveals a thyroglossal duct cyst. The thyroid is normal, and there is no lymphadenopathy. A Sistrunk procedure is performed. Intraoperatively, the thyroid is visually and palpably normal, and there is no palpable adenopathy. Pathologic evaluation of the specimen reveals a 5-mm focus of welldifferentiated papillary carcinoma within the wall of the cyst. Margins are uninvolved. The next step in management should be (A) observation (B) isthmusectomy (C) subtotal thyroidectomy with central neck dissection (D) total thyroidectomy followed by radioactive iodine ablation therapy (E) radioactive iodine ablation therapy without further operation page 1 American College of Surgeons SESAP 13 Critique for Category 1 Item 1 The most common congenital cystic lesion of the neck is the thyroglossal duct cyst, comprising 70% of all congenital abnormalities of the neck. It presents most commonly in the first decade of life as a cystic anterior midline neck mass. It is usually located at or below the level of the hyoid bone; however, it may be found anywhere between the base of the tongue and the suprasternal notch. The thyroid gland develops from a midline endodermal invagination of the foregut (foramen cecum). Thyroglossal duct cysts are remnants of the tract along which the thyroid gland descends. Indications for operation include increasing size, the risk for cyst infection, or suspicion of carcinoma. High-resolution ultrasound study can be useful for preoperative evaluation to confirm the diagnosis, to confirm the presence of a normal thyroid gland, and to detect possible malignant change within the cyst (solid component). The classic treatment is the Sistrunk procedure, which is based on the embryogenesis of the thyroid gland, tongue, and hyoid bone. The cyst is completely excised in continuity with its tract, the central portion of the hyoid bone, and the tissue above the hyoid bone extending to the base of the tongue. A lesser procedure, such as simple cyst excision, is associated with a high risk of recurrence. Approximately 1% of thyroglossal duct cysts harbor carcinoma. The most common type is thyroid papillary carcinoma (80%), followed by squamous cell carcinoma. Most thyroglossal duct carcinoma presents as an asymptomatic mass and is diagnosed incidentally from histopathologic evaluation of the surgical specimen after surgical excision of the thyroglossal duct cyst. The mean age of presentation is the fourth decade of life. Women are affected slightly more often than men. The Sistrunk operation is adequate for treatment of most thyroglossal duct cyst tumors, which are incidentally discovered well-differentiated papillary thyroid carcinomas associated with a normal thyroid gland. More aggressive therapy, which may include total thyroidectomy with or without neck dissection, radioactive iodine ablation therapy, and TSH suppression, is required with more advanced disease. Prognosis is excellent with adequately treated disease. REFERENCES 1.Ahuja AT, Wong KT, King AD, Yuen EH: Imaging for thyroglossal duct cyst: the bare essentials. Clin Radiol 60:141-148, 2005 2.Motamed M, McGlashan JA: Thyroglossal duct carcinoma. Curr Opin Otolaryngol Head Neck Surg 12:106-109, 2004 3.Roseman BJ, Clark OH: Neck mass, in Souba WW, Fink MP, Jurkovich GJ, Kaiser LR, et al (eds): ACS Surgery: Principles and Practice 2006. New York, WebMD, Head and Neck, sec 3, chap 2, pp 158-169 4.Warner BW: Pediatric surgery, in Townsend CM Jr (ed): Sabiston Textbook of Surgery: The Biological Basis of Modern Surgical Practice, ed 17. Philadelphia, Saunders, 2004, pp 2097-2135 page 2 American College of Surgeons SESAP 13 Category 1 Item 2 A 54-year-old man with a 30-pack year history of smoking and heavy alcohol use has a painless 2cm nodule on the anterior border of the sternocleidomastoid inferior to the angle of the mandible. Which of the following would be the MOST appropriate means of obtaining a tissue diagnosis? (A) Endoscopic biopsy (B) Fine-needle aspiration (FNA) (C) Core needle biopsy (D) Incisional biopsy (E) Excisional biopsy page 3 American College of Surgeons SESAP 13 Critique for Category 1 Item 2 The patient described presents with a solitary neck mass. A systematic approach is required for diagnosis of the etiology of such a lesion. History and physical examination can often point to the underlying cause, as seen in the chart. In this particular case, several items suggest that the mass is a metastatic squamous cell neoplasm originating from the upper aerodigestive tract: older adult age (malignancy), male sex (squamous cell cancer), history of tobacco and heavy alcohol use (squamous cell cancer), upper neck location (metastatic head and neck cancer), painless character (metastatic disease), and unilateral appearance (malignancy). Other important factors to consider would include history of head and neck cancer, rapid growth, fixed location, and upper aerodigestive tract symptoms, eg, odynophagia, otalgia, hoarseness. Tissue sampling can often establish the diagnosis, assisting in therapeutic planning. Fine-needle aspiration (FNA) remains the first-line approach because of its safety, accuracy (a definitive diagnosis can be made in 90% of cases), and simplicity. Because of the risk of poor wound healing and possible spread of disease, FNA is always preferred over open biopsy for diagnosing squamous cell carcinoma. Although core needle biopsy using an 18-gauge needle with ultrasound guidance has been advocated for diagnosis of suspected lymphomas, an initial FNA is often used to help differentiate whether the process is lymphoproliferative or frankly malignant. Due to poor sampling, however, open biopsy is required in about 10% of all malignant masses. Combining FNA with ultrasound guidance may help reduce this percentage. Ultrasound-guided FNA has higher accuracy (24.3 %) and results in better sampling (84% reduction in nondiagnostic results) compared with traditional palpation-guided FNA. Panendoscopy (bronchoscopy, endoscopy, and laryngoscopy) with biopsies is an essential part of the work-up of patients with a suspected head and neck tumor, but it can be nondiagnostic. Incisional and excisional biopsies are modalities used for establishing a tissue diagnosis after repeated nondiagnositic FNA or core sampling. REFERENCES 1.Jereczek-Fossa BA, Jassem J, Orecchia R: Cervical lymph node metastases of squamous cell carcinoma from an unknown primary. Cancer Treat Rev 30:153-164, 2004 2.Roseman BJ, Clark OH: Neck mass, in Souba WW, Fink MP, Jurkovich GJ, Kaiser LR, et al (eds): ACS Surgery: Principles and Practice 2006. New York, WebMD, Head and Neck, sec 3, chap 2, pp 158-169 3.Senchenkov A, Staren ED: Ultrasound in head and neck surgery: thyroid, parathyroid, and cervical lymph nodes. Surg Clin North Am 84:973-1000, 2004 page 4 American College of Surgeons SESAP 13 Category 1 Item 3 A 45-year-old female smoker has a painless 1-cm nodule in her right anterior cervical triangle found to be squamous cell carcinoma. Physical examination of the head and neck is otherwise unremarkable. The most likely source for the primary tumor is (A) tongue (B) tonsil (C) palate (D) pharynx (E) larynx page 5 American College of Surgeons SESAP 13 Critique for Category 1 Item 3 The neck is one of the most common locations for metastatic disease from a carcinoma of unknown primary. Of all head and neck cancers, 2% to 9% present as cervical masses from an unknown primary. The location of the mass in the neck is helpful in determining the potential location of the primary. An upper neck metastasis tends to arise from cancers within the head and neck, whereas lower neck lesions usually result from malignancies located below the clavicles. Specific cancers also tend to produce cervical metastases at specific levels of the neck Given the location of the mass in this patient, an upper aerodigestive primary is the likely source. Computed tomography (CT) of the head and neck can identify a lesion in up to 20% of such cases. When imaging or physical examination is suggestive of the primary location, panendoscopy with biopsies can properly identify the source in about 65% of cases. When neither suggests the location of the primary, the localization rate is 20%. The tonsils are the most common primary source in head and neck cancers from an unknown location. In fact, some experts recommend bilateral tonsillectomy if work-up fails to reveal a primary lesion because up to 25% will have a tonsillar origin. Patients presenting with metastatic nodes in the subdigastric, submandibular, and midjugular region are considered at high risk for such occult tonsillar malignancies. REFERENCES 1.Jereczek-Fossa BA, Jassem J, Orecchia R: Cervical lymph node metastases of squamous cell carcinoma from an unknown primary. Cancer Treat Rev 30:153-164, 2004 2.Koch WM, Bhatti N, Williams MF, Eisele DW: Oncologic rationale for bilateral tonsillectomy in head and neck squamous cell carcinoma of unknown primary source. Otolaryngol Head Neck Surg 124:331-333, 2001 3.Mendenhall WM, Mancuso AA, Parsons JT, Stringer SP, et al: Diagnostic evaluation of squamous cell carcinoma metastatic to cervical lymph nodes from an unknown head and neck primary site. Head Neck 20:739-744, 1998 4.Roseman BJ, Clark OH: Neck mass, in Souba WW, Fink MP, Jurkovich GJ, Kaiser LR, et al (eds): ACS Surgery: Principles and Practice 2006. New York, WebMD, Head and Neck, sec 3, chap 2, pp 158-169 page 6 American College of Surgeons SESAP 13 Category 1 Item 4 A 55-year-old otherwise asymptomatic man presents with a 2-cm ulcerated lesion on his right lateral mobile tongue and a firm 2- by 1-cm mass in the right neck. Which of the following statements is TRUE? (A) Induction therapy with fluorouracil (5-FU) and cisplatin is indicated (B) Five-year survival rate is 60% (C) Adjuvant radiotherapy is indicated (D) Resection with selective neck dissection is indicated (E) Synchronous malignancy in the lung or upper aerodigestive track occurs in 20% of cases page 7 American College of Surgeons SESAP 13 Critique for Category 1 Item 4 This lesion is almost certainly a squamous cell carcinoma of the tongue with a nodal metastasis. Men over 40 with a history of smoking and alcohol intake are at highest risk. Aspiration cytology of the neck mass is useful for diagnosis. Although computed tomographic (CT) scan of the neck may provide further information, clinical staging of this patient is T2N1M0 or stage III. When oral tongue squamous cell carcinoma has progressed to the neck, survival rates decrease by 50%. Progression or recurrence at the primary site, regional neck recurrence, and distant metastases (particularly to the lung) are the most common manifestations of failure of therapy. These patients have a fairly high incidence of other tumors of the upper aerodigestive tract; 4% to 10% have synchronous tumors and 27% have metachronous tumors, most frequently of the lung and esophagus. En bloc resection of the tongue with modified or radical neck dissection (depending on the nature of the cervical lymph node) is the most appropriate treatment for this patient. Because the jugulodigastric and the jugular nodes are most commonly involved in metastasis from oral tongue cancer, the decision whether to remove the jugular vein, accessory nerve, or sternocleidomastoid muscle will be made by analysis of CT findings or by findings at operation. Selective neck dissection, limited to the area at highest likelihood of being involved with the tumor by micro metastasis, is only indicated in the clinically negative N0 neck. In the presence of a palpable positive lymph node, modified or radical neck dissection is required. Adjuvant radiotherapy modestly increases survival of stage III oral cancer and significantly decreases the risk of local regional recurrence, particularly failure in the neck. Although interstitial radiotherapy (implantation of the radioactive sources into the tumor) has been used both as definitive and adjuvant therapy, postoperative external-beam radiation therapy (teletherapy) is the most common form of adjuvant therapy for squamous cell carcinoma of the upper aerodigestive tract. The oral cavity, oropharynx, and neck are treated with 60 to 65 Gy. Adjuvant radiotherapy should be initiated within 6 weeks after operation. Chemotherapy, particularly combinations of cisplatin and its analogs and fluorouracil (5-FU), has been widely used in organ-sparing regimens, especially for the larynx. It has not been shown to increase survival or decrease locoregional failure when used in combination with operation and radiotherapy, and is not part of the standard therapy for resectable stage III disease. REFERENCES 1.Million RR, Cassisi NJ, Mancuso AA: Oral cavity, in Million RR, Cassisi NJ (eds): Management of Head & Neck Cancer: A Multidisciplinary Approach, ed 2. Philadelphia, Lippincott Williams & Wilkins, 1994, pp 321-400 2.Wein RO, Chandra RK, Weber RS: Disorders of the head and neck, in Brunicardi FC, Andersen DK, Billiar TR, Dunn DL, et al (eds): Schwartz’s Principles of Surgery, ed 8. New York, McGraw-Hill, 2005, pp 501-544 page 8 American College of Surgeons SESAP 13 Category 1 Item 5 Lateral aberrant thyroid is (A) abnormally descended thyroid tissue in the lateral neck (B) associated with thyroglossal duct cyst (C) cervical lymph node metastases from papillary cancer (D) cervical lymph node enlargement as a result of Hashimoto’s disease (E) asymmetric goiter protruding predominantly to one side of the neck page 9 American College of Surgeons SESAP 13 Critique for Category 1 Item 5 Lateral aberrant thyroid is cervical lymph node metastases from a papillary carcinoma of the thyroid. Papillary carcinoma most commonly presents as a thyroid nodule. Some lesions may be subclinical. Despite their small size, these lesions may demonstrate metastases. Up to 80% of papillary cancers will be associated with lymph node metastases; however, most are microscopic and do not appear to influence long-term survival. About 35% of lymph node metastases are palpable at the time of presentation, and lateral aberrant thyroid is a subset of these. Because papillary thyroid carcinoma is a well-differentiated tumor, biopsy of a lateral neck mass may appear microscopically as normal thyroid tissue in an abnormal location, thus the term lateral aberrant thyroid. Although lymph node metastasis is not a strong prognostic indicator, the presence of lymph node metastases does influence local recurrence rates. Some authors recommend prophylactic central neck dissection at the time of initial thyroidectomy. Modified neck dissection (sparing the sternocleidomastoid muscle, internal jugular vein, and spinal accessory nerve) is warranted if palpable lymph node metastases are present and clinically are not fixed and at operation are not invading any of these structures. Embryologically, the thyroid descends from the foramen cecum at the base of the tongue to its permanent location in the midline of the neck. Congential anomalies are midline when they occur. A cyst, the most common of which is a thyroglossal duct cyst, may form along this track of descent. However, this is not associated with functional thyroid tissue. Hashimoto’s disease is a benign autoimmune disorder that gradually leads to hypothyroidism. There is no thyroid tissue in the cervical lymph nodes with this disease. By definition, goiter is enlargement of the thyroid itself; it may extend laterally, but there is connection to the central component. REFERENCES 1.De Jong SA, Demeter JG, Jarosz H, Lawrence AM, et al: Primary papillary thyroid carcinoma presenting as cervical lymphadenopathy: the operative approach to the “lateral aberrant thyroid.” Am Surg 59:172-176, 1993 2.Feinmesser R, Smith O, Mullen B, Freeman J: Aberrant thyroid in the lateral neck. Ear Nose Throat J 69:769-770, 1990 3.Nishiyama RH: Overview of surgical pathology of the thyroid gland. World J Surg 24:898-906, 2000 page 10 American College of Surgeons SESAP 13 Category 1 Item 6 Which of the following statements about the treatment of Warthin’s tumor is TRUE? (A) Observation alone is sufficient (B) Surgical enucleation is contraindicated (C) Total parotidectomy involves resection of the facial nerve (D) Postoperative radiation therapy is necessary for high-grade lesions (E) Superficial parotidectomy is inadequate because of the high rate of recurrence page 11 American College of Surgeons SESAP 13 Critique for Category 1 Item 6 Benign salivary gland tumors are uncommon, representing only 2% of all human tumors. The location of tumor occurrence is proportional to gland size, with 65% to 80% found in the parotid gland, 10% in the submandibular gland, and the rest in the minor salivary glands. The likelihood of malignancy follows the opposite pattern and is inversely proportional to gland size: 15% to 30% of parotid tumors are malignant, 40% in the submandibular, and 70% to 90% in the sublingual gland. Pleomorphic adenoma or mixed benign tumor is the most common, making up nearly half of all salivary gland tumors and over 80% of benign lesions. Whereas most are found in the parotid, they are less common in the submandibular and relatively rare in the minor glands. Warthin’s tumor is the second most common (10% to 12% of all benign tumors) salivary gland neoplasm and arises almost exclusively in the parotid. Also known as papillary cystadenoma lymphomatosum, this tumor is always benign and has a 2% recurrence rate after resection. Despite a slight female preponderance among most salivary gland tumors, for unknown reasons Warthin’s tumor is much more common in males (by 5:1). Up to 10% are bilateral or multifocal. Smokers are at 8-fold greater risk for developing Warthin’s tumor. Among malignant lesions, mucoepidermoid carcinoma is the most common, representing 15% of all salivary tumors. These are usually found in the parotid and have a variable clinical course. Low-grade lesions invade locally and recur in 15% of cases, but they rarely metastasize (5 year survival > 90%), Conversely, high-grade lesions are difficult to excise, recur in 30% of cases, and metastasize in 30% (5 year survival ~50%). All parotid tumors have a similar clinical presentation, regardless of histologic pattern. They produce a distinctive, painless swelling anterior to the ear that is usually mobile to palpation. Often 4 to 6 cm in diameter at the time of diagnosis, parotid neoplasms (both benign and malignant) grow slowly and have usually been present a long time prior to presentation. Lesions with acute onset or painful swelling are usually inflammatory or due to duct obstruction. Because no clinical criteria are available to differentiate benign from malignant lesions, cytologic evaluation is necessary, either fine-needle aspiration biopsy or parotidectomy. The recommended treatment of all benign salivary neoplasms, including Warthin’s tumor, is complete excision to avoid recurrences. Enucleation is not recommended because of the high recurrence rate. In the parotid, tumor excision should include a cuff of normal tissue and careful dissection and preservation of the facial nerve unless malignancy is confirmed on permanent section. Suspicion of malignancy on frozen section does not warrant sacrifice of the facial nerve. Even larger lesions confined to the superficial lobe may be adequately treated with a superficial parotidectomy. However, tumors extending to the deep lobe will require total parotidectomy with facial nerve preservation. For lesions in the submandibular and minor salivary glands, complete excision of gland is required. Inadequate excision or tumor multicentricity may explain tumor recurrence. Malignant lesions are treated with en bloc surgical excision. High-grade tumors and those demonstrating extraglandular disease, perineural invasion, or regional metastases should also receive postoperative radiation therapy. For low-grade parotid malignancies confined to the superficial lobe, a lateral lobectomy with facial nerve preservation is adequate. In the case of gross facial nerve involvement, a temporal bone resection should be performed and the nerve sacrificed proximally to obtain a negative margin. page 12 American College of Surgeons SESAP 13 REFERENCES 1.Hanna EY, Lee, S, Fan CY, Suen JY: Benign neoplasms of the salivary glands, in Cummings CW, Flint PW, Haughey BH, et al: Otolaryngology: Head & Neck Surgery, ed 4. St. Louis, Mosby, 2005, pp 1348-1375 2.Khatri VP, Asensio JA: Parotidectomy, in Khatri VP, Asensio JA (eds): Operative Surgery Manual. Philadelphia, Saunders, 2003, pp 21-26 3.Lingen MW, Kumar V: Head and neck, in Kumar V, Abbas AK, Fausto N (eds): Robbins & Cotran Pathologic Basis of Disease, ed 7. Philadelphia, Elsevier Saunders, 2005, pp 773-796 4.Lorenz RR, Netterville JL, Burkey BB: Head and neck, in Townsend CM Jr (ed): Sabiston Textbook of Surgery: The Biological Basis of Modern Surgical Practice, ed 17. Philadelphia, Saunders, 2004, pp 833-866 page 13 American College of Surgeons SESAP 13 Category 1 Item 7 A 76-year-old man has a slowly growing lump in his left neck, anterior to the sternocleidomastoid muscle, just below the angle of the mandible. He has no history of cancer and no constitutional symptoms. Cytology from fine-needle aspiration is consistent with squamous cell carcinoma. The test most likely to identify the primary lesion would be (A) chest x-ray (B) 2-[18F] fluoro-2-deoxy-D-glucose/positron emission tomography (FDG-PET) scan (C) contrast-enhanced computed tomography (CT) of the soft tissues of the neck (D) magnetic resonance imaging (MRI) of the head and neck (E) panendoscopy under anesthesia with random biopsie page 14 American College of Surgeons SESAP 13 Critique for Category 1 Item 7 Diagnosis begins with a thorough examination of the upper aerodigestive tract. Important risk factors include history of tobacco and alcohol use as well as sun exposure. Signs and symptoms of otalgia, trismus, hoarseness, dysphagia, and weight loss are clinical clues that can direct further evaluation. Physical examination should include palpation of the entire neck, tonsillar fossae, and base of the tongue. Additional information can possibly be gained from flexible fiberoptic endoscopy and fineneedle aspiration of the node should be obtained if not done previously. The specific location of the node helps to locate primary sites because specific regions of the aerodigestive tract usually drain to specific nodal basins. The anterior triangle of the neck contains lymph node regions II, III, and IV, whereas the posterior triangle contains lymph node region V. The central lymph node groups are I, VI, and VII. Region I nodes are located in the submental triangle and region VI nodes are centrally located, medial to the lateral borders of the strap muscles. Region VII nodes are posterior to the manubrium. Nodal involvement from occult squamous cell carcinoma, as in this case, typically arises in the jugulodigastric nodes. Jugulodigastric nodes in region II typically drain the soft palate, tonsil, base of tongue, pyriform sinus, and supraglottic larynx. The middle jugular nodes (region III) drain the supraglottic larynx, inferior piriform sinus, and postcricoid region, and the most inferior jugular nodes (region IV) drain the thyroid, trachea, and cervical esophagus. Supraclavicular nodes also drain regions below the clavicle; nodes found here are more likely to have arisen from primaries below than above the clavicles. Adenopathy from a primary located in the nasopharynx typically arises in the posterior triangle (region V). There is significant overlap and skip lesions can occur. Regions II and III tend to be involved more frequently than the others, and lesions are more commonly unilateral than bilateral (10%). Radiographic analysis is required and should include computed tomography (CT) of the soft tissues of the neck and chest x-ray or CT of the chest, the latter especially with supraclavicular nodes. CT scans have a detection rate of 15% to 20%. Fluorodeoxyglucose-positron emission tomography (FDGPET) has had mixed results in identifying the primary tumor, with rates ranging from 8% to 42%. The resolution of FDG-PET is limited to 5 to 10 mm or larger, and this study is unable to specifically localize the tumor. PET-CT provides some improvement, but the overall consensus seems to be that FDG-PET imaging is not a required modality for evaluation of unknown primaries. With adequate history, physical, and imaging, primaries have been detected in up to 50% of cases without further work-up. The remaining patients should undergo further diagnostic modalities. Most agree that panendoscopy with directed random biopsies should be performed to evaluate for unknown primaries if standard imaging and office evaluation fails. Specifically, bronchoscopy, rigid cervical esophagoscopy, direct laryngoscopy, and evaluation of the nasopharynx should be performed with directed biopsies of specific sites. Biopsy sites should include the nasopharynx, hypopharynx, pyriform sinus, postcricoid region, tonsils, and base of the tongue. Physical examination and panendoscopy with biopsies detects 65% of primary tumors. Recent studies have shown that the most common sites for the primary lesion are the tonsillar fossa or base of tongue (82%), not the nasopharynx or hypopharynx. Some studies advocate bilateral tonsillectomy because morbidity is not significantly increased in bilateral versus unilateral procedures. Treatment is controversial and may depend on nodal staging, but similar outcomes (30% to 50% 5year survival) have been found regardless of therapy. Single modality therapy, recommended for N1 or N2a, may be either neck dissection alone or radiation therapy alone. If extracapsular spread is page 15 American College of Surgeons SESAP 13 found after neck dissection, most would treat that side with radiation therapy also. The optimal radiation field is controversial. For neck dissection, selective dissection of the involved site with contiguous sites and involved structures seems to be appropriate. Combined therapy is also controversial, in that the optimal timing of radiation therapy, whether prior to or after neck dissection, is not known. Chemotherapy can be recommended with distant metastasis or inoperable disease, but the results are mixed. Outcome is related to many different variables, the most important being nodal status. Some studies show no difference in outcome regardless of primary lesion with similar nodal stage, while others report poorer outcomes when the primary is discovered. Other prognostically important factors include number of lymph nodes, grading, extracapsular extension, patient age, and weight loss before treatment. REFERENCES 1.Grau C, Johansen LV, Jakobsen J, Geertsen P, et al: Cervical lymph node metastases from unknown primary tumours. Results from a national survey by the Danish Society for Head and Neck Oncology. Radiother Oncol 55:121-129, 2000 2.Jereczek-Fossa BA, Jassem J, Orecchia R: Cervical lymph node metastases of squamous cell carcinoma from an unknown primary. Cancer Treat Rev 30:153-164, 2004 3.Jungehulsing M, Scheidhauer K, Damm M, Pietrzyk U, et al: 2[F]-fluoro-2-deoxy-D-glucose positron emission tomography is a sensitive tool for the detection of occult primary cancer (carcinoma of unknown primary syndrome) with head and neck lymph node manifestation. Otolaryngol Head Neck Surg 123:294-301, 2000 4.Mendenhall WM, Mancuso AA, Amdur RJ, Stringer SP, et al: Squamous cell carcinoma metastatic to the neck from an unknown head and neck primary site. Am J Otolaryngol 22:261-267, 2001 5.Orel SG, Weinstein SP, Schnall MD, Reynolds CA, et al: Breast MR imaging in patients with axillary node metastases and unknown primary malignancy. Radiology 212:543-549, 1999 6.Randall DA, Johnstone PA, Foss RD, Martin PJ: Tonsillectomy in diagnosis of the unknown primary tumor of the head and neck. Otolaryngol Head Neck Surg 122:52-55, 2000 7.Sinnathamby K, Peters LJ, Laidlaw C, Hughes PG: The occult head and neck primary: to treat or not to treat? Clin Oncol 9:322-329, 1997 page 16 American College of Surgeons SESAP 13 Category 1 Item 8 A 65-year-old man has had a painless, gradually enlarging mass in the region of the left parotid for 6 months, as shown. He can no longer depress the left corner of his mouth or close his left eye fully. He is a heavy smoker and drinker, but has no aerodigestive tract symptoms. Examination reveals a firm, nontender mass overlying the left angle of the jaw, facial asymmetry with left facial weakness, and no cervical lymphadenopathy. The next step should be (A) superficial parotidectomy (B) radical parotidectomy with neck dissection (C) MRI of the head and neck and fine-needle aspiration (FNA) (D) panendoscopy (E) metastatic work-up page 17 American College of Surgeons SESAP 13 Critique for Category 1 Item 8 One in eight patients with parotid tumors presents with facial nerve paralysis. Such a presentation indicates a poor prognosis, with a mean survival of less than 3 years from the onset of the paralysis. Such tumors are usually high-grade mucoepidermoid carcinomas or adenoid cystic carcinomas; the former have a 1 in 4 chance of occult nodal metastases at diagnosis. Magnetic resonance imaging (MRI) and fine-needle aspiration (FNA) are the best first management steps, to determine nerve involvement and rule out the rare cases of benign tumors or metastases to the gland. Panendoscopy would be helpful in ruling out a head and neck cancer, but not as a first step in this clinical presentation. Metastatic work-up might be necessary depending on what FNA reveals. Operation usually consists of radical parotidectomy. Nerve sacrifice with reconstruction (often using the sural nerve) will likely be required. The benefit of even selected neck dissection is more controversial and may be obviated by the fact that most of these patients will require postoperative radiation therapy. Chemotherapy does not yet have a defined role in such highly malignant parotid tumors. REFERENCES 1.Hocwald E, Korkmaz H, Yoo GH, Adsay V, et al: Prognostic factors in major salivary gland cancer. Laryngoscope 111:1434-1439, 2001 2.Shannon KF, O’Brien CJ: Parotid gland tumor, in Khatri VP (ed): Clinical Scenarios in Surgical Oncology. Philadelphia, Lippincott Williams& Wilkins, 2006, pp 23-28 page 18 American College of Surgeons SESAP 13 Category 1 Item 9 Derived from thyroid c-cells (A) Papillary thyroid cancer (B) Follicular thyroid cancer (C) Medullary thyroid cancer (D) Anaplastic thyroid cancer (E) Follicular variant of papillary thyroid cancer page 19 American College of Surgeons SESAP 13 Critique for Category 1 Item 9 Papillary thyroid cancer is the most common thyroid cancer, comprising over 70% of all cases. The second most common type is follicular thyroid cancer. Both papillary and follicular thyroid cancers are derived from thyroid follicular cells and are often grouped together as well-differentiated thyroid cancers. Medullary thyroid cancer is a neuroendocrine tumor derived from thyroid c-cells. Whereas 80% of medullary thyroid cancers are of the sporadic variety, 20% are associated with mutations in the RET proto-oncogene. These familial medullary cancer syndromes include familial medullary thyroid cancer, multiple endocrine neoplasia type 2A (MEN2A), and multiple endocrine neoplasia type 2B (MEN2B). Anaplastic thyroid cancer is the least common and most aggressive thyroid cancer, with a median survival of less than 6 months. Treatment options include external-beam radiation and/or chemotherapy. Follicular variant of papillary thyroid cancer is a subtype of papillary thyroid cancer. Although most thyroid cancers can be diagnosed by fine-needle aspiration (FNA), follicular thyroid cancers cannot. Follicular thyroid cancers have a benign counterpart called follicular adenomas. The only difference between a follicular thyroid cancer and a follicular adenoma is a vascular or capsular invasion, which can only be seen on permanent histology. REFERENCES 1.Chen H, Nicol TL, Udelsman R: Follicular lesions of the thyroid. Does frozen section evaluation alter operative management? Ann Surg 222:101-106, 1995 2.Udelsman R, Chen H: The current management of thyroid cancer. Adv Surg 33:1-27, 1999 page 20 American College of Surgeons SESAP 13 Category 1 Item 10 Thyroidectomy not indicated (A) Papillary thyroid cancer (B) Follicular thyroid cancer (C) Medullary thyroid cancer (D) Anaplastic thyroid cancer (E) Follicular variant of papillary thyroid cancer page 21 American College of Surgeons SESAP 13 Critique for Category 1 Item 10 Papillary thyroid cancer is the most common thyroid cancer, comprising over 70% of all cases. The second most common type is follicular thyroid cancer. Both papillary and follicular thyroid cancers are derived from thyroid follicular cells and are often grouped together as well-differentiated thyroid cancers. Medullary thyroid cancer is a neuroendocrine tumor derived from thyroid c-cells. Whereas 80% of medullary thyroid cancers are of the sporadic variety, 20% are associated with mutations in the RET proto-oncogene. These familial medullary cancer syndromes include familial medullary thyroid cancer, multiple endocrine neoplasia type 2A (MEN2A), and multiple endocrine neoplasia type 2B (MEN2B). Anaplastic thyroid cancer is the least common and most aggressive thyroid cancer, with a median survival of less than 6 months. Treatment options include external-beam radiation and/or chemotherapy. Follicular variant of papillary thyroid cancer is a subtype of papillary thyroid cancer. Although most thyroid cancers can be diagnosed by fine-needle aspiration (FNA), follicular thyroid cancers cannot. Follicular thyroid cancers have a benign counterpart called follicular adenomas. The only difference between a follicular thyroid cancer and a follicular adenoma is a vascular or capsular invasion, which can only be seen on permanent histology. REFERENCES 1.Chen H, Nicol TL, Udelsman R: Follicular lesions of the thyroid. Does frozen section evaluation alter operative management? Ann Surg 222:101-106, 1995 2.Udelsman R, Chen H: The current management of thyroid cancer. Adv Surg 33:1-27, 1999 page 22 American College of Surgeons SESAP 13 Category 1 Item 11 Associated with multiple endocrine neoplasia type 2 (MEN2) (A) Papillary thyroid cancer (B) Follicular thyroid cancer (C) Medullary thyroid cancer (D) Anaplastic thyroid cancer (E) Follicular variant of papillary thyroid cancer page 23 American College of Surgeons SESAP 13 Critique for Category 1 Item 11 Papillary thyroid cancer is the most common thyroid cancer, comprising over 70% of all cases. The second most common type is follicular thyroid cancer. Both papillary and follicular thyroid cancers are derived from thyroid follicular cells and are often grouped together as well-differentiated thyroid cancers. Medullary thyroid cancer is a neuroendocrine tumor derived from thyroid c-cells. Whereas 80% of medullary thyroid cancers are of the sporadic variety, 20% are associated with mutations in the RET proto-oncogene. These familial medullary cancer syndromes include familial medullary thyroid cancer, multiple endocrine neoplasia type 2A (MEN2A), and multiple endocrine neoplasia type 2B (MEN2B). Anaplastic thyroid cancer is the least common and most aggressive thyroid cancer, with a median survival of less than 6 months. Treatment options include external-beam radiation and/or chemotherapy. Follicular variant of papillary thyroid cancer is a subtype of papillary thyroid cancer. Although most thyroid cancers can be diagnosed by fine-needle aspiration (FNA), follicular thyroid cancers cannot. Follicular thyroid cancers have a benign counterpart called follicular adenomas. The only difference between a follicular thyroid cancer and a follicular adenoma is a vascular or capsular invasion, which can only be seen on permanent histology. REFERENCES 1.Chen H, Nicol TL, Udelsman R: Follicular lesions of the thyroid. Does frozen section evaluation alter operative management? Ann Surg 222:101-106, 1995 2.Udelsman R, Chen H: The current management of thyroid cancer. Adv Surg 33:1-27, 1999 page 24 American College of Surgeons SESAP 13 Category 1 Item 12 Embolization therapy (A) Figure A (B) Figure B (C) Both (D) Neither page 25 American College of Surgeons SESAP 13 Critique for Category 1 Item 12 Patient (A) has an arteriovenous malformation of the external carotid artery system. Patient (B) has Sturge-Weber syndrome, the external manifestation of which is a port wine stain found at birth over the ophthalmic, maxillary, and sometimes mandibular cutaneous distributions of the trigeminal nerve. Arteriovenous malformations (AVM) are usually congenital, but may occasionally be related to trauma. They usually increase in size with time and may recruit new vascular territories, particularly if they are incompletely resected. Definitive therapy almost always requires total excision because persistence of any abnormal malformation will usually quickly enlarge and may recruit new vascular territories into the lesion. Embolization therapy may be useful immediately prior to operation to decrease blood loss, but is rarely adequate as definitive therapy because of the copious collateral circulation in the head and neck. External carotid ligation is almost never useful for the same reason. High-flow AVM may result in congestive heart failure because of the increased cardiac work required to compensate for vascular shunting through the malformation. The port wine stain characteristic of Sturge-Weber is a congenital vascular hamartoma. Initially superficial and of low flow, it tends to hypertrophy with time into a verruciform pattern and may cause deformity of the affected structures, usually by gravitational deformation. Although resection of the entire lesion is seldom useful, selective resection or debulking may be appropriate to improve function or appearance. Intracranial calcifications and other deformities usually result in mental retardation and seizure activity, and blindness often supervenes from glaucoma. Because this is a low flow malformation, embolization is not indicated and there is certainly no indication for ligation of the external carotid artery. REFERENCES 1.Hurwitz DJ, Kerber CW: Hemodynamic considerations in the treatment of arteriovenous malformations of the face and scalp. Plast Reconstr Surg 67:421-434, 1981 2.Leikensohn JR, Epstein LI, Vasconez LO: Superselective embolization and surgery of noninvoluting hemangiomas and A-V malformations. Plast Reconstr Surg 68:143-152, 1981 3.Mulliken JB: Cutaneous vascular anomalies, in McCarthy JG (ed): Plastic Surgery. Philadelphia, WB Saunders, 1990, pp 3191-3274 page 26 American College of Surgeons SESAP 13 Category 1 Item 13 Congestive heart failure (A) Figure A (B) Figure B (C) Both (D) Neither page 27 1990. embolization is not indicated and there is certainly no indication for ligation of the external carotid artery. and blindness often supervenes from glaucoma. usually by gravitational deformation. Plast Reconstr Surg 68:143-152.Hurwitz DJ. Patient (B) has Sturge-Weber syndrome. Definitive therapy almost always requires total excision because persistence of any abnormal malformation will usually quickly enlarge and may recruit new vascular territories into the lesion.American College of Surgeons SESAP 13 Critique for Category 1 Item 13 Patient (A) has an arteriovenous malformation of the external carotid artery system. High-flow AVM may result in congestive heart failure because of the increased cardiac work required to compensate for vascular shunting through the malformation. The port wine stain characteristic of Sturge-Weber is a congenital vascular hamartoma. Epstein LI. and sometimes mandibular cutaneous distributions of the trigeminal nerve. in McCarthy JG (ed): Plastic Surgery.Mulliken JB: Cutaneous vascular anomalies. REFERENCES 1. it tends to hypertrophy with time into a verruciform pattern and may cause deformity of the affected structures. Because this is a low flow malformation. Philadelphia. maxillary. Embolization therapy may be useful immediately prior to operation to decrease blood loss. the external manifestation of which is a port wine stain found at birth over the ophthalmic. WB Saunders. Vasconez LO: Superselective embolization and surgery of noninvoluting hemangiomas and A-V malformations. Plast Reconstr Surg 67:421-434. selective resection or debulking may be appropriate to improve function or appearance. Initially superficial and of low flow. Intracranial calcifications and other deformities usually result in mental retardation and seizure activity. Although resection of the entire lesion is seldom useful. 1981 2. but may occasionally be related to trauma. External carotid ligation is almost never useful for the same reason. pp 3191-3274 page 28 . Arteriovenous malformations (AVM) are usually congenital. but is rarely adequate as definitive therapy because of the copious collateral circulation in the head and neck. 1981 3. particularly if they are incompletely resected.Leikensohn JR. They usually increase in size with time and may recruit new vascular territories. Kerber CW: Hemodynamic considerations in the treatment of arteriovenous malformations of the face and scalp. American College of Surgeons SESAP 13 Category 1 Item 14 Surgical debulking (A) Figure A (B) Figure B (C) Both (D) Neither page 29 . Plast Reconstr Surg 68:143-152. Kerber CW: Hemodynamic considerations in the treatment of arteriovenous malformations of the face and scalp. pp 3191-3274 page 30 . REFERENCES 1. Because this is a low flow malformation. and sometimes mandibular cutaneous distributions of the trigeminal nerve. Although resection of the entire lesion is seldom useful. Plast Reconstr Surg 67:421-434. it tends to hypertrophy with time into a verruciform pattern and may cause deformity of the affected structures. Embolization therapy may be useful immediately prior to operation to decrease blood loss. maxillary.Leikensohn JR. Patient (B) has Sturge-Weber syndrome. usually by gravitational deformation. 1981 2. 1981 3. Definitive therapy almost always requires total excision because persistence of any abnormal malformation will usually quickly enlarge and may recruit new vascular territories into the lesion. and blindness often supervenes from glaucoma. WB Saunders. selective resection or debulking may be appropriate to improve function or appearance. Epstein LI. embolization is not indicated and there is certainly no indication for ligation of the external carotid artery. Intracranial calcifications and other deformities usually result in mental retardation and seizure activity. High-flow AVM may result in congestive heart failure because of the increased cardiac work required to compensate for vascular shunting through the malformation. Arteriovenous malformations (AVM) are usually congenital. the external manifestation of which is a port wine stain found at birth over the ophthalmic. particularly if they are incompletely resected. External carotid ligation is almost never useful for the same reason. Vasconez LO: Superselective embolization and surgery of noninvoluting hemangiomas and A-V malformations. Initially superficial and of low flow. but is rarely adequate as definitive therapy because of the copious collateral circulation in the head and neck. in McCarthy JG (ed): Plastic Surgery.Mulliken JB: Cutaneous vascular anomalies.American College of Surgeons SESAP 13 Critique for Category 1 Item 14 Patient (A) has an arteriovenous malformation of the external carotid artery system. They usually increase in size with time and may recruit new vascular territories. 1990.Hurwitz DJ. Philadelphia. but may occasionally be related to trauma. The port wine stain characteristic of Sturge-Weber is a congenital vascular hamartoma. American College of Surgeons SESAP 13 Category 1 Item 15 Ligation of external carotid artery (A) Figure A (B) Figure B (C) Both (D) Neither page 31 . Plast Reconstr Surg 68:143-152.Leikensohn JR. Philadelphia. Patient (B) has Sturge-Weber syndrome. the external manifestation of which is a port wine stain found at birth over the ophthalmic. but is rarely adequate as definitive therapy because of the copious collateral circulation in the head and neck.Hurwitz DJ. High-flow AVM may result in congestive heart failure because of the increased cardiac work required to compensate for vascular shunting through the malformation. Vasconez LO: Superselective embolization and surgery of noninvoluting hemangiomas and A-V malformations. particularly if they are incompletely resected. WB Saunders. Definitive therapy almost always requires total excision because persistence of any abnormal malformation will usually quickly enlarge and may recruit new vascular territories into the lesion. maxillary. but may occasionally be related to trauma. Intracranial calcifications and other deformities usually result in mental retardation and seizure activity.Mulliken JB: Cutaneous vascular anomalies. Kerber CW: Hemodynamic considerations in the treatment of arteriovenous malformations of the face and scalp. REFERENCES 1. it tends to hypertrophy with time into a verruciform pattern and may cause deformity of the affected structures. Arteriovenous malformations (AVM) are usually congenital. The port wine stain characteristic of Sturge-Weber is a congenital vascular hamartoma. 1981 2. They usually increase in size with time and may recruit new vascular territories. Because this is a low flow malformation. and sometimes mandibular cutaneous distributions of the trigeminal nerve. usually by gravitational deformation. Initially superficial and of low flow. Embolization therapy may be useful immediately prior to operation to decrease blood loss. Although resection of the entire lesion is seldom useful. 1990.American College of Surgeons SESAP 13 Critique for Category 1 Item 15 Patient (A) has an arteriovenous malformation of the external carotid artery system. Plast Reconstr Surg 67:421-434. pp 3191-3274 page 32 . 1981 3. Epstein LI. and blindness often supervenes from glaucoma. embolization is not indicated and there is certainly no indication for ligation of the external carotid artery. in McCarthy JG (ed): Plastic Surgery. External carotid ligation is almost never useful for the same reason. selective resection or debulking may be appropriate to improve function or appearance. Which of the following statements is NOT true? (A) The local recurrence rate is higher for this procedure than for a nonskin-sparing mastectomy (B) SSM allows the native skin envelope to be used for immediate reconstruction (C) If she smokes. she is at higher risk for flap necrosis (D) The prior excisional biopsy site must be completely excised to minimize local recurrence (E) Nipple-areolar excision is necessary page 33 .American College of Surgeons SESAP 13 Category 2 Item 1 A 35-year-old woman has a 1-cm infiltrating ductal grade I carcinoma of her right breast diagnosed by excisional biopsy. She is requesting a skin-sparing mastectomy (SSM) and sentinel lymph node biopsy with immediate reconstruction. et al: Skin-sparing mastectomy.Bleicher RJ. Ann Surg 225: 570-578. Selection bias is one major limitation to the retrospective studies because most included patients with T1 and T2 tumors. 1997 3.Carlson GW. Arch Surg 136:1069-1075. in DeVita VT Jr. Tissue flap necrosis is a potential complication and is related to the patient’s history of smoking as well as previous incisions for reduction mammoplasties. To better ensure negative pathologic margins. pp 1-10 2. Rosenberg SA (eds): Cancer: Principles and Practice of Oncology. In performing the operation. Some surgeons excise core biopsy sites to minimize the potential effect of biopsy track seeding because most of these patients will not be receiving chest wall radiation. The local recurrence is related to the pathologic characteristics of the breast cancer and the stage of disease. 2003 vol 17. Giuliano AE: Skin-sparing mastectomies. the plastic surgeon selects the mode of reconstruction to fill the “envelope” of skin. Gabram SG: Skin-sparing mastectomy with sentinel lymph node dissection: less is more. Moore B. no 7. not an inadequate surgical excision. improving the aesthetic result. Angelats J. Styblo TM. 2001 page 34 . New York. Smaller skin incisions are used around the nipple-areolar complex to preserve the breast envelope of skin. a counter axillary incision is preferred for better exposure. Ahn M. Patients with these histories should be cautioned about this potential complication. Multiple retrospective studies report local recurrence rates ranging from 0 to 7%.American College of Surgeons SESAP 13 Critique for Category 2 Item 1 Skin-sparing mastectomy (SSM) is being offered with increased frequency to patients with early stage breast cancer who are candidates for or desire a mastectomy in lieu of breast conservation. Lippincott Williams & Wilkins. Bostwick J III. traditional mastectomy landmarks are used as the flaps are elevated (see figure A). The nipple-areolar complex is completely excised in a circular or elliptical fashion (see figure B). For a positive sentinel node. ed 6. Oncologic and reconstructive considerations.Stradling BL. If a sentinel node biopsy is performed. the completion dissection can be done through the mastectomy or axillary incisions (see figure D). REFERENCES 1. Routine excision of core biopsy sites is not universally supported because local recurrence rates are low even without radiation therapy. some of whom had more favorable tumor characteristics. the prior recent excision biopsy sites should be excised (see figure C). After preoperative planning. Hellman S. which is similar to historical control groups in whom wider skin removal was used. Core biopsy shows stromal hyperplasia. the final pathology examination shows pseudoangiomatous hyperplasia with a 1-mm margin.American College of Surgeons SESAP 13 Category 2 Item 2 A 37-year-old woman has an enlarging 3-cm palpable left breast mass. After wide excision. The next step in her treatment should be (A) simple mastectomy (B) radiation therapy (C) tamoxifen (D) observation (E) re-excision with sentinel node biopsy page 35 . American College of Surgeons SESAP 13 Critique for Category 2 Item 2 Pseudoangiomatous stromal hyperplasia (PASH) is a non-neoplastic stromal lesion that presents as a distinct mass. Cranor ML. Local excision of the mass is recommended. Rosen PP: Pseudoangiomatous stromal hyperplasia (PASH). 1995 page 36 .Hoda SA. Am J Surg Pathol 19:270-277. the recurrence rate is low (7%). PASH is characterized by a round or oval well-circumscribed noncalcified mass and on ultrasound study it is a hypoechoic mass with cystic spaces. Gopalan A: Evolving entities and changing concepts in breast pathology. helping to distinguish it from the benign entity of PASH. and no further treatment is necessary. Biopsy is necessary to confirm the diagnosis. After excision with a negative margin. Whitman GJ.Powell CM. Breast Cancer 10:294-300. The pathogenesis of PASH is unknown but may be related to reactivity of myofibroblasts to endogenous or exogenous hormones. wider excision is often necessary to exclude another pathologic entity. Am J Clin Oncol 25:213-216. especially when the lesion is enlarging and core only shows stromal proliferation. 2002 2. and postmenopausal women not receiving hormone replacement therapy. florid PASH may be mistaken for angiosarcoma. First described in 1986. Because the complex anastomosing pattern at times simulates a vascular lesion. A mammary stromal tumor with myofibroblastic differentiation. A neoplastic versus a developmental process has been postulated because of the incidental microscopic finding of multicentricity and the rarely seen recurrent lesions. the disease has been found in teenagers. In very unusual cases. immunosuppressed patients. area of thickening of breast tissue. Chung SM. men. On mammogram. Also. Sahin AA: Pseudoangiomatous stromal hyperplasia of the breast. or as an incidental microscopic finding on core breast biopsy. 2003 3. The tumor cells of angiosarcoma are immunoreactive with endothelial markers. however.Castro CY. most PASH tumors are not ER/PR receptor positive. If PASH is found incidentally on core biopsy. REFERENCES 1. simple mastectomy may be required after multiple recurrences. the characteristic histologic finding is anastomosing slit-like empty spaces outlined by stromal cells. Her mother first noticed the mass 2 months previously. Management includes all of the following options EXCEPT (A) ultrasound imaging (B) fine-needle aspiration (FNA) (C) core biopsy (D) excisional biopsy (E) in-depth history for review of systemic symptoms page 37 .American College of Surgeons SESAP 13 Category 2 Item 3 An 8-year-old girl has a 2.5-cm breast mass that is enlarging just underneath her nipple-areolar complex. Pediatr Radiol 24:81-84. which is often the case in a young patient. REFERENCES 1. In children. and miscellaneous benign lesions. Primary adenocarcinoma of the breast is extremely rare in children and occurs in less than 1% of presenting childhood breast masses. rhabdomyosarcoma. 2005 page 38 . benign neoplastic masses.American College of Surgeons SESAP 13 Critique for Category 2 Item 3 Breast masses in children can be grouped into physiological. inflammatory. Single discrete lesions on ultrasound examination should be initially approached with fine-needle aspiration. Excisional biopsy as an initial diagnostic procedure should be avoided because the developing breast bud can be mistakenly removed. leukemia. Paik NS. Cho KJ. malignant breast lesions are more likely to be due to a primary tumor of nonbreast origin (such as lymphoma or rhabdomyosarcoma) or metastatic disease from lymphoma. Chung JH.McDivitt RW. 95% of these tumors in children are benign.5% to 2% of all malignant breast tumors. The most common lesions of the breast in childhood are fibroadenomas and cystosarcoma phyllodes. In contrast. Occasionally a subareolar nodule of up to 1 cm may persist due to the infant’s own hormonal secretions. et al: Breast mass in a 3-year-old girl: differentiation of secretory carcinoma versus abnormal thelarche by fine needle aspiration biopsy. core needle biopsy can lead to deformity but is occasionally necessary to obtain a larger sample for histologic diagnosis. Stewart FW: Breast carcinoma in children. metastatic tumors in adults account for 0. Carty H: Breast masses in childhood and adolescence. It is superior to mammography in detecting masses in dense fibroglandular versus fatty breast tissue. 1994 2. leading to significant deformity in the developing breast at a later age. primary malignant tumors. and secondary malignant tumors (see table). To a lesser extent. A presentation of 17 cases and a review of the literature. Although cystosarcoma phyllodes may present on a spectrum of benign to malignant lesions. and ultrasound guidance for this procedure may enhance tissue yield. Surgery 137:109110.Noh WC. Physiologically neonatal breast “nodules” may present in girls and boys and are due to the stimulation of maternal hormones. Ultrasonography is ideal for initial assessment of a pediatric patient with a breast mass. 1966 3. and neuroblastoma.Boothroyd A. JAMA 195:388-390. American College of Surgeons SESAP 13 Category 2 Item 4 A 48-year-old woman has had spontaneous clear discharge from the left breast for 3 months. Mammography and breast examination including staining for occult blood are normal. The best option for treatment would be (A) observation (B) core biopsy of the duct (C) central duct excision (D) ductogram with directed duct excision (E) magnetic resonance imaging (MRI) followed by central duct excision page 39 . the radiologist may perform a preoperative ductogram to identify an obstructing lesion deeper in the breast. A central duct excision refers to a nondirected procedure. A circumareolar cosmetic incision is made and the ducts are divided between ties until the offending duct is identified (either with the probe or dye). Staining for occult blood may be used to differentiate bloody pathologic discharge from dark brown physiologic discharge. Its use in predicting cancer is less accurate. or bloody is associated with an increased risk of carcinoma. although some can occur deeper in the breast. Regardless of the imaging findings. Ductal carcinoma in situ (DCIS) is the malignancy most commpnly found in intraductal lesions with pathologic discharge. Magnetic resonance imaging (MRI) is less sensitive for DCIS than for invasive cancer and would not be indicated in this patient prior to surgical duct excision. with only a 20% positive predictive value. REFERENCES 1. et al: Nonsurgical evaluation of pathologic nipple discharge. On the day of excision. is not as accurate in identifying the offending duct. Most papillomas are located within 2 to 3 cm of the nipple areolar complex. The most common etiology for pathologic nipple discharge is intraductal papilloma or mammary duct ectasia. open surgical biopsy is preferred. At times. Adamovich T. and can often lead to missing the lesion on excision biopsy. spontaneous. Excision continues beyond this area and a cylindrical piece of tissue extending 2 to 3 cm deep is then excised. Determining whether the “lesion” is more proximal or distal by palpation of the periareolar area is usually straightforward. Nipple aspirate cytology has also been used. Singletary SE. then the next step is directed duct excision. If palpation within the periareolar area elicits discharge. which is performed in the operating room by cannulating the duct with lacrimal probes or injecting the duct with methylene blue to identify the duct to be excised. may be required. If clinical examination or mammography indicates that the intraductal lesion resides deeper in the breast.Simmons R. Hunt KK. and persistent is classified as pathologic because of its higher association with an underlying pathologic process. needle localization of this deeper lesion. the incision is closed. After orienting the specimen for pathology. Christos P. J Am Coll Surg 196:354-364. serous (yellow). a papilloma may be diagnosed on core biopsy of a nodular density in the breast. 2003 2. Brennan M.Cabioglu N. serosanguinous (pink). Stephens TW. not within 2 to 3 cm of the nipple areolar complex. although excision is still required to rule out papillary carcinoma. 2003 page 40 . but its high falsenegative rates and the need for a surgical biopsy if a patient indeed has pathologic discharge have contributed to the decreasing use of this modality.American College of Surgeons SESAP 13 Critique for Category 2 Item 4 Nipple discharge that is unilateral. ie. Ann Surg Oncol 10:113-116. et al: Surgical decision making and factors determining a diagnosis of breast carcinoma in women presenting with nipple discharge. Spontaneous nipple discharge that is clear. from a single duct. mastectomy is the best option (C) An underlying carcinoma is associated with this disease process in 90% of cases (D) Disease deeper in the breast may be missed radiographically (E) Diagnosis can be made by punch biopsy page 41 . Which of the following statements about her condition is NOT true? (A) Breast conservation is contraindicated (B) If she has an underlying deep palpable mass with highly suspicious mammographic findings.American College of Surgeons SESAP 13 Category 2 Item 5 An 80-year-old woman has the left breast clinical findings shown. Surgical treatment has traditionally been mastectomy with or without axillary dissection. if patients do not have evidence of deeper disease on clinical examination and mammography. Deeper disease in the breast may be missed radiographically and there may be a role for magnetic resonance imaging to better identify those patients with localized disease who could undergo breast conservation. however. Martin D. et al: Paget’s disease of the breast: there is a role for breast-conserving therapy. Keratin positive staining Paget’s cells are pictured (see histologic slide). REFERENCES 1. Because more than 90% of patients have an underlying carcinoma (either in situ or invasive). The second theory holds that the cells originate from an intraductal cancer and spread toward the nipple (epidermotropic theory). 2004 2. Lopez-Ruiz JA. 2004 page 42 . The first. Buchholz TA. Tucker SL.American College of Surgeons SESAP 13 Critique for Category 2 Item 5 This patient has Paget’s disease and the diagnosis can be confirmed in the office with punch biopsy of the nipple-areolar complex. Dimaio DJ. Systemic treatment should be based on staging after the surgical procedure. About 50% of patients with Paget’s disease will present with a palpable mass.Marcus E: The management of Paget’s disease of the breast. Ann Surg Oncol 12:391-397. the epidermotropic theory is favored. There are two main theories regarding the origin of Paget’s disease. Br J Radiol 77:1036-1039. mastectomy and nodal sampling either with sentinel node biopsy or complete axillary dissection is indicated. 2005 3. If this is deep to the nipple-areolar complex with associated suspicious mammographic findings. Imaz I. then complete excision of the nipple-areolar complex and any underlying tumor (central lumpectomy) with radiation therapy results in similar local control and survival rates.Echevarria JJ. et al: Usefulness of MRI in detecting occult breast cancer associated with Paget’s disease of the nipple-areolar complex. proposes that the cells have an intra-epidermal origin that then progresses to malignant cells. Sir James Paget described the disease in 1874. Patients classically present with eczematous changes of the nipple that can progress to bleeding and ulceration as pictured in this case. the in situ transformation theory. Curr Treat Options Oncol 5:153-160.Kawase K. disk-like mass underneath his nippleareolar complex. mobile. All of the following are appropriate initially EXCEPT (A) inquiring about a history of trauma (B) taking a family history of breast and ovarian cancer (C) inquiring about illicit drug use (D) performing a testicular examination (E) recommending excisional biopsy page 43 .American College of Surgeons SESAP 13 Category 2 Item 6 A 15-year-old boy has a unilateral firm. rubbery. For the young person described here. Male breast cancer is associated with the BRCA2 genetic mutation. observation until at least age 18 is appropriate as long as his clinical examination is consistent with gynecomastia. peaking at age 13 to 14 from an imbalance of estrogen and testosterone. N Engl J Med 328:490-495. Numerous medications can cause gynecomastia. Most commonly this will be gynecomastia. FSH. The 3 distinct peaks in age distribution are: transiently during the neonatal period because of transplacental passage of estrogens. J Am Coll Surg 200:255-269. Because testicular cancer may present as gynecomastia. Kalter R: Male breast disease. For the older age patient with no obvious etiology who presents with a unilateral discrete mass. REFERENCES 1. and during adulthood. It is not uncommon for male patients to present to surgeons with a unilateral breast mass for evaluation. however. It may present as an asymptomatic enlargement or as a painful unilateral or bilateral tender breast “mass”’ just below the areolar region. which is more diffuse in nature. prolactin level (to exclude hyperprolactinemia secondary to prolactinoma). Roorda AK. notably marijuana. 2005 page 44 . A mammogram may also be helpful in screening for male breast cancer and if read as suspicious.Braunstein GD: Gynecomastia. Illicit drug use. even though it is unilateral. the breast “mass” should be biopsied. a family history of female or male breast and ovarian cancer would perhaps lead to other diagnostic modalities if his examination were more suspicious. from age 20 and beyond. ie. gynecomastia presents with a disk-like discrete soft mass as compared with fatty enlargement (pseudogynecomastia). changing this medication may alleviate symptoms. On clinical examination. is one of the etiologies and it is appropriate in a young person to inquire about use of this drug. creatinine (to exclude renal disease). If a patient has symptomatic gynecomastia from a medication recently started.Wise GJ. Fine-needle or core biopsy can be diagnostic in this clinical setting and will often yield a diagnosis without requiring an excisional biopsy. Bilateral mammography may demonstrate characteristic findings of gynecomastia that may be either unilateral or bilateral. thyroid hormone (to exclude hyperthyroidism). category 4 or 5 on the American College of Radiology’s Breast Imaging Reporting and Database System (BI-RADS). some of which are listed in the second table. DHEA (to exclude primary and secondary gonadal failure as well as adrenal carcinoma). a work-up including liver function tests (to exclude liver disease). 1993 2. Although a breast neoplasm would be extremely uncommon in this young patient. BUN. and testosterone (to exclude enzymatic defects of testosterone production) is appropriate. The first table summarizes a comprehensive list of conditions associated with gynecomastia. during puberty.American College of Surgeons SESAP 13 Critique for Category 2 Item 6 Gynecomastia is benign hypertrophy of the male breast due to proliferation of the glandular component. LH. a complete testicular examination is indicated even in this young patient. Core biopsy reveals infiltrating ductal carcinoma.American College of Surgeons SESAP 13 Category 2 Item 7 A 55-year-old woman has a nonpalpable 1-cm mass in her left breast. Left needle localization lumpectomy and sentinel lymph node biopsy is recommended. Which of the following statements is NOT true? (A) The recommended procedure is appropriate for patients with early breast cancer if performed by an experienced surgical team (B) The major concern for performing the procedure without experience is the false-negative rate (C) The risk of lymphedema is 1% to 3% (D) The risk of local axillary recurrence is 8% to 10% if her sentinel lymph nodes are negative (E) Frozen-section examination of the lymph node in the operating room has a false-negative rate of 10% to 15% and a false-positive rate 1% page 45 . There is no axillary or supraclavicular adenopathy on physical examination. 6th edition). but limitations of the test as well as certain types of breast histology (lobular carcinoma) contribue to the 5% to 10% chance the metastatic deposit will not be identified at the time of operation. isolated tumor deposits identified in the sentinel node < 0. after an informed discussion about the low yield of identifying other positive nodes in this setting as well as the role of adjuvant therapies that might control the nodal disease. Frozen-section examination of the lymph node can be performed at the time of operation. page 46 . or a combination to identify the primary afferent lymphatic drainage channel (s) leading to the sentinel lymph node(s). According to the AJCC Cancer Staging Manual (2002. An informed discussion with the patient about whether to perform a completion axillary dissection in this circumstance is indicated. radioactive dye. The American Society of Breast Surgeons recommends that a surgeon perform 20 cases of sentinel node biopsy in a mentored situation or with a complete axillary lymph node dissection to minimize the false-negative rate (and increase the identification rate). Patients can still present with this clinical finding after operation.American College of Surgeons SESAP 13 Critique for Category 2 Item 7 Sentinel lymph node surgery was first applied to breast cancer patients in 1991 when isosulfan blue dye-directed intraoperative lymphatic mapping was used. The risk of paresthesias is also decreased because the technique of identifying the sentinel lymph node involves following a lymphatic channel to the node with the ability to retract the intercostal brachial nerves away from the sentinel nodes. Others have reported similarly low incidences of recurrence on a longer term basis (less than 2%). and today surgeons use blue dye. Sentinel lymph node surgery decreases axillary morbidity. The procedure was originally described for determining the groin lymphatic drainage pattern for penile cancer.2 mm to 2 mm in size.2 mm in size is considered N0 disease. Micrometastatic disease is designated pN1mic. Technetium sulfur colloid radioactive mapping was successfully used in 1993 for breast patients. Micrometastasis in the lymph nodes is defined as a deposit measuring 0. However. The major limitation of the procedure is the false-negative rate. however. especially if a more in-depth surgical dissection is necessary to find a deep sentinel lymph node. In this case a full axillary dissection is indicated. which is the number of times all blue and/or radioactive lymph nodes are removed and yet a remaining non-sentinel lymph node harbors a metastasis. These patients should be followed clinically for potential local axillary recurrence. One study of axillary recurrence compared patients who had a sentinel lymph node procedure alone if the sentinel node was negative versus those randomly assigned to receive a complete axillary dissection regardless of the histologic status of the sentinel lymph node. the standard of care recommendation is to return to the operating room for a complete axillary lymph node dissection in order to determine the number of positive lymph nodes and to ensure that gross nodal disease is not left inadequately treated in the axilla. The sentinel node identification rate is higher and the false-negative rate lower when the combination is used. The incidence of lymphedema remains at 1% to 3%. and should be discussed with the patient. but its first widespread clinical use was for cutaneous melanoma. the patient may decide against further surgical treatment. The axillary recurrence rate was 0% in the sentinel lymph node only group. If a metastatic deposit is found on routine hematoxylin and eosin staining postoperatively. 1993 6. 2005. 1977 4. 2003 page 47 . Cancer 39:456-466. Consensus statement on guidelines for performing sentinel lymph node dissection in breast cancer.Anderson BO: Sentinel lymphadenectomy in breast cancer: an update on the NCCN clinical practice guidelines. Weaver DL. N Engl J Med 349:546-553. Luini A.Cabanas RM: An approach for the treatment of penile carcinoma. Giuliano AE: The time has come to change the algorithm for the surgical management of early breast cancer.Veronesi U.org 2.Singletary SE.Krag DN. Viale G. Fairbank JT: Surgical resection and radiolocalization of the sentinel lymph node in breast cancer using a gamma probe. Grube BJ. Surg Oncol 2:335-340. Available at: www.The American Society of Breast Surgeons. Arch Surg 137:1131-1135.American College of Surgeons SESAP 13 REFERENCES 1.Hansen NM.breastsurgeons. CA Cancer J Clin 56:37-47. et al: A randomized comparison of sentinel-node biopsy with routine axillary dissection in breast cancer. 2002 5. Paganelli G. J Natl Comprehensive Cancer Network 1:S64-S70. Connolly JL: Breast cancer staging: working with the sixth edition of the AJCC Cancer Staging Manual. 2003 3. Alex JC. 2006 7. estrogen and progesterone receptor negative.American College of Surgeons SESAP 13 Category 2 Item 8 A 55-year-old otherwise healthy woman has a 6-cm biopsy-proven infiltrating ductal carcinoma and an ipsilateral clinically palpable suspicious lymph node. Which of the following statements is NOT true? (A) She is a candidate for primary systemic chemotherapy (B) Modified radical mastectomy will ultimately be required regardless of her response to chemotherapy (C) A systemic work-up including computed tomographic (CT) scan of her abdomen and bone scan is indicated prior to initiating chemotherapy (D) Maximal chemotherapy should be given preoperatively providing the tumor is not progressing during treatment (E) Preoperative chemotherapy in this clinical setting is at least as good as postoperative chemotherapy and may offer improved survival page 48 . The tumor is grade III. Mammography demonstrates an ill-defined suspicious mass corresponding to the 6-cm palpable mass. J Clin Oncol 21:4165-4174. or extensive regional lymph node involvement. 2003 2. In all of these clinical situations. a systemic work-up including a bone scan and abdominal computed tomography are indicated to rule out systemic disease prior to treatment in order to plan the most optimal surgical care after chemotherapy. 2003 4. 1998 3. Valero V.2006.National Comprehensive Cancer Network: Clinical Practice Guidelines in Oncology v. et al: International expert panel on the use of primary (preoperative) systemic treatment of operable breast cancer: review and recommendations. www. von Minckwitz G. eg. inflammatory breast cancer or non-inflammatory cancer with skin.American College of Surgeons SESAP 13 Critique for Category 2 Item 8 Despite public awareness of the importance of screening mammography and vigilant examination. and delivering maximal chemotherapy first provides an opportunity to study the clinical effectiveness of a particular chemotherapy regimen.nccn.1.org/professionals/physician_gls/default.Kaufmann M. Wolmark N. chest wall.Fisher B. Chemotherapy given prior to operation was historically termed neoadjuvant therapy. An anthracycline-based chemotherapy regimen with the addition of a taxane as primary systemic therapy is preferred in patients without metastatic disease. All modalities are important.asp page 49 . Brown A. This terminology is dated because it implies that chemotherapy is of secondary importance to surgical resection or radiation therapy. for patients presenting with T3N1Mx disease (the clinical stage of the patient in this question). 2004 5. J Clin Oncol 16:2672-2685. treating patients with chemotherapy prior to operation is appropriate. et al: The effect on tumor response of adding sequential preoperative docetaxel to preoperative doxorubicin and cyclophosphamide: preliminary results from National Surgical Adjuvant Breast and Bowel Project Protocol B-27. The preferred term for this approach is primary systemic therapy as opposed to induction therapy or preoperative systemic therapy. 10% to 30% of all primary breast cancers are diagnosed as locally advanced.Lafreniere R: What’s new in general surgery: surgical oncology. Anderson S. REFERENCES 1. or are carcinomas with an unfavorable breast-to-tumor size ratio that prohibits initial breast conservation methods. Mamounas E. Smith R. et al: Effect of preoperative chemotherapy on the outcome of women with operable breast cancer. According to the National Comprehensive Cancer Network Clinical Practice Guidelines. Bryant J. J Clin Oncol 21:2600-2608. Smith R.Bear HD. J Am Coll Surg 198:966-988. examination. The next step should be (A) re-excision to obtain clear margins (B) lumpectomy and sentinel lymph node biopsy (C) lumpectomy.American College of Surgeons SESAP 13 Category 2 Item 9 A 45-year-old woman whose mother had breast cancer undergoes a stereotactic right breast core biopsy. and mammography in 6 months (E) radiation therapy of the right breast page 50 . sentinel lymph node biopsy. Pathologic examination demonstrates lobular carcinoma in situ. and radiation therapy (D) observation. Saunders. bilateral total mastectomy would be the procedure of choice because the incidence of cancer is equal for both breasts. et al: Clinical management of lobular carcinoma in situ. Should a patient concerned about cancer risk insist on a surgical procedure. REFERENCES 1. Hamilton. Robb GL. London. Li CL. Kaelin CM: Diseases of the breast. Patients should be informed that LCIS is associated with the possible development of carcinoma. in Singletary SE. 2004.Anderson BO. Lawton TJ.Iglehart JD. pp 867-928 page 51 .American College of Surgeons SESAP 13 Critique for Category 2 Item 9 Lobular carcinoma in situ (LCIS) is usually an incidental finding at biopsy and occurs in younger premenopausal women. it can be considered a marker for the increased susceptibility of breast cancer for which the rate of occurrence can be estimated at 1% per year. Hortobagyi GN (eds): Advanced Therapy of Breast Disease. Although LCIS is not a breast cancer itself. sentinel lymph node biopsy. Tamoxifen has been associated with a nearly 50% reduction in cancer formation in women with LCIS and should be considered in this patient. Philadelphia. pp 269-279 2. Ontario. ed 2. BC Decker Inc. Re-excision of this lesion. 2004. or irradiation is not appropriate in this circumstance. The patient’s family history of breast cancer mandates close observation. ed 17. Several studies have documented that the probability of developing carcinoma at 20 to 30 years after diagnosis of LCIS may be as high as 20% to 25%. and that this risk occurs throughout life and increases over time. in Townsend CM Jr (ed): Sabiston Textbook of Surgery: The Biological Basis of Modern Surgical Practice. Rinn KJ. American College of Surgeons SESAP 13 Category 2 Item 10 A 45-year-old woman whose mother and 2 maternal aunts have breast cancer has a breast imaging reporting and data system (BI-RADS) 4 lesion on mammography. the chance of malignancy is 75% (D) A BI-RADS 4 lesion requires additional imaging (E) Strong family history will alter the BI-RADS classification page 52 . it is most likely to be a fibroadenoma (B) If the lesion is malignant. it is most likely to be ductal carcinoma in situ (DCIS) (C) If the lesion is associated with a mass. Which of the following statements is TRUE? (A) If the lesion is benign. It is based on the mammographic findings on any individual mammogram and the classification is made in coordination with any previous mammogram done on the patient. Reynolds C. Proceeding to biopsy without additional imaging is strongly indicated for BIRADS 4 or 5. and architectural distortion or dilatation of ducts. Ductal carcinoma in situ (DCIS) is the most common malignancy. Hollingsworth AB: Positive predictive value of the Breast Imaging Reporting and Data System. as shown in the tables.Lacquement MA. REFERENCES 1. followed by invasive ductal carcinoma.Orel SG. This order is reversed for BI-RADS 5 lesions. a developing density. Kay N. Biopsy is not required for BI-RADS categories 1. Radiology 211:845-850. Strong family history or other outside predictive factors are not a component of BI-RADS classification. The frequency of carcinoma is 25% to 30% in BI-RADS 4 lesions associated with a mass. microcalcifications. 1999 2. if the lesion is benign it is most likely a proliferative or nonproliferative fibrocystic process. J Am Coll Surg 189:34-40. 2. Although a significant number of malignancies are found in BI-RADS 4 patients. this percentage falls significantly in BI-RADS 4 cases relative to fibrocystic disease. BI-RADS classification depends on the mammographic finding of mass. Mitchell D. asymmetric density. and 3. Whereas fibroadenoma may occur in a higher percentage of cases of BI-RADS 3 patients. Sullivan DC: BI-RADS categorization as a predictor of malignancy. BI-RADS is intended to standardize the terminology in mammographic reporting and recommendations for the necessity of biopsy.American College of Surgeons SESAP 13 Critique for Category 2 Item 10 The breast imaging reporting and data system (BI-RADS) was created to standarize mammographic results and provide comparison across mammographic facilities. 1999 page 53 . The predictive value of BI-RADS has been evaluated in studies correlating pathologic findings after biopsy for various BI-RAD lesions. BI-RADS 0 indicates technical concerns requiring additional imaging. Lumpectomy and sentinel lymph node biopsy confirm negative surgical margins. The tumor is poorly differentiated. Additional initial treatment should include (A) tamoxifen (B) methotrexate (C) anastrazole (Arimidex) (D) trastuzumab (E) orathecin (Rubitecan) page 54 . and estrogen and progesterone receptor negative.American College of Surgeons SESAP 13 Category 2 Item 11 A 35-year-old woman with no co-morbidities presents with a 1. HER2 positive. and lymph nodes are negative for tumor.5-cm invasive breast cancer diagnosed by core biopsy. After HER2/neu amplification has occurred. this risk-benefit scenario is likely to apply to many patients with nodepage 55 . Patients with HER2-positive breast cancer are likely to have poorly differentiated tumors with a high proliferative rate. each with a different natural history and each requiring a different treatment. The National Surgical Adjuvant Breast and Bowel Project trial and the North Central Cancer Treatment Group trials documented that trastuzumab therapy was associated with a 33% reduction in the risk of death (p=0.American College of Surgeons SESAP 13 Critique for Category 2 Item 11 Breast cancer is not a single disease but a group of several important tumor subtypes. 95% CI 0.4%). which means that testing for HER2 can be performed on either the primary tumor or a metastatic tumor deposit. Because laboratory and clinical studies indicated that the inhibition of cell growth by trastuzumab is limited to HER2-positive cancers. was developed after it had been recognized that HER2 overexpression served as both a marker of aggressive disease and a target for treatment. should also be offered the antibody. positive axillary lymph nodes. On the basis of these results. and decreased expression of estrogen and progesterone receptors. Patients with negative lymph nodes. absolute benefit in terms of disease-free survival at 2 years of 8. unless the antibody is clearly contraindicated. These characteristics are associated with an increased risk of disease recurrence and death.015) among women with HER2-positive breast cancer. HER2 testing became routine in the care of patients with breast cancer. HER2 became clinically relevant with the demonstration that HER2-positive breast cancers have a worse prognosis than HER2-negative tumors.0001. Of invasive breast cancers. patients with lymph-node-positive. a humanized monoclonal antibody against HER2 created by inserting portions of the antigen-binding site of a mouse monoclonal antibody against HER2 into a human monoclonal antibody. whose estimated risk of recurrence after optimal chemotherapyand endocrine therapy comfortably exceeds the risk of the cardiac toxic effects of trastuzumab. The international multicenter randomized trial compared 1 or 2 years of trastuzumab given every 3 weeks with observation in patients with HER2-positive breast cancer who had completed locoregional therapy and at least 4 cycles of neoadjuvant or adjuvant chemotherapy. generally with similar results. HER2-positive breast cancer should receive trastuzumab as part of optimal adjuvant systemic therapy. Because most HER2-positive tumors have other adverse prognostic factors.67. the HER2 phenotype is believed to be fixed for the remaining natural history of the invasive tumor. Overexpression of HER2 (which derives its name from human epidermal growth factor receptor 2) defines one of these unique subtypes. The absolute difference in disease-free survival between the trastuzumab group and the control group was 12% at 3 years. 15% to 20% are HER2-positive. Trastuzumab. testing tumors for expression of HER2 became integral to the selection of patients for clinical trials of the efficacy of trastuzumab.54. One year of treatment with trastuzumab significantly improved disease-free survival (hazard ratio 0. p<0. amplification of the gene and the resultant overexpression of HER2 protein occur during the in situ stage of tumor development. Overall survival was not significantly different. Because of the success of these trials. This confirmed that trastuzumab combined with paclitaxel after doxorubicin and cyclophosphamide improves outcomes among women with surgically removed HER2-positive breast cancer (see graph).43-0. et al: Trastuzumab plus adjuvant chemotherapy for operable HER2-positive breast cancer.Slamon DJ. N Engl J Med 353:1659-1672. trastuzumab contributes to apoptosis. Science 235:177-182. There may also be an extracellular effect. Suman VJ. 2005 2. the scientific basis for the effects of trastuzumab is uncertain. Wong SG. Bryant J. 2005 4. 1987 page 56 . in mediating antibody-dependent immune recognition.American College of Surgeons SESAP 13 Although the clinical observations are clear. Leyland-Jones B. N Engl J Med 353:1673-1684. 2005 3.Burstein HJ: The distinctive nature of HER2-positive breast cancers. suppresses the production of the angiogenic factor vascular endothelial growth factor (VEGF). Clark GM. tamoxifen and anastrazole would not be indicated. alters downstream signaling and regulatory pathways in the cell cycle. causes downregulation of surface HER2 expression.Piccart-Gebhart MJ. Goldhirsch A. REFERENCES 1. Perez EA. et al: Human breast cancer: correlation of relapse and survival with amplification of the HER-2/neu oncogene. N Engl J Med 353:1652-1654. After binding to the HER2 protein. Procter M.Romond EH. Because this patient is estrogen and progeserone receptor negative. Orathecin (Rubitecan) is an oral camptothecin analog currently in phase II clinical trials for the treatment of metastatic breast cancer. et al: Trastuzumab after adjuvant chemotherapy in HER2-positive breast cancer. Laboratory studies and limited data from clinical trials suggest myriad possible mechanisms of action. and potentiates the effects of chemotherapy. Levin WJ. American College of Surgeons SESAP 13 Category 2 Item 12 Which of the following statements about digital mammography is TRUE? (A) It is most beneficial for women with radiologically dense breasts (B) It is better than film mammography for patients over 50 (C) It is a better screening test for breast cancer than film mammography (D) It results in a higher radiation dose than film mammography (E) The two systems are similar in cost page 57 . The use of digital mammography is therefore justified in these groups. Image processing of digital data allows the degree of contrast in the image to be manipulated. Gatsonis C. women aged 40 to 49 at entry had a smaller but significant reduction of 15% to 20%. The Digital Mammographic Imaging Screening Trial (DMIST) was designed to measure relatively small but potentially clinically important differences in diagnostic accuracy between digital and film mammography in 49.Pisano ED. Meta-analyses of 8 large randomized trials found a reduction in the mortality rate of 16% to 35% among women 50 to 69 years old who were assigned to screening mammography. which was developed in part to address some of the limitations of film mammography. more rapidly growing tumors. REFERENCES 1.528 asymptomatic women. 2005 page 58 .American College of Surgeons SESAP 13 Critique for Category 2 Item 12 There is now general agreement that screening mammography reduces the rate of death from breast cancer among women who are 40 or older. previous trials have not found digital mammography to be significantly more accurate than film mammography in the diagnosis of breast cancer. 2005 2. and storage of images. Hendrick E. retrieval. and the greater radiographic density of breast tissue in women under 50. so that contrast can be increased in the dense areas of the breast with the lowest contrast. but digital mammography was more accurate in women under 50. Digital mammography. However. et al: Diagnostic performance of digital versus film mammography for breast-cancer screening. these studies included only one type of digital detector and had insufficient statistical power to identify relatively small differences in diagnostic accuracy. separates image acquisition and display. improving both. Despite the apparent differences between the two approaches. despite its higher cost. Yaffe M. those with radiographically dense breasts. and the use of a lower average dose of radiation without a compromise in diagnostic accuracy. The overall diagnostic accuracy of digital and film mammography for breast cancer screening was similar. Greater density reduces the sensitivity of mammography and increases the risk of breast cancer. The smaller benefit of screening in younger women is probably due to a lower incidence of breast cancer.Dershaw DD: Film or digital mammographic screening? N Engl J Med 353:1846-1847. N Engl J Med 353:1773-1783. and premenopausal and perimenopausal women. improved means of transmission. Digital mammography offers other advantages over film mammography: easier access to images and computer-assisted diagnosis. American College of Surgeons SESAP 13 Category 2 Item 13 Factors involved in the Gail model assessment of breast cancer risk include all of the following EXCEPT (A) presence of atypical hyperplasia (B) age at menses (C) number of first-degree relatives with breast cancer (D) number of second-degree relatives with breast cancer (E) number of prior breast biopsies page 59 . 2001 page 60 . number of prior breast biopsies. 1989 3. et al: Projecting individualized probabilities of developing breast cancer for white females who are being examined annually. age at menarche. et al: Validation of the Gail et al model of breast cancer risk prediction and implications for chemoprevention.Rockhill B. The number of second-degree relatives with breast cancer is not considered a risk factor. the model calculates an individual’s risk of developing breast cancer over the next 5 years and her lifetime risk. Anderson S. Tamoxifen does have potentially serious complications. 1999 2. when the 5-year risk is >1. J Natl Cancer Inst 91:1541-1548. J Natl Cancer Inst 81:18791886. These risks need to be discussed with the patient prior to beginning chemoprevention. including thromboembolic events.American College of Surgeons SESAP 13 Critique for Category 2 Item 13 The Breast Cancer Detection Demonstration Project developed the Gail model to assess a woman’s risk of developing invasive breast cancer. J Natl Cancer Inst 93:358-366. Pee D.Costantino JP. Tamoxifen is contraindicated in women who continue to smoke because of the increased risk of thromboembolic disease. and increased risk of endometrial cancer. tamoxifen is indicated and reduces the breast cancer risk by about 50%. Brinton LA. Gail MH. From the standpoint of chemoprevention. Corle DK. The original model evaluated many factors that could have influenced breast cancer occurrence. number of first-degree relatives with breast cancer. Byar DP. REFERENCES 1. Byrne C. et al: Validation studies for models projecting the risk of invasive and total breast cancer incidence. Using this information. Spiegelman D. A subsequent modification of the model (to use SEER baseline age-specific breast cancer rates rather than BCDDP rates) was used in the NSABP Breast Cancer Prevention trial (P-01).Gail MH. Hunter DJ. Factors ultimately found to be associated with a significant increased risk were age. such as deep venous thrombosis and pulmonary embolism.6%. and the presence of atypical ductal hyperplasia in a breast biopsy. American College of Surgeons SESAP 13 Category 2 Item 14 Radiation therapy after mastectomy for breast cancer is indicated for (A) all patients (B) patients with any metastases to lymph nodes (C) patients with 4 or more lymph node metastases (D) patients with at least 10 or more lymph node metastases (E) patients with high-grade ductal carcinoma in situ lesions > 5 cm in diameter page 61 . those with fewer than 4 positive lymph nodes. Costa J. Int J Radiat Oncol Biol Phys 44:989-990. J Natl Cancer Inst 93:979-989. or invasion of the skin or chest wall. Techniques to avoid the heart and great vessels. J Clin Oncol 19:1539-1569.Harris JR. Edge SB. Mendenhall NP. Halpin-Murphy P. 2000. Postmastectomy adjuvant radiation is only indicated for high-risk groups. A higher incidence of death from cardiovascular events has also been reported with adjuvant radiotherapy. et al: National Institutes of Health Consensus Development Conference Statement: adjuvant therapy for breast cancer. November 1-3. and the subsequent vascular events. 2001 2.Eifel P. ie. Solin LJ. It was common practice several decades ago to use radiation therapy after modified radical mastectomy. REFERENCES 1. 2001 page 62 . 1999 3. and studies showing that the risk of locoregional recurrence after mastectomy is very high for certain groups have led to renewed interest in postmastectomy adjuvant radiation therapy.American College of Surgeons SESAP 13 Critique for Category 2 Item 14 Radiation therapy is the standard of care after breast-conserving surgery. but this may be related to the inclusion of the heart and great vessels in the radiation field in some of these studies. Axelson JA. et al: Consensus Statement on postmastectomy radiation therapy. Patients with invasive tumors < 5 cm or not involving the skin or chest wall. High-risk groups include patients with 4 or more positive lymph node and patients with advanced invasive tumors. Crowley J. > 5 cm.Recht A. are part of the more modern approach to adjuvant postoperative radiotherapy. et al: Postmastectomy radiotherapy: clinical practice guidelines of the American Society of Clinical Oncology. McNeese M. or those with DCIS alone are not candidates. Robinson DS. Meta-analyses of several randomized trials comparing postmastectomy adjuvant radiation therapy versus mastectomy with chemotherapy have demonstrated a 70% to 90% decrease in locoregional failure and an improvement in overall survival for these high-risk groups. At her postoperative check-up. her breast is warm and erythematous. The most appropriate treatment now would be (A) simple mastectomy (B) lumpectomy and axillary dissection (C) removal of the foreign body (D) isoniazid (E) corticosteroids page 63 .American College of Surgeons SESAP 13 Category 2 Item 15 A 24-year-old woman has 3 new palpable masses in her right breast that have increased in size over the past 4 months. Pathologic examination of an excisional biopsy specimen reveals multiple necrotizing granulomata surrounded by lymphoplasmacytic infiltrate and focal duct disruption. The mammogram shown is obtained. breast mass. which may be responsible for up to 3% of benign breast lesions in developing countries. use of methotrexate has been described. 2002 3. J Clin Pathol 56:519-521. an autoimmune syndrome has also been reported Classically. REFERENCES 1. Clinical symptoms may include galactorrhea. Granulomatous mastitis is an uncommon breast lesion. 2004 2. Kumar M. Mammography and sonography may demonstrate nodular opacities and hypoechoic nodules. Conditions that must be considered include infection with Bartonella henselae or mycobacteria. epitheloid cells.American College of Surgeons SESAP 13 Critique for Category 2 Item 15 The mammogram shows a nipple ring that likely caused an ascending ductal infection leading to several chronic granulomatous reactions along the duct.Doughty JC. There is no role for axillary dissection in this nonmalignant process.Khanna R. and skin induration or ulceration. Histologic features include signs of a chronic inflammation with giant cells. 2003 page 64 . The absence of necrosis and a predominantly neutrophilic infiltrate in the background support a diagnosis of granulomatous mastitis. radiological or cytological examination: a cautionary tale. Poon CS. eventually in combination with anti-inflammatory drugs or colchicines. leucocytes. Postgrad Med J 78:422—424. et al: Fine needle aspiration cytology of granulomatous mastitis. When tuberculous mastitis is diagnosed based on histology or polymerase chain reaction confirmation of Mycobacterium tuberculosis. treatment of noninfectious granulomatous mastitis consists of complete surgical excision combined with oral corticosteroid therapy. and macrophages. granulomatous mastitis presents as a breast lump with or without an associated sinus. inflammation. This diagnosis should also be considered when epithelioid histiocytes are abundant on smears. as well as abscess cavities.Tse GM. Breast 13:261. Prasanna GV. especially in the US. Reported correlations include breast-feeding and the use of oral contraceptives in the premenopausal postpartum female. systemic antitubercular therapy should be instituted. even in the absence of granulomas. Immunosuppressive therapy should be performed until complete remission is seen because recurrence rates can approach 50%. Law BK. et al: Mammary tuberculosis: report on 52 cases. Wilson CR. Mallon EA: Granulomatous mastitis can mimic breast cancer on clinical. Pang LM. Definitive diagnosis and therapy for granulomatous mastitis depends on fine-needle cytology/histology and microbiology. If no microbiologic etiology can be confirmed. Gupta P. American College of Surgeons SESAP 13 Category 2 Item 16 Patients with the BRCA1/2 gene mutation and a family history of breast cancer (A) are more likely to develop contralateral cancer than women with comparable-stage cancer who do not have BRCA1 (B) do not benefit from prophylactic oophorectomy (C) do not benefit from prophylactic bilateral mastectomy (D) have a low incidence of undetected premalignant lesions in prophylactic mastectomy specimens (E) have a worse prognosis than women with comparable-stage cancer who do not have BRCA1 page 65 . Neuhausen SL. Hulick M. histopathologic localization of previously unidentified high-risk neoplastic lesions is common.000 individuals. Deffenbaugh AM. should be discussed with patients. Reid JE. supporting the potential benefits of prophylactic mastectomy. Although the use of prophylactic mastectomy is still debated. Bilateral prophylactic oophorectomy reduces the risk of breast cancer in women with BRCA1 or BRCA2 mutations. N Engl J Med 346:1616-1622. et al: Clinical characteristics of individuals with germline mutations in BRCA1 and BRCA2: analysis of 10. premalignant lesions are frequently found in mastectomy specimens. and the risks and advantages of operation. et al: Prophylactic oophorectomy in carriers of BRCA1 or BRCA2 mutations.Frank TS. Narod SA. Lynch HT. Issues regarding life expectancy. 2002 2. J Clin Oncol 20:1480-1490. When prophylactic mastectomies are performed in patients over 40.Rebbeck TR. Breast cancer in patients with the BRCA1 or BRCA2 mutation appears to have a similar prognosis to that of equally staged cancers in women without the mutation. 2002 page 66 . Each patient should receive information adequate to make her own informed choices about cancer prevention strategies. REFERENCES 1.American College of Surgeons SESAP 13 Critique for Category 2 Item 16 Women with breast cancer who have the BRCA1 or BRCA2 gene mutations are at increased risk for contralateral breast cancer at 5 years compared with women with breast cancer who do not have the mutations (31% versus 4%). American College of Surgeons SESAP 13 Category 2 Item 17 Phyllodes tumors (A) are easily distinguished from fibroadenomas on physical examination (B) commonly involve the nipple-areolar complex (C) are more common in males (D) require axillary dissection (E) may require mastectomy page 67 . Damak T. Surg Oncol 9:49-52. Most occur in the upper outer quadrant (46%).Chen WH. McNeese MD. Cohen JM. Chargui R. Yang TL. et al: Surgical treatment of phyllodes tumors of the breast: retrospective review of 172 cases. The nipple-areolar complex is rarely involved. although 4% of phyllodes tumors are central and patients may present with bloody nipple discharge or nipple retraction. Cancer 89:1502-1511. REFERENCES 1.American College of Surgeons SESAP 13 Critique for Category 2 Item 17 Presenting as a firm. Cheng SP. J Surg Oncol 91:185-194. Because the primary goal for treatment of localized phyllodes tumor is to achieve wide negative margins.August DA. palpable adenopathy is usually reactive in nature. 2000 2.Ben Hassouna J. Am J Surg 192:141-147. Zagars GK. or lumpectomy plus irradiation. 2005 page 68 . et al: Phyllodes tumors of the breast: a case series of 106 patients.McGrath MH: Benign tumors of the teenage breast. well-circumscribed and mobile mass. or for repeated local recurrences despite adequate margins. most can be treated by excision with a rim of at least 1 cm of uninvolved tissue. Even in the case of malignant phyllodes tumors. smooth. 2000 4. lumpectomy. Even with mammography or sonography. Gamoudi A.Chaney AW. Kearney T: Cystosarcoma phyllodes: mastectomy.Valdes EK. Plast Reconstr Surg 105:218-222. 2000 6. Because these tumors do not metastasize via lymphatic drainage. 2005 5. Tzen CY. removal of axillary or other regional lymph nodes is not indicated unless they are clearly involved with tumor. Total mastectomy may be required for lesions too big to achieve a 1-cm margin without markedly deforming the breast. et al: Primary treatment of cystosarcoma phyllodes of the breast. 2006 3. Boolbol SK. seldom painful. and these have been associated with gynecomastia. Feldman SM: Malignant transformation of a breast fibroadenoma to cystosarcoma phyllodes: case report and review of the literature. Am Surg 71:348-353. Few have been reported in men. Pollack A. phyllodes tumors are most often diagnosed after biopsy of a breast mass presumed to be a fibroadenoma. differentiating fibroadenoma from phyllodes tumor is challenging. a 36-year-old gravida 2.American College of Surgeons SESAP 13 Category 2 Item 18 During a routine prenatal visit. para 1 woman in her 14th week of an uncomplicated pregnancy mentions finding a right breast mass and is referred for surgical consultation.x 2-cm mass is easily palpable in the subareolar area of the right breast along with multiple nodes in the right axilla. A 3. Management now should consist of (A) core needle biopsy (B) diagnostic mammography (C) excisional biopsy of the mass (D) magnetic resonance imaging (MRI) of the breast (E) no diagnostic work-up until after 32 weeks gestation page 69 .5. An ultrasound study of the lesion is shown. pp1476-1490 3. treatment options can be considered. Magnetic resonance imaging (MRI) offers no additional information in this clinical scenario. Copeland EM III (eds): The Breast: Comprehensive Management of Benign and Malignant Disorders. Hurd TC: Breast cancer in pregnancy: a literature review. The halo effect or boundary echoes are prominent. which may interfere with subsequent sentinel node biopsy and may compromise the opportunity for induction chemotherapy. Curr Treat Options Oncol 4:165-173. Yu T. Diagnostic mammography of the involved breast is not a substitute for tissue diagnosis.American College of Surgeons SESAP 13 Critique for Category 2 Item 18 Ten percent of women under age 40 who develop breast cancer will be pregnant. 2004. Saunders. St. Gwyn KM.4 mRad. Mammography may be useful for evaluation of the contralateral breast. J Am Coll Surg 194:54-64. Sundaram M. as is the heterogeneous texture of the irregular mass effect. et al: Multidisciplinary management of breast cancer concurrent with pregnancy. REFERENCES 1. Lakin GE: Carcinoma of the breast in pregnancy and lactation. Arch Surg 138:91-98. in Bland KI. Total two-view mammography results in a fetal dose of only 0.Woo JC. Theriault RL. Fine-needle aspiration or core needle biopsy is preferred to excisional biopsy. 2003 4. This patient’s ultrasound is a classic example of an invasive ductal carcinoma. Delay in diagnosis increases the risk of lymphatic metastasis and systemic disease and should be avoided. Louis. including breast conservation therapy. Tissue diagnosis remains the gold standard. 2003 page 70 . It may be done at any point during gestation.Rugo HS: Management of breast cancer diagnosed during pregnancy. and can be performed safely during pregnancy.Keleher AJ. ed 3.Robinson DS. Excisional biopsy alone is inadequate treatment. Once the diagnosis is established. Hunt KK. 2002 2. 5. The first step in the definitive management of this patient should be (A) mastectomy (B) metastatic work-up (C) induction chemotherapy (D) sentinel lymph node biopsy (E) termination of the pregnancy page 71 .x 2-cm mass in the subareolar region of her left breast. Core needle biopsy demonstrates the histopathologic appearance shown. It is fixed to the subareolar area but not to the chest wall.American College of Surgeons SESAP 13 Category 2 Item 19 A 36-year-old gravida 2. para 1 woman entering her 20th week of an uncomplicated pregnancy has a 2. sentinel node identification. Termination of pregnancy is no longer recommended. J Am Coll Surg 194:54-64. which is an abortifacient and the leading cause of chemotherapy-induced birth defects. Sundaram M. and sampling if they are appropriate candidates. 2003 page 72 .Rugo HS: Management of breast cancer diagnosed during pregnancy. induction chemotherapy should be recommended to this patient because of the size of the lesion (>1 cm even if node negative) and the fixation to the subareolar area of the breast. and is generally reserved until the second trimester. Copeland EM III (eds): The Breast: Comprehensive Management of Benign and Malignant Disorders. pp1476-1490 3. Lakin GE: Carcinoma of the breast in pregnancy and lactation. Its use in pregnant women with breast cancer has been evaluated only on a very limited basis.American College of Surgeons SESAP 13 Critique for Category 2 Item 19 The histopathologic examination demonstrates that this patient has an invasive ductal carcinoma. alkylating agents) can be excreted in breast milk and should not be administered to mothers who are breast-feeding. et al: Multidisciplinary management of breast cancer concurrent with pregnancy. 2003 4. The radiation dose to the fetus should theoretically be small based on known doses of the radioisotope in the mother’s breast tissue and the exposure to operating room personnel. 2002 2. Chemotherapy during the first trimester when viscerogenesis is occurring is associated with a higher incidence of birth defects. Sentinel lymph node biopsy has become commonplace for the non-gravid patient with breast cancer and has minimized unnecessary axillary dissection for node-negative women.Robinson DS. ed 3. Methotrexate. Abortion has not been shown to improve survival. the growth and development of children born to mothers who received chemotherapy during pregnancy do not appear to have been adversely affected. REFERENCES 1. Curr Treat Options Oncol 4:165-173. Hurd TC: Breast cancer in pregnancy: a literature review.5%.Woo JC. fluorouracil. Isosulfan blue has not been tested in pregnant animals or humans and most authors do not recommend its use. Arch Surg 138:91-98. Theriault RL. should be avoided. Hunt KK. and there is some evidence that it in fact reduces survival. Yu T. Whether sentinel node biopsy should be performed before the administration of chemotherapy has yet to be determined. The risk of allergic reaction in women is reported to be only 1. Saunders. Certain agents (methotrexate. 2004. St. Pregnant women with breast cancer should not be denied the opportunity to participate in clinical trials involving lymphatic mapping. While definitive surgical management of breast cancer can be performed safely at any point during gestation. Metastatic work-up at this point would be unnecessary in this symptomatic patient because the yield is low and there is no improvement in survival. in Bland KI. Gwyn KM.Keleher AJ. Louis. After two decades of follow-up. the galactogram shown was obtained.American College of Surgeons SESAP 13 Category 2 Item 20 A 49-year-old woman develops spontaneous bloody discharge from her left nipple. Prior to referral for surgical consultation. The next step in management should be (A) observation only (B) excisional biopsy (C) fine-needle aspiration (D) nipple-areolar resection (E) ductal lavage and cytologic examination page 73 . Zuiani C. Radiology 238:801-808. Lawton TJ: Papillary lesions of the breast with and without atypical ductal hyperplasia: can we accurately predict benign behavior from core needle biopsy? Am J Clin Pathol 122:440-443.Mercado CL. 2003 7. Excisional biopsy provides tissue for definitive histopathologic examination. with biopsy also looks promising but is not widely available and its impact has not been thoroughly evaluated. 2004 page 74 . Oken SM.Renshaw AA. Fine-needle aspiration might miss significant disease and is a poor choice. Mammoscopy. 2005 4. Given the small but real risk for malignancy. Ductal lavage and cytology are promising diagnostic techniques. Singer CI: Papillary lesions of the breast at percutaneous core-needle biopsy.Di Cristofano C. A tissue diagnosis is necessary to exclude a malignant lesion and to identify lesions believed to be premalignant. Oncology 65:311-315. ultrasound and large core biopsy in the diagnostic evaluation of papillary breast lesions. or ductal endoscopy. Nipple-areolar resection would be reserved for high-grade lesions that could not be removed by simple duct excision alone. Namba K. Am J Clin Pathol 123:36-44. Hamele-Bena D. Kawakami Y. observation is not a wise option. Yeh IT: Myoepithelial cell staining patterns of papillary breast lesions: from intraductal papillomas to invasive papillary carcinomas. surgical excision upstages disease to atypia or in situ malignancy with such frequency (~21% in one series) that excisional biopsy is the best first approach. A few studies correlating the results from benign histopathology on core needle biopsy have indicated good concordance with subsequent radiographic imaging and patient outcome. et al: Papillary lesions of the breast: a molecular progression? Breast Cancer Res Treat 90:71-76. Tizol-Blanco DM. ductal hyperplasia with atypia. However.Matsunaga T.American College of Surgeons SESAP 13 Critique for Category 2 Item 20 All unilateral nipple discharge should be critically analyzed to rule out cancer. and eliminates the troublesome symptoms of a nipple discharge. but the patients for whom this technology will be most useful have not been defined.Agoff SN. Valent F. Derhagopian RP.Puglisi F. 2005 3. Bertacca G. Gould EW: Papillomas and atypical papillomas in breast core needle biopsy specimens: risk of carcinoma in subsequent excision. Fujii M: Intraductal biopsy for diagnosis and treatment of intraductal lesions of the breast. 2006 6. Bazzocchi M. 2004 2. Am J Clin Pathol 122:217-221. et al: Role of mammography. but is premature at this point in the diagnostic work-up. Cancer 101:2164-2169. Image-directed core needle biopsy has been advocated for lesions without radiographic signs suggesting malignancy.Hill CB. eg. Zavaglia K. Mrad K. This patient’s galactogram demonstrates significant ductal ectasia (images 1 and 2) and a papillary lesion (images 3 and 4) within the duct. REFERENCES 1. 2004 5. The most likely diagnosis is (A) hematoma (B) carcinoma (C) cystic disease (D) giant fibroadenoma (E) juvenile hypertrophy page 75 . and is slightly tender to palpation.American College of Surgeons SESAP 13 Category 2 Item 21 A 17-year-old high school swimmer has noted rapid enlargement of her right breast over the past 6 weeks. There has been no nipple discharge. She has taken oral contraceptives for the past 18 months. There is a mass effect in the lateral upper and lower right breast. The right breast is nearly 2 1/2 times as large as the left. as shown. Silfen R. these tumors usually exceed 5 cm. They are characterized by rapid and impressive growth over several weeks. Recurrence is rare and the breast will usually regain its normal shape.American College of Surgeons SESAP 13 Critique for Category 2 Item 21 Giant fibroadenomas of the breast are rare and generally occur in women age 15 to 20 years old. 2004. and occasionally skin necrosis. ed 3. Bland KI: Evaluation and treatment of benign breast disorders.Beenken SW. Louis. but unilateral enlargement occasionally occurs. pp 233-260 3. 2003 page 76 . but there is no history of trauma. 2000 2. rotational flaps. 1999 4. Erk Y: Unilateral virginal breast hypertrophy. Treatment is by local excision and can frequently be accomplished through an inframammary incision for cosmetic reasons. in Bland KI. Breast J 9:53. A hematoma is certainly possible. Venous engorgement of the skin is often seen.Agaoglu G. Hudson DA: Florid juvenile (cellular) fibroadenomatosis in the adolescent: a case for subcutaneous mastectomy? Aesthetic Plast Surg 23:413-415. These are benign lesions and there has not been an association with malignant degeneration. Copeland EM III (eds): The Breast: Comprehensive Management of Benign and Malignant Disorders. REFERENCES 1. pain. Lakhar B. Ittoop A. Observation is not indicated for a giant fibroadenoma because rapid growth can lead to infarction of the lesion. In adolescents. and prosthesis should be avoided. Ozgur F. Ann Plast Surg 45:451-453. Saunders.Zacharia TT. tissue expanders. Both epithelial and connective tissue components are seen on histopathology and atypia is not a feature. Additional diagnostic studies contribute nothing to the management of this problem. St. Juvenile hypertrophy is usually bilateral. Menachery J: Giant fibroadenoma. Carcinoma is extremely rare in this age group and does not have this clinical course. Cystic changes are more commonly associated with older women during the mature reproductive years and early involution. Mastectomy. Skoll PJ. undergoing bilateral mastectomy for BRCA2 expression. body mass index (BMI) 25.American College of Surgeons SESAP 13 Category 2 Item 22 32-year-old basketball coach. (A) Latissimus dorsi musculocutaneous flap(s) (B) Pedicle to transverse rectus abdominis musculocutaneous flap(s) (C) Tissue expander(s) and implant(s) (D) Transverse rectus abdominis musculocutaneous free flap(s) (E) Deep inferior epigastric vessel free flap(s) page 77 . Patients undergoing bilateral mastectomy. The flap can be moved as a pedicle flap based on the superior epigastric and rotated up into the chest over the costal margin or as a free flap with the flap vessels being the deep inferior epigastric vessels and the recipient vessels either the page 78 . Placement of a tissue expander at operation and subsequent gradual stretching of the skin by repeated inflations of the expander will create a skin envelope large enough to accommodate a larger implant. The skin of the lower abdominal wall (often lax because of previous pregnancies) is used to create a breast mound without an implant. The simplest goals of breast reconstruction are to form a breast mound that approximates the opposite normal size. The transverse rectus abdominis musculocutaneous flap (TRAM) has been an enormous innovation in breast reconstruction. Silicone implants can be placed beneath the skin flaps of the mastectomy as a primary form of reconstruction. This is useful only when skin resection is minimal and the contralateral breast is round without ptosis. and wishes.American College of Surgeons SESAP 13 Critique for Category 2 Item 22 Breast reconstruction at the time of definitive cancer therapy or in a delayed fashion has been an integral part of the treatment of breast cancer for 20 years. and are thus subject to infection and possible extrusion or to capsular contracture. diabetic. The blood supply to this skin comes from the perforating vessels through the rectus abdominis muscle from its axial blood supplies. Several studies have demonstrated that immediate breast reconstruction is not associated with an increase in local recurrence or delay in diagnosis. medical history. a modification of the latissimus known as the fleur de lis pattern allows inclusion of enough skin and fat from the posterolateral chest wall to form an adequate breast mound without the need for an implant. ie. In moderately obese patients. as well as the patient’s body habitus. but still usually creates a round breast reconstruction that requires contralateral mastectomy or reduction to achieve symmetry in most parous females. are excellent candidates for implants and tissue expansion. Preoperative irradiation or postoperative adjuvant radiation also increases the risk of complications. The timing and type of reconstruction chosen should thus depend on how adjuvant therapy will affect the patient and the reconstruction. There are several methods of breast reconstruction. The latissimus muscle and an island of skin overlying it can be rotated from the back to the anterior chest wall to replace skin removed in the mastectomy and to drape muscle over an implant to create a breast mound. The risk of infection and extrusion of the expanders or implants placed at operation is increased greatly in patients who are obese. Even when damage to those vessels from previous operation or radiotherapy is a concern. and the superior epigastric vessels. Federal law mandates insurance coverage for breast reconstruction as part of the treatment of cancer and includes operation on the normal breast to achieve symmetry. This flap receives its blood supply from the thorocodorsal branch of the subscapular vessels. careful elevation of the flap. the deep inferior epigastric. formation of a hard capsule around the implant that may cause pain or deform the breast. Loss of the latissimus muscle action does not impair function noticeably except in the most vigorous athletes. or smokers. Autogenous tissue in the form of the latissimus dorsi musculocutaneous flap is the second major method of breast reconstruction. especially for prophylaxis or early stage disease. will allow the flap to be transferred and receive its blood supply by retrograde flow. Special care to ensure the viability of the mastectomy skin flaps is important in the use of implants. All foreign bodies elicit the inflammatory response. preserving the connection of the latissimus to the branch of the thorocodorsal that supplies the serratus muscle. or to create symmetrical breast mounds by reconstructing the absent breast and altering the opposite breast. Moulds JE. In the lower abdomen. A refinement of free tissue transfer known as the perforator flap has recently been described and popularized. Tondu T. incisions that transect the upper abdominal wall preclude the use of the flap as a location flap based on the superior epigastrics. Br J Surg 87:1455-1472. diabetics. Radiat Oncol Investig 6:81-89. Van Landuyt K. REFERENCES 1.American College of Surgeons SESAP 13 internal mammary vessels or the thorocodorsal vessels. Berg CD: Radiation therapy and breast reconstruction. Purushotham AD: Immediate breast reconstruction after mastectomy for cancer. Blondeel P. et al: Bilateral autogenous breast reconstruction using perforator free flaps: a single center's experience.Hamdi M. This allows preservation of the rectus muscle and obviates the common abdominal weakness and occasional (1% to 8%) hernia associated with the conventional TRAM flap. Obviously. virtually the same territory of skin can be harvested on one or two perforators from the deep inferior epigastric as from the conventional TRAM flap that includes the entire muscle or the whole segment of muscle below the skin island. 1998 page 79 . making a TRAM free flap the superior choice to avoid skin and fat necrosis in patients with potentially impaired blood supply (smokers. 2000 3. This technique harvests the skin area supplied by one or two vessels perforating through the muscle and allows dissection of these vessels out of the muscle down to the underlying axial blood supply and free tissue transfer based on these vessels. The blood supply via the deep inferior epigastric vessels is somewhat greater than via the superior epigastric vessels. 2004 2. obese patients).Malata CM. McIntosh SA. Plast Reconstr Surg 114:83-89. undergoing modified radical mastectomy for invasive ductal cancer (A) Latissimus dorsi musculocutaneous flap(s) (B) Pedicle to transverse rectus abdominis musculocutaneous flap(s) (C) Tissue expander(s) and implant(s) (D) Transverse rectus abdominis musculocutaneous free flap(s) (E) Deep inferior epigastric vessel free flap(s) page 80 .5 packs of cigarettes per day.American College of Surgeons SESAP 13 Category 2 Item 23 55-year-old. BMI 30. smokes 1. medical history. Silicone implants can be placed beneath the skin flaps of the mastectomy as a primary form of reconstruction.American College of Surgeons SESAP 13 Critique for Category 2 Item 23 Breast reconstruction at the time of definitive cancer therapy or in a delayed fashion has been an integral part of the treatment of breast cancer for 20 years. Patients undergoing bilateral mastectomy. This is useful only when skin resection is minimal and the contralateral breast is round without ptosis. There are several methods of breast reconstruction. The latissimus muscle and an island of skin overlying it can be rotated from the back to the anterior chest wall to replace skin removed in the mastectomy and to drape muscle over an implant to create a breast mound. Placement of a tissue expander at operation and subsequent gradual stretching of the skin by repeated inflations of the expander will create a skin envelope large enough to accommodate a larger implant. and the superior epigastric vessels. will allow the flap to be transferred and receive its blood supply by retrograde flow. The skin of the lower abdominal wall (often lax because of previous pregnancies) is used to create a breast mound without an implant. Federal law mandates insurance coverage for breast reconstruction as part of the treatment of cancer and includes operation on the normal breast to achieve symmetry. This flap receives its blood supply from the thorocodorsal branch of the subscapular vessels. The transverse rectus abdominis musculocutaneous flap (TRAM) has been an enormous innovation in breast reconstruction. preserving the connection of the latissimus to the branch of the thorocodorsal that supplies the serratus muscle. The risk of infection and extrusion of the expanders or implants placed at operation is increased greatly in patients who are obese. The flap can be moved as a pedicle flap based on the superior epigastric and rotated up into the chest over the costal margin or as a free flap with the flap vessels being the deep inferior epigastric vessels and the recipient vessels either the page 81 . All foreign bodies elicit the inflammatory response. or to create symmetrical breast mounds by reconstructing the absent breast and altering the opposite breast. careful elevation of the flap. Even when damage to those vessels from previous operation or radiotherapy is a concern. In moderately obese patients. Loss of the latissimus muscle action does not impair function noticeably except in the most vigorous athletes. but still usually creates a round breast reconstruction that requires contralateral mastectomy or reduction to achieve symmetry in most parous females. Autogenous tissue in the form of the latissimus dorsi musculocutaneous flap is the second major method of breast reconstruction. formation of a hard capsule around the implant that may cause pain or deform the breast. or smokers. as well as the patient’s body habitus. The simplest goals of breast reconstruction are to form a breast mound that approximates the opposite normal size. and wishes. a modification of the latissimus known as the fleur de lis pattern allows inclusion of enough skin and fat from the posterolateral chest wall to form an adequate breast mound without the need for an implant. Special care to ensure the viability of the mastectomy skin flaps is important in the use of implants. especially for prophylaxis or early stage disease. and are thus subject to infection and possible extrusion or to capsular contracture. Several studies have demonstrated that immediate breast reconstruction is not associated with an increase in local recurrence or delay in diagnosis. Preoperative irradiation or postoperative adjuvant radiation also increases the risk of complications. The timing and type of reconstruction chosen should thus depend on how adjuvant therapy will affect the patient and the reconstruction. The blood supply to this skin comes from the perforating vessels through the rectus abdominis muscle from its axial blood supplies. diabetic. the deep inferior epigastric. ie. are excellent candidates for implants and tissue expansion. making a TRAM free flap the superior choice to avoid skin and fat necrosis in patients with potentially impaired blood supply (smokers. Van Landuyt K. The blood supply via the deep inferior epigastric vessels is somewhat greater than via the superior epigastric vessels. This technique harvests the skin area supplied by one or two vessels perforating through the muscle and allows dissection of these vessels out of the muscle down to the underlying axial blood supply and free tissue transfer based on these vessels. et al: Bilateral autogenous breast reconstruction using perforator free flaps: a single center's experience. A refinement of free tissue transfer known as the perforator flap has recently been described and popularized. obese patients). Tondu T. diabetics. Blondeel P. This allows preservation of the rectus muscle and obviates the common abdominal weakness and occasional (1% to 8%) hernia associated with the conventional TRAM flap.Malata CM.Hamdi M. Plast Reconstr Surg 114:83-89. 2000 3. Berg CD: Radiation therapy and breast reconstruction. 1998 page 82 . McIntosh SA.Moulds JE. In the lower abdomen. virtually the same territory of skin can be harvested on one or two perforators from the deep inferior epigastric as from the conventional TRAM flap that includes the entire muscle or the whole segment of muscle below the skin island. Obviously. 2004 2. incisions that transect the upper abdominal wall preclude the use of the flap as a location flap based on the superior epigastrics.American College of Surgeons SESAP 13 internal mammary vessels or the thorocodorsal vessels. Purushotham AD: Immediate breast reconstruction after mastectomy for cancer. Radiat Oncol Investig 6:81-89. REFERENCES 1. Br J Surg 87:1455-1472. American College of Surgeons SESAP 13 Category 2 Item 24 35-year-old. athletically active. seeking delayed reconstruction 2 years after mastectomy and adjuvant radiotherapy (A) Latissimus dorsi musculocutaneous flap(s) (B) Pedicle to transverse rectus abdominis musculocutaneous flap(s) (C) Tissue expander(s) and implant(s) (D) Transverse rectus abdominis musculocutaneous free flap(s) (E) Deep inferior epigastric vessel free flap(s) page 83 . preserving the connection of the latissimus to the branch of the thorocodorsal that supplies the serratus muscle. The blood supply to this skin comes from the perforating vessels through the rectus abdominis muscle from its axial blood supplies. ie.American College of Surgeons SESAP 13 Critique for Category 2 Item 24 Breast reconstruction at the time of definitive cancer therapy or in a delayed fashion has been an integral part of the treatment of breast cancer for 20 years. The timing and type of reconstruction chosen should thus depend on how adjuvant therapy will affect the patient and the reconstruction. The risk of infection and extrusion of the expanders or implants placed at operation is increased greatly in patients who are obese. In moderately obese patients. Even when damage to those vessels from previous operation or radiotherapy is a concern. or smokers. Patients undergoing bilateral mastectomy. will allow the flap to be transferred and receive its blood supply by retrograde flow. The flap can be moved as a pedicle flap based on the superior epigastric and rotated up into the chest over the costal margin or as a free flap with the flap vessels being the deep inferior epigastric vessels and the recipient vessels either the page 84 . and the superior epigastric vessels. Special care to ensure the viability of the mastectomy skin flaps is important in the use of implants. There are several methods of breast reconstruction. The latissimus muscle and an island of skin overlying it can be rotated from the back to the anterior chest wall to replace skin removed in the mastectomy and to drape muscle over an implant to create a breast mound. Placement of a tissue expander at operation and subsequent gradual stretching of the skin by repeated inflations of the expander will create a skin envelope large enough to accommodate a larger implant. All foreign bodies elicit the inflammatory response. and wishes. the deep inferior epigastric. diabetic. The transverse rectus abdominis musculocutaneous flap (TRAM) has been an enormous innovation in breast reconstruction. Preoperative irradiation or postoperative adjuvant radiation also increases the risk of complications. and are thus subject to infection and possible extrusion or to capsular contracture. medical history. Silicone implants can be placed beneath the skin flaps of the mastectomy as a primary form of reconstruction. Federal law mandates insurance coverage for breast reconstruction as part of the treatment of cancer and includes operation on the normal breast to achieve symmetry. This is useful only when skin resection is minimal and the contralateral breast is round without ptosis. The skin of the lower abdominal wall (often lax because of previous pregnancies) is used to create a breast mound without an implant. or to create symmetrical breast mounds by reconstructing the absent breast and altering the opposite breast. but still usually creates a round breast reconstruction that requires contralateral mastectomy or reduction to achieve symmetry in most parous females. Loss of the latissimus muscle action does not impair function noticeably except in the most vigorous athletes. especially for prophylaxis or early stage disease. Several studies have demonstrated that immediate breast reconstruction is not associated with an increase in local recurrence or delay in diagnosis. formation of a hard capsule around the implant that may cause pain or deform the breast. as well as the patient’s body habitus. This flap receives its blood supply from the thorocodorsal branch of the subscapular vessels. are excellent candidates for implants and tissue expansion. careful elevation of the flap. a modification of the latissimus known as the fleur de lis pattern allows inclusion of enough skin and fat from the posterolateral chest wall to form an adequate breast mound without the need for an implant. Autogenous tissue in the form of the latissimus dorsi musculocutaneous flap is the second major method of breast reconstruction. The simplest goals of breast reconstruction are to form a breast mound that approximates the opposite normal size. Purushotham AD: Immediate breast reconstruction after mastectomy for cancer. Van Landuyt K. Radiat Oncol Investig 6:81-89. In the lower abdomen. incisions that transect the upper abdominal wall preclude the use of the flap as a location flap based on the superior epigastrics. diabetics. Tondu T. This technique harvests the skin area supplied by one or two vessels perforating through the muscle and allows dissection of these vessels out of the muscle down to the underlying axial blood supply and free tissue transfer based on these vessels. Br J Surg 87:1455-1472. McIntosh SA. making a TRAM free flap the superior choice to avoid skin and fat necrosis in patients with potentially impaired blood supply (smokers. obese patients).American College of Surgeons SESAP 13 internal mammary vessels or the thorocodorsal vessels. The blood supply via the deep inferior epigastric vessels is somewhat greater than via the superior epigastric vessels. Plast Reconstr Surg 114:83-89. A refinement of free tissue transfer known as the perforator flap has recently been described and popularized. 1998 page 85 .Hamdi M.Moulds JE. Blondeel P. 2000 3. et al: Bilateral autogenous breast reconstruction using perforator free flaps: a single center's experience. Obviously. REFERENCES 1.Malata CM. virtually the same territory of skin can be harvested on one or two perforators from the deep inferior epigastric as from the conventional TRAM flap that includes the entire muscle or the whole segment of muscle below the skin island. This allows preservation of the rectus muscle and obviates the common abdominal weakness and occasional (1% to 8%) hernia associated with the conventional TRAM flap. Berg CD: Radiation therapy and breast reconstruction. 2004 2. American College of Surgeons SESAP 13 Category 2 Item 25 Tamoxifen increases survival (A) Ductal carcinoma in situ (DCIS) (B) Invasive ductal cancer (C) Both (D) Neither page 86 . Petruzella S. without a difference in overall survival. and mastectomy. NSABP B-06 compared excision alone. Cha I. J Natl Cancer Inst 95:1692-1702.Davidson NE. Harris JR: Local management of invasive cancer: breast. tamoxifen decreased the odds of recurrence by 25% and the odds of death by 16%. in Harris JR. ed 3. Osborne CK (eds): Diseases of the Breast. should prompt referral to a high-risk protocol. 2004 3. In a meta-analysis comparing over 20 trials evaluating tamoxifen plus chemotherapy versus chemotherapy alone in patients with invasive breast cancer. NSABP B-17 demonstrated a reduction in recurrence from 31. This suggests that either diagnosis. Tamoxifen improves survival and decreases recurrence in patients with invasive breast cancer.Morrow M.Claus EB. 2004 6. 2005 2. 2004. in combination with a personal or family history. excision and radiotherapy. Matloff E. pp 945-950 4. The Van Nuys prognostic index—originally nuclear grade combined with the presence or absence of comedo necrosis—now also includes tumor size and margin width. The addition of radiotherapy to excision alone decreased local recurrence from 42% to 12% at 9 years. Morrow M. or a history of thromboembolic disease. Goonewardene SA. With respect to epidemiology.Cornfield DB. the side effects of tamoxifen may offset the relative decrease in cancer recurrence. Lippincott Williams & Wilkins.Leonard GD.7% to 15. J Natl Cancer Inst 96:906-920. a history of stroke.Kerlikowske K. pp 719-744 page 87 . ed 3. 2003 5. but does not improve overall survival. there are important differences as well as similarities in epidemiology and treatment of women with this spectrum of disease. Carter D: Prevalence of BRCA1 and BRCA2 mutations in women diagnosed with ductal carcinoma in situ. REFERENCES 1. Molinaro A. Philadelphia. et al: The prognostic significance of multiple morphologic features and biologic markers in ductal carcinoma in situ of the breast: a study of a large cohort of patients treated with surgery alone. Osborne CK: Adjuvant systemic therapy treatment guidelines. Lippman ME. Overall 12-year survival (86%-87%) was not different. complexities and challenges. Morrow M. the prevalence of mutations in both BRCA1 and BRCA2 is now known to be similar in women with either DCIS or invasive ductal cancer (1% to 3% of all cancers).American College of Surgeons SESAP 13 Critique for Category 2 Item 25 Despite the well-known association of ductal carcinoma in situ (DCIS) and invasive breast cancer. Palazzo JP. Philadelphia.7% when radiation therapy was added to excision for women with DCIS. Swain SM: Ductal carcinoma in situ. Similar results have been obtained in women with invasive cancer. et al: Characteristics associated with recurrence among women with ductal carcinoma in situ treated by lumpectomy. Osborne CK (eds): Diseases of the Breast. It is used to assess the risk of DCIS to recur or present as invasive cancer after excision. 2004. JAMA 293:964-969. tamoxifen may decrease recurrence of cancer (NSABP B-24). not to describe prognosis for women with invasive cancer. Lippman ME. In patients with DCIS. Schwartz GF. Cancer 100:2317-2327. In patients with endometrial cancer. Lippincott Williams & Wilkins. in Harris JR. American College of Surgeons SESAP 13 Category 2 Item 26 Radiation therapy increases survival (A) Ductal carcinoma in situ (DCIS) (B) Invasive ductal cancer (C) Both (D) Neither page 88 . Cornfield DB. the side effects of tamoxifen may offset the relative decrease in cancer recurrence. the prevalence of mutations in both BRCA1 and BRCA2 is now known to be similar in women with either DCIS or invasive ductal cancer (1% to 3% of all cancers). Osborne CK (eds): Diseases of the Breast. 2004. Swain SM: Ductal carcinoma in situ.Morrow M. excision and radiotherapy.Claus EB. Lippincott Williams & Wilkins. Goonewardene SA. Tamoxifen improves survival and decreases recurrence in patients with invasive breast cancer. Osborne CK (eds): Diseases of the Breast. ed 3. Cha I. Morrow M. not to describe prognosis for women with invasive cancer. Overall 12-year survival (86%-87%) was not different. Cancer 100:2317-2327. in combination with a personal or family history. Morrow M. In a meta-analysis comparing over 20 trials evaluating tamoxifen plus chemotherapy versus chemotherapy alone in patients with invasive breast cancer. tamoxifen decreased the odds of recurrence by 25% and the odds of death by 16%. Harris JR: Local management of invasive cancer: breast. Palazzo JP. In patients with endometrial cancer. The addition of radiotherapy to excision alone decreased local recurrence from 42% to 12% at 9 years. Molinaro A. in Harris JR. 2004 3. 2005 2. This suggests that either diagnosis. Schwartz GF. Osborne CK: Adjuvant systemic therapy treatment guidelines. JAMA 293:964-969. Lippman ME. should prompt referral to a high-risk protocol. Similar results have been obtained in women with invasive cancer. It is used to assess the risk of DCIS to recur or present as invasive cancer after excision. Lippman ME. 2004 6. et al: The prognostic significance of multiple morphologic features and biologic markers in ductal carcinoma in situ of the breast: a study of a large cohort of patients treated with surgery alone. pp 945-950 4. Lippincott Williams & Wilkins. 2004.7% when radiation therapy was added to excision for women with DCIS. NSABP B-06 compared excision alone. but does not improve overall survival.Leonard GD. pp 719-744 page 89 .Kerlikowske K. complexities and challenges. REFERENCES 1. ed 3. Philadelphia. In patients with DCIS. 2003 5. Matloff E. without a difference in overall survival.7% to 15. With respect to epidemiology. tamoxifen may decrease recurrence of cancer (NSABP B-24). The Van Nuys prognostic index—originally nuclear grade combined with the presence or absence of comedo necrosis—now also includes tumor size and margin width. and mastectomy. Carter D: Prevalence of BRCA1 and BRCA2 mutations in women diagnosed with ductal carcinoma in situ. there are important differences as well as similarities in epidemiology and treatment of women with this spectrum of disease. Philadelphia. a history of stroke.American College of Surgeons SESAP 13 Critique for Category 2 Item 26 Despite the well-known association of ductal carcinoma in situ (DCIS) and invasive breast cancer. NSABP B-17 demonstrated a reduction in recurrence from 31. Petruzella S. in Harris JR. et al: Characteristics associated with recurrence among women with ductal carcinoma in situ treated by lumpectomy. J Natl Cancer Inst 95:1692-1702.Davidson NE. J Natl Cancer Inst 96:906-920. or a history of thromboembolic disease. American College of Surgeons SESAP 13 Category 2 Item 27 Prevalence of BRCA2 mutation 1% to 3% (A) Ductal carcinoma in situ (DCIS) (B) Invasive ductal cancer (C) Both (D) Neither page 90 . 2004. Osborne CK (eds): Diseases of the Breast. 2004 6. The addition of radiotherapy to excision alone decreased local recurrence from 42% to 12% at 9 years. The Van Nuys prognostic index—originally nuclear grade combined with the presence or absence of comedo necrosis—now also includes tumor size and margin width. NSABP B-06 compared excision alone. 2004. This suggests that either diagnosis. Matloff E.Claus EB. NSABP B-17 demonstrated a reduction in recurrence from 31. Osborne CK: Adjuvant systemic therapy treatment guidelines. Harris JR: Local management of invasive cancer: breast. and mastectomy.Morrow M. et al: The prognostic significance of multiple morphologic features and biologic markers in ductal carcinoma in situ of the breast: a study of a large cohort of patients treated with surgery alone. or a history of thromboembolic disease. In a meta-analysis comparing over 20 trials evaluating tamoxifen plus chemotherapy versus chemotherapy alone in patients with invasive breast cancer. Lippincott Williams & Wilkins. JAMA 293:964-969. 2004 3. complexities and challenges. not to describe prognosis for women with invasive cancer. Lippincott Williams & Wilkins. Tamoxifen improves survival and decreases recurrence in patients with invasive breast cancer. It is used to assess the risk of DCIS to recur or present as invasive cancer after excision.Kerlikowske K. REFERENCES 1. Similar results have been obtained in women with invasive cancer. should prompt referral to a high-risk protocol. pp 719-744 page 91 .American College of Surgeons SESAP 13 Critique for Category 2 Item 27 Despite the well-known association of ductal carcinoma in situ (DCIS) and invasive breast cancer. Schwartz GF. Overall 12-year survival (86%-87%) was not different. Morrow M. Lippman ME. a history of stroke. Lippman ME. excision and radiotherapy. J Natl Cancer Inst 96:906-920. et al: Characteristics associated with recurrence among women with ductal carcinoma in situ treated by lumpectomy. in combination with a personal or family history. but does not improve overall survival. Carter D: Prevalence of BRCA1 and BRCA2 mutations in women diagnosed with ductal carcinoma in situ. Palazzo JP.7% when radiation therapy was added to excision for women with DCIS. In patients with endometrial cancer. 2003 5. Philadelphia. J Natl Cancer Inst 95:1692-1702. Petruzella S. in Harris JR. Cancer 100:2317-2327. pp 945-950 4.Davidson NE.Leonard GD. the prevalence of mutations in both BRCA1 and BRCA2 is now known to be similar in women with either DCIS or invasive ductal cancer (1% to 3% of all cancers).Cornfield DB. Osborne CK (eds): Diseases of the Breast. there are important differences as well as similarities in epidemiology and treatment of women with this spectrum of disease. tamoxifen may decrease recurrence of cancer (NSABP B-24). Philadelphia. tamoxifen decreased the odds of recurrence by 25% and the odds of death by 16%. Swain SM: Ductal carcinoma in situ. 2005 2. Cha I. in Harris JR. With respect to epidemiology. ed 3. In patients with DCIS. Molinaro A. ed 3. Morrow M. the side effects of tamoxifen may offset the relative decrease in cancer recurrence. Goonewardene SA. without a difference in overall survival.7% to 15. American College of Surgeons SESAP 13 Category 2 Item 28 Lumpectomy with clear margins (A) Lobular carcinoma in situ (B) Ductal carcinoma in situ (C) Both (D) Neither page 92 . There appears to be a higher incidence of osteoporosis of the bone with these agents. Tamoxifen reduces the development of invasive cancer in patients with DCIS and LCIS by up to 50%. WebMD. Souba WW: Breast complaints. These effects have been demonstrated in the contralateral breast of women with documented invasive cancer. This class of drugs is currently being studied for DCIS therapy.Lind DS. Walts D. recurrence is usually at the initial site. anti-estrogen therapy is used to reduce progression to invasive lesions. Land SR. 2005 5. et al (eds): ACS Surgery: Principles and Practice 2006. Cancer 100:238-244. The newer agents are associated with less risk of thrombotic events and endometrial cancer than tamoxifen. DCIS is believed to be the precursor of nearly all invasive ductal carcinomas. Mamounas E. these drugs do not have FDA approval for this use. and may also reduce invasive recurrences.Kalidas M. Am J Surg 189:610-615. Clear margins are not required. and probably are associated with missed foci of microinvasion on lumpectomy specimens. which in postmenopausal women is dependent on proteins called aromatases. Jurkovich GJ. Clin Breast Cancer 6:27-37. REFERENCES 1. Many prefer the term lobular neoplasia to indicate this difference in behavior. Both breasts are at equal risk. Addition of radiation therapy reduces local recurrence by 50%. Like tamoxifen. but with very different patterns of behavior that affect clinical management. Skin. Fisher B.American College of Surgeons SESAP 13 Critique of Category 2 Item 28 Ductal carcinoma in situ (DCIS) and lobular carcinoma in situ (LCIS) are both predictors of invasive breast carcinoma. They are highly effective for postmenopausal women with metastatic cancer or treated invasive breast cancer. 1994 3. and Soft page 93 . New York. and ipsilateral mastectomy is not indicated. LCIS is viewed as a predictor of increased risk of development of invasive breast cancer. When it is incompletely excised. Many physicians presume that because they are so effective in invasive breast cancer that has the estrogen receptor. in the absence of research results to show their benefit in this setting. et al: Risk counseling and management in patients with lobular carcinoma in situ.Fisher ER. and local recurrences of the type seen with DCIS are uncommon. Aromatase inhibitors have not been adequately studied in LCIS. Breast. et al: Pathologic findings from the National Surgical Adjuvant Breast and Bowel Project: twelve-year observations concerning lobular carcinoma in situ. Lymph node dissection is not considered necessary in most cases. World J Surg 18:45-57. Brown P: Aromatase inhibitors for the treatment and prevention of breast cancer. Radiation therapy has no known benefit in the prevention of future carcinogenesis in these patients. but not as an anatomic marker of the actual site of cancer development. Currently. Look R. The likelihood of invasive disease increases with time. However. There is no role for axillary dissection in the absence of invasive disease. Aromatase inhibitors are a new class of drugs used to treat breast cancer. as with DCIS. 2004 2. and clear margins are the sine qua non of breast-conserving surgery. in Souba WW. 2005 4. Fink MP. Bland KI: Management of in situ and minimally invasive breast carcinoma. Smith BL.Frykberg ER. It is not recommended for premenopausal women. Kaiser LR. Axillary metastases are rare (1% to 2%). Nelson J. they may be effective for DCIS when the estrogen receptor is present. they block the production of estrogen.Garreau JR. Parker RG. 1998 page 94 . et al: Axillary node dissection in ductal carcinoma in situ. Rees K. Leung KM. pp 205-223 6. sec 1. chap 3. Berkbigler D.American College of Surgeons SESAP 13 Tissue. Am J Clin Oncol 21:109-110. American College of Surgeons SESAP 13 Category 2 Item 29 Aromatase inhibitors (A) Lobular carcinoma in situ (B) Ductal carcinoma in situ (C) Both (D) Neither page 95 . It is not recommended for premenopausal women. Am J Surg 189:610-615. Aromatase inhibitors have not been adequately studied in LCIS. Aromatase inhibitors are a new class of drugs used to treat breast cancer. et al (eds): ACS Surgery: Principles and Practice 2006. Jurkovich GJ.Fisher ER. recurrence is usually at the initial site. REFERENCES 1. Like tamoxifen. Many prefer the term lobular neoplasia to indicate this difference in behavior. Many physicians presume that because they are so effective in invasive breast cancer that has the estrogen receptor. Axillary metastases are rare (1% to 2%). in Souba WW.Kalidas M. Clear margins are not required. WebMD. There is no role for axillary dissection in the absence of invasive disease. and probably are associated with missed foci of microinvasion on lumpectomy specimens. Addition of radiation therapy reduces local recurrence by 50%. LCIS is viewed as a predictor of increased risk of development of invasive breast cancer. Brown P: Aromatase inhibitors for the treatment and prevention of breast cancer. Mamounas E. and ipsilateral mastectomy is not indicated. Tamoxifen reduces the development of invasive cancer in patients with DCIS and LCIS by up to 50%. Kaiser LR. 2004 2. Currently. which in postmenopausal women is dependent on proteins called aromatases. they block the production of estrogen.Lind DS.Garreau JR. 2005 5. Look R.American College of Surgeons SESAP 13 Critique of Category 2 Item 29 Ductal carcinoma in situ (DCIS) and lobular carcinoma in situ (LCIS) are both predictors of invasive breast carcinoma. DCIS is believed to be the precursor of nearly all invasive ductal carcinomas. they may be effective for DCIS when the estrogen receptor is present. Radiation therapy has no known benefit in the prevention of future carcinogenesis in these patients. 2005 4. and clear margins are the sine qua non of breast-conserving surgery. The newer agents are associated with less risk of thrombotic events and endometrial cancer than tamoxifen. Clin Breast Cancer 6:27-37. Souba WW: Breast complaints. These effects have been demonstrated in the contralateral breast of women with documented invasive cancer. Breast. and Soft page 96 . these drugs do not have FDA approval for this use. Nelson J. Land SR. New York. Smith BL. When it is incompletely excised. Cancer 100:238-244. anti-estrogen therapy is used to reduce progression to invasive lesions. Walts D. There appears to be a higher incidence of osteoporosis of the bone with these agents. and may also reduce invasive recurrences. Skin. Bland KI: Management of in situ and minimally invasive breast carcinoma. However. et al: Pathologic findings from the National Surgical Adjuvant Breast and Bowel Project: twelve-year observations concerning lobular carcinoma in situ. World J Surg 18:45-57. Both breasts are at equal risk. Fink MP. et al: Risk counseling and management in patients with lobular carcinoma in situ. as with DCIS. They are highly effective for postmenopausal women with metastatic cancer or treated invasive breast cancer. Fisher B. Lymph node dissection is not considered necessary in most cases.Frykberg ER. in the absence of research results to show their benefit in this setting. but not as an anatomic marker of the actual site of cancer development. The likelihood of invasive disease increases with time. 1994 3. and local recurrences of the type seen with DCIS are uncommon. This class of drugs is currently being studied for DCIS therapy. but with very different patterns of behavior that affect clinical management. Am J Clin Oncol 21:109-110. et al: Axillary node dissection in ductal carcinoma in situ. Rees K. Berkbigler D. sec 1. pp 205-223 6. Leung KM.American College of Surgeons SESAP 13 Tissue. 1998 page 97 .Parker RG. chap 3. American College of Surgeons SESAP 13 Category 2 Item 30 Bilateral risk (A) Lobular carcinoma in situ (B) Ductal carcinoma in situ (C) Both (D) Neither page 98 . Like tamoxifen. they block the production of estrogen. anti-estrogen therapy is used to reduce progression to invasive lesions. Look R. Lymph node dissection is not considered necessary in most cases. Clin Breast Cancer 6:27-37. 1994 3. Both breasts are at equal risk. The likelihood of invasive disease increases with time.Frykberg ER. Fink MP. and local recurrences of the type seen with DCIS are uncommon. et al: Risk counseling and management in patients with lobular carcinoma in situ. Skin. However. Jurkovich GJ.American College of Surgeons SESAP 13 Critique of Category 2 Item 30 Ductal carcinoma in situ (DCIS) and lobular carcinoma in situ (LCIS) are both predictors of invasive breast carcinoma. Tamoxifen reduces the development of invasive cancer in patients with DCIS and LCIS by up to 50%. REFERENCES 1. Mamounas E. Nelson J. Currently. Souba WW: Breast complaints. Am J Surg 189:610-615. Kaiser LR. and clear margins are the sine qua non of breast-conserving surgery. et al (eds): ACS Surgery: Principles and Practice 2006. these drugs do not have FDA approval for this use. Many prefer the term lobular neoplasia to indicate this difference in behavior. 2004 2. Aromatase inhibitors are a new class of drugs used to treat breast cancer. but with very different patterns of behavior that affect clinical management. in Souba WW. 2005 5.Kalidas M. they may be effective for DCIS when the estrogen receptor is present. LCIS is viewed as a predictor of increased risk of development of invasive breast cancer. in the absence of research results to show their benefit in this setting. New York.Lind DS. There is no role for axillary dissection in the absence of invasive disease. and Soft page 99 . Cancer 100:238-244.Fisher ER. Axillary metastases are rare (1% to 2%). Bland KI: Management of in situ and minimally invasive breast carcinoma. There appears to be a higher incidence of osteoporosis of the bone with these agents. which in postmenopausal women is dependent on proteins called aromatases. as with DCIS. DCIS is believed to be the precursor of nearly all invasive ductal carcinomas. Fisher B. Walts D. Breast. The newer agents are associated with less risk of thrombotic events and endometrial cancer than tamoxifen. World J Surg 18:45-57. and ipsilateral mastectomy is not indicated. It is not recommended for premenopausal women. and may also reduce invasive recurrences. and probably are associated with missed foci of microinvasion on lumpectomy specimens.Garreau JR. Addition of radiation therapy reduces local recurrence by 50%. These effects have been demonstrated in the contralateral breast of women with documented invasive cancer. Land SR. Many physicians presume that because they are so effective in invasive breast cancer that has the estrogen receptor. Smith BL. This class of drugs is currently being studied for DCIS therapy. Clear margins are not required. WebMD. When it is incompletely excised. but not as an anatomic marker of the actual site of cancer development. Aromatase inhibitors have not been adequately studied in LCIS. et al: Pathologic findings from the National Surgical Adjuvant Breast and Bowel Project: twelve-year observations concerning lobular carcinoma in situ. recurrence is usually at the initial site. 2005 4. Brown P: Aromatase inhibitors for the treatment and prevention of breast cancer. They are highly effective for postmenopausal women with metastatic cancer or treated invasive breast cancer. Radiation therapy has no known benefit in the prevention of future carcinogenesis in these patients. 1998 page 100 .Parker RG. et al: Axillary node dissection in ductal carcinoma in situ. Rees K. Leung KM. Berkbigler D.American College of Surgeons SESAP 13 Tissue. sec 1. chap 3. Am J Clin Oncol 21:109-110. pp 205-223 6. esophagogastroduodenoscopy. He has never had an abdominal operation. He has no diverticula in his colon and no internal or external hemorrhoids.American College of Surgeons SESAP 13 Category 3 Item 1 A 65-year-old man has had 6 episodes of lower gastrointestinal bleeding over the past 2 years manifested by passage of maroon stools. Other medical problems include hypertension and coronary artery disease. No bleeding source has been defined during these episodes. each of which required hospitalization and transfusion. and colonoscopy have all been unremarkable. selective mesenteric angiogram. to a total of 28 units of blood. tagged red cell scan. He is now referred for consultation to try to define and treat bleeding. abdominal computed tomographic (CT) scan. Which of the following options is the LEAST appealing from a risk-benefit standpoint? (A) Exploratory laparotomy with push enteroscopy (B) Capsule endoscopy (C) Meckel’s scan (D) Selective mesenteric angiogram with a heparin challenge (E) Laparotomy and abdominal colectomy page 101 . no increase in mortality rates has been reported. a Meckel scan is a harmless test with virtually no risk. a rapid bowel preparation is usually administered before the study.1 mL/minute). Abdominal computed tomographic (CT) scan is of limited use in localizing gastrointestinal bleeding unless a mass is detected. Tagged RBC scans require a lower rate of bleeding to be positive (0. Aspiration of bile is necessary for an upper gastrointestinal source to be definitively ruled out. surgical options would be limited to undersewing of the individual lesions under guidance of the endoscopist to localize them. A selective mesenteric angiogram will require bleeding at the rate of 0. The source of occult lower gastrointestinal bleeding can be identified in over one third of patients.04 to 0. This approach has had limited success. but the results are often misleading or equivocal from the standpoint of localizing the bleeding site. For that reason. options for localizing the source of bleeding depend on the rate of bleeding. A swallowed capsule acquires video images as it moves through the gastrointestinal tract and is excreted in the stool. Selective mesenteric angiography performed after a heparin challenge to induce bleeding that can then be localized angiographically is another diagnostic option. If the lesions are more diffuse or multiple. This technique requires time-consuming viewing and image analysis. If hypotension and tachycardia are present. and cannot be page 102 . Colonoscopy is also useful. laparotomy and abdominal colectomy would be an unwise choice. The endoscopic images are transmitted through electronic signals to a computer for processing. however. Capsule endoscopy is the newest method to identify occult small intestinal sources of bleeding. however. The remaining options are all reasonable ones that have shown modest success in patients with low volume chronic recurrent bleeding. Tissue plasminogen activator or tolazoline may also be used to induce bleeding.5 to 1 mL/minute for extravasation of contrast to be visible. even though the likely benefit is small. In a patient of this age group. but here too. The patient described in this scenario has had negative colonoscopy without even diverticula being noted. Although this method might stimulate uncontrolled bleeding. Patients who have had a complete evaluation during bleeding episodes and yet continue to have recurrent admissions and require multiple units of blood present a particularly difficult problem and may require unusual measures for diagnosis. The most common causes of colonic bleeding. In the absence of bile aspiration. recurrent lower gastrointestinal hemorrhage can be one of the most difficult problems to diagnose and treat correctly. upper endoscopy may be necessary to ensure that the source is not in the stomach or duodenum. esophagogastroduodenoscopy has been negative and so an upper gastrointestinal source is unlikely. If angiodysplasia is confined to one area. the bleeding source might be upper gastrointestinal (above the ligament of Treitz) with rapid transit of blood through the bowel. Exploratory laparotomy with intraoperative push enteroscopy is a method of inspecting the entire small intestine and is helpful to identify sites of angiodysplasia in the small intestine. Passage of a nasogastric tube to ensure that an upper gastrointestinal source is not present is important. diverticula and angiodysplasia. the results may be misleading if blood is still present in the lumen of the bowel. the first step is obviously resuscitation and stabilization. The small bowel remains then the most likely site of the bleeding.American College of Surgeons SESAP 13 Critique for Category 3 Item 1 Chronic. For this reason. a Meckel’s diverticulum would be extremely unlikely. Similarly. should be visible on a colonoscopy and are therefore unlikely to be the sources of his problem. that segment can be resected. When a patient presents with bright red blood or maroon stools. If an upper gastrointestinal source has been ruled out. Doherty GM. and capsule endoscopy is costly and not universally available. Dumbleton SA. Lippincott Williams & Wilkins.Ryan JM.Kim LT. Lillemoe KD. however.Soderman C. Tumino E. Smith TP: Nonlocalized lower gastrointestinal bleeding: provocative bleeding studies with intraarterial tPA. Turnage RH: Acute gastrointestinal hemorrhage. and tolazoline. Parisi G. 2006. et al: The role of video capsule endoscopy for evaluating obscure gastrointestinal bleeding: usefulness of early use. J Gastroenterol 40:256-259. in Mulholland MW. pp 1050-1065 3. heparin. REFERENCES 1. Uribe A: Enteroscopy as a tool for diagnosing gastrointestinal bleeding requiring blood transfusion. J Vasc Interv Radiol 12:1273-1277.American College of Surgeons SESAP 13 used if the patient has areas of stricture or narrowing that might obstruct passage of the capsule through the intestine. In a patient without previous abdominal operations and with no obstructing lesions seen on CT scan. The capsule may miss a lesion. 2001 4.Bresci G. 2005 2. Maier RV. ed 4. Philadelphia. Bertoni M. a capsule endoscopy would be a reasonable diagnostic test to try to localize the source of chronic hemorrhage. Surg Laparosc Endosc Percutan Tech 11:97-102. Heldmann M. et al (eds): Greenfield's Surgery: Scientific Principles and Practice. Key SM. 2001 page 103 . American College of Surgeons SESAP 13 Category 3 Item 2 Which of the following is the most effective treatment for chronic anal fissure? (A) Nitroglycerin ointment (B) Fissurectomy (C) Internal anal sphincterotomy (D) Botulinum toxin injection (E) Diltiazam ointment page 104 . 2005 2. Because even internal anal sphincterotomy has a small risk of incontinence to flatus and stool. Nevertheless. most physicians will reasonably prescribe nonoperative treatment prior to recommending surgical sphincterotomy. Because these treatments are associated with little or no risk and lateral internal sphincterotomy still has possible complications. Dis Colon Rectum 47:422-431. Several meta-analyses have demonstrated that medical therapy for chronic anal fissure and even for acute fissure has a chance of cure that is only marginally better than placebo. Although superficial acute fissures may respond well to nonoperative management. should be associated with a less than 5% risk of impairing sphincter continence. Options for medical treatment of fissures include nitroglycerin ointment.Acheson AG.American College of Surgeons SESAP 13 Critique for Category 3 Item 2 Anal fissures are painful and debilitating and represent a source of misery far out of proportion to their minute size. diltiazam cream.Nelson R: A systematic review of medical therapy for anal fissure. and anal dilation or stretch. Scholefield JH: Anal fissure: the changing management of a surgical condition. internal anal sphincterotomy is the most effective treatment for chronic anal fissure and. A multitude of studies have compared these treatments in randomized controlled trials. Anal dilation and fissurectomy have essentially been abandoned because both are associated with unacceptable rates of incontinence and the fissure healing rate is also poor. and botulinum toxin injections. chronic anal fissures are more refractory. Three distinct surgical procedures have been used to treat fissures: fissurectomy. conservative nonoperative methods have been favored as a first option for patients with chronic anal fissures. REFERENCES 1. 2004 page 105 . if performed properly. lateral internal anal sphincterotomy. Langenbecks Arch Surg 390:1-7. American College of Surgeons SESAP 13 Category 3 Item 3 An 82-year-old nursing home resident is evaluated for fecal incontinence. She has Alzheimer’s dementia and chronic obstructive pulmonary disease treated with corticosteroids and supplemental oxygen. The most appropriate operative strategy would be (A) transabdominal rectosigmoid resection and rectopexy (B) anterior mesh rectopexy (Ripstein procedure) (C) laparoscopic rectosigmoid resection and rectopexy (D) perineal rectosigmoidectomy (E) Thiersch wire page 106 . Air contrast barium enema shows a redundant colon and no masses. Physical examination reveals the abnormality shown in the photograph. constipation often remains a vexing problem postoperatively. Anderson JH. The operation can be performed laparoscopically or by an open approach. and the prolapse should be eliminated if it can be done safely. the levators can also be plicated to narrow the pelvic outlet sufficiently that the incontinence can be improved. The rectum should be mobilized down into the pelvis and retracted cephalad. Arch Surg 140:63-73. perineal approaches are reasonable options.Madiba TE. For patients who are young and fit. frail patient. A more appropriate and successful approach is a perineal rectosigmoidectomy. Although the incontinence may not be reversed when the prolapse is repaired. the transabdominal approach is most appropriate. 2005 page 107 . Both of these approaches have over a 90% likelihood of success. perineal procedures are better tolerated by the elderly. Rectal prolapse present for a significant period will result in fecal incontinence. For this reason. This procedure has largely been abandoned because of the difficulty in regulating bowel function and the possibility of making the wire too tight as well as the possibility of infection developing. 2004 2. Wexner SD: Surgical management of rectal prolapse. A very old and seldom used technique in the present day is the use of Thiersch wire placed submucosally near the top of the anal canal. some return of function once the prolapse has been eliminated is not uncommon. Because patients with rectal prolapse often have a redundant sigmoid colon and pre-existing constipation. REFERENCES 1. many people favor resection of the rectosigmoid with a primary anastomosis and suture rectopexy to the sacrum. Dis Colon Rectum 47:103-107. Finlay IG: Strategy for selection of type of operation for rectal prolapse based on clinical criteria. This procedure can frequently be performed with regional or even local anesthetic in the prone jackknife position.American College of Surgeons SESAP 13 Critique for Category 3 Item 3 The patient shown in this photograph has a full-thickness rectal prolapse. McKee RF. Before the anastomosis is performed. Rectal prolapse is uncomfortable and can cause both bleeding and obstruction of bowel function. It can be fixed to the sacrum either by sutures directly or by placement of a strip of mesh secured first to the sacrum and then to the rectum well below the peritoneal reflection. Baig MK. The redundant rectum is prolapsed from the anus and the rectum is detached approximately 1 cm above the dentate line. Although the risk of prolapse recurrence is higher.Brown AJ. If the patient is elderly and frail with medical problems that preclude safe abdominal surgery. The redundant rectum is then dissected and resected with a primary interrupted anastomosis performed from the proximal rectum to the dentate line. When considering palliative operation. endoscopic. a dilated stomach.American College of Surgeons SESAP 13 Category 3 Item 4 A 45-year-old man has an unresectable pancreatic carcinoma. He has intractable vomiting from duodenal obstruction. or surgical biliary drainage may improve his appetite and lessen his pruritis (C) The patient should participate in the choice of treatment plan after being informed of options and risks (D) If the patient desires. which of the following statements is NOT true? (A) Gastrojejunostomy may palliate his intractable vomiting (B) Percutaneous. his family should assist him in making decisions (E) Palliative intervention or operation is not indicated because his survival is less than 3 months page 108 . and symptomatic jaundice. There are no other co-morbid factors. patient’s wish for surgery. interventional radiology. Surgical bypass may be more desirable in healthier patients who have longer life expectancy and is the preferred approach for patients who have co-existing duodenal obstruction. well-defined radiographic locus of obstruction. procedures may involve multiple specialties such as surgical subspecialties. radiation therapy. improvement of the patient’s and family’s well-being. a palliative procedure would be reasonable. and physical/occupational therapy. poor general condition. patient refusal. Life expectancy projections are often inaccurate.Ross EL. and marked abdominal distention that suggests absence of extensive infiltrative disease. absence of previously failed chemotherapy and no previous abdominal radiation therapy. Surgical bypass and endoscopic stenting are equally effective for relief of symptoms caused by obstructive jaundice due to malignancy. Currently patients may be offered procedures prior to urgent presentation to improve quality of life. anesthesia. and preserving the individual’s function when possible are additional appropriate goals for any palliative procedure.American College of Surgeons SESAP 13 Critique for Category 3 Item 4 Palliative surgery can be used to treat intractable symptoms for patients with unresectable disease. During the assessment of psychosocial factors. and multiple sites of obstruction. absence of ascites. Surgical palliative care is not synonymous with end-of-life care. Surg Clin North Am 85:191207. 2005 2. As a result of these goals. the patient and if the patient desires. normal nutritional status. REFERENCES 1. psychosocial factors. assistance with the activities of daily living. pain management specialties. at a later time interval for patients treated endoscopically. and available health care resources. Along with symptom management. Favorable factors for operative management of malignant intestinal obstruction include: age under 65. should be available to assist him in making decisions. Given this patient’s age and symptoms. his family. The old approach for palliative surgery was to consider an intervention only when the patient showed an urgent need for symptom control. this patient meets criteria for operative management and he is a candidate for operation if he so desires. Abrahm J: Preparation of the patient for palliative procedures. Complications occur earlier when surgical bypass is selected and on the contrary. Contraindications include recent demonstration of extensive intra-abdominal disease at laparotomy. Surg Clin North Am 85:169-190.Dunn GP: Surgical palliative care: an enduring framework for surgical care. what is feasible and what is possible. 2005 page 109 . medical oncology. absence of palpable abdominal masses. The assessment for a palliative procedure is a four-step process that includes: patient factors. specialties with endoscopic expertise. American College of Surgeons SESAP 13 Category 3 Item 5 A 30-year-old previously healthy woman has burning peri-umbilical pain radiating to her back. The next step in her treatment should be (A) repeat CT scan in 2 months and operation if the lesion is increasing in size (B) percutaneous drainage to evacuate the entire cyst (C) cyst gastrostomy and biopsy of the cyst wall (D) exploratory laparotomy and enucleation of the mass (E) pancreatoduodenectomy page 110 . Endoscopic ultrasonography with aspiration of fluid yields benign inflammatory cells on cytology that are mucicarmine-positive. The abdominal computed tomographic (CT) scan shown is obtained. The fluid aspirate carcinoembryonic antigen (CEA) level is 656 mg/dL with a normal amylase level. Fromwiller TE. While exploration with cyst gastrostomy and biopsy of the cyst wall is appropriate for pancreatic pseudocysts. Initial size alone or location of the cyst is not predictive of malignancy. Daniel RA. REFERENCES 1. 2004 2. those that increase under observation may be malignant. Pickleman J: The increasing problem of unusual pancreatic tumors. Other surgeons believe that these complications are rare. or if benign may cause symptoms that warrant surgical excision. Aspiration as a treatment option will only lead to recurrence of the lesion. Unlike adenocarcinoma of the pancreas. Ann Surg 239:678-687. Latona C. Thayer SP. Because cystic neoplasms have been misdiagnosed as post-inflammatory pseudocysts in up to 37% of patients. et al: Cystic pancreatic neoplasms: observe or operate. However. and the results might influence the next steps in the treatment options. 2002 page 111 . Enucleation is possible for lesions remote from the pancreatic duct that are proven to be benign.Spinelli KS.Sheehan M. in this patient who has no history of pancreatitis and features of the computed tomography that suggest a more complex lesion.American College of Surgeons SESAP 13 Critique for Category 3 Item 5 Cystic lesions of the pancreas are increasingly diagnosed because of advancements in technology with ultrasonography. and magnetic resonance imaging. the presence of symptoms on initial presentation. Arch Surg 135:644-650.Salvia R. and a more aggressive procedure should be performed. computed tomography. Bassi C. Kiely JM. et al: Main-duct intraductal papillary mucinous neoplasms of the pancreas: clinical predictors of malignancy and long-term survival following resection. excellent long-term survival for both noninvasive and invasive papillary neoplasms and mucinous cystadenocarcinomas with an aggressive surgical approach has been documented. Surgical resection of these mucinous tumors can be performed with minimal morbidity and mortality in experienced centers. differentiating these entities is essential. Vogt DP. Predictors of malignancy include age over 70. but some of these lesions will have malignant potential. exploration is favored over observation because of the large size of the lesion and symptoms on initial presentation. Preoperative aspiration of the lesion is controversial. Ann Surg 239:651-657. 2004 4. Surgery 132:628-634. et al: Prospective preoperative determination of mucinous pancreatic cystic neoplasms. 2000 3. Aranha G. Some patients with mucinous cystic neoplasms and intraductal papillary mucinous neoplasms may present with pancreatitis based on viscous secretions occluding the pancreatic duct. Fernandez-del Castillo C. abscess formation. and enlargement of the lesion over time. the likelihood that this is a pseudocyst is very low.Walsh RM. Half of these patients will be asymptomatic. Potential complications include hemorrhage. Baker ME. and the possibility of malignant track seeding. Henderson JM. especially if the aspiration is performed with a fine needle under ultrasound guidance. An elevated CEA level on the fluid aspirate is consistent with a malignant process. For the patient described here. Biopsy of the lesion is positive for low-grade dysplasia.American College of Surgeons SESAP 13 Category 3 Item 6 A 55-year-old man with chronic gastrointestinal reflux undergoes upper endoscopy and is found to have a 4-cm segment of Barrett’s esophagus. Current surveillance recommendations include four-quadrant biopsies every (A) 3 months (B) 6 months (C) 9 months (D) 12 months (E) 24 months page 112 . surveillance. Wagh MS.Pacifico RJ. REFERENCES 1. J Clin Gastroenterol 39:S33-S41. The American College of Gastroenterology’s updated guidelines concerning surveillance regimens (see the second table) recommend repeat biopsy to confirm the diagnosis followed by yearly endoscopy with four-quadrant biopsies every 2 cm along the length of the Barrett’s segment for low-grade dysplasia. 2002 3. Barrett’s segments have a typical pink salmon color and velvety appearance.to 125-fold increased risk of developing invasive esophageal adenocarcinoma. Goyal RK.Sampliner RE. et al: Surveillance and screening for Barrett esophagus and adenocarcinoma.Mashimo H. Practice Parameters Committee of the American College of Gastroenterology: Updated guidelines for the diagnosis. It is considered to be the first change in a stepwise histologic progression to malignant transformation.American College of Surgeons SESAP 13 Critique for Category 3 Item 6 Barrett’s esophagus involves metaplastic transformation of any aspect of the squamous epithelial lining of the esophagus into intestinal columnar cells. Barrett’s occurs in up to 10% of patients with gastroesophageal reflux disease (GERD) and is believed to develop as a result of chronic injury to the distal mucosal lining of the esophagus from gastric acid. Surg Clin North Am 82:683-695. A biopsy revealing evidence of intestinal metaplasia after adequate treatment of any erosive esophagitis is required to verify the diagnosis. On endoscopic evaluation of the esophagus. 2002 page 113 . 2005 2. Deschamps C. as shown in the figure. and therapy of Barrett’s esophagus. Am J Gastroenterol 97:1888-1895. Barrett’s is associated with an approximately 30. The risk of cancer is increased when dysplasia is also present (see the first table). Wang KK: Management of Barrett’s esophagus with high-grade dysplasia. Barrett’s is often subdivided into long segment (> 3 cm) and short segment (< 3 cm) categories based on the extent of esophageal involvement from the gastroesophageal junction. Upper endoscopy reveals some retained food in the distal esophagus but is otherwise normal. The most appropriate management for this patient would be (A) botulinum toxin injection of the LES (B) calcium channel blocker therapy (C) esophageal myotomy (D) esophageal myotomy with fundoplication (E) pneumatic dilatation of the LES page 114 . The barium swallow and esophageal manometry shown are obtained. he has regurgitated food that is several days old. Over the last several weeks.American College of Surgeons SESAP 13 Category 3 Item 7 A 34-year-old man has a progressive history of dysphagia to solids and liquids. patients often complain of a sensation of food sticking in their chest. Achalasia is characterized by the progressive loss of myenteric ganglia cells within the esophagus. Botulinum injection of the LES improves symptoms in up to 85 % of patients. Rapid return of symptoms and poor response to therapy are negative predictors of successful outcome following subsequent dilatations. Early on. Botulinum injection also causes fibrosis at the LES and increases the risk of perforation at subsequent operation. 2004 page 115 . Phillips E: Current status of an antireflux procedure in laparoscopic Heller myotomy. Esophageal manometry confirms the diagnosis by revealing aperistalsis along the length of the esophagus. With each dilatation. Oelschlager BK: Achalasia. Thoman D.5 to 2 cm. Oelschlager BK. Dysphagia. it has an incidence of only 0. Steiner JP.American College of Surgeons SESAP 13 Critique of Category 3 Item 7 Although achalasia is the most common of the primary esophageal motility disorders. especially after eating meals or while lying flat. Pellegrini CA: Surgical management of esophageal motility disorders. Pneumatic dilatation of the LES relieves symptoms in up to 90% of patients and remains effective in 70% one year after intervention. is a cardinal symptom. A recent randomized controlled trial revealed that laparoscopic cardiomyotomy with combined anterior fundoplication decreased the rate of documented gastroesophageal reflux in patients by 9-fold compared with cardiomyotomy alone. Pellegrini CA. et al: Heller myotomy versus Heller myotomy with Dor fundoplication for achalasia: a prospective randomized double-blind clinical trial. J Surg Res 117:34-43. resulting in aperistalsis of the esophageal musculature and tonic contraction of the lower esophageal sphincter (LES) after swallowing. but recurrent symptoms develop within 6 months in up to half of treated patients. Khaitan L. Surg Endosc 17:554-558. Nonoperative treatment options include relaxation of the LES using nitrites and calcium channel blockers.0 in 100.5 to 1. REFERENCES 1.Richards WO. It is not as effective.Woltman TA. the risk of perforation is around 2%. The laparoscopic approach remains the preferred route. Surg Clin North Am 85:483-493. however.Lyass S. Neither class of drug is effective or consistent in relieving symptoms. in younger patients. Later symptoms include regurgitation. Holzman MD.000 in the United States. progressing from solids to liquids. 2004 3. Ann Surg 240:405-415. Upper endoscopy helps rule out an occult malignancy causing pseudo-achalasia. Surgical intervention via cardiomyotomy produces excellent results in up to 95 % of patients. and pulmonary complaints related to aspiration.Woltman TA. Barium swallow reveals the classic bird’s-beak appearance of smooth tapering of the esophagus at the level of the LES. 2003 2. Torquati A. weight loss. The myotomy should involve 6 to 7 cm of the thickened musculature of the distal esophagus and extend onto the proximal stomach for 1. 2005 4. American College of Surgeons SESAP 13 Category 3 Item 8 A 47-year-old woman has acid reflux that has not responded to therapy with proton pump inhibitors. Esophageal manometry shows good progression of peristalsis with normal lower esophageal sphincter (LES) tone and relaxation. Upper endoscopy reveals grade 1 esophagitis. The procedure MOST likely to help this patient overall would be (A) total fundoplication (B) partial fundoplication (C) gastric bypass (D) endoscopic antireflux procedure (E) vertical banded gastroplasty page 116 . Her body mass index (BMI) is 43 and she is hypertensive and diabetic. A bariatric procedure allows for weight loss and improvement or resolution of other associated conditions.Patterson EJ. Obes Surg 9:527-531. diabetes. Awais O. 2003 2. et al: Outcome of esophageal function and 24-hour esophageal pH monitoring after vertical banded gastroplasty and Roux-en-Y gastric bypass. Richards WO. and hyperlipidemia that a severely obese person will have. GERD is usually only one of many co-morbidities such as sleep apnea.American College of Surgeons SESAP 13 Critique for Category 3 Item 8 Gastroesophageal reflux disease (GERD) is seen in 37% to 72% of obese persons versus 20% of the general population. 2003 5. Sala C. Gonzalez QH. Mora F. but its effectiveness compared with total fundoplication is unknown. hypertension. degenerative joint disease. REFERENCES 1. 1999 3. Because of its other beneficial effects in this population. Laparoscopic adjustable band also seems to improve GERD symptoms. and associated symptoms appear to be more intense in the severely obese. it has a higher morbidity and is technically more difficult. Vertical banded gastroplasty (VBG) is much less effective than gastric bypass in treating GERD. Swanstrom LL: Comparison of objective outcomes following laparoscopic Nissen fundoplication versus laparoscopic gastric bypass in the morbidly obese with heartburn.Ortega J. et al: Gastrointestinal symptoms are more intense in morbidly obese patients and are improved with laparoscopic Roux-en-Y gastric bypass. Obes Surg 13:610-614. nor does it do anything to help with the other co-morbid conditions. Luketich JD: Laparoscopic gastric bypass after antireflux surgery for the treatment of gastroesophageal reflux in morbidly obese patients: initial experience. primary gastric bypass should be considered in obese patients qualifying for bariatric surgery who have GERD meriting operation. Khajanchee Y. Laparoscopic gastric bypass appears to improve objective (DeMeester score) and subjective (symptoms) measures of GERD as effectively as laparoscopic total fundoplication in the obese population. Escudero MD. 2004 page 117 . Gastric bypass is superior to adjustable band and VBG in promoting resolution of diabetes. O’Brien PE: Gastroesophageal reflux in obesity: the effect of lap-band placement. Although laparoscopic conversion of fundoplication to gastric bypass is possible.Clements RH. Davis DG. and surgical or endoscopic fundoplication directed solely at treating GERD does not adequately address the underlying cause.Raftopoulos I. Surg Endosc 17:1561-1565. Obes Surg 14:1373-1380.Dixon JB. Courcoulas AP. Obes Surg 14:1086-1094. Foster A. 2004 4. She has lost 50 lb since the procedure. a 32-year-old woman has severe periumbilical abdominal pain and vomiting. She has a large abdominal pannus and mild tenderness to palpation on examination.American College of Surgeons SESAP 13 Category 3 Item 9 Four months after laparoscopic gastric bypass. The next step in diagnostic work-up should be (A) computed tomographic (CT) scan of the abdomen (B) upper gastrointestinal swallow study (C) water-soluble enema (D) abdominal exploration (E) upper endoscopy page 118 . She is afebrile and mildly tachycardic. Pinto D.Westling A. Symptoms typically include acute onset of abdominal pain with nausea and vomiting. In one large series. Small bowel obstruction can develop at any time after gastric bypass. REFERENCES 1.Higa KD. Obes Surg 11:284-292. although some patients may have chronic intermittent pain or only vomiting.3%. 2001 page 119 . Am J Surg 189:532-535. Blood chemistries and radiographic imaging do not reliably identify small bowel obstruction in these patients. antegastric approach had a rate of 1. Gustavsson S: Laparoscopic vs open Roux-en-Y gastric bypass: a prospective. 2004 5. treatment and prevention. Ho T.6%.Lauter DM: Treatment of nonadhesive bowel obstruction following gastric bypass. In the open series.1% rate from a large open gastric bypass series. Because of the potentially devastating complications that can result from late diagnosis and intervention. Another large laparoscopic series performed via the antecolic. et al: Computed tomography patterns in small bowel obstruction after open distal gastric bypass. Carrodeguas L. small bowel obstruction tended to occur much later postoperatively. These percentages compare favorably with a 2. randomized trial. Obes Surg 14:811-822. early abdominal exploration is indicated after gastric bypass in patients with symptoms suggesting small bowel obstruction.American College of Surgeons SESAP 13 Critique for Category 3 Item 9 Small bowel obstruction after gastric bypass is a potentially life-threatening complication requiring prompt intervention. the rate of negative abdominal exploration was 2% among symptomatic patients after gastric bypass. Boone KB: Internal hernias after laparoscopic Roux-en-Y gastric bypass: incidence. Obes Surg 13:350-354 2003 3.Cho M. the incidence of small bowel obstruction was 3. Keskey T. 2005 4. J Am Coll Surg 202:262-268. 2006 2.Srikanth MS. The WBC count can be within normal limits. et al: Diagnosis and management of partial small bowel obstruction after laparoscopic antecolic antegastric Roux-en-Y gastric bypass for morbid obesity. Oh KH. antegastric approach. Lascano C. Fox SR. In one large series of laparoscopic gastric bypass performed via a retrocolic. It is often secondary to internal herniation of small bowel in the upper abdomen or cicatrical scarring at the level of the mesocolon rather than adhesive band formation. Plain abdominal films and abdominal computed tomography (CT) may not reveal abnormalities. After four days in the hospital.800 cells/mm3. fever. He is NPO and is receiving intravenous antibiotics.American College of Surgeons SESAP 13 Category 3 Item 10 A 37-year-old obese man has a 1-day history of change in bowel movements. and left lower quadrant pain. His WBC count is 11. Abdominal computed tomographic (CT) scan shows thickened sigmoid with stranding of the surrounding fat. Which of the following should be recommended to this patient? (A) Elective sigmoid colectomy before discharge (B) Elective sigmoid colectomy in 6 weeks (C) Colonoscopy in 6 weeks (D) Repeat CT scan in 6 weeks (E) Dietary counseling page 120 . he is clinically improved without leukocytosis or fever. but no intraperitoneal air or extracolonic fluid collections. recurrent/chronic diverticulitis was more common in younger patients. Veenstra DL. is growing. Hyman N: Diverticulitis in young patients: is resection after a single attack always warranted? Dis Colon Rectum 47:1187-1191. 2004 4. in Souba WW.5%. admission to the hospital for intravenous antibiotics and no oral intake is required. et al (eds): ACS Surgery: Principles and Practice 2006. and cost savings of $5249 per patient. or fistula. 2% fewer colostomies. Dis Colon Rectum 46:1110-1114. The most common problem is to miss a colon cancer that presents with signs and symptoms consistent with diverticulitis (see picture and algorithm). sec 12. is present.Welch JP. Kaiser LR.Guzzo J. Surg Clin North Am 85:1924. New York.1% fewer deaths. operation should be deferred. and altered defecation. 2003 3. Physical examination will reveal left lower quandrant tenderness to palpation with or without rebound or guarding. In one large retrospective study. rates of urgent surgery were equal in both age groups. These authors suggest waiting until after the second attack before surgical intervention. Jurkovich GJ. Flum DR: The timing of elective colectomy in diverticulitis: a decision analysis. Fink MP. Colonoscopy several weeks later to rule out an alternative diagnosis is necessary. In more severe cases. Classic symptoms of an acute episode of uncomplicated diverticulitis include left lower quadrant pain. Gastrointestinal Tract and Abdomen. Mild cases can be treated with low-residue liquid diets and oral antibiotics.Salem L. pp 540-553 page 121 . WebMD. abscess.American College of Surgeons SESAP 13 Critique for Category 3 Item 10 Although diverticular disease increases in frequency with increasing age.Senagore AJ: Laparoscopic sigmoid colectomy for diverticular disease. and the subsequent rate of diverticular free perforation in young patients who did not undergo elective operation after their initial episode was only 0. especially obese males. Abdominal and pelvic computed tomography (CT) should be obtained to determine whether complicated disease.McConnell EJ. fever. Sullivan SD. In this young patient with uncomplicated diverticulitis responsive to medical management. recent studies have challenged this notion. mounting evidence suggests that the incidence of diverticulitis among patients under 40 or 50 years old. 2005 5. perforation. ie. Cohen JL: Diverticulitis. REFERENCES 1. J Am Coll Surg 199:904-912. among patients requiring surgical resection. Tessier DJ. In another study looking at patients admitted for sigmoid diverticulitis. chap 5. A recent decision and cost analysis using a Markov model of young and old patients concluded that waiting until the fourth episode of diverticulitis for elective resection would result in 0. Wolff BG: Population-based incidence of complicated diverticular disease of the sigmoid colon based on gender and age. Although elective interval resection following an initial attack has traditionally been advocated in young patients because of the perceived increased severity of disease. 2004 2. He underwent aggressive resuscitation with stabilization of his blood pressure. his blood pressure was 83/60 and his hematocrit was 20%. Nuclear scintigraphy with tagged RBCs was positive for pooling of material in the left colon with reflux into the right colon. Proper therapy at this time would include (A) emergent right colectomy (B) emergent subtotal colectomy (C) emergent left colectomy (D) continued ICU care with transfusion support (E) intravenous vasopressin page 122 . Colonoscopy revealed blood throughout the large bowel without visualization of active bleeding. Arteriography of the mesenteric vascular did not reveal an active source. On initial presentation. He is now on his 6th unit of packed red blood cells. Work-up for his bleeding has included a negative upper endoscopy.American College of Surgeons SESAP 13 Category 3 Item 11 A 73-year-old man with a history of angina was admitted to the ICU 3 days ago for massive bright red blood per rectum. upper endoscopy should be considered to verify that the source is not above the ligament of Treitz. and it has high diagnositic yields. REFERENCES 1.American College of Surgeons SESAP 13 Critique for Category 3 Item 11 Lower gastrointestinal bleeding is defined as hemorrhage within the bowel distal to the ligament of Treitz. but is not very good at localizing the exact anatomic site of bleeding. Two large-bore intravenous lines and a urinary catheter should be placed for fluid resuscitation and monitoring. Philadelphia. but can localize bleeding sources accurately. It is not as sensitive as radionuclide scanning. but severe bleeding may make localization difficult. in Souba WW. including myocardial infarction. Radionuclide scanning using technetium-99m labeled RBCs is another accurate means of identifying bleeding at rates as low as 0. Several modalities may be used to localize the source of bleeding. anorectal disease. Surgical intervention is used as a last resort. Bleeding tends to occur more frequently in men and increases with age. Jurkovich GJ. Arteriovenous malformations.Rosen M. Patients should be cross-matched and undergo transfusion as needed. Hemodynamically unstable patients should be monitored in the ICU. Coagulation parameters and serial hematocrits should be obtained. the bleeding stops spontaneously. Segmental resection based solely on radionuclide scanning results should be avoided because errors in removal can occur in up to 40% of cases. Diverticulosis is the most common cause of colon hemorrhage. and colectomy. but it approaches 50% for a third episode.0 mL/min. then emergency subtotal colectomy should be performed because segmental resections have rebleeding rates up to 75% with mortality reaching 50%. 2004. Selective mesenteric angiography requires bleeding rates of 1. Up to 11 % of episodes of bright red blood per rectum originate above the ligament of Treitz. New York. et al (eds): ACS Surgery: Principles and Practice 2006. and interventional hemostasis. It is generally considered when more than 4 units are transfused in 24 hours. or bleeding recurs within a week. re-bleeding rates approach 50% and its vasoconstrictive effects can lead to cardiac complications. Kaiser LR. The risk of a second diverticular hemorrhage is around 25%. a lower source of bleeding is likely. in Cameron JL (ed): Current Surgical Therapy. Management of patients presenting with lower gastrointestinal bleeding focuses on patient stabilization with resuscitation. ed 8. Mosby. pp 285-291 2. vasopressin administration or embolization via angiography. Ponsky JL: Lower gastrointestinal bleeding. If the exact anatomic location of bleeding cannot be determined preoperatively or intraoperatively (using endoscopy). Although the colon is often the source.1 mL/min. neoplasms. WebMD.Chan EL. and inflammatory bowel disease can also lead to colon bleeding. bleeding persists for more than 72 hours. In up to 90% of cases. Gastrointestinal page 123 . Therapy for lower gastrointestinal bleeding includes endoscopic treatment using electrocautery or epinephrine injection via colonoscopy. Galandiuk S: Lower gastrointestinal bleeding. Although vasopressin is effective at stopping bleeding. Otherwise. It can be used to detect bleeding up to 48 hours after injection. Gastric lavage should be performed to rule out an upper gastrointestinal source. Fink MP. both the small bowel and the upper gastrointestinal tract can be involved. If the nasogastric aspirate is bilious. Vasopressin is usually avoided in patients at risk for cardiac events. Colonoscopy is useful for identifying a colon or distal ileal lesion. localization of bleeding. pp 466-476 page 124 .American College of Surgeons SESAP 13 Tract and Abdomen. sec 6. chap 5. Endoscopic biopsy of these lesions demonstrates dysplasia without evidence of malignancy. The most appropriate management would be (A) observation with surveillance endoscopic biopsies at 6-month intervals (B) endoscopic resection of all lesions (C) photodynamic therapy (D) transduodenal excision (E) pancreaticoduodenectomy page 125 .American College of Surgeons SESAP 13 Category 3 Item 12 A 54-year-old woman undergoes colectomy for familial adenomatous polyposis. The findings at the time of duodenoscopy are shown. and according to the experience with isolated villous tumors of the duodenum. In patients with FAP.Gallagher MC. the polyps are often multiple and numerous. large. Arch Surg 137:557-563. Russell RC. 2004 2. complete clearance of polyps within the duodenum is required. The incidence of duodenal adenocarcinoma in FAP patients is 200 to 300 times greater than in the general population. Donohue JH. In patients with familial adenomatous polyposis (FAP). Photodynamic therapy is not efficacious in this setting. Observation with surveillance endoscopic biopsies every 6 months would be likely to miss development of invasive adenocarcinoma until it is far too advanced. Br J Surg 91:1157-1164. et al: Pylorus-preserving pancreaticoduodenectomy for advanced duodenal disease in familial adenomatous polyposis. Duodenal adenocarcinoma is the most common cause of malignancy-associated death in patients with FAP despite proctocolectomy for polyposis.American College of Surgeons SESAP 13 Critique for Category 3 Item 12 Duodenal polyposis is an uncommon diagnosis that occurs in two different clinical settings. REFERENCES 1. Thompson GB. In patients with isolated villous tumors. the polyps are usually single. 2002 page 126 . Nagorney DM. Transduodenal excision or endoscopic resection would not be appropriate because recurrence rates are high and recurrences frequently present as invasive carcinomas. Surgical resection is the only effective means to achieve complete clearance. Shankar A. these growths clearly have increased malignant potential. Accepting the adenoma-carcinoma sequence.Sarmiento JM. et al: Pancreas-sparing duodenectomy for duodenal polyposis. and centered around the papilla of Vater. Groves CJ. American College of Surgeons SESAP 13 Category 3 Item 13 Which of the following statements about inflammatory conditions of the colon is TRUE? (A) The risk of malignancy with pancolonic ulcerative colitis (UC) is 1% to 2% per year (B) UC is a mucosal disease that is associated with the development of strictures despite medical therapy (C) Perianal lesions are relatively common in severe UC. but the rectum is usually spared (D) Nonsteroidal anti-inflammatory agents are effective in the treatment of UC (E) Oral contraceptives are associated with the development of inflammation of the colon that mimics Crohn’s disease histologically page 127 . 2003.Garcia Rodriguez LA. ed 2. Rothenberger DA: Colon.Bullard KM. Aliment Pharmacol Ther 22:309-315. 2005 3. chronic infections. The most common causes are inflammatory bowel disease. rectum. The risk of malignancy in pancolonic ulcerative colitis is approximately 0.American College of Surgeons SESAP 13 Critique for Category 3 Item 13 Colitis is a nonspecific description of inflammation of the colon. and anus. Perianal lesions are uncommon in ulcerative colitis despite the fact that the rectum is virtually always involved. Johansson S. McGraw-Hill. and 18% after 30 years. pp 480-488 page 128 . REFERENCES 1. New York. in Friedman SL. Gonzalez-Perez A. Grendell JH (eds): Current Diagnosis & Treatment in Gastroenterology. Dunn DL. Chronic ulcerative colitis is the most common form of inflammatory colitis and presents clinically as abdominal pain and bloody diarrhea. 8% after 20 years. There have been interesting reports that oral contraceptives may be associated with inflammatory lesions in the colon that mimic the findings in ulcerative colitis. New York. A similar association between the development of Crohn’s disease and the use of hormone replacement therapy has also been suggested. Lashner BA: Miscellaneous diseases of the colon. Because the inflammation in ulcerative colitis is confined to the mucosa. and strictures are uncommon findings in ulcerative colitis. pp 1055-1118 2. McQuaid KR. McGraw-Hill. Andersen DK. fistulas. in Brunicardi FC. ed 8.2% per year or about 2% at 10 years. and radiation enteritis. there is a significant risk of malignant degeneration in areas of chronic inflammation.1% to 0. Billiar TR. 2005. et al (eds): Schwartz’s Principles of Surgery. Wallander MA: Risk factors for inflammatory bowel disease in the general population.Thota PN. ischemia. In addition. Nonsteroidal anti-inflammatory agents have been implicated in the induction of a form of ulcerative colitis and are not effective in the treatment. perforation. American College of Surgeons SESAP 13 Category 3 Item 14 Which of the following statements about neuroendocrine tumors (NETs) of the rectum is TRUE? (A) More than 70% of those > 2 cm are associated with distant metastases (B) They appear to be more likely to secrete vasoactive substances than are NETs in other locations (C) Local excision should not be used because of the aggressive nature of even small rectal NETs (D) Patients with distant metastases rarely survive more than 12 months (E) Rectal NETs are much less frequent and their prognosis is worse than colon NETs page 129 . Rothenberger DA: Colon. ed 8. arise from a variety of neuroendocrine cells located in the mucosa throughout the gastrointestinal tract. and about 5% metastasize. 2005. Rectal carcinoids are less likely than carcinoids in other locations to secrete vasoactive substances. 2005. Most appear during endoscopy as small (< 1 cm). Doherty GM. solitary submucosal tumors than can often be safely removed endoscopically. More than 14 different cell types produce different hormones and show nonrandom distribution in the gastrointestinal tract. and tumors > 2 cm have metastases in > 70%. New York. McGraw-Hill.Jensen RT: Endocrine tumors of the gastrointestinal tract and pancreas. Best Pract Res Clin Gastroenterol 19:507-517. Lippincott Williams & Wilkins. with 66% to 80% being <1 cm in diameter. Maier RV. Bresalier R: Colonic polyps and polyposis syndromes. Rectal carcinoids are found in only 1 of every 2500 proctoscopies. pp 1055-1118 3. Anlauf M: Epidemiology. pp 2220-2230 4. Fauci AS. in Kasper DL. REFERENCES 1. McGraw-Hill. Andersen DK. 2005 page 130 . ed 4. Carcinoid tumors of the rectum are not typically poorly differentiated tumors.American College of Surgeons SESAP 13 Critique for Category 3 Item 14 Neuroendocrine tumors (NETs).Boland CR. Philadelphia. in Brunicardi FC. and anus. tumour biology and histopathological classification of neuroendocrine tumours of the gastrointestinal tract.Kloppel G. et al (eds): Schwartz’s Principles of Surgery. Hauser SL. also frequently called carcinoid tumors. et al (eds): Greenfield's Surgery: Scientific Principles and Practice. Dunn DL.Bullard KM. Lillemoe KD. Nearly all occur 4 to 13 cm above the dentate line. 2006. Rectal NETs are more frequent and have a better prognosis than NETs found in the colon. The etiology is unknown. Rectal NETs comprise about 10% of all gastrointestinal NETs. ed 16. in Mulholland MW. pp1080-1102 2. rectum. Most are small. New York. Tumors between 1 and 2 cm have metastases in 5% to 30% of patients. et al (eds): Harrison's Principles of Internal Medicine. Braunwald E. Billiar TR. Even patients with metastatic deposits of NETs have a higher survival rate than those with adenocarcinoma primaries. He does not have hemorrhoids. His abdomen is soft. nontender. Which of the following is the most like source of his bleeding? (A) Angiodysplasia (B) Ulcerative colitis (C) Juvenile polyps (D) Meckel’s diverticulum (E) Crohn’s disease page 131 . Physical examination is normal. He was told he had hemorrhoids. He had a similar episode a year ago and underwent nondiagnostic colonoscopy and upper gastrointestinal endoscopy. His hematocrit is 32 mg/dL. He is otherwise in good health and takes no medications.American College of Surgeons SESAP 13 Category 3 Item 15 A 19-year-old student presents to the emergency department with bright red blood per rectum. and without masses. Jacobson BC. 2005 2. Angiodysplasia of the small bowel and right colon is a common source of obscure gastrointestinal bleeding in adults but is rare in children and teens. 2004-2005. Hirota W. Juvenile polyps may cause lower gastrointestinal bleeding in children but occur with less frequency than Meckel’s. Goldstein J. Wireless video capsule endoscopy has been used with some success in the evaluation of the small bowel. 2004. The production of acid by functional gastric mucosa leads to peptic ulceration of the adjacent small bowel mucosa with resultant bleeding. in Habermann TM (ed): Mayo Clinic Internal Medicine Board Review. Mutignani M. 2001 page 132 . Meckel’s diverticulum is the most common source of lower gastrointestinal bleeding in children and young adults. Marano AF. pp 253-330 4. J Am Coll Surg 192:658-662. Gastrointest Endosc 61:779-781. 2003 3. It is the most prevalent congenital anomaly of the gastrointestinal tract. Both capsule endoscopy and double-balloon enteroscopy have been used for successful diagnosis of Meckel’s diverticulum. Philadelphia. There is a strong male preponderance.Yahchouchy EK. REFERENCES 1. or bleeding of unknown origin that persists or recurs after an initial negative endoscopic evaluation.American College of Surgeons SESAP 13 Critique for Category 3 Item 15 Obscure gastrointestinal bleeding. Ulcerative colitis may present with massive lower gastrointestinal bleeding but is uncommon in children and teenagers and would be detected by colonoscopy. is usually caused by lesions between the ligament of Treitz and the ileocecal valve. Crohn’s disease may cause gastrointestinal bleeding but is usually associated with other abdominal symptoms. heterotopic gastric and pancreatic mucosa may be found in the diverticulum. Lippincott Williams & Wilkins. Cammarota G. Poterucha JJ: Gastroenterology and hepatology. In symptomatic patients. et al: Obscure gastrointestinal bleeding.Viggiano TR. affecting 2% of the general population. Double-balloon enteroscopy is a new technique that combines radiologic imaging with enteroscopy to provide access to the entire small bowel. Fingerhut AL: Meckel’s diverticulum.Leighton JA. Gastrointest Endosc 58:650-655. Sedlack RE. et al: Double-balloon enteroscopy for diagnosis of a Meckel’s diverticulum in a patient with GI bleeding of obscure origin. Like ulcerative colitis.Gasbarrini A. which is inaccessible to routine endoscopy. Di Caro S. Etienne JC. American College of Surgeons SESAP 13 Category 3 Item 16 Which of the following statements about malignant tumors of the appendix is TRUE? (A) Right hemicolectomy is the recommended treatment for all appendiceal carcinoid tumors (B) Carcinoid is the second most frequent appendiceal malignancy (C) Hepatic metastases can be demonstrated in most patients with even small appendiceal carcinoids (D) Appendiceal carcinoids have a lower 5-year survival than noncarcinoid malignancies of the appendix (E) Appendiceal goblet cell carcinoids < 1 cm can be safely treated with simple appendectomy page 133 . the histologic grade of the tumor. location of the tumor at the base of the appendix rather than the tip. the presence of metastases. 2005. pp 1119-1138 2. Rothmund M: Neuroendocrine tumours (carcinoids) of the appendix. O’Connell JB. Berger DH: The appendix.to 2-cm NETs is controversial. Mucinous adenocarcinoma is the number 1 malignancy. 2005 3. Maggard MA. and mucinous and signet ring types. Neuroendocrine tumor (NET) is currently the recommended term for carcinoids. Andersen DK. leading to the hypothesis that appendiceal NETs arise from a different cell type.Stinner B. The treatment of appendiceal malignancy should be guided by cell type. in Doherty GM (ed): Current Surgical Diagnosis and Treatment. Billiar TR.Jaffe BM. Non. Gastrointestinal NETs arise from a diffuse population of neuroendocrine cells. Dis Colon Rectum 48:2264-2271. New York.neuroendocrine tumors include adenocarcinoma. 2006. Considerations for more aggressive surgical treatment include involvement of the mesoappendix. Goblet cell carcinoids. 2005 4. have an intermediate malignant potential. ed 12. et al (eds): Schwartz’s Principles of Surgery. et al: Malignancies of the appendix: beyond case series reports. Dunn DL. New York. Some evidence suggests that appendiceal NETs have a better prognosis than do NETs in other parts of the gastrointestinal tract.Way LW: Appendix. and NET is the second most frequent. Kang H. but many surgeons recommend right hemicolectomy.McGory ML. pp 648-653 page 134 . The treatment of 1. McGraw-Hill. Small appendiceal carcinoids tumors (< 1 cm) have a virtually nonexistent risk of metastases and may be safely treated by simple appendectomy. and the patient’s overall health. Best Pract Res Clin Gastroenterol 19:729-38.American College of Surgeons SESAP 13 Critique for Category 3 Item 16 Malignant tumors of the appendix can be classified into carcinoid tumors and non-carcinoid tumors. Treatment is controversial. McGraw-Hill. ed 8. REFERENCES 1. or so-called adenocarcinoids. in Brunicardi FC. Right hemicolectomy is recommended for all non-NET malignancies and for all NETs > 2 cm. She has no abdominal pain or constipation. Vital signs include: temperature.5°C. blood pressure. 20/min. 80/min. 120/70. In addition to resuscitation.American College of Surgeons SESAP 13 Category 3 Item 17 A 23-year-old otherwise healthy woman presents with hematochezia. the next step should be (A) exploratory laparotomy (B) highly selective vagotomy (C) colonoscopy (D) bleeding scan (E) repeat esophagogastroduodenoscopy page 135 . and has taken no recent trips. heart rate. Flexible esophagogastroduodenoscopy shows evidence of recent hemorrhage She remains hemodynamically stable and is scheduled for gastrointestinal work-up when she develops hematemesis. and respiratory rate. 37. Overall permanent hemostasis was achieved in 18 patients (78%). or ethanolamine injection. heater probe. Paroutoglou G. In the patients treated with dextrose 50% plus epinephrine injection. There were no side effects related to endoscopic therapy. 4 of 10 had recurrent bleeding and 1 had unsuccessful initial hemostasis. Sung JJ: Dieulafoy's lesion.American College of Surgeons SESAP 13 Critique for Category 3 Item 17 Esophagogastroduodenosopy (EGD) is the initial diagnostic test for any patient presenting with hematemesis. These authors concluded that endoscopic methods should be regarded as the first choice of therapy for Dieulafoy's lesions. hemoclipping in 8 patients. 2005 2. Bleeding scans are time consuming and nonspecific and would not be indicated in this patient with active hematemesis. and inability to approach the lesion. Highly selective vagotomy is indicated for peptic ulcer disease. In a recent series of 23 patients who had endoscopic therapy for gastrointestinal bleeding from Dieulafoy's lesions. Initial hemostasis was successful with no recurrent bleeding in patients treated with hemoclipping. Rom J Gastroenterol 14:79-82. 2003 page 136 . a Dieulafoy’s lesion should be suspected. Endoscopic hemoclipping has also been used successfully to control bleeding due to colonic Dieulafoy’s from the cecum to the rectum. Leong RW. Walmsley RS. Mimidis K. Of the 4 patients who had re-bleeding. Gastrointest Endosc 58:236-243. 3 had unsuccessful hemostasis with similar treatment. REFERENCES 1. It has a high sensitivity and specificity for diagnosing the source of hemorrhage and frequently allows for therapeutic interventions to control the bleeding.Gheorghe C: Endoscopic clipping focused on "triclip" for bleeding Dieulafoy's lesion in the colon. None of the patients in whom permanent hemostasis was achieved presented with re-bleeding from Dieulafoy's lesion over a mean follow-up of 30 months. Five patients required surgical treatment for uncontrolled bleeding. Hematemesis is due to a bleeding source in the proximal gastrointestinal tract (above the ligament of Treitz) and thus colonoscopy is not indicated. In this patient with a normal EGD. Exploratory laparotomy is associated with significant morbidity and would only be indicated if the patient were exsanguinating.Lee YT. and ethanolamine oleate in 2 patients. heater probe in 2 patients. 2005 3. recurrent bleeding with unsuccessful re-treatment. Co-morbid conditions were present in 17 patients (74%). Beltsis A.Katsinelos P. and recommended endoscopic hemoclipping for bleeding. hemostasis was attempted by intramural injection of dextrose 50% plus epinephrine in 10 patients. et al: Endoscopic treatment and follow-up of gastrointestinal Dieulafoy's lesions. World J Gastroenterol 11:6022-6026. American College of Surgeons SESAP 13 Category 3 Item 18 A 22-year-old unrestrained driver has multiple injuries after a motor vehicle crash in which he was ejected from the car. truncal vagotomy and antrectomy (D) closure of the ulcer and highly selective vagotomy (E) subtotal gastrectomy page 137 . a 2cm gastric perforation along the greater curvature of the body is discovered. and a right humerus fracture. Ten days after injury he develops an acute abdomen with pneumoperitoneum. At laparotomy. a left femur fracture. The best management would be (A) closure with a Graham patch (B) excision of the perforation and primary closure (C) closure of the ulcer. Injuries include a mangled right leg requiring below-knee amputation. and erosions. burns covering more than 35% of body surface area. many intensivists have questioned whether this normalization contributes to nosocomial pneumonia by allowing bacterial colonization of the upper aerodigestive tract. cardiopulmonary bypass. Perforated gastric ulcer has a mortality rate of 40% versus 10% for perforated duodenal ulcer. Johnson type I gastric ulcer. because patients with perforated gastric ulcer tend to be older and sicker.American College of Surgeons SESAP 13 Critique for Category 3 Item 18 Acid secretion is variable in patients with gastric ulcer. Both type II and type III gastric ulcers are associated with normal or increased gastric acid secretion. The lesions can be distributed throughout the gastric mucosa. Operation is almost always indicated. Type II gastric ulcer is associated with active or quiescent duodenal ulcer disease. Bleeding is the clinical presenting sign in critically ill patients. Four types of gastric ulcer are described. The bleeding may be massive and life threatening. Although early trials appeared to confirm this fear. and surgical intervention for control of bleeding is now a rare procedure. Type IV gastric ulcers occur near the gastroesophageal junction. Also known as stress gastritis. a more recent large comparative trial showed no increased incidence in pneumonia in patients receiving H2 receptor therapy rather than sucralfate (while clinically important bleeding episodes were significantly decreased). The mortality rate for perforated gastric ulcer is higher than that for duodenal ulcer. acute mucosal ischemia. petechiae. although nonsurgical treatment can occasionally be used in the stable patient without peritonitis when radiologic studies document a sealed perforation. These patients usually have normal or decreased acid secretion. Patients with type I or IV gastric ulcers may have weak mucosal defenses that permit an abnormal amount of injurious acid back-diffusion into the mucosa. but are predominantly identified in the body of the stomach. coma following head injury or intracranial operation. and type III gastric ulcer is prepyloric. and coagulopathy. Particular risk factors include sepsis. erosive gastritis. and the larger size of gastric ulcers. The most common. Activated pepsin is a proteolytic enzyme that directly digests the mucosal lining and is inactivated at a pH of approximately 4. increased co-morbidities.5. close to the border between the antrum and the body of the stomach. This understanding has led to broad application of prophylaxis against stress ulceration in critical care practice. This increased mortality is generally attributed to more advanced patient age. and acid secretion is normal or below normal. Acute gastric mucosal lesions (AGMLs) include a broad category of acute erosive mucosal conditions that develop in critically ill patients. Despite the proven efficacy of gastric pH neutralization for reducing clinically significant bleeding. is typically located near the angularis incisura on the lesser curvature. They are characterized by stigmata of mucosal injury evidenced by mucosal pallor. hemodynamic instability. The pathogenesis of AGML is related to a combination of both gastric acid and activated pepsin injuring the gastric mucosa and is exacerbated by mucosal ischemia secondary to hypoperfusion. delay in seeking medical attention. or stress ulceration. The condition is particularly lethal in postoperative cardiovascular patients and in patients with sepsis and multiorgan failure. The current recommendation calls for gastric neutralization with either H2 receptor antagonists or proton-pump inhibitors. these conditions share a common epidemiology and clinical presentation. page 138 . Patients at risk for AGML not associated with NSAIDs include virtually all critically ill patients in medical and surgical ICUs. respiratory failure. multiple trauma. Distal gastrectomy often eradicates Helicobacter pylori. Billiar TR. closure. Dunn DL. which could partially explain this finding. Saunders. inasmuch as lower ulcer recurrence rates have been reported after simple patch closure of perforated gastric ulcer with effective eradication of H pylori. 2004 . 2005. truncal vagotomy. Andersen DK. depending on ulcer type. local excision and closure. pp 933-996 2. McGraw-Hill. or biopsy. Excision and primary closure with antisecretory medication postoperatively should be adequate treatment. Simple patch closure has a higher short-term complication rate (20% versus 5%) and a higher ulcer recurrence rate (25% versus 10%) than distal gastrectomy. Vagotomy is usually performed for type II and III gastric ulcers.Dempsey DT: Stomach. Philadelphia. in Brunicardi FC. many patients with perforated gastric ulcer are H pylori negative. All perforated gastric ulcers. pp 1265-1322 page 139 . ed 17. in Townsend CM Jr (ed): Sabiston Textbook of Surgery: The Biological Basis of Modern Surgical Practice. and drainage. should be biopsied if they are not removed at operation. Robinson EK: Stomach.American College of Surgeons SESAP 13 All perforated gastric ulcers are best treated by distal gastric resection with or without truncal vagotomy. even those in the prepyloric position. Highly selective vagotomy should generally not be used for perforated gastric ulcer. Alternative operations include patch closure with biopsy. However. ed 8. et al (eds): Schwartz’s Principles of Surgery. REFERENCES 1. New York.Mercer DW. This patient has a perforated stress ulcer. Heart rate is 100/min and blood pressure is 120/80. The most appropriate study now would be (A) magnetic resonance cholangiopancreatography (MRCP) (B) colonoscopy (C) visceral angiography (D) nuclear bleeding scan (E) dynamic helical computed tomography (CT) page 140 . Esophagogastroduodenoscopy shows blood in the duodenum. a 68-year-old woman presents with hematemesis. Digital rectal examination yields melenic stool.American College of Surgeons SESAP 13 Category 3 Item 19 Forty-eight hours after laparoscopic-guided biopsy of a liver lesion. and inadequate drainage. In a recent review. Hepatic resection may be necessary for failed arterial ligation or for cases of severe trauma or tumor. Once hemobilia is suspected. but percutaneous transhepatic biliary drainage procedures yield an incidence of 2% to 10%. the use of packs. surgical exploration of the biliary tree can result in hemobilia from direct injury or arterial pseudoaneurysm. no further studies are necessary. A number of cases of hemobilia after cholecystectomy have been reported in recent years. biliary colic (70%). There are isolated reports of successful management of hemobilia with endoscopic coagulation. Percutaneous liver biopsy results in hemobilia in fewer than 1% of cases. MRCP is indicated for hepatobiliary tumors and to evaluate for a pancreatic ductal injury. upper gastrointestinal hemorrhage. hematemesis (60%). The classic triad of symptoms and signs of hemobilia are upper abdominal pain. Clinical sequelae of hemobilia are related to blood loss and the formation of potentially occlusive blood clots in the biliary tree. Colonoscopy is not indicated for initial investigation of hematemesis. page 141 . A nuclear bleeding scan is indicated to localize intestinal bleeding as originating in the upper intestine or lower intestine (defined as proximal or distal to the ligament of Treitz). Upper gastrointestinal bleeding seen in conjunction with biliary symptoms must always raise the suspicion of hemobilia. Minor hemobilia generally runs an uneventful asymptomatic clinical course. or nonselective ligation of a main hepatic artery.American College of Surgeons SESAP 13 Critique for Category 3 Item 19 In recent years. Recent reviews put iatrogenic etiologies related to liver biopsy procedures as the cause of hemobilia in 40% to 60% of cases.Likewise. Even in patients for whom laparotomy may be mandated for other reasons. excision of aneurysm. TAE is still the therapy of choice for hemobilia because of its lower morbidity. Reported success rates of TAE in this setting are 80% to 100%. and so is not specific enough to be of value in this setting. but is less sensitive than visceral angiography. Upper endoscopy is only diagnostic of hemobilia in about 10% of cases. If upper endoscopy is diagnostic and conservative management is planned. Surgical approaches generally involve ligation of bleeding vessels. the ratio of hemobilia attributed to intentional or unintentional trauma compared with iatrogenic trauma was 2:1. all three were present in 22% of patients. It has not been used to diagnose intrahepatic pseudoaneurysms. Hemorrhage from the gallbladder or hemorrhagic cholecystitis mandates cholecystectomy. Hemobilia secondary to accidental trauma is more common with blunt than penetrating abdominal trauma. which rules out other sources of hemorrhage and may visualize bleeding from the ampulla of Vater. Operation is indicated when conservative therapy and TAE have failed. and jaundice (60%). Dynamic helical computed tomography (CT) might demonstrate a pseudoaneurysm of the intrahepatic artery. The incidence of documented hemobilia after major hepatic trauma ranges from 0. The symptoms and signs of major hemobilia are melena (90%). somatostatin. Prior to the 1980s.2% to 3%. Visceral angiography is the diagnostic study of choice and also allows control of the hemorrhage by transarterial embolization (TAE) of the involved vessel. iatrogenic trauma to the liver and biliary tree has become the most common cause of hemobilia. Risk factors for the development of hemobilia after trauma include central hepatic rupture with a cavity. and vasopressin. The management of hemobilia after percutaneous transhepatic biliary drainage usually consists of removal of the catheter or replacement with larger catheters. the first evaluation should be upper gastrointestinal endoscopy. however. and jaundice. Philadelphia. Jamieson NV: Haemobilia. pp 1241-1264 2. 2004.American College of Surgeons SESAP 13 REFERENCES 1. in Townsend CM Jr (ed): Sabiston Textbook of Surgery: The Biological Basis of Modern Surgical Practice. Saunders. Duell RM. ed 17. Br J Surg 88:773. Philadelphia. pp 1513-1574 3.Bass BL. Turner DJ: Acute gastrointestinal hemorrhage. in Townsend CM Jr (ed): Sabiston Textbook of Surgery: The Biological Basis of Modern Surgical Practice.D’Angelica M. ed 17. Saunders. Fong Y: The liver. 2004. 2001 page 142 .Green MH. Johnson CD. 000/mm3. The most appropriate management would be (A) laparotomy (B) colonoscopy (C) laparoscopy (D) upper gastrointestinal contrast study (E) nasogastric decompression and serial examinations page 143 . static peak compliance. positive end-expiratory pressure. 15 cm H2O. She is receiving full nutritional support via a small-bore nasointestinal feeding tube when she develops abdominal tenderness. WBC count is 18. She is currently intubated for adult respiratory distress syndrome (ARDS) (FiO2. The abdominal computed tomographic (CT) scan shown is obtained. 40 cm H2O) and receiving steroids for the asthma.American College of Surgeons SESAP 13 Category 3 Item 20 A 58-year-old woman has been in the ICU for 2 weeks because of respiratory failure after an exacerbation of asthma. and diarrhea. 0. distention.60. They can occur anywhere along the gastrointestinal tract. colonic ischemia is rarely associated with major arterial or venous occlusion. Diagnostic laparoscopy may confirm the diagnosis. any segment of the colon may be affected. A wet appearance with heterogeneous areas of edema was seen in 33 patients (61%). or not at all. Although the splenic flexure is the most common site of ischemic colitis. Pneumatosis may be primary or secondary. diarrhea (commonly bloody). arrhythmias. they are most common in the jejunum. The CT appearance of the colonic wall varied. The clinical course in this patient is highly suggestive of an acute intra-abdominal process. Wall thickness varied between 2 and 20 mm (mean. A number of predisposing or precipitating factors have been delineated. In addition. Intestinal ischemia occurs most commonly in the colon.American College of Surgeons SESAP 13 Critique for Category 3 Item 20 Pneumatosis intestinalis is an uncommon condition presenting as multiple gas-filled cysts of the gastrointestinal tract. rarely. hypotensive episodes. segmental involvement of the colon was seen in 48 patients (89%). myocardial infarction. and colonic obstruction. mucosal damage. muscularis layer and vary from microscopic to several centimeters in diameter. but has been of limited utility with distended bowel loops and is highly dependent on the experience of the surgeon. Risk factors include vascular disease. including arteriosclerotic heart disease. vasculitis. Extraintestinal structures such as mesentery. The abdomen may be tender diffusely. cardiac and aortic surgery. and the falciform ligament may also be involved. with a mean length of involvement of 19 cm (range. bacterial. There is an equal incidence among males and females. About 15% of cases are primary and idiopathic. a dry appearance with mild homogeneous thickening was seen in 18 patients (33%). In a review of computed tomographic (CT) findings in 54 patients with ischemic colitis. Occasionally. peritoneum. The rectum is relatively spared because of its rich collateral circulation. and hypotension. digitalis treatment. page 144 . left lower quadrant). Delayed laparotomy is associated with increased morbidity and mortality. diabetes mellitus. of which the mechanical. submucosa. vasculitis. without a marked sex predilection. Ischemic colitis is a form of nonocclusive ischemic disease usually seen in older patients. and. and intramural air was present in 3 patients (6%). All parts of the colon were involved. which is confirmed by the finding of pneumatosis intestinalis on the abdominal CT scan. A number of theories have been proposed. and pulmonary theories appear to be the most promising. however. most colonic ischemia appears to result from low flow and/or small vessel occlusion. and this condition most commonly occurs in the fourth to seventh decades of life. from the esophagus to the rectum. followed by the ileocecal region and the colon. It also would not allow assessment of bowel wall edema seen with early mesenteric ischemia. Unlike small bowel ischemia. The cause of pneumatosis intestinalis has not been completely delineated. colonoscopy and an upper GI series would yield minimal additional information and delay definitive treatment. Patients with ischemic colitis have an abrupt onset of abdominal pain. These authors concluded that CT can be used to confirm the clinical suspicion of ischemic colitis. in a localized area (eg. the cysts are submucosal and usually are limited to the left colon. Secondary pneumatosis comprises 85% of cases and is typically associated with ischemic colitis. ligation of the inferior mesenteric artery during aortic surgery predisposes to colonic ischemia. The cysts may be located in the subserosa. Instead. 8 mm). 5 to 38 cm). thrombosis or embolism may cause ischemia. and systemic symptoms. Sigild U. pp 480-488 page 145 .Houe T. New York. McQuaid KR. Radiology 211:381388.American College of Surgeons SESAP 13 REFERENCES 1.Thota PN. McGraw-Hill. Eur J Vasc Endovasc Surg 19:304-307. Lashner BA: Miscellaneous diseases of the colon. 1999 2. Grendell JH (eds): Current Diagnosis & Treatment in Gastroenterology. Liisberg-Larsen O.Balthazar EJ. Thorboll JE. in Friedman SL. 2000 3. Yen BC. 2003. et al: Can colonoscopy diagnose transmural ischaemic colitis after abdominal aortic surgery? An evidence-based approach. ed 2. Gordon RB: Ischemic colitis: CT evaluation of 54 cases. One month after her operation she began to have intermittent emesis. Workup has revealed no anatomic cause for her emesis. The most likely cause of her neurologic symptoms is a deficiency of (A) iron (B) folate (C) vitamin B1 (thiamine) (D) vitamin B6 (pyridoxine) (E) vitamin B12 (cobalmin) page 146 .American College of Surgeons SESAP 13 Category 3 Item 21 A 25-year-old woman underwent Roux-en-Y gastric bypass and was discharged on postoperative day 2. She presents 6 months later with persistent emesis and now bilateral symmetric thigh numbness and difficulty walking. mental status changes) are relatively uncommon. glossitis. Vitamin B12 levels should be checked 3 to 6 months after bariatric operations and annually or semiannually thereafter. and is not reported in bariatric patients. In one report. numbness and tingling of hands and feet. E. but can become severe and should be suspected with anatomic difficulties such as anastomotic stenosis. REFERENCES 1. and headache. Bodziner RA: Peripheral polyneuropathy from thiamine deficiency following laparoscopic Roux-en-Y gastric bypass. Anemia due to folate deficiency is somewhat less common than vitamin B12 deficiency. Mild protein-calorie deficiency is not uncommon. brittle nails. The pathologic mechanism is axonal degeneration. Vitamin D deficiency aggravates calcium malabsorption. Vitamin B12 deficiency occurs. vitamins A. with an estimated incidence of 14% to 16% after Roux-enY gastric bypass and 21% to 26% after biliopancreatic diversion. again because calcium is preferentially absorbed in the duodenum and proximal jejunum. and is more malabsorptive. Calcium deficiency is also common after bariatric procedures. Because the total body store of vitamin B12 is approximately 2000 µg and daily needs are 2 µg/day. restricting food intake. Obes Surg 15:890-892. requiring additional iron supplementation. Thiamine absorption is in the duodenum and proximal jejunum.Alvarez-Leite JI: Nutrient deficiencies secondary to bariatric surgery. Symptoms include microcytic anemia. Nutrient deficiencies are more likely with shorter absorptive limbs and are proportional to weight loss. Deficiency can occur even in patients taking multivitamins. Iron deficiency occurs because iron absorption depends on acid exposure to dietary iron and because iron is absorbed mainly in the duodenum and proximal jejunum. 2005 page 147 . Patients present with symmetric numbess and tingling over the anterior thighs bilaterally progressing to bilateral lower extremity weakness and even paralysis if left untreated.American College of Surgeons SESAP 13 Critique for Category 3 Item 21 Roux-en-Y gastric bypass produces weight loss through both restrictive and malabsorptive mechanisms. Curr Opin Clin Nutr Metab Care 7:569-575. deficiency may not present until months to years after operation. particularly after restrictive procedures. Biliopancreatic diversion with and without duodenal switch combines a more modest restriction of gastric size with a longer biliopancreatic limb. Most cases of neurologic beriberi after bariatric surgery are related to persistent emesis or hyperemesis. A small (typically 15 to 30 mL) gastric pouch is formed and connected to a Roux limb of jejunum via a small (10 to 15 mm) anastomosis.Angstadt JD. Thiamine (B1) deficiency is also known as beriberi. but symptoms of deficiency (anemia. Vitamin D and other fat-soluble vitamins are also not absorbed well. Nutrients travel down the digestive limb and are not effectively absorbed until mixed with biliopancreatic secretions in the common limb. and occurs because folate absorption occurs preferentially in the proximal part of the small intestine. Cardiac beriberi presents with cardiac failure. Overall prevalence of vitamin B12 deficiency is estimated at 12% to 33%. Mineral and multivitamin supplementation should be routine in patients after bariatric procedures. and K were deficient in most patients after biliopancreatic diversion. Iron deficiency is the most common nutrient deficiency seen after bariatric procedures. 2004 2. Vitamin B6 deficiency has not been described after bariatric procedures. and comes in two forms. Schirmer BD: Morbid obesity. 2004. in Townsend CM Jr (ed): Sabiston Textbook of Surgery: The Biological Basis of Modern Surgical Practice.American College of Surgeons SESAP 13 3. ed 17. Saunders. pp 357-400 page 148 . Philadelphia. American College of Surgeons SESAP 13 Category 3 Item 22 Which of the following is more common after laparoscopic Roux-en-Y gastric bypass than an open procedure? (A) Anastomotic leak (B) Death (C) Failure of weight loss (D) Internal hernia (E) Wound complication page 149 . and in most studies. Wilson SE. Obes Surg 13:350-354. Wilson SE. Stevens CM. 2005 3.Marema RT. Arch Surg 138:957-961. treatment and prevention.Podnos YD. 2003 4. Buffington CK: Comparison of the benefits and complications between laparoscopic and open Roux-en-Y gastric bypass surgeries. Left untreated. both laparoscopic and open Roux-en-Y gastric bypasses are being performed in record numbers. Austrheim-Smith IT. The incidence of internal hernia is lower when the Roux limb is placed in an antecolic position. REFERENCES 1. Jimenez JC. Diagnosis of internal hernia requires a high index of suspicion. 2003 2.Higa KD. Internal hernia can cause chronic abdominal pain (typically periumbilical) and can be difficult to diagnose because contrast studies and computed tomography (CT) are often normal. et al: Complications after laparoscopic gastric bypass: a review of 3464 cases. Boone KB: Internal hernias after laparoscopic Roux-en-Y gastric bypass: incidence. Perez M. The laparoscopic approach has a higher rate of internal hernia because these defects are more difficult to close laparoscopically and also because adhesion formation is decreased compared with the open procedure. specifically wound infection and incisional hernia. Wolfe BM.Puzziferri N. Mortality has been higher for open procedures in some studies. Ann Surg 243:181-188. Surg Endosc 19:525-530. anastomotic leak rates. Laparoscopic and open procedures have equivalent effectiveness of weight loss.American College of Surgeons SESAP 13 Critique for Category 3 Item 22 Although third-party payers have begun taking steps to limit eligibility of patients. are more prevalent with open procedures. mortality. Ho T. Most repairs can be performed laparoscopically. 2006 page 150 . The laparoscopic approach requires slightly longer operative times and is costlier. et al: Three-year follow-up of a prospective randomized trial comparing laparoscopic versus open gastric bypass. internal hernia can lead to bowel obstruction and/or small bowel necrosis. at the opening in the transverse mesocolon through which the retrocolic Roux limb is passed. likely due to the higher incidence of internal hernias seen after laparoscopic procedures. Wound complications. and behind the Roux limb (Petersen hernia). although overall hospital costs are equivalent for both approaches because of the shortened stay after laparoscopic bypass. and hospital stay is predictably longer. Bowel obstruction is also more common. Internal hernia occurs after Roux-en-Y gastric bypass through three defects: at the jejunojejunal anastomosis. He has a temperature of 99. The abdomen is minimally tender.0°F and oxygen saturation is 88%. Vital signs are blood pressure 130/70. pulse 120 beats/minute. and respiratory rate 20/minute. Over the next several hours he becomes tachypnic with respirations of 40 breaths/minute and requires intubation and mechanical ventilatory support. The following morning he appears anxious and complains of left shoulder pain. Breath sounds are coarse on the left.American College of Surgeons SESAP 13 Category 3 Item 23 A 55-year-old diabetic man with a body mass index of 50 kg/m2 undergoes laparoscopic Roux-enY gastric bypass. The next step should be (A) bronchoscopy (B) upper gastrointestinal study (C) chest and abdominal spiral computed tomography (CT) (D) emergent re-exploration (E) systemic heparin and observation page 151 . detecting only 22% of documented leaks. Upper gastrointestinal studies have particularly poor sensitivity. 2004 2. Fink MP. body mass index > 50 kg/m2) men who have multiple co-morbid conditions are at particular risk. most authors agree that patients with respiratory failure or hemodynamic instability after gastric bypass should be reexplored to rule out this complication. These cardiopulmonary manifestations are due to sepsis. oliguria. Surgical treatment of leaking gastrojejunal anastomosis usually consists of identification of the leak if possible. and leukocytosis. these studies can be falsely negative. particularly anastomotic stricture and gastro-gastric fistula. Diagnosis requires a high index of suspicion. Gallagher SF. New York. pp 477-485 page 152 . Leftsided pleural effusion is particularly troublesome. Jurkovich GJ. Signs can also include fever. hypotension. et al (eds): ACS Surgery: Principles and Practice 2006. and marked anxiety. Obese patients may present with a normal abdominal examination despite the presence of peritonitis. leaks at the gastrojejunal anastomosis can cause signs and symptoms that suggest pulmonary embolism: tachypnea.Gonzalez R. WebMD.American College of Surgeons SESAP 13 Critique for Category 3 Item 23 Leaking of the gastrojejunal anastomosis is the most dreaded complication of Roux-en-Y gastric bypass. It is often mistakenly diagnosed as postoperative pneumonia and is likely caused by irritation of the left diaphragm due to leaking gastric contents. heavier (ie. In addition. pelvic or scrotal pain. chap 5. Older. Repair can be attempted but often fails because of the local inflammation. sec 7. Nelson LG. Other symptoms of leaking gastrojejunal anastomosis include shoulder pain. in Souba WW. Although an upper gastrointestinal series with gastrograffin and computed tomographic (CT) scan can demonstrate the presence of a leak. REFERENCES 1. and placement of large-bore drains near the leak. back pain. Murr MM: Anastomotic leaks after laparoscopic gastric bypass.Sugerman HJ: Morbid obesity. copious irrigation to remove enteric contents from the peritoneal cavity. Kaiser LR. Obes Surg 14:1299-1307. tachycardia. Because delayed diagnosis of leaking gastrojejunostomy markedly increases morbidity and mortality. Patients with leaking at the gastrojejunostomy may also have increases in late complications. and respiratory failure. Gastrointestinal Tract and Abdomen. American College of Surgeons SESAP 13 Category 3 Item 24 Risk factors for Clostridium difficile-associated diarrhea include all of the following EXCEPT (A) advanced age (B) previous Clostridium difficile colitis (C) immunosuppressive agents (D) asymptomatic carrier status (E) nasogastric intubation page 153 . N Engl J Med 346:334-339. the patient becomes an asymptomatic carrier or develops C difficile-associated disease. Of these patients. Proliferation and elaboration of large amounts of toxin result in active disease. Patients who develop a second episode are even more likely (50% to 65%) to develop additional episodes. Results are not available for 24 to 48 hours.American College of Surgeons SESAP 13 Critique for Category 3 Item 24 Antibiotic-associated diarrhea occurs in up to 25% of patients receiving oral or parenteral antibiotics. and bleach must be used for environmental disinfection. If the patient’s colonic flora is disrupted by antibiotic administration. Oral vancomycin may be more efficacious. and peritonitis have severe disease and should be treated with oral vancomycin and colectomy as necessary. The gold standard is the tissue culture cytotoxic assay. Patients with sepsis. Only 10% to 20% of these cases are caused by Clostridium difficile. The spectrum of disease ranges from nuisance diarrhea to toxic megacolon requiring colectomy. one study revealed a failure rate of 78% for patients treated with oral metronidazole. leading to recurrent disease. 2002 2. and this test is no longer widely used. with false negatives of at most 6%. Approximately 12% to 24% of patients develop a second episode of C difficile-associated disease within two months of the initial episode.Bartlett JG: Clinical practice. REFERENCES 1. nasogastric intubation. mild disease should be initially treated with oral metronidazole to avoid vancomycin resistance. The asymptomatic carrier state occurs in patients with a high serum antibody response to one of the C difficile toxins. and has been implicated in numerous outbreaks of severe C difficile-associated disease throughout North America. increasing severity of disease. C difficile spores are not killed by alcohol. Preventive or prophylactic treatment before laboratory diagnosis is confirmed is not recommended. Patients must be carefully observed for progression of disease. enzyme immunoassay. broad-spectrum antibiotics (except aminoglycosides). and the patient is exposed to C difficile spores or vegetative organisms (usually while in the hospital). Antibiotic-associated diarrhea. Risk factors for symptomatic disease include age > 65 years. and protects against symptomatic disease. paralytic ileus. Gerding DN: Clostridium difficile-associated diarrhea. has a false negative rate of 10% to 20%. Clin Infect Dis 26:1027-1034. and treating with oral metronidazole or oral vancomycin. Second and even third samples can be sent if C difficileassociated disease is strongly suspected and initial results are negative. Most laboratories will not accept solid stool samples because the rate of colonization is high. Antiperistaltic agents should be completely avoided when C difficile diarrhea is suspected. The recently identified North American pulsed-field gel electrophoresis type 1 (NAP 1) strain of C difficile elaborates 16 times more toxin A and 23 times more toxin B than other strains. half are believed to have germination of spores persisting after anti-C difficile therapy and half are believed to have acquired a new strain of C difficile. Narcotics and antiperistaltic agents are believed to increase the risk of progression to toxic megacolon. immunosuppressive or antineoplastic therapy. Prevention of C difficile-associated disease involves frequent handwashing for all caregivers and contact precautions for symptomatic patients. Even so. Paralytic ileus can complicate C difficile colitis and paradoxically decrease diarrhea. 1998 page 154 . Diagnosis can be difficult because the most common and rapid test. toxin A. volume depletion. Treatment consists of withdrawing the inciting antibiotics whenever possible. and in some studies anti-ulcer medications.Johnson S. 2006 page 155 . Am Fam Physician 71:921-928.Schroeder MS: Clostridium difficile-associated diarrhea.American College of Surgeons SESAP 13 3. McDonald LC: Clostridium difficile-associated disease: new challenges from an established pathogen. Cleve Clin J Med 73:187-197. 2004 4.Sunenshine RH.Poutanen SM. Simor AE: Clostridium difficile-associated diarrhea in adults. CMAJ 171:51-58. 2005 5. American College of Surgeons SESAP 13 Category 3 Item 25 Appropriate surgical management of a 32-year-old woman suspected of having appendicitis with the intraoperative findings shown includes (A) ileocecal resection with primary anastomosis (B) diverting ileostomy (C) appendectomy (D) Meckel’s diverticulectomy (E) mesenteric lymph node biopsy. bone marrow biopsy and aspirate page 156 . Colombani PM. Mesenteric fat wrapping from chronic transmural inflammation is generally not observed in infectious ileitis or primary small intestinal lymphoma. Endoscopy 37:42-47. in Oldham KT.Sato TT. diversion. while the terminal ileum is grossly inflamed with an edematous mesentery. REFERENCES 1. Newer diagnostic imaging studies such as magnetic resonance enteroclysis and capsule endoscopy are being used with increasing frequency to aid in the diagnosis of inflammatory bowel disease. 2005 2. Resection of an incidentally found Meckel’s diverticulum is not indicated. but do not allow for definitive histologic diagnosis. medical management is often successful for controlling the presenting symptoms to the extent that intestinal resection is not immediately required. pp1313-1328 page 157 . These diagnostic techniques are less invasive than endoscopy. in the absence of acute involvement of the appendix or cecum. or biopsy should be avoided in the untreated patient diagnosed with terminal ileitis because the diagnosis can be made with stool cultures (in the setting of infection) or endoscopy with mucosal biopsy if inflammatory bowel disease is suspected. appendectomy can be performed safely and will eliminate appendicitis as a potential cause of recurrent right lower quadrant pain. However.Herfarth H. ed 2. lymphadenopathy. Philadelphia. 2005. Kugathasan S: Crohn’s disease. Skinner MA (eds): Principles and Practice of Pediatric Surgery. Rogler G: Inflammatory bowel disease.American College of Surgeons SESAP 13 Critique for Category 3 Item 25 Crohn’s disease of the terminal ileum may present with signs and symptoms of acute right lower quadrant pain and tenderness consistent with a diagnosis of appendicitis. Intestinal resection. and mesenteric fat wrapping. the appendix is found to be normal. Foglia RP. After histologic diagnosis of Crohn’s disease. Other causes of terminal ileitis include infection with Yersinia or Campylobacter and other infectious agents. Lippincott Williams & Wilkins. In this setting. American College of Surgeons SESAP 13 Category 3 Item 26 Management of uncomplicated appendicitis in a pregnant woman includes all of the following EXCEPT (A) perioperative fetal monitoring (B) pneumatic compression devices (C) antibiotics and interval appendectomy (D) laparoscopic approach based on surgeon experience and uterine size (E) ultrasonography page 158 . fetal loss rates approach 20%. pp 1381-1399 2. Philadelphia. Fetal loss rates of 3% to 5% are observed with acute appendicitis. ed 17. The use of ionizing radiation associated with computed tomographic (CT) scan or fluoroscopic imaging should be minimized if possible. Pregnancy is a risk factor for DVT and pneumatic compression devices may be appropriate. Clinical suspicion of appendicitis during pregnancy requires prompt surgical intervention in all trimesters.Rollins MD. Clin Obstet Gynecol 45:553561.American College of Surgeons SESAP 13 Critique for Category 3 Item 26 Appendicitis is the most common nonobstetric surgical condition during pregnancy. Mattox KL (eds): Sabiston Textbook of Surgery: The Biological Basis of Modern Surgical Practice. 2004 3.Lally KP. Andrassy RJ: Appendix. Saunders. Evers BM. Nonoperative management using antibiotics and interval appendectomy is no longer warranted in pregnancy. 2004. in Townsend CM Jr.Stone K: Acute abdominal emergencies associated with pregnancy. The choice of open versus laparoscopic appendectomy should be based on surgeon experience and the size of the uterus. The risk of premature labor is greatest during the first postoperative week and mother and fetus should be monitored closely. 2002 page 159 . Ultrasonography is a useful diagnostic adjunct to physical examination findings and will also allow for fetal evaluation. Chan KJ. Price RR: Laparoscopy for appendicitis and cholelithiasis during pregnancy: a new standard of care. REFERENCES 1. Surg Endosc 18:237-241. Beauchamp RD. Symptoms of pain and the location of tenderness vary in pregnancy based on the position of the appendix and its cephalad shift during progressive uterine enlargement. Cox CS. with perforated appendicitis and progression of intra-abdominal infection. 500/mm3) and markedly increased BUN (84 mg/dL) and creatinine (5. The abdominal computed tomographic (CT) scan shown is obtained. He is hypotensive (blood pressure 70/40. 42. but no evidence of peritonitis. He was previously hospitalized for pneumonia and received a 2-week course of intravenous and oral antibiotics.5 mg/dL). Physical examination confirms diffuse abdominal tenderness. The next step in management of this patient should be (A) emergent laparotomy and colectomy (B) initiation of antibiotic therapy for Clostridium difficile colitis (C) diagnostic colonoscopy to evaluate for inflammatory bowel disease (D) therapeutic decompressive colonoscopy (E) mesenteric angiogram to evaluate for colonic ischemia page 160 . and the patient’s blood pressure and WBC count improve. heart rate 130) and anuric. Laboratory examination reveals a significant leukocytosis (WBC count. Aggressive fluid resuscitation is initiated.American College of Surgeons SESAP 13 Category 3 Item 27 A 54-year-old man presents with diffuse abdominal pain associated with fever and diarrhea. and it was higher among patients who did not respond fully to an initial course of therapy versus those who did (33% vs 21%). A significant number of patients (22%) continued to have symptoms of colitis for >10 days despite treatment. The mainstay of diagnosis is detection of C difficile toxin A. A recent prospective observational study of 207 patients treated with metronidazole for C difficile colitis documented that only 103 patients (50%) were cured by the initial course of therapy with no recurrence of disease. and commonly occurs in postoperative surgical patients. morbidity. and some patients (28%) responded initially but had a recurrence within the ensuing 90 days. Therapeutic decompressive colonoscopy is not indicated in the absence of colonic dilation. The mortality rate among patients who developed C difficile colitis was 27%. Recent reports of accelerated incidence. More recent reports have shown a much higher rate of treatment failure. If the first assay is negative and C difficile-associated diarrhea is strongly suspected. Earlier studies suggested that > 90% of patients responded to treatment with metronidazole or vancomycin. or both with a cytotoxin test or enzyme immunoassay of the stool.000/mm3) and an elevated serum creatinine (> 2. C difficile colitis has increased in prevalence. Colonoscopy in this setting of severe colitis also puts the patient at risk for complications.5% in patients who presented with a high leukocyte count (> 20. oral metronidazole can be used in mild disease. C difficile colitis has recently emerged as a prominent cause of disease in hospitalized patients. and rectal vancomycin as well. The reported rate of infection is 3. and failure of the standard treatments support page 161 . This spore-forming toxigenic bacterium is one of the most common causes of infectious diarrhea and colitis in the United States. Operation is indicated for severe infections refractory to medical management or for complications such as toxic megacolon or colonic perforation. with recurrent disease developing in 10% to 15% of cases. A mesenteric angiogram to evaluate for colonic ischemia is not indicated because the abdominal CT scan shows no evidence of colonic ischemia.4 to 8. Another study of 1721 cases of C difficile-associated disease in Quebec documented a mortality rate of 25. The only intravenous therapy available is metronidazole. toxin B. A recent study of 415 C difficile isolates determined that 6.1% manifested intermediate resistance to vancomycin. Clinical presentation varies from asymptomatic colonization to mild diarrhea to fulminant severe colitis. including mucosal bleeding and perforation.3% were resistant to metronidazole and 3. and both the rate and severity of infection appear to be increasing. Standard treatment has been oral vancomycin or metronidazole with equal efficacy.4:1000 hospital admissions. The appearance of metronidazole-resistant clinical C difficile isolates has discouraged the use of this firstline treatment. with a history of previous antimicrobial therapy and a severe leukocytosis.. Intravenous immunoglobulin can be effective in patients with immune deficiency or low pretreatment levels of serum antitoxin. Diagnostic colonoscopy to evaluate for inflammatory bowel disease is not warranted because stool assay for C difficile toxin is accurate. Most patients with C difficile infection have received prior antimicrobial therapy. Oral metronidazole had been recommended as the preferred therapy for most initial infections. Computed tomographic (CT) scan confirms diffuse colitis with thickened bowel wall and no evidence of colonic ischemia. Oral vancomycin is preferred in patients with severe infections. a second assay should be performed. Mortality in patients who require total abdominal colectomy is 48%.American College of Surgeons SESAP 13 Critique for Category 3 Item 27 This patient has a classic presentation for severe Clostridium difficile diarrhea and colitis.0 mg/dL). which could represent a major shift in the epidemiology of C difficile-associated disease. waterless alcohol-based products are not effective in removing C difficile spores. 2005 page 162 . Vigilant handwashing is necessary. 2002 6. Garg R. 2005 2. Killgore G. is a presumptive explanation why all NAP1/027 isolates had a strikingly elevated production of toxin (16 times for toxin A. et al: A predominantly clonal multi-institutional outbreak of Clostridium difficile-associated diarrhea with high morbidity and mortality. Cochrane Database Syst Rev (1):CD004610. Loza A. diagnosis. Miller MA. Efforts to investigate alternative treatment such as binding of toxin with polymers and vaccination are underway. Nelson R. et al: Reassessment of Clostridium difficile susceptibility to metronidazole and vancomycin. Hamill RJ. 2004 4. This strain has been confirmed in 67% of isolates from healthcare facilities and in 37% of communityacquired isolates. The epidemic C difficile variant strain (NAP1/ribotype 027. Oughton M. Dis Colon Rectum 47:1620-1626. et al: Antibiotic treatment for Clostridium difficile-associated diarrhea in adults. 2005 3. Poirier L. Alonso R.Warny M. 2005 5. 23 times for toxin B) compared with nondominant isolates. Antimicrob Agents Chemother 46:1647-1650. Deletion in the gene tcdC. Alcala L. Mazuski JE. Virgo KS.Loo VG. Lee P. toxinotype III) produces substantially more toxin A and B than most strains. REFERENCES 1. Pepin J. Lancet 366:1079-1084. Lancet Infect Dis 5:549-557. and treatment of the disorder.Pelaez T. Rodriguez-Creixems M. et al: Toxin production by an emerging strain of Clostridium difficile associated with outbreaks of severe disease in North America and Europe. a putative down regulator of toxin A and B genes. Evidence indicates that this emerging toxinotype III strain is highly transmissible and more virulent. et al: Outcome after colectomy for Clostridium difficile colitis. Musher DM: Treatment of Clostridium difficile-associated disease: old therapies and new strategies. Fang A.Longo WE.American College of Surgeons SESAP 13 the clinical impression of a more aggressive disease. N Engl J Med 353:24422449. Clinicians need to be vigilant in the prevention.Aslam S.Bricker E. fluid resuscitation (C) surgical pancreatic debridement (D) fine-needle aspiration of peripancreatic fluid (E) prophylactic antifungal agents page 163 . There is no evidence of cholelithiasis or cholecystitis. He is afebrile and hemodynamically normal. He has had no prior episodes of pancreatitis. but is anuric and serum creatinine is 3.0 mg/dL. A computed tomographic (CT) scan confirms necrotizing pancreatitis with a large peripancreatic phlegmon.American College of Surgeons SESAP 13 Category 3 Item 28 A 40-year-old man with no co-morbidities presents with diffuse abdominal pain and distention and a 2-day history of nausea and vomiting. The next step in management should be (A) total parenteral nutrition (TPN) (B) bowel rest. et al: Antibiotic prophylaxis in patients with severe acute pancreatitis. et al: IAP Guidelines for the Surgical Management of Acute Pancreatitis.02) and reduction of pancreatic sepsis (infected necrosis. life-threatening illness within 2 to 3 days. Hirata K. 2006 3. The most important initial treatment priority is to ensure adequate fluid resuscitation to maintain organ perfusion. double-blind trial.Bassi C. Kahl S. A 2004 international consensus conference developed recommendations for the management of the critically ill patient with severe acute pancreatitis.Isaji S. Larvin M. Cook DJ. self-limited course to a rapidly progressive. Enteral nutrition is superior to parenteral nutrition in patients with severe acute pancreatitis. The mortality rate of severe acute pancreatitis exceeds 20% and some patients diagnosed with mild to moderate acute pancreatitis at the onset of the disease may progress to a severe. 2002 6. Most of these studies were limited by small sample size and lack of double blinding. and to improve major organ perfusion in an effort to prevent multiple organ failure. et al: JPN Guidelines for the management of acute pancreatitis: surgical management. which included 4 studies. severe illness. et al: Management of the critically ill patient with severe acute pancreatitis. p=0.51. OR 0. Hepatobiliary Pancreat Dis Int 4:23-27. and delay of operative necrosectomy and/or drainage at least 2 to 3 weeks to allow for demarcation of the necrotic pancreas. Zhu YQ. 2004 4. Crit Care Med 32:2524-2536. Pancreatology 2:565-573. Li YM. p=0. The 2003 Cochrane systematic review. Beale RJ. no pancreatic debridement or drainage for sterile necrosis. limitation of debridement or drainage to those with infected pancreatic necrosis and/or abscess confirmed by radiologic evidence of gas or results of fine-needle aspirate of peripancreatic fluid. Villatoro E: Antibiotic therapy for prophylaxis against infection of pancreatic necrosis in acute pancreatitis. 2005 page 164 .Uhl W. Warshaw A. defined as acute pancreatitis with organ dysfunction. A recent double-blind placebo-controlled multicenter study demonstrated no benefit of antibiotic prophylaxis with respect to the risk of development of infected pancreatic necrosis. and further studies are required to provide adequate data and further define whether antibiotics have any role in this challenging patient population. et al: Prophylactic antibiotic treatment in patients with predicted severe acute pancreatitis: a placebo-controlled. Imrie C. demonstrated an advantage for antibiotic therapy for survival (OR 0. Xue ZL. Runzi M.American College of Surgeons SESAP 13 Critique for Category 3 Item 28 Acute pancreatitis represents a spectrum of disease ranging from a mild. These guidelines are similar to previous guidelines developed by the International Association of Pancreatology and more recent guidelines published by the Japanese. Takada T. 2003 2. 2004 5.Isenmann R. J Hepatobiliary Pancreat Surg 13:48-55.04). Curtis JR. Kawarada Y. Recommendations included no routine use of prophylactic systemic antibacterial or antifungal agents in patients with necrotizing pancreatitis. REFERENCES 1.32. Cochrane Database Syst Rev (4):CD002941. The potential role of prophylactic antibiotics for severe acute pancreatitis remains controversial. to maintain pancreatic perfusion and limit the degree of necrosis. Kron M.Zhou YM.Nathens AB. Bassi C. Gastroenterology 126:997-1004. Proctoscopy shows melena. The next diagnostic study should be (A) colonoscopy (B) radionuclide imaging (C) catheter angiography (D) computed tomographic (CT) angiography (E) capsule endoscopy page 165 . Esophagogastroduodenoscopy and rigid proctoscopy to 20 cm do not identify a source of bleeding.American College of Surgeons SESAP 13 Category 3 Item 29 A hemodynamically normal 68-year-old man presents with a 2-day history of melena. Hoedema RE. Nishioka NS: Effectiveness of current technology in the diagnosis and management of lower gastrointestinal hemorrhage. et al: Urgent colonoscopy for evaluation and management of acute lower gastrointestinal hemorrhage: a randomized controlled trial. with an accuracy of around 45%. but should only be used for patients who are hemodynamically normal. It is more sensitive than angiography but less specific than a positive endoscopic or angiographic study.5 mL/min. a video capsule endoscopy can image the entire small bowel and identify the site of bleeding in 55% to 65% of patients. Radionuclide imaging is well tolerated by patients but is limited by its variable ability to localize bleeding. CT angiography has not been adequately evaluated for detecting the source of lower gastrointestinal bleeding. This study requires a bleeding rate of at least 1 mL/min. REFERENCES 1. Endoscopic therapy can reduce the risk of re-bleeding and the need for operation. Am J Gastroenterol 100:2395-2402. 1995 page 166 . Portwood G. The bleeding site can often be identified by the presence of a fresh clot or active bleeding. When a bleeding site is identified. Radionuclide imaging can detect active bleeding of 0. 2005 2. Dis Colon Rectum 48: 2010-2024. Endoscopy is performed only if the patient presents with hematemesis or bloody nasogastric tube aspirate.1 to 0. colonoscopy yielded a diagnosis in 85% of patients. Radionuclide imaging and angiography may be used when urgent colonoscopy cannot identify a bleeding source or when the bleeding is too rapid to permit colonoscopy.Green BT. Tagged RBC scintigraphy can detect intermittent bleeding by performing serial scans over the lifetime of the radionuclide. Luchtefeld MA: The management of lower gastrointestinal hemorrhage. Christensen MR.American College of Surgeons SESAP 13 Critique for Category 3 Item 29 Diagnostic colonoscopy is the study of choice in patients with acute lower gastrointestinal hemorrhage. Lower gastrointestinal bleeding is sometimes a manifestation of upper gastrointestinal bleeding. In a recent study. Kaplan LM. When work-up of a lower gastrointestinal bleed is negative and an upper endoscopy with push enteroscopy does not reveal a source of bleeding. Catheter-based angiography has a specificity of 100% but a sensitivity of only 30% to 47%. Rockey DC. Tarnasky PR. 2005 3.Richter JM. intra-arterial infusion of vasopressin (Pitressin) or arterial embolization may be performed via the catheter. Gastrointest Endosc 41:93-98. hyperlipidemia.American College of Surgeons SESAP 13 Category 3 Item 30 A 73-year-old man with a history of hypertension. He remains afebrile with a normal WBC count. and severe abdominal distention with mild tenderness and tympany. chronic obstructive pulmonary disease. The abdominal x-ray shown is obtained. abdominal pain. On postoperative day 5. he develops intolerance to food. The most likely diagnosis is (A) sigmoid volvulus (B) cecal bascule (C) gastric torsion (D) small bowel obstruction (E) pancreatitis page 167 . The patient had an appendectomy 50 years ago. and severe peripheral vascular disease undergoes coronary artery bypass grafting for symptomatic coronary artery disease. Although abdominal pain and massive distention are common to both entities. The absence of dilated small bowel on this film is against the diagnosis of small bowel obstruction.Rozycki GS: Special feature: Image of the month. and cecostomy is associated with complications in 52% of cases. a previous abdominal operation. 2001 3. is more often associated with cecal bascule. The signs and symptoms of a cecal bascule are similar to those of cecal volvulus. requiring prolonged ventilatory support. 1992 2.Yarbrough DE. With the cecum flipped upward on itself. Donohue JH: Abdominal pain with bloating won’t subside. originally used to describe a type of balanced drawbridge. The treatment is primarily surgical. Although the patient has risk factors for developing bowel ischemia. Plain abdominal x-ray is the mainstay of the diagnosis. in combination with massive distention. redundant mesentery or hypofixation. elevated WBC count. a deep crease forms across the bowel. In a cecal bascule. Zarroug AE. a bent inner tube shadow in the right upper quadrant would be seen with sigmoid volvulus. Adhesions between the anterior wall of the cecum and the ascending colon result in bascule formation. However. the cecum is 16 cm in diameter. Cecal bascule. The plain x-ray appearance of cecal volvulus would typically be a coffee-bean sign. Arch Surg 136:835-836. REFERENCES 1. leading to occlusion of the gut lumen and bowel obstruction. the recurrence rate is as high as 40% with cecopexy. Contemp Surg 61:559-562. the characteristic intramural air on film. Neither is present here because there is no axial torsion of the bowel. especially appendectomy.American College of Surgeons SESAP 13 Critique for Category 3 Item 30 Bascule is a French word meaning seesaw.Pousada L: Cecal bascule: an overlooked diagnosis in the elderly. allows the cecum to fold onto itself. consisting preferably of resection. J Am Geriatr Soc 40:65-67. Patients with cecal bascule are frequently critically ill. In this patient’s x-ray. Cecopexy and cecostomy are acceptable alternatives in a critically ill patient. and fever that would be seen with bowel ischemia are absent. 2005 page 168 . Gallstone ileus is suspected.American College of Surgeons SESAP 13 Category 3 Item 31 A 91-year-old man presents with small bowel obstruction. the best treatment approach would be (A) conservative management with nasogastric suction (B) laparotomy and milking of the stone into the large bowel (C) cholecystectomy. common bile duct exploration. and extraction of the stone through an enterotomy (D) removal of the stone through a proximal enterotomy (E) colonoscopic disruption and extraction of the stone page 169 . repair of the small bowel component of the fistula. After resuscitation. If the stone can be dislodged proximally. 2003 page 170 . Zovak M. J Hepatobiliary Pancreat Surg 10:299-302. Atmatzidis KS. Sebastian JJ.American College of Surgeons SESAP 13 Critique for Category 3 Item 31 Gallstone ileus represents only 1% of all cases of small bowel obstruction. The stone is most often lodged within the terminal ileum and is beyond the reach of a colonoscope.Pavlidis TE. The one-stage procedure including cholestectomy and fistula repair should be reserved for highly selected patients with absolute indications. cholecystectomy and fistula repair can be performed as a delayed operation. Colonoscopy and stone extraction have been described in a few cases in which the gallstone traversed the ileocecal valve and caused large bowel obstruction. World J Surg 27:400-404. et al: Successful endoscopic relief of large bowel obstruction in a case of a sigmoid colon gallstone ileus.. et al: Comparison of surgical treatments of gallstone ileus: preliminary report. Because these stones often cause irritation and ulceration. an enterotomy is performed and the stone is removed. In these cases. A common bile duct exploration is not necessary because the gallstone usually travels through the fistula between the gallbladder and intestine. Papaziogas BT. Uribarrena R. Glavan E. Papaziogas TB: Management of gallstone ileus. REFERENCES 1. 1997 3. they may be impacted and therefore not amenable to proximal dislodgement. Solanilla P. The small bowel component of the cholecystenteric fistula can be repaired at the time of cholecystectomy. These stones are large (diameter > 2. Simple enterotomy should be the procedure of choice for patients with gallstone ileus.0 cm) and cannot be milked through the ileocecal valve. Kopljar M.Doko M. but accounts for 15% of complete obstruction in patients over age 70. 2003 2.Garcia-Lopez S. Removal of the gallbladder is not necessarily indicated during the first operation unless biliary symptoms are present. J Clin Gastroenterol 24:291-292. a limited small bowel resection of the segment containing the stone is appropriate. Addressing only the small bowel obstruction is acceptable if the patient is unstable. a 60-yearold man has the computed tomographic-positron emission tomographic (CT/PET) scan shown. the most appropriate treatment would be (A) transhiatal esophagectomy (B) total esophagectomy with extended lymph node dissection (C) transhiatal esophagectomy with liver resection (D) additional chemotherapy only (E) additional chemotherapy and radiation therapy page 171 . Of the following.American College of Surgeons SESAP 13 Category 3 Item 32 Following preoperative chemoradiation therapy for a distal esophageal adenocarcinoma. Some also advocate endoscopic ultrasound evaluation to determine depth of tumor involvement. Chemotherapy can be used in patients with metastatic esophageal cancer. Unlike colorectal carcinoma. and discontinuous lymph node involvement. but not beyond. superior to the level of the carina. bronchoscopy is also indicated to rule out tracheobronchial involvement. Staging for esophageal adenocarcinoma after definitive diagnosis by endoscopy and biopsy includes a CT scan of the chest. Radiology 236:841-851. and would be this patient’s only viable option for treatment. 2005 3. The presumption is that malignant cells will be more metabolically active than the surrounding tissue and will show up as hot spots on PET. ways to achieve palliation. especially with surrounding structures. but PET is helpful is confirming malignancy in lesions that are equivocal by CT alone. Curr Opin Gastroenterol 21:472-477. and FDG PET for assessment of response to neoadjuvant therapy—systematic review. recent meta-analyses demonstrate a modest survival advantage. Whether PET has sufficient sensitivity and specificity to be used as a screening test for metastatic cancer in a patient with no overt signs of metastatic spread is controversial. identifies unresectable disease. Metabolically active cells take up the glucose and cannot excrete it because of the fluoride moiety. The presence of any distant metastases. REFERENCES 1. Hoekstra OS. liver resection does not provide a survival advantage in patients with hepatic metastases from adenocarcinoma of the esophagus. peritoneum.Law S. endoscopic US. J Gastrointest Surg 9:291-310.von Rahden BH. a CT scan of the head is warranted. radiation therapy is considered local therapy and is also not indicated except in a palliative role. ie. PET uses a fluorinated glucose molecule with a radioactive phosphorus moiety. There is no role for palliative esophagectomy. such as an endoscopically placed stent. Either treatment option is predicated on disease being localized to the esophagus and regional lymph nodes. Because of the radioactive phosophorus moiety. Stein HJ: Staging and treatment of advanced esophageal cancer. and pelvis. Wong J: Current management of esophageal cancer. Although the value of neoadjuvant therapy with respect to improved survival is controversial. positive celiac lymph nodes with a cancer in the proximal esophagus. abdomen. Involvement of a discontinuous lymph node basin. primarily liver. Reitsma JB. There are other. 2005 2. The presence of distant metastasis makes this patient’s tumor unresectable.American College of Surgeons SESAP 13 Critique for Category 3 Item 32 This patient’s computed tomographic-positron emission tomographic (CT/PET) scan demonstrates a hypermetabolic focus corresponding to a liver mass in a patient with a known esophageal adenocarcinoma. van Westreenen HL. If there are any presenting neurologic symptoms. and lung. et al: Esophageal cancer: CT.Westerterp M. Like operation. areas of high metabolic activity are identified. If the lesion is in the proximal esophagus. eg. is also considered M1 disease. There are two acceptable treatment options for esophageal adenocarcinoma: surgical resection alone or preoperative (neoadjuvant) chemoradiation. less invasive. 2005 page 172 . American College of Surgeons SESAP 13 Category 3 Item 33 A hemodynamically normal patient has abdominal pain and distention. Which of the following statements is TRUE? (A) This condition typically occurs in young women (B) Initial management should be nasogastric decompression and bowel rest (C) This patient is a candidate for colon wall stenting (D) This abnormality could have been detected on a colonoscopy 6 months previously (E) The patient does not require emergent surgical management page 173 . The plain films and contrast enema shown are obtained. but no peritoneal signs. pp 315-320 2. A redundant sigmoid colon with a narrow base of attachment to its mesentery is a common anatomical finding in these patients. endoscopic decompression with either rigid or more commonly flexible endoscopy is performed. 2004. in Fazio VW. distention. Nelson H: Colonic volvulus. Colon cancer is the most common cause of large bowel obstruction in the US and should be distinguished from colonic volvulus. vomiting. in Cameron JL (ed): Current Surgical Therapy. Retrograde gastrografin enema in the setting of volvulus shows a bird’s-beak deformity with smooth mucosa. including assessment of colonic viability. Barium enema reveals a tapering deformity at the rectosigmoid junction known as a bird’s-beak sign. Mosby.American College of Surgeons SESAP 13 Critique for Category 3 Item 33 Sigmoid volvulus is the most common location of colonic volvulus and is the third most common cause of large bowel obstruction in Western countries. whereas colon cancer more typically shows an apple-core lesion with shouldering edges and an abnormal mucosal pattern. Sigmoid colectomy is the procedure of choice. Otherwise. behind carcinoma and diverticulitis. Colon cancers arise from adenomatous polyps over a period of years. Emergent laparotomy is indicated in patients believed to have nonviable colon or perforation. fluid and electrolyte resuscitation. and obstipation. ed 2. Sigmoid volvulus typically presents with abdominal pain. and either endoscopic or surgical decompression. Selected cases of obstructing colon cancer can be palliated with endoscopic dilatation and stent placement. 2005. Computed tomographic (CT) scan may reveal spiraling of the mesentery known as the whirl sign. Delaney CP (eds): Current Therapy in Colon and Rectal Surgery. Recurrence after resection is extremely rare. Had this scenario represented a patient with an obstructing colon cancer. The typical patient is 60 to 70 years old with a long-standing history of constipation or laxative abuse. nausea. Stryker SJ: Volvulus of the colon.Bimston DN. This is not only diagnostic but therapeutic. Church JM. ed 8. REFERENCES 1. and elective resection on the same hospitalization is recommended. oriented from the left lower quadrant to the right upper quadrant. this lesion would have been detected on a colonoscopy 6 months previously. Philadelphia. urgent operation is required. Stenting has no role in the management of sigmoid volvulus. Mosby. Characteristic findings on plain film include a markedly distended anhaustral loop of airfilled colon. which approximates the appearance of a bent inner tube. Initial management of sigmoid volvulus involves rapid evaluation. Philadelphia. pp 179-182 page 174 . Sigmoid volvulus treated with endoscopic decompression alone has a recurrence rate of 40% to 50%. with reported success rates of 75% to 90%.Drelichman ER. If endoscopic evaluation fails or demonstrates extensive mucosal ischemia. The most appropriate management would be (A) a draining seton (B) saucerization (C) fibrin glue (D) a cutting seton (E) an endorectal advancement flap page 175 .American College of Surgeons SESAP 13 Category 3 Item 34 A 45-year-old woman with active Crohn’s proctitis has a symptomatic anterior transphincteric anovaginal fistula. Jagelman DG: The use of transanal rectal advancement flaps in the management of fistulas involving the anorectum. Fazio VW. 1995 4. Drainage of the abscess may be followed by a loose draining seton and medical therapy in hopes of resolving the active inflammation in preparation for definitive surgical repair.Jones IT. Fibrin glue repair is an appealing healing option in that it is a simple and repeatable sphincter-preserving technique. Factors to be considered include associated disease activity and anatomic relations of the fistula. REFERENCES 1.Michelassi F. Multiple therapeutic options are available. Am J Surg 173:95-98.American College of Surgeons SESAP 13 Critique of Category 3 Item 34 Perianal fistulas are a common manifestation of Crohn’s disease. Fistula symptoms are often promulgated by areas of undrained infection. Surgical repairs such as saucerization or placement of a cutting seton will not heal in the presence of grossly active proctitis and may result in debilitating. 1997 2. Br J Surg 82:603-606. however. 1987 3. superficial fistulas. and the amount of sphincter traversed by the fistula. Melis M. especially in fistulas that are short and/or associated with Crohn’s disease. Dis Colon Rectum 30:919-923. Hurst RD: Surgical treatment of anorectal complications in Crohn’s disease. In the setting of minimal or no proctitis. Becker HD. including level of internal opening.Hull TL. Perianal Crohn’s disease is a chronic relapsing condition that places the anal sphincter at lifelong risk of injury. Jehle EC. Surgery 128:597-603. 2000 page 176 . and with disease-related diarrhea places these patients at high risk for potential fecal incontinence. Fazio VW: Surgical approaches to low anovaginal fistula in Crohn's disease. Fistulotomy should be reserved for short. it has a high failure rate. Starlinger M: Clinical course after transanal advancement flap repair of perianal fistula in patients with Crohn's disease.Makowiec F. Rubin M. associated tracts. an endorectal advancement flap has been successful in 50% to 70% of patients with Crohn’s fistulas. slow-healing wounds. All efforts should be made at sphincter-preserving treatments. American College of Surgeons SESAP 13 Category 3 Item 35 The distal margin of resection for sigmoid diverticulitis should be (A) 2 cm distal to the colon inflammation (B) the sacral promontory (C) uninflamed distal sigmoid (D) the peritoneal reflection (E) normal rectum page 177 . The incidence increases with age. Berho M. Surgical goals include resection of the diverticulitis with margins of uninvolved colon in each side of the inflamed segment of bowel. The distal margin of resection for sigmoid diverticulitis should be normal rectum because the rectum does not have any tinea coli and the incidence of rectal diverticulitis is exceedingly rare. The risk of recurrent diverticulitis is considerably lower when the distal margin of resection is to the rectum instead of the distal sigmoid colon. Diverticulitis is a peridiverticular infection resulting from perforation of a diverticulum and subsequent leakage of stool from the colon. or medical intractability. et al: Determinants of recurrence after sigmoid resection for uncomplicated diverticulitis. Wolff BG. fistula. The process usually resolves with a short course of antibiotics. REFERENCES 1. 2003 page 178 .Benn PL. Dis Colon Rectum 46:385-388. 1986 2. Operation for uncomplicated diverticulitis is generally reserved for patients who have had multiple recurrences.Thaler K. Am J Surg 151:269-271. Ilstrup DM: Level of anastomosis and recurrent colonic diverticulitis. Surgical resection of the inflamed segment of colon is indicated for patients with complicated diverticulitis as manifested by generalized peritonitis.American College of Surgeons SESAP 13 Critique for Category 3 Item 35 Diverticular disease is a common acquired anatomical condition of the colon rarely occurring before age 30. Baig MK. abscess formation. and this condition is present in 75% of Americans over age 80. Weiss EG. American College of Surgeons SESAP 13 Category 3 Item 36 The use of computed tomography (CT) in suspected appendicitis (A) lowers the incidence of negative appendectomy (B) has become more accurate over the past 10 years (C) eliminates the need for serial re-examinations (D) has decreased the rate of perforated appendicitis at operation (E) may detect pelvic abnormalities in women page 179 . Wilson EB. REFERENCES 1. Nipper ML. Cole JC. but the negative appendectomy rate had actually increased from 12% to 17%. Wilbanks TO. Technical improvements in CT do not appear to have increased its accuracy. and ethical considerations would make such a study unlikely. et al: Liberal use of computed tomography scanning does not improve diagnostic accuracy in appendicitis.American College of Surgeons SESAP 13 Critique for Category 3 Item 36 Although computed tomography (CT) is now more commonly used for suspected appendicitis. its diagnostic value remains questionable. the use of CT had increased fourfold. et al: Computed tomography and ultrasonography in the diagnosis of appendicitis: when are they indicated? Arch Surg 136:670-675. Salem L. Jorgensson D. Am J Surg 185:194-197. The rate of perforated appendicitis was 13% in 2000 and 11% in 1994. which is consistent with many other studies in which CT was not used. The negative appendectomy rate was 17%. 2003 3. CT and re-examination resulted in a change in diagnosis in one third of cases.Perez J. Accuracy of the CT was 80% in 2000 and 81% in 1994. and the diagnosis of pelvic inflammatory disease or ruptured ovarian cyst could be made in 6 women.Anderson BA. Flum DR: A systematic review of whether oral contrast is necessary for the computed tomography diagnosis of appendicitis in adults. In a study of CT (and ultrasound) in 100 consecutive patients in which CT was followed by re-examination. Cooney DR. Barone JE. No study has compared CT alone with re-examination alone. 2005 2. 2001 page 180 . In a study comparing 100 consecutive cases in 2000 with 100 consecutive cases in 1994. The only significant effect found was an increase in the length of stay in the emergency department. Re-examination remains valuable for suspected appendicitis. Am J Surg 190:474-478. although this was not significant. 6-cm homogeneous mural mass in the stomach without evidence of metastasis. An abdominal computed tomographic (CT) scan shows a solid.American College of Surgeons SESAP 13 Category 3 Item 37 An otherwise healthy 63-year-old man presents with upper gastrointestinal hemorrhage. Endoscopic evaluation reveals a 6-cm intramural tumor with stigmata of recent bleeding along the greater curvature of the body of the stomach. The most appropriate management at this time would be (A) a trial of imatinib (Gleevec) (B) subtotal gastrectomy (C) wedge excsion of the tumor with grossly clear margins (D) enucleation (E) endoscopic ultrasound (EUS) page 181 . Ann Surg 231:51-58. Lewis JJ. and molecular genetic study of 1765 cases with long-term follow-up.DeMatteo RP. LeungD. Am J Surg Pathol 29:52-68. These tumors are best staged with abdominal computed tomographic (CT) scans to assess metastasis to the liver and/or direct invasion of adjacent structures. Metastatic disease is the only reliable proof of malignant potential.Miettinen M. 2000 2. 2005 page 182 . Lymphatic metastasis is rare. but these criteria were refined by a recent study from the Armed Forces Institute of Pathology that identified key parameters as tumor size > 10 cm and > 5 mitoses/50 HPF. et al: Two hundred gastrointestinal stromal tumors: recurrence patterns and prognostic factors for survival.American College of Surgeons SESAP 13 Critique for Category 3 Item 37 Current studies suggest that the prevalence of gastrointestinal stromal tumors (GIST) has increased. Although most GISTs respond to imitanib (Gleevec). Mudan SS. Surgical excision is the best treatment for most patients with these tumors. For many gastric GISTs. The frequency of c-kit positivity was 90%. Sobin LH. Tumors > 10 cm and with > 5 mitoses/50 HPF had a recurrence rate of 85% versus 10% to 15% for tumors over 10 cm with low mitoses or tumors less than 10 cm but with high mitoses. Lasota J: Gastrointestinal stromal tumors of the stomach: a clinicopathologic. and adjuvant use of Gleevec is not warranted with such a low chance for metastatic recurrence. Assessing the malignant potential of GISTs is problematic. Local recurrences were rare in this series. cure does not occur. Tumor-specific mortality was 17%. Tumors < 10 cm and with < 5 mitoses/50 HPF had a recurrence rate of only 2%. and would add nothing to the care of the patient described in this item. Histologic evaluation of the neoplasm for c-kit will predict the likelihood of response to Gleevec. Traditional pathologic criteria predicting malignant potential were tumor size > 5 cm and number of mitoses/50 high-power fields (HPF). and simple excision with clear margins is appropriate. REFERENCES 1. immunohistochemical. it usually does not affect management. this can be accomplished with wedge resections. Although endoscopic ultrasound (EUS) effectively assesses the depth of invasion. and vomiting.American College of Surgeons SESAP 13 Category 3 Item 38 A 28-year-old women arrives in the emergency department with a 72-hour history of diffuse abdominal pain. The computed tomographic (CT) scan shown is obtained. All of the following would be indicated EXCEPT (A) insertion of a nasogastric tube (B) endoscopy (C) corticosteroids (D) enteral nutrition (E) exploratory laparotomy page 183 . She has been able to tolerate very little oral intake without emesis. nausea. Dunn DL. New York. and immunomodulating (infliximab) drugs. 2006 5. Church M. in Brunicardi FC.Schraut WH: The surgical management of Crohn's disease. thickening. especially in the case of failure or contraindication of medical therapies including corticosteroids. Andersen DK. Practitioners should be aware of this efficacy.American College of Surgeons SESAP 13 Critique for Category 3 Item 38 Crohn’s disease is an immune-mediated chronic inflammatory disease that may affect any part of the intestinal tract. et al: European evidence based consensus on the diagnosis and management of Crohn's disease: current management. nor is extraluminal air. et al: Effect of resection margins on the recurrence of Crohn's disease of the small bowel. Prior to any operation. Gastroenterol Clin North Am 31:255263. The CT scan does not indicate an acute need for a surgical exploration. Zinner MJ: Small intestine. Granulomas and lymphoid aggregates occurring distal to active disease are suggestive of the diagnosis. but the small bowel alone is involved in 30% to 35% of cases and the colon alone in 25% to 35% of cases. REFERENCES 1. 1996 3. Oresland T. The comparable European Cooperative Crohn’s Disease Study of 105 patients achieved 83% remission with 6-methylprednisolone compared with 38% taking placebo over 18 weeks. The National Co-operative Crohn’s Disease Study randomized 162 patients. pp 10171054 page 184 . blunting. CT permits accurate demonstration of any abscess. et al (eds): Schwartz’s Principles of Surgery. compared with 30% receiving placebo. Operation may acutely relieve symptoms and improve the quality of life. Ann Surg 224:563-571. but diarrhea or malnutrition may result from the loss of bowel and recurrence risk is not reduced. immunosuppressing. and a strategy that may prevent a later requirement for ostomy. The CT images show acute right lower quadrant inflammation. Lemann M. Goldblum JR. sometimes with intervening skip areas.Travis SP. Progression affects segments of the small bowel. Stange EF. The ability of Crohn’s disease to affect any part of the intestine at any time has shaped our treatment philosophy.Whang EE. A clear abscess is not present. Marchetti F. Three fourths of Crohn’s patients undergo operation after 20 years and 90% after 30 years of symptoms. It is diagnosed by a combination of clinical. Operation is not a cure for Crohn’s disease. and should be reserved for specific complications. biological. Gut 55 (Suppl 1):i16-i35. The condition usually affects the terminal ileum and cecum.Fazio VW. 2005. and straightening of the valvulae conniventes is accompanied by inflammation of the bowel wall and small ulcers. the extent of disease should be documented by contrast or endoscopy studies. 2001 2. A randomized controlled trial. During the initial nonstenotic phase. and morphologic features. Billiar TR. culture. Enteral nutrition has a significant efficacy in adults with acute Crohn’s disease. McGraw-Hill. Two important trials established corticosteroids as effective therapy for inducing remission in Crohn’s disease.Delaney CP. ed 8. allowing for preliminary drainage. achieving 60% remission with prednisone. Surg Clin North Am 81:137-158. Fazio VW: Crohn's disease of the small bowel. Computed tomographic (CT) scanning in patients with suspected sepsis is even more important. 2002 4. Ashley SW. He has never had symptoms like this before. Chest x-ray demonstrates mediastinal fluid. The most appropriate management would be (A) esophagoscopy (B) tube thoracostomy (C) video-assisted thoracic surgery (VATS) with tube drainage (D) expectant management (E) thoracotomy page 185 .American College of Surgeons SESAP 13 Category 3 Item 39 A 51-year-old man with gastroesophageal reflux disease arrives in the emergency department with a 6-hour history of persistent chest pain that began after repeated episodes of emesis. ed 8. Accurate diagnosis and effective treatment depend on early recognition of clinical features and accurate interpretation of diagnostic imaging. Tiu AT. Dunn DL.Reeder LB. McGraw-Hill. A collective review of 13 series published between 1980 and 1990 disclosed an overall mortality of 22% from esophageal perforation. Most injuries are iatrogenic and the increasing use of endoscopic procedures will likely increase the incidence of esophageal perforation.esophageal stenting as an alternative to surgical repair. No current data demonstrate that tube decompression or thoracoscopic exploration and drainage without surgical repair is better than surgical repair in esophageal perforations less than 24 hours old. in Brunicardi FC. 2003 2. Andersen DK. Am J Surg 190:161-165.Hill AG. pp 835-932 3. Outcome is determined by the cause and location of the injury. and the interval between perforation and initiation of therapy. ANZ J Surg 73:1008-1010. There are limited data looking at esophagoscopy +/. Mortality after drainage was 34% compared to 15% for suture closure. but these are small case series that have typically included delayed clinical presentations without long-term follow-up with respect to patient outcomes. DeMeester TR: Esophagus and diaphragmatic hernia. 2005 page 186 . Billiar TR. with or without a buttress of endogenous tissue. Martin IG: Boerhaave's syndrome: 10 years experience and review of the literature. The overall mortality associated with esophageal perforation approaches 20%. Surgical primary repair.Richardson JD: Management of esophageal perforations: the value of aggressive surgical treatment. the trend in the literature favors an aggressive approach. Operative therapy for esophageal perforation remains controversial. 1995 4. REFERENCES 1. Am J Surg 169:615-617.American College of Surgeons SESAP 13 Critique for Category 3 Item 39 Esophageal perforation is a devastating event that is difficult to diagnose and manage.Peters JH. the presence of concomitant esophageal disease. While some patients meet established guidelines for conservative treatment. One of the most controversial areas of management relates to whether drainage alone (either open or closed) will suffice for treatment. Spontaneous esophageal rupture (Boerhaave's syndrome) is a rare condition. 2005. Ferguson MK: Current results of therapy for esophageal perforation. and treatment delay of more than 24 hours after perforation doubles mortality. et al (eds): Schwartz’s Principles of Surgery. is the most successful treatment option and reduces mortality by 50% to 70% compared with other interventional therapies. New York. DeFilippi VJ. His WBC count is slightly elevated.American College of Surgeons SESAP 13 Category 3 Item 40 A 22-year-old man arrives in the emergency department with the sudden onset of acute abdominal pain in the mid-epigastric region and associated emesis. On abdominal examination he is diffusely tender to palpation. His vital signs are pulse. The next step should be (A) administration of a Fleets enema (B) insertion of a nasogastric tube (C) diagnostic colonoscopy (D) emergency laparotomy (E) vascular arteriogram of his mesenteric vessels page 187 . blood pressure. 110/80. He has had intermittent but much less severe abdominal pain over the past few months. 125/min. 18/minute. and respiratory rate. The image (left lateral decubitus view) shown is obtained. Treatment remains the Ladd procedure as originally described in 1936. 2003 page 188 . there is nothing pathologically wrong with the vasculature of the intestinal contents. and often emergent laparotomy. appropriate diagnostic studies. 2002 2. Grundfest-Broniatowski S. Johnson RK. Fleets enema has no role in the setting of an acute abdomen. Symptomatic patients present either acutely with bowel obstruction and intestinal ischemia with a midgut or cecal volvulus. It is rare for malrotation to present in adulthood. et al: Intestinal malrotation: a rare but important cause of bowel obstruction in adults. While malrotation is a function of the midgut incompletely rotating around the superior mesenteric artery. and immediate operation is indicated. first described by Ladd in 1932.American College of Surgeons SESAP 13 Critique for Category 3 Item 40 Midgut malrotation is a congenital anomaly in which rotation of the intestines around the axis of the superior mesenteric artery during fetal development does not occur or is incomplete.Dietz DW. Complete resolution of acute obstruction or chronic abdominal pain is the result of a high index of suspicion for malrotation. and the true incidence in adults is difficult to estimate. REFERENCES 1. Rothwell M: Adult malrotation: a case report and review of the literature. or chronically with vague abdominal pain. Chronic symptoms can often make the diagnosis difficult. Symptoms are caused by peritoneal bands. Lavery IC. angiography.Gamblin TC. Dis Colon Rectum 45:1381-1386. Findings diagnostic of malrotation are described using several modalities such as barium studies. Curr Surg 60:517-520. Stephens RE Jr. Diagnostic colonoscopy would identify clinical pathology to the level of the terminal ileum but would be unhelpful in diagnosing or alleviating the clinical condition of small bowel obstruction and midgut volvulus. Most patients remain asymptomatic and their conditions are never diagnosed. Nasogastric decompression is a component of the management of a patient who presents with an acute abdomen secondary to a midgut volvulus. and aggressive definitive surgical treatment. Most patients present with bilious vomiting in the first month of life because of duodenal obstruction or a volvulus. computed tomographic (CT) scans. Walsh RM. The critical factor in this clinical presentation suggests that the patient’s intestinal viability could be compromised. running from the cecum to the right lateral abdominal wall. Four 1-cm submucosal tumors are identified in the body of the stomach. The best management would be (A) total gastrectomy (B) proton pump inhibitors (C) antrectomy with gastroduodenostomy (D) proximal gastrectomy with esophagogastrostomy (E) enucleation of all 4 tumors page 189 . and associated hypergastrinemia. Biopsies show carcinoid tumor. Further work-up establishes a diagnosis of gastric carcinoid tumor associated with gastric achlorhydria. pernicious anemia. Fasting serum gastrin concentration is 950 pg/mL (normal. 100 pg/mL).American College of Surgeons SESAP 13 Category 3 Item 41 A 66-year-old otherwise healthy man with epigastric abdominal pain and anemia is evaluated with upper endoscopy. serum gastrin levels are normal in the sporadic carcinoid group. and partial gastrectomies and lymphadenectomies are indicated for these patients. histologic confirmation of atrophic parietal cell mucosa. Baybick JH. present in this patient. An immunohistochemical and clinicopathologic study of 104 patients. Elsayed AM.Woodside KJ. Sporadic gastric carcinoid tumors are potentially lethal. These patients tend to have multiple small gastric carcinoids in the body of the stomach caused by the trophic effect of gastrin on intestinal mucosa. REFERENCES 1.Thomas RM. treatment should focus on eliminating hypergastrinemia. not ZES. and because treatment depends on the etiology. the most cost-effective test to determine the cause of hypergastrinemia in the patient described in this scenario is one that confirms atrophic gastritis. J Gastrointest Surg 8:742-756. 2004 2. Serum levels of chromogranin A should be measured in all carcinoid patients. By definition. 2004 page 190 . Low serum levels of vitamin B12. Cancer 73:2053-2058. making enucleation or resection of multiple small gastric carcinoids unnecessary. Sobin LH: Gastric carcinoids. Finally. Sweeney J: Gastric carcinoids in patients with hypergastrinemia. work-up must be directed at determining the underlying etiology. Evers BM: Current management of gastrointestinal carcinoid tumors. and total gastrectomy is unnecessarily radical treatment. is associated with gastric achlorhydria. some gastric carcinoids occur in the setting of ZollingerEllison syndrome (ZES) and hypergastrinemia. Townsend CM Jr. Antisecretory agents such as proton pump inhibitors or histamine H2 receptor antagonists have no role in treatment of these patients. pernicious anemia. which usually occur as larger. Proximal gastrectomy adds morbidity and does not address the etiology of the disease. Multiple small tumors in the setting of hypergastrinemia have low (but not zero) malignant potential. solitary tumors. A second type. Because atrophic gastritis is much more common than ZES. Barroso A. Antrectomy (resection of all gastrin-producting cells) and normalization of serum gastrin levels will result in regression of all tumors less than 1 cm in most patients with atrophic gastritis. J Am Coll Surg 199:552-555. and high gastric pH all confirm the diagnosis and/or rule out ZES. 1994 3.Jordan PH Jr. Baseline and follow-up levels can be used to monitor these patients. Some patients have sporadic tumors.American College of Surgeons SESAP 13 Critique of Category 3 Item 41 Gastric carcinoid tumors occur in three distinct settings. Octreotide scans would be indicated only for patients with ZES or large sporadic tumors. Unless the tumors are larger than 1 to 2 cm. and associated hypergastrinemia. The most appropriate treatment for this patient would be (A) oral imatinib mesylate (Gleevec) (B) total gastrectomy (C) partial gastrectomy (D) radiation therapy (E) distal pancreatectomy page 191 .American College of Surgeons SESAP 13 Category 3 Item 42 A 53-year-old man with dyspepsia has the abdominal computed tomographic (CT) scan shown. and/or liver. peritoneum. J Clin Oncol 21:4342-4349. LeungD. Corless CL. and a limited gastric resection that achieves a negative margin is adequate.American College of Surgeons SESAP 13 Critique for Category 3 Item 42 Gastrointestinal stromal tumors (GISTs) represent 1% of all gastrointestinal malignancies and are the most common sarcoma of the gastrointestinal tract. Imatinib mesylate (Gleevec). Sarlomo-Rikala M. More than 95% of tumor cells in GISTs will possess activating mutations of the c-KIT tyrosine kinase receptor. Recent data suggest that the true incidence is as high as 4500 to 6000 cases per year in the United States. N Engl J Med 344:1052-1056. an oral tyrosine kinase inhibitor. Andersson LC. REFERENCES 1. and preoperative biopsy before resection is generally unnecessary and potentially unsafe. Demetri GD. 2000 2.DeMatteo RP. However. Until recently the cellular origin of these tumors was unknown. Lewis JJ. Operation is the treatment of choice for localized tumors. This patient should undergo partial gastrectomy with negative margins and does not require either pancreatic resection or total gastrectomy. External-beam radiation therapy is not indicated. Roberts PJ. In patients with localized GIST. GISTs are usually identified as well-circumscribed masses (~5 cm) on imaging studies. 2001 page 192 . GISTs are most commonly found in the stomach and are associated with upper gastrointestinal symptoms or bleeding. The goal of operation is to obtain a histologically negative margin. et al: Effect of the tyrosine kinase inhibitor STI571 in a patient with a metastatic gastrointestinal stromal tumor. the 5-year actuarial survival rate is ~54% if complete resection of gross disease is achieved. Blanke CD. Survival is predicted by tumor size but not microscopic margins of resection. is effective in treating locally recurrent or metastastic GIST. These neural pacemaker cells are responsible for smooth muscle activity in the gastrointestinal tract. which can be identified with immunohistochemical staining of the specimen (figure). et al: Kinase mutations and imatinib response in patients with metastatic gastrointestinal stromal tumor.Joensuu H. As demonstrated by the serial computed tomographic (CT) images presented in this case.Heinrich MC. Ann Surg 231:51-58. it is currently not recommended for patients with surgically resectable disease. this tumor is arising from the stomach and not the pancreas. et al: Two hundred gastrointestinal stromal tumors: recurrence patterns and prognostic factors for survival. 2003 3. but it is now believed that GISTs arise from the interstitial cells of Cajal. Mudan SS. Recurrence of disease after resection is usually intra-abdominal and involves the original tumor site. American College of Surgeons SESAP 13 Category 3 Item 43 A 65-year-old man has a biopsy-proven gastric carcinoma on the lesser curve of the stomach 5 cm distal to the esophagogastric junction. The most appropriate surgical therapy for this patient would be (A) esophagogastrectomy with colonic interposition (B) subtotal gastrectomy with a Billroth II anastomosis (C) total gastrectomy (D) total gastrectomy and splenectomy (E) esophagogastrectomy with jejunal interposition page 193 . Staging laparoscopy confirms these findings. Computed tomography (CT) identifies enlarged lymph nodes along the lesser curve of the stomach without other evidence of metastatic disease. The tumor in this patient is clearly arising from the stomach without involvement of the GEJ (type III). 1999 2. Weeden S. Fielding J. 1996 6. 1989 page 194 .Cuschieri A. Total gastrectomy is needed in this case because the tumor is within 6 to 8 cm of the gastroesophageal junction (GEJ). tumor epicenter within the esophagus. To achieve an R0 gastric resection. Resection of the esophagus is not required unless the tumor involves the GEJ. Hermans J. Staging laparoscopy will identify metastatic disease in up to 30% of patients. Hermans J. Craven J. Sasako M. Lancet 345:745-748. et al: Extended lymph-node dissection for gastric cancer.Bozzetti F. the proximal margin must be at least 6 cm from tumor to the cut edge. et al: Patient survival after D1 and D2 resections for gastric cancer: long-term results of the MRC randomized surgical trial. Ann Surg 210:596-602. postoperative morbidity and mortality were significantly increased if the spleen and/or pancreas were resected.Bonenkamp JJ. tumor epicenter at the cardia with tumor involvement at the GEJ. Ann Surg 196:685-690. The difficulty in determining the extent of lymphadenectomy occurs because the presence of micrometastatic nodal disease cannot be identified at the time of operation. et al: Adequacy of margins of resection in gastrectomy for cancer. Accordingly. In randomized trials examining extended lymphadenectomy. 1982 4. Songun I.Maruyama K. Bancewicz J.Cuschieri A. Okabayashi K. Surgical Co-operative Group.American College of Surgeons SESAP 13 Critique for Category 3 Item 43 The central controversies in the surgical management of gastric carcinoma are focused on the extent of gastric resection. type II. et al: Postoperative morbidity and mortality after D1 and D2 resections for gastric cancer: preliminary results of the MRC randomised controlled surgical trial. 1999 5. Br J Cancer 79:1522-1530. and the extent of lymphadenectomy. Fielding J. REFERENCES 1.Bonenkamp JJ. the resection of adjacent structures such as the spleen and pancreas. et al: Lymph node metastases of gastric cancer. Menotti V. Lancet 347:995-999. which is the distance shown to have the least chance of a pathologically positive margin.The Surgical Cooperative Group. Bufalino R. increasing the risk of unsuspected R1 (margin positive) resections. Sasako M. patients with type II or III tumors would require a gastric resection and patients with type I or II tumors require esophageal resection. Sasako M. making removal of the esophagus unnecessary. based on the classifications defined by Siewart : type III. et al: Randomised comparison of morbidity after D1 and D2 dissection for gastric cancer in 996 Dutch patients. N Engl J Med 340:908-914. Gunven P. van de Velde CJ. Fayers P. General pattern in 1931 patients. tumor epicenter in stomach below the cardia without involvement of the GEJ. Bonfanti G. Gastric adenocarcinoma cells infiltrate beneath normal-appearing mucosa. or type I. 1995 3. and routine removal of the spleen or pancreas is not recommended. The margins of resection are negative.American College of Surgeons SESAP 13 Category 3 Item 44 A 65-year-old man has a total gastrectomy forT2N1M0 gastric adenocarcinoma. This patient should also receive (A) external-beam radiation therapy (B) fluorouracil-based chemotherapy (C) external-beam radiation therapy and fluorouracil-based chemotherapy (D) cisplatinum and external-beam radiation therapy (E) no additional therapy page 195 . Local and regional failure can be expected in 40% to 65% of these patients and nearly always results in 5-year survival of 20% for these patients. Although cisplatin and radiation therapy in combination have achieved some favorable results in patients with gastric cancer. Beart RW. Bufalino R. 1982 2. Ann Surg 196:685-690. Bonfanti G. 1986 3. O'Connell MJ. Adjuvant chemotherapy or radiation therapy alone after resection has not resulted in higher survival rates than operation alone. Adding fluorouracil to radiation therapy improves tumor response over radiation therapy alone. Int J Radiat Oncol Biol Phys 12:661-665. 1982 4. Menotti V. and is currently recommended for all patients after potentially curative resection of gastric cancer. Int J Radiat Oncol Biol Phys 8:1-11.Gunderson LL. External-beam radiation therapy or chemotherapy without operation can achieve palliation in patients with locally advanced gastrointestinal malignancies.Gunderson LL.Macdonald JS. Hundahl SA.American College of Surgeons SESAP 13 Critique of Category 3 Item 44 This patient has undergone operation for a stage III gastric cancer. et al: Adequacy of margins of resection in gastrectomy for cancer. et al: Local control and survival in locally advanced gastrointestinal cancer. A recent prospective trial (Intergroup Trial 0116) demonstrated improved outcomes for patients who receive an R0 (negative margin) surgical resection of gastric cancers followed by postoperative chemotherapy (bolus fluorouracil) and radiation therapy (45 Gy delivered 2 cm beyond all involved nodal basins). Sosin H: Adenocarcinoma of the stomach: areas of failure in a re-operation series (second or symptomatic look) clinicopathologic correlation and implications for adjuvant therapy. Smalley SR. Benedetti J. N Engl J Med 345:725-730. they have not been examined in a randomized controlled fashion and this regimen is not recommended outside of an investigational protocol.Bozzetti F. 2001 page 196 . et al: Chemoradiotherapy after surgery compared with surgery alone for adenocarcinoma of the stomach or gastroesophageal junction. Martin JK. but local control and long-term survival are infrequent. REFERENCES 1. He has no peritoneal signs on examination. During the procedure the patient was noted to have had a poor bowel preparation. The chest x-ray shown is obtained. The most appropriate management now would be (A) exploratory laparotomy (B) gastrografin enema (C) repeat colonoscopy (D) intravenous antibiotics and observation (E) abdominal computed tomography page 197 .American College of Surgeons SESAP 13 Category 3 Item 45 A 65-year-old man presents with progressive abdominal pain 12 hours after undergoing screening colonoscopy and biopsy of a large sessile polyp. and management. This approach often includes enema-examination or abdominal computed tomography with water-based contrast to detect free intraperitoneal leakage. or radiographically. becomes generalized.Farley DR. or is associated with tenderness. Management of colonoscopic perforations is controversial and no prospective data are available. Beart RW Jr: Colonoscopic perforation and bleeding of the colon can be page 198 . the free intraperitoneal air and persistent abdominal pain warrant surgical exploration. 2000 2. diagnosis. Am J Gastroenterol 95:3418-3422. visualizing the intraperitoneal space through the scope. general health of the patient. including fever. Etiology. tachycardia. subcutaneous emphysema. Mayo Clin Proc 72:729-733. Pemberton JH. especially if it intensifies. also point to perforation. adequacy of bowel preparation. If none is detected and the patient’s condition does not deteriorate. as in this case. Wasserberg N. Zietlow SP.032% to 0. eg. Dis Colon Rectum 39:1308-1314.1%. et al: Management of colonoscopic perforations. Rantis PC. Signs associated with peritoneal irritation (guarding and generalized tenderness) should also prompt surgical exploration. 1997 4. This patient presented with progressive pain that persisted several hours after the procedure and was associated with a large amount of intraperitoneal air. Even without signs indicating peritonitis on physical examination. Vernava AM III. No data are available to guide the duration of antibiotic therapy or observation. Longo WE: Colonoscopic perforations.American College of Surgeons SESAP 13 Critique for Category 3 Item 45 In recent large retrospective series. 1996 3.Ker TS. the patient receives intravenous antibiotics and is observed. Bannon MP. Repeat colonoscopy will likely increase perforation size and spill and has no place in this management algorithm. Poor bowel preparation predicts a higher likelihood of fecal spill and sepsis after perforation and also favors surgical treatment. and time of diagnosis relative to time of perforation. Both operative and nonoperative management approaches have been used for these patients and described in retrospective studies of a large number of patients receiving colonoscopy. REFERENCES 1. which has been shown to predict that the perforation will likely not close and heal with nonoperative management.Damore LJ II. Pasha TM. ie. and a deteriorating clinical condition during treatment with antibiotics should prompt surgical exploration. Pain after the completion of a colonoscopy is a warning sign of perforation. the incidence of colonoscopic perforation of colon was 0. The choice between operative and nonoperative management is often based on factors such as mechanism and size of perforation. large amount of free intraperitoneal air. underlying pathologic process. Additional factors supporting surgical exploration in this case include the presence of pathology at the site of biopsy. In selected patients without these findings and circumstances. guarding. a nonoperative strategy has been successfully applied. or rigidity on abdominal examination. a localized perforation may not result in pneumoperitoneum.Anderson ML. Plain films of the abdomen and upright chest x-ray may identify extravasated air confined to the bowel wall. Clinical signs of sepsis. Leighton JA: Endoscopic perforation of the colon: lessons from a 10-year study. Operation is definitely indicated in the presence of a large perforation demonstrated either colonoscopically. or even pneumothorax. Additionally. free intraperitoneal air (as in this case). and leukocytosis. Am Surg 70:922-924.American College of Surgeons SESAP 13 treated safely without surgery. 2004 page 199 . Which of the following statements about squamous cell carcinoma of the anal canal is TRUE? (A) The sensitivity of computed tomographic (CT) imaging and clinical examination for involved lymph nodes is only 50% (B) Tumor size at initial diagnosis is the most reliable predictor of survival after medical or surgical treatment (C) Inguinal lymph node dissection is indicated for patients with clinical lymphadenopathy or positive sentinel lymph node biopsy at initial presentation (D) Combined chemotherapy and radiation therapy is associated with similar local control and improved survival compared with radiation therapy alone (E) Liver and bone are the most common sites of distant metastases for anal cancer page 200 .American College of Surgeons SESAP 13 Category 3 Item 46 A 56-year-old woman has a 2-month history of pain with bowel movements and spotting of blood. Biopsy of an anal canal ulcer identified on anoscopy reveals invasive squamous cell carcinoma. Cancer 92:77-84. Bartolo DC: Management of anal canal cancer. 2001 3. The liver and the lungs are the most common sites of distant metastasis. and immunosuppression. depth of micro invasion. REFERENCES 1. and anus. Morignat E.Sato H. et al: Management of inguinal lymph node metastases in patients with carcinoma of the anal canal: experience in a series of 270 patients treated in Lyon and review of the literature. 2004 2. lymph node involvement. rectum. Combined chemoradiation therapy is the primary form of treatment for squamous cell carcinoma of the anal canal. and distant metastasis. Hartley A.Clark MA. The sensitivity of computed tomographic (CT) imaging and clinical examination for involvement of lymph nodes is only 50%. 5% to 10% of patients will have inguinal lymph node metastasis.American College of Surgeons SESAP 13 Critique for Category 3 Item 46 Squamous cell carcinoma of the anal canal comprises 1% to 5% of all cancers of the colon. HIV infection. Samiei F.Gerard JP. Combined radiation and chemotherapy controls synchronous inguinal lymph node metastasis in 90% of patients. Dis Colon Rectum 48:1301-1315. Chapet O. At presentation. and is preferred over surgical inguinal lymph node dissection as the initial modality for the management of inguinal lymph node metastasis. Lancet Oncol 5:149-157. Combined chemoradiation therapy provides superior local control and improved survival over radiation therapy alone. Lymph node involvement and distant metastasis are the most important prognostic factors. Koh PK. Risk factors for developing anal cancer include human papillomavirus (HPV) infection. The staging of anal cancers depends on tumor size. 2005 page 201 . Geh JI: Cancer of the anal canal. American College of Surgeons SESAP 13 Category 3 Item 47 Which of the following statements about recurrent rectal prolapse is TRUE? (A) Rectopexy with nonabsorbable mesh is more effective than suture rectopexy (B) Recurrence after suture rectopexy is approximately 5% at 5 years (C) The perineal approach for full-thickness rectal prolapse should be reserved for patients who are not candidates for the transabdominal approach (D) Recurrence rates are higher after operation for recurrent rectal prolapse compared with primary rectal prolapse (E) Repeat perineal rectosigmoidectomy may be performed for recurrent prolapse page 202 . Pikarsky AJ. Wexner SD. The best clinical data indicate that sutured rectopexy is equivalent to rectopexy with mesh at preventing recurrent prolapse. 2005 page 203 . Senagore AJ. A previous perineal rectosigmoidectomy does not preclude a subsequent perineal rectosigmoidectomy for recurrent rectal prolapse. 2000 3. Joo JS. Di Giuro G.Bachoo P. however. 2000 2. A previous sigmoid resection. The primary treatment of rectal prolapse is surgical.Raftopoulos Y. Some studies have reported a higher rate of anal incontinence with perineal rectosigmoidectomy. no clear advantage of one procedure over all others has been established. Cochrane Database Syst Rev (2):CD001758. Perineal rectosigmoidectomy has a similar recurrence rate to rectopexy procedures performed through an abdominal approach. Factors associated with the development of rectal prolapse include constipation.American College of Surgeons SESAP 13 Critique for Category 3 Item 47 Rectal prolapse or procidentia is a full-thickness intussusception of the rectum. female sex. Dis Colon Rectum 48:1200-1206. Although the varied surgical approaches have their proponents. Grant A: Surgery for complete rectal prolapse in adults. postmenopausal status. Among the most commonly used surgical options are transabdominal rectopexy with or without sigmoid resection. more distal anastomosis at risk. Brazzelli M. placing the new. and perineal rectosigmoidectomy (Altemeier procedure). Operative outcomes are similar for primary and recurrent prolapse. yet on average 25% of patients will experience a recurrence of the rectal prolapse within 5 years. Reported recurrence rates for surgical management of rectal prolapse vary. may complicate a subsequent perineal resection because the initial operation may have disrupted collateral vessels. REFERENCES 1. Weiss EG. Bergamaschi R. and neurologic disorders. multiparity. et al: Recurrence rates after abdominal surgery for complete rectal prolapse: a multicenter pooled analysis of 643 individual patient data. et al: Recurrent rectal prolapse: what is the next good option? Dis Colon Rectum 43:1273-1276. American College of Surgeons SESAP 13 Category 3 Item 48 Which of the following statements about the treatment of hemorrhoids is TRUE? (A) Surgical hemorrhoidectomy is appropriate first-line therapy for grade III or grade IV hemorrhoids (B) Rubber band ligation. sclerotherapy. and infrared coagulation are equivalent options for in-office treatment of hemorrhoids (C) Similar postprocedure pain occurs after stapled hemorrhoidectomy and rubberband ligation (D) Excision or incision and clot evacuation are equally effective for treatment of thrombosed external hemorrhoids (E) Approximately 20% of patients will experience pain due to thrombosed external hemorrhoids after rubber band ligation page 204 . This is superior to simple incision and clot evacuation because the lesion typically consists of multiple small intravascular thromboses rather than a single hematoma. 2003 page 205 . Complication rates with rubber band ligation are low and thrombosis of external hemorrhoids secondary to rubber band ligation is uncommon. Dis Colon Rectum 46:291-297. 2005 3. Gregorcyk S.Cataldo P. Ellis CN.Peng BC. et al: Complications of rubber band ligation of symptomatic internal hemorrhoids. Jayne DG. For patients with thrombosed external hemorrhoids. Dis Colon Rectum 36:287-290. Dis Colon Rectum 48:189-194. REFERENCES 1. Surgical hemorrhoidectomy is appropriate for primary treatment of grade IV and large grade III hemorrhoids. Ho YH: Randomized trial of rubber band ligation vs stapled hemorrhoidectomy for prolapsed piles. those who have been symptomatic for less than 48 to 72 hours can be treated with surgical excision of the thrombosed hemorrhoid.Bat L. Hyman N. as shown in the table.American College of Surgeons SESAP 13 Critique for Category 3 Item 48 Five percent of the population complains of hemorrhoids. Rubber band ligation is the most effective office procedure for symptomatic internal hemorrhoids. When compared with rubber band ligation. Melzer E. and has a lower rate of recurrence than sclerotherapy or infrared coagulation. Hemorrhoids above the dentate line are designated as internal hemorrhoids and classified by grade. et al: Practice parameters for the management of hemorrhoids (revised). stapled hemorrhoidectomy resulted in more postoperative pain for grade III internal hemorrhoids. Dreznick Z. Koler M. 1993 2. Primary surgical hemorrhoidectomy is also appropriate for combined internal and external hemorrhoids with significant prolapse. and stool softeners is the best initial management (B) Topical nitrates are only marginally effective at healing anal fissures (C) Topical calcium channel blockers are more effective than oral calcium channel blockers at healing anal fissures (D) Recurrence is common after initial successful treatment with botulinum toxin (E) Lateral internal sphincterotomy should be recommended only after topical nitrates. calcium channel blockers. or botulinum therapy has failed page 206 .American College of Surgeons SESAP 13 Category 3 Item 49 Which of the following statements about anal fissures is NOT true? (A) Nonoperative treatment with increased fluid intake. fiber ingestion. Diagnosis is made by history and confirmed by examination of the anal canal. et al: Practice parameters for the management of anal fissures (revised). Topical nitrates such as 0. controlled trial. In most cases the fissure is caused by an episode of constipation with passage of a hard stool. Dis Colon Rectum 40:1318-1320.Oettle GJ: Glyceryl trinitrate vs sphincterotomy for treatment of chronic fissure-in-ano: a randomized. Rakinic J. The availability of topical calcium channel blockers may be limited. Neal KR.Orsay C.American College of Surgeons SESAP 13 Critique for Category 3 Item 49 An anal fissure is a longitudinal tear in the squamous lining of the anal canal between the anal verge and the dentate line. and increased fluid intake has not been successful. surgical sphincterotomy remains an accepted alternative to pharmacologic treatment when conservative treatment with stool softeners. The principal side effect of topical nitrates is headache. fiber supplements. Operation may be appropriately offered to such patients without a trial of pharmacologic therapy. and have fewer side effects than nitrates. and increased fluid intake. Patients may also experience some degree of bleeding. 2004 page 207 . Topical calcium channel blockers can also promote healing. Hyman N. fiber supplements. REFERENCES 1. which can limit its effectiveness. Injection of botulinum toxin into the internal sphincter allows for healing of up to 80% of chronic anal fissures. however. Scholefield JH: A randomized trial of oral vs topical diltiazem for chronic anal fissures.Jonas M. 1997 3. Dis Colon Rectum 47:2003-2007. but are marginally associated with a healing rate superior to placebo.2% nitroglycerin ointment can help to lessen the pain of anal fissures. Due to the relatively low healing rates and high recurrence rates with topical treatments. but recurrence is common. and oral calcium channel blockers are not as effective as topical application. 2001 2. Perry WB. Because most will heal spontaneously. Dis Colon Rectum 44:1074-1078. Abercrombie JF. Fissures typically result in severe anal pain that is often excruciating during defecation. acute anal fissures are best treated with conservative measures consisting of stool softeners. American College of Surgeons SESAP 13 Category 3 Item 50 A 56-year-old woman has rectal cancer found on colonoscopy performed for evaluation of rectal bleeding. Which of the following is the primary determinant of the feasibility of avoiding a permanent colostomy? (A) Distance of the tumor from the proximal anal canal (B) Presence of enlarged lymph nodes on computed tomographic (CT) scan (C) Distance of tumor from dentate line (D) Distance of tumor from anal verge (E) Extension of tumor into posterior vaginal wall or bladder page 208 . The patient’s primary concern is whether operation will result in a permanent colostomy. ed 4. Philadelphia. 2005 2.Soderman C. then an abdominal perineal resection is required. For an appropriate oncologic resection while still preserving the sphincters (to avoid a permanent stoma). The distance of the tumor from the top of the sphincter mechanism or the proximal anal canal is critical in detecting whether an adequate cancer resection can be performed without excising the anal sphincters. REFERENCES 1. and appropriate lymph node dissection. and thus into the sphincter mechanisms. 2001 4. pp 1050-1065 3. Surg Laparosc Endosc Percutan Tech 11:97-102. 2001 page 209 . Turnage RH: Acute gastrointestinal hemorrhage. If the distal resection margin extends beyond the superior margin of the anal canal. in Mulholland MW. Tumino E. Bertoni M. Most surgeons would agree that a 2-cm distal margin with wide lateral dissection is sufficient.American College of Surgeons SESAP 13 Critique for Category 3 Item 50 The objectives for surgical management of rectal cancer are the achievement of adequate surgical margins. J Gastroenterol 40:256-259. Heldmann M. Lippincott Williams & Wilkins. Uribe A: Enteroscopy as a tool for diagnosing gastrointestinal bleeding requiring blood transfusion. The length of the anal canal and the relative position of the dentate line vary from person to person and are less reliable landmarks. J Vasc Interv Radiol 12:1273-1277. Total mesorectal excision is the current standard to achieve adequate radial margins and an optimal lymph node dissection. The appropriate length of the distal margin of resection is a subject of controversy.Ryan JM. Lillemoe KD. some clinical evidence suggests a 1-cm margin may be equally effective for attaining local control and improving survival. the distal resection margin cannot encompass the anal sphincter mechanism. Maier RV. heparin. et al (eds): Greenfield's Surgery: Scientific Principles and Practice. Key SM.Bresci G. Dumbleton SA. Smith TP: Nonlocalized lower gastrointestinal bleeding: provocative bleeding studies with intraarterial tPA. 2006. Local tumor involvement of the posterior vaginal wall or bladder involvement does not necessarily eliminate the ability to perform a coloanal anastomosis. however.Kim LT. and tolazoline. Parisi G. et al: The role of video capsule endoscopy for evaluating obscure gastrointestinal bleeding: usefulness of early use. Presence of lymph node metastasis should not affect the decision to create a permanent stoma. Doherty GM. The optimal method for assessing whether preoperative combined modality therapy is indicated in this patient would be (A) endorectal ultrasound (EUS) (B) physical examination (C) computed tomography (CT) (D) positron-emission tomography (PET) (E) endoscopic biopsy page 210 .American College of Surgeons SESAP 13 Category 3 Item 51 A 56-year-old woman has rectal cancer found on colonoscopy performed for evaluation of rectal bleeding. J Gastrointest Surg 8:139-149. computed tomography (CT). and endoscopic biopsy cannot accurately determine the depth of penetration of the primary tumor. 2004 page 211 .American College of Surgeons SESAP 13 Critique of Category 3 Item 51 Multiple randomized controlled studies have demonstrated the value of adjuvant chemoradiation therapy in improving both local control and long-term survival for patients with stage II and stage III adenocarcinomas of the rectum. Clin Colorectal Cancer 4:124-132. 2004 3. Beets GL. Although EUS is the preferred modality for determining the T stage of rectal adenocarcinomas. Nelemans PJ. Engelen SM. positron-emission tomography (PET).Tait D: Advances in chemoradiation therapy in rectal cancer: the impact of imaging. Wong WD: Endorectal ultrasound in the preoperative evaluation of rectal cancer. Fazio VW: Management of rectal cancer. endorectal ultrasound (EUS) is 90% accurate in determining the T stage of rectal adenocarcinomas. 2005 4. Br J Radiol 78 (Spec No 2):S131-S137. Physical examination. REFERENCES 1.Lahaye MJ. et al: Imaging for predicting the risk factors—the circumferential resection margin and nodal disease—of local recurrence in rectal cancer: a metaanalysis. magnetic resonance imaging (MRI) is an acceptable alternative for determining depth of penetration of the primary tumor. Semin Ultrasound CT MR 26:259-268. making preoperative staging critical. In experienced hands. Preoperative combined chemoradiation is preferred to postoperative.Schaffzin DM. Patients with stage I rectal cancer do not require adjuvant therapy.Wu JS. Those patients with tumor invasion through the muscular propria as determined by preoperative EUS would be eligible for neoadjuvant therapy. 2005 2. American College of Surgeons SESAP 13 Category 3 Item 52 Which of the following statements about gastric polyps is TRUE? (A) Hyperplastic polyps are not associated with an increased risk of gastric cancer (B) Hyperplastic polyps most frequently occur in the fundus (C) Gastric adenomas are typically pedunculated (D) Gastric adenomas are associated with an increased risk for cancer throughout the gastric mucosa (E) Tubular adenomas without evidence of dysplasia can be managed with surveillance endoscopies page 212 . the risk for developing an invasive adenocarcinoma within a gastric tubuloadenoma is sufficiently high that all should be removed either endoscopically or surgically. 2004 2.Ljubicic N. location. pedunculated tubular adenomas are less common. Although gastric hyperplastic polyps are not neoplastic. Even so. also known as regenerative polyps. Banic M. and age and sex distribution. with tubuloadenoma being less likely to harbor a cancer than a tubulovillous or a villous polyp. 2000 page 213 .American College of Surgeons SESAP 13 Critique for Category 3 Item 52 Hyperplastic gastric polyps. Virchows Arch 437:581-590. and polypectomy is recommended. 2002 3. Coll Antropol 26:55-60. Rev Gastroenterol Peru 24:50-74.Oberhuber G. dysplastic changes and invasive cancers can develop within the polyp in rare cases. Multiple hyperplastic polyps are associated with an increased risk for adenocarcinoma. Hyperplastic polyps develop in 50% of cases of Helicobacter pylori infection with atrophic gastritis. Gastric adenomas are typically sessile lesions. Roic G. Large. REFERENCES 1.Espejo Romero LH. Benign gastric adenomas are a marker for an increased risk of gastric cancer throughout the remainder of the stomach. Kujundzic M. Risk for adenocarcinoma within an adenomatous polyp is dependent on the size and histology of the polyp. are most commonly found in the antrum. Navarrete Siancas J: [Gastric epithelial polyps (part two)]. et al: Benign epithelial gastric polyps—frequency. Stolte M: Gastric polyps: an update of their pathology and biological significance. American College of Surgeons SESAP 13 Category 3 Item 53 Which of the following statements about infliximab is TRUE? (A) Beneficial for treatment of Crohn’s disease but not ulcerative colitis (B) Risk of antibody development reduced by pretreatment with mercaptopurine (6-MP) (C) Helps resolve Crohn’s-related perirectal abscesses (D) Contraindicated in patients who have had an antibody reaction to infliximab (E) Initial treatment doses of infliximab should be lowered to reduce the risk of antibody formation page 214 . 2005 3. Feagan BG. and associated infusion reactions. et al: Therapeutic efficacy of multiple intravenous infusions of anti-tumor necrosis factor alpha monoclonal antibody combined with low-dose weekly methotrexate in rheumatoid arthritis. 1998 4. Paradoxically. 2003 2. N Engl J Med 348:601-608. Van Assche G. Noman M. and most are not mediated by anti-infliximab antibodies. Smolen JS. et al: Influence of immunogenicity on the long-term efficacy of infliximab in Crohn’s disease. Severe infusion reactions such as IGE-mediated reactions with bronchospasm and hypotension are rare.Baert F. 2005 page 215 .Cheifetz A. The use of infliximab in the treatment of fulminant ulcerative colitis is currently under investigation. About 20% of patients treated with infliximab will develop antibodies to the medication. et al: Infliximab for induction and maintenance therapy for ulcerative colitis.Rutgeerts P. Breedveld FC. Arthritis Rheum 41:1552-1563. Kalden JR.Maini RN. Pretreatment with mercaptopurine (6-MP) decreases the risk for development of anti-infliximab antibodies. Mt Sinai J Med 72:250-256. Mayer L: Monoclonal antibodies. REFERENCES 1. N Engl J Med 353:2462-2476. Reinisch W.American College of Surgeons SESAP 13 Critique for Category 3 Item 53 Infliximab is a chimeric monoclonal IgG antibody against tumor necrosis factor that has been FDA approved for use in moderate to severe Crohn’s disease. Reactions to infliximab infusion are common. The development of antibodies does not prevent continued use of the medication. immunogenicity. Sandborn WJ. but may diminish the effectiveness of the treatment. Vermeire S. A recent multicenter trial has shown that it is also effective at inducing and maintaining remission of moderate to severe ulcerative colitis. anti-infliximab antibodies are most likely to occur with the administration of smaller rather than larger doses. Infliximab is contraindicated in the presence of sepsis and should not be used in patients with undrained intraabdominal or perirectal abscesses. Which of the following statements is TRUE? (A) Angiographic embolization is associated with an 80% success rate and infrequent ischemic complications (B) The use of heparin or urokinase may increase the sensitivity of angiography with minimal increase in complication rates (C) Vasopressin infusion has similar efficacy and fewer ischemic complications compared with angiographic embolization (D) Tagged RBC scan should be performed routinely prior to angiography (E) Vascular ectasias account for 30% of massive lower gastrointestinal hemorrhages page 216 . Colonoscopy demonstrates continued brisk bleeding. Gastric lavage returns clear bilious fluid. but no definite bleeding site is identified.American College of Surgeons SESAP 13 Category 3 Item 54 A previously healthy 58-year-old man with lower gastrointestinal hemorrhage is hemodynamically stable. et al: Evaluation of transarterial embolization for lower gastrointestinal bleeding. Moore BA.Vernava AM III.Luchtefeld MA. Shetty PC. Sharma RP. Semin Colon Rectal Surg 8:164-171. 2000 6. et al: Selective arterial embolization for the control of lower gastrointestinal bleeding. Burke TH.American College of Surgeons SESAP 13 Critique for Category 3 Item 54 Angiographic embolization is a relatively new modality used for the treatment of severe colonic hemorrhage. Kerlan RK. When patients such as the one described in this clinical scenario are known to have active brisk bleeding. Dis Colon Rectum 43:532-534. 1997 7.Bandi R.Horiguchi J. et al: Morphologic and histopathologic changes in the bowel after super-selective transcatheter embolization for focal lower gastrointestinal hemorrhage. J Vasc Interv Radiol 12:1399-1405. Dis Colon Rectum 40:846-858. Heparin or urokinase can increase the sensitivity of angiography for identifying the site of bleeding. Johnson FE: Lower gastrointestinal bleeding. Ischemic complications after angiographic embolization are rare when superselective techniques are used. Ahl KL. Angiographic embolization is as effective as vasopressin infusion in controlling lower gastrointestinal hemorrhage and avoids the vasoconstrictive side effects of vasopressin that can occur in 10% of cases. 1997 3. Am J Surg 174:24-28. Dig Dis Sci 32:248-253. but is associated with a 20% complication rate and should be used only in highly selected patients.Molgaard CP: Mesenteric angiography for the diagnosis and treatment of lower gastrointestinal bleeding. 1997 page 217 . Tagged RBC scans can be useful in identifying patients who are likely to be actively bleeding and thus would warrant angiography. REFERENCES 1. Fukuda H. Benner KG. Longo WE. 2001 2. Fedeson B. Nakashige A.Gordon RL. Kozak BE: Aggressive angiographic diagnosis in acute lower gastrointestinal hemorrhage. Szomstein M. their incidence as a cause of massive lower gastrointestinal hemorrhage is 1% to 3%. then angiography should be performed without the bleeding scan. 2003 4. Rosch J. Although arteriovenous malformations or vascular ectasias have been extensively reported.Koval G. but lack adequate accuracy in locating the site of bleeding. The most common causes of massive lower gastrointestinal bleed are diverticular disease and inflammatory bowel disease. Senagore AJ. Multiple reports indicate an 80% success rate of stopping the hemorrhage with this procedure. Acta Radiol 44:334-339. Wilson MW. Naito A. 1987 5. et al: Superselective arterial embolization for the treatment of lower gastrointestinal hemorrhage. Colonoscopy and small bowel follow-through were reported as normal. The abdominal plain film shown was obtained 24 hours previously. she had an extensive work-up at a referring institution. Three weeks ago.American College of Surgeons SESAP 13 Category 3 Item 55 A 52-year-old woman with Crohn’s disease and a 4-month history of right lower quadrant discomfort is seeking a second opinion. The best management now would be (A) elective laparotomy if her condition does not improve (B) urgent exploratory laparotomy (C) capsule endoscopy (D) computed tomographic (CT) scan of the abdomen and pelvis (E) diagnostic laparoscopy page 218 . then surgical removal is indicated. the retained capsule is asymptomatic and additional time can be given to allow for spontaneous passage of the device. or if the capsule does not pass within a reasonable amount of time. batteries. Am Surg 71:455-458. The images are transmitted to a data recorder worn by the patient.Chang PK.Maieron A. a light source. The standard device used with capsule endoscopy measures 11 mm by 16 mm and weighs about 4 g. Hubner D. Two images per second are taken with 57. 2004 page 219 . If the patient develops obstructive symptoms. Deutsch C. Blaha B. Retention of the capsule occurs in 4% of studies performed. The appearance is consistent with a retained device after capsule endoscopy.000 frames for the standard 8-hour study. In cases of known Crohn’s disease.American College of Surgeons SESAP 13 Critique for Category 3 Item 55 The plain film shows a metallic foreign body in the right hemi abdomen. Endoscopy 36:864-868. REFERENCES 1. and an antenna. De Villiers WJ. a transmitter. to diagnose Crohn’s disease. In most cases. and to identify suspected tumors. Boulanger BR: A new complication from a new technology: what a general surgeon should know about wireless capsule endoscopy. a small bowel follow-through should be performed to exclude strictures that may impede the passage of the capsule. 2005 2. Capsule endoscopy is indicated as a means of detecting the source of obscure gastrointestinal bleeding. et al: Multicenter retrospective evaluation of capsule endoscopy in clinical routine. Holt EG. Within a plastic housing are a digital video camera. The abdominal discomfort is worse if she lies supine within an hour or so after eating. Her stool is negative for occult blood. She has lost 25 lb in the past 6 months. Upper gastrointestinal endoscopy is normal. She frequently vomits partially digested food and bilious material. The symptoms have progressed to the point that the discomfort occurs after every meal.American College of Surgeons SESAP 13 Category 3 Item 56 A 62-year-old woman has an 8-month history of early satiety with severe abdominal fullness after eating. On physical examination she is small and frail. The computed tomographic (CT) scan shown is obtained Surgical therapy for this condition will most likely include (A) gastrojejunostomy (B) duodenojejunostomy (C) choledochoduodenostomy (D) pancreaticoduodenectomy (E) combined biliary-duodenal bypass page 220 . Bender GN. Up to 90% of patients who have this procedure are improved. Duodenal compression may have various causes. Heitkamp DE. or direct trauma to the SMA. reducing the aortomesenteric angle to < 25 degrees (normal. Wilkie’s series of 75 patients is the largest reported and the syndrome has been given his eponym. Lappas JC. Contrast upper gastrointestinal imaging suggests the diagnosis by the rather abrupt cut off of duodenal emptying at the third portion. Endoscopic ultrasound (EUS) has some proponents. and duodenojejunostomy. The diagnosis can be a challenge and some patients have been labeled with psychiatric eating disorders. et al: Multidetector-row helical CT enteroclysis. An operative approach should focus on bypassing the duodenal obstruction. Howard TJ. hyperextension of the spine (as occurred in children in spica casts). 2003 6. Weiss W. et al: Current concepts in imaging of small bowel obstruction.Ahmed AR. Int J Eat Disord 21:103-114. the former usually associated with an acute illness with weight loss. Radiol Clin North Am 41:263-283. Kelvin FM. Females predominate 2:1.Adson DE. Physical examination frequently shows only an asthenic habitus and a paucity of specific abdominal findings. 2003 page 221 . Most patients report an exacerbation of symptoms in the supine position and some relief in the knee-chest position. Computed tomographic (CT) angiography allows direct measurement of the aortomesenteric angle and the aortomesenteric distance (normal 10 to 20 mm versus 2 to 8 mm in SMA syndrome). The syndrome occurs in both acute and chronic forms. et al: Superior mesenteric artery syndrome: an uncommon cause of intestinal obstruction. 1997 3. 2000 4. Dunn J. loss of innervation of the abdominal wall or of the abdominal wall itself (large ventral hernia). Trenkner SW: The superior mesenteric artery syndrome and acute gastric dilatation in eating disorders: a report of two cases and a review of the literature. et al: Superior mesenteric artery syndrome: diagnosis and treatment from the gastroenterologist’s view. No additional information would be provided by enteroclysis and there is no indication for colonoscopy in this case. vomiting of partially digested food. Taylor I: Superior mesenteric artery syndrome. South Med J 93:606-608. Hannig C. 38 to 60 degrees). It was commonly seen in children placed is spica casts for orthopaedic injuries and was known as the spica cast syndrome.Baltazar U. REFERENCES 1. Heitkamp DE. Radiol Clin North Am 41:249-262. Postgrad Med J 73:776-778. Other causes include a congenitally abnormal aortomesenteric angle.Maglinte DD. Floresguerra C. either directly or via Roux-en-Y. 2002 5.American College of Surgeons SESAP 13 Critique of Category 3 Item 56 The patient described fits the classic picture of superior mesenteric artery (SMA) syndrome: postprandial epigastric discomfort. Allescher HD. and weight loss.Lippl F. is the indicated operative procedure. but there is no evidence for biliary tract disease and ERCP in not indicated. Mitchell JE.Maglinte DD. The SMA compresses the duodenum against the spine because of a loss of the normal fat pad (acute weight loss). J Gastroenterol 37:640-643. It is preferred by many as the diagnostic procedure with the highest yield (see image A and image B). Schmidt L. 1997 2.. Which of the following statements about his care is TRUE? (A) Surgical treatment of gastric cancer in patients over 80 is not appropriate (B) Stage I disease has a 90% 5-year survival (C) Total gastrectomy (including spleen and omentum) and extended lymphadenectomy is the treatment of choice (D) Palliative resection is contraindicated (E) Neoadjuvant chemoradiation will improve his resectability page 222 .American College of Surgeons SESAP 13 Category 3 Item 57 An 81-year-old man presents with epigastric pain. Endoscopy reveals an antral mass. but the anemia worsens. and biopsy confirms gastric carcinoma. anemia. Treatment with proton pump inhibitors relieves the pain. and progressive food intolerance. et al: Total or subtotal gastrectomy for gastric carcinoma? A study of quality of life. subtotal gastrectomy also offers the best palliation. and for those confined to the submucosa. 2004 4. J Am Coll Surg 198:1013-1021. REFERENCES 1. However. by decreasing the risk of duodenal obstruction and/or bleeding. Tumor-free margins are a minimum requirement for curative resection. Brennan MF: Defining palliative surgery in patients receiving noncurative resections for gastric cancer. radical subtotal (50% to 85%) gastrectomy (including gastrocolic omentum. The potential value of chemoradiotherapy is under investigation. For a distal lesion that does not directly invade the hilum of the spleen.American College of Surgeons SESAP 13 Critique for Category 3 Item 57 Gastric cancer. Patients are often elderly and debilitated. Sue-Ling H. World J Surg 24:465-472. survival. Cancer Treat Rev 26:243-255. McGraw-Hill. and mesentery) is the operation of choice. a biologically aggressive malignancy. 2005. Fiveyear survival for in situ cancers approaches 99%. Operative mortality rates are directly related to the extent of gastrectomy. Although investigations continue. operation may be curative.Miner TJ. 1998 2. Karpeh MS. even in the elderly. 70% long-term survival rates are reported after radical operation.Davies J. Andersen DK. ed 8. associated lymph nodes. 2000 page 223 . Billiar TR. New York. Dunn DL. et al: Surgical mortality. pp 933-996 3.Wu CW. 2000 5. Neoadjuvant chemoradiotherapy does not improve resectability. but this should not preclude adequate treatment. is uncommon in the United States but is the leading cause of cancer deaths in Japan. 93%. curative treatment requires operation.Roukos DH: Current status and future perspectives in gastric cancer management. but no improvement in 5year survival has been shown. Johnston D. Even with lymph node involvement. Jaques DP. Lo SS. and quality of life after resection for gastric cancer in the elderly. World J Surg 22:1048-1055. in Brunicardi FC.Dempsey DT: Stomach. Hsieh MC. Young S. Shen KH. et al (eds): Schwartz’s Principles of Surgery. For the occasional stage I tumor. American College of Surgeons SESAP 13 Category 3 Item 58 Dieulafoy’s lesion is (A) seen only in foregut-derived viscera (B) an arterio-venous malformation implicated in gastrointestinal bleeding (C) preferably treated by surgical resection (D) usually accompanied by Helicobacter pylori infection (E) painless page 224 . Katsinelos P. but the lesion is painless. ed 17. colon. and rectum. Williams LF: Hemorrhage from a Dieulafoy type ulcer of the colon: a new cause of lower gastrointestinal bleeding. nor of Helicobacter pylori infection. 2004. Hisanaga Y. histologic examination typically shows no sign of inflammation. rubber band ligation. Lamb R. Endoscopy 37:449-453. Kumada T. Baillie J: Dieulafoy lesions: a review of 6 years of experience at a tertiary referral center. Labitzke HG. Toyoda H. Successful outcomes have been achieved with a combination of endoscopic and angiographic methods—cautery. small intestine. Originally described in 1896.Bech-Knudsen F. clips. REFERENCES 1. In recent years. Toftgaard C: Exulceratio simplex dieulafoy. and embolization—and surgical resection is now reserved for patients who do not respond to less invasive methods. Massive hemorrhage may occur. 1993 2. Saunders. et al: Dieulafoy-like lesion of the colon presenting with massive lower gastrointestinal bleeding. Dieulafoy’s lesion was for a long period believed to be found exclusively in the stomach. Philadelphia. 2001 7. 1984 3. Am J Gastroenterol 79:195-200. Paroutoglou G. 1988 6. Surg Gynecol Obstet 176:139-143. Robinson EK: Stomach. et al: Endoscopic management and follow up of Dieulafoy lesion in the upper gastrointestinal tract. Am Surg 54:121-124. 2004 4.Schmulewitz N. pp 1265-1322 5. in Townsend CM Jr (ed): Sabiston Textbook of Surgery: The Biological Basis of Modern Surgical Practice. Galanis I. 2005 page 225 .Mercer DW. Surg Endosc 18:346. Pilipilidis I.Richards WO. it has been found in the esophagus. Grove-Mahoney D. Am J Gastroenterol 96:1688-1694. sclerotherapy.Juler GL.Sone Y. Allen R: The pathogenesis of Dieulafoy’s gastric erosion. Because the ulcerative process is mechanical. however.American College of Surgeons SESAP 13 Critique of Category 3 Item 58 Dieulafoy’s lesion is a combination of an abnormally tortuous submucosal artery and ulceration of the overlying mucosa. American College of Surgeons SESAP 13 Category 3 Item 59 Controlled clinical trials comparing laparoscopic and open colectomy demonstrate that (A) time to recovery is not significantly different (B) incisional metastases are more common with laparoscopic resection (C) complications occur with equal frequency (D) significantly fewer lymph nodes are retrieved with laparoscopic resections (E) overall survival is more favorable with open resections page 226 . Trials comparing laparoscopic and open resections were completed and reported in 2004. Delaney CP.Senagore AJ.Curet MJ: Port site metastases. 2004 2. J Am Coll Surg 199:675-679. Brady KM. complications occurred with equal frequency. Fazio VW: Standardized approach to laparoscopic right colectomy: outcomes in 70 consecutive cases. Gholghesaei M. and overall survival was similar. and operative times are significantly longer. Bonjer HJ. but age is not an absolute contraindication. Liver metastases are not a contraindication to laparoscopic liver resection. Obesity and previous abdominal operation increase difficulty. Concern was also expressed regarding the completeness of oncologic resection possible with the techniques and surgical skills of that era. They indicated that postperative recovery is significantly faster in the laparoscopic group. 1996 4. Long operating times associated with laparoscopy may be inadvisable in the very elderly and/or infirm. as surgeons were actively expanding the envelope for laparoscopic surgery.Veldkamp R. Fetter JE. operating times. the median number of nodes retrieved was 12.Clinical Outcomes of Surgical Therapy Study Group: A comparison of laparoscopically assisted and open colectomy for colon cancer. Rohde DC. bulky tumor and locally advanced disease was the most common indication for conversion to open operation. In a large prospective European study. disturbing reports of problems with laparoscopic resection of colonic malignancies began to surface. in each group. et al: Laparoscopic resection of colon cancer: consensus of the European Association of Endoscopic Surgery (EAES. 2004 page 227 .) Surg Endosc 18:11631185. J Clin Oncol 14:1950-1956. A national moratorium was advised until prospective trials could demonstrate safety and efficacy. Laparoscopic colon surgery is technically demanding. as compared with an historical rate of only 1% in conventional laparotomy incisions. et al: Port site recurrences after laparoscopic and thoracoscopic procedures in malignancy. 2004 3. Of particular concern were case reports of port site metastases in up to 20% of cases. REFERENCES 1. 2004 5. N Engl J Med 350:2050-2059. recurrence rates in surgical wounds were less than 1% regardless of approach. Am J Surg 187:705-712. Meijer DW. Swartz SE.Johnstone PA. and conversion to open resection.American College of Surgeons SESAP 13 Critique for Category 3 Item 59 In the mid-1990s. American College of Surgeons SESAP 13 Category 3 Item 60 Endoscopic therapy for treatment of Barrett’s esophagus with low-grade dysplasia includes all of following EXCEPT (A) endoscopic plication (B) multipolar electrocoagulation (C) endoscopic mucosal resection (D) photodynamic therapy (E) argon beam coagulation page 228 . For macroscopically visible lesions. transfers electrical energy by means of the ionized gas argon to achieve very high temperatures and desiccation. 2005 page 229 . Gut 54:875-884. Treatment options for BE include medical therapy. Strictures are associated with this technique in 1% of cases. Patients with BE have a 0. Endoscopic plication is currently being used for treatment of gastroesophageal reflux disease (GERD). Possible complications include strictures (3% to 17%) and perforations (2% to 9%). Proton pump inhibitors and Nissen fundoplication are used for treatment of Barrett’s without dysplasia. not Barrett’s. 2005 2. The choice of technique continues to be controversial and randomized trials are rare. Multipolar electrocoagulation (MPEC) relies on current passing between electrodes and achieves a depth of damage of 1. but strictures can occur in as many as 36% of cases.7 to 4. endoscopic resection as opposed to ablation allows analysis of the actual pathologic specimen. Current guidelines recommend esophagectomy in medically fit patients with high-grade dysplasia. Once dysplasia or cancer is identified. Photodynamic therapy (PDT) uses systemic administration of 5-aminolevulinic acid (5-ALA). complete resection of areas not visible or those that are multifocal and long is often difficult. Sampliner RE: Randomized trials in the treatment of Barrett’s esophagus. APC’s depth of damage is 1 to 3 mm. Cryoablation and YAG lasers have also been used as ablative techniques. endoscopic ablative techniques. The depth of damage is 2 to 6 mm. Laser light of the appropriate wavelength exposed to the photosensitized cells causes a cytotoxic reaction mediated by oxygen. endoscopic ablation/resection or esophagectomy is the primary treatment modality. REFERENCES 1. a photosensitizing agent that acts selectively on rapidly proliferating or malignant cells. Perforations and deaths have resulted. Perforations are rare.Barr H.5% per year risk of developing adenocarcinoma of the esophagus.American College of Surgeons SESAP 13 Critique for Category 3 Item 60 Barrett’s esophagus (BE) is a premalignant condition in which normal squamous epithelium in the distal esophagus is replaced by intestinal metaplasia. Endoscopic mucosal resection is performed by piecemeal resection of Barrett’s segments under direct visualization. and endoscopic mucosal resection. reserving endoscopic treatments for poor-risk patients and for those involved in clinical trials. Stone N.Faybush EM. Dis Esophagus 18:291-297. operation. PDT in combination with omeprazole has been statistically shown to decrease the progression of cancer from 28% to 13% in patients with high-grade dysplasia.8 mm. Rembacken B: Endoscopic therapy for Barrett’s oesophagus. However. another endoscopic ablation technique. Argon beam plasma coagulation (APC). randomized trials comparing laparoscopic versus open colectomy for cancer suggest all of the following are true EXCEPT (A) no statistical difference in overall and 5-year survival (B) no statistical difference in intraoperative complications (C) similar parenteral narcotics utilization (D) similar port site/incisional recurrence rates (E) no difference in 30-day mortality page 230 .American College of Surgeons SESAP 13 Category 3 Item 61 Prospective. randomized multicenter trial comparing laparoscopic colectomy with open colectomy for treatment of cancer was published in 2004 (Clinical Outcomes of Surgical Therapy Study Group). Surg Endosc 19:643-649. The overall complication rate and 30-day mortality were similar for both techniques. with similar overall survival and disease-free survival as open colectomies. Recurrences at surgical wounds were rare.Clinical Outcomes of Surgical Therapy Study Group: A comparison of laparoscopically assisted and open colectomy for colon cancer. N Engl J Med 350:2050-2059. Laparoscopic colectomy also resulted in a shorter median hospital stay (5 days versus 6 days) and decreased parenteral and oral analgesic utilization. 2005 page 231 . but was not significant. REFERENCES 1. 2004 2. Salky BP: Laparoscopic colectomy for colon adenocarcinoma: an 11-year retrospective review with 5-year survival rates. but similar between the two groups. The first prospective. as was the overall survival (86% and 85%. debunking the idea of trocar site recurrences.American College of Surgeons SESAP 13 Critique for Category 3 Item 61 Laparoscopic colectomy is now an acceptable treatment for colon cancer in experienced hands. respectively). Retrospective analyses have demonstrated its safety. Three-year recurrence rates were similar for both techniques.Jacob BP. A trend toward higher intraoperative complications such as bleeding and bowel injury was noted with the laparoscopic technique. American College of Surgeons SESAP 13 Category 3 Item 62 Which of the following statements about nonoperative treatment of clinically diagnosed appendiceal phlegmon is TRUE? (A) Interval appendectomy should be scheduled within 6 weeks of discharge (B) Interval appendectomy is probably not necessary (C) Recurrence of acute appendicitis is over 50% (D) Recurrence and complication rate is higher if the patient initially had peritonitis or abscess formation (E) Length of hospital stay is longer for patients with recurrence than it is for interval appendectomy page 232 . Further. Liu IL. Of these 1012 patients. Applebaum H. REFERENCES 1. length of hospital stay for interval appendectomy was not only longer than it was for a recurrence. but also longer than it was for patients who were treated by emergency appendectomy on the first admission. 2005 page 233 .Friedell ML. Perez-Izquierdo M: Is there a role for interval appendectomy in the management of acute appendicitis? Am Surg 66:1158-1162. With a median follow-up of 4 years. Arch Surg 140:897-901. 3% (1012 patients) did not have an appendectomy. 85% (864) did not have an interval appendectomy.Kaminski A. 2000 2. et al: Routine interval appendectomy is not justified after initial nonoperative treatment of acute appendicitis. only 5% of these 864 patients had a recurrence.American College of Surgeons SESAP 13 Critique for Category 3 Item 62 In a study of nearly 33. and one third of this group were successfully managed nonoperatively and none had a second recurrence.000 patients with appendicitis. Patients in this large study who presented initially with abscess or peritonitis did not have any higher incidence of recurrence. Lee SL. American College of Surgeons SESAP 13 Category 3 Item 63 Seventy percent of anorectal fistulas are (A) intersphincteric (B) transphincteric (C) suprasphinteric (D) extrasphinteric (supralevator) (E) not related to the sphincter mechanism page 234 . Kimmins MH. or cancer or its treatment. Hyman N. A clear understanding of these anatomic variants is essential to preserving continence while effectively managing this common surgical problem REFERENCES 1. traversing the rectum above the levator ani through the ischiorectal fossa to the perianal skin.Whiteford MH. 2005 page 235 . Isler JT. Nelson JM. et al: Practice parameters for the treatment of perianal abscess and fistula-in-ano (revised). originating at the dentate line. Transphincteric fistulas result from ischiorectal abscesses and account for an additional 23% of cases. Crohn’s disease. A supralevator abscess may result in suprasphincteric fistulas (5%) when the tract. extends above the puborectalis and tracks lateral to the external sphincter to the distal perianal skin.American College of Surgeons SESAP 13 Critique of Category 3 Item 63 Most anorectal fistulas (70%) result from a perianal abscess. tracking within the intersphinteric plane. These unusual fisulas may result from trauma (including iatrogenic probing during surgical fistulotomy). diverticulitis. et al: The diagnosis and management of common anorectal disorders. The remaining 2% of fistulas are extrasphincteric. Kilkenny J III. Dis Colon Rectum 48:1337-1342. Buie WD. Curr Probl Surg 41:586-645.Billingham RP. 2004 2. American College of Surgeons SESAP 13 Category 3 Item 64 Use of a seton should be considered in the management of anorectal fistulas associated with any of the following characteristics EXCEPT (A) those tracking > 30% to 50% of the external sphincter (B) anterior fistulas in women (C) recurrent fistulas (D) those associated with Crohn’s disease (E) intersphincteric fistulas page 236 . 2005 page 237 . Dis Colon Rectum 48:1337-1342. anterior fistulas in women.American College of Surgeons SESAP 13 Critique for Category 3 Item 64 The goals of fistula operation are to eliminate the fistula. prevent recurrent disease. Isler JT.Billingham RP. REFERENCES 1.Whiteford MH. or impaired continence. et al: Practice parameters for the treatment of perianal abscess and fistula-in-ano (revised). previous sphincter injury. If the fistula tract traverses the sphincter mechanism proximal to the dentate line. Buie WD. et al: The diagnosis and management of common anorectal disorders. Kilkenny J III. Crohn’s disease. and preserve sphincter function. Kimmins MH. The lower portion of the internal sphincter and anal skin is divided. Patients in whom seton placement should be strongly considered include those with recurrent or multiple fistulas. placement of a seton in conjunction with a partial fistulotomy is a safe option. Intersphincteric fistulas do not involve the external sphincter mechanisms and can be managed safely with simple fistulotomy. Curr Probl Surg 41:586-645. inserting a vessel loop or suture around the external sphincter into the portion of the tract above the dentate line. The seton will stimulate granulation tissue. Nelson JM. 2004 2. This requires identification of the primary opening and limiting the amount of external sphincter muscle divided. reducing sphincter retraction at the time of secondary fistulotomy. Hyman N. American College of Surgeons SESAP 13 Category 3 Item 65 All of the following are contraindications to neostigmine therapy for suspected colonic pseudoobstruction EXCEPT (A) bradycardia (B) history of bronchospasm (C) renal failure (D) abdominal pain (E) mechanical bowel obstruction page 238 . pain is not a contraindication in the absence of other signs and symptoms suggestive of intestinal ischemia or perforation. 2005 2. Mechanical obstruction should be excluded as a contributing factor prior to administration of neostigmine. Kimmey MB: Neostigmine for the treatment of acute colonic pseudoobstruction. neostimine increases airway secretions and bronchial reactivity. J Clin Gastroenterol 39:17-20.Fazel A. colonoscopic decompression is preferable to neostigmine.American College of Surgeons SESAP 13 Critique for Category 3 Item 65 Randomized controlled trials support the use of intravenous neostigmine as a treatment for acute colonic pseudo-obstruction. Aliment Pharmacol Ther 22:917-925. 1999 3. which may exacerbate bronchospasm. Although the presence of abdominal pain in association with colonic distention would be a concern.Ponec RJ. This is most easily accomplished with a retrograde single-contrast water-soluble enema. Verne GN: New solutions to an old problem: acute colonic pseudo-obstruction. If obstruction cannot be excluded. Because renal excretion accounts for 50% of drug clearance. Similarly.Saunders MD. 2005 page 239 . Saunders MD. patients with renal impairment may have an increased or prolonged vagomimetic response after treatment. documenting a lack of proximal distention/distal collapse. Kimmey MB: Systematic review: acute colonic pseudo-obstruction. Patients with underlying bradyarrhythmias or those receiving adrenergic antagonists may be more susceptible to neostigmine-induced bradycardia. REFERENCES 1. Clinical response is defined as a prompt evacuation of flatus and/or stool with reduction in abdominal distention. N Engl J Med 341:137-141. American College of Surgeons SESAP 13 Category 3 Item 66 Neoadjuvant chemoradiotherapy for esophageal adenocarcinoma offers a potential survival benefit for patients with clinical tumor stage (A) T1N0M0 (B) T2N0M0 (C) T2N2M1 (D) T3N1M0 (E) T3N2M1 page 240 . and identification of distant metastatic disease (M). 2003 page 241 . Buck DA. the presence of suspicious or biopsy proven local-regional nodal involvement (N). BMC Med 2:35.Urschel JD.Frechette E. or combined chemoradiation treatments have shown no added benefit over operation alone for patients with tumor into but not through the muscularis propria and without evidence for nodal or distant metastasis (T2N0M0).Malthaner RA. et al: Surgery alone versus chemoradiotherapy followed by surgery for resectable cancer of the oesophagus: a randomised controlled phase III trial. or T4. Wong RK. 2004 3. Lancet Oncol 6:659-668. Studies of patients with comparable clinical stage and pathology have evaluated the relative impact of chemotherapy. Zuraw L. Smithers BM. et al: Neoadjuvant or adjuvant therapy for resectable esophageal cancer: a clinical practice guideline.American College of Surgeons SESAP 13 Critique for Category 3 Item 66 Treatment modalities for adenocarcinoma of the distal esophagus continue to evolve. In addition. 2004 5. Gebski V. operation. Although lymph node involvement becomes increasingly more likely with deeper tumor penetration. J Surg Oncol 87:68-74. REFERENCES 1. as may arise in Barrett’s epithelium or those with tumors that penetrate up to but not through the submucosa (T1N0M0) can be cured with surgical resection alone. this subgroup is the subject of ongoing trials. Recommendations depend largely on the pretreatment clinical stage. et al: Neoadjuvant or adjuvant therapy for resectable esophageal cancer: a systematic review and meta-analysis. Am J Surg 185:538-543. lesions pentrating into adjacent structures such as the trachea) with limited local or regional nodal involvement. however. present with larger tumors (T3. the aim of this preoperative therapy is to control local disease in the bed of the proposed surgical resection. et al: Esophageal cancer: outcomes of surgery. radiation therapy. BMC Cancer 4:67. Involvement of > 5 lymph nodes has been associated with systemic recurrence. After pathologic confirmation. transmural tumors pentrating into periesophageal fat. Patients with high-grade dysplasia. Zuraw L. because neoadjuvant chemoradiation does not reduce the incidence of systemic recurrence.Malthaner RA. or a multimodality approach and have begun to identify subgroups of patients who benefit from a specific strategy. 2005 2. Rumble RB. 2004 4. and three-dimension conformal radiotherapy. a pre-treatment clinical stage may be established by combining abdominal and chest computed tomography (CT) with endoscopic ultrasound (EUS) and cytologic biopsy of suspicious local nodal disease. defined as the depth of primary tumor (T). Those patients who show a complete pathologic response at the time of surgical resection have a substantial improvement in survival (up to 59% at 3 years). Chung TD. A significant number of patients. neoadjuvant chemotherapy. Fitzgerald L. Wong RK. Surgical resection should not be offered to patients who have clear evidence for distant metastases at the time of presentation. Rumble RB. radiation therapy. Kaplan BJ. Vasan H: A meta-analysis of randomized controlled trials that compared neoadjuvant chemoradiation and surgery to surgery alone for resectable esophageal cancer. Current recommendations are for chemotherapy with or without added radiotherapy and management of dysphagia with endoscopic stent. neoadjuvant chemotherapy.Burmeister BH. the boy develops respiratory distress and requires intubation. the next step in management should be (A) abdominal exploration (B) total parenteral nutrition (TPN) (C) corticosteroids (D) fluoroscopy and string placement (E) pneumatic dilation page 242 . he has erythema and moderate edema to the hard palate and pharynx. On examination. A follow-up chest x-ray and gastrograffin swallow do not suggest perforation. During the course of evaluation. An initial attempt to scope the esophagus failed because of edema and circumferential ulcerations of the esophagus. In addition to antibiotics.American College of Surgeons SESAP 13 Category 3 Item 67 A 9-year-old boy is brought to the emergency department after drinking half a bottle of liquid drain cleaner. Debridement of all necrotic tissue is the initial treatment. Gastrointest Endosc 54: 368-372. REFERENCES 1. Contact time is longer for the stomach than the esophagus. Corticosteroids have no role in the management of this condition of tissue necrosis. Second-look explorations are often necessary and reconstruction can wait until the patient recovers from the hypovolemic and possibly septic shock. et al: Combined antegrade and retrograde dilation: A new endoscopic technique in the management of complex esophageal obstruction. No further attempts at manipulating the injured esophagus should be made because of the extreme risk of perforation. and there is a substantial risk for gastric perforation.Hawkins DB: Dilation of esophageal strictures: comparative morbidity of antegrade and retrograde methods. Laparotomy will identify the extent of necrosis. Ann Otol Rhinol Laryngol 97:460-465. String placement is used for esophageal stricture. Swanson SJ. as shown by the findings at gentle endoscopy and respiratory failure.Bueno R. Lukanich JM.American College of Surgeons SESAP 13 Critique for Category 3 Item 67 This boy is critically ill. A feeding jejunostomy can be placed in hemodynamically normal patients when re-operation is not believed to be indicated. Although total parenteral nutrition (TPN) may be needed. abdominal exploration is required. 2001 2. which is a late complication of caustic ingestion. Jaklitsch MT. 1988 page 243 . American College of Surgeons SESAP 13 Category 3 Item 68 A 47-year-old woman presents with a 1. mild epigastric pain on swallowing solid food. Which of the following is appropriate for this patient? (A) Esophageal lengthening procedure (B) Observation (C) Partial gastrectomy (D) Stamm gastrostomy (E) Surgical reduction of the hernia page 244 .to 2-month history of postprandial. The x-rays shown are obtained. Either an open or laparoscopic approach may be used. with the esophagogastric junction at or below the diaphragm. Sliding hernias are associated with heartburn symptoms. Partial gastrectomy may be necessary if gastric necrosis has occurred. both require surgical repair. In type II hernias. 2003 page 245 . Surg Clin North Am 85:411-432. In type IV hernias. A gastrostomy can be added to repair of the hiatus to help fix the stomach and esophagogastric junction below the diaphragm. more than the stomach is involved. Surg Endosc 17:1372-1375. as shown. Patients may present with subtle signs of gastritis. but this is needed only in rare cases. but paraesophageal hernias occur when the stomach twists up into the chest through the esophageal diaphragmatic hiatus. Distention of the stomach within the chest with secretions or gas causes colic and epigastric pain. including anemia or heme-positive stools due to vascular congestion within the trapped portion of the stomach.Leeder PC. The esophagogastric junction is displaced into the chest in type III hernias. the esophagus is usually in the normal position. Swallowing can be difficult when the incarcerated stomach compresses the esophagus. The main risk is strangulation of the incarcerated organ(s). 2005 2. Smith G. REFERENCES 1. Dehn TC: Laparoscopic management of large paraesophageal hiatal hernia. Santos R: Management of paraesophageal hernias. and can cause a variety of digestive complaints. As with type II hernias. but is not indicated if the stomach is viable. Del Pino M. The diagnosis can be confirmed with a barium swallow. They are more common in the elderly. which requires surgical reduction. Sliding hernias are type I.Landreneau RJ.American College of Surgeons SESAP 13 Critique for Category 3 Item 68 The chest x-ray suggests a paraesophageal hernia. Observation of symptomatic patients is warranted only for the significantly infirm. The esophagus does not need to be lengthened. A Helicobacter pylori test is negative.American College of Surgeons SESAP 13 Category 3 Item 69 A 52-year-old man has an upper gastrointestinal endoscopy for ulcer-type pain 15 years after a highly selective vagotomy. The most likely explanation for his recurrent ulcer is (A) antral G cell hyperplasia (B) bile reflux (C) gastrinoma (D) incomplete vagotomy (E) occasional use of nonsteroidal anti-inflammatory drugs page 246 . as shown. Endoscopy confirms a recurrent duodenal ulcer. When endoscopy shows a recurrent duodenal ulcer. not recurrent duodenal ulceration. Nonsteroidal medications are erosive and prostaglandin inhibition alters the stomach’s mucosal barrier. Zinner MJ: Reoperation for recurrent peptic ulcer disease. They are associated with gastric ulcers. 1987 2. World J Surg 11:274-282. Blackett RL: Recurrent peptic ulcers. G-cell hyperplasia occurs after vagotomy. The right-posterior vagus nerve can branch high in the chest. J Gastrointest Surg 7:606-626. this conservative approach may miss significant branches to the posterior stomach.Johnston D. and bile reflux does not cause duodenal ulcers. However. Spechler S. REFERENCES 1. the etiology is almost certainly acid-related. Cryer B. but does not cause acid hypersecretion or ulcers. A gastrinoma with complications is rare in comparison to an incomplete vagotomy. 2003 page 247 .American College of Surgeons SESAP 13 Critique for Category 3 Item 69 A highly selective vagotomy spares the branches to the liver and biliary system.McFadden DW.Turnage RH. and these separate branches can be the source of continued acid stimulation and recurrent ulcerations. Surg Clin North Am 71:7792. the so-called criminal nerve of Grassi. The duodenum is quite used to bile. 1991 3. Sarosi G. et al: Evaluation and management of patients with recurrent peptic ulcer disease after acid-reducing operations: a systematic review. and superior mesenteric artery embolectomy (D) placement of a pulmonary artery catheter and improvement of cardiac output (E) revascularization of mesenteric vessels using autogenous conduit page 248 .American College of Surgeons SESAP 13 Category 3 Item 70 Four days after an uneventful coronary artery bypass graft operation. a 59-year-old man has congestive heart failure and atrial fibrillation. An arteriogram shows normal proximal mesenteric vessels.000/mm3 and slightly elevated potassium and amylase. He develops acute epigastric/periumbilical pain. He has abdominal distention but no peritoneal signs. A computed tomographic (CT) scan is normal. resection of compromised bowel. The MOST appropriate management would be (A) catheter-directed arterial lytic therapy (B) laparoscopic fluorescein test with bowel resection as indicated (C) laparotomy. Arterial blood gases demonstrate no significant metabolic acidosis. but gradual tapering of the distal mesenteric arteries. The patient has a WBC count of 14. especially if corrected early. If abdominal findings progressed or acidosis developed despite medical therapy. Improved cardiac output will usually reverse the intestinal ischemia. Arch Intern Med 164:1054-1062.Yasuhara H: Acute mesenteric ischemia: the challenge of gastroenterology. Rodenberg TJ.Oldenburg WA. 2004 2. Edmonds HJ. and normal CT do not suggest bowel necrosis and urgent laparotomy would not be indicated. the ideal filling pressure for optimal cardiac output. Lau LL. 2005 page 249 . Systemic vascular resistance can also be managed with vasoactive drugs. Surg Today 35:185195. ie. and may potentially be salvaged with optimal hemodynamics. REFERENCES 1. A monitoring device such as a pulmonary artery catheter (Swan-Ganz) will allow for careful titration of the Starling curve. Bowel resection during abdominal exploration is needed if necrotic bowel is present. Lytic therapy or revascularization is not needed for this low-flow state. minimal abdominal findings. then exploration would be appropriate. et al: Acute mesenteric ischemia: a clinical review. The patient's normal arterial blood gas. thereby limiting the workload of the heart. Fluorescein is used at operation to help determine which segment or segments of bowel are marginal yet remain viable.American College of Surgeons SESAP 13 Critique for Category 3 Item 70 Patent vessels and no occlusive lesions on mesenteric angiography is most consistent with lowflow mesenteric ischemia from insufficient cardiac output. 5-cm ulcerated lesion with a raised border. external to the anal opening. Examination reveals a 1.American College of Surgeons SESAP 13 Category 3 Item 71 A 68-year-old otherwise healthy man presents with a lesion near the anus. Digital rectal examination is normal. Biopsy shows this lesion is an invasive squamous carcinoma. The best treatment for this condition would be (A) topical fluorouracil (5-FU) (B) chemoradiation (Nigro protocol) (C) wide local excision (D) downstaging radiotherapy followed by abdominoperineal resection (E) abdominoperineal resection page 250 . Most of these squamous cancers are well-differentiated.Dev VR. The anal canal extends from the upper border to the lower border of the internal anal sphincter. Large defects after wide local excision may require reconstruction with grafts or flaps. mitomycin C) according to the Nigro protocol. from the pelvic floor to the anal verge. such as the one described here. Surg Oncol Clin N Am 13:321-338. REFERENCES 1. are called anal margin tumors. whereas recurrent anal margin cancer is usually treated by repeat excision. 2004 page 251 . It is important to differentiate anatomically the anal margin from the anal canal because location influences tumor behavior and has a direct impact on treatment. Rodriguez-Bigas MA. Neither anal margin nor anal canal cancer should be treated with initial abdominoperineal resection. They grow slowly and metastasize to the inguinal lymph nodes. ie. whereas anal canal cancer is treated initially with chemoradiation (fluorouracil. Lesions located outside of the anal verge. Gordon PH: Surgical considerations in anal cancer. Gupta A: Plastic and reconstructive surgery approaches in the management of anal cancer. The recommended treatment for anal margin cancer is wide local excision.Skibber J. Abdominoperineal excision is reserved for recurrent anal canal cancer.American College of Surgeons SESAP 13 Critique for Category 3 Item 71 The lesion described and shown here is located on the anal margin. Topical fluorouracil has no role in the treatment of anal margin or anal canal cancer. Surg Oncol Clin N Am 13:339-353. 2004 2. The patient feels intra-rectal and low pelvic pressure. The end of the vibrator is barely palpable by digital rectal examination.American College of Surgeons SESAP 13 Category 3 Item 72 A 45-year-old man has been unable to remove a vibrator inserted in his rectum in the course of sexual foreplay 24 hours ago. There is no free air in the abdomen.to upper rectum. but the vibrator is palpable in the lower abdomen. but has no abdominal pain. Abdominal films confirm the presence of the vibrator in the mid. The next step should be (A) transanal removal under local perianal block (B) transanal removal under conscious sedation (C) transanal removal under monitored anesthesia (D) transanal removal with laparoscopic assisted abdominal pressure under general anesthesia (E) laparotomy with colotomy and transabdominal extraction under general anesthesia page 252 . Abdominal examination reveals no peritoneal signs. 2005. pp 149-151 page 253 . High-lying objects at or proximal to the rectosigmoid junction are best removed in lithotomy position under spinal anesthesia. 0.000). observation for 24 hours may be indicated to rule out delayed perforation. regional anesthesia. perforations. and injecting slowly. additionally. monitored anesthesia care. or general anesthesia. and grasping instruments may be used to gently remove the foreign body. Lowlying objects located in the rectal ampulla can almost always be removed via the transanal route. In cases of difficult extraction. scopes. Following extraction. This allows for abdominal counterpressure and external displacement of the foreign body. Enemas should not be used. This provides total sphincter relaxation and allows for maximal dilation of the anus for manual extraction of the foreign body. Extraction should be done cautiously so as not to damage the bowel further. conscious sedation. or hemorrhage have not occurred. as long as an adequate perianal block is obtained. pressure necrosis. a comfortable and awake patient may be able to help by performing a Valsalva maneuver. In-patient observation is not necessary if the extraction was easy and postextraction endoscopy shows no mucosal injury. Laparotomy may be necessary when the foreign body is wedged. the bowel must be inspected directly endoscopically to confirm that tears. Sedation or anesthesia is often unnecessary. Clinical judgment must be exercised. This is done by infiltrating the anal sphincter with 40 mL of a local anesthetic (eg.Sohn N. Church JM. Alternatively. REFERENCES 1. Presentation is often delayed. or when prolonged impaction is present.American College of Surgeons SESAP 13 Critique for Category 3 Item 72 Patients presenting with retained anorectal foreign bodies are seen regularly in the emergency departments of most large urban centers. turn along their axis. The need for sedation and instrumentation is usually minimal. Foreign bodies may migrate proximally. Aronoff JS: Rectal foreign bodies.Kann BR. Life-threatening injuries from bowel perforation or hemorrhage must be excluded. Gentleness is paramount. Philadelphia. Semin Colon Rectal Surg 15:119-124. or become lodged above a curve in the rectosigmoid colon or sacrum. Hicks TC: Anorectal foreign bodies: evaluation and treatment. or general anesthesia may be helpful. Delaney CP (eds): Current Therapy in Colon and Rectal Surgery. 10 mL is used as a deposit in each of the 4 perianal quadrants. or when extensive manipulation was required. Elsevier Mosby. making extraction more difficult. If unsuccessful. Delayed presentation also increases the chances of ischemia and pressure necrosis. 2004 2. because they increase edema and may precipitate perforation.25% marcaine with epinephrine 1:100. in Fazio VW. ed 2. using a 2-in 25gauge needle. retractors. 300/mm3 ) with left shift (90.6% neutrophils).American College of Surgeons SESAP 13 Category 3 Item 73 A 68-year-old man presents with a 6-day history of right lower quadrant abdominal pain. as shown. An abdominal and pelvic computed tomographic (CT) scan is obtained. He is hemodynamically normal. WBC count is increased (16. Abdominal examination confirms localized right lower quadrant abdominal tenderness and no peritonitis. The preferred management of this patient would include (A) emergent laparotomy (B) emergent diagnostic laparoscopy (C) intravenous antibiotics. bowel rest (D) diagnostic colonoscopy (E) upper gastrointestinal series with small bowel follow-through page 254 . making the diagnosis of perforated appendicitis likely. Dig Surg 17:508512.Yamini D. Shperber Y. Vargas H. 864 of the 1012 patients in whom nonoperative treatment was used initially did not have an interval appendectomy. initial computed tomographic (CT) scan shows a complex phlegmon in the right lower quadrant and the appendix is not visualized. 1994 5. Emergent laparotomy in this setting is associated with high complication rates. et al: Routine interval appendectomy is not justified after initial nonoperative treatment of acute appendicitis. Orda R: Cecal diverticulitis: a diagnostic challenge. Klein S. 2005 2. Two weeks later. Diagnostic laparoscopy and diagnostic colonoscopy are not indicated because the etiology of the phlegmon need not be determined at this time. leukocytosis. and percutaneous drainage with ultrasound or CT guidance would most commonly be used. Arch Surg 140:897-901. Applebaum H. inflammatory bowel diseases. when the patient presents with fever. Surgical intervention with intestinal resection is not necessary in the absence of evidence of peritonitis.American College of Surgeons SESAP 13 Critique for Category 3 Item 73 The differential diagnosis of a right lower quadrant inflammatory mass includes perforated appendicitis. Lee SL.Keidar S. Chew FS: Radiological reasoning: recurrent right lower quadrant inflammatory mass. but should not be performed at this time. when management of the infectious process must be the priority. Liu IL. cecal diverticulitis. Because the patient is hemodynamically normal and has no evidence of peritonitis. Averbook A. AJR Am J Roentgenol 185:S188-S194. a nonoperative approach with intravenous antibiotics and observation is appropriate. REFERENCES 1. and localized pain to the right lower quadrant. et al: Perforated appendicitis: is it truly a surgical urgency? Am Surg 64:970-975.Kaminski A.Vargas HI. Upper gastrointestinal series with small bowel follow-through may be helpful as a diagnostic strategy for inflammatory bowel disease. In the case presented. Bongard F. Abscess drainage is indicated at this time.Bittle MM. 1998 page 255 . Recurrent appendicitis occurred in only 39 patients (5%) after a median follow-up of 4 years. appendiceal carcinoma. Nonoperative management with intravenous antibiotics is an accepted management strategy if the leading clinical diagnosis is perforated appendicitis. and other rare diseases.938 patients with acute appendicitis. Am Surg 60:753-758. Stamos MJ: Appendiceal mass: conservative therapy followed by interval laparoscopic appendectomy. cecal carcinoma. Pappo I. and should be reserved for patients who do not respond to abscess drainage. Subsequent interval appendectomy is controversial. 2000 4. 2005 3. CT scan imaging demonstrates a large abscess in the right lower quadrant. In a recent study of 32. Meckel’s diverticulum. An abdominal and pelvic computed tomographic (CT) scan is obtained.American College of Surgeons SESAP 13 Category 3 Item 74 A 68-year-old man presents with a 6-day history of right lower quadrant abdominal pain. Abdominal examination confirms localized right lower quadrant abdominal tenderness and no peritonitis. A repeat CT scan is obtained. fever to 102 F. and leukocytosis to 22.000/mm3.6% neutrophils). He is hemodynamically normal.300/mm3 ) with left shift (90. The preferred management of this patient would be (A) right colectomy and ileocolostomy (B) right colectomy and end-ileostomy (C) total abdominal colectomy and ileostomy (D) abscess drainage and diverting loop ileostomy (E) abscess drainage only page 256 . WBC count is increased (16. the patient presents with increased abdominal distention with localized right lower quadrant abdominal pain. as shown in the previous item. as shown. Two weeks later. Kaminski A. when the patient presents with fever.Bittle MM. when management of the infectious process must be the priority. but should not be performed at this time. In a recent study of 32. Shperber Y. leukocytosis. Dig Surg 17:508512. 1994 5. Applebaum H. CT scan imaging demonstrates a large abscess in the right lower quadrant. Orda R: Cecal diverticulitis: a diagnostic challenge. Bongard F. a nonoperative approach with intravenous antibiotics and observation is appropriate. 1998 page 257 . Vargas H. Abscess drainage is indicated at this time. In the case presented. appendiceal carcinoma. Two weeks later. 2000 4. 864 of the 1012 patients in whom nonoperative treatment was used initially did not have an interval appendectomy. Nonoperative management with intravenous antibiotics is an accepted management strategy if the leading clinical diagnosis is perforated appendicitis. Upper gastrointestinal series with small bowel follow-through may be helpful as a diagnostic strategy for inflammatory bowel disease. Averbook A. making the diagnosis of perforated appendicitis likely. cecal carcinoma. 2005 3. and other rare diseases.Yamini D.American College of Surgeons SESAP 13 Critique for Category 3 Item 74 The differential diagnosis of a right lower quadrant inflammatory mass includes perforated appendicitis. Because the patient is hemodynamically normal and has no evidence of peritonitis. Am Surg 60:753-758. inflammatory bowel diseases.Keidar S. Lee SL. 2005 2. AJR Am J Roentgenol 185:S188-S194. and should be reserved for patients who do not respond to abscess drainage. cecal diverticulitis.Vargas HI. Arch Surg 140:897-901. Meckel’s diverticulum. Emergent laparotomy in this setting is associated with high complication rates. Recurrent appendicitis occurred in only 39 patients (5%) after a median follow-up of 4 years. Subsequent interval appendectomy is controversial.938 patients with acute appendicitis. initial computed tomographic (CT) scan shows a complex phlegmon in the right lower quadrant and the appendix is not visualized. Liu IL. et al: Routine interval appendectomy is not justified after initial nonoperative treatment of acute appendicitis. Diagnostic laparoscopy and diagnostic colonoscopy are not indicated because the etiology of the phlegmon need not be determined at this time. Chew FS: Radiological reasoning: recurrent right lower quadrant inflammatory mass. et al: Perforated appendicitis: is it truly a surgical urgency? Am Surg 64:970-975. Stamos MJ: Appendiceal mass: conservative therapy followed by interval laparoscopic appendectomy. and localized pain to the right lower quadrant. Surgical intervention with intestinal resection is not necessary in the absence of evidence of peritonitis. Pappo I. and percutaneous drainage with ultrasound or CT guidance would most commonly be used. REFERENCES 1. Klein S. American College of Surgeons SESAP 13 Category 3 Item 75 A 30-year-old man is brought into the emergency department 30 minutes after drinking drain cleaner. The patient is hemodynamically normal and able to control his airway. intravenous fluids (B) serial chest and abdominal x-rays (C) immediate administration of neutralizing agents (D) upper endoscopy within 24 hours to determine extent of injury (E) laryngoscopy within 24 hours to evaluate for epiglottis injury/edema page 258 . He has throat and chest pain. and pain with swallowing. Initial management of this patient includes all of the following EXCEPT (A) observation in the ICU. Staszak VM. Endoscopy is useful to determine the extent of injury and presence of esophageal or gastric necrosis. Delayed problems include stricture and partial esophageal function. Am J Gastroenterol 87:337-341. Laryngoscopy aids in determining epiglottis injury and/or edema. Although corticosteroids were formerly recommended. Dees J. 1989 2. Am J Surg 157:116-120. Early (within 24 hours) upper endoscopy and laryngoscopy should be used to determine the extent of injury. Dilation can be performed. and strictures related to caustic injury are usually long. The stomach is also frequently injured. Neutralizing agents should never be used because of the risk of further damage from chemical or thermal means from exothermic reactions. 2004 3. Esophagectomy is indicated when dilation is contraindicated or has failed. Gastrointest Endosc 60:372-377. and an increased risk of subsequent carcinoma. complete obstruction. providing intravenous access and fluid resuscitation. Nagi B. and monitoring of vital signs and urine output. Esophageal myotomy does not address the underlying problem of a transmural cicatrix. and resection is also indicated if cancer cannot be definitively ruled out. rather than blindly. Mehta S. lye or other cleaning agents. Kochhar R. MK. 1992 page 259 . eg. Serial chest and abdominal x-rays are useful in assessing for mediastinal or peritoneal free air. Pneumatic dilation is best for short strictures.Zargar SA. Patients who suffer caustic injuries are at higher risk of esophageal cancer. the patient will require immediate surgical intervention.Poley JW. The extent of injury at the time of endoscopy may determine subsequent stricture formation. battery acid. but acid-related injuries are generally more severe that alkaline-related injuries. Initial management should include ensuring a secure airway.Ferguson. and alkaline agents. as in this case. although this is not mandatory for mild injuries. Kuipers EJ. and cannot be used as a conduit for reconstruction. which may help determine the need for early intubation to secure an airway. REFERENCES 1.American College of Surgeons SESAP 13 Critique of Category 3 Item 75 Ingestion of caustic materials usually occurs accidentally (in children who find such materials stored in cabinets) or as a result of a suicide attempt (usually in adults). their use does not change the ultimate outcome. Steyerberg EW. Strictures can be very difficult to treat. The immediate concerns with caustic injuries are esophageal necrosis with subsequent perforation and the multiple problems such as shock and infection caused by caustic and esophageal contents in the mediastinal and pleural spaces. but should be done over a guidewire. The extent of the injury can vary. Corticosteroids have shown no benefit in lessening the extent of the stricture. et al: Ingestion of acid and alkaline agents: outcome and prognostic value of early upper endoscopy. These injuries are generally divided into those caused by acidic agents. Migliore M. Little AG: Early evaluation and therapy for caustic esophageal injury. such as a Savory wire. et al: Ingestion of strong corrosive alkalis: spectrum of injury to upper gastrointestinal tract and natural history. If frank necrosis is present. Most patients will require ICU monitoring. eg. and pain with swallowing.American College of Surgeons SESAP 13 Category 3 Item 76 A 30-year-old man is brought into the emergency department 30 minutes after drinking drain cleaner. The esophagogram shown is obtained. He has throat and chest pain. The patient recovers from the acute illness.Within a few weeks. The patient is hemodynamically normal and able to control his airway. The best treatment would be (A) repeated esophageal dilation with bougies (B) high-dose corticosteroids (C) endoscopic pneumatic dilation (D) long esophageal myotomy (E) esophagectomy with colon as conduit page 260 . he develops severe dysphagia. eg. Pneumatic dilation is best for short strictures. as in this case. Early (within 24 hours) upper endoscopy and laryngoscopy should be used to determine the extent of injury. their use does not change the ultimate outcome. 2004 3. Delayed problems include stricture and partial esophageal function. lye or other cleaning agents. Steyerberg EW. Kuipers EJ. Patients who suffer caustic injuries are at higher risk of esophageal cancer. Nagi B.American College of Surgeons SESAP 13 Critique for Category 3 Item 76 Ingestion of caustic materials usually occurs accidentally (in children who find such materials stored in cabinets) or as a result of a suicide attempt (usually in adults). REFERENCES 1. Corticosteroids have shown no benefit in lessening the extent of the stricture. and monitoring of vital signs and urine output. the patient will require immediate surgical intervention. Kochhar R. but acid-related injuries are generally more severe that alkaline-related injuries. These injuries are generally divided into those caused by acidic agents. Neutralizing agents should never be used because of the risk of further damage from chemical or thermal means from exothermic reactions. such as a Savory wire. 1992 page 261 . The immediate concerns with caustic injuries are esophageal necrosis with subsequent perforation and the multiple problems such as shock and infection caused by caustic and esophageal contents in the mediastinal and pleural spaces. Strictures can be very difficult to treat. et al: Ingestion of strong corrosive alkalis: spectrum of injury to upper gastrointestinal tract and natural history. Initial management should include ensuring a secure airway. battery acid. rather than blindly. Little AG: Early evaluation and therapy for caustic esophageal injury. The stomach is also frequently injured. Serial chest and abdominal x-rays are useful in assessing for mediastinal or peritoneal free air. although this is not mandatory for mild injuries. complete obstruction. Esophageal myotomy does not address the underlying problem of a transmural cicatrix. Dilation can be performed. Mehta S.Poley JW. Am J Gastroenterol 87:337-341. and an increased risk of subsequent carcinoma. If frank necrosis is present. The extent of the injury can vary. and resection is also indicated if cancer cannot be definitively ruled out. and cannot be used as a conduit for reconstruction. eg. Laryngoscopy aids in determining epiglottis injury and/or edema. Although corticosteroids were formerly recommended. Staszak VM. and alkaline agents. 1989 2. which may help determine the need for early intubation to secure an airway. Most patients will require ICU monitoring. Endoscopy is useful to determine the extent of injury and presence of esophageal or gastric necrosis. et al: Ingestion of acid and alkaline agents: outcome and prognostic value of early upper endoscopy.Ferguson. Am J Surg 157:116-120.Zargar SA. providing intravenous access and fluid resuscitation. but should be done over a guidewire. Gastrointest Endosc 60:372-377. The extent of injury at the time of endoscopy may determine subsequent stricture formation. Migliore M. Esophagectomy is indicated when dilation is contraindicated or has failed. Dees J. MK. and strictures related to caustic injury are usually long. She has no history of abdominal operation. lasting only an hour or so. but there are no peritoneal signs. There are no hernias. On physical examination her abdomen is distended.American College of Surgeons SESAP 13 Category 3 Item 77 A 40-year-old woman presents with a 12-hour history of colicky abdominal pain and vomiting. WBC count is elevated with a left shift. Previous episodes over the past year were self-limiting. tympanitic. and tender. These findings are most consistent with (A) lesser sac abscess (B) pancreatic pseudocysts (C) pseudomyxoma peritonei (D) cystic teratoma of the ovary (E) proximal jejunal intussusception page 262 . The abdominal computed tomographic (CT ) scan shown is obtained. vomiting. Nausea. the incidence of lead points increases exponentially. and gastrointestinal hemorrhage are less common. Rajesh A. 2004 5. Clinical reports of this process in older patients are limited to small case series. Govender P. Smith RC: Adult intussusception: case reports and review of literature. Reduction is not always possible and runs the risk of iatrogenic perforation and spillage of intestinal content. 2005 2. 2004 6. edematous walls. Geoghegan T. and they may present with symptoms that have been present for several days. Simple bypass is contraindicated for the same reason. Lappas J: Proximal small bowel intussusceptions in adults: CT appearance and clinical significance. Kopecky KK. Abdom Imaging 29:653-657.Sandrasegaran K.Eisen LK. Sudha P: Radiology for the surgeon: soft-tissue case 56. Only 50% of adults present with symptoms of acute intestinal obstruction. Pain is the most common symptom reported and is present in over 80%. O'Keefe SJ: Small bowel intussusception and brown bowel syndrome in association with severe malnutrition.American College of Surgeons SESAP 13 Critique for Category 3 Item 77 This patient’s computed tomographic (CT) images show dilated loops of small bowel with thickened. Clin Radiol 60:39-46. 1996 3. J Am Coll Surg 188:390-395. an abdominal mass can usually be palpated. REFERENCES 1. Aufses AH Jr: Intussusception in adults: institutional review. Small-bowel intussusception. The diagnosis is rarely made preoperatively in adults.Yalamarthi S. Lyburn ID. the intussusceptiens. Postgrad Med J 81:174-177. 1999 4. constipation. and the process can generally be managed nonoperatively.Byrne AT. Unlike pediatric patients.Vijay S. and further work-up is indicated. resection is recommended. 2005 page 263 .Drake WM. Intussusception is seen in children more commonly than adults. a lead point is found in up to 50% of adults. Small bowel involvement is prevalent in adults. Am J Gastroenterol 91:1450-1452. Intestinal plication and detorsion have no role in the management of small bowel intussusception. Dahiya N. Winter TA. In children under 2 years old. Considering the high association with intestinal pathology and malignancy. An intussusceptum (segment of small bowel and its mesentery) can be seen in both images within a contiguous but distal segment of small bowel. One report details the fact that small bowel intussusception in adults may be an incidental finding on CT scan. et al: The imaging of intussusception. Over the age of 2. Cunningham JD. Can J Surg 47:453-455. Price SK. American College of Surgeons SESAP 13 Category 3 Item 78 A 40-year-old woman presents with a 12-hour history of colicky abdominal pain and vomiting. tympanitic. Previous episodes over the past year were self-limiting. The abdominal computed tomographic (CT ) scan shown is obtained. and tender. Management should consist of (A) cyst gastrostomy (B) percutaneous drainage (C) unilateral oophorectomy (D) enterectomy and primary anastomosis (E) bilateral salpingo-oophorectomy and appendectomy page 264 . WBC count is elevated with a left shift. but there are no peritoneal signs. On physical examination her abdomen is distended. She has no history of abdominal operation. lasting only an hour or so. There are no hernias. Rajesh A. One report details the fact that small bowel intussusception in adults may be an incidental finding on CT scan. Reduction is not always possible and runs the risk of iatrogenic perforation and spillage of intestinal content. Intussusception is seen in children more commonly than adults. constipation. Postgrad Med J 81:174-177. Cunningham JD. Abdom Imaging 29:653-657. Unlike pediatric patients. Clinical reports of this process in older patients are limited to small case series. Lyburn ID. Over the age of 2. Winter TA. Can J Surg 47:453-455. Price SK. J Am Coll Surg 188:390-395. and further work-up is indicated. Am J Gastroenterol 91:1450-1452. Pain is the most common symptom reported and is present in over 80%. 2004 6. 2004 5.Drake WM. resection is recommended. REFERENCES 1. 2005 page 265 . Nausea. Dahiya N. vomiting. Govender P. 1999 4. Sudha P: Radiology for the surgeon: soft-tissue case 56. An intussusceptum (segment of small bowel and its mesentery) can be seen in both images within a contiguous but distal segment of small bowel. an abdominal mass can usually be palpated. the incidence of lead points increases exponentially. Only 50% of adults present with symptoms of acute intestinal obstruction. 1996 3. Lappas J: Proximal small bowel intussusceptions in adults: CT appearance and clinical significance. Small bowel involvement is prevalent in adults. Kopecky KK. and they may present with symptoms that have been present for several days.Vijay S. Aufses AH Jr: Intussusception in adults: institutional review. Geoghegan T. Clin Radiol 60:39-46. et al: The imaging of intussusception.Byrne AT. Considering the high association with intestinal pathology and malignancy.American College of Surgeons SESAP 13 Critique for Category 3 Item 78 This patient’s computed tomographic (CT) images show dilated loops of small bowel with thickened. 2005 2. Simple bypass is contraindicated for the same reason.Sandrasegaran K.Yalamarthi S.Eisen LK. edematous walls. the intussusceptiens. a lead point is found in up to 50% of adults. O'Keefe SJ: Small bowel intussusception and brown bowel syndrome in association with severe malnutrition. Smith RC: Adult intussusception: case reports and review of literature. Intestinal plication and detorsion have no role in the management of small bowel intussusception. and the process can generally be managed nonoperatively. Small-bowel intussusception. and gastrointestinal hemorrhage are less common. In children under 2 years old. The diagnosis is rarely made preoperatively in adults. American College of Surgeons SESAP 13 Category 3 Item 79 Occurs almost exclusively in women of child-bearing age (A) Hepatic hemangioma (B) Angiosarcoma in the liver (C) Hepatic adenoma (D) Focal nodular hyperplasia (E) Hepatocellular carcinoma page 266 . Lippincott Williams & Wilkins. pp 956-977 3. Lillemore KD. Han JK. et al: Hepatic hemangioma: contrast-enhancement pattern during the arterial and portal venous phases of spiral CT. Howard ER: Angiosarcoma of the liver in children.Yun EJ. Davenport M. Choi BI. Jang HJ.Awan S. Philadelphia. Curley SA: Hepatic neoplasms. J Pediatr Surg 31:1729-1732. 1999 page 267 . 1996 2. Abdom Imaging 24:262-266. ed 4. Maier RV.American College of Surgeons SESAP 13 Critique for Category 3 Item 79 REFERENCES 1.Delman KA. 2005. Portmann B. in Mulholland MW. et al (eds): Greenfield’s Surgery: Scientific Principles and Practice. Doherty GM. American College of Surgeons SESAP 13 Category 3 Item 80 Markedly elevated alpha-fetoprotein is diagnostic (A) Hepatic hemangioma (B) Angiosarcoma in the liver (C) Hepatic adenoma (D) Focal nodular hyperplasia (E) Hepatocellular carcinoma page 268 . and appropriate laboratory tests and proper imaging studies are obtained. Han JK. Most masses in the liver are benign and can be accurately diagnosed by a combination of a history and appropriate radiologic studies.Awan S. ed 4. 1996 2. When the lesion cannot be radiographically distinguished from hepatic adenoma or hepatocellular carcinoma.American College of Surgeons SESAP 13 Critique for Category 3 Item 80 Hepatic tumors can almost always be precisely characterized if a thorough history is taken. These lesions may be difficult to distinguish from another benign condition. Focal nodular hyperplasia is also predominantly seen in women but is less commonly symptomatic and not at risk for malignant degeneration. resection or further intervention can be avoided. Maier RV. MRI is also highly accurate to define a hemangioma. on radiologic studies. et al: Hepatic hemangioma: contrast-enhancement pattern during page 269 .Yun EJ. Hepatic hemangiomas are the most common benign tumor in the liver. Davenport M. They are highly vascular on computed tomographic (CT) scans. Focal nodular hyperplasia may be diagnosed if a central scar is seen. 2005. the risk of rupture is associated with lesions over 5 cm or rapid growth. Portmann B. Curley SA: Hepatic neoplasms. pp 956-977 3. J Pediatr Surg 31:1729-1732. but surgical resection is recommended for larger lesions or ones that do not shrink after cessation of oral contraceptive use. resection is indicated. If a definitive diagnosis cannot be reached in a large lesion. Philadelphia. in Mulholland MW. Many of these tumors are symptomatic. The clinical course of hemangiomas is benign and surgical resection is only indicated for symptomatic lesions that are large enough to cause pain or early satiety. They may be unresectable at diagnosis and neither chemotherapy nor radiotherapy is effective. A history of hepatitis or alcoholism is suggestive of hepatocellular carcinoma. Choi BI. Doherty GM. If the pathognomonic central scar can be identified on CT or MRI.Delman KA. presenting with abdominal pain. and a significant number of them show bleeding into the tumor or even rupture into the abdominal cavity. a needle biopsy may be undertaken. Large bilateral tumors are often seen at presentation and are commonly unresectable. Howard ER: Angiosarcoma of the liver in children. REFERENCES 1. they most frequently occur in the third and fourth decades of life and affect women more often than men. Hepatic adenomas almost always occur in women of child-bearing age who are taking oral contraceptives. focal nodular hyperplasia. a dynamic or spiral CT is the preferred examination and shows early enhancement of the periphery of the lesion. Both lesions demonstrate rapid contrast enhancement followed by rapid washout of contrast within the tumor on CT and magnetic resonance imaging (MRI). et al (eds): Greenfield’s Surgery: Scientific Principles and Practice. Resection of large lesions is also indicated because hepatocellular carcinoma may develop in large adenomas. The lesions may be most definitively diagnosed on CT using intravenous contrast. Jang HJ. Lillemore KD. They are most commonly identified as an asymptomatic mass found incidentally on investigation for other purposes. Hepatic angiosarcoma is a rare tumor seen primarily in children. a markedly elevated alpha-fetoprotein is diagnostic. Smaller adenomas (under 4 cm) may be managed by cessation of oral contraceptives. Like hepatic adenoma and focal nodular hyperplasia. Lippincott Williams & Wilkins. 1999 page 270 . Abdom Imaging 24:262-266.American College of Surgeons SESAP 13 the arterial and portal venous phases of spiral CT. American College of Surgeons SESAP 13 Category 3 Item 81 (A) Hepatic hemangioma (B) Angiosarcoma in the liver (C) Hepatic adenoma (D) Focal nodular hyperplasia (E) Hepatocellular carcinoma page 271 . Jang HJ. Resection of large lesions is also indicated because hepatocellular carcinoma may develop in large adenomas. Han JK. Most masses in the liver are benign and can be accurately diagnosed by a combination of a history and appropriate radiologic studies. Howard ER: Angiosarcoma of the liver in children. Hepatic hemangiomas are the most common benign tumor in the liver. They may be unresectable at diagnosis and neither chemotherapy nor radiotherapy is effective. If a definitive diagnosis cannot be reached in a large lesion. Davenport M. they most frequently occur in the third and fourth decades of life and affect women more often than men. Doherty GM. the risk of rupture is associated with lesions over 5 cm or rapid growth. Maier RV. on radiologic studies. and a significant number of them show bleeding into the tumor or even rupture into the abdominal cavity. These lesions may be difficult to distinguish from another benign condition. A history of hepatitis or alcoholism is suggestive of hepatocellular carcinoma. When the lesion cannot be radiographically distinguished from hepatic adenoma or hepatocellular carcinoma. focal nodular hyperplasia. Focal nodular hyperplasia may be diagnosed if a central scar is seen. in Mulholland MW. but surgical resection is recommended for larger lesions or ones that do not shrink after cessation of oral contraceptive use. Many of these tumors are symptomatic. Smaller adenomas (under 4 cm) may be managed by cessation of oral contraceptives. Philadelphia. J Pediatr Surg 31:1729-1732. resection is indicated. Like hepatic adenoma and focal nodular hyperplasia. and appropriate laboratory tests and proper imaging studies are obtained. presenting with abdominal pain. Both lesions demonstrate rapid contrast enhancement followed by rapid washout of contrast within the tumor on CT and magnetic resonance imaging (MRI). 1996 2. Lippincott Williams & Wilkins. Choi BI. ed 4. et al: Hepatic hemangioma: contrast-enhancement pattern during page 272 . 2005.Yun EJ. et al (eds): Greenfield’s Surgery: Scientific Principles and Practice. Lillemore KD. The lesions may be most definitively diagnosed on CT using intravenous contrast. They are most commonly identified as an asymptomatic mass found incidentally on investigation for other purposes. Large bilateral tumors are often seen at presentation and are commonly unresectable. Portmann B. REFERENCES 1. Hepatic angiosarcoma is a rare tumor seen primarily in children. The clinical course of hemangiomas is benign and surgical resection is only indicated for symptomatic lesions that are large enough to cause pain or early satiety.Awan S. a dynamic or spiral CT is the preferred examination and shows early enhancement of the periphery of the lesion.Delman KA. resection or further intervention can be avoided. Curley SA: Hepatic neoplasms.American College of Surgeons SESAP 13 Critique for Category 3 Item 81 Hepatic tumors can almost always be precisely characterized if a thorough history is taken. MRI is also highly accurate to define a hemangioma. Focal nodular hyperplasia is also predominantly seen in women but is less commonly symptomatic and not at risk for malignant degeneration. They are highly vascular on computed tomographic (CT) scans. a markedly elevated alpha-fetoprotein is diagnostic. pp 956-977 3. Hepatic adenomas almost always occur in women of child-bearing age who are taking oral contraceptives. If the pathognomonic central scar can be identified on CT or MRI. a needle biopsy may be undertaken. American College of Surgeons SESAP 13 the arterial and portal venous phases of spiral CT. 1999 page 273 . Abdom Imaging 24:262-266. American College of Surgeons SESAP 13 Category 3 Item 82 Associated with multiple endocrine neoplasia type 1 (MEN1) (A) Gastric carcinoid (B) Small bowel carcinoid (C) Colon carcinoid (D) Appendiceal carcinoid (E) Rectal carcinoid page 274 . Type 2 tumors are often multiple and display a low grade of malignant behavior. in Clark O. REFERENCES 1. hypotension.Thompson GB. Elsevier Saunders. Prob Gen Surg 20:125-133. but lack the enzyme to convert it to the active form of serotonin. Gastric carcinoids secrete small amounts of 5-hydroxytryptamine.American College of Surgeons SESAP 13 Critique for Category 3 Item 82 There are three types of gastric carcinoids. Philadelphia. resulting in flushing. Type 3 carcinoids are the sporadic variety that are often large and metastatic. Chen H: An update on carcinoid tumors. and serotonin and urinary 5-hydroxyindoleacetic acid (5-HIAA) levels are often normal. About 50% of these tumors are diagnosed incidentally on routine endoscopy. Type 1 is the most common. Type 1 tumors tend to be nonfunctional and are believed to be due to overproduction of gastrin. Kebebew E (eds): Textbook of Endocrine Surgery. 2003 2. These tumors tend to be very large and are most commonly located in the right colon. Type 2 gastric carcinoids are rare and arise in the setting of multiple endocrine neoplasia type 1 (MEN1) combined with ZollingerEllison syndrome. 5-HIAA testing is very sensitive for the diagnosis of these lesions. The carcinoid syndrome is rare in patients with colonic. Small intestine is the most common location for carcinoids. appendiceal. and bronchial constriction. Approximately 70% of these tumors are metastatic at the time of presentation. Rectal carcinoids are becoming more and more common due to enhanced screening. accounting for 63% to 75% of all cases. with over half occurring in the distal ileum. and rectal carcinoids. It tends to develop in patients with type A chronic gastritis. ed 2. 2005. Duh QY. Colonic carcinoids are rare and typically present during the seventh decade of life. Young WF Jr: Multiple endocrine neoplasia type I. These may cause the atypical carcinoid syndrome.Sippel RS. pp 673-690 page 275 . American College of Surgeons SESAP 13 Category 3 Item 83 Atypical carcinoid syndrome (A) Gastric carcinoid (B) Small bowel carcinoid (C) Colon carcinoid (D) Appendiceal carcinoid (E) Rectal carcinoid page 276 . American College of Surgeons SESAP 13 Critique for Category 3 Item 83 There are three types of gastric carcinoids. The carcinoid syndrome is rare in patients with colonic. Kebebew E (eds): Textbook of Endocrine Surgery. Chen H: An update on carcinoid tumors. Duh QY. Rectal carcinoids are becoming more and more common due to enhanced screening. Type 2 gastric carcinoids are rare and arise in the setting of multiple endocrine neoplasia type 1 (MEN1) combined with ZollingerEllison syndrome. Prob Gen Surg 20:125-133. Type 1 is the most common. appendiceal. and rectal carcinoids. Young WF Jr: Multiple endocrine neoplasia type I. It tends to develop in patients with type A chronic gastritis.Thompson GB. ed 2. and serotonin and urinary 5-hydroxyindoleacetic acid (5-HIAA) levels are often normal. 2003 2. in Clark O. Gastric carcinoids secrete small amounts of 5-hydroxytryptamine.Sippel RS. Type 1 tumors tend to be nonfunctional and are believed to be due to overproduction of gastrin. About 50% of these tumors are diagnosed incidentally on routine endoscopy. Type 3 carcinoids are the sporadic variety that are often large and metastatic. accounting for 63% to 75% of all cases. Type 2 tumors are often multiple and display a low grade of malignant behavior. These may cause the atypical carcinoid syndrome. pp 673-690 page 277 . 5-HIAA testing is very sensitive for the diagnosis of these lesions. Colonic carcinoids are rare and typically present during the seventh decade of life. These tumors tend to be very large and are most commonly located in the right colon. Philadelphia. hypotension. with over half occurring in the distal ileum. Approximately 70% of these tumors are metastatic at the time of presentation. and bronchial constriction. but lack the enzyme to convert it to the active form of serotonin. Elsevier Saunders. REFERENCES 1. 2005. resulting in flushing. Small intestine is the most common location for carcinoids. American College of Surgeons SESAP 13 Category 3 Item 84 Urinary 5-hydroxyindoleacetic acid (5-HIAA) sensitive for diagnosis (A) Gastric carcinoid (B) Small bowel carcinoid (C) Colon carcinoid (D) Appendiceal carcinoid (E) Rectal carcinoid page 278 . and serotonin and urinary 5-hydroxyindoleacetic acid (5-HIAA) levels are often normal. in Clark O. resulting in flushing. ed 2. appendiceal. hypotension. Philadelphia. Rectal carcinoids are becoming more and more common due to enhanced screening. These may cause the atypical carcinoid syndrome. but lack the enzyme to convert it to the active form of serotonin. Gastric carcinoids secrete small amounts of 5-hydroxytryptamine. It tends to develop in patients with type A chronic gastritis. These tumors tend to be very large and are most commonly located in the right colon. Colonic carcinoids are rare and typically present during the seventh decade of life. accounting for 63% to 75% of all cases. Kebebew E (eds): Textbook of Endocrine Surgery. and rectal carcinoids. The carcinoid syndrome is rare in patients with colonic. Duh QY. Type 1 is the most common. 2005. Type 3 carcinoids are the sporadic variety that are often large and metastatic. Prob Gen Surg 20:125-133. Elsevier Saunders.Sippel RS. and bronchial constriction. Approximately 70% of these tumors are metastatic at the time of presentation.American College of Surgeons SESAP 13 Critique for Category 3 Item 84 There are three types of gastric carcinoids. Type 1 tumors tend to be nonfunctional and are believed to be due to overproduction of gastrin. 5-HIAA testing is very sensitive for the diagnosis of these lesions. with over half occurring in the distal ileum. Small intestine is the most common location for carcinoids. pp 673-690 page 279 . Chen H: An update on carcinoid tumors. Type 2 gastric carcinoids are rare and arise in the setting of multiple endocrine neoplasia type 1 (MEN1) combined with ZollingerEllison syndrome.Thompson GB. 2003 2. REFERENCES 1. About 50% of these tumors are diagnosed incidentally on routine endoscopy. Type 2 tumors are often multiple and display a low grade of malignant behavior. Young WF Jr: Multiple endocrine neoplasia type I. American College of Surgeons SESAP 13 Category 3 Item 85 Indicate(s) presence of hiatal hernia (A) Distal esophageal web (B) Cervical esophageal web (C) Both (D) Neither page 280 . or sideropenic dysphagia. Maier RV. but can be difficult to differentiate from stricture caused by gastroesophageal reflux when present. Bhutani M: Esophagus.Kucharczuk JC. Savage C. and patients are typically older edentulous women also suffering from atrophic oral mucosa. REFERENCES 1. Distal esophageal web.Hoffman RM. Treatment consists of correction of iron deficiency and dilation of cervical webs if present. an antireflux medical regimen should be added and if surgical therapy is necessary an antireflux operation should be performed in conjunction with either dilation or resection. Philadelphia. is cervical esophageal dysphagia associated with iron-deficiency anemia. ed 17. about 10% of patients develop squamous cell carcinoma of the hypopharynx. Lippincott Williams & Wilkins. Lillemoe KD. In the setting of reflux. and koilonychia (brittle. Jaffe PE: Plummer-Vinson syndrome. Kaiser LR: Esophageal injury. et al (eds): Greenfield's Surgery: Scientific Principles and Practice. oral cavity. A case report and literature review. spoon-shaped fingernails). and neoplasms. or Schatzki’s ring. Philadelphia. Dysphagia is usually present when the diameter of the ring is 13 mm or less as measured on barium esophagram. The condition is commonly associated with cervical esophageal webs. pp 691-708 3.American College of Surgeons SESAP 13 Critique for Category 3 Item 85 Plummer-Vinson syndrome. Saunders. in Townsend CM Jr (ed): Sabiston Textbook of Surgery: The Biological Basis of Modern Surgical Practice. ed 4. is an annular constriction of the esophagus that indicates the presence of a sliding hiatal hernia. diverticula. or esophagus. Plummer-Vinson syndrome is considered a premalignant condition. Schatzki’s ring typically projects into the esophageal lumen at a right angle and occurs at the squamocolumnar junction. Arch Intern Med 155:2008-2011. Doherty GM.Zwischenberger JB. 2006. Schatzki’s ring does not necessarily indicate the presence of gastroesophageal reflux or esophagitis. 2004. glossitis. It is most common in Scandinavia and Great Britain. Periodic esophageal dilation is effective when Schatzki’s ring is not associated with reflux. in Mulholland MW. pp 1091-1150 page 281 . 1995 2. American College of Surgeons SESAP 13 Category 3 Item 86 Associated with iron deficiency anemia (A) Distal esophageal web (B) Cervical esophageal web (C) Both (D) Neither page 282 . The condition is commonly associated with cervical esophageal webs. Doherty GM. and koilonychia (brittle. Distal esophageal web. 2004. Dysphagia is usually present when the diameter of the ring is 13 mm or less as measured on barium esophagram. in Townsend CM Jr (ed): Sabiston Textbook of Surgery: The Biological Basis of Modern Surgical Practice. Maier RV.Hoffman RM. pp 1091-1150 page 283 . In the setting of reflux. and patients are typically older edentulous women also suffering from atrophic oral mucosa. Philadelphia. 1995 2. Philadelphia. It is most common in Scandinavia and Great Britain. is an annular constriction of the esophagus that indicates the presence of a sliding hiatal hernia. Lillemoe KD. Kaiser LR: Esophageal injury. Jaffe PE: Plummer-Vinson syndrome. pp 691-708 3. in Mulholland MW. ed 17. 2006. Lippincott Williams & Wilkins. Periodic esophageal dilation is effective when Schatzki’s ring is not associated with reflux. about 10% of patients develop squamous cell carcinoma of the hypopharynx. is cervical esophageal dysphagia associated with iron-deficiency anemia. ed 4. and neoplasms. but can be difficult to differentiate from stricture caused by gastroesophageal reflux when present. Plummer-Vinson syndrome is considered a premalignant condition. REFERENCES 1. or sideropenic dysphagia.American College of Surgeons SESAP 13 Critique for Category 3 Item 86 Plummer-Vinson syndrome. or Schatzki’s ring. et al (eds): Greenfield's Surgery: Scientific Principles and Practice. an antireflux medical regimen should be added and if surgical therapy is necessary an antireflux operation should be performed in conjunction with either dilation or resection.Kucharczuk JC. A case report and literature review.Zwischenberger JB. spoon-shaped fingernails). Bhutani M: Esophagus. Arch Intern Med 155:2008-2011. oral cavity. glossitis. Savage C. diverticula. Schatzki’s ring does not necessarily indicate the presence of gastroesophageal reflux or esophagitis. Treatment consists of correction of iron deficiency and dilation of cervical webs if present. Saunders. or esophagus. Schatzki’s ring typically projects into the esophageal lumen at a right angle and occurs at the squamocolumnar junction. American College of Surgeons SESAP 13 Category 3 Item 87 Indicate(s) presence of gastroesophageal reflux disease (GERD) (A) Distal esophageal web (B) Cervical esophageal web (C) Both (D) Neither page 284 . Lillemoe KD. Jaffe PE: Plummer-Vinson syndrome. et al (eds): Greenfield's Surgery: Scientific Principles and Practice. in Townsend CM Jr (ed): Sabiston Textbook of Surgery: The Biological Basis of Modern Surgical Practice. about 10% of patients develop squamous cell carcinoma of the hypopharynx. It is most common in Scandinavia and Great Britain. but can be difficult to differentiate from stricture caused by gastroesophageal reflux when present.Kucharczuk JC. Kaiser LR: Esophageal injury. glossitis.Zwischenberger JB. 2004. A case report and literature review. Bhutani M: Esophagus. an antireflux medical regimen should be added and if surgical therapy is necessary an antireflux operation should be performed in conjunction with either dilation or resection. Treatment consists of correction of iron deficiency and dilation of cervical webs if present. Distal esophageal web. 1995 2. is an annular constriction of the esophagus that indicates the presence of a sliding hiatal hernia. and patients are typically older edentulous women also suffering from atrophic oral mucosa. Periodic esophageal dilation is effective when Schatzki’s ring is not associated with reflux. Plummer-Vinson syndrome is considered a premalignant condition. or sideropenic dysphagia. Doherty GM. and neoplasms. ed 17. Schatzki’s ring does not necessarily indicate the presence of gastroesophageal reflux or esophagitis. In the setting of reflux. REFERENCES 1.American College of Surgeons SESAP 13 Critique for Category 3 Item 87 Plummer-Vinson syndrome. diverticula. pp 691-708 3. or Schatzki’s ring. Saunders. ed 4. Maier RV. Schatzki’s ring typically projects into the esophageal lumen at a right angle and occurs at the squamocolumnar junction. is cervical esophageal dysphagia associated with iron-deficiency anemia. and koilonychia (brittle. Savage C. Dysphagia is usually present when the diameter of the ring is 13 mm or less as measured on barium esophagram. Lippincott Williams & Wilkins. spoon-shaped fingernails). oral cavity. or esophagus. Arch Intern Med 155:2008-2011. in Mulholland MW. 2006. Philadelphia. pp 1091-1150 page 285 . Philadelphia.Hoffman RM. The condition is commonly associated with cervical esophageal webs. American College of Surgeons SESAP 13 Category 3 Item 88 Nutcracker esophagus (A) Adenocarcinoma of the esophagus (B) Squamous cell carcinoma of the esophagus (C) Both (D) Neither page 286 . Nyren O: Symptomatic gastroesophageal reflux as a risk factor for esophageal adenocarcinoma. gastroesophageal junction.8% to 4%. Sorensen HR: Does achalasia predispose to cancer of the esophagus? Chest 102:1013-1016. Lindgren A. although one study found a 7% rate. but now adenocarcinoma is seen more frequently.Appelqvist P.American College of Surgeons SESAP 13 Critique of Category 3 Item 88 Most cancers involving the esophagus are either adenocarcinoma or squamous cell carcinoma. squamous cell carcinoma of the esophagus is believed to be associated with tobacco and alcohol use. Bergstrom R. even decades. 1992 2. One reason that adenocarcinoma is becoming more common is believed to be its association with gastroesophageal reflux disease (GERD). The interval between diagnosis and treatment of the achalasia and the development of the cancer is many years. This increased risk persists even after successful surgical treatment of the achalasia. to carcinoma in situ. Population-based studies have demonstrated that the longer the duration of GERD symptoms and the more severe. on average 6 years. At one time. to high-grade dysplasia. Like other squamous carcinomas of the upper aerodigestive tract. N Engl J Med 340:825-831. a change of the normal squamous epithelium of the lower esophagus to intestinal metaplasia. Lye ingestion and the subsequent stricture and architectural damage of the esophagus have been associated with the development of squamous cell carcinoma of the esophagus. GERD is associated with Barrett’s esophagus. Follow-up is difficult because persistent symptoms mimicking carcinoma are common after the lye ingestion and the radiographic appearance is similar to a cancer. the higher the risk of adenocarcinoma of the lower esophagus. The motility disorder of achalasia is associated with an increased risk of developing squamous cell carcinoma of the mid-esophagus. Holm JC. 1980 3.Lagergren J. The incidence of squamous cell cancer in these patients is 0. Cancer 45:2655-2658. squamous cell carcinoma was the most common type. and gastric cardia. the interval between ingestion and development of the cancer is usually measured in years. Salmo M: Lye corrosion carcinoma of the esophagus: a review of 63 cases. Barrett’s metaplasia is believed to progress in steps to low-grade dysplasia. achalasia patients need to be continuously monitored. No other motility disorder is associated with increased risk of carcinoma of the esophagus. 1999 page 287 . As with achalasia. and the incidence of GERD has increased greatly in recent years. REFERENCES 1. As with Barrett’s esophagus.Aggestrup S. and finally to invasive adenocarcinoma. American College of Surgeons SESAP 13 Category 3 Item 89 Gastroesophageal reflux disease (A) Adenocarcinoma of the esophagus (B) Squamous cell carcinoma of the esophagus (C) Both (D) Neither page 288 . to high-grade dysplasia. As with Barrett’s esophagus.Appelqvist P. GERD is associated with Barrett’s esophagus. 1980 3. 1992 2. At one time. As with achalasia. and finally to invasive adenocarcinoma. and the incidence of GERD has increased greatly in recent years. The incidence of squamous cell cancer in these patients is 0.Aggestrup S. Nyren O: Symptomatic gastroesophageal reflux as a risk factor for esophageal adenocarcinoma. a change of the normal squamous epithelium of the lower esophagus to intestinal metaplasia. 1999 page 289 .8% to 4%. Sorensen HR: Does achalasia predispose to cancer of the esophagus? Chest 102:1013-1016. even decades. achalasia patients need to be continuously monitored. squamous cell carcinoma was the most common type. Lindgren A. The interval between diagnosis and treatment of the achalasia and the development of the cancer is many years. No other motility disorder is associated with increased risk of carcinoma of the esophagus. and gastric cardia. One reason that adenocarcinoma is becoming more common is believed to be its association with gastroesophageal reflux disease (GERD). to carcinoma in situ. the higher the risk of adenocarcinoma of the lower esophagus. on average 6 years. The motility disorder of achalasia is associated with an increased risk of developing squamous cell carcinoma of the mid-esophagus. although one study found a 7% rate. Cancer 45:2655-2658. the interval between ingestion and development of the cancer is usually measured in years. Holm JC. Like other squamous carcinomas of the upper aerodigestive tract. Bergstrom R. Population-based studies have demonstrated that the longer the duration of GERD symptoms and the more severe. Barrett’s metaplasia is believed to progress in steps to low-grade dysplasia. but now adenocarcinoma is seen more frequently. REFERENCES 1.Lagergren J. Lye ingestion and the subsequent stricture and architectural damage of the esophagus have been associated with the development of squamous cell carcinoma of the esophagus. Follow-up is difficult because persistent symptoms mimicking carcinoma are common after the lye ingestion and the radiographic appearance is similar to a cancer. gastroesophageal junction. This increased risk persists even after successful surgical treatment of the achalasia. squamous cell carcinoma of the esophagus is believed to be associated with tobacco and alcohol use.American College of Surgeons SESAP 13 Critique for Category 3 Item 89 Most cancers involving the esophagus are either adenocarcinoma or squamous cell carcinoma. Salmo M: Lye corrosion carcinoma of the esophagus: a review of 63 cases. N Engl J Med 340:825-831. American College of Surgeons SESAP 13 Category 3 Item 90 Lye ingestion (A) Adenocarcinoma of the esophagus (B) Squamous cell carcinoma of the esophagus (C) Both (D) Neither page 290 . achalasia patients need to be continuously monitored. GERD is associated with Barrett’s esophagus. and finally to invasive adenocarcinoma. 1992 2. 1980 3. even decades. squamous cell carcinoma of the esophagus is believed to be associated with tobacco and alcohol use. but now adenocarcinoma is seen more frequently. The incidence of squamous cell cancer in these patients is 0. Follow-up is difficult because persistent symptoms mimicking carcinoma are common after the lye ingestion and the radiographic appearance is similar to a cancer. to carcinoma in situ. At one time.Aggestrup S. Salmo M: Lye corrosion carcinoma of the esophagus: a review of 63 cases. N Engl J Med 340:825-831. the interval between ingestion and development of the cancer is usually measured in years. The interval between diagnosis and treatment of the achalasia and the development of the cancer is many years. on average 6 years. This increased risk persists even after successful surgical treatment of the achalasia. Nyren O: Symptomatic gastroesophageal reflux as a risk factor for esophageal adenocarcinoma. and gastric cardia. Bergstrom R.American College of Surgeons SESAP 13 Critique for Category 3 Item 90 Most cancers involving the esophagus are either adenocarcinoma or squamous cell carcinoma. squamous cell carcinoma was the most common type. No other motility disorder is associated with increased risk of carcinoma of the esophagus. Barrett’s metaplasia is believed to progress in steps to low-grade dysplasia. REFERENCES 1. As with Barrett’s esophagus. As with achalasia. Lindgren A. and the incidence of GERD has increased greatly in recent years. One reason that adenocarcinoma is becoming more common is believed to be its association with gastroesophageal reflux disease (GERD). gastroesophageal junction. Population-based studies have demonstrated that the longer the duration of GERD symptoms and the more severe. Lye ingestion and the subsequent stricture and architectural damage of the esophagus have been associated with the development of squamous cell carcinoma of the esophagus. Like other squamous carcinomas of the upper aerodigestive tract. Sorensen HR: Does achalasia predispose to cancer of the esophagus? Chest 102:1013-1016.Lagergren J. a change of the normal squamous epithelium of the lower esophagus to intestinal metaplasia.8% to 4%. 1999 page 291 .Appelqvist P. Holm JC. to high-grade dysplasia. although one study found a 7% rate. the higher the risk of adenocarcinoma of the lower esophagus. Cancer 45:2655-2658. The motility disorder of achalasia is associated with an increased risk of developing squamous cell carcinoma of the mid-esophagus. American College of Surgeons SESAP 13 Category 3 Item 91 Diagnosis of active infection (A) Helicobacter pylori serology (B) H pylori stool antigen assay (C) Both (D) Neither page 292 . breath. As more patients are treated for H pylori infection. uses techniques that show active infection with the H pylori organism. or urine for an IgG antibody to H pylori.Vaira D. Ricci C. and a positive serology test may only indicate past infection. Serologic testing is the preferred method because it is easier and less expensive. the IgG antibodies remain after infection has been eradicated.Vaira D. et al: Peptic ulcer and Helicobacter pylori: update on testing and treatment. Treatment of the organism significantly decreases the incidence of gastric and duodenal ulcers and their associated complications. 2005 2. Gut 48:287-289. Tampieri A. the distinction between active and passive tests is important to avoid over-treatment with antibiotics in patients who are not actively infected with H pylori. stool. diagnosis was based on invasive testing involving endoscopy and gastric biopsy. Postgrad Med 117:17-22. However. J Pediatr 146:S21S26. including the urea breath test and the stool antigen test. 2001 page 293 . Gatta L. money.American College of Surgeons SESAP 13 Critique for Category 3 Item 91 Helicobacter pylori has been firmly established as a cause for peptic ulcer disease.Czinn SJ: Helicobacter pylori infection: detection. relying on the urease produced by the H pylori organism to split this carbon off. which can then be detected in the breath. 2005 3. Passive testing. and management. The urea breath test involves the ingestion of 13-C labeled urea. Vakil N: Blood. Active or passive noninvasive testing for H pylori infection is now available. and Helicobacter pylori. Initially. saliva. The stool antigen immunoassay detects an antigen on H pylori in the stool. Active testing. which involves the evaluation of serum. REFERENCES 1. urine. These tests can also be used to determine when H pylori has been eradicated. is useful during active infection. 46. investigation. American College of Surgeons SESAP 13 Category 3 Item 92 Diagnosis of previous infection (A) Helicobacter pylori serology (B) H pylori stool antigen assay (C) Both (D) Neither page 294 . 2005 2. 2005 3. breath. and Helicobacter pylori. Passive testing. Treatment of the organism significantly decreases the incidence of gastric and duodenal ulcers and their associated complications. relying on the urease produced by the H pylori organism to split this carbon off. Vakil N: Blood. REFERENCES 1. Initially.Czinn SJ: Helicobacter pylori infection: detection. Ricci C. diagnosis was based on invasive testing involving endoscopy and gastric biopsy. However. 2001 page 295 . money. Serologic testing is the preferred method because it is easier and less expensive. et al: Peptic ulcer and Helicobacter pylori: update on testing and treatment. The urea breath test involves the ingestion of 13-C labeled urea. Gatta L. Postgrad Med 117:17-22.Vaira D. Tampieri A. which can then be detected in the breath. or urine for an IgG antibody to H pylori. stool. Active testing. The stool antigen immunoassay detects an antigen on H pylori in the stool. which involves the evaluation of serum. including the urea breath test and the stool antigen test. Gut 48:287-289. the distinction between active and passive tests is important to avoid over-treatment with antibiotics in patients who are not actively infected with H pylori. These tests can also be used to determine when H pylori has been eradicated. uses techniques that show active infection with the H pylori organism.Vaira D. the IgG antibodies remain after infection has been eradicated. 46. saliva.American College of Surgeons SESAP 13 Critique for Category 3 Item 92 Helicobacter pylori has been firmly established as a cause for peptic ulcer disease. J Pediatr 146:S21S26. Active or passive noninvasive testing for H pylori infection is now available. is useful during active infection. investigation. and management. As more patients are treated for H pylori infection. urine. and a positive serology test may only indicate past infection. American College of Surgeons SESAP 13 Category 3 Item 93 Diagnosis of eradication (A) Helicobacter pylori serology (B) H pylori stool antigen assay (C) Both (D) Neither page 296 . and management.Czinn SJ: Helicobacter pylori infection: detection. including the urea breath test and the stool antigen test. urine. et al: Peptic ulcer and Helicobacter pylori: update on testing and treatment. J Pediatr 146:S21S26. saliva. which can then be detected in the breath. As more patients are treated for H pylori infection. uses techniques that show active infection with the H pylori organism. 2005 2. money. Vakil N: Blood. Postgrad Med 117:17-22. Gut 48:287-289. REFERENCES 1. or urine for an IgG antibody to H pylori. Gatta L. The stool antigen immunoassay detects an antigen on H pylori in the stool. and a positive serology test may only indicate past infection. investigation. Ricci C. These tests can also be used to determine when H pylori has been eradicated. the IgG antibodies remain after infection has been eradicated. stool. diagnosis was based on invasive testing involving endoscopy and gastric biopsy. is useful during active infection. relying on the urease produced by the H pylori organism to split this carbon off. 46.American College of Surgeons SESAP 13 Critique for Category 3 Item 93 Helicobacter pylori has been firmly established as a cause for peptic ulcer disease. and Helicobacter pylori. which involves the evaluation of serum. Initially. Tampieri A. breath. 2001 page 297 . However. 2005 3.Vaira D. The urea breath test involves the ingestion of 13-C labeled urea. the distinction between active and passive tests is important to avoid over-treatment with antibiotics in patients who are not actively infected with H pylori.Vaira D. Serologic testing is the preferred method because it is easier and less expensive. Treatment of the organism significantly decreases the incidence of gastric and duodenal ulcers and their associated complications. Active testing. Passive testing. Active or passive noninvasive testing for H pylori infection is now available. American College of Surgeons SESAP 13 Category 4 Item 1 Which of the following is the most accurate predictor of increased risk for morbidity and morality in a cirrhotic after abdominal operation? (A) Child’s class (B) Child-Turcotte-Pugh score (C) Model end-stage liver disease (MELD) score (D) APACHE II (E) APACHE III page 298 . Palmer DE. REFERENCES 1. APACHE II and APACHE III scoring systems in predicting hospital mortality of patients with liver cirrhosis. APACHE II and III scores were used to predict short-term mortality in 200 hospital admissions of cirrhotic patients.Chatzicostas C. The MELD score is calculated from serum creatinine. Vlachonikolis IG. The Model for EndStage Liver Disease score (MELD) was created to estimate survival of patients undergoing the transjugular intrahepatic portosystemic shunt (TIPS) procedure.unos. the CTP score was a better predictor of mortality. Developed as systems to predict prognosis in critically ill patients. Arch Surg 140: 650-654. 2005 2. but has been found to be reliable enough to be used as the scoring system for allocation of livers for transplantation. a not surprising finding considering that the CTP system was developed specifically to assess the patients with cirrhosis. The MELD score has been confirmed to be more precise in categorizing risks of cirrhotic patients for both hepatic and other abdominal operations and to be superior to the CTP score. The MELD score is the most precise tool to assess operative risk in this patient population. et al: A comparison of Child-Pugh. Notas G. These developments result in surgeons increasingly facing a need to assess the risk of an elective or emergency operation.asp?index=98. and etiology of cirrhosis. 17 for APACHE III) and specific parameters of liver function (bilirubin and albumin) are included only in APACHE III. Although the formula is unwieldy to calculate. More precise risk assessment systems than the CTP score have been developed both within the field of hepatology and in the critical care realm to help guide patient care decisions. This assessment often includes deciding whether the expected benefit outweighs the risk of the procedure. total serum bilirubin. The high prevalence of hepatitis C infection in the population and more success with anti-viral treatment is producing an ever-increasing number and age of cirrhotics. When CTP. however. Roussomoustakaki M. the APACHE (Acute Physiology. 2003 page 299 . et al: The safety of intra-abdominal surgery in patients with cirrhosis. an online worksheet is available at http://www. international normalized ratio (INR) for prothrombin time. Longo W. However. BMC Gastroenterol 3:7.Befeler A. with CTP class C patients having prohibitive risk for most operations. Early studies identified that the Child’s class or the Child-Turcotte-Pugh (CTP) score correlated with the risk of abdominal operations in cirrhotics.American College of Surgeons SESAP 13 Critique for Category 4 Item 1 Cirrhosis of the liver is associated with increased morbidity and mortality when a patient must undergo operation. the risk varies markedly depending on the severity of the cirrhosis. and is superior to the CTP score. Although studies from 20 years ago found a 30% mortality in cirrhotics undergoing abdominal operation. These systems include a large number of physiologic variables (12 for APACHE II. they do contain subjective elements such as degree of ascites and hepatic encephalopathy that may decrease their precision. The Child’s class and CTP score are generally reliable predictors of mortality. given that anesthesia and the physiologic exacerbations produced by an operation may precipitate hepatic decompensation.org/resources/MeldPeldCalculator. Age and Chronic Health Evaluation) II and III scores have been found to apply to patients with a broad range of conditions admitted to the ICU. Hoffman M. The model was derived from data from patients at four US medical centers and validated from an independent group of Dutch patients. Farnsworth N. Awad SS: Child-Turcotte-Pugh versus MELD score as a predictor of outcome after elective and emergent surgery in cirrhotic patients. et al: A model to predict survival in patients with end-stage liver disease. Fagan SP. Hepatology 33:464-470. Wiesner RH. Kremers W. Malinchoc M. 2001 page 300 . Am J Surg 188:580583. 2004 4.American College of Surgeons SESAP 13 3.Kamath PS. Berger DH. American College of Surgeons SESAP 13 Category 4 Item 2 Which of the following statements about bile duct injury during laparoscopic cholecystectomy is NOT true? (A) The incidence of bile duct injury requiring reconstruction is higher after laparoscopic than open cholecystectomy (B) Use of intraoperative cholangiography is associated with a decreased rate of bile duct injury during laparoscopic cholecystectomy (C) Long-term results are best when biliary reconstruction of a laparoscopic bile duct injury is performed by a surgeon who specializes in hepatobiliary surgery (D) Major biliary injury is one of the most commonly litigated surgical complications (E) Bile duct injuries are preventable if basic principles for performing laparoscopic cholecystectomy are followed page 301 . 5%. REFERENCES 1. et al: Common bile duct injury during laparoscopic cholecystectomy and the use of intraoperative cholangiography: adverse outcome or preventable error? Arch Surg 136:1287-1292. Whereas major bile duct injury occurred in only about 0. 2005 3. 1998 2. careful dissection of the gallbladder cystic duct junction.Flum DR. laparoscopic equipment was developed with advanced optics and more sophisticated instruments. The need for biliary reconstruction confers a significant risk of anastomotic stricture requiring secondary surgical or radiologic interventions. Way LW: Bile duct injuries during laparoscopic cholecystectomy. Phillips EH: Common bile duct injuries during laparoscopic cholecystectomy that result in litigation. Koepsell T. Diggs BS.1%. conversion to an open cholecystectomy is advised. Intraoperative cholangiography is associated with a decreased rate of significant bile duct injury. Birth M. Sheppard BC. As surgeons developed more experience performing the procedure. Additionally. Arch Surg 130:1123-1128. such as that using the Nationwide Inpatient Sample of over 7 million patient records per year. If a significant bile duct injury requiring biliary reconstruction does occur. If anatomy is unclear at any time during the dissection. Hunter JG: Ten-year trend in the national volume of bile duct injuries requiring operative repair. bile duct injuries are one of the most commonly litigated surgical procedures. The most recent population-based studies. and use of cholangiography before dividing any structures. details of how to perform it safely were described and disseminated. the mortality of the procedure increases tenfold from 0. The incidence of significant major bile duct injuries slowly declined but has not entirely disappeared. 2001 4. Although it is rare.45% to 4.4% during the laparoscopic procedures. Because most bile duct injuries are perceived to be preventable. outcome analysis demonstrates that experienced hepatobiliary surgeons obtain superior results. 1995 page 302 .Carroll BJ. Key maneuvers include traction laterally at the gallbladder base to open up Calot’s triangle. Factors that influence the results of treatment. the incidence of significant bile duct injury during cholecystectomy initially increased significantly. Studies analyzing the mechanism of bile duct injuries have resulted in a set of guidelines that will help prevent such injuries. and secondary biliary cirrhosis can develop.American College of Surgeons SESAP 13 Critique for Category 4 Item 2 When laparoscopic cholecystectomy was first introduced in the late 1980s. estimate the incidence of bile duct injury requiring ductal reconstruction to be identical to that for open cholecystectomy at 0. bile duct injury may eventually require liver transplantation. Heagerty P. some series reported a stated incidence of as high as 1.Stewart L.1% of patients during open cholecystectomy. Surg Endosc 19: 967-973. Sinanan M. Surg Endosc 12: 310-313.Dolan JP. When significant bile duct injury does occur during either open or laparoscopic cholecystectomy. retraction of the fundus of the gallbladder cranially. American College of Surgeons SESAP 13 Category 4 Item 3 A 70-year-old woman presents with right upper quadrant pain. The next procedure should be (A) laparoscopic cholecystectomy (B) open cholecystectomy (C) percutaneous biopsy of the gallbladder (D) open cholecystectomy with en bloc partial hepatectomy (E) cholecystostomy page 303 . The ultrasound and computed tomographic (CT) scan shown are obtained. Cholecystectomy alone is appropriate only for stage I gallbladder carcinomas.Jarnagin WR.Weber SM. Cancer 98:1689-1700. is the only potentially curative therapy. However. DeMatteo RP. Fong Y.de Groen PC. LaRusso NF. Blumgart LH. N Engl J Med 341:1368-1378. Analysis of 100 patients. Klimstra D. and open cholecystectomy with en bloc partial hepatectomy should be the next procedure. which are usually incidental findings. These findings are consistent with advanced gallbladder malignancy. et al: Biliary tract cancers. which will be present in over 30% of patients with gallbladder cancer. et al: Staging laparoscopy in patients with extrahepatic biliary carcinoma. Ruo L. Laparoscopy is appropriate to rule out peritoneal carcinomatosis. 2003 3. et al: Patterns of initial disease recurrence after resection of gallbladder carcinoma and hilar cholangiocarcinoma: implications for adjuvant therapeutic strategies. Radical cholecystectomy. 2002 page 304 . Percutaneous drainage of the gallbladder or operative cholecystostomy would be indicated only for patients with significant symptoms and a prohibitively high operative risk. a negative biopsy of the gallbladder should not dissuade the surgeon from radical resection with the findings shown here and is superfluous. with en bloc resection of liver segments IVB and V and regional lymph nodes. 1999 2. Gores GJ. Ann Surg 235:392-399.American College of Surgeons SESAP 13 Critique for Category 4 Item 3 The computed tomographic (CT) scan shows asymmetrical gallbladder wall thickening and no evidence of pericholecystic inflammation. Percutaneous biopsy of the gallbladder should be reserved for evaluating any suspected metastases seen on axial imaging. REFERENCES 1. Gunderson LL. Gallbladder cancer is more common in older women and in patients with stones. Little SA. An elevated serum CA19-9 may support a diagnosis of cancer. The ultrasound study reveals intraluminal masses and gallbladder wall thickening. American College of Surgeons SESAP 13 Category 4 Item 4 A 40-year-old previously healthy man presents with epigastric abdominal pain and cyclical fevers. The computed tomographic (CT) scan shown is obtained. Which of the following statements is TRUE? (A) Percutaneous drainage is associated with a shorter hospital stay compared with surgical drainage (B) This lesion cannot be successfully treated with percutaneous drainage (C) Percutaneous sclerotherapy is appropriate treatment (D) 30% of patients will have no clearly identifiable etiology (E) Oral metronidazole therapy will cure this process page 305 . in Brunicardi FC. Staphylococcus and Streptococcus species are more common. hepatosplenic candidiasis must be considered along with more conventional pyogenic etiologies. and the remaining cases are culture negative. Commonly encountered organisms include Escherichia coli. gram-positive aerobes. In patients with solitary abscesses. New York. Curr Surg 58:381-384. but most amoebic abscesses are large and solitary. TD.Balint TD. Mendelson KG. and anaerobic or facultative anaerobic species such as Bacteroides fragilis. Interestingly. Rarely. in patients with recalcitrant lesions. The clinical presentation of patients with pyogenic liver abscess is rarely subtle. and repeated catheter interventions may be required to obtain satisfactory drainage. laparoscopic or open surgical drainage may be required if percutaneous drainage fails. The placement of a percutaneous drainage catheter at the time of aspiration is beneficial for patients with a complex abscess or an abscess containing particularly thick fluid. and anaerobes are the predominant organisms found in liver abscesses. based on the presumed etiology of the primary infection and subsequently tailored based on the culture results after aspiration of the abscess. Percutaneous and surgical drainage are equally efficacious in achieving resolution of the infection. aspiration alone may be sufficient if the abscess can be drained. Right upper quadrant abdominal pain. Although percutaneous drainage is usually associated with less morbidity than surgical drainage. surgical intervention is typically unnecessary. Klebsiella pneumoniae. McGraw-Hill. 2005. In patients with systemic infections from subacute bacterial endocarditis and indwelling catheter infections. In an immunosuppressed patient who has multiple abscesses. a primary source of infection will not be identified in as many as one third of patients with pyogenic liver abscesses. However. Chung AY. Pofahl W: Hepatic abscess: Current concepts in diagnosis and treatment. Chow PK. Enterococcus faecalis. Curley SA: Liver. Bailey BM. Antimicrobial therapy should be empiric. 2001 2.Sielaff. In patients with intra-abdominal sources leading to hepatic abscesses. pp 1139-1186 3. et al: An appraisal of surgical and percutaneous drainage for pyogenic liver abscesses larger than 5 cm. and systemic infections such as bacterial endocarditis. Because percutaneous aspiration is effective in 80% to 90% of patients. Ann Surg 241: 485-490. Oral metronidazole therapy would be used for patients with amoebic abscesses. Dunn DL.Tan YM. 2005 page 306 . Cheow PC. Monomicrobial abscesses are found in 40% of patients. formal liver resection may be necessary. and occasionally jaundice are present. Billiar TR. diverticular disease. Andersen DK. it is associated with longer hospitalizations. et al (eds): Schwartz’s Principles of Surgery.American College of Surgeons SESAP 13 Critique for Category 4 Item 4 The common causes of pyogenic liver abscess include biliary tract manipulation. Percutaneous sampling is occasionally not possible in patients presenting with multiple small abscesses. fever. though it has been considered as initial therapy for symptomatic liver cysts and echinococcal cysts. ed 8. inflammatory bowel disease. 40% are polymicrobial. REFERENCES 1. gram-negative aerobes. Percutaneous sclerotherapy is not appropriate for the management of abscesses. Liver test abnormalities if present are mild. Abdominal examination is unremarkable. scan (C) endoscopic ultrasound study (D) laparoscopic cholecystectomy (E) open cholecystectomy page 307 . Right upper quadrant ultrasound examination reveals a 6-mm fixed. eg. HIDA.American College of Surgeons SESAP 13 Category 4 Item 5 A 51-year-old man has intermittent bloating and epigastric pain associated with eating fried foods. The preferred management at this time would be (A) repeat ultrasound study in 3 months (B) biliary excretion. solitary projection in the gallbladder lacking an acoustic shadow. Philadelphia. ed 8. Li JC. Mosby. pp 439-443 page 308 . The presence of a risk factor for malignancy. Pitt HA: Biliary tract.Lee KF. Wong J. lack displacement. Treatment should take such risk factors into account (see algorithm). Asymptomatic polyps < 10 mm can generally be observed with serial ultrasound studies every 3 to 6 months within the first few years. 2004 3. the presence of symptoms. pp 1597-1642 2. Spillage of gallbladder contents should be avoided. they can be removed laparoscopically. Because most malignant polyps < 18 mm usually do not invade deeper than the muscle layer of the gallbladder wall. but they can harbor malignancy in up to 8% of cases (see table). Most are cholesterol polyps. concomitant gallstones. Risk factors for malignancy include age > 50. Lai PB: Polypoid lesions of the gallbladder. in Townsend CM Jr (ed): Sabiston Textbook of Surgery: The Biological Basis of Modern Surgical Practice. and do not have an acoustic shadow. Philadelphia.Russell SE. Any significant increase in size should prompt removal of the gallbladder. Age and size appear to be the two most important factors. 2004. 2004. in Cameron JL (ed): Current Surgical Therapy. On ultrasound study. should also prompt excision of polyps < 10 mm. Am J Surg 188:186-190. An open approach should be used for lesions > 18 mm. and solitary polyps. size > 10 mm. Zinner MJ: Tumors of the gallbladder. especially age > 50. All symptomatic polyps and polyps > 10 mm should also be removed. Saunders. REFERENCES 1. ed 17. appear fixed.American College of Surgeons SESAP 13 Critique for Category 4 Item 5 Polypoid lesions of the gallbladder are present in 3% to 7% of patients undergoing ultrasound evaluation of the gallbladder and in 2% to 12% of cholecystectomy specimens. they have the same echogenicity as the gallbladder wall.Ahrendt SA. American College of Surgeons SESAP 13 Category 4 Item 6 A 24-year-old woman with a warm antibody autoimmune hemolytic anemia that has not responded to medical therapy is scheduled for laparoscopic splenectomy. The most likely location for an accessory spleen would be the (A) splenic hilum (B) small bowel mesentery (C) splenocolic ligament (D) tail of the pancreas (E) greater omentum page 309 . ed 8. If splenectomy is indicated as treatment for hemolytic anemia. small bowel mesentery. REFERENCES 1. chap 5. New York. et al (eds): ACS Surgery: Principles and Practice 2006. McGraw-Hill.Park AE. then a thorough search for accessory spleens must be accomplished. and ovary (see diagram. WebMD. occurring in up to 20% of the population.American College of Surgeons SESAP 13 Critique for Category 4 Item 6 Accessory spleen is a common anomaly. Andersen DK. Jurkovich GJ.Schlachta CM. pp 1297-1316 2. Gastrointestinal Tract and Abdomen. Approximately 80% of accessory spleens are located within the splenic hilum or its vascular pedicle. especially in idiopathic thrombocytopenic purpura (ITP). in Brunicardi FC. tail of the pancreas. splenocolic ligament. A diligent search for any accessory spleens is prudent whenever splenectomy is performed for hematologic disease. Accessory spleens can lead to relapse. New York. 2005. It can occur in up to 30% of patients with a hematologic disorder. Kaiser LR. Dunn DL. et al (eds): Schwartz's Principles of Surgery. pp 719-736 page 310 . in Souba WW. picture). Fink MP. Other areas include the greater omentum. sec 25. Billiar TR. McKinlay R: Spleen. Mamazza J: Splenectomy. His platelet count is 20.000/mm3. A course of corticosteroid therapy produced no response. He is HIV negative. Which of the following BEST predicts that he will have a favorable response to splenectomy? (A) Age (B) Response to glucocorticoids (C) HIV status (D) Platelet count (E) Time since diagnosis page 311 .American College of Surgeons SESAP 13 Category 4 Item 7 A 23-year-old man is scheduled for splenectomy 1 year after diagnosis of idiopathic thrombocytopenic purpura (ITP). Duperier T. Several preoperative indicators have been suggested for predicting a positive response to splenectomy. Brody F. Walsh RM. Initial therapy consists of oral glucocorticoids. high preoperative platelet count. in Brunicardi FC. REFERENCES 1. pp 1297-1316 page 312 . and surgical complications. 2004 3. 2004 2. Arch Surg 139:61-66. Felsher J. 2005. ITP can be associated with other chronic immune conditions such as HIV infection. Billiar TR. but data are mixed at best. prediction of response. George JN: Splenectomy for adult patients with idiopathic thrombocytopenic purpura: a systematic review to assess long-term platelet count responses. A recently developed prediction score for successful splenectomy based on age and preoperative platelet count found them to be predictive of a positive response on multivariate analysis. positive HIV status. Andersen DK. Terrell DR. It is usually a diagnosis of exclusion after other causes have been ruled out. ed 8. it tends to affect more women than men.American College of Surgeons SESAP 13 Critique of Category 4 Item 7 Idiopathic thrombocytopenic purpura (ITP) is an autoimmune disorder resulting in low platelet counts secondary to the development of IgG antiplatelet antibodies. and young age. including short interval between diagnosis and operation. et al (eds): Schwartz's Principles of Surgery. McKinlay R: Spleen. Vesely SK. initial response to glucocorticoid therapy. McGraw-Hill. A systematic review of 56 case series that included 4146 patients found that young age was the most commonly encountered positive predictor. et al: Predictive factors for successful laparoscopic splenectomy in patients with immune thrombocytopenic purpura. Splenectomy is indicated for failure of medical management. Dunn DL.Park AE. Blood 104:2623-2634. New York.Kojouri K. Intravenous immunoglobin is usually reserved for refractory cases associated with bleeding. In adults. American College of Surgeons SESAP 13 Category 4 Item 8 Which of the following statements about exposure of the thoracolumbar spine is TRUE? (A) One vertebral body above and below the fracture site must be exposed (B) A right-sided thoracoabdominal incision is preferred to a left-sided approach to reduce the risk of injury to the thoracoabdominal aorta (C) The supine position should be used to facilitate exposure of the anterior spinal ligament (D) Retrograde ejaculation is identified in more than 25% of patients (E) An extrapleural approach versus conventional transthoracic incisions has been associated with an increased risk of persistent postoperative pain page 313 . Injury to intercostal nerves can lead to a bulge in the abdominal wall adjacent to the incision. Diekerhof CH. et al: The anterior extrapleural approach to the thoracolumbar junction revisited. Webb JW. 2004 page 314 . complications and outcome.Barone GW.American College of Surgeons SESAP 13 Critique for Category 4 Item 8 During the last decade. van der Tweel I. Injury to the sympathetic nerves can lead to retrograde ejaculation. it is important to achieve sufficient exposure to allow appropriate fixation of the orthopaedic hardware to vertebral bodies both above and below the site of injury. et al: Surgical treatment of traumatic fractures of the thoracic and lumbar spine: a systematic review of the literature on techniques. but it is uncommon. 1998 2. Eidt JF. In general. Care must be taken to avoid injury to the ipsilateral ureter during the retroperitoneal dissection. An extrapleural approach may reduce the incidence of postoperative pulmonary complications. The patient is placed in a modified right lateral decubitus position to facilitate medial rotation of the abdominal viscera. REFERENCES 1. The left thoracoabdominal approach is selected in most cases to avoid the more easily injured vena cava. Buskens E. Spine 29:803-814. Am Surg 64:372-375.Verlaan JJ. General and vascular surgeons are increasingly called on to perform the exposure of the lower thoracic and abdominal spine though a thoracoabdominal exposure. Hudec WA. spine surgeons have increasingly used direct fixation of the thoracolumbar spine to treat both acute and chronic conditions. The obstruction is a firm intraluminal mass. there is an inflammatory mass in the right upper quadrant and a point of bowel obstruction in the distal ileum 10 cm proximal to the ileocecal valve. distention. She had 2 episodes of emesis within the last 24 hours. and obstipation. At laparotomy. cholecystectomy (E) ileocecectomy with primary anastomosis page 315 . cholecystostomy (B) small bowel resection with primary anastomosis (C) enterotomy with removal of the mass (D) enteroenterostomy to bypass the mass. An abdominal series reveals pneumobilia. She had an appendectomy during childhood. Of the following.American College of Surgeons SESAP 13 Category 4 Item 9 A 75-year-old woman presents with a 3-day history of crampy abdominal pain. the best management would be (A) small bowel resection with ileostomy. pp 1187-1220 page 316 . Billiar TR. but stones that cause gallstone ileus are almost always 2. The proximal intestine must be carefully inspected for the presence of a second calculus that might cause a postoperative recurrence. The gallbladder should be left undisturbed at the original operation. An enteroenterostomy would relieve the obstruction.American College of Surgeons SESAP 13 Critique for Category 4 Item 9 Gallstone ileus is mechanical intestinal obstruction caused by a large gallstone lodged in the lumen. plain films of the abdomen may show a radiopaque gallstone. New York. the ectopic stone can be a puzzling finding. in Townsend CM Jr (ed): Sabiston Textbook of Surgery: The Biological Basis of Modern Surgical Practice. pp 573-601 3. In most patients. rarely. Pitt HA: Biliary Tract. 2006. Andersen DK. et al (eds): Schwartz's Principles of Surgery. In about 40% of cases. interval cholecystectomy will be required in about 30% of patients. ed 8. Hunter JG: Gallbladder and the extrahepatic biliary system. The gallbladder may contain one or several stones. Both are associated with increased morbidity and mortality in the elderly patient. McGraw-Hill. ed 17. and the average age is about 70. a manifestation of the cholecystoenteric fistula. or. The obstructing gallstone enters the intestine through a cholecystoenteric fistula located in the duodenum. If the obstructing stone became dislodged. or may occur even after the stone has traversed the entire small bowel. Philadelphia. it might cause another intestinal obstruction. 2005. the findings on physical examination are typical of distal small bowel obstruction.Doherty GM. Saunders. The fistula itself is rarely the source of trouble and closes spontaneously in most patients. pp 1597-1643 2. 2004. REFERENCES 1. an elective cholecystectomy should be scheduled if the patient has chronic gallbladder symptoms. the stomach or jejunum. McGraw-Hill. careful examination of the film will reveal gas in the biliary tree. in Brunicardi FC. but would require prolonged anesthesia and additional dissestion. It is seen most often in women. colon. and unless one is alert to the possibility of gallstone ileus. Dunn DL. but not address the intraluminal gallstone. On this basis. Obstruction of the large intestine may follow passage of a gallstone through a fistula at the hepatic flexure. The patient usually presents with obvious small bowel obstruction.Oddsdóttir M. ed 12. In addition to dilated small intestine.5 cm or more in diameter. either partial or complete.Ahrendt SA. The proper treatment is emergency laparotomy and removal of the obstructing stone through a small enterotomy. Small bowel resection with a primary anastomsis or ileostomy and ileocecectomy would relieve the obstruction. Once the patient has recovered. in Doherty GM (ed): Current Surgical Diagnosis & Treatment. Way LW: Biliary tract. The lumen in the proximal bowel will allow most of these large calculi to pass caudally until the ileum is reached. New York. and he is hydrated with intravenous fluids. The computed tomographic (CT) scan shown is obtained. Oral intake is discontinued. He has minimal upper abdominal tenderness without peritoneal signs. and right upper quadrant rebound tenderness. he has a worsening leukocytosis with left shift. Seventy-two hours after admission. elevated amylase.American College of Surgeons SESAP 13 Category 4 Item 10 A 35-year-old man is admitted with acute pancreatitis secondary to hypertriglyceridemia. The most appropriate management would be (A) laparotomy (B) nasogastric tube decompression and broad-spectrum antibiotics (C) somatostatin therapy (D) repeat CT scan in 48 to 72 hours (E) percutaneous drainage page 317 . and colonic involvement with the inflammatory process on CT scan mandate laparotomy. 2003 7.Law NM. A computed tomographic (CT) scan of the pancreas using intravenous contrast media should be obtained in any patient with acute pancreatitis that does not resolve after 48 to 72 hours.American College of Surgeons SESAP 13 Critique for Category 4 Item 10 One of the central issues in understanding the cellular events leading to acute pancreatitis is how duct obstruction results in intra-acinar cell enzyme activation. pancreatic necrosis. stomach. One of the most widely accepted theories to explain this coupling is the co-localization hypothesis. 1998 4. 1996 3. The other four choices will not address the extrapancreatic organ involvement. N Engl J Med 330:1198-1210. According to the colocalization hypothesis. 2003 5. 1999 2. Diamond T: Management of severe acute pancreatitis. When the clinical course of acute pancreatitis worsens. Mostafavi A. One of the earliest changes noted was co-localization of digestive enzyme zymogens such as trypsinogen with lysosomal hydrolases such as cathepsin B inside cytoplasmic vacuoles. leukocytosis. The inflammatory process may extend into adjacent organs. This patient’s CT scan demonstrates a thickened and edematous transverse colon with a mesenteric fluid collection.Yousaf M. and to couple pancreatitis to extrapancreatic events such as lung and renal injury. Rattner DW. The presence of peritonitis on physical examination. Digestion 1:19-21. cathepsin B can activate trypsinogen and trypsin can activate the other zymogens. cathepsin B-mediated intra-acinar cell activation of the digestive enzymes leads to acinar cell injury and triggers an intrapancreatic inflammatory response.Fernandez-del Castillo C. 1994 6.Bradley EL III: Operative vs nonoperative therapy in necrotizing pancreatitis.Steinberg W. The intensity of that inflammatory response appears to regulate the severity of the pancreatitis. et al: Debridement and closed packing for the treatment of necrotizing pancreatitis. The radiologic findings may be consistent with any of the following: relatively normal-appearing pancreas. early imaging with CT scan aids in identification of complicated pancreatitis and involvement of adjacent orgrans. the autodigestive process may be associated with peritonitis due to infarction or abscess formation. McCallion K. REFERENCES 1. Makary MA. which is based on a number of studies that have used experimental models of pancreatitis induced in laboratory animals.Yadav D. Ann Surg 228:676-684. Br J Surg 90:407420. pancreatic phlegmon. Under these conditions. or pancreatic pseudocyst or abscess formation. and jejunum. 2003 page 318 . pancreatic phlegmon with extension of the inflammatory process to adjacent extrapancreatic spaces. Pancreas 13:219-223. When this occurs. Pitchumoni CS: Issues in hyperlipidemic pancreatitis. including the transverse colon. Gastroenterol Clin North Am 32:1169-1194. Freeman ML: Emergency complications of acute and chronic pancreatitis. J Clin Gastroenterol 36:54-62. Tenner S: Acute pancreatitis.Bradley EL III: Indications for debridement of necrotizing pancreatitis. 000/mm3 . Of the following. the most appropriate management would be (A) open debridement (B) cystgastrostomy (C) Roux-en-Y cystjejunostomy (D) enteral feeding distal to the ligament of Trietz (E) CT-guided percutaneous drainage page 319 . he was discharged after an admission for uncomplicated acute pancreatitis. The abdominal computed tomographic (CT) scan shown is obtained.American College of Surgeons SESAP 13 Category 4 Item 11 A 35-year-old man has epigastric pain and emesis. He has been receiving corticosteroids since a cadaveric renal transplant 4 years ago. 1000 IU (normal 80 to190 IU). Four weeks previously. amylase. 11. Pertinent data include: WBC count. and normal serum creatinine and BUN. consisting of a swollen pancreas and contiguous viscera (a phlegmon). and internal operative drainage may be difficult or even impossible. Acute pseudocysts are often irregular in shape. the early stages of pseudocyst formation are often demonstrated radiographically before specific clinical findings appear. chronic pseudocysts are most often circular or nearly so.to 12-week delay. Catheter drainage at this stage is meddlesome. or when symptoms return after improving for a time. because their cyst wall is tough (mature) enough to hold sutures and allow an anastomosis with the gut. Waiting 8 to12 weeks allows for development of a mature wall and also for spontaneous involution of the pseudocyst. The cysts are single in 85% of cases and multiple in the remainder. uncomplicated pseudocyst identified on the computed tomographic (CT) scan. weight loss. CT scan is the diagnostic study of choice. A few have jaundice. Most cysts can be promptly eliminated by surgical drainage into the stomach or intestine. page 320 . tenderness. of complications. Enteral alimentation distal to the ligament of Trietz has been associated with reduced septic morbidity and is also less expensive than TPN. An enlarged pancreatic duct may be demonstrated in patients with chronic pancreatitis. In the absence of symptoms or radiographic evidence of enlargement (and regardless of cyst size). Recent data indicate that the natural history of these lesions is more benign than previously believed. albeit low. The chances of spontaneous resolution are about 40%. the mass may subside.American College of Surgeons SESAP 13 Critique for Category 4 Item 11 This patient’s symptoms are caused by the large. Internal drainage with either a cystgastrostomy or Roux-en-Y cystjejunostomy at this time would be difficult due to ongoing inflammatory process. Regardless of the type of prodromal phase. A pseudocyst should be suspected when a patient with acute pancreatitis fails to recover after a week of treatment. pain is the most common finding. Fever. a manifestation of obstruction of the intrapancreatic segment of the bile duct. immature. Either total parenteral nutrition(TPN) or enteral alimentation may be provided. and a few untreated cysts resolve spontaneously even after being stable for months. Pseudocysts develop in about 2% of patients with acute pancreatitis. The first clinical manifestation is usually a palpable tender mass in the epigastrium. but if it persists it most likely represents a pseudocyst. With time. This eliminates the need for prolonged follow-up with repeated ultrasound or CT scans and avoids the risks. Jaundice in a patient with a pseudocyst is usually caused by pressure from the cyst on the bile duct. and a palpable mass are present in about half of patients. Expectant management is especially important in the first 6 to12 weeks for cysts that have arisen during an attack of acute pancreatitis. Because obtaining a CT scan early in an attack of severe acute pancreatitis is now fairly routine. Patients who present with a symptomatic pseudocyst and no history of recent acute pancreatitis may be treated without the 6. Any anastomosis would be difficult at 4 weeks because of the lack of a fibrotic or mature wall. Open debridement is reserved for complicated necrotizing pancreatitis associated with a phlegmon or abscess. The size and shape of the cyst and its relationship to other viscera can be seen. expectant management is not unreasonable. Introduction of a percutaneous drain is associated with increased infection of the pseudocyst and/or retroperitoneum. Heider R.Grendell JH: Nonsurgical therapy of acute pseudocysts. pp 191-196 3. 1999 page 321 . in Bradley EL III (ed): Acute Pancreatitis: Diagnosis and Therapy.Cooperman AM: Surgical treatment of pancreatic pseudocysts. New York.American College of Surgeons SESAP 13 REFERENCES 1. 1994. Behrns KE: Percutaneous drainage of pancreatic pseudocysts is associated with a higher failure rate than surgical treatment in unselected patients. 2001 2. Surg Clin North Am 81:411-419. Meyer AA. Raven Press. Galanko JA. Ann Surg 229:781-787. Last bowel movement was 4 hours ago. Physical examination reveals abdominal distention with hypoactive bowel sounds and a tender mass in the right groin below the inguinal ligament. an incarcerated femoral hernia is reduced. She had an appendectomy at age12. The preferred repair would be (A) Bassini (B) Cooper’s ligament (C) Lichtenstein (D) mesh plug in femoral canal (E) Marcy repair page 322 .American College of Surgeons SESAP 13 Category 4 Item 12 A 78-year-woman with long-standing emphysema presents with a 24-hour history of abdominal distention and emesis. At groin exploration. Bowel is viable. or the neck of the defect for a direct hernia. Either nonabsorbable or long-acting absorbable suture should be used. using interrupted sutures. with the upper tail being placed on top of the lower. The initial steps of the Lichtenstein repair are similar to those of the Bassini repair. A femoral hernia presents as a swelling below the inguinal ligament and just lateral to the pubic tubercle. The wide overlap of the pubic tubercle is important because pubic tubercle recurrences are commonly seen with other operations. A single interrupted suture is used to secure the lower edge of the superior tail to the lower edge of the inferior tail. A Bassini repair involves reconstruction of the posterior wall by suturing the transversalis fascia. Femoral hernias are overdiagnosed because of the presence of a prominent femoral fat pad. The tails are positioned around the cord structures and placed beneath the external oblique aponeurosis laterally to about the anterior superior iliac spine. The medial end is rounded to correspond to the patient's anatomy and secured to the anterior rectus sheath a minimum of 2 cm medial to the pubic tubercle. A mesh prosthesis (with a minimum size of 15 by 8 cm for an adult) is positioned over the inguinal floor. securing the prosthesis to either side of the pubic tubercle (not into it). Displacing the cord structures laterally allows the placement of sutures through the muscular and fascial layers. page 323 . Operation is not indicated. in effect creating a shutter valve at the internal ring. a so-called femoral pseudohernia. The suture is continued in a running locking fashion laterally. The hernia sac is dissected away from surrounding structures and reduced back into the preperitoneal space. and then the shelving edge of the inguinal ligament. represents a surgical emergency. a wide one (two thirds) above and a narrower (one third) below. Femoral hernias are much more common in women than in men. Millikan. A flat sheet of polypropylene mesh is rolled up like a cigarette and held in place with suture. Its main indication is in Nyhus type I indirect inguinal hernias when the internal ring is normal. these hernias are prone to incarceration and strangulation. and possibly the iliopubic tract. The groin is entered through a standard anterior approach. They are asymptomatic and disappear spontaneously when the patient assumes a supine position. or one who develops an acute incarceration that remains exquisitely tender. The essential features of this operation are high ligation of the hernia sac plus narrowing of the internal ring. The suture is tied at the internal ring. The Marcy repair is the simplest nonprosthetic repair performed today. the internal oblique muscle medially to the inguinal ligament laterally. the transversus abdominis muscle. Femoral hernias comprise about one third of groin hernias in women and about 2% of groin hernias in men. commercially available prosthesis.American College of Surgeons SESAP 13 Critique for Category 4 Item 12 Femoral hernias descend through the femoral canal beneath the inguinal ligament. A slit is made at the lateral end of the mesh creating two tails. It is appropriate for children and young adults in whom long-term effects of prosthetic material remains a concern. Rutkow and Robbins recommend the use of a prefabricated. Thin patients commonly have prominent bilateral bulges below the inguinal ligament medial to the femoral vessels. The mesh plug technique was developed by Gilbert and then modified by Rutkow and Robbins. individualized for each patient by removing some of the petals to avoid unnecessary bulk. but in both sexes femoral hernias are less common than inguinal hernias. This plug is inserted into the defect and secured to either the internal ring for an indirect hernia. Because of their narrow necks. and others. A patient with an incarcerated inguinal hernia exhibiting signs of a bowel obstruction. with the notable exception of a femoral hernia in a patient with specific contraindications to the use of mesh. 2005. Interrupted sutures beginning at the pubic tubercle and continuing laterally along Cooper's ligament progressively narrow the femoral ring. the pectineus fascia. as in the Bassini repair. pp 1353-1394 page 324 . et al (eds): Schwartz's Principles of Surgery. ed 12. A mesh plug is contraindicated because of the potential contamination.American College of Surgeons SESAP 13 A Cooper ligament repair is similar to the Bassini repair. New York. This stitch closes the femoral ring.Fitzgibbons RJ Jr. New York. except that Cooper's ligament instead of the inguinal ligament is used for the medial portion of the repair. Many surgeons believe this results in more pain than other herniorrhaphies. Filipi CJ. Quinn TH: Inguinal hernias. pp 765-778 2. treatment of a femoral hernia). and predisposes to recurrence. eg. Billiar TR. McGraw-Hill.Deveney KE: Hernias and other lesions of the abdominal wall. Marcy. Andersen DK. in Doherty GM (ed): Current Surgical Diagnosis & Treatment. the operation is rarely chosen. and Lichtenstein repairs do not address a femoral defect. and this constitutes its most common application (ie. Dunn DL. A transition stitch includes the transversalis fascia. It also provides a smooth transition or step-up to the iliopubic tract over the femoral vessel so that the repair can be continued laterally. infection. in Brunicardi FC. Bassini. A relaxing incision should always be used given the considerable tension required to span such a large distance. 2006. McGraw Hill. ed 8. REFERENCES 1. For this reason. and the iliopubic tract. American College of Surgeons SESAP 13 Category 4 Item 13 Which of the following statements about immune thrombocytopenic purpura (ITP) in children is TRUE? (A) Usually requires splenectomy (B) Females are more frequently affected than males (C) Peak age is 12 years (D) Onset typically follows infectious illness (E) Incidence of intracranial hemorrhage is high page 325 . lymphoproliferative disorders. Holtman MD. The peak age for onset of childhood ITP is 5 years. or drugs such as heparin and quinidine. when necessary. Treatment of children with ITP is controversial. or corticosteroids. REFERENCES 1. There is no proof that therapy prevents intracranial hemorrhage. 2004. but the actual incidence is 0. In the United States it is diagnosed in approximately 25. Intravenous immune globulin (IVIG) is appropriate when there is internal bleeding. 2002 page 326 .American College of Surgeons SESAP 13 Critique for Category 4 Item 13 Immune thrombocytopenic purpura (ITP) is an autoimmune disorder characterized by low platelet count and bleeding. Most cases of intracranial hemorrhage occur within the first 4 weeks after diagnosis. Anti-D immune globulin is also effective in Rh-positive patients but is more costly. or emergency surgery. many within the first week. Adults generally require treatment at presentation because half present with platelet counts less than 10.Beauchamp RD. and can consist of intravenous methylpredisone. immunodeficiency states. The disease in children differs markedly from that seen in adults. anti-D immune globulin. Megathrombocytes (large. IVIG. In 70% of affected children. Boys and girls are equally affected. with 50% to 75% of patients responding over the course of 3 weeks after diagnosis. regardless of treatment. Splenectomy should be considered. permanent or lengthy remission is rare.2% to 1%. Saunders. Fabian TC: Spleen. Bleeding can be treated with aminocaproic acid or activated factor VII. secondary ITP may be accompanied by symptoms or signs of the underlying condition. Splenectomy is generally recommended for severe disease or when high doses of corticosteroids. Most are previously healthy and have sudden onset of petechiae or purpura a few days or weeks after an infectious illness.000 patients/year. HIV or hepatitis C infections. Fear of intracranial hemorrhage causes physicians to treat and limit patients’ physical activity. Primary disease is marked only by low platelet count and bleeding. A peripheral blood smear is necessary to rule out pseudothrombocytopenia. immature platelets) are often seen.000 and are at risk for internal bleeding. Adult ITP has a more insidious onset and is a chronic condition affecting twice as many women as men. ie. internal bleeding. ITP resolves within 6 months. antiphospholipid syndrome.000 for more than 3 to 6 months. Most children recover completely within a few weeks and without treatment. Treatment. Philadelphia. or when the patient has extensive or progressive purpura. Although 80% of patients respond to IVIG.Cines DB. is usually with IVIG. when platelet count remains below 5000 despite corticosteroid treatment. in Townsend CM Jr (ed): Sabiston Textbook of Surgery: The Biological Basis of Modern Surgical Practice. The condition can be acute or chronic. are required to maintain platelets > 30. and other hematologic disorders. inherited giant platelet syndromes. Approximately half of the cases of ITP each year occur in children. Vincristine can be added. Urgent medical treatment is required for adults and children with neurologic symptoms. First-line treatment is with oral prednisone. ITP can be either primary or secondary to other conditions such as systemic lupus erythematosis. N Engl J Med 346:995-1008. ed 17. 10 to 20 mg/day of prednisone. pp 1679-1708 2. and platelet administration. Blanchette VS: Immune thrombocytopenic purpura. and subsequently skin grafted. allowed to granulate.American College of Surgeons SESAP 13 Category 4 Item 14 A 28-year-old patient with Crohn's disease developed necrotizing pancreatitis and subsequently a necrotizing infection of the abdomen that was packed. as shown. the most appropriate for definitive closure would be (A) component separation and advancement closure (B) mesh closure of the defect (C) placement of tissue expanders. skin expansion. Of the following methods. and skin advancement flaps over synthetic mesh (D) tensor fasciae latae free flap (E) gracilis musculocutaneous transposition flap page 327 . Axial incisions are made in the lateral rectus fascia to disconnect it from fascial contributions from the external and internal oblique fascia. based either cephalad or caudad depending on the nature of the injury. allowing relaxation and mobilization toward the midline to permit the medial borders of the rectus abdominis fascia to be approximated. fascial replacement is usually done with some form of mesh. but the arcs of rotation do not permit them to be used as rotation or transposition flaps. these can be pedicled. and the amount of visceral protrusion outside the abdominal cavity. It is unlikely to be effective in the face of page 328 . increasing the midline shift. eg. Consolidation and protection of the intra-abdominal and intra-thoracic viscera is the primary concern. eg. The most useful of these methods comes from the quadratis femoris muscle group in the leg. enterolysis and mobilization of the fascia to the midline is the obvious choice. amount of muscle and fascia remaining. one that has been skin grafted and is to be later closed with mesh. the presence or absence of exposed or infected synthetic mesh. is a superior method. Computed tomographic (CT) scan of the chest. the rectus femoris. Other muscle flaps may be used as free flaps. Depending on the patient’s physiologic status. A modification of this technique frees the posterior rectus sheath from the rectus muscle and allows its medial border to be approximated to the lateral border of the anterior rectus sheet. The component separation procedure facilitates closure of large gaps (up to 10 cm) with completely autogenous tissue. closed hernia defect. These axial incisions do not transect the intercostal nerve. or definitively. particularly the rectus abdominis muscle. Division of perforators between the rectus and the skin depends on whether access to the lateral rectus sheath is gained by raising a skin flap or by direct lateral incision through the skin. When the normal structures. silo. the presence or absence of enterocutaneous fistula. In defects that cannot be closed by advancement or component separation. The tensor fasciae latae. Analysis of the defect most significantly involves presence or absence of skin cover. and the condition of the subjacent viscera. The only fascial reconstruction that also brings full-thickness skin to the abdominal wall is transposition either by pedicle or free flap technique of musculocutaneous flaps. For infra-umbilical defects. The underlying operational concept is to replace skin coverage and restore dehiscent fascia to its normal anatomic position if still present or to bridge the defect with appropriate or synthetic or autogenous materials to effect closure. this can be done temporarily. Positive experience with acellular cadaver dermis has been described. and the tensor fasciae latae. is the only other useful rotation flap. are present but dehiscent and the loss of domain is not excessive. which provides thick fascia muscle and full-thickness skin. and the remaining deep inferior epigastric vessels are often adequate recipient vessels. This seems to serve as a substrate for subsequent regrowth of fascia and appears to have fewer problems with fistula or infection than the various synthetic meshes.American College of Surgeons SESAP 13 Critique for Category 4 Item 14 The approach to reconstruction of abdominal wall defects requires careful analysis of the tissue loss and assignment of an appropriate goal for the point in time in terms of the natural history of the disease process. The rectus abdominis. abdomen and pelvis is useful to define the presence and position of muscle and fascia and to analyze the position of the intraabdominal contents. nonadherent dressings and packing. Tissue expansion is occasionally useful to cover a long-standing. which at that level is arborizing in between the internal oblique and transverse abdominis muscles. Larger defects require free tissue transfer. Careful physical examination should note the amount of skin and fascia loss. Synthetic polypropylene meshes of various types have been used and are still useful. however.American College of Surgeons SESAP 13 stomas. Steinwald PM.Mathes SJ. Because of the large size of the wound. Hackney FL. et al: Complex abdominal wall reconstruction: a comparison of flap and mesh closure. component separation would be inadequate and expansion and synthetic mesh unlikely to work. The tensor fasciae latae as a free flap would. Plast Reconstr Surg 105:202-216. Lowe JB. Dumanian GA: Definitive surgical treatment of infected or exposed ventral hernia mesh. et al: An algorithm for abdominal wall reconstruction. Foster RD. or other contaminated fields because of increased risk of infection.Rohrich RJ. be an excellent choice. REFERENCES 1. Bowman JL. Ann Surg 232:586-596.Szczerba SR. and the presence of a stoma and infected mesh. the loss of skin and rectus muscles. 2000 2. infected mesh. 2000 3. Ann Surg 237:437-441. The gracilis muscle is much too small to fill the patient's defect and the arc of rotation as a transposition flap does not reach the abdomen. Hoffman WY. 2003 page 329 . American College of Surgeons SESAP 13 Category 4 Item 15 Transjugular intrahepatic portosystemic shunt (TIPS) is indicated for (A) refractory variceal bleeding (B) Caroli’s disease (C) primary prophylaxis of variceal hemorrhage (D) prehepatic portal hypertension (E) correction of hypersplenism and thrombocytopenia page 330 . which is associated with high risk of severe hemorrhage. Shiffman ML. Only one third of patients with cirrhosis and esophageal varices will experience variceal hemorrhage. In a systematic prospective study of 60 subjects in whom baseline platelets were obtained. Gastroenterology 124:1700-1710. Purdum PP. mean platelet counts were unchanged after TIPS. 1995 page 331 . Both polycystic liver disease and Caroli’s disease may be complicated by portal hypertension and variceal hemorrhage. but these disorders represent a contraindication to TIPS. The data do not justify TIPS in routine clinical practice for this indication.Shiffman ML. and is not indicated for the primary prophylaxis of variceal hemorrhage. Pharmacologic treatment with nonselective beta blockers achieves a decrease in bleeding frequency with relatively little morbidity. 2003 2. invasive procedure with significant morbidity and mortality.Boyer TD: Transjugular intrahepatic portosystemic shunt: current status. Prehepatic portal hypertension results most commonly from portal vein thrombosis or tumors at the porta hepatis. and prevention and treatment of complications. especially those with limited hepatic reserve who might become transplant candidates. In comparison. REFERENCES 1. Thrombocytopenia in patients with cirrhosis has primarily been ascribed to splenic engorgement and sequestration. The mere presence of esophagogastric varices does not represent an indication for TIPS. Hepatology 22:1591-1597. Tralka TS: The role of transjugular intrahepatic portosystemic shunt for treatment of portal hypertension and its complications: a conference sponsored by the National Digestive Diseases Advisory Board. Hoofnagle JH. 1996 3.Sanyal AJ. TIPS is presently the procedure of choice for patients who are poor operative candidates. Hepatology 23:32-39. et al: The hematologic consequences of transjugular intrahepatic portosystemic shunts. Failure to control bleeding despite adequate endoscopic and pharmacologic treatment is an indication to move rapidly to definitive salvage treatment such as transjugular intrahepatic portosystemic shunt (TIPS) or operation. Freedman AM. TIPS is an expensive. The creation of an intrahepatic tract may involve going through a cyst. Jeffers L.American College of Surgeons SESAP 13 Critique for Category 4 Item 15 Initial therapy for acute variceal bleeding is aimed at hemodynamic resuscitation. low-grade fever. and malaise has the computed tomographic (CT) scan shown. The best treatment would be (A) corticosteroids (B) cytotoxic chemotherapy (C) hospice care (D) bypass of the mass (E) radical resection of the mass including small bowel and mesentery page 332 .American College of Surgeons SESAP 13 Category 4 Item 16 A 60-year-old man being evaluated for chronic abdominal pain. and displacement without invasion of the bowel. Resection is both unnecessary and potentially dangerous. Tissue biopsy should be obtained because of the need to exclude potentially treatable malignancies. Hartmann R.Parra-Davila E.Genereau T. or sclerosing lipogranulomatosis. sclerosing peritonitis. Bellin MF. primarily for the diagnosis of the soft tissue tumors. Initially. Nayak V: Treatment of sclerosing mesarteritis with corticosteroids and azothioprine. Most treatments have centered around anti-inflammatory or immunosuppressive agents. et al: Demonstration of efficacy of combining corticosteroids and colchicines in two patients with idiopathic sclerosing mesarteritis. et al: Mesenteric pancreatitis: case report and literature review. REFERENCES 1. Can J Gastroenterol 15:533. The differential diagnosis includes inflammatory pseudotumor. Am Surg 64:768-771. Despite their size and intimate involvement with the small bowel mesentery. mesenteric lipodystrophy. Coderre SP. This rare disease of unknown etiology progresses through three phases: degeneration of mesentery fat (the mesenteric lipodystrophy phase). Severity of symptoms also progresses. and malaise. symptoms can become more severe and bowel obstruction may occur. Sleeman D. Spontaneous resolution may even occur in this phase. As the disease progresses to the retractile mesenteritis phase. optimum treatment has not been determined. lymphoma involving the small bowel mesenteric lymph nodes. also referred to as sclerosing mesenteritis. computed tomographic (CT) scanning usually leads to the discovery of the mesenteric mass. but laparoscopic or open incisional biopsy is usually required to definitively rule out lymphoma and diagnose mesenteric panniculitis.Bala A.American College of Surgeons SESAP 13 Critique for Category 4 Item 16 This patient has mesenteric panniculitis. but treatment response varies and these treatments are often unsuccessful. and soft tissue sarcomas. inflammatory reaction (the mesenteric panniculitis phase). Dig Dis Sci 41:684. CTguided core-needle biopsy may be helpful. retractile mesenteritis. CT features include an ill-defined mass involving the mesentery. if the disease progresses. desmoid tumor. 1998 page 333 . 1996 3. McKenney MG. these lesions do not cause bowel obstruction. idiopathic retroperitoneal fibrosis. the most common symptoms are abdominal pain. Although physical examination can demonstrate a vague. Wechsler B. Corticosteroids and cholchicine appear to have the best efficacy for alleviation of symptoms. involvement (but no true encasement) of the mesenteric vessels. Because of the rarity of the disease and lack of knowledge about its etiology. however. 2001 2. and finally fibrosis of adipose tissue (retractile mesenteritis phase). Johnson DR. or even large abdominal mass. patients may be asymptomatic with the abnormality found incidentally by imaging. low-grade fever. American College of Surgeons SESAP 13 Category 4 Item 17 Surgical exploration for possible resection of a solid cystic pancreatic tumor will most likely result in (A) complete resection with low risk of recurrence (B) partial resection with frequent local recurrence (C) late distant recurrence. despite complete resection (D) unresectability because of extent of local invasion (E) frequent uncontrolled hemorrhage due to neovascularization page 334 . distinguishing them from serous and cyst adenomas. Despite their large size. Surgery 108:475-480. these tumors are infrequently associated with obstructive jaundice when located in the head of the pancreas. The solid component appears as mural nodules. Sometimes referred to as papillary-cystic tumors. Erlandson RA.American College of Surgeons SESAP 13 Critique for Category 4 Item 17 Solid cystic tumors of the pancreas are very rare. the so-called floating cloud sign. Although some authors cite a higher incidence in women of color. 1988 3.Pezzi CM. Cameron JL: Solid and papillary epithelial neoplasms of the pancreas. They average 10 cm in diameter on presentation and can be located throughout the pancreas. and is the treatment of choice. Computed tomographic (CT) scan usually shows a very large. et al: CT manifestations and features of solid cystic tumors of the pancreas. Zhan Y. Prognosis is excellent. J Surg Oncol 37:278-285. They are frequently asymptomatic and are discovered as incidental findings on imaging studies obtained for other reasons. 1990 page 335 . long-term survival the norm.Miao F. These tumors do not invade the surrounding structures. with disease-free. there are no other known risk factors. well-circumscribed tumor with either predominantly cystic or equal solid and cystic components. When symptoms do occur. Schuerch C. which usually occur in middle-aged women. 2002 2. there may be internal septations in the cystic component. adjuvant therapy is not required.Zinner MJ. Wang X-Y. Hepatobiliary Pancreat Dis Int 1:465-468. they are generally vague abdominal pain or pressure. Once resected. Deitrick J: Papillary-cystic neoplasm of the pancreas. these tumors have a very distinct presentation and occur almost exclusively in young women. Wang D-B. REFERENCES 1. Shurbaji MS. Pancreatic resection is feasible in most cases. American College of Surgeons SESAP 13 Category 4 Item 18 The single best test to determine a hepatic hemangioma is (A) transabdominal ultrasound study (B) endoscopic ultrasound study (C) computed tomography (D) magnetic resonance imaging (E) laparoscopy page 336 . but ultrasound has an accuracy only in the range of 70% to 80%. Nguyen MH: The diagnosis and management of benign hepatic tumors. J Am Coll Surg 197:392-402. well-circumscribed mass with posterior acoustic enhancement. specificities. and outcomes of 115 patients with hepatic hemangioma. although liver cysts are the most common structural abnormalities. Charny CK. Resection is only indicated in patients with symptomatic lesions. J Clin Gastroenterol 39:401-412. 2003 page 337 . 1990 3. et al: Diagnosis. and complete isoattenuating fill-in occurring not less than 3 minutes or more that 60 minutes after contrast administration. and most are found in women. hyperechoic. There are no other known predisposing factors. right upper quadrant pain or discomfort. progressive opacification toward the center. REFERENCES 1. Hepatic hemangiomas can occur at any age. and accuracies of more than 90% in diagnosing hemangiomas and is the single best test.Choi BY. However. asymptomatic lesions can be safely observed. Jarnagin WR. liver function tests are not abnormal and jaundice does not occur. these tend to be dull. although detection is good with proper administration of a contrast agent and appropriately timed scan. only 50% to 80% of hemangiomas meet these criteria. Usually. Strict CT criteria include relative hypoattenuation compared with the normal liver. Radiology 176:11-13. although even very large lesions can be asymptomatic. Hepatic hemangiomas can be detected by a variety of radiologic studies.Yoon SS.American College of Surgeons SESAP 13 Critique for Category 4 Item 18 Hepatic hemangiomas are the most common tumor of the liver. Fong Y. The classic ultrasonographic appearance is that of a homogeneous. With an MRI diagnosis of a hepatic hemangioma. When symptoms do occur. Most lesions are less than 5 cm in diameter. Standard computed tomographic (CT) techniques can miss hemangiomas.Nelson RC. management. Magnetic resonance imaging (MRI) has sensitivities. 2005 2. Hemangiomas can be solitary or multiple. Chezmar JL: Diagnostic approach to hepatic hemangiomas. The most common presentation is an incidental finding when imaging is done for other reasons. early peripheral enhancement. American College of Surgeons SESAP 13 Category 4 Item 19 A 45-year-old man with Crohn’s disease has an asymptomatic intersphincteric fistula-in-ano. The most appropriate management would be (A) observation (B) fibrin glue injection (C) cutting seton (D) fistulotomy (E) botulinum toxin page 338 . Goldberg SM: Fistula-in-ano in Crohn's disease. Salvati EP: Seton management of complex anorectal fistulas in patients with Crohn's disease. Hurst RD: Surgical treatment of anorectal complications in Crohn's disease. Fibrin glue.Michelassi F.American College of Surgeons SESAP 13 Critique for Category 4 Item 19 Perianal fistulas are a common manifestation of Crohn’s disease. Rubin M. the success rate in patients with Crohn’s disease is reduced by half compared with those whose fistula is not associated with Crohn’s disease. a newer alternative to fistulotomy. Botulinum toxin has been successfully used as a nonsurgical treatment of anal fissure. However. but has no role in the management of fistula-in-ano. Results of aggressive surgical treatment. 2000 3. Kodner IJ. does not divide sphincter. 1991 page 339 . and is easily repeatable. Eisenstat TE. low.Williams JG. 1990 4. short fistulas that have little sphincter at risk. Surgery 128:597-603. Goals of therapy are to palliate symptoms. Dis Colon Rectum 34:378-384. Timmcke A: Techniques and results in the management of anal and perianal Crohn's disease. is appealing because it is simple to perform. Melis M. reserving fistulotomy for simple. Shemesh EI. REFERENCES 1. Symptomatic Crohn’s fistula-in-ano is ideally treated with sphincter-preserving techniques. Surg Gynecol Obstet 168:42-48.Fry RD. which is a chronic inflammatory gastrointestinal disease typified by an unpredictable relapsing pattern that has no known cure.White RA. 1989 2. Rubin RJ. Rothenberger DA. Surgical therapy including cutting seton is not necessary in the asymptomatic patient. Dis Colon Rectum 33:587-589. Nemer FD. American College of Surgeons SESAP 13 Category 4 Item 20 A 56-year-old woman with a history of ulcerative colitis develops a painful area inferior to her ileostomy. The most appropriate management would be (A) cholestyramine powder and reduction of the size of the ileostomy appliance opening (B) antifungal powder (C) high-dose systemic corticosteroids (D) debridement followed by wound vac closure (E) wide local excision with stoma relocation page 340 . as shown. fistula.3 Online access November 1. The presence of spreading ulceration with underlying violaceous edges is one of the hallmarks of the disease. and therefore re-siting is not considered a primary treatment modality.Peppercorn MA: Skin and eye manifestations of inflammatory bowel disease UpToDate version 13. conjunctivitis. and sclerosing cholangitis.Sheldon DG.American College of Surgeons SESAP 13 Critique for Category 4 Item 20 Inflammatory bowel disease may be associated with several extraintestinal manifestations.asp 2. REFERENCES 1. Debridement or excision is not indicated. Hughes LE: Parastomal pyoderma gangrenosum in inflammatory bowel disease. 2005. arthritis. ankylosing spondylitis.com/index. most often involving the extremities. Excessive ileal effluent can sometimes be treated with cholestyramine powder and reduction of the size of the ileostomy appliance opening.Tjandra JJ. Available at: http://uptodate. iritis. The lesion pictured is pyoderma gangrenosum. Dis Colon Rectum 37:938-942. Re-siting the stoma to a new location often is followed by the development of pyoderma gangrenosum around the new ileostomy. the diagnosis is made on clinical grounds. including pyoderma gangrenosum. Thirlby RC: Twenty cases of peristomal pyoderma gangrenosum: diagnostic implications and management. Kozarek RA. a painful chronic ulcerating skin condition. Arch Surg 135:564-569.The diagnosis of pyoderma gangrenosum is challenging and must be differentiated from these causes. 1994 page 341 . Antifungal powder is a treatment for Candidiasis. candidiasis. and is often caused by unintended contact of the ileal effluent around an ill-fitting ostomy appliance. Peristomal skin irritation and ulceration is not uncommon in patients with inflammatory bowel disease. most commonly high-dose corticosteroids. and other dermatoses. Because pyoderma has no pathognomonic histologic findings on biopsy. 2000 3. The therapy should be immunosuppressive agents. Sawchuk LL. erythema nodosum. American College of Surgeons SESAP 13 Category 4 Item 21 Five years after a restorative proctocolectomy for familial adenomatous polyposis. a 28-year-old man has a single episode of gastroenteritis that completely resolved after a visit in the emergency department. The most appropriate treatment now would be (A) sulindac (B) imatinib (Gleevec) (C) external-beam radiation (D) surgical enucleation (E) radical excision page 342 . The computed tomographic (CT) scan shown was obtained. Landgrebe JC. Operation is not considered a mainstay of treatment for intra-abdominal desmoids because of a very high recurrence rate and a high perioperative surgical morbidity rate related to hemorrhage. Although most desmoids are indolent or demonstrate cyclical patterns of growth and regression.Church JM: Desmoid tumors.American College of Surgeons SESAP 13 Critique for Category 4 Item 21 Desmoid tumors develop in 10% to15% of patients with familial adenomatous polyposis (FAP). Cancer 95:2373-2379. Patients whose tumors continue to grow or those with life-threatening symptoms can be offered anti-sarcoma chemotherapeutic regimens. pp 355-358 2. yet local infiltration and mass effect can cause significant morbidity and mortality. Philadelphia. McGinn C. Desmoid tumors have not consistently responded to radiation therapy and the risk of radiation enteritis in the surrounding bowel limits doses. they remain the second most common cause of death in FAP patients. No treatment is required for an asymptomatic desmoid showing no growth.Mace J. These tumors of proliferating myofibroblasts have no metastatic potential. 1999 3. Biermann JS. Br J Surg 86:1185-1189.Clark SK. implicating surgical trauma as a contributing factor. et al: Response of extraabdominal desmoid tumors to therapy with imatinib mesylate. in Fazio VW. The natural history of desmoid tumors is unpredictable. Treatment of desmoids continues to evolve. Church JM. Imatinib (Gleevec). a receptor tyrosine kinase inhibitor. Up to 80% of desmoids occur in the postoperative setting. Delaney CP (eds): Current Therapy in Colon and Rectal Surgery. fistulas. The first line of treatment for mildly symptomatic patients includes sulindac and/or tamoxifen. This FAP variant is also called Gardner’s syndrome. Elsevier Mosby. REFERENCES 1. 2005. 2002 page 343 . Neale KF. is an effective therapy for gastrointestinal stromal tumors (GISTs). Sondak V. Phillips RK: Desmoid tumours complicating familial adenomatous polyposis. FAP patients with mutations near the 3-prime end of the APC gene are prone to desmoid formation. and the potential for short-bowel syndrome. ed 2. but data are insufficient to recommend its use in desmoids. American College of Surgeons SESAP 13 Category 4 Item 22 The most common cause of Budd-Chiari syndrome in the US is (A) hematologic disorders (B) oral contraceptives (C) vena caval webs (D) right heart failure (E) alcoholic cirrhosis page 344 . REFERENCES 1. In the United States. accounting for more than 50% of the cases of Budd-Chiari syndrome. but are rare in the United States. Kaplowitz N.Slakey DP. ranging from chronic hepatic disease to acute fulminant hepatic failure. vague abdominal complaints are accompanied by the onset of ascites. The primary goals of treatment are palliation of symptoms and elimination of hepatic congestion.American College of Surgeons SESAP 13 Critique for Category 4 Item 22 Budd-Chiari syndrome (BCS) is the sequelae of hepatic vein thrombosis or mechanical obstruction of the hepatic veins. pp 25172525 2. In the most common presentation. Ann Surg 233:522-527. Use of oral contraceptive pills is a lesser risk factor. Alpers DH. the leading risk factor for hepatic vein thrombosis is polycythemia vera and other myeloproliferative disorders. Philadelphia. Vena caval webs that act as a hepatic vein outlet obstruction are the most common cause of Budd-Chiari syndrome in Asia. Klein AS. Laine L.DeLeve LD: Vascular diseases of the liver. in Yamada T. Cameron JL: Budd-Chiari syndrome: current management options. 2001 page 345 . whereas right heart failure and cirrhosis do not seem to be causative factors. et al (eds): Textbook of Gastroenterology. BCS manifests over a wide spectrum of disease patterns. Venbrux AC. Lippincott Williams & Wilkins. ed 4. Treatment is highly individualized and usually consists of some form of surgical or image-guided vascular intervention up to and including liver transplantation. Most patients also require lifelong anticoagulation to manage the hypercoagulable state. 2003. American College of Surgeons SESAP 13 Category 4 Item 23 A 36-year-old previously healthy woman who was wearing a seatbelt is hemodynamically normal on arrival after a motor vehicle crash. Abdominal computed tomography (CT) shows a mass in her liver. The best recommendation would be (A) no intervention (B) percutaneous drainage (C) colonoscopy to identify a primary malignancy (D) embolization of the right hepatic artery (E) resection of the right lobe of the liver page 346 . The abdominal magnetic resonance image (MRI ) shown was obtained 2 days later. hepatic artery ligation. Montagnini AL.Herman P. hepatic adenoma. although these drugs have a trophic effect. marked thrombocytopenia. cirrhotic-like mass in otherwise normal liver. radiation therapy. It is generally a single. nodular. Pugliese V. Metastatic lesions to the liver have typical features that distinguish them from the primary lesions of the liver. When treatment is indicated. FNH does not result from the ingestion of oral contraceptives. et al: Hepatic adenoma and focal nodular hyperplasia: differential diagnosis and treatment. particularly if there is a potential for exposure to trauma. subcapsular lesion. and FNH. Machado MA. Endoscopy 35:27-35.Bond JH: Colon polyps and cancer. FNH is the most common benign hepatic epithelial hamartoma seen in children and is more frequent in females. the lesion occurs in both sexes and does not recur after excision. The classic imaging finding is a stellate scar that can be demonstrated on either CT or MRI. Indications for surgical intervention include a palpable mass. Asymptomatic FNH should be treated conservatively because there are no convincing reports showing that these tumors can bleed or undergo malignant degeneration. surgical resection provides the only consistently effective method. The Kasabach-Merritt syndrome with platelet trapping and a large hemangioma has been an indication for resection in a few cases. Although each has its own distinct pathologic and clinical features. pain or epigastric discomfort. percutaneous biopsy is rarely necessary. rapid growth. Each nodule has a characteristic fibrous stellate central scar with slender septa to the periphery. 2000 3. and corticosteroids have limited success and are occasionally associated with morbidity. and rupture with intraperitoneal bleeding. Embolization. REFERENCES 1.Chen MF: Hepatic resection for benign tumours of the liver. Sex hormones are unlikely to be factors in infants and children. Resection is required when the diagnosis of FNH cannot be established and when FNH is symptomatic. 2000 page 347 . hepatic adenoma and FNH are frequently difficult to differentiate. The pathogenesis is uncertain. J Gastroenterol Hepatol 15:587-592. Most patients with hepatic hemangioma should not require major operation. nonencapsulated. World J Surg 24:372-376.American College of Surgeons SESAP 13 Critique for Category 4 Item 23 The magnetic resonance image (MRI) shows an incidental focal nodular hyperplasia (FNH). 2003 2. The differential diagnosis includes hemangioma. but may be multiple in 12% to 20% of cases and pedunculated in 5%. FNH is usually a well circumscribed. Both occur primarily in young women and are composed of hepatocytes. The hernia is 30 cm long x 15 cm wide.American College of Surgeons SESAP 13 Category 4 Item 24 A 53-year-old man with a history of a gunshot wound to the abdomen presents with a recurrent hernia despite previous repair with prolene mesh. Factors that can limit the success of component separation for repair of this abdominal hernia include all of the following EXCEPT (A) previous colostomy (B) disruption of the rectus muscle from the gunshot wound (C) non-midline fascial defects (D) ulceration of the thin scar overlying the hernia (E) significant smoking history page 348 . The hernia continues to enlarge and is interfering with activities of daily living. Smoking can compromise wound healing and may result in partial or complete necrosis of the large skin flaps that are raised during component separation. Plast Reconstr Surg 105:720-729. Dellon AL: "Components separation" method for closure of abdominal-wall defects: an anatomic and clinical study. Rohrich RJ. Plast Reconstr Surg 105:202-216. Plast Reconstr Surg 86:519-526.American College of Surgeons SESAP 13 Critique for Category 4 Item 24 The goals of abdominal reconstruction include restoration of the function and integrity of the musculofascial abdominal wall. This procedure is ideal for midline musculofascial defects greater than 3 cm in size. Thinning of the scar overlying the hernia with bleeding and chronic ulceration is an indication to repair the hernia. 2000 page 349 . Bowman JL.Garza JR. Hackney FL. REFERENCES 1. prevention of visceral eventration.Rohrich RJ. et al: An algorithm for abdominal wall reconstruction. Removal of prosthetic material is an advantage of component separation in the presence of a wound. Ruas E. or damage to the rectus muscle from previous trauma will also limit the advancement of vascularized musculofascial units.Ramirez OM.Lowe JB. Component separation uses a series of fascial incisions for maximal tissue advancement with preservation of the continuity of the internal oblique fascia. but is of limited reconstructive use in non-midline defects. previous colostomy. Lowe JB. and provision of dynamic muscle support. Transverse incisions. et al: Endoscopically assisted "components separation" for closure of abdominal wall defects. 2000 2. 1990 3. Bowman JL. American College of Surgeons SESAP 13 Category 4 Item 25 A 65-year-old man is undergoing closure of a large abdominal hernia using component separation when the anesthesiologist notices that airway pressure has increased significantly. The most likely diagnosis is (A) right mainstem intubation (B) pulmonary edema (C) pneumothorax (D) pulmonary embolus (E) loss of domain page 350 . Johnson RR: The separation of anatomic components technique for the reconstruction of massive midline abdominal wall defects: anatomy.Tobias AM. and limitations revisited.Shestak KC. Plast Reconstr Surg 112:766-776. there is lateral migration of the rectus abdominus muscle in conjunction with flank muscle contraction. 2000 2. Low DW: The use of a subfascial vicryl mesh buttress to aid in the closure of massive ventral hernias following damage-control laparotomy. One of the primary problems that results from delayed reconstruction of the “damage control abdomen” is loss of domain. Edington HJ.Carlson GW. there will be a rise in pulmonary pressure and difficulty ventilating. Opening the repair verifies the diagnosis and requires that an alternative strategy be used to accomplish closure. Ann Plast Surg 44:147-153. surgical technique. However. Losken A. REFERENCES 1. When the midline fascia is not re-approximated. even in some cases in which the abdomen can be closed. Elwood E. applications. but are left with massive midline hernias. Plast Reconstr Surg 105:731-738. This will be noticed by the anesthesiologist and coincides with abdominal closure. The goal of component separation is to achieve a tensionless repair. 2003 page 351 . but unlikely at this time in the scenario.American College of Surgeons SESAP 13 Critique for Category 4 Item 25 The trend toward damage control laparotomy has resulted in increased numbers of patients who survive abdominal trauma or sepsis. The other options listed are all possible in this patient population. Galloway JR: The role of tissue expansion in abdominal wall reconstruction. This reduces the volume of the abdominal cavity and worsens the protrusion of the viscera. 2000 3. American College of Surgeons SESAP 13 Category 4 Item 26 Which of the following statements about gastrointestinal carcinoid is TRUE? (A) Carcinoid syndrome occurs from metastatic tumor in the liver (B) Valvular disease in carcinoid syndrome affects primarily the mitral and aortic valves (C) Once metastatic liver disease is present. survival is usually < 1 year (D) Carcinoid is embryologically derived from the mesenchymal layer (E) Carcinoid crisis may be treated medically with calcium channel blockers page 352 . are used for medical treatment of carcinoid crisis. These cells can produce vasoactive peptides such as serotonin. 2005 2. Hellman P.Rorstad O: Prognostic indicators for carcinoid neuroendocrine tumors of the gastrointestinal tract. pp 555-618 page 353 . and carcinoidal valvular fibrosis usually occurs in the right side of the heart. or venous drainage from the tumor bypasses the liver. which differentiate from the neural crest. Hessman O. but gut carcinoids will not produce carcinoid syndrome until liver metastases have overwhelmed the liver’s ability to detoxify the vasoactive peptides.Schoen FJ: The heart. REFERENCES 1. Philadelphia. palliative surgical therapy to debulk disease can significantly prolong life and alleviate symptoms of carcinoid syndrome even without curative resection. such as in a foregut or ovarian carcinoid. Fausto N (eds): Robbins & Cotran Pathologic Basis of Disease. in Kumar V. Osmak L: Management of midgut carcinoids.American College of Surgeons SESAP 13 Critique for Category 4 Item 26 Carcinoids are slow-growing tumors derived from amine precursor uptake and decarboxylation (APUD) cells. J Surg Oncol 89:161-169.Akerstrom G. Abbas AK. a protean group of cells derived from ectoderm. 2005 3. not calcium channel blockers. Elsevier Saunders. Somatostatin analogs (octreotide). The lung has some ability to detoxify vasoactive peptides as well. 2005. J Surg Oncol 89:151-160. Because carcinoids tend to be slow-growing. ed 7. Patients with unresectable disease and even hepatic metastases may live for years. weight loss. diarrhea. bulging sphincter of Oddi (B) Invasive cancer is rarely present at diagnosis (C) Peripheral lesions are more commonly associated with invasive disease than are main pancreatic duct lesions (D) Total pancreatectomy is preferred to partial pancreatectomy (E) Abdominal pain. and jaundice are present in most cases page 354 .American College of Surgeons SESAP 13 Category 4 Item 27 Which of the following statements about the neoplastic process in the pancreas illustrated is TRUE? (A) Mucin production leads to a patulous. et al: Classification of types of intraductal papillary-mucinous neoplasm of the pancreas: a consensus study. like the progression from polyp to cancer that is well-delineated in colon cancer. The disease has only been recognized for a few decades. This approach is given credence by the absence of clinical data showing a distinct benefit in survival or recurrence of disease in patients with microscopically negative margins. 2005 3.American College of Surgeons SESAP 13 Critique for Category 4 Item 27 Intraductal papillary mucinous neoplasia of the pancreas may be considered a premalignant condition of the pancreas. Adsay VN. and optimal surgical treatment is still somewhat controversial. Virchows Arch 447:794-799. Ann Surg 239:788-797. 2005 2. J Clin Oncol 23:4518-4523. Klöppel G. Up to half of these patients have invasive cancer at the time of diagnosis. REFERENCES 1. leading to its characteristic appearance with multiple cysts and dilated ducts. and jaundice is present in only 15%.Furukawa T. carcinoma in situ and severe dysplasia are also common. Most of the hallmark pathologic features of the disease relate to copious production of thick mucin. Patients usually have nonspecific symptoms. Yeo CJ.Conlon KC: Intraductal papillary mucinous tumors of the pancreas. Hruban RH. 2004 page 355 . Albores-Saavedra J.Sohn TA. Cameron JL. Invasive disease is more prevalent in patients whose disease is primarily located within the main pancreatic duct. et al: Intraductal papillary mucinous neoplasms of the pancreas: an updated experience. Most experts believe that partial pancreatectomy is preferable to total pancreatectomy even when microscopic disease is present at the surgical margin. She has no abdominal complaints and her liver and pancreatic laboratory values have returned to normal. She is scheduled for laparoscopic cholecystectomy.American College of Surgeons SESAP 13 Category 4 Item 28 A 43-year-old woman has gallstone pancreatitis that resolves in 2 days with conservative treatment. Which of the following statements is TRUE? (A) Intraoperative cholangiography is associated with a decreased risk of biliary tract injury (B) The procedure should be scheduled for 6 weeks after resolution of symptoms (C) Intraoperative cholangiography in this patient will identify choledocholithiasis in 50% of cases (D) Preoperative endoscopic retrograde cholangiopancreatography (ERCP) should be performed (E) The sensitivity of magnetic resonance cholangiopancreatography (MRCP) for choledocholithiasis in this patient is less than 50% page 356 . Routine preoperative ERCP will subject 70% to 80% of patients to an unnecessary procedure. Stabile BE. or jaundice can decrease the incidence of injury to the common bile duct by identifying unclear anatomy.Chang L. Am J Gastroenterol 99:2417page 357 . Ann Surg 229:449-457. Almela P. and is not related to the severity of the initial presentation or evidence of choledocholithiasis. Lewis RJ. Casillas J. This benefit of intraoperative cholangiography in complex cases is also seen in open cholecystectomy. Although most patients will not be harmed by waiting up to 30 days.American College of Surgeons SESAP 13 Critique for Category 4 Item 28 Up to 25% of patients with gallstone pancreatitis will have a recurrent attack within 30 days if they do not undergo cholecystectomy. If ERCP is to be used in an algorithm to manage patients with gallstone pancreatitis.Hallal AH. It is difficult to show that routine intraoperative cholangiography is beneficial in reducing biliary tract injury in uncomplicated cases because biliary duct injury occurs infrequently. selective postoperative ERCP minimizes exposure to an unnecessary procedure. Ann Surg 231:82-87. pancreatitis. This is related to both cannulation failure rate (reported at 5% to 10% for laparoscopic cholangiography) and misinterpretation of the cholangiogram obtained. choledocholithiasis. J Am Coll Surg 200:869-875. Ann R Coll Surg Engl 86:358-362. but an additional procedure may then be required to perform stone extraction. Pascual I. Valinsky LJ. Recurrence may take the form of biliary colic. Common duct stones in patients with gallstone pancreatitis can be identified by MRCP. potentially increasing cost and complications. et al: Recurrence of acute gallstone pancreatitis and relationship with cholecystectomy or endoscopic sphincterotomy. cholecystitis. Hobbs MS. or intraoperative cholangiography. or pancreatitis. However. 2005 5. Goodman AJ: Delayed cholecystectomy for gallstone pancreatitis: re-admissions and outcomes. 2004 2. et al: Complications of cholecystectomy: risks of the laparoscopic approach and protective effects of operative cholangiography: a population-based study. ERCP. Amortegui JD. et al: Magnetic resonance cholangiopancreatography accurately detects common bile duct stones in resolving gallstone pancreatitis. et al: Preoperative versus postoperative endoscopic retrograde cholangiopancreatography in mild to moderate gallstone pancreatitis: a prospective randomized trial.Fletcher DR.Cameron DR.Hernandez V. 2000 3. although the incidence of retained stones is not decreased when routine intraoperative cholangiography is used in either the open or laparoscopic approach. making the cost-benefit of routine MRCP questionable. routine intraoperative cholangiography performed during laparoscopic cholecystectomy in patients with cholecystitis. MRCP is 80% to 100% sensitive in detecting stones > 2 mm and can be used to identify retained stones. The decision to perform routine versus selective intraoperative cholangiography during the operative procedure centers on two potentially preventable outcomes—retained common duct stones and injury to the extrahepatic biliary system. Routine intraoperative cholangiography is performed to identify patients with choledocholithiasis. Jeroukhimov IM. Tan P. Anon R. Selection may be based on symptoms. 1999 4. Lo S. REFERENCES 1. laboratory evaluation. recurrence may be seen as early as 2 weeks after initial presentation. or results of intraoperative cholangiography. Naik KS. et al: Magnetic resonance cholangiopancreatography accurately predicts the presence or absence of choledocholithiasis.American College of Surgeons SESAP 13 2423. 1998 page 358 . Rofsky NM. 2004 6. Dobryansky M. J Gastrointest Surg 2:573-579.Hochwald SN. An abdominal computed tomographic (CT) scan reveals splenic vein thrombosis.American College of Surgeons SESAP 13 Category 4 Item 29 A 45-year-old woman is admitted with acute gallstone pancreatitis. Appropriate treatment now would be (A) therapeutic anticoagulation (B) observation (C) splenic artery embolization (D) splenectomy (E) upper endoscopy page 359 . However. Although esophageal varices in patients with isolated splenic vein thrombosis are even less common. particularly gallstone pancreatitis. Therapeutic anticoagulation is likely to exacerbate bleeding from any varices present. although effective for esophageal varices. occurring in fewer than 10% of patients. the recognized incidence is increasing with increased use of abdominal computed tomography (CT) in the care of patients with acute pancreatitis.McDermott VG. Newman GE: Case report: bleeding gastric varices secondary to page 360 . REFERENCES 1. most of these varices do not progress. Azeem S. there is no evidence that varices seen by esophagogastroduodenoscopy are any more likely to bleed compared with those seen only on CT. If an unrelated abdominal procedure is planned in a patient with known splenic vein thrombosis and varices. perhaps explaining why most patients do not develop variceal complications. with the prothrombotic state resolving after resolution of the pancreatitis. and there would be no recommended change in treatment based on results of esophagogastroduodenoscopy. Because the risk of bleeding in the absence of anticoagulation is low. in the setting of isolated splenic vein thrombosis it may do more harm than good. Galanko JA. this study is needed only in patients with occult or overt upper gastrointestinal bleeding. An improved understanding of the natural history of this disease has led to a change in management recommendations. Ann Surg 239:876-882. This may explain the difference in natural history of isolated splenic vein thrombosis secondary to acute pancreatitis. suggesting that splenectomy in these cases may have been somewhat prophylactic.American College of Surgeons SESAP 13 Critique for Category 4 Item 29 Isolated splenic vein thrombosis secondary to acute pancreatitis is most often a local phenomenon. In patients with gastric varices secondary to splenic vein thrombosis who have undergone repeat esophagogastroduodenoscopy. Bleeding complications from either esophageal or gastric varices in patients with isolated splenic vein thrombosis are infrequent. Splenectomy should be reserved for patients who have developed complications from gastric varices. England RE. Partial thrombosis is more common than complete thrombosis. In patients with splenic vein thrombosis secondary to acute pancreatitis. Fewer than half of patients with isolated splenic vein thrombosis go on to develop gastric varices. are not effective in controlling bleeding from gastric varices. As with patients who have esophageal varices secondary to portal hypertension. 2004 2. and splenic vein thrombosis secondary to other causes. This should be the primary treatment offered. upper gastrointestinal bleeding in patients with known gastric varices is as likely to come from gastritis or peptic ulcer disease as it is from the varices. Behrns KE: The natural history of pancreatitis-induced splenic vein thrombosis. they can occur if the coronary vein joins the splenic vein proximal to the thrombosis. Many series reporting higher rates of splenectomy in this condition include other operative procedures in up to one third of patients. sclerotherapy and banding. the incidence of isolated splenic vein thrombosis is less than 50%. The incidence of isolated splenic vein thrombosis in patients with acute pancreatitis is unknown because most patients are asymptomatic. Splenic artery embolization may provide an alternative to splenectomy in patients too ill to undergo an operative procedure. consideration should be given to performing a splenectomy. the infarcted spleen may lead to an abscess in up to 25% of patients.Heider TR. In patients with chronic pancreatitis. Weber SM.American College of Surgeons SESAP 13 splenic vein thrombosis successfully treated by splenic artery embolization. Br J Radiol 68:928-930. 2003 page 361 . World J Surg 27:1271-1274. 1995 3. Rikkers LF: Splenic vein thrombosis and gastrointestinal bleeding in chronic pancreatitis. American College of Surgeons SESAP 13 Category 4 Item 30 Which of the following statements about the surgical management of chronic pancreatitis is TRUE? (A) Puestow pancreaticojejunostomy results in lasting pain relief in 85% of patients (B) Pancreaticoduodenectomy rarely results in long-term pain relief in patients with chronic pancreatitis (C) Local resection of the head of the pancreas combined with longitudinal pancreaticojejunostomy (Frey procedure) is associated with a high risk of pancreatic endocrine insufficiency (D) Thoracoscopic splanchnicectomy is ineffective as a means of pain relief for chronic pancreatitis (E) Patients who continue to drink alcohol after a Frey procedure have a much higher risk for continued pain than those who discontinue alcohol consumption page 362 . pp 490-494 2. Mann O. Patients who continue to drink alcohol after the Frey procedure do not appear to be at a higher risk for recurrent pain. Bloechle C. Mosby. et al: Long-term follow-up of a randomized trial comparing the Beger and Frey procedures for patients suffering from chronic pancreatitis. Patients who are not good candidates for a ductal drainage procedure or who have recurrent pain after surgical drainage of the pancreatic duct can be treated with thoracoscopic splanchnicectomy. Denham W: Chronic pancreatitis.Strate T. This procedure can be performed with minimal morbidity. Ann Surg 241:591-598. and the results are more durable than the celiac plexus block.Izbicki JR. ed 8. Philadelphia. et al: Extended drainage versus resection in surgery for chronic pancreatitis: a prospective randomized trial comparing the longitudinal pancreaticojejunostomy combined with local pancreatic head excision with the pylorus-preserving pancreatoduodenectomy. but may be associated with a higher risk of morbidity and mortality. The Frey procedure appears to be equal to pancreaticoduodenectomy in relieving pain. 2005 page 363 .Bell RH Jr. The Frey procedure is associated with a 60% risk of pancreatic endocrine insufficiency. Ann Surg 228:771-779. 2005 3. Bloechle C. Knoefel WT.Frey CF. but only about one third of patients will remain pain free long-term. Reber HA: Local resection of the head of the pancreas with pancreaticojejunostomy. Pancreaticoduodenectomy is effective at producing long-term pain relief. J Gastrointest Surg 9:863-868.American College of Surgeons SESAP 13 Critique for Category 4 Item 30 A variety of surgical procedures have been used to treat the pain associated with chronic pancreatitis. Taherpour Z. in Cameron JL (ed): Current Surgical Therapy. Broering DC. REFERENCES 1. Lateral pancreaticojejunostomy or the Puestow procedure is associated with relatively low morbidity. Pancreatic exocrine insufficiency is seen in most patients with chronic pancreatitis regardless of the treatment used. 1998 4. 2004. The kidney.American College of Surgeons SESAP 13 Category 4 Item 31 A 46-year-old man had a nasogastric tube placed at elective sigmoid colectomy for diverticular disease. Which of the following statements is TRUE? (A) Correct placement of a nasogastric tube would have allowed return of gastrointestinal motility more rapidly (B) The lack of effective gastric aspiration in this patient increases the risk of anastomotic leak (C) This patient requires immediate re-exploration (D) This finding is associated with an increased risk of fascial dehiscence (E) This finding may resolve even if an etiology is not diagnosed or treated page 364 . Seven days later his abdomen is distended and nontender. and bladder (KUB) study shown is obtained. ureter. Immediate re-exploration is not required. Despite the common use of nasogastric tubes. Schein M: Early postoperative small bowel obstruction. urinary tract infection. Tse B. patients improve spontaneously without any underlying cause of the ileus being identified. infection. earlier return of bowel function occurred without the use of a nasogastric tube. a recent meta-analysis found that whereas vomiting and distention were more common in patients without nasogastric tubes. 2005 2. Often. Br J Surg 92:673-680. pneumonia. Underlying causes of persistent postoperative ileus should be investigated in this patient. No difference was found in the incidence of pulmonary complications or anastomotic leak rate. electrolyte imbalance.American College of Surgeons SESAP 13 Critique for Category 4 Item 31 This patient’s x-ray shows distended loops of small bowel with air throughout the small bowel and colon consistent with a postoperative ileus. return of bowel function. The nasogastric tube is coiled within the proximal stomach. however. Edwards S: Systematic review of prophylactic nasogastric decompression after abdominal operations. wound infection.Nelson R. 2004 page 365 . herniation. In subgroup analysis of patients undergoing colon resection. REFERENCES 1. fascial dehiscence. all other variables such as pulmonary aspiration. including intra-abdominal abscess. Br J Surg 91:683-691.Sajja SB. and excessive narcotic use. and prolonged hospital stay occurred more frequently with their use. wound separation. Which of the following statements is TRUE? (A) This is a rare complication of the previous operation (B) This diagnosis can be made with physical examination (C) Recurrence rate is lowest for a direct repair with suture (D) Esophagogastroduodenoscopy is indicated (E) A laparoscopic approach is not possible page 366 . Representative cuts are shown. A computed tomographic (CT) scan reveals no inflammatory changes between the aortic graft and the bowel.American College of Surgeons SESAP 13 Category 4 Item 32 A 67-year-old man presents with vague complaints of abdominal pain. He had a juxtarenal aneurysm repair many years ago. The success of this repair appears to be due to lack of tension on the wound. or hemodynamic instability. 2002 2. use of internal retention sutures. and the Nuttall procedure involving transposition of the rectus abdominus and its enveloping fascia. occurs far less frequently and usually presents with a history of melena. Various repairs for ventral hernia have been described. frank gastrointestinal bleeding. Key features of this approach include lateral placement of the trochars and at least a 3cm overlap of the mesh with the fascial edge. Dixon AK. most commonly with polypropylene and expanded polytetrafluoroethylene (PTFE).American College of Surgeons SESAP 13 Critique for Category 4 Item 32 The computed tomographic (CT) scan shows a large ventral hernia. Br J Surg 86:1243-1249. Recurrence rates range from 0 to 10% during the first year of followup.Toms AP. REFERENCES 1. Murphy JM. Given the size of the defect and the patient’s body habitus. In addition. hematemesis. Aortoduodenal fistula. including a modified Mayo technique with overlap of fascial edges. the CT scan demonstrates a clean tissue plane between the duodenum and the aorta. Esophagogastroduodenoscopy is not indicated in this patient. Recurrence rates from these procedures ranged from 7% to 44%. a Keel procedure using relaxing incisions in the lateral aspect of the anterior rectus sheath. Mesh repairs have been used since 1963. Br J Surg 89:534-545.Cassar K. this should be easily palpated on physical examination. Munro A: Surgical treatment of incisional hernia. Ventral hernia is one of the most common complications after abdominal aortic aneurysm repair and may occur in up to 20% of patients followed-up long-term. Successful laparoscopic repair of large incisional hernias has been described with the intraperitoneal placement of mesh as well. one of the most dreaded complications. Jamieson NV: Illustrated review of new imaging techniques in the diagnosis of abdominal wall hernias. 1999 page 367 . There are no peritoneal signs. She has some nausea but no vomiting. Additional diagnostic studies should include (A) colonoscopy (B) thoracentesis (C) bronchoscopy (D) upper gastrointestinal endoscopy (E) none page 368 . but does not radiate. Vital signs are normal. On physical examination she has diminished breath sounds in the lower right hemithorax and the absence of bowel sounds. as shown. The patient has not had a bowel movement nor passed flatus since the onset of symptoms. Anterior-posterior and lateral chest x-rays are obtained. It waxes and wanes. The pain began the morning after she did strenuous yard work. She has had no previous similar episodes and has had no surgical procedures.American College of Surgeons SESAP 13 Category 4 Item 33 A 49-year-old woman presents with a 24-hour history of upper abdominal discomfort. thoracentesis would be contraindicated due to the risk of visceral perforation and potential contamination. Ann Thorac Surg 77:1956-1959. Considering the classic appearance of the chest x-rays. Foramen of Morgagni hernia. Deans KJ. Although more commonly diagnosed in children. et al: Laparoscopy is useful in the diagnosis and management of foramen of Morgagni hernia in children. Bergamini TM: Foramen of Morgagni hernia: surgical consideration. Whether the approach is through a thoracotomy or an upper abdominal incision and using open or minimally invasive technique depends on surgical preference. a defect exists in the triangular space of Larrey where the costal and sternal fibrotendinous fibers fail to fuse with the central tendon of the diaphragm. REFERENCES 1. delayed presentation until adulthood is not unusual. Most of these defects occur on the right side (90%) and a few cases of bilateral defects have been reported. Nobuhara K. 2004 4. Lukish JR. Men outnumber women 4 to 1.Hinshaw LJ. pregnancy. little would be gained by any additional diagnostic studies. Surg Laparosc Endosc Percutan Tech 12:375-377. These findings are consistent with a foramen of Morgagni hernia. Powell D. Kim P. Semin Roentgenol 37:34.Minneci PC. 2000 3. a sudden increase in abdominal pressure. 2002 page 369 .Meredith K. Computed tomography (CT) may help identify which abdominal viscera will be found in the defect at exploration. Symptoms may vary from none or mild and nonspecific to those of overt strangulating obstruction. Allen J.American College of Surgeons SESAP 13 Critique for Category 4 Item 33 This patient’s anterior-posterior and lateral chest x-rays demonstrate air-fluid filled loops of bowel above the right hemidiaphragm.Ponsky TA. Mathisen DJ: Foramen of Morgagni hernia: changes in diagnosis and treatment. obesity. Anatomically. 2002 2. but most authors agree that reduction and repair are indicated once patients become symptomatic. Richardson JD. Collins J: Case of the season. or trauma. Frequently. herniation of abdominal viscera follows strenuous exercise. In fact. J Ky Med Assoc 98:286-288. but is not considered essential. American College of Surgeons SESAP 13 Category 4 Item 34 Randomized trials comparing laparoscopic versus open inguinal hernia repair suggest all of the following EXCEPT (A) increased incidence of intraoperative complications (B) similar incidence of life-threatening complications (C) similar rate of long-term complications (D) improved pain in short-term follow-up (E) increased recurrence rates at 2 years for surgeons with < 250 cases page 370 . suggesting a learning curve. Br J Surg 92:1085-1091. The VA group identified greater numbers of intraoperative complications with laparoscopic repair consisting of anestheticrelated problems (because laparoscopic repair must be performed under general anesthesia) and injury to vessels.Arvidsson D. with a trend toward increased neuralgia in the open group. randomized trials comparing laparoscopic with open repairs until 2004. Compared with open techniques. A similar prospective randomized trial comparing open Shouldice repair with TAPP repair found similar recurrence rates (6%) for the 2 groups at 5 years. Both require general anesthesia and mesh repair. it has been found to reduce immediate postoperative pain and shorten the time until return to work. 2004 page 371 . long-term complication rates appeared similar. portsite hernias. 2005 2. However. Jonasson O. Transabdominal preperitoneal (TAPP) and totally extraperitoneal (TEP) are the 2 laparoscopic techniques currently being used. et al: Randomized clinical trial comparing 5year recurrence rate after laparoscopic versus Shouldice repair of primary inguinal hernia. Return to daily activities was shorter among those having the laparoscopic repair. as well as an increased risk of lifethreatening complications as a result of myocardial infarction. Giobbie-Hurder A. REFERENCES 1. hemorrhage requiring re-operation. Surgeon experience of over 250 cases was associated with a decreased rate of recurrence in the laparoscopic group. Recurrence rates at 2 years also were higher for laparoscopic (10%) compared with open (5%) inguinal hernia repair. and respiratory insufficiency. N Engl J Med 350:1819-1827. et al: Open mesh versus laparoscopic mesh repair of inguinal hernia.Neumayer L. They also found a trend toward increased postoperative complications such as urinary retention and seroma formation in the laparoscopic group. but seemed to equalize after 3 months. Despite excellent retrospective results suggesting its safety. This Swedish study had an independent observer grade the surgeon’s performance and found a correlation between recurrence and operative technique. Pain control was statistically improved in the laparoscopic group in the immediate and early postoperative period. Leijonmarck CE. when laparoscopic inguinal hernia repair was compared with open Lichtenstein repair in a VA population. Berndsen FH. No such link to a surgeon’s experience was found in the open group.American College of Surgeons SESAP 13 Critique for Category 4 Item 34 Laparoscopic inguinal hernia repair has been used since the 1990s. Fitzgibbons R Jr. Larsson LG. there were no prospective. American College of Surgeons SESAP 13 Category 4 Item 35 Characteristics of patients who develop recurrences after laparoscopic ventral hernia repair (LVHR) include all of the following EXCEPT (A) larger hernia size (B) longer operative time (C) male sex (D) previous hernia repair (E) body mass index (BMI) > 40 page 372 . Voeller G: Laparoscopic repair of ventral hernias: nine years' experience with 850 consecutive hernias. intestinal/bladder injuries. longer operative times. 2003 2. et al: Prognosis factors in incisional hernia surgery: 25 years of experience. 2005 page 373 . Recurrences were not more common in males.7%. Ann Surg 238:391-399. the overall complication rate was 13%.American College of Surgeons SESAP 13 Critique for Category 4 Item 35 In the largest series studying laparoscopic ventral hernia repairs (LVHR) of 850 patients. Schaper A. seroma. Patients with recurrences were more likely to have larger hernias.Heniford BT. Kulle B. Park A. previous hernia repairs. Ramshaw BJ. REFERENCES 1. and pain.Langer C. and were obese (body mass index > 40). The overall recurrence rate was 4. Liersch T. The most common complications of LVHR were ileus. Hernia 9:16-21. On arrival. After receiving 4 units of packed RBCs. as shown. The most appropriate therapy now would be (A) activated factor VII (B) endoscopic variceal banding obliteration (C) transjugular intrahepatic portosystemic shunt (TIPS) (D) portosystemic shunt (E) beta blockade page 374 . Upper endoscopy is obtained. he is pale and tachycardic.American College of Surgeons SESAP 13 Category 4 Item 36 A 58-year-old man with a history of severe alcohol abuse is brought to the emergency department after an initial episode of hematemesis. he becomes stable. upper endoscopy is indicated to search for the etiology. After appropriate resuscitation. Activated factor VII is used in exceptional circumstances to control coagulopathic bleeding. Available at: http://www. and stricture. nor would this patient meet transplant criteria at this time. Liver transplantation is never considered in the acute situation. Sanyal AJ: Portal hypertensive bleeding. gastric varices. 2003 3.endoskopischer-atlas. Zervos EE: Management of variceal hemorrhage. and the treatment chosen depends on the etiology. Band ligation is usually chosen because it is associated with fewer complications such as perforation. esophageal varices. but would not be indicated in a patient for whom conservative measures have been successful. Potential causes include Mallory-Weiss tear. Gastroenterol Clin North Am 32:1079-1105.Atlas of Gastroenterological Endoscopy: Rubber band ligature.de/o21e. Methods for ablating varices without open operation include endoscopic sclerotherapy and endoscopic band ligation. or bleeding from peptic ulcer disease.American College of Surgeons SESAP 13 Critique for Category 4 Item 36 In this patient with presumed cirrhosis and upper gastrointestinal bleeding. Transjugular intrahepatic portosystemic shunt (TIPS) and portosystemic shunting would be options if endoscopic varix ablation is unsuccessful.Comar KM. mediastinitis. REFERENCES 1. the source of the bleeding must be identified.htm for banding 2. Ablative therapy to eliminate the varices is appropriate for hemorrhage secondary to esophageal varices. 2003 page 375 .Rosemurgy AS. Curr Probl Surg 40:263-343. American College of Surgeons SESAP 13 Category 4 Item 37 The standard for location of mesh implantation for open ventral hernia repair is (A) superficial to the anterior rectus sheath with at least 4 cm overlap (B) between the anterior rectus sheath and the anterior rectus muscle with at least 4 cm overlap (C) sutured to the fascial edge of the defect (D) deep to the posterior rectus sheath/transversalis fascia with at least 4 cm overlap (E) mesh reinforcement of fascial margins with linear closure page 376 . 2005 2. Kercher KW. REFERENCES 1. Mesh with barrier properties to prevent adherence to abdominal viscera should be used. Surg Clin North Am 83:1223-1234. there is still pressure applied to the mesh only through the hernia defect itself. If the mesh is placed anterior to the anterior fascia.American College of Surgeons SESAP 13 Critique for Category 4 Item 37 Several different techniques have been used for implantation of mesh into a ventral hernia defect. 2003 page 377 . Surg Clin North Am 85:91-103. which increases the amount of pressure on a smaller area of mesh. and can be placed by either an open or laparoscopic approach. Mesh in this position has been shown to be associated with the lowest recurrence rate. The goal of the repair is to implant the mesh to establish a tension-free repair.Cobb WS.Millikan KW: Incisional hernia repair. Posteriorly placed mesh is the only repair that distributes pressure over a larger area of mesh. even with 2 cm of overlap. using the mechanics of the abdominal wall to distribute pressure on the mesh over as large an area as possible. Heniford BT: Laparoscopic repair of incisional hernias. This similar principle applies to placement of the mesh anterior to the rectus muscle and also sutured to the fascial edge. a 65-year-old man presents with chronic right groin pain. Physical examination and computed tomographic (CT) scan do not reveal a recurrent hernia.American College of Surgeons SESAP 13 Category 4 Item 38 Nine months after right inguinal hernia repair. with the mesh placed in the anterior position. He has reproducible pain with percussion just medial to his right anterior superior iliac spine. The most appropriate step now would be (A) magnetic resonance imaging (MRI) of the lumbar spine (B) groin exploration (C) mesh excision and native tissue hernia repair (D) ilioinguinal and iliohypogastric nerve blocks (E) neurontin page 378 . The operative report describes routine tension-free herniorrhaphy with mesh. and complications from mesh placement are becoming more frequent. Am J Surg 189:283-287. prevention. The ilioinguinal and iliohypogastric nerve are at risk with an anterior approach. the ilioinguinal and iliohypogastric nerves should be blocked to see whether the patient’s pain is relieved.Amid PK: Causes. Hernia 8:343-349. 2004 2. Chronic pain from the mesh implantation itself tends not to be reproduced with percussion over the ilioinguinal nerve. Madura JA II.Madura JA. Physical examination and computed tomographic (CT) scanning are both reasonable options to make this evaluation. Worth RM: Inguinal neurectomy for inguinal nerve entrapment: an experience with 100 patients. Referral to a chronic pain clinic would not be appropriate. In the absence of any evidence of hernia on either of these examinations. For patients with chronic groin pain after hernia repair. and injury to one of the nerves that bring sensation to the groin and medial thigh. REFERENCES 1.American College of Surgeons SESAP 13 Critique for Category 4 Item 38 Mesh is almost universally accepted as the repair of choice for inguinal hernia. Early complications include wound infection and acute hernia recurrence. This evaluation looks for a nerve entrapment syndrome. but would not be indicated at this time. then ilioinguinal and/or iliohypogastric neurectomy would be indicated. It is a more dull chronic pain that exists in the inguinal floor itself. ruling out an occult hernia as an etiology is important. with the ilioinguinal nerve being involved most frequently. This patient’s physical findings do not indicate a need for lumbar spine MRI. Late complications include hernia recurrence. Copper CM. blind groin exploration is not indicated. and surgical treatment of postherniorrhaphy neropathic inguinodynia: triple neurectomy with proximal end implantation. Neurontin is used for groin pain after hernia repair. chronic pain from the mesh implantation itself. 2005 page 379 . Excision of the mesh and repair of the hernia with autogenous tissue is indicated in a patient with such a syndrome who presents 9 months after a hernia repair with mesh. To complete the patient’s work-up for postoperative hernia pain. If injection relieves the pain. American College of Surgeons SESAP 13 Category 4 Item 39 After a 4-day bout of diarrhea. an 85-year-old paraplegic resident of a chronic care facility is admitted with increasing abdominal distention over the last 48 hours. Abdominal examination reveals diffuse tympany without evidence for peritoneal irritation. The abdominal x-ray shown is obtained. The MOST likely contributing factor is (A) anemia (B) hyponatremia (C) hyperkalemia (D) previous abdominal operation (E) atherosclerosis page 380 . stroke. diffuse colonic distention in the absence of abdominal pain or peritoneal signs in this scenario make this an unlikely diagnosis. J Clin Gastroenterol 39:17-20. acute renal failure. myocardial infarction. Patients treated with diuretics or other medications affecting electrolytes often demonstrate abnormal colonic motility and present with diffuse colonic distention. who are relatively immobile. Verne. are often observed in elderly patients confined to nursing facilities. GN: New solutions to an old problem: acute colonic pseudo-obstruction.Ponec RJ. Although visceral atherosclerosis might contribute to bowel ischemia. Kimmey MB: Neostigmine for the treatment of acute colonic pseudoobstruction.American College of Surgeons SESAP 13 Critique for Category 4 Item 39 The x-ray is most consistent with acute colonic pseudo-obstruction. 2005 page 381 . Acute colonic pseudo-obstruction occurs in association with precipitating medical or surgical conditions but in the absence of obvious colonic disease or mechanical obstruction. burns. in particular hyokalemia and hyponatremia. The presence of anemia would not be expected to contribute to the development of this syndrome in the absence of other predisposing factors. 2005 2. and metastatic malignancy. sepsis. trauma. Electrolyte abnormalities. congestive heart failure. Although prior abdominal operation is a predisposing factor for adhesive mechanical obstruction. it would not present with this radiologic picture of diffuse colonic distention. evidenced by gas throughout the dilated colon and rectum.Saunders MD. Kimmey MB: Systematic review: acute colonic pseudo-obstruction.Fazel A. REFERENCES 1. Predisposing factors include acute surgical procedures. Saunders MD. pneumonia. acute respiratory failure. 1999 3. Aliment Pharmacol Ther 22:917-925. N Engl J Med 341:137-141. the patient develops upper gastrointestinal bleeding from esophageal varices and requires 4 units of packed RBCs. At operation. a defect containing infarcted omentum is identified and repaired. On postoperative day 1.American College of Surgeons SESAP 13 Category 4 Item 40 A 52-year-old diabetic woman with Child’s C cirrhosis undergoes repair of an incarcerated umbilical hernia. On the fifth postoperative day. her abdominal wound is painful and erythematous. All of the following are significant potential risk factors for developing this surgical site infection EXCEPT (A) intraoperative hyperglycemia (B) perioperative hypoxemia (C) postoperative transfusion (D) cirrhosis (E) age page 382 . but the effect may be due to postoperative anemia and poor tissue oxygen delivery. several inherent and modifiable risk factors have also been identified that dramatically affect the incidence of surgical site infections. prolonging hospital stays and increasing total surgical costs. and avoidance of perioperative hypoxemia are independent practice factors that can be manipulated to decrease the chance of developing a surgical site infection. malnutrition. Lermite E. particularly cirrhosis. cardiac. 2002 2. Gannon C. et al: Risk factors for prediction of surgical site infections in "clean surgery. et al: Surgical site infections: reanalysis of risk factors. and the presence of a preoperative urinary catheter) that more than double the incidence of surgical site infections postoperatively. J Surg Res 103:89-95. Maintenance of perioperative normothermia.Pessaux P. Tracy JK. Atallah D. 2003 page 383 .Malone DL.American College of Surgeons SESAP 13 Critique for Category 4 Item 40 Surgical site infections remain a major source of postoperative morbidity. Msika S. Numerous studies have documented that postoperative blood transfusion is associated with an increase in surgical site infections. respiratory and/or renal failure. Hay JM. 2005 3. Msika S.” Am J Infect Control 33:292-298. Although appropriately timed antibiotic prophylaxis (administered within 1 hour prior to incision and discontinued within 24 hours or after 1 or 2 doses) effectively decreases infectious wound complications. Arch Surg 138:314-324. Multivariate analyses have identified variables independent of preoperative factors (such as evidence of chronic illnesses.Pessaux P. Atalla D. strict glycemic control. et al: Risk factors for postoperative infectious complications in noncolorectal abdominal surgery: a multivariate analysis based on a prospective multicenter study of 4718 patients. Genuit T. age is not a risk factor for surgical site infection or deep parietal infections. REFERENCES 1. Although age leads to a progressive reduction in immunity and has been identified as a risk factor for global postoperative infectious complications. Low partial pressure of oxygen in contaminated tissue has been hypothesized to define an ideal infectious environment. American College of Surgeons SESAP 13 Category 4 Item 41 Which of the following statements about the risks and benefits of prophylactic cholecystectomy for asymptomatic paraplegics found to have cholelithiasis on routine ultrasound screening for renal stones is TRUE? (A) They are at greater risk than the general population for developing acute cholecystitis (B) The conversion rate from a laparoscopic to open approach is no different than for the general population (C) A diagnosis of cholecystitis would likely be delayed due to insensate abdomen (D) The incidence of cholelithiasis symptoms increases by at least 10% per year (E) They are at greater risk than the general population for developing biliary complications such as pancreatitis page 384 . Whereas the proportion of spinal cord injury patients with cholelithiasis is greater than in the general population.Moonka R. J Am Coll Surg 189:274-281. et al: The prevalence and natural history of gallstones in spinal cord injured patients. Rudderow DJ. et al: Cholecystectomy in patients with previous spinal cord injury. including gallstone pancreatitis. The 6.American College of Surgeons SESAP 13 Critique for Category 4 Item 41 Patients with spinal cord injury should undergo annual screening with ultrasonography due to a significantly increased incidence of asymptomatic nephrolithiasis. Patients with cervical and high thoracic lesions are often capable of describing classic symptoms of biliary colic. 1999 page 385 . Am J Surg 178:246-250. Stelzner M: The presentation of gallstones and results of biliary surgery in a spinal cord injured population. 1999 3. Eubank WB.Ahmed HU. and virtually all patients with cholecystitis. Smith JB. Although some studies have suggested that patients with insensate abdomens are at risk for diagnostic delays and thus may present with more advanced intraperitoneal inflammatory diseases. 2002 2. does not appear to be higher. choledocholithiasis. the incidence and severity of biliary disease. REFERENCES 1. McDonald JM. asymptomatic cholelithiasis is often found incidentally.Moonka R. Resnick WJ. this assumption does not extend to biliary disease. Stiens SA. Am J Surg 184:452-459.3% annual cholecystectomy rate among patients with spinal cord injury exceeds that of the general population. Longo WE. Stiens SA. but prophylactic cholecystectomy would not be warranted. and pancreatitis have clinical presentations highly suggestive of the correct diagnosis. inferior to the inguinal ligament. with an exquisitely tender mass in the right groin. vomiting. On physical examination.American College of Surgeons SESAP 13 Category 4 Item 42 An obese 53-year-old woman presents acutely with nausea. The BEST treatment strategy would be (A) a transverse incision above the typical herniorrhaphy incision (B) incision and drainage (C) an attempt at manual reduction after generous sedation (D) total extraperitoneal patch (TEPP) hernia repair (E) laparoscopic transabdominal pre-peritoneal (TAPP) hernia repair page 386 . she appears quite ill. The patient’s WBC count is 16.1°F. and abdominal distention.000/mm3 and her temperature is 102. Colborn GL. transabdominal approach. At operation. 2005 page 387 .American College of Surgeons SESAP 13 Critique for Category 4 Item 42 This patient is most likely systemically ill due to an incarcerated and gangrenous femoral hernia. Division of the inguinal ligament is often helpful. nor should permanent mesh be used in the setting of a bowel resection with contamination of the operative field. Mangiante EC.Mirilas P. 1991 2. Though this condition may be confused with a thigh abscess. The method of repair is dictated by the operative findings. Britt LG: Preperitoneal herniorrhaphy for the acutely incarcerated groin hernia. and bowel resection will almost certainly be necessary. Voeller GR. The best control of the ischemic. control of the proximal and distal ends of the involved bowel is obtained prior to releasing the hernia contents. with pressure pneumoperitoneum and permanent mesh. Any attempt at reduction would be associated with the risk of bowel perforation and generalized peritonitis. If primary tissue repair cannot be accomplished. Arch Surg 140:90-94. McClusky DA 3rd. then use of nonpermanent biological materials should be considered as an option in a contaminated field. The segment of incarcerated bowel must be controlled before reduction and inspected carefully for viability. and risk generalized peritonitis in the search for the involved segment. Both total extraperitoneal patch (TEPP) and transabdominal pre-peritoneal (TAPP) hernia repairs are typically done laparoscopically. incarcerated bowel is via an open. et al: The history of anatomy and surgery of the preperitoneal space. vomiting and abdominal distention point to entrapped bowel with ischemia. REFERENCES 1. Skandalakis LJ. the nausea. Am Surg 57:139-141. The relaxation of anesthesia plus pneumoperitoneum may reduce the gangrenous bowel prior to control.George SM Jr. 1.8 mg/dL. The next step in her evaluation should be (A) repeat ultrasound study (B) biliary excretion. bilirubin is 4. serum bilirubin. eg. Physical examination shows mild epigastric tenderness without guarding or rebound. The following day. HIDA. scan (C) abdominal computed tomography (CT) (D) liver biopsy (E) endoscopic retrograde cholangiopancreatography (ERCP) page 388 .6 mg/dL. Her temperature and vital signs are normal except for sinus tachycardia of 105.American College of Surgeons SESAP 13 Category 4 Item 43 A 55-year-old woman with a body mass index of 32 presents to the emergency department with an 18-hour history of right upper quadrant pain radiating to the back. an abdominal ultrasound study showed small gallstones with a normal biliary tree. Two months ago. 354 U/L. A repeat ultrasound confirms gallstones without evidence of cholecystitis and a common bile duct diameter of 8 mm. and alkaline phosphatase. She looks uncomfortable. and alkaline phosphatase has increased to 175 U/L. Pertinent data include serum amylase. her pain has not improved. amylase has decreased to 215 U/L. 86 U/L. Dig Surg 15:323-327. might be useful if the biliary tree were not adequately visualized or if there were a concern for ongoing or unresolved pancreatitis. and may be used as a screening study for patients at high risk for choledocholithiasis. the patient’s chance of having a common bile duct stone is at least 30%. Ranstam J: Factors predictive of bile duct stones in patients with acute calculous cholecystitis. it has a 1% to 2% incidence of significant complications (pancreatitis.Golub R. Andersen DK. Although her amylase has decreased. which is a 4fold increase compared with patients without rising values. or even an abdominal computed tomographic (CT) scan. Cantu R Jr. and potentially time-consuming. 1998 4. Two other alternatives are also worth reviewing. her chances of having choledocholithiasis are 60% to 80%. 1998 page 389 . Although ERCP is fairly safe. REFERENCES 1. bilirubin and alkaline phosphatase have both increased. HIDA. Liver biopsy is used to evaluate parenchymal diseases of the liver such as hepatitis or cirrhosis. bleeding. Preoperative endoscopic retrograde cholangiopancreatography (ERCP) should be used to evaluate the common bile duct and retrieve stones prior to laparoscopic cholecystectomy. 2006 3. it requires significant expertise for accurate interpretation. Tan M: The prediction of common bile duct stones using a neural network. In this scenario with rising laboratory values. Am J Gastroenterol 96:3305—3311. Biliary excretion. A second alternative would be to perform an endoscopic ultrasound (EUS) and proceed to ERCP only if a common duct stone was detected. Ideally. et al: Prediction of bile duct stones and complications in gallstone pancreatitis using early laboratory trends.Hammarstrom L. expensive. MRCP is nontherapeutic. Duffy AJ: Choledocholithiasis: evolving standards for diagnosis and management. Wells CK. Given her age and the diameter of her common bile duct. Magnetic resonance cholangiopancreatography (MRCP) has roughly the same accuracy for diagnosing choledocholithiasis as ERCP and intraoperative cholangiogram. This allows therapy to follow immediately. scan is unlikely to change the management of this patient because it would not provide clinical or sonographic evidence for cholecystitis. World J Gastroenterol 12:3162-3167.American College of Surgeons SESAP 13 Critique of Category 4 Item 43 This patient's presentation is most consistent with gallstone pancreatitis. eg. Bell RL. J Am Coll Surg 187:584-590. ERCP would be performed only on patients with documented common bile duct stones. neither of which is suggested by the history.Freitas ML. perforation). and has a success rate of 90% for common bile duct stone removal. Slezak L. 2001 2.Cohen ME. A repeat ultrasound study. her epigastric pain resolves. 1. Laparoscopic cholecystectomy should be attempted (A) after endoscopic retrograde cholangiopancreatography (ERCP) and sphincterotomy (B) prior to discharge (C) once her amylase is normal (D) 4 to 6 weeks later (E) only if the patient develops recurrent pancreatitis page 390 . After 2 days of medical management. Pertinent data include amylase.000/mm3. 15. bilirubin. Her amylase is 340 U/L and her bilirubin and WBC count have returned to normal.American College of Surgeons SESAP 13 Category 4 Item 44 A 39-year-old woman is admitted with gallstone pancreatitis and epigastric pain.2 mg/dL. and WBC count. 2000 U/L. each with approximately a 5% risk for severe pancreatitis. This patient is statistically unlikely to have a retained common bile duct stone and would not require preoperative endoscopic retrograde cholangiopancreatography (ERCP). In good-risk operative candidates with resolving mild biliary pancreatitis. Such patients should have a cholecystectomy prior to discharge because they are at risk for repeated attacks. et al: Timing of cholecystectomy for acute biliary pancreatitis: outcomes of cholecystectomy on first admission and after recurrent biliary pancreatitis. Imrie C.Taylor E. Both her symptoms and abnormal laboratory data are resolving.American College of Surgeons SESAP 13 Critique for Category 4 Item 44 This patient had an episode of gallstone pancreatitis. waiting for the amylase to return to normal is unnecessary and only adds to delay. 2003 2. Stabile BE. Ann Surg 231:82-87. Lewis RJ. Wong C: The optimal timing of laparoscopic cholecystectomy in mild gallstone pancreatitis. 2004 4. delaying operation for 4 to 6 weeks subjects the patient to a 30% to 65% risk of recurrent symptoms or pancreatitis. 2000 3.Chang L. et al: IAP Guidelines for the Surgical Management of Acute Pancreatitis. The success rate of laparoscopic cholecystectomy in this setting is 85% to 100% with a morbidity rate of 10% compared with a morbidity rate of 40% to 45% for laparoscopic cholecystectomy after a recurrent episode. Lo S. Ozkan OV. Am Surg 70:971-975. REFERENCES 1. World J Surg 27:256-259. et al: Preoperative versus postoperative endoscopic retrograde cholangiopancreatography in mild to moderate gallstone pancreatitis: a prospective randomized trial. Akcakaya A. Bassi C. Selective rather than routine use of preoperative ERCP is associated with a significant reduction in hospital stay and costs.Alimoglu O. Warshaw A.Uhl W. Sahin M. Pancreatology 2:565—573. 2002 page 391 . Although many practitioners have advocated a cooling-off period. American College of Surgeons SESAP 13 Category 4 Item 45 When compared with open repair. laproscopic ventral hernia repair has increased (A) postoperative complications (B) postoperative ileus (C) wound problems (D) length of stay (E) operative time page 392 . Ventral hernias may either be primary hernias or the result of failure of a prior midline laparotomy incision (which occurs in up to 20% of all laparotomies). Finlayson SR. Sanchez LJ. Surg Endosc 20:1030-1035. Surg Endosc 17:17781780. mesh repair was demonstrated to be superior to suture repair for ventral hernias.Bencini L. et al: Incisional hernia repair: retrospective comparison of laparoscopic and open techniques. Most studies show longer operative times with laparoscopic ventral hernia repair.Lomanto D.McGreevy JM. which may be due to the lower narcotic use with laparoscopic repair. Shabbir A. laparoscopic ventral hernia repair has a lower complication rate (8% to 26% versus 21% to 44%) due principally to decreased wound problems and a reduction in ileus. open hernia repair with mesh has been associated with a modest complication rate and significant postoperative pain. et al: A prospective study comparing the complication rates between laparoscopic and open ventral hernia repairs. Cheah WK: Laparoscopic versus open ventral hernia mesh repair: a prospective study. In a prospective randomized trial.000 ventral hernia repairs are performed annually in the United States. Boffi B. 2003 page 393 . reducing the recurrence rate by half. However. Surg Endosc 17:1546-1551. 2006 3. Farsi M. Mesh hernia repair has become the preference in many centers.American College of Surgeons SESAP 13 Critique for Category 4 Item 45 Over 100. Iyer SG. Reduced complication rates have translated into a shorter length of stay. In most studies. Birkmeyer CM. Goodney PP. REFERENCES 1. 2003 2. Laparoscopic mesh hernia repair adheres to the same principles as open – retrofascial placement with a large overlap – while avoiding the morbidity of a larger incision. After transfusion of 3 units of packed RBCs. The blood seems to come from the edge of the stoma. which are not bleeding actively. Flexible ileoscopy of the distal 25 cm of small bowel reveals no intraluminal blood. her ostomy bag continues to fill up with blood. Gastroscopy shows only esophageal varices. The best initial management would be (A) intravenous medical therapy to reduce portal pressure (B) direct suture-ligation of the bleeding site (C) transjugular intrahepatic portosystemic shunt (TIPS) (D) emergency ileostomy revision (E) emergency surgical portosystemic shunt page 394 . A picture of the stoma is shown.American College of Surgeons SESAP 13 Category 4 Item 46 A 55-year-old alcoholic with refractory ascites and an end-ileostomy is admitted for massive gastrointestinal bleeding. St. 1999 page 395 . or portosystemic shunting with transjugular intrahepatic portosystemic shunt (TIPS) to decrease the portosystemic pressure gradient. recurrent bleeding is common. and portal hypertension. Nivatvongs S (eds): Principles and Practice of Surgery for the Colon. Predisposing factors include primary sclerosing cholangitis. may be necessary. REFERENCES 1. and Anus. Brophy DP. in Gordon PH. Adrenalin-soaked sponges may be helpful. 1999. Bubrick MP: Intestinal stomas. Stomal varices are portosystemic collaterals between the high-pressure portal system and the low-pressure abdominal wall venous system at the mucocutaneous junction between the ostomy and the skin. cirrhosis. Definitive surgical portal decompression is associated with a high morbidity and mortality risk. Even with successful initial control. Quality Medical Publishing Inc. Rectum.Shibata D. The typical appearance is shown here as a caput medusae. Bleeding from ileal varices can be massive. these vessels are quite small. Sutureligation of the bleeding site or injection with sclerosing agents usually controls the acute hemorrhage. Immediate control of the bleeding is obtained by direct pressure on the bleeding site. Dis Colon Rectum 42:15811585. Stoma revision to disconnect the parastomal portosystemic collaterals. pp 1117-1180 2. Rolstad BS. et al: Transjugular intrahepatic portosystemic shunt for treatment of bleeding ectopic varices with portal hypertension. with circumferential bluish peristomal discoloration. One fourth to one half of patients with varices have significant hemorrhage. Gordon FD.American College of Surgeons SESAP 13 Critique for Category 4 Item 46 Ectopic parastomal varices are seen in up to 50% of patients with stomas and portal hypertension. and is not recommended unless all other reasonable avenues have been exhausted. Intravenous medical therapy to reduce portal pressure is time-consuming and ineffective in the face of acute hemorrhage.Gordon PH. Anastopoulos HT. ed 2. Louis. and mortality from bleeding is 3% to 4%. Unlike esophageal varices. Computed tomography (CT) demonstrates a large splenic cyst.American College of Surgeons SESAP 13 Category 4 Item 47 A 20-year-old woman has had increasingly bothersome left upper quadrant discomfort over the last several months. The single best treatment would be (A) nonsteroidal anti-inflammatory agents (B) percutaneous aspiration (C) angiographic embolization (D) partial splenectomy (E) total splenectomy page 396 . Primary cysts are generally classified as parasitic. The spleen is the third most common site of such infestation after the liver and lung. Neoplastic cysts represent cystic degeneration of primary or metastatic tumors of the spleen. REFERENCES 1. Moller AC: Splenic cysts. 2004 2. Neoplastic and dermoid cysts require splenectomy. The most common cause of the parasitic cysts is infection with Echinococcus spp. often referred to as simple cysts. as shown. and are truly pseudocysts. Some authors lump the congenital and neoplastic cysts in a nonparasitic category. These represent cystic dilation of embryonic inclusions of epithelial cells from adjacent structures. Approximately 75% of all splenic cysts are posttraumatic in nature.Hansen MB. with the goal of conserving as much normal spleen as possible. Dermoid cysts are cystic teratomas with all three germ cell layers and are the most rare of all splenic cysts. followed by resection. Asymptomatic cysts over 5 cm in diameter should also be treated with partial splenectomy. Parasitic cysts should be treated medically as any other hytadid cyst. Surg Laporosc Endosc Percutan Tech 14:316-322. Symptomatic cysts should be treated with partial splenectomy. J Am Coll Surg 194:306-314.Morgenstern L: Nonparasitic splenic cysts: pathogenesis. NSAIDs and angiographic embolization have no role. Treatment is based on cyst type. Over 30% of patients with such pseudocysts cannot recall the traumatic event. Percutaneous aspiration is often followed by cyst recurrence and potential bleeding problems. such as hemangiomas or lymphangiomas. Of these. primarily to rule out neoplasia. the epidermoid cysts are the most common. 2002 page 397 . congenital.American College of Surgeons SESAP 13 Critique for Category 4 Item 47 Splenic cysts overall are very uncommon. There are a variety of causes of splenic cysts and a useful classification scheme has been developed. classification. and neoplastic. Congential cysts are the most common type of primary cyst. Endodermoid cysts represent cystic vascular lesions. Secondary pseudocyst and small epidermoid or endodermoid cysts that are asymptomatic may be safely observed. and treatment. 400/mm3.hour history of excruciating epigastric pain. blood pressure is 90/60. Pulse is 115/min. The abdominal computed tomographic (CT) scan shown is obtained. Immediate management should include (A) peritoneal dialysis (B) exploratory laparotomy (C) needle aspiration (D) fluid resuscitation (E) endoscopic retrograde cholangiopancreatography (ERCP) page 398 . and WBC count is 16. He is afebrile and not jaundiced.American College of Surgeons SESAP 13 Category 4 Item 48 A 42-year-old previously healthy man arrives in the emergency department with a 12. Acute pseudocysts are enclosed by a wall of fibrous or granulation tissue. This was not a statistically significant difference. surgical debridement and antibiotics are recommended. Antibiotics may have a role in patients with necrotizing pancreatitis and should be used when infection is proven in the pancreatic bed. A pseudocyst is a cystic cavity bound to the pancreas by inflammatory tissue. usually in proximity with the pancreas. Chronic pseudocysts are enclosed by a wall of fibrous or granulation tissue. If infected necrosis is confirmed. leucocytosis. Protracted courses of antibiotics should not be used routinely. An acute pseudocyst is a fluid collection arising in association with an episode of acute pancreatitis. and should be suspected clinically when there is increased abdominal pain. In patients with signs of sepsis. fever. infectious complications account for approximately 80% of deaths from acute pancreatitis. It is beneficial in biliary pancreatitis with biliary obstruction or cholangitis. cardiovascular. There is no consensus about the timing of operation. The formation of a pseudocyst usually requires 4 or more weeks from the onset of acute pancreatitis. CT. but its role in severe acute biliary pancreatitis without evidence of biliary obstruction or cholangitis is unclear. although the current trend is toward delayed operation. However. the mortality rate was 58% in the early operation group versus 27% in the late operation group. but offers no advantage over appropriate fluid resuscitation. and the study was stopped because of a high complication rate in the early operation group. Infection in a necrotic pancreas significantly increases the risk of death. Endoscopic retrograde cholangiopancreatography (ERCP) is the most valuable method of diagnosing and treating ductal stones. Respiratory. Delayed operation offers better demarcation of necrotic tissue in a resuscitated patient. and/or organ failure. Ultrasonography is ineffective for the grading of acute pancreatitis or detection of necrosis because the gland is poorly visualized in up to 30% of cases. arising as a consequence of acute pancreatitis or pancreatic trauma. Early operation identifies a friable necrotic mass without normal tissue planes and the distinction between viable and nonviable tissue may be impossible. Pancreatic abscess is a circumscribed intra-abdominal collection of pus. A pseudocyst is usually rich in pancreatic enzymes and is most often sterile. The value of prophylactic antibiotics has been debated for over half a century. and intensive support of any failing organ or system. oxygen supplementation. Infections occur in 30% to 40% of patients who have over 30% necrosis of the pancreas. Furthermore. Peritoneal dialysis has been used. lasting more than 4 weeks and surrounded by a defined wall. Early ERCP is not indicated for mild acute pancreatitis or for nonbiliary pancreatitis. Fluid collections lasting less than 4 weeks that lack a page 399 . but its role in the management of acute pancreatitis is debatable. and renal function must be carefully monitored.or ultrasonographically guided aspiration of fluid should be performed for examination and culture to confirm infected necrosis. It may also stimulate pancreatitis and in general is not used unless biliary obstruction is still believed to be present. Rapid restoration and maintenance of intravascular fluid volume requires the administration of fluids at a rate of 300 to 500 mL/hour and simultaneous correction of electrolyte abnormalities. In a randomized clinical trial comparing early versus late necrosectomy in patients with necrotizing pancreatitis.American College of Surgeons SESAP 13 Critique for Category 4 Item 48 The treatment of severe acute pancreatitis includes aggressive fluid resuscitation. there are no specific signs or symptoms to differentiate sterile from infected necrosis. American College of Surgeons SESAP 13 defined wall are more properly termed acute fluid collections. Dervenis C: Pancreatic pseudocysts in the 21st century. REFERENCES 1. van Westerloo DJ. JOP 5:8-24. Only 1 of 13 pseudocysts present for 7 to 12 weeks resolved spontaneously. van Dorp WT. Aspiration of a pseudocyst is usually not performed because of the risk of an external fistula forming. 2003 page 400 . Pseudocysts < 5 cm in size were more likely to resolve than larger ones.Bruno MJ. Uhl W. spontaneous resolution occurred in 10 (42%) of 24 patients in whom a pseudocyst was present for less than 6 weeks. Diamond T: Management of severe acute pancreatitis. Pseudocysts of more than 4 cm resolved spontaneously at a mean of 3 months after diagnosis. Buchler MW: Management of infection in acute pancreatitis. 2000 4. Chronic pseudocysts have a well-defined wall usually detected nonivasively by a CT scan. Dekker W. and possible aetiology. 2004 2. clinical course. anatomic considerations and treatment. Dervenis C: Pancreatic pseudocysts in the 21st century. Br J Surg 90:407420. pathophysiology.Andren-Sandberg A. Cysts presenting more than 6 weeks after an attack of acute pancreatitis are unlikely to resolve. overall resolution of the pseudocyst occurred in 57% of the 24 patients with satisfactory radiographic follow-up.Andren-Sandberg A. 2002 5. Part I: classification. JOP 5:64-70.Yousaf M. Werner J. In a series of 93 patients with acute pancreatitis and 62 who had chronic disease. and 38% resolved more than 6 months after diagnosis. outcome. 2004 3. In a series of 68 patients treated expectantly. et al: Acute pancreatitis in peritoneal dialysis and haemodialysis: risk. J Hepatobiliary Pancreat Surg 9:423-428. Gut 46:385-389. Part II: natural history. McCallion K.Hartwig W. The median time to regression was 29 weeks (range 2 to143) and the independent predictive factor of pseudocyst resolution or asymptomatic course was size < 4 cm. and WBC count is 16.hour history of excruciating epigastric pain. Pulse is 115/min.400/mm3. The abdominal computed tomographic (CT) scan shown in the previous item is obtained. He is afebrile and not jaundiced. Vital signs are normal. Four weeks later. The computed tomographic (CT) scan shown is obtained. blood pressure is 90/60. Treatment now should be (A) endoscopic retrograde cholangiopancreatography (ERCP) (B) surgical decompression (C) percutaneous aspiration (D) 14-day course of antibiotics (E) repeat CT scan in 30 days page 401 .American College of Surgeons SESAP 13 Category 4 Item 49 A 42-year-old previously healthy man arrives in the emergency department with a 12. the patient returns to the emergency department with abdominal fullness. It is beneficial in biliary pancreatitis with biliary obstruction or cholangitis. but offers no advantage over appropriate fluid resuscitation. Peritoneal dialysis has been used. The formation of a pseudocyst usually requires 4 or more weeks from the onset of acute pancreatitis. arising as a consequence of acute pancreatitis or pancreatic trauma. and/or organ failure.or ultrasonographically guided aspiration of fluid should be performed for examination and culture to confirm infected necrosis. cardiovascular. but its role in the management of acute pancreatitis is debatable. Pancreatic abscess is a circumscribed intra-abdominal collection of pus. infectious complications account for approximately 80% of deaths from acute pancreatitis. Early ERCP is not indicated for mild acute pancreatitis or for nonbiliary pancreatitis. An acute pseudocyst is a fluid collection arising in association with an episode of acute pancreatitis. However. In a randomized clinical trial comparing early versus late necrosectomy in patients with necrotizing pancreatitis. and intensive support of any failing organ or system. Antibiotics may have a role in patients with necrotizing pancreatitis and should be used when infection is proven in the pancreatic bed. Delayed operation offers better demarcation of necrotic tissue in a resuscitated patient. and renal function must be carefully monitored. Acute pseudocysts are enclosed by a wall of fibrous or granulation tissue. surgical debridement and antibiotics are recommended. Rapid restoration and maintenance of intravascular fluid volume requires the administration of fluids at a rate of 300 to 500 mL/hour and simultaneous correction of electrolyte abnormalities. Respiratory. Early operation identifies a friable necrotic mass without normal tissue planes and the distinction between viable and nonviable tissue may be impossible. There is no consensus about the timing of operation. Infection in a necrotic pancreas significantly increases the risk of death. Protracted courses of antibiotics should not be used routinely. The value of prophylactic antibiotics has been debated for over half a century. CT. It may also stimulate pancreatitis and in general is not used unless biliary obstruction is still believed to be present. and the study was stopped because of a high complication rate in the early operation group. usually in proximity with the pancreas. but its role in severe acute biliary pancreatitis without evidence of biliary obstruction or cholangitis is unclear. This was not a statistically significant difference. Furthermore. In patients with signs of sepsis. Infections occur in 30% to 40% of patients who have over 30% necrosis of the pancreas. Endoscopic retrograde cholangiopancreatography (ERCP) is the most valuable method of diagnosing and treating ductal stones. lasting more than 4 weeks and surrounded by a defined wall. the mortality rate was 58% in the early operation group versus 27% in the late operation group. although the current trend is toward delayed operation. leucocytosis. A pseudocyst is a cystic cavity bound to the pancreas by inflammatory tissue.American College of Surgeons SESAP 13 Critique for Category 4 Item 49 The treatment of severe acute pancreatitis includes aggressive fluid resuscitation. Fluid collections lasting less than 4 weeks that lack a page 402 . If infected necrosis is confirmed. oxygen supplementation. A pseudocyst is usually rich in pancreatic enzymes and is most often sterile. fever. and should be suspected clinically when there is increased abdominal pain. Chronic pseudocysts are enclosed by a wall of fibrous or granulation tissue. there are no specific signs or symptoms to differentiate sterile from infected necrosis. Ultrasonography is ineffective for the grading of acute pancreatitis or detection of necrosis because the gland is poorly visualized in up to 30% of cases. Diamond T: Management of severe acute pancreatitis. REFERENCES 1. van Dorp WT. and 38% resolved more than 6 months after diagnosis. Pseudocysts < 5 cm in size were more likely to resolve than larger ones. Aspiration of a pseudocyst is usually not performed because of the risk of an external fistula forming. 2000 4. Dervenis C: Pancreatic pseudocysts in the 21st century. outcome. 2004 2. Br J Surg 90:407420. Buchler MW: Management of infection in acute pancreatitis.Hartwig W.Andren-Sandberg A. 2004 3. and possible aetiology. clinical course.Andren-Sandberg A. J Hepatobiliary Pancreat Surg 9:423-428.Yousaf M. 2003 page 403 . Pseudocysts of more than 4 cm resolved spontaneously at a mean of 3 months after diagnosis. 2002 5. anatomic considerations and treatment. Part II: natural history. JOP 5:64-70. van Westerloo DJ. Chronic pseudocysts have a well-defined wall usually detected nonivasively by a CT scan. spontaneous resolution occurred in 10 (42%) of 24 patients in whom a pseudocyst was present for less than 6 weeks. Werner J.American College of Surgeons SESAP 13 defined wall are more properly termed acute fluid collections. Gut 46:385-389. In a series of 93 patients with acute pancreatitis and 62 who had chronic disease. Part I: classification. et al: Acute pancreatitis in peritoneal dialysis and haemodialysis: risk. Cysts presenting more than 6 weeks after an attack of acute pancreatitis are unlikely to resolve. Dervenis C: Pancreatic pseudocysts in the 21st century. JOP 5:8-24. Dekker W.Bruno MJ. Uhl W. overall resolution of the pseudocyst occurred in 57% of the 24 patients with satisfactory radiographic follow-up. McCallion K. pathophysiology. Only 1 of 13 pseudocysts present for 7 to 12 weeks resolved spontaneously. The median time to regression was 29 weeks (range 2 to143) and the independent predictive factor of pseudocyst resolution or asymptomatic course was size < 4 cm. In a series of 68 patients treated expectantly. Preoperative sperm analysis is normal. The right side is recurrent after repair 20 years previously. He is interested in starting a family. open approach) is the procedure of choice (B) totally extraperitoneal (TEP) laparoscopic mesh repair eliminates risk of injury to vas deferens and spermatic vessels (C) mesh repairs have been implicated as a cause of obstructive azoöspermia (D) Bassini repairs will minimize the potential for infertility (E) his risk of ischemic orchitis and testicular atrophy is < 1% page 404 . The left-sided hernia extends into the scrotum. He should be advised that (A) bilateral tension-free mesh repair (anterior.American College of Surgeons SESAP 13 Category 4 Item 50 A recently married. otherwise healthy 34-year-old man presents with symptomatic bilateral hernias. and is concerned that operation may affect fertility. Hernia 8:252-254. It is easily mistaken for a hernia on physical examination.American College of Surgeons SESAP 13 Critique for Category 4 Item 50 Despite more than a century of modern approaches to the treatment of groin hernias. A recent article implicating polypropylene mesh in the development of azoöspermia. 2004 2. Heniford BT. Marucci DD. Lipshultz LI. nerve entrapment. hematoma/hemorrhage. the Shouldice repair. Most experienced hernia surgeons recommend their removal. but the risk increases from less than 1% to more than 5% with recurrent and/or large scrotal hernias such as a sliding hernia with sigmoid colon as a component. Garcia-Ruiz A. Surg Endosc 13:585-587. recurrence. Ischemic orchitis is an uncommon result in straightforward primary hernias. which can be done safely without increasing morbidity. Alper A. attempts to resect the distal sac (beyond the pubic tubercle) in large scrotal hernias significantly increase the risk of testicular devascularization. Variables to consider include open vs laparoscopic. 1999 5. Wantz GE: Prevention of ischemic orchitis during inguinal hernioplasty. and may be a single sheet.Carilli S. 2002 7. in Canada and elsewhere. et al: Inguinal canal “lipoma. Ann Surg 235:586-590.Lilly MC. is essential. 2002 6. 2005 3. Arregui ME: Lipomas of the cord and round ligament. a plug. has many faithful adherents. and ischemic orchitis. In the former. Less frequently cited is the potential for infertility. has raised concerns. or a device incorporating mesh attached to a pre-formed flexible ring. Lipomas may be missed at laparoscopic repair. A pitfall of transabdominal preperitoneal laparoscopic hernia repair. Surg Gynecol Obstet 174:399-402. Commonly recognized complications include infection. leading to re-exploration. which may have been compromised by previous dissection. meticulous attention to the spermatic vessels. Ann Surg 241:553-558. Emre A: Inguinal cord lipomas.Heller CA.Fitzgibbons RJ Jr: Can we be sure polypropylene mesh causes infertility? Ann Surg 241:559-561.” Clin Anat 15:280-285. Barr EM. presumably secondary to an intense fibrotic reaction around the vas deferens. Ponsky JL: Missed lipoma of the spermatic cord. “Lipoma” of the spermatic cord is probably a misnomer. Almost all mesh repairs use nonabsorbable materials. 1992 4.Gersin KS.Shin D. incorporating multiple layers of tissue instead of mesh. tension-free mesh vs tissue repair. can cause pain. 2005 8. a consensus has yet to be reached on the optimal technique of repair. Dunn T. Tormey S. 2005 page 405 .Fong Y. and anterior vs preperitoneal vs intraperitoneal approach. REFERENCES 1. Walsh TN: Lipoma of the cord and round ligament: an overlooked diagnosis? Hernia 9:245-247. However. Most surgeons in North America have adopted some type of prosthetic mesh repair as their operation of choice for primary hernias in adults. There is no evidence that a Bassini repair reduces the incidence of fertility. and the subsequent persistent groin mass has been interpreted as a recurrence. The lesion is actually herniation of preperitoneal fat through the lateral aspect of the internal ring. In the latter. Goldstein M. and when large may sufficiently dilate the ring to predispose to visceral herniation. a composite (PTFE and polypropylene). et al: Herniorrhaphy with polypropylene mesh causing inguinal vasal obstruction: a preventable cause of obstructive azoospermia.Nasr AO. Barme GA. He is interested in starting a family. and is concerned that operation may affect fertility. Exploration of the left groin for a left inguinal hernia identifies a sliding hernia with chronically incarcerated sigmoid colon and a large lipoma of the cord.American College of Surgeons SESAP 13 Category 4 Item 51 A recently married. Treatment should be guided by the knowledge that (A) removal of the lipoma increases the risk of testicular devascularization (B) removal of the distal sac increases the risk of testicular devascularization (C) a cord lipoma is an asymptomatic inconsequential finding. The right side is recurrent after repair 20 years previously. and does not need to be removed (D) presence of a sliding hernia mandates mesh repair (E) repair of sliding hernias increases the risk of injury to the vas deferens page 406 . otherwise healthy 34-year-old man presents with symptomatic bilateral hernias. The left-sided hernia extends into the scrotum. Preoperative sperm analysis is normal. 2002 6. Almost all mesh repairs use nonabsorbable materials. Variables to consider include open vs laparoscopic. Ischemic orchitis is an uncommon result in straightforward primary hernias. Emre A: Inguinal cord lipomas. but the risk increases from less than 1% to more than 5% with recurrent and/or large scrotal hernias such as a sliding hernia with sigmoid colon as a component.Shin D. and ischemic orchitis. Most experienced hernia surgeons recommend their removal.Fong Y. Ann Surg 235:586-590. leading to re-exploration. Heniford BT. Alper A. In the latter. Lipshultz LI. The lesion is actually herniation of preperitoneal fat through the lateral aspect of the internal ring. is essential. nerve entrapment.American College of Surgeons SESAP 13 Critique of Category 4 Item 51 Despite more than a century of modern approaches to the treatment of groin hernias. and when large may sufficiently dilate the ring to predispose to visceral herniation. which can be done safely without increasing morbidity. Arregui ME: Lipomas of the cord and round ligament.Nasr AO. Tormey S. or a device incorporating mesh attached to a pre-formed flexible ring. In the former. and may be a single sheet. Dunn T. Commonly recognized complications include infection. Less frequently cited is the potential for infertility. has many faithful adherents.Lilly MC. REFERENCES 1. which may have been compromised by previous dissection. Barr EM. 2005 page 407 . 1992 4.Fitzgibbons RJ Jr: Can we be sure polypropylene mesh causes infertility? Ann Surg 241:559-561. A pitfall of transabdominal preperitoneal laparoscopic hernia repair. Wantz GE: Prevention of ischemic orchitis during inguinal hernioplasty. a plug. Most surgeons in North America have adopted some type of prosthetic mesh repair as their operation of choice for primary hernias in adults. 2005 3. It is easily mistaken for a hernia on physical examination. A recent article implicating polypropylene mesh in the development of azoöspermia. incorporating multiple layers of tissue instead of mesh. 2004 2. Walsh TN: Lipoma of the cord and round ligament: an overlooked diagnosis? Hernia 9:245-247. tension-free mesh vs tissue repair. a composite (PTFE and polypropylene).Heller CA. attempts to resect the distal sac (beyond the pubic tubercle) in large scrotal hernias significantly increase the risk of testicular devascularization. Goldstein M. 2002 7. can cause pain. meticulous attention to the spermatic vessels. Barme GA. a consensus has yet to be reached on the optimal technique of repair. and the subsequent persistent groin mass has been interpreted as a recurrence. Ponsky JL: Missed lipoma of the spermatic cord.” Clin Anat 15:280-285. has raised concerns. Marucci DD.Gersin KS. Garcia-Ruiz A. Surg Gynecol Obstet 174:399-402. Lipomas may be missed at laparoscopic repair. et al: Inguinal canal “lipoma. hematoma/hemorrhage. There is no evidence that a Bassini repair reduces the incidence of fertility. 1999 5. Hernia 8:252-254.Carilli S. the Shouldice repair. However. “Lipoma” of the spermatic cord is probably a misnomer. 2005 8. Surg Endosc 13:585-587. recurrence. Ann Surg 241:553-558. et al: Herniorrhaphy with polypropylene mesh causing inguinal vasal obstruction: a preventable cause of obstructive azoospermia. presumably secondary to an intense fibrotic reaction around the vas deferens. in Canada and elsewhere. and anterior vs preperitoneal vs intraperitoneal approach. American College of Surgeons SESAP 13 Category 4 Item 52 Rarely involves bowel (A) Spigelian hernia (B) Lumbar hernia (C) Trocar hernia (D) Obturator hernia (E) Richter’s hernia page 408 . most commonly involve retroperitoneal tissues. 2002. Obturator hernias occur along the obturator canal. Obturator hernia is the most common pelvic hernia and the type most likely to be seen by a general surgeon.American College of Surgeons SESAP 13 Critique for Category 4 Item 52 Spigelian and lumbar hernias are the most common of the unusual abdominal wall hernias. Most Spigelian hernias occur in the area between the level of umbilicus and the level of the arcuate (semicircular) line. both the hernia sac and the orifice may be difficult to detect on palpation. Pain extending down the medial aspect of the thigh to the knee is one of the most common symptoms. 2003 page 409 . Most postlaparoscopy trocar hernias are Richter’s hernias. This pain pattern is known as the Howship-Romberg sign or obturator neuralgia. between the fascial layers of the abdominal wall. WB Saunders Co. pp 810-813 2. ie. REFERENCES 1.Itani KMF: Uncommon abdominal wall hernias.Montes IS. which are subdivided into hernias of the superior or the inferior lumbar space. These hernias do not have a peritoneal lining and usually contain small or large intestine or omentum. Because the hernia often develops intramurally. but not to the point of involving the entire circumference of the intestine. Lumbar hernias. The presence of bowel in a lumbar hernia would be extremely unusual. The boom in minimally invasive surgery has been associated with increased prevalence of a newly recognized form of abdominal wall hernia. Surg Clin North Am 83:12351253. correct preoperative diagnosis is rare. Flexion of the hip usually exacerbates the pain. Philadelphia. the postlaparoscopy trocar hernia. Deysine M: Spigelian and other uncommon hernia repairs. but the most common location is at the site of femoral hernia. A Richter’s hernia can occur within any type of abdominal wall or pelvic hernia. where the Spigelian fascia is widest and weakest. involving partial protrusion of the antimesenteric intestine into the hernia sac. in Bland KI (ed): The Practice of General Surgery. In the absence of this pain. and occurs in 50% of patients. which is the portion of the obturator foramen through which the obturator nerve and vessels exit the pelvis. American College of Surgeons SESAP 13 Category 4 Item 53 Most common location is at the site of a femoral hernia (A) Spigelian hernia (B) Lumbar hernia (C) Trocar hernia (D) Obturator hernia (E) Richter’s hernia page 410 . which are subdivided into hernias of the superior or the inferior lumbar space. and occurs in 50% of patients. This pain pattern is known as the Howship-Romberg sign or obturator neuralgia. Obturator hernias occur along the obturator canal. most commonly involve retroperitoneal tissues. These hernias do not have a peritoneal lining and usually contain small or large intestine or omentum. the postlaparoscopy trocar hernia. 2002.Itani KMF: Uncommon abdominal wall hernias. but the most common location is at the site of femoral hernia. A Richter’s hernia can occur within any type of abdominal wall or pelvic hernia. Pain extending down the medial aspect of the thigh to the knee is one of the most common symptoms. pp 810-813 2. The boom in minimally invasive surgery has been associated with increased prevalence of a newly recognized form of abdominal wall hernia. Deysine M: Spigelian and other uncommon hernia repairs. WB Saunders Co. Most Spigelian hernias occur in the area between the level of umbilicus and the level of the arcuate (semicircular) line. 2003 page 411 . Most postlaparoscopy trocar hernias are Richter’s hernias. In the absence of this pain. Obturator hernia is the most common pelvic hernia and the type most likely to be seen by a general surgeon. between the fascial layers of the abdominal wall. where the Spigelian fascia is widest and weakest. which is the portion of the obturator foramen through which the obturator nerve and vessels exit the pelvis. but not to the point of involving the entire circumference of the intestine. both the hernia sac and the orifice may be difficult to detect on palpation. Lumbar hernias. Philadelphia. ie.American College of Surgeons SESAP 13 Critique for Category 4 Item 53 Spigelian and lumbar hernias are the most common of the unusual abdominal wall hernias. Surg Clin North Am 83:12351253. The presence of bowel in a lumbar hernia would be extremely unusual. involving partial protrusion of the antimesenteric intestine into the hernia sac. Flexion of the hip usually exacerbates the pain. correct preoperative diagnosis is rare. Because the hernia often develops intramurally.Montes IS. REFERENCES 1. in Bland KI (ed): The Practice of General Surgery. American College of Surgeons SESAP 13 Category 4 Item 54 Often develops at the level of the arcuate (semicircular) line (A) Spigelian hernia (B) Lumbar hernia (C) Trocar hernia (D) Obturator hernia (E) Richter’s hernia page 412 . involving partial protrusion of the antimesenteric intestine into the hernia sac. Deysine M: Spigelian and other uncommon hernia repairs. which are subdivided into hernias of the superior or the inferior lumbar space. 2002. REFERENCES 1. These hernias do not have a peritoneal lining and usually contain small or large intestine or omentum. Philadelphia. Because the hernia often develops intramurally. In the absence of this pain. which is the portion of the obturator foramen through which the obturator nerve and vessels exit the pelvis.Montes IS. pp 810-813 2. Lumbar hernias. and occurs in 50% of patients. 2003 page 413 . but the most common location is at the site of femoral hernia. between the fascial layers of the abdominal wall. Most Spigelian hernias occur in the area between the level of umbilicus and the level of the arcuate (semicircular) line. Most postlaparoscopy trocar hernias are Richter’s hernias. The boom in minimally invasive surgery has been associated with increased prevalence of a newly recognized form of abdominal wall hernia. The presence of bowel in a lumbar hernia would be extremely unusual. most commonly involve retroperitoneal tissues. A Richter’s hernia can occur within any type of abdominal wall or pelvic hernia. both the hernia sac and the orifice may be difficult to detect on palpation. WB Saunders Co. Pain extending down the medial aspect of the thigh to the knee is one of the most common symptoms. but not to the point of involving the entire circumference of the intestine.American College of Surgeons SESAP 13 Critique for Category 4 Item 54 Spigelian and lumbar hernias are the most common of the unusual abdominal wall hernias. the postlaparoscopy trocar hernia. correct preoperative diagnosis is rare.Itani KMF: Uncommon abdominal wall hernias. Obturator hernia is the most common pelvic hernia and the type most likely to be seen by a general surgeon. Flexion of the hip usually exacerbates the pain. where the Spigelian fascia is widest and weakest. Surg Clin North Am 83:12351253. Obturator hernias occur along the obturator canal. This pain pattern is known as the Howship-Romberg sign or obturator neuralgia. ie. in Bland KI (ed): The Practice of General Surgery. American College of Surgeons SESAP 13 Category 4 Item 55 Acute colonic pseudo-obstruction (A) Abdominal colectomy (B) Abdominoperineal resection (C) Restorative ileoanal anastomosis (D) Cecostomy (E) Stricturoplasty page 414 . Wood WC (eds): Oxford Textbook of Surgery. although the most common operation performed for Crohn’s disease is segmental resection of small bowel. Philadelphia. page 415 . and discontinuing analgesics. Cecostomy can sometimes be done under local anesthesia for high-risk patients. in Cameron JL (ed): Current Surgical Therapy. REFERENCES 1. Oxford. If stricturoplasty is required. is still the most common surgical procedure today for toxic megacolon. Recent trials have demonstrated that intravenous neostigmine is effective in 70% to 95% of cases. in Morris PJ. Although some authors advocate abdominal colectomy and ileorectal anastomosis. 2000. This nonmechanical obstructive process is initially managed conservatively by removing oral nutrition. Fischer JE (eds): Mastery of Surgery. pp 1361-1369 4.Cohen AM: Colorectal tumours. and other medications known to cause pseudoobstruction of the colon. Oxford University Press.Cook TA. Most surgeons currently prefer total restorative proctocolectomy with a pelvic pouch-anal anastomosis because it attempts to eliminate the at-risk rectal mucosa while preserving the sphincters. 2004. Oxford.Hull TL. this resective option would be appropriate only for patients who have few if any polyps in the rectum and agree to continuous surveillance of the remaining rectum. Fazio VW: Surgery for toxic megacolon.Becker JM. in Baker RJ. 2000. ed 2. Historically. pp 1471-1510 3. the mortality rate was 63% for ileostomy alone compared with 10% to 15% for abdominal colectomy. small bowel stricturoplasty has been shown to be safe and effective. advocated by Crile and Thomas in 1951. ed 8. Restorative ileoanal anastomosis is performed after an abdominal colectomy and resection of the rectal mucosa for familial adenomatous polyposis (FAP). This ileal-pouch anal reconstruction has had good long-term results and is the procedure of choice for patients with FAP. Stricturoplasty sites should be marked with metal clips for future reference. Stucchi AF: Inherited colorectal polyposis syndromes. The guiding principle in the operative management of Crohn’s disease is preservation of intestinal length. although it is currently also associated with fulminant Clostridium difficile colitis. inserting a nasogastric tube. anticholinergic agents. When indicated. calcium channel blockers. Mortensen N: Surgery for Crohn’s disease of the small intestine.American College of Surgeons SESAP 13 Critique for Category 4 Item 56 Total abdominal colectomy is indicated for a patient with toxic megacolon. Longer strictures (10 to 20 cm) would likely require a Finney-type stricturoplasty. Toxic megacolon initially was considered a complication of inflammatory bowel disease. Careful decompressive colonoscopy is also an initial treatment of choice. Total abdominal colectomy and ileostomy with preservation of the rectum (along with construction of an ileostomy and mucous fistula). Mosby. There are very few indications for cecostomy. and should be attempted prior to performing a cecostomy if possible. ed 2. pp 200-210 2. Wood WC (eds): Oxford Textbook of Surgery. in Morris PJ. it is usually done in patients with Crohn’s disease in an attempt to preserve a strictured segment of bowel. A Heineke-Mikulicz-type strictureplasty is recommended for short strictures (<10 cm). Oxford University Press. Abdominoperineal resection is usually indicated for low-lying rectal malignant neoplasms such as adenocarcinomas. but it remains an option for colonic pseudoobstruction (Ogilvie’s syndrome). American College of Surgeons SESAP 13 Category 4 Item 56 Familial adenomatous polyposis (A) Abdominal colectomy (B) Abdominoperineal resection (C) Restorative ileoanal anastomosis (D) Cecostomy (E) Stricturoplasty page 416 . 2000. it is usually done in patients with Crohn’s disease in an attempt to preserve a strictured segment of bowel. 2000. and should be attempted prior to performing a cecostomy if possible. Oxford University Press. Cecostomy can sometimes be done under local anesthesia for high-risk patients. Philadelphia.Hull TL. calcium channel blockers. page 417 .Cohen AM: Colorectal tumours. Wood WC (eds): Oxford Textbook of Surgery.American College of Surgeons SESAP 13 Critique for Category 4 Item 56 Total abdominal colectomy is indicated for a patient with toxic megacolon. Although some authors advocate abdominal colectomy and ileorectal anastomosis. This nonmechanical obstructive process is initially managed conservatively by removing oral nutrition. in Morris PJ. Abdominoperineal resection is usually indicated for low-lying rectal malignant neoplasms such as adenocarcinomas. Stricturoplasty sites should be marked with metal clips for future reference. Fischer JE (eds): Mastery of Surgery. advocated by Crile and Thomas in 1951. Oxford. although it is currently also associated with fulminant Clostridium difficile colitis. Restorative ileoanal anastomosis is performed after an abdominal colectomy and resection of the rectal mucosa for familial adenomatous polyposis (FAP). Recent trials have demonstrated that intravenous neostigmine is effective in 70% to 95% of cases.Becker JM. Historically. When indicated. is still the most common surgical procedure today for toxic megacolon. small bowel stricturoplasty has been shown to be safe and effective. Toxic megacolon initially was considered a complication of inflammatory bowel disease. pp 1471-1510 3. pp 1361-1369 4. Fazio VW: Surgery for toxic megacolon. Mortensen N: Surgery for Crohn’s disease of the small intestine. Total abdominal colectomy and ileostomy with preservation of the rectum (along with construction of an ileostomy and mucous fistula). and other medications known to cause pseudoobstruction of the colon. Oxford University Press. and discontinuing analgesics. ed 2. although the most common operation performed for Crohn’s disease is segmental resection of small bowel. 2004. ed 8. If stricturoplasty is required. Careful decompressive colonoscopy is also an initial treatment of choice. REFERENCES 1. pp 200-210 2.Cook TA. Longer strictures (10 to 20 cm) would likely require a Finney-type stricturoplasty. The guiding principle in the operative management of Crohn’s disease is preservation of intestinal length. Wood WC (eds): Oxford Textbook of Surgery. ed 2. anticholinergic agents. this resective option would be appropriate only for patients who have few if any polyps in the rectum and agree to continuous surveillance of the remaining rectum. Most surgeons currently prefer total restorative proctocolectomy with a pelvic pouch-anal anastomosis because it attempts to eliminate the at-risk rectal mucosa while preserving the sphincters. in Baker RJ. Mosby. in Cameron JL (ed): Current Surgical Therapy. in Morris PJ. Stucchi AF: Inherited colorectal polyposis syndromes. but it remains an option for colonic pseudoobstruction (Ogilvie’s syndrome). the mortality rate was 63% for ileostomy alone compared with 10% to 15% for abdominal colectomy. Oxford. This ileal-pouch anal reconstruction has had good long-term results and is the procedure of choice for patients with FAP. A Heineke-Mikulicz-type strictureplasty is recommended for short strictures (<10 cm). inserting a nasogastric tube. There are very few indications for cecostomy. American College of Surgeons SESAP 13 ed 4. in Cameron JL (ed): Current Surgical Therapy. 2001. Mosby. ed 8. pp 1497-1506 5. Lippincott Williams & Wilkins. Philadelphia. DeBarros J: Ogilvie’s syndrome (colonic pseudo-obstruction). 2004. pp 176-178 page 418 .Vernava AM III. Philadelphia. American College of Surgeons SESAP 13 Category 4 Item 57 Toxic megacolon (A) Abdominal colectomy (B) Abdominoperineal resection (C) Restorative ileoanal anastomosis (D) Cecostomy (E) Stricturoplasty page 419 . There are very few indications for cecostomy. page 420 .Cook TA. 2000. ed 2. Stricturoplasty sites should be marked with metal clips for future reference. small bowel stricturoplasty has been shown to be safe and effective. The guiding principle in the operative management of Crohn’s disease is preservation of intestinal length. and discontinuing analgesics.Becker JM. Although some authors advocate abdominal colectomy and ileorectal anastomosis. If stricturoplasty is required. in Baker RJ. Wood WC (eds): Oxford Textbook of Surgery. Philadelphia. Careful decompressive colonoscopy is also an initial treatment of choice. When indicated.Hull TL. This ileal-pouch anal reconstruction has had good long-term results and is the procedure of choice for patients with FAP. inserting a nasogastric tube. calcium channel blockers. Oxford. this resective option would be appropriate only for patients who have few if any polyps in the rectum and agree to continuous surveillance of the remaining rectum. Historically. but it remains an option for colonic pseudoobstruction (Ogilvie’s syndrome). Restorative ileoanal anastomosis is performed after an abdominal colectomy and resection of the rectal mucosa for familial adenomatous polyposis (FAP). in Morris PJ. Stucchi AF: Inherited colorectal polyposis syndromes. Abdominoperineal resection is usually indicated for low-lying rectal malignant neoplasms such as adenocarcinomas. ed 2. A Heineke-Mikulicz-type strictureplasty is recommended for short strictures (<10 cm). Wood WC (eds): Oxford Textbook of Surgery. in Cameron JL (ed): Current Surgical Therapy. This nonmechanical obstructive process is initially managed conservatively by removing oral nutrition. pp 1361-1369 4. and should be attempted prior to performing a cecostomy if possible.Cohen AM: Colorectal tumours. Total abdominal colectomy and ileostomy with preservation of the rectum (along with construction of an ileostomy and mucous fistula). Oxford University Press. Oxford University Press. Mosby. anticholinergic agents. Fazio VW: Surgery for toxic megacolon. 2000. Cecostomy can sometimes be done under local anesthesia for high-risk patients. REFERENCES 1. Oxford. although it is currently also associated with fulminant Clostridium difficile colitis. ed 8. although the most common operation performed for Crohn’s disease is segmental resection of small bowel. and other medications known to cause pseudoobstruction of the colon. it is usually done in patients with Crohn’s disease in an attempt to preserve a strictured segment of bowel. is still the most common surgical procedure today for toxic megacolon. pp 1471-1510 3. Longer strictures (10 to 20 cm) would likely require a Finney-type stricturoplasty. Most surgeons currently prefer total restorative proctocolectomy with a pelvic pouch-anal anastomosis because it attempts to eliminate the at-risk rectal mucosa while preserving the sphincters. 2004. Fischer JE (eds): Mastery of Surgery. Mortensen N: Surgery for Crohn’s disease of the small intestine.American College of Surgeons SESAP 13 Critique for Category 4 Item 57 Total abdominal colectomy is indicated for a patient with toxic megacolon. the mortality rate was 63% for ileostomy alone compared with 10% to 15% for abdominal colectomy. pp 200-210 2. in Morris PJ. Toxic megacolon initially was considered a complication of inflammatory bowel disease. advocated by Crile and Thomas in 1951. Recent trials have demonstrated that intravenous neostigmine is effective in 70% to 95% of cases. 2004. in Cameron JL (ed): Current Surgical Therapy. Philadelphia. pp 176-178 page 421 . 2001.Vernava AM III. pp 1497-1506 5. DeBarros J: Ogilvie’s syndrome (colonic pseudo-obstruction). Mosby. Lippincott Williams & Wilkins. Philadelphia. ed 8.American College of Surgeons SESAP 13 ed 4. thrombocytopenia (A) Immune (idiopathic) thrombocytopenic purpura (ITP) (B) Thrombotic thrombocytopenic purpura (TTP) (C) Felty’s syndrome (D) β-thalassemia (E) Spherocytosis page 422 .American College of Surgeons SESAP 13 Category 4 Item 58 Normal platelet production. Plasmapheresis is the treatment of choice. pregnancy. Best Pract Res Clin Rheumatol 18:631-645.American College of Surgeons SESAP 13 Critique for Category 4 Item 58 Immune (idiopathic) thrombocytopenic purpura (ITP) is a diagnosis of exclusion. Schlinkert RT: Thrombocytopenia and other hematologic disorders. cutaneous leg ulcers. Sarr MG. Splenectomy is reserved for patients who do not respond to medical therapy or have recurrent disease.Bisharat N. Philadelphia. The syndrome is not well understood. and is quite similar clinically to hemolytic uremic syndrome. Splenectomy should be delayed until the age of 10 if possible. and rheumatoid arthritis. ITP is usually self-limited in children. a structural RBC membrane protein. Balint PV: Felty’s syndrome. 2004. Omari H. which eventually leads to hypersplenism and anemia. These patients have an elevated risk of postsplenectomy sepsis. REFERENCES 1. 2001 3. and systemic lupus erythematosis all produce similar syndromes. but patients will sometimes benefit from splenectomy if medical therapy fails. Thrombotic thrombocytopenic purpura (TTP) classically presents with thrombocytopenia. and accumulation of intracellular material leads to structurally abnormal red blood cells. Spherocytosis is the result of genetically defective spectrin. Felty’s syndrome is the clinical triad of thrombocytopenia. Saunders. and central nervous system dysfunction. 2004 2. and ITP can be assigned as a diagnosis only after other causes have been ruled out. β-thalassemia is a hereditary blood disorder characterized by a defective β-globulin chain. αglobulin production is normal. HIV. Hinder RA (eds): Mayo Clinic Gastrointestinal Surgery. a hemolytic anemia. Lavi I. in Kelly KA. drugs.Swain JM. Splenectomy is reserved for severe complications or high blood transfusion requirements. Platelets are normally produced in ITP. The abnormal cells exhibit a high rate of sequestration and destruction by the spleen. J Infect 43:182-186. fever.Balint GP. pp 365-374 page 423 . Raz R: Risk of infection and death among post-splenectomy patients. renal disease. and adults may respond to glucocorticoid or gamma globulin therapy. hemolytic anemia. renal disease.American College of Surgeons SESAP 13 Category 4 Item 59 Fever. thrombocytopenia. central nervous system dysfunction (A) Immune (idiopathic) thrombocytopenic purpura (ITP) (B) Thrombotic thrombocytopenic purpura (TTP) (C) Felty’s syndrome (D) β-thalassemia (E) Spherocytosis page 424 . The abnormal cells exhibit a high rate of sequestration and destruction by the spleen. J Infect 43:182-186. β-thalassemia is a hereditary blood disorder characterized by a defective β-globulin chain. Platelets are normally produced in ITP. HIV. and systemic lupus erythematosis all produce similar syndromes. 2004 2.Swain JM. Omari H. Felty’s syndrome is the clinical triad of thrombocytopenia. fever. Schlinkert RT: Thrombocytopenia and other hematologic disorders. REFERENCES 1. a structural RBC membrane protein.Bisharat N. 2004. but patients will sometimes benefit from splenectomy if medical therapy fails. Raz R: Risk of infection and death among post-splenectomy patients. Spherocytosis is the result of genetically defective spectrin. pp 365-374 page 425 . and accumulation of intracellular material leads to structurally abnormal red blood cells. drugs. cutaneous leg ulcers. Sarr MG. Plasmapheresis is the treatment of choice. and rheumatoid arthritis. Saunders. Splenectomy is reserved for patients who do not respond to medical therapy or have recurrent disease. and ITP can be assigned as a diagnosis only after other causes have been ruled out. a hemolytic anemia. 2001 3. and central nervous system dysfunction. which eventually leads to hypersplenism and anemia. Splenectomy is reserved for severe complications or high blood transfusion requirements. These patients have an elevated risk of postsplenectomy sepsis. αglobulin production is normal. and is quite similar clinically to hemolytic uremic syndrome. pregnancy. Splenectomy should be delayed until the age of 10 if possible. renal disease. Thrombotic thrombocytopenic purpura (TTP) classically presents with thrombocytopenia. Best Pract Res Clin Rheumatol 18:631-645. The syndrome is not well understood.American College of Surgeons SESAP 13 Critique for Category 4 Item 59 Immune (idiopathic) thrombocytopenic purpura (ITP) is a diagnosis of exclusion. Balint PV: Felty’s syndrome. Lavi I. Philadelphia.Balint GP. and adults may respond to glucocorticoid or gamma globulin therapy. Hinder RA (eds): Mayo Clinic Gastrointestinal Surgery. ITP is usually self-limited in children. in Kelly KA. defective spectrin. structurally abnormal RBCs (A) Immune (idiopathic) thrombocytopenic purpura (ITP) (B) Thrombotic thrombocytopenic purpura (TTP) (C) Felty’s syndrome (D) β-thalassemia (E) Spherocytosis page 426 .American College of Surgeons SESAP 13 Category 4 Item 60 Thrombocytopenia. normal α-globulin. Thrombotic thrombocytopenic purpura (TTP) classically presents with thrombocytopenia. Spherocytosis is the result of genetically defective spectrin. drugs.American College of Surgeons SESAP 13 Critique for Category 4 Item 60 Immune (idiopathic) thrombocytopenic purpura (ITP) is a diagnosis of exclusion. Splenectomy is reserved for patients who do not respond to medical therapy or have recurrent disease. 2004. fever.Balint GP. Schlinkert RT: Thrombocytopenia and other hematologic disorders. a structural RBC membrane protein. which eventually leads to hypersplenism and anemia. Splenectomy should be delayed until the age of 10 if possible. Omari H. These patients have an elevated risk of postsplenectomy sepsis. Best Pract Res Clin Rheumatol 18:631-645. β-thalassemia is a hereditary blood disorder characterized by a defective β-globulin chain. a hemolytic anemia. J Infect 43:182-186. Plasmapheresis is the treatment of choice. and accumulation of intracellular material leads to structurally abnormal red blood cells. Raz R: Risk of infection and death among post-splenectomy patients.Bisharat N. 2004 2. αglobulin production is normal. and rheumatoid arthritis. pp 365-374 page 427 . Sarr MG. HIV. Philadelphia. Balint PV: Felty’s syndrome. REFERENCES 1. in Kelly KA. and systemic lupus erythematosis all produce similar syndromes. and central nervous system dysfunction. Hinder RA (eds): Mayo Clinic Gastrointestinal Surgery. cutaneous leg ulcers. and ITP can be assigned as a diagnosis only after other causes have been ruled out. The abnormal cells exhibit a high rate of sequestration and destruction by the spleen. and is quite similar clinically to hemolytic uremic syndrome. Saunders. Lavi I. but patients will sometimes benefit from splenectomy if medical therapy fails. 2001 3. and adults may respond to glucocorticoid or gamma globulin therapy. ITP is usually self-limited in children. Felty’s syndrome is the clinical triad of thrombocytopenia. renal disease. Splenectomy is reserved for severe complications or high blood transfusion requirements. pregnancy. Platelets are normally produced in ITP. The syndrome is not well understood.Swain JM. American College of Surgeons SESAP 13 Category 4 Item 61 Focal nodular hyperplasia (A) Figure A (B) Figure B (C) Figure C (D) Figure D (E) Figure E page 428 . No imaging modality page 429 . seen either on CT or MRI. Gadolinium enhances vascular structures and is therefore most helpful in highlighting hemangiomas and metastases. although it may be too sensitive and is often unnecessary. The adenoma should be hyperintense on T2-weighted magnetic resonance images. homogeneous. or mildly hyperintense on CT. A similar appearance is seen on MRI with gadolinium contrast. superparamagnetic iron oxide decreases the intensity of the liver signal and highlights even small lesions. The lack of uptake of radionuclide tracer due to lack of Kupffer cell activity differentiates hepatic adenoma from focal nodular hyperplasia. the gallbladder and extrahepatic biliary system should be imaged with whatever modality is chosen. enhance brightly with contrast. and hyperechoic without posterior shadowing. Ultrasonography is the least expensive and most versatile. CT will reveal marked early enhancement with washout on delayed images. isointense. particularly with the advances in CT. The lesions are initially of low attenuation. Radionuclide imaging is primarily an adjunctive modality. pertechnetate. On ultrasonography hemangiomas are usually well circumscribed. as the hemangioma fills in completely. Intraoperative use can aid in identification of lesions and guide resection margins. Early enhancement on CT due to increased vascularity is a feature of hepatic adenoma (image C). Hemangiomas (image B) are characterized by peripheral enhancement with centripetal filling. Differentiation between primary and metastatic disease is most often based on history. computed tomography (CT). Technetium 99 sulfur colloid. Hepatocellular carcinoma has classic radiologic features in multiphase CT or MRI with gadolinium. it is not sufficiently sensitive to detect lesions < 2 cm. CT must be done with intravenous contrast. it is primarily helpful in differentiating a solid from a cystic lesion. Metastatic lesions (image E) appear as hypointense. and become hypodense on delayed images.American College of Surgeons SESAP 13 Critique for Category 4 Item 61 Ultrasonography. MRI with superparamagnetic iron oxide or technetium 99m-labeled sulfur colloid will demonstrate increased uptake due to the Kupffer cell activity. as well as determining the relationship of lesions to vascular structures. with the characteristic central scar and surrounding fibrous tissue present in about 60% of cases. Gadolinium or superparamagnetic iron oxide contrast is used. MRI is usually unnecessary unless multiple small abscesses are indistinguishable from surrounding fatty liver. a more homogeneous picture is easily confused with focal nodular hyperplasia. If the early phase of the image is missed the lesion itself may be missed. with some also hyperintensity on T1 images. Because the biliary tree is often the source of pyogenic abscesses. Air within a lesion in the absence of prior biliary tract operation is diagnostic of abscess. If there is bleeding within the adenoma (10% to 15%). this enhancement may be heterogeneous. Focal nodular hyperplasia (image A) is characterized by increased Kupffer cell activity. and gallium 67 are most helpful in making the distinction between focal nodular hyperplasia and adenoma. and multiple phase contrastenhanced multislice CT provides the greatest sensitivity and specificity. The different appearances depend on the vascularity of the tumor. particularly if small. although this is rarely needed. MRI is the most accurate modality for visualizing and differentiating liver pathology. The use of a gadolinium flash sequence on T2-weighted images improves the ability to visualize the lesion. colorectal metastases are relatively avascular and will have a different appearance than neuroendocrine metastases. CT characteristics include an enhancing rim. necessitating the use of intravenous contrast. and magnetic resonance imaging (MRI) are the primary modalities used for imaging the liver. Ultrasonographic characteristics of abscess (image D) include a heterogeneous center that may be multiloculated. Lillemoe KD.Molmenti EP. 2006. ed 4. Lippincott Williams & Wilkins. et al (eds): Greenfield's Surgery: Scientific Principles and Practice. ed 4. Philadelphia. in Mulholland MW. Adjunctive serology along with an appropriate clinical history can aid in this differentiation.American College of Surgeons SESAP 13 can differentiate between pyogenic and amebic abscesses. Maier RV.Delman KA. pp 909-924 page 430 . Curley SA: Hepatic neoplasms. Maier RV. pp 956-977 2. et al (eds): Greenfield's Surgery: Scientific Principles and Practice. Doherty GM. Doherty GM. REFERENCES 1. Lillemoe KD. Philadelphia. Klein AS: Hepatic infection and acute hepatic failure. 2006. Lippincott Williams & Wilkins. in Mulholland MW. American College of Surgeons SESAP 13 Category 4 Item 62 Metastasis (A) Figure A (B) Figure B (C) Figure C (D) Figure D (E) Figure E page 431 . Because the biliary tree is often the source of pyogenic abscesses. necessitating the use of intravenous contrast. and magnetic resonance imaging (MRI) are the primary modalities used for imaging the liver. this enhancement may be heterogeneous. Differentiation between primary and metastatic disease is most often based on history. Gadolinium enhances vascular structures and is therefore most helpful in highlighting hemangiomas and metastases. it is not sufficiently sensitive to detect lesions < 2 cm. MRI with superparamagnetic iron oxide or technetium 99m-labeled sulfur colloid will demonstrate increased uptake due to the Kupffer cell activity. The use of a gadolinium flash sequence on T2-weighted images improves the ability to visualize the lesion. although this is rarely needed. MRI is the most accurate modality for visualizing and differentiating liver pathology. No imaging modality page 432 . and become hypodense on delayed images. MRI is usually unnecessary unless multiple small abscesses are indistinguishable from surrounding fatty liver. The lack of uptake of radionuclide tracer due to lack of Kupffer cell activity differentiates hepatic adenoma from focal nodular hyperplasia. Metastatic lesions (image E) appear as hypointense. Gadolinium or superparamagnetic iron oxide contrast is used. If the early phase of the image is missed the lesion itself may be missed. Air within a lesion in the absence of prior biliary tract operation is diagnostic of abscess. and multiple phase contrastenhanced multislice CT provides the greatest sensitivity and specificity. The different appearances depend on the vascularity of the tumor. although it may be too sensitive and is often unnecessary. and gallium 67 are most helpful in making the distinction between focal nodular hyperplasia and adenoma. as the hemangioma fills in completely. it is primarily helpful in differentiating a solid from a cystic lesion. as well as determining the relationship of lesions to vascular structures. and hyperechoic without posterior shadowing. Technetium 99 sulfur colloid. A similar appearance is seen on MRI with gadolinium contrast. with the characteristic central scar and surrounding fibrous tissue present in about 60% of cases. enhance brightly with contrast. homogeneous.American College of Surgeons SESAP 13 Critique for Category 4 Item 62 Ultrasonography. with some also hyperintensity on T1 images. Early enhancement on CT due to increased vascularity is a feature of hepatic adenoma (image C). If there is bleeding within the adenoma (10% to 15%). a more homogeneous picture is easily confused with focal nodular hyperplasia. or mildly hyperintense on CT. particularly if small. CT must be done with intravenous contrast. Radionuclide imaging is primarily an adjunctive modality. The lesions are initially of low attenuation. Hepatocellular carcinoma has classic radiologic features in multiphase CT or MRI with gadolinium. colorectal metastases are relatively avascular and will have a different appearance than neuroendocrine metastases. CT characteristics include an enhancing rim. Ultrasonography is the least expensive and most versatile. seen either on CT or MRI. isointense. pertechnetate. Focal nodular hyperplasia (image A) is characterized by increased Kupffer cell activity. Hemangiomas (image B) are characterized by peripheral enhancement with centripetal filling. The adenoma should be hyperintense on T2-weighted magnetic resonance images. Intraoperative use can aid in identification of lesions and guide resection margins. CT will reveal marked early enhancement with washout on delayed images. particularly with the advances in CT. the gallbladder and extrahepatic biliary system should be imaged with whatever modality is chosen. Ultrasonographic characteristics of abscess (image D) include a heterogeneous center that may be multiloculated. superparamagnetic iron oxide decreases the intensity of the liver signal and highlights even small lesions. computed tomography (CT). On ultrasonography hemangiomas are usually well circumscribed. ed 4. 2006. Philadelphia. Doherty GM. Klein AS: Hepatic infection and acute hepatic failure. Adjunctive serology along with an appropriate clinical history can aid in this differentiation. Lillemoe KD. REFERENCES 1. in Mulholland MW.American College of Surgeons SESAP 13 can differentiate between pyogenic and amebic abscesses. Lippincott Williams & Wilkins.Delman KA. Maier RV. et al (eds): Greenfield's Surgery: Scientific Principles and Practice. Curley SA: Hepatic neoplasms. Philadelphia. Lippincott Williams & Wilkins. 2006. pp 909-924 page 433 . ed 4. Maier RV. Doherty GM. in Mulholland MW.Molmenti EP. et al (eds): Greenfield's Surgery: Scientific Principles and Practice. pp 956-977 2. Lillemoe KD. American College of Surgeons SESAP 13 Category 4 Item 63 Adenoma (A) Figure A (B) Figure B (C) Figure C (D) Figure D (E) Figure E page 434 . A similar appearance is seen on MRI with gadolinium contrast. it is primarily helpful in differentiating a solid from a cystic lesion. The different appearances depend on the vascularity of the tumor. The lesions are initially of low attenuation. Technetium 99 sulfur colloid. Ultrasonography is the least expensive and most versatile. The use of a gadolinium flash sequence on T2-weighted images improves the ability to visualize the lesion. it is not sufficiently sensitive to detect lesions < 2 cm. computed tomography (CT). and gallium 67 are most helpful in making the distinction between focal nodular hyperplasia and adenoma. as the hemangioma fills in completely. as well as determining the relationship of lesions to vascular structures. superparamagnetic iron oxide decreases the intensity of the liver signal and highlights even small lesions. No imaging modality page 435 . a more homogeneous picture is easily confused with focal nodular hyperplasia. Early enhancement on CT due to increased vascularity is a feature of hepatic adenoma (image C). Air within a lesion in the absence of prior biliary tract operation is diagnostic of abscess. The adenoma should be hyperintense on T2-weighted magnetic resonance images. this enhancement may be heterogeneous. pertechnetate. The lack of uptake of radionuclide tracer due to lack of Kupffer cell activity differentiates hepatic adenoma from focal nodular hyperplasia. Radionuclide imaging is primarily an adjunctive modality. Gadolinium or superparamagnetic iron oxide contrast is used.American College of Surgeons SESAP 13 Critique for Category 4 Item 63 Ultrasonography. necessitating the use of intravenous contrast. colorectal metastases are relatively avascular and will have a different appearance than neuroendocrine metastases. If the early phase of the image is missed the lesion itself may be missed. Gadolinium enhances vascular structures and is therefore most helpful in highlighting hemangiomas and metastases. Differentiation between primary and metastatic disease is most often based on history. seen either on CT or MRI. CT must be done with intravenous contrast. isointense. CT characteristics include an enhancing rim. MRI with superparamagnetic iron oxide or technetium 99m-labeled sulfur colloid will demonstrate increased uptake due to the Kupffer cell activity. the gallbladder and extrahepatic biliary system should be imaged with whatever modality is chosen. and multiple phase contrastenhanced multislice CT provides the greatest sensitivity and specificity. enhance brightly with contrast. particularly if small. MRI is the most accurate modality for visualizing and differentiating liver pathology. Intraoperative use can aid in identification of lesions and guide resection margins. Hepatocellular carcinoma has classic radiologic features in multiphase CT or MRI with gadolinium. and magnetic resonance imaging (MRI) are the primary modalities used for imaging the liver. particularly with the advances in CT. with the characteristic central scar and surrounding fibrous tissue present in about 60% of cases. homogeneous. although it may be too sensitive and is often unnecessary. although this is rarely needed. and hyperechoic without posterior shadowing. Hemangiomas (image B) are characterized by peripheral enhancement with centripetal filling. If there is bleeding within the adenoma (10% to 15%). Ultrasonographic characteristics of abscess (image D) include a heterogeneous center that may be multiloculated. with some also hyperintensity on T1 images. MRI is usually unnecessary unless multiple small abscesses are indistinguishable from surrounding fatty liver. Focal nodular hyperplasia (image A) is characterized by increased Kupffer cell activity. On ultrasonography hemangiomas are usually well circumscribed. Metastatic lesions (image E) appear as hypointense. Because the biliary tree is often the source of pyogenic abscesses. CT will reveal marked early enhancement with washout on delayed images. or mildly hyperintense on CT. and become hypodense on delayed images. ed 4.Molmenti EP. Lillemoe KD. Adjunctive serology along with an appropriate clinical history can aid in this differentiation. Lillemoe KD. in Mulholland MW.Delman KA. Philadelphia. Lippincott Williams & Wilkins. Maier RV. 2006. REFERENCES 1. Philadelphia. Klein AS: Hepatic infection and acute hepatic failure.American College of Surgeons SESAP 13 can differentiate between pyogenic and amebic abscesses. et al (eds): Greenfield's Surgery: Scientific Principles and Practice. Lippincott Williams & Wilkins. et al (eds): Greenfield's Surgery: Scientific Principles and Practice. pp 909-924 page 436 . ed 4. pp 956-977 2. in Mulholland MW. Doherty GM. Curley SA: Hepatic neoplasms. Doherty GM. Maier RV. 2006. American College of Surgeons SESAP 13 Category 4 Item 64 Polymicrobial infection (A) Primary peritonitis (B) Secondary peritonitis (C) Both (D) Neither page 437 . et al (eds): ACS Surgery: Principles and Practice 2006. et al (eds): Schwartz's Principles of Surgery. Saunders. Mosby. which requires operation. et al: Intra-abdominal infection. and a cell count with differential can be helpful in some cases. mesentery. A frequent cause of secondary bacterial peritonitis is acute perforated appendicitis. in Souba WW. ed 17. Inflammatory adhesions often wall off collections of purulent material allowing inter-loop abscess formation. the ascitic fluid grows mixed aerobes and anaerobes. particularly gram-negative enteric organisms. lactate. Andersen DK. Smith R. and less commonly congestive heart failure. pp 1592-1620 4. pp 1171-1198 page 438 . and retroperitoneum. Dunn DL. Abscesses may occur anywhere in the peritoneal cavity and extend into the retroperitoneum. measurements of glucose. surgically treatable process. McDonald JC: Abdominal wall. Although SBP is usually associated with cirrhosis. Chong T. Secondary peritonitis describes an infection of the peritoneal cavity secondary to perforation of a hollow viscus or transmural necrosis of the gastrointestinal tract. protein. Broad-spectrum antimicrobiotics are initiated for empiric therapy. fungal. Fink MP. or imaging studies suggest bowel perforation. mesenteries. pH < 7. or tertiary. secondary. ed 8. Jurkovich GJ. but more culture-directed therapy is preferred. Primary peritonitis is an infection of the peritoneal cavity without perforation of the gastrointestinal tract that results from a bacterial. defined as a bacterial infection of ascitic fluid in the absence of an intra-abdominal. chlamydial. Critical Care. or myobacterial infection. ed 8. Because SBP has a uniorganism etiology. Paracentesis is required to make the diagnosis of SBP.Turnage RH. Surgical intervention for spontaneous peritonitis is indicated when medical management is unsuccessful. it can occur with the nephrotic syndrome. in Cameron JL (ed): Current Surgical Therapy. 2005. Tertiary peritonitis represents an infection of the peritoneal cavity resulting from failure to definitively manage secondary peritonitis. Philadelphia. Kaiser LR. REFERENCES 1. multiple bacterial isolates. Billiar TR. although other measurements such as pH determinations. omentum. McGraw-Hill.Christou NV: Intraabdominal infections. and retroperitoneum.Bell RL. 2004. and lactate > 32 ng/mL are very suggestive of primary peritonitis. chap 8. Seymour NE: Abdominal wall. peritoneum. Barkun JS. omentum. Philadelphia. p 1118-1121 3.American College of Surgeons SESAP 13 Critique for Category 4 Item 64 Peritonitis can be described as primary. sec 18. 2004. in Townsend CM Jr (ed): Sabiston Textbook of Surgery: The Biological Basis of Modern Surgical Practice. Spontaneous bacterial peritonitis (SBP) is a prototypical example of primary peritonitis. New York. Neutrophil count of > 250/mm³. Li BDL. in Brunicardi FC. New York.35. WebMD. along with a poor response to empiric antimicrobial therapy is diagnostic of secondary peritonitis. umbilicus. fluid is primarily sent for gram stain and culture.Sawyer RG. pp 1317-1328 2. American College of Surgeons SESAP 13 Category 4 Item 65 Operation the treatment of choice (A) Primary peritonitis (B) Secondary peritonitis (C) Both (D) Neither page 439 . in Cameron JL (ed): Current Surgical Therapy. ed 8. in Townsend CM Jr (ed): Sabiston Textbook of Surgery: The Biological Basis of Modern Surgical Practice. lactate. 2005. et al (eds): ACS Surgery: Principles and Practice 2006. Inflammatory adhesions often wall off collections of purulent material allowing inter-loop abscess formation. mesentery. Secondary peritonitis describes an infection of the peritoneal cavity secondary to perforation of a hollow viscus or transmural necrosis of the gastrointestinal tract. fluid is primarily sent for gram stain and culture. 2004.Turnage RH.Sawyer RG. Li BDL. ed 17. Andersen DK.American College of Surgeons SESAP 13 Critique for Category 4 Item 65 Peritonitis can be described as primary.Christou NV: Intraabdominal infections. p 1118-1121 3. et al: Intra-abdominal infection. defined as a bacterial infection of ascitic fluid in the absence of an intra-abdominal. ed 8. Billiar TR. Mosby. it can occur with the nephrotic syndrome. Saunders. Philadelphia. 2004. sec 18. et al (eds): Schwartz's Principles of Surgery. McDonald JC: Abdominal wall. WebMD. or tertiary. Although SBP is usually associated with cirrhosis. Neutrophil count of > 250/mm³. chap 8. Dunn DL. pH < 7. REFERENCES 1. in Souba WW. pp 1171-1198 page 440 . pp 1592-1620 4. New York. Abscesses may occur anywhere in the peritoneal cavity and extend into the retroperitoneum. Kaiser LR. surgically treatable process.35. Because SBP has a uniorganism etiology. Chong T. Broad-spectrum antimicrobiotics are initiated for empiric therapy. Barkun JS. Seymour NE: Abdominal wall. but more culture-directed therapy is preferred. and a cell count with differential can be helpful in some cases. and less commonly congestive heart failure. omentum. Spontaneous bacterial peritonitis (SBP) is a prototypical example of primary peritonitis. and retroperitoneum. chlamydial. multiple bacterial isolates. Tertiary peritonitis represents an infection of the peritoneal cavity resulting from failure to definitively manage secondary peritonitis. which requires operation. mesenteries.Bell RL. along with a poor response to empiric antimicrobial therapy is diagnostic of secondary peritonitis. in Brunicardi FC. protein. Critical Care. and lactate > 32 ng/mL are very suggestive of primary peritonitis. Jurkovich GJ. measurements of glucose. peritoneum. pp 1317-1328 2. McGraw-Hill. secondary. although other measurements such as pH determinations. the ascitic fluid grows mixed aerobes and anaerobes. or imaging studies suggest bowel perforation. Primary peritonitis is an infection of the peritoneal cavity without perforation of the gastrointestinal tract that results from a bacterial. Paracentesis is required to make the diagnosis of SBP. A frequent cause of secondary bacterial peritonitis is acute perforated appendicitis. Smith R. New York. or myobacterial infection. fungal. umbilicus. particularly gram-negative enteric organisms. and retroperitoneum. Surgical intervention for spontaneous peritonitis is indicated when medical management is unsuccessful. Fink MP. Philadelphia. omentum. American College of Surgeons SESAP 13 Category 4 Item 66 Paracentesis required for diagnosis (A) Primary peritonitis (B) Secondary peritonitis (C) Both (D) Neither page 441 . Barkun JS. pH < 7. in Townsend CM Jr (ed): Sabiston Textbook of Surgery: The Biological Basis of Modern Surgical Practice. Smith R. lactate. Secondary peritonitis describes an infection of the peritoneal cavity secondary to perforation of a hollow viscus or transmural necrosis of the gastrointestinal tract. fluid is primarily sent for gram stain and culture. peritoneum. WebMD. Spontaneous bacterial peritonitis (SBP) is a prototypical example of primary peritonitis.Sawyer RG. McDonald JC: Abdominal wall. Critical Care. umbilicus. Tertiary peritonitis represents an infection of the peritoneal cavity resulting from failure to definitively manage secondary peritonitis. Li BDL. but more culture-directed therapy is preferred. particularly gram-negative enteric organisms. protein. Jurkovich GJ. chlamydial. which requires operation. the ascitic fluid grows mixed aerobes and anaerobes. multiple bacterial isolates. Billiar TR. p 1118-1121 3. in Souba WW.American College of Surgeons SESAP 13 Critique for Category 4 Item 66 Peritonitis can be described as primary. Broad-spectrum antimicrobiotics are initiated for empiric therapy. or tertiary. although other measurements such as pH determinations. and a cell count with differential can be helpful in some cases. et al (eds): Schwartz's Principles of Surgery. omentum. Surgical intervention for spontaneous peritonitis is indicated when medical management is unsuccessful. pp 1171-1198 page 442 . Abscesses may occur anywhere in the peritoneal cavity and extend into the retroperitoneum. it can occur with the nephrotic syndrome. Dunn DL. Saunders. Although SBP is usually associated with cirrhosis. mesentery. Fink MP.Turnage RH. sec 18. pp 1592-1620 4. Because SBP has a uniorganism etiology. and retroperitoneum. measurements of glucose. et al: Intra-abdominal infection. New York. A frequent cause of secondary bacterial peritonitis is acute perforated appendicitis.35. omentum. 2004. Neutrophil count of > 250/mm³. chap 8. Philadelphia. Chong T. or imaging studies suggest bowel perforation. secondary.Bell RL. 2005. mesenteries. ed 17. surgically treatable process. ed 8. and retroperitoneum. or myobacterial infection. defined as a bacterial infection of ascitic fluid in the absence of an intra-abdominal. fungal. and lactate > 32 ng/mL are very suggestive of primary peritonitis. Andersen DK. in Cameron JL (ed): Current Surgical Therapy. Inflammatory adhesions often wall off collections of purulent material allowing inter-loop abscess formation. 2004. and less commonly congestive heart failure. pp 1317-1328 2. Primary peritonitis is an infection of the peritoneal cavity without perforation of the gastrointestinal tract that results from a bacterial.Christou NV: Intraabdominal infections. Kaiser LR. in Brunicardi FC. Paracentesis is required to make the diagnosis of SBP. New York. along with a poor response to empiric antimicrobial therapy is diagnostic of secondary peritonitis. Mosby. Seymour NE: Abdominal wall. et al (eds): ACS Surgery: Principles and Practice 2006. REFERENCES 1. McGraw-Hill. Philadelphia. ed 8. American College of Surgeons SESAP 13 Category 4 Item 67 Associated with increased urinary 5-hydroxyindoleacetic acid (5-HIAA) (A) Carcinoid syndrome diarrhea (B) Carcinoid valvular disease (C) Both (D) Neither page 443 . a somatostatin analog. cytotoxic chemotherapy has little effect on the clinical manifestations of the carcinoid syndrome.American College of Surgeons SESAP 13 Critique of Category 4 Item 67 The carcinoid syndrome is a paraneoplastic syndrome related to metastatic carcinoid tumors. Patients with the carcinoid syndrome typically have liver metastases that deliver carcinoid mediators to the systemic circulation. suppresses the release of serotonin from metastatic tumors. Octreotide. Rubin J.Modlin IM. Connolly HM. 2003 page 444 . but does not prevent progression of carcinoid valvular disease. Both the diarrhea of the carcinoid syndrome and carcinoid valvular disease can occur with liver metastasis. but again does not prevent development of carcinoid valvular disease. Cancer 79:813-829. This agent is effective at controlling the diarrhea of the carcinoid syndrome. bronchospasm. Both are associated with elevated urinary 5-hydroxyindoleacetic acid (5-HIAA) levels. N Engl J Med 348:1005-1015. and valvular heart disease. flushing. REFERENCES 1. et al: Factors associated with progression of carcinoid heart disease. the primary gastrointestinal carcinoid tumors do not result in the development of the carcinoid syndrome because the mediators of the syndrome are readily metabolized by the liver.Kulke MH. 1999 2. Embolization or ablation of hepatic metastases can also provide symptomatic relief from the diarrhea associated with the carcinoid syndrome. Surveillance for cardiac valvular disease is required regardless of the symptomatic response to therapy. Sandor A: An analysis of 8305 cases of carcinoid tumors. Seward JB. 1997 3. N Engl J Med 340:858-868. Without metastasis. Mayer RJ: Carcinoid tumors. Primary mediators of the carcinoid syndrome include serotonin and substance P.Moller JE. Because metastatic carcinoid is relatively insensitive to cytoxic chemotherapy. Symptoms include diarrhea. American College of Surgeons SESAP 13 Category 4 Item 68 Embolization of hepatic metastases effective for control (A) Carcinoid syndrome diarrhea (B) Carcinoid valvular disease (C) Both (D) Neither page 445 . N Engl J Med 340:858-868.Kulke MH. Octreotide. Cancer 79:813-829. cytotoxic chemotherapy has little effect on the clinical manifestations of the carcinoid syndrome. Primary mediators of the carcinoid syndrome include serotonin and substance P. a somatostatin analog. Mayer RJ: Carcinoid tumors. Seward JB. Both are associated with elevated urinary 5-hydroxyindoleacetic acid (5-HIAA) levels. the primary gastrointestinal carcinoid tumors do not result in the development of the carcinoid syndrome because the mediators of the syndrome are readily metabolized by the liver. Symptoms include diarrhea. Because metastatic carcinoid is relatively insensitive to cytoxic chemotherapy. 1999 2. suppresses the release of serotonin from metastatic tumors. Without metastasis. et al: Factors associated with progression of carcinoid heart disease. but again does not prevent development of carcinoid valvular disease.Modlin IM. bronchospasm. Rubin J. Surveillance for cardiac valvular disease is required regardless of the symptomatic response to therapy.Moller JE. This agent is effective at controlling the diarrhea of the carcinoid syndrome. Sandor A: An analysis of 8305 cases of carcinoid tumors. Patients with the carcinoid syndrome typically have liver metastases that deliver carcinoid mediators to the systemic circulation. Both the diarrhea of the carcinoid syndrome and carcinoid valvular disease can occur with liver metastasis. Embolization or ablation of hepatic metastases can also provide symptomatic relief from the diarrhea associated with the carcinoid syndrome. but does not prevent progression of carcinoid valvular disease.American College of Surgeons SESAP 13 Critique for Category 4 Item 68 The carcinoid syndrome is a paraneoplastic syndrome related to metastatic carcinoid tumors. REFERENCES 1. N Engl J Med 348:1005-1015. and valvular heart disease. flushing. Connolly HM. 1997 3. 2003 page 446 . American College of Surgeons SESAP 13 Category 4 Item 69 Cytotoxic chemotherapy effectively controls symptoms (A) Carcinoid syndrome diarrhea (B) Carcinoid valvular disease (C) Both (D) Neither page 447 . Octreotide. Cancer 79:813-829. but does not prevent progression of carcinoid valvular disease. cytotoxic chemotherapy has little effect on the clinical manifestations of the carcinoid syndrome. Mayer RJ: Carcinoid tumors. the primary gastrointestinal carcinoid tumors do not result in the development of the carcinoid syndrome because the mediators of the syndrome are readily metabolized by the liver. but again does not prevent development of carcinoid valvular disease. Symptoms include diarrhea. N Engl J Med 348:1005-1015. Because metastatic carcinoid is relatively insensitive to cytoxic chemotherapy. Both the diarrhea of the carcinoid syndrome and carcinoid valvular disease can occur with liver metastasis. bronchospasm. This agent is effective at controlling the diarrhea of the carcinoid syndrome. 1997 3. and valvular heart disease. Rubin J. Primary mediators of the carcinoid syndrome include serotonin and substance P. et al: Factors associated with progression of carcinoid heart disease.Moller JE. suppresses the release of serotonin from metastatic tumors.American College of Surgeons SESAP 13 Critique for Category 4 Item 69 The carcinoid syndrome is a paraneoplastic syndrome related to metastatic carcinoid tumors. flushing. Embolization or ablation of hepatic metastases can also provide symptomatic relief from the diarrhea associated with the carcinoid syndrome. Seward JB. 1999 2. Sandor A: An analysis of 8305 cases of carcinoid tumors.Kulke MH. Without metastasis. REFERENCES 1. 2003 page 448 . Both are associated with elevated urinary 5-hydroxyindoleacetic acid (5-HIAA) levels. N Engl J Med 340:858-868. a somatostatin analog.Modlin IM. Surveillance for cardiac valvular disease is required regardless of the symptomatic response to therapy. Patients with the carcinoid syndrome typically have liver metastases that deliver carcinoid mediators to the systemic circulation. Connolly HM. American College of Surgeons SESAP 13 Category 4 Item 70 Experimental treatment (A) Radiofrequency ablation of liver tumors (B) Cryoablation of liver tumors (C) Both (D) Neither page 449 . 5 cm can be ablated with a single electrode. The overall 5year survival after hepatic resection with curative intent ranges from 25% to 50%.American College of Surgeons SESAP 13 Critique for Category 4 Item 70 Radiofrequency ablation and cryoablation have been used successfully in the treatment of metastatic liver disease for years. Izzo F: Colorectal cancer metastatic to the liver: nonresective destructive techniques. Recurrence after radiofrequency ablation and cryoablation consistently exceeds 50% REFERENCES 1. The ions in the tumor tissues start to oscillate along the alternating current. All vascular structures less than 3 mm are either destroyed or thrombosed. ed 8. 2005 page 450 . Izzo F. Clary BM: Management of hepatic metastases from colorectal cancer. with gradual loss of intracellular fluid and ions. Slow tumor cooling causes freezing of the extracellular fluid. Cryoablation achieves cell death via two different mechanisms. et al: Intraoperative radiofrequency ablation or cryoablation for hepatic malignancies.Pearson AS. The results from both techniques are inferior to formal hepatic resection for cancer. The repeat freeze-thaw cycle is increasingly effective as the previously frozen liver is more susceptible to additional cryoablation because of increased thermoconductivity. Larger tumors may need repositioning and several applications for complete tumor eradication. pp 341-347 2. creating frictional heat in the tumor. Fleming RY. A radiofrequency electrode is inserted into liver tumors and high frequency alternating currents are applied. The cryoprobe is placed in contact with the tumor tissue and a freeze-thaw cycle is initiated. Tumors less than 2. 2004.Curley SA. Am J Surg 178:592-599. Philadelphia. with a recurrence rate of 50%. Rapid tumor cooling next to the cryoprobe creates intracellular ice crystals that destroy intracellular organelles and membranes.Sheth KR. Local tissue temperature increases of 60oC to 100oC create coagulation necrosis. in Cameron JL (ed): Current Surgical Therapy. Protein denaturation and membrane disruption follow. Clin Colon Rectal Surg 18:215-223. Radiofrequency energy is used to produce heat and destroy tissues. Peripheral lesions less than 3 cm are ideally suited for cryoabalation. Mosby. Ellis LM. 1999 3. American College of Surgeons SESAP 13 Category 4 Item 71 Inferior results to formal resection (A) Radiofrequency ablation of liver tumors (B) Cryoablation of liver tumors (C) Both (D) Neither page 451 . American College of Surgeons SESAP 13 Critique for Category 4 Item 71 Radiofrequency ablation and cryoablation have been used successfully in the treatment of metastatic liver disease for years. The ions in the tumor tissues start to oscillate along the alternating current. The results from both techniques are inferior to formal hepatic resection for cancer. with a recurrence rate of 50%. pp 341-347 2. Rapid tumor cooling next to the cryoprobe creates intracellular ice crystals that destroy intracellular organelles and membranes. Recurrence after radiofrequency ablation and cryoablation consistently exceeds 50% REFERENCES 1. Clin Colon Rectal Surg 18:215-223. The cryoprobe is placed in contact with the tumor tissue and a freeze-thaw cycle is initiated. ed 8.Sheth KR. 2004. Peripheral lesions less than 3 cm are ideally suited for cryoabalation. Mosby. Izzo F: Colorectal cancer metastatic to the liver: nonresective destructive techniques. Am J Surg 178:592-599. in Cameron JL (ed): Current Surgical Therapy. 2005 page 452 . Fleming RY. A radiofrequency electrode is inserted into liver tumors and high frequency alternating currents are applied. Ellis LM. 1999 3.Pearson AS. Radiofrequency energy is used to produce heat and destroy tissues. Tumors less than 2. The overall 5year survival after hepatic resection with curative intent ranges from 25% to 50%. Protein denaturation and membrane disruption follow. creating frictional heat in the tumor. Local tissue temperature increases of 60oC to 100oC create coagulation necrosis. The repeat freeze-thaw cycle is increasingly effective as the previously frozen liver is more susceptible to additional cryoablation because of increased thermoconductivity. et al: Intraoperative radiofrequency ablation or cryoablation for hepatic malignancies. Cryoablation achieves cell death via two different mechanisms.5 cm can be ablated with a single electrode. Slow tumor cooling causes freezing of the extracellular fluid. with gradual loss of intracellular fluid and ions.Curley SA. Izzo F. All vascular structures less than 3 mm are either destroyed or thrombosed. Philadelphia. Clary BM: Management of hepatic metastases from colorectal cancer. Larger tumors may need repositioning and several applications for complete tumor eradication. American College of Surgeons SESAP 13 Category 4 Item 72 Coagulation necrosis (A) Radiofrequency ablation of liver tumors (B) Cryoablation of liver tumors (C) Both (D) Neither page 453 . creating frictional heat in the tumor. Recurrence after radiofrequency ablation and cryoablation consistently exceeds 50% REFERENCES 1. ed 8.Sheth KR. Tumors less than 2. Radiofrequency energy is used to produce heat and destroy tissues. Slow tumor cooling causes freezing of the extracellular fluid. 2004. Cryoablation achieves cell death via two different mechanisms. The results from both techniques are inferior to formal hepatic resection for cancer. The repeat freeze-thaw cycle is increasingly effective as the previously frozen liver is more susceptible to additional cryoablation because of increased thermoconductivity. 2005 page 454 . with a recurrence rate of 50%. All vascular structures less than 3 mm are either destroyed or thrombosed. Am J Surg 178:592-599. The overall 5year survival after hepatic resection with curative intent ranges from 25% to 50%. A radiofrequency electrode is inserted into liver tumors and high frequency alternating currents are applied. Clary BM: Management of hepatic metastases from colorectal cancer. Izzo F. Fleming RY. Protein denaturation and membrane disruption follow.Pearson AS. Peripheral lesions less than 3 cm are ideally suited for cryoabalation. Clin Colon Rectal Surg 18:215-223. Rapid tumor cooling next to the cryoprobe creates intracellular ice crystals that destroy intracellular organelles and membranes.American College of Surgeons SESAP 13 Critique for Category 4 Item 72 Radiofrequency ablation and cryoablation have been used successfully in the treatment of metastatic liver disease for years. Mosby.Curley SA. The cryoprobe is placed in contact with the tumor tissue and a freeze-thaw cycle is initiated. Larger tumors may need repositioning and several applications for complete tumor eradication. Local tissue temperature increases of 60oC to 100oC create coagulation necrosis. with gradual loss of intracellular fluid and ions. Izzo F: Colorectal cancer metastatic to the liver: nonresective destructive techniques. in Cameron JL (ed): Current Surgical Therapy. Philadelphia. 1999 3. pp 341-347 2. The ions in the tumor tissues start to oscillate along the alternating current.5 cm can be ablated with a single electrode. et al: Intraoperative radiofrequency ablation or cryoablation for hepatic malignancies. Ellis LM. and 90% stenosis of the left ICA (E) 60-year-old diabetic man with unexplained right eye blindness and 50% stenosis of the left ICA page 455 . left ventricular ejection fraction of 20%. episodes of intermittent right arm weakness.40). and a 60% asymptomatic stenosis of the left ICA (D) 72-year-old man with chronic obstructive pulmonary disease (FEV1 =0.American College of Surgeons SESAP 13 Category 5 Item 1 Which of the following is the most appropriate candidate for protected carotid-artery stenting? (A) Healthy 60-year-old man with amaurosis fugax of the left eye and an 80% stenosis of the left internal carotid artery (ICA) (B) Healthy 70-year-old woman with a 50% asymptomatic stenosis of the left ICA (C) 69-year-old man with severe congestive heart failure. J Vasc Surg 33:728-732. making carotid angioplasty with stenting a more feasible option in certain high-risk patients. contralateral laryngeal nerve palsy. These conditions included clinically significant cardiac disease (congestive heart failure. carotid stenting with an emboli-protection device was compared with carotid endarterectomy. or need for open-heart surgery). and length of hospital stay were greater for the endarterectomy group than for the stenting group. In the SAPPHIRE trial.Ouriel K. REFERENCES 1. All of the patients had either symptoms plus carotid stenosis of more than 50% of the luminal diameter or no symptoms plus stenosis of more than 80% of the luminal diameter. 2001 3. The primary finding of the trial was that carotid-artery stenting with use of an emboli-protection device is not inferior to carotid endarterectomy with respect to prevention of stroke. The rates of cranial-nerve palsy. The rates of bleeding complications were similar in the two groups. The Stenting and Angioplasty with Protection in Patients at High Risk for Endarterectomy (SAPPHIRE) trial was conducted to determine a revascularization strategy for patients with severe carotid-artery stenosis and co-morbidities that would have increased the risk associated with carotid endarterectomy. Nickel-titanium (nitinol) crush-resistant stents and emboli-protection devices have been developed to address these problems. Hopkins LN. A second trial (ARCHeR) concluded that carotid artery stenting can be done safely with embolic filter protection. 2006 2. further studies are needed to assess the appropriateness of stenting in low-risk patients. Hertzer NR. et al: Preprocedural risk stratification: identifying an appropriate population for carotid stenting. These results were true for the overall study population and in the subgroups with asymptomatic or symptomatic stenosis.Yadav JS. stroke. N Engl J Med 351:1493-1501. previous radical neck surgery or radiation therapy to the neck. or myocardial infarction in high-risk patients. or age > 80 years. severe pulmonary disease. Fayad P. All patients were required to have at least one co-existing condition that potentially increased the risk of carotid endarterectomy. Its use has been limited by the risks of stent compression and embolization of debris to the brain. abnormal stress test.American College of Surgeons SESAP 13 Critique for Category 5 Item 1 Several trials have shown carotid endarterectomy to be superior to medical management for the prevention of stroke in patients with severe symptomatic or asymptomatic carotid stenosis [North American Symptomatic Carotid Endarterectomy Trial (NASCET) and Asymptomatic Carotid Atherosclerosis Study (ACAS)].Gray WA. et al: Protected carotid stenting in high-surgical risk patients: the ARCHeR results. Kuntz RE. Carotid angioplasty with stenting is a less invasive revascularization strategy than endarterectomy and has been used to treat patients at high surgical risk. et al: Protected carotid-artery stenting versus endarterectomy in high-risk patients. Davis T. The results of the study cannot be generalized to patients at low surgical risk. myocardial infarction) that were equivalent to or lower than those among patients who underwent endarterectomy. Yadav S. contralateral carotid occlusion. recurrent stenosis after endarterectomy. death. J Vasc Surg 44:258-268. Stenting resulted in rates of complications for all adverse events (death. O'hara PJ. Wholey MH. need for further revascularization. Beven EG. 2004 page 456 . The next step in her management should be (A) esophagogastroduodenoscopy (B) capsule endoscopy (C) visceral angiogram (D) ileocecal resection (E) colonoscopy page 457 . and a long history of Crohn’s disease present with a 20-lb weight loss and deep.American College of Surgeons SESAP 13 Category 5 Item 2 A 68-year-old woman with hypertension. She has never been operated on for Crohn’s disease of the terminal ileum. soft. nondistended abdomen without masses. coronary artery disease. which has required only one hospitalization and has been previously wellcontrolled with mesalamine (Pentasa). Rectal examination is normal and stool is hemoccult negative. Abdominal films show no evidence of obstruction. severe enough that she has markedly decreased her food intake. Physical examination shows a flat. The abdominal-pelvic computed tomographic (CT) scan shown is obtained. aching abdominal pain after eating. Lee WA. ed 4. Cherry KJ Jr. with aching (rather than cramping) abdominal pain after eating. Although duplex scanning of the aorta and visceral vessels may adequately image the celiac and mesenteric vessels in some vascular diagnostic laboratories. Seeger JM: Abdominal aortic aneurysms. These patients commonly undergo an extensive evaluation before the possibility of chronic mesenteric ischemia is entertained. conventional angiography with both anteroposterior and lateral views is the gold standard. J Vasc Surg 35:853-859. Clark RC.Huber TS. pp 1711-1747 3. 2006. et al: Current results of open revascularization for chronic mesenteric ischemia: a standard for comparison. capsule endoscopy. or percutaneous approaches with transluminal balloon angioplasty with or without stenting. Philadelphia. Doherty GM. There is certainly no indication for esophageal gastroduodenoscopy. Ozaki CK. Lillemoe KD. ileocecal resection. retrograde bypass grafting from the infrarenal aorta or one of the iliac arteries. including antegrade bypass grafting from the aorta to the affected artery or arteries distal to the occlusion. an event that is unpredictable but to be avoided if possible. her abdominal films do not show evidence of obstruction and her abdominal/pelvic computed tomographic (CT) scan does not show thickened small bowel in the region of the terminal ileum. in Mulholland MW. Once the possibility has been raised. Heart Dis 3:231-235. REFERENCES 1. and food fear.Char D. Lippincott Williams & Wilkins. her postprandial abdominal pain could be due to partial obstruction from her Crohn’s disease in the terminal ileum. They have frequently lost such a significant amount of weight that intra-abdominal malignancy is a concern. Maier RV. However. Whether all or only the symptomatic mesenteric vessels need be re-vascularized is controversial. Certainly. 2001 2. weight loss. postprandial epigastric pain 30 to 45 minutes after eating. Visceral angina is commonly seen in women with the classic triad of weight loss. expeditious evaluation and treatment should be undertaken to prevent progression to infarction. and avoidance of eating. Most studies favor complete revascularization of all of the diseased vessels. Chua HK. Hines G: Chronic mesenteric ischemia: diagnosis and treatment. et al (eds): Greenfield’s Surgery: Scientific Principles and Practice. The CT scan images show heavy calcification in her aorta and image 2 shows dense calcification of her superior mesenteric artery takeoff.Park WM. 2002 page 458 . Her history is suggestive of chronic mesenteric ischemia. or colonoscopy.American College of Surgeons SESAP 13 Critique for Category 5 Item 2 This patient’s history of Crohn’s disease over a long period of time is somewhat confusing. A visceral angiogram with lateral views of the aorta should be performed to delineate the exact nature and extent of her visceral occlusive disease. endarterectomy of the affected vessels. however. A number of therapeutic approaches can be used. He is diabetic and hypertensive. He can only walk 1 block before becoming dyspneic. as shown. The most effective nonoperative therapy for this patient would be (A) elastic compression stockings (Jobst) (B) paste gauze (Unna) boot (C) polyurethane foam dressings (D) hydrocolloid dressings (DuoDerm) (E) intermittent pneumatic compression page 459 .American College of Surgeons SESAP 13 Category 5 Item 3 A 58-year-old man has multiple sores on his right lower leg. He first noted the sores about 2 months ago. On physical examination he has an ulceration measuring 3 x 5 cm on the anteromedial aspect of his right lower leg above the medial malleolus. The surrounding skin is brawny with hemosiderin deposition. Similar skin changes are noted on the anteromedial aspect of the left lower leg. He has palpable dorsalis pedis pulses bilaterally. Moneta GL: Nonoperative treatment of chronic venous insufficiency. Philadelphia. 2005. J Vasc Surg 7:478-483. Elsevier Saunders. and elastic compression wraps. A trial of nonoperative therapy is indicated in this high-risk surgical patient. pp 2241-2245 2. There is no evidence of arterial insufficiency. which may have influenced the healing rates. Schuler JJ. IPC stockings alone have not been compared with Unna boots. 1988 page 460 . many studies have shown compression therapy to be an effective treatment for venous hypertension.Giswold ME. a 70% success rate was achieved with Unna boots versus a 40% healing rate with hydrocolloid dressings. polyurethane foam dressings. ed 6. Several prospective randomized studies have found that paste gauze (Unna) boots are superior to elastic compression stockings (Jobst). REFERENCES 1. Meyer JP. Compressive therapy is the most effective nonoperative treatment for chronic venous insufficiency. in Rutherford RB (ed): Vascular Surgery. randomized trial of Unna’s boots versus hydroactive dressing in the treatment of venous stasis ulcers. et al: A prospective. Durham JR. Although the exact mechanism is unknown.Kikta MJ.American College of Surgeons SESAP 13 Critique for Category 5 Item 3 This patient has venous stasis ulcers. the IPC patients also had periods of leg elevation. In a trial comparing Unna boots with hydrocolloid (DuoDerm) dressing. However. The combination of intermittent pneumatic compression (IPC) stockings and elastic compression wraps was found to be superior to elastic compression alone in healing venous stasis ulcers. he was able to climb 4 flights of stairs without difficulty. He complains of numbness and tingling below the knee. Before his toe turned blue. On physical examination. his first toe is bluish. He also has a palpable pulsatile mass in his right popliteal fossa. and he has palpable pedal pulses bilaterally. The most appropriate treatment would be (A) systemic anticoagulation (B) ligation of the aneurysm with bypass graft (C) endovascular stent (D) operative resection of the aneurysm with primary repair (E) aneurysmorrhaphy with interposition graft page 461 .American College of Surgeons SESAP 13 Category 5 Item 4 A 53-year-old man has a 1-week history of a painful blue right first toe. Ultrasound examination confirms the presence of a 4-cm right popliteal aneurysm. with a mean follow-up of approximately 4 years. Chiou AC. venous congestion. and edema. numbness. in Rutherford RB (ed): Vascular Surgery. Most complications occur in aneurysms with diameters > 2 cm. This alternative approach is indicated for larger aneurysms causing any compressive symptoms. and then the bypass graft is placed within the bed of the aneurysm. the placement of stents at the knee joint in a younger person with a low operative risk has not been thoroughly studied. Elsevier Saunders. Systemic anticoagulation may reduce the risk of thromboembolic events from popliteal aneurysm. et al: Graft patency is not the only clinical predictor of success after exclusion and bypass of popliteal artery aneurysms.American College of Surgeons SESAP 13 Critique for Category 5 Item 4 This patient suffered a thromboembolic complication from a large popliteal aneurysm. but would have no effect on any compressive symptoms. because they do not resolve with simple proximal and distal ligation. The aneurysm is ligated proximally and distally. Decaprio JD. Surgical techniques for treatment of popliteal aneurysms include exclusion of the aneurysm with a bypass graft. and early operation for asymptomatic aneurysms larger than 2 cm is indicated.Van Bockel JH. These aneurysms can result in loss of limb. In a combined review of 536 asymptomatic patients with popliteal aneurysm. The durability and long-term patency of these stents at a major joint site is still under question. 2003 2. The associated amputation rate was 25%. Progression to compressive symptoms would almost always be expected in an aneurysm of this size. ed 6. followed by ligation of any collateral vessels. Vascular stenting has recently begun to be used in more widespread cases. Aneurysmorrhaphy with decompression eliminates the possibility of recurrence or continued expansion of the aneurysm. 35% developed thromboembolic complications. However. 2005.Jones WT III. Hamming JF: Lower extremity aneurysms. the aneurysm can be incised. paresthesia. Alternatively. J Vasc Surg 37:392-398. pp 1534-1550 page 462 . Hagino RT. Philadelphia. then continuity is re-established using an extra-aneurysmal bypass graft. REFERENCES 1. Symptoms of compression include lower leg pain. Which of the following statements about her condition is TRUE? (A) Higher incidence in females (B) Often accompanied by neurologic symptoms (C) Not successfully treated with anticoagulation therapy (D) Definitively treated with catheter-directed thrombolysis (E) Associated with venous gangrene of the upper extremities page 463 .American College of Surgeons SESAP 13 Category 5 Item 5 A 28-year-old female body builder has the acute onset of a swollen right arm. 2005. pp 840-845 page 464 . in Rutherford RB (ed): Vascular Surgery. catheterdirected thrombolysis should be initiated once the diagnosis of acute subclavian vein thrombosis has been made. This condition typically occurs in young patients and has a 2:1 male:female ratio. Elsevier Saunders. ed 8.Green RM: Subclavian-axillary vein thrombosis.Shepard AD. ed 6. However. Parikshak M: Thoracic outlet syndrome. The traditional treatment of this condition consisted of elevation of the upper extremity and anticoagulation. The use of thrombolytic therapy gained great favor after several authors reported poor results with operative thrombectomy. persistent symptoms in up to 80% of patients have led to adoption of a more aggressive approach. In the absence of any contraindication. REFERENCES 1. The etiology is believed to be compression of the subclavian vein at the thoracic outlet. Mosby. The problem often becomes chronic. Philadelphia. Venous gangrene of the upper extremity is extremely rare with this condition.American College of Surgeons SESAP 13 Critique for Category 5 Item 5 The patient described in this item has a primary axillary-subclavian vein thrombosis. because the vein and nerves are at the opposite ends of the thoracic outlet. and most patients will have debilitating symptoms of venous outflow obstruction. 2004. This should be followed by dynamic venography and operative thoracic outlet decompression. Neurologic symptoms do not usually accompany primary venous thrombosis. in Cameron JL (ed): Current Surgical Therapy. Philadelphia. A history of strenuous or repetitive upper extremity activity often precedes thrombosis. pp 1371-1384 2. and hyperglycemia. The plantar surface of the foot is pictured. leukocytosis. Which of the following statements about management of this problem is TRUE? (A) Fever is a reliable indicator of the severity of infection (B) Antimicrobial therapy alone will resolve 50% of cases (C) Swab cultures of purulent drainage are adequate (D) Magnetic resonance imaging (MRI) will not reliably diagnose osteomyelitis in this patient (E) Immediate transmetatarsal amputation is required page 465 .American College of Surgeons SESAP 13 Category 5 Item 6 A 44-year-old man with diabetes mellitus complicated by peripheral neuropathy presents with malaise. He has received 1 week of antibiotic therapy. American College of Surgeons SESAP 13 Critique for Category 5 Item 6 Foot infections are a frequent complication of diabetes mellitus, accounting for up to 20% of diabetes-related hospital admissions among the 16 million diabetics in the US. Foot ulcers are even more common, occurring in 15% of all patients with diabetes at some time during their lifetimes. Of patients with diabetes who develop foot ulcers, 15% also develop underlying osteomyelitis, and a systematic approach to treatment is imperative. Because of the chronic immunosuppression that accompanies diabetes mellitus, the usual physical signs and symptoms of infection are often not present, and fever is not a reliable indicator of the severity of the underlying foot infection. Temperatures > 100°F are typically present in only 50% of diabetic patients presenting with significant foot infections. Diabetic foot wounds must first be probed to ascertain the presence of underlying sinus tracts or abscesses; the presence of deep or proximal extension along fascial planes; and whether bones and joints are involved in the deep infection. Probing to bone by gently advancing a sterile surgical probe into the depth of the wound has a positive predictive value of 89% for the diagnosis of osteomyelitis in diabetic patients presenting with foot infections. Initial deep cultures, aerobic and anaerobic, can usually be taken at the bedside in these frequently neuropathic patients. Diabetic patients presenting with foot infections and possible osteomyelitis typically have mixed infections involving a variety of different organisms, and broad-spectrum antibiotic therapy should initially cover Staphylococcus aureus, Streptococcus, Enterobacter, and Bacteroides fragilis. Three to five organisms are typically recovered from deep tissue cultures of infected foot wounds. Swab cultures of purulent drainage, which often miss organisms located within the deeper tissues, are never indicated. A variety of techniques can be used to confirm the clinical diagnosis of osteomyelitis. Magnetic resonance imaging (MRI) is a sensitive and specific indicator of true bone marrow infection, but previous trauma, operation, or Charcot osteoarthropathy significantly reduces the specificity of MRI because of the chronic, noninfectious marrow inflammation present in these subgroups. For these patients, combining bone scintigraphy with leukocyte scans has greater accuracy than MRI, and has a specificity in excess of 80%. Antimicrobial therapy alone will rarely suffice in managing most foot infections. This point is succinctly made in the IDSA guidelines for treatment of diabetic foot infections. Management should always include avoidance of weightbearing, immediate drainage and debridement procedures, hyperglycemic control, and management of ischemia. Diabetic patients with serious foot infections benefit from aggressive early surgical management. Early surgical debridement or limited partial foot amputation with adjunctive antimicrobial therapy results in an overall shorter length of hospital stay, as well as a reduced need for major amputations. Delaying operation for more than 3 days increases the incidence of major amputations, morbidity, and mortality. REFERENCES 1.Frykberg RG: An evidence-based approach to diabetic foot infections. Am J Surg 186:44S-54S; discussion 61S-64S, 2003 2.Lipsky BA, Berendt AR, Deery HG, Embil JM, et al: Diagnosis and treatment of diabetic foot infections. Clin Infect Dis 39:885-910, 2004 page 466 American College of Surgeons SESAP 13 3.Rauwerda JA: Surgical treatment of the infected diabetic foot. Diabetes Metab Res Rev 20:S41S44, 2004 4.Treiman GS, Oderich GS, Ashrafi A, Schneider PA: Management of ischemic heel ulceration and gangrene: an evaluation of factors associated with successful healing. J Vasc Surg 31:1110-1118, 2000 5.van Baal JG: Surgical treatment of the infected diabetic foot. Clin Infect Dis 39:S123-S128, 2004 page 467 American College of Surgeons SESAP 13 Category 5 Item 7 A 45-year-old woman undergoes angiography for severe hypertension, as shown. Which of the following statements about the lesion(s) shown is TRUE? (A) The most likely etiology is atherosclerosis (B) Lesions > 2.0 cm are associated with a high incidence of rupture (C) The most common location is in the main renal artery (D) Surgical repair improves control of hypertension in 60% of patients (E) Lesions < 2.0 cm should be managed nonoperatively page 468 American College of Surgeons SESAP 13 Critique for Category 5 Item 7 The angiogram demonstrates an aneurysm of the renal artery. Renal artery aneurysms are uncommon, occurring in approximately 0.09% of the general population, and are often encountered as an incidental finding on imaging studies performed for other diseases. These patients have a different demographic profile than patients with aneurysms in other locations. The most common etiology of renal artery aneurysms is arterial fibrodysplasia. The underlying arterial matrix disruption leads to aneurysm formation, particularly at branch points where discontinuities in the internal elastic lamina are common, even in healthy patients. Arterial fibrodysplasia occurs more commonly in women than in men. Women who are multiparous are more prone to the development of arterial fibrodysplasia. During the latter stages of gestation, an alteration in hormone and enzyme activity contributes to the tissue relaxation necessary for parturition. Sustained release of these matrix-altering substances during the latter stages of pregnancy might predispose to formation of renal artery aneurysms by causing irreversible changes to the elastic tissues at arterial bifurcations, including the renal arteries. In a 35-year clinical experience with 252 renal artery aneurysms in 168 patients at the University of Michigan, most renal artery aneurysms were asymptomatic. The average size of the aneurysms was 1.5 cm. Importantly, 60% of patients undergoing repair had a significant decline in blood pressure postoperatively, while taking fewer antihypertensive medications. Late renal artery rupture did not occur in the nonoperative patients, but this group saw no change in their hypertension. The average number of renal artery aneurysms per patient was 1.5. Most patients had solitary aneurysms, but 53 had multiple renal artery aneurysms. Bilateral aneurysms occurred in 32 patients. The most common location for renal artery aneurysms was at the bifurcation of the renal artery or the first order renal artery branch, rather than in the main renal artery itself. Lesions > 2 cm in diameter are not associated with a higher incidence of rupture compared with lesions < 2 cm. The factors relating to rupture appear to be more complicated than size alone. One of the most important findings of the Michigan study was the reduction in hypertension in patients who underwent aneurysmectomy and renal artery reconstruction, suggesting that the absolute size of the renal artery aneurysm should not be the deciding factor regarding surgical or nonsurgical management. In fact, patients with aneurysms < 2 cm in diameter can have a significant improvement in their hypertension and should be offered surgical resection. In the Michigan series, repair of renal artery aneurysms 1 cm or larger in patients with difficult-to-control hypertension was efficacious. Most aneurysms 1.5 to 2 cm in diameter and all those larger than 2 cm in diameter, regardless of blood pressure status, should be treated surgically if their anatomic characteristics suggest a relatively noncomplex renal artery construction and the procedure is undertaken by an experienced surgeon. REFERENCES 1.Cherr GS, Hansen KJ, Craven TE, Edwards MS, et al: Surgical management of atherosclerotic renovascular disease. J Vasc Surg 35:236-245, 2002 2.Chiesa R, Astore D, Guzzo G, Frigerio S, et al: Visceral artery aneuryms. Ann Vasc Surg 19:42-48, 2005 3.English WP, Pearce JD, Craven TE, Wilson DB, et al: Surgical management of renal artery aneurysms. J Vasc Surg 40:53-60, 2004 4.Henke PK, Cardneau JD, Welling TH III, Upchurch GR Jr, et al: Renal artery aneurysms: a 35-year page 469 American College of Surgeons SESAP 13 clinical experience with 252 aneurysms in 168 patients. Ann Surg 234:454-463, 2001 5.Pfeiffer T, Reiher L, Grabitz K, Grunhage B, et al: Reconstruction for renal artery aneurysm: operative techniques and long-term results. J Vasc Surg 37:293-300, 2003 6.Saltzberg SS, Maldonado TS, Lamparello PJ, Cayne NS, et al: Is endovascular therapy the preferred treatment for all visceral artery aneurysms? Ann Vasc Surg 19:507-515, 2005 7.Sessa C, Tinelli G, Porcu P, Aubert A, et al: Treatment of visceral artery aneurysms: description of a retrospective series of 42 aneurysms in 34 patients. Ann Vasc Surg 18:695-703, 2004 page 470 American College of Surgeons SESAP 13 Category 5 Item 8 A 56-year-old diabetic man with a history of transmetatarsal amputation presents with fever, chills, fatigue, malaise, leukocytosis, and hyperglycemia. His transmetatarsal amputation site is pictured. The dorsalis pedis and posterior tibial pulses are absent. Proper management of this condition would be broad-spectrum antibiotics and (A) revision of transmetatarsal amputation (B) immediate surgical debridement of soft tissue only (C) immediate surgical debridement, and vascular reconstruction of pedal blood flow (D) immediate surgical debridement, with negative pressure dressing to improve blood flow (E) immediate below-knee amputation page 471 American College of Surgeons SESAP 13 Critique for Category 5 Item 8 The illustration shows a clinically infected, gangrenous transmetatarsal amputation site. The patient has systemic signs and symptoms of sepsis, including leukocytosis and hyperglycemia. Given the absence of dorsalis pedis and posterior tibial pulses and the clinical burden of infection within the remaining foot, the immediate need is to halt the spread of sepsis before it is fatal. Proper management includes immediate broad-spectrum antibiotic therapy and immediate guillotine amputation in the distal third of the leg. Revision of the transmetatarsal amputation would be inappropriate in this setting of systemic sepsis, nor will immediate debridement of soft tissue be sufficient to remove all infection present, leaving the patient at risk of developing worsening sepsis, multi-organ system failure, and death. Anything that would delay immediate below-knee amputation, which in this clinical setting can be lifesaving, would be contraindicated. Once the sepsis is controlled, revision of the amputation can be undertaken. Negative pressure dressings do not improve blood flow to ischemic tissues. REFERENCES 1.Faries PL, Teodorescu VJ, Morrissey NJ, Hollier LH, et al: The role of surgical revascularization in the management of diabetic foot wounds. Am J Surg 187:34S-37S, 2004 2.Lepantalo M, Biancari F, Tukiainen E: Never amputate without consultation of a vascular surgeon. Diabetes Metab Res Rev 16:S27-S32, 2000 3.Rivers SP, Scher L, Veith FJ: Indications for distal arterial reconstruction in the presence of palpable pedal pulses. J Vasc Surg 12:552-557, 1990 4.Treiman GS, Oderich GS, Ashrafi A, Schneider PA: Management of ischemic heel ulceration and gangrene: an evaluation of factors associated with successful healing. J Vasc Surg 31:1110-1118, 2000 page 472 American College of Surgeons SESAP 13 Category 5 Item 9 Which of the following statements about antithrombotic treatments is TRUE? (A) In the absence of antithrombin III, unfractionated heparin has no significant anticoagulant effect (B) Low molecular weight heparins share the ability to accelerate the activity of factor Xa (C) Warfarin inhibits the absorption of vitamin K from the intestinal tract (D) Hirudin, a synthetic thrombin inhibitor, is dependent on antithrombin III for anticoagulant activity (E) The addition of aspirin to heparin increases the anticoagulant effect without increasing the risk of hemorrhagic side effects page 473 American College of Surgeons SESAP 13 Critique for Category 5 Item 9 Unfractionated heparin is an indirect thrombin inhibitor; its chief antithrombotic effect is mediated through acceleration of the interaction of antithrombin III with thrombin (factor IIa). In the absence of antithrombin, unfractionated heparin has no significant antithrombotic effect. Antithrombin III levels can be increased by the infusion of fresh frozen plasma. The anticoagulant efficacy of low molecular weight heparins is due primarily to inhibition of factor Xa. Inhibition of factor Xa results in down-regulation of the production of thrombin, which is the central bioregulatory enzyme in hemostasis. Warfarin, which was developed under a grant from the Wisconsin Alumni Research Foundation (WARF), inhibits vitamin K-dependent terminal carboxylation of factors II, VII, IX, and X in the liver. Warfarin is an oral anticoagulant, but has no impact on gastrointestinal absorption of vitamin K. Hirudin, isolated from the saliva of the medical leech and reproduced with recombinant technology, is a direct thrombin inhibitor that is not dependent on the activity of antithrombin III. It is increasingly used as an alternative to heparin, particularly in patients with heparin-associated antibodies. The addition of aspirin to heparin may increase the antithrombotic efficacy, but does so at the cost of increased hemorrhagic side effects. REFERENCES 1.Liem TK, Silver D: Hemostasis and thrombosis, in Moore WS (ed): Vascular Surgery: A Comprehensive Review, ed 6. Philadelphia, WB Saunders, 2002, pp 71-90 2.Meissner MH: Antithrombotic therapy, in Rutherford RB (ed): Vascular Surgery, ed 6. Philadelphia, Elsevier Saunders, 2005, pp 511-529 page 474 American College of Surgeons SESAP 13 Category 5 Item 10 Which of the following statements about management of abdominal aortic aneurysm (AAA) is TRUE? (A) Elective operation should be considered for patients with symptomatic AAA in the absence of significant co-morbidities (B) The risk of rupture is higher in women than men for small aneurysms (C) In an otherwise healthy 75-year-old man, a 4.5-cm aneurysm should be repaired (D) An unreliable patient who is unlikely to comply with lifelong surveillance should be preferentially offered endograft versus open repair (E) Mortality is not related to the hospital’s volume of AAA repairs performed page 475 American College of Surgeons SESAP 13 Critique for Category 5 Item 10 The mortality of untreated rupture of abdominal aortic aneurysms (AAA) approaches 100%. AAA presenting with abdominal and/or back pain (most common) should be repaired urgently in almost all patients regardless of co-morbidity because of the excessive risk of rupture if left untreated. To avoid complex operations in the middle of the night with a less skilled operating team, some authors have advocated delaying repair until morning in selected hemodynamically stable symptomatic patients without evidence of free rupture on computed tomographic (CT) scan. The UK Small Aneurysm Trial demonstrated a significantly higher risk of rupture for small aneurysms in women compared with similar sized aneurysms in men. Patients undergoing endograft repair of AAA must be willing to comply with rigorous lifelong surveillance to assure the proper function of the endograft. Surveillance is designed to detect graft-related complications such as graft migration, endoleak, and limb obstruction. The annual risk of rupture of a 4.5-cm AAA in a 75-year-old man is approximately 1% per year according to the VA-sponsored Aneurysm Detection and Management (ADAM) trial. No survival advantage was demonstrated for open repair of small (4.0 to 5.5 cm) AAA in two large trials in both the United States (ADAM) and Great Britain (UK Small Aneurysm Trial). The average life expectancy for an 80-year-old man after successful repair of AAA is approximately 7 years, or about half the life expectancy for an age-matched man without AAA repair. Mortality is lower with higher hospital volume. REFERENCES 1.Brown PM, Zelt DT, Sobolev B: The risk of rupture in untreated aneurysms: the impact of size, gender, and expansion rate. J Vasc Surg 37:280-284, 2003 2.Schermerhorn ML, Cronenwett JL: Abdominal aortic and iliac aneurysms, in Rutherford RB (ed): Vascular Surgery, ed 6. Philadelphia, Elsevier Saunders, 2005, pp 1408-1451 3.United Kingdom Small Aneurysm Trial Participants: Long-term outcomes of immediate repair compared with surveillance of small abdominal aortic aneurysms. N Engl J Med 346:1445-1452, 2002 page 476 American College of Surgeons SESAP 13 Category 5 Item 11 Which of the following statements about thromboangiitis obliterans (Buerger’s disease) is TRUE? (A) Women outnumber men 4:1 (B) It is caused by a primary arterial infection with chlamydia (C) In a smoker, tobacco withdrawal should be gradual to avoid rebound arterial vasospasm (D) It is characterized by thrombotic occlusions of small- and medium-sized arteries (E) Despite complete abstinence from nicotine, most patients eventually progress to major limb loss page 477 American College of Surgeons SESAP 13 Critique for Category 5 Item 11 Thromboangiitis obliterans (Buerger’s disease) is a chronic arterial inflammatory condition of unknown etiology that leads to arterial occlusion in the medium and small extremity arteries. Major risk factors for atherosclerosis must be absent, and proximal sources of emboli must be excluded, as well as underlying autoimmune disease and hypercoagulable states. Both men and women can be affected, but most patients are male. Many patients develop a form of superficial thrombophlebitis that can be helpful in establishing the diagnosis. Complete abstinence from tobacco in all forms (including chewing tobacco and snuff) is the only known effective treatment. Most patients are unable to quit smoking. If nicotine exposure is not curtailed, amputation rates can approach 80% to 90%. Cessation of tobacco use can reduce this rate by 50%. REFERENCES 1.Landry GJ, Abou-Zamzam AM Jr, Edwards JM, Porter JM: Nonatherosclerotic vascular disease, in Moore WS (ed): Vascular Surgery: A Comprehensive Review, ed 6. Philadelphia, WB Saunders, 2002, pp 119-153 2.Olin JW: Thromboangiitis obliterans (Buerger’s disease), in Rutherford RB (ed): Vascular Surgery, ed 6. Philadelphia, Elsevier Saunders, 2005, pp 404-418 page 478 American College of Surgeons SESAP 13 Category 5 Item 12 Which of the following statements about the natural history of intermittent claudication is TRUE? (A) Five-year survival is > 90% (B) Most patients eventually require revascularization to avoid amputation (C) One in 4 patients will eventually undergo major amputation (D) Intermittent claudication is a risk factor for adverse cardiovascular events (E) Abstinence from tobacco does not improve the symptoms of intermittent claudication page 479 American College of Surgeons SESAP 13 Critique for Category 5 Item 12 The term claudication is derived from the root word meaning “to limp” and is defined by the clinical development of reproducible leg pain that is relieved with rest. Intermittent claudication is relatively common in elderly patients in Western societies. The ankle:brachial index (ABI) is the ratio of the ankle pressure to the arm pressure; normal values range from 0.9 to 1.2. Peripheral arterial disease, as defined by an ABI < 0.9, is detectable in approximately 25% of elderly patients seen in a primary care practice. In approximately half of these patients, the peripheral arterial disease is asymptomatic. Intermittent claudication is relatively benign, leading to amputation in only a small fraction (5% to 7%) of affected patients. Only about 25% of patients need either surgical or catheter-based intervention. Intermittent claudication is a marker for patients at risk of future adverse cardiovascular events, and initial treatment should be directed at modifying the risk factors of systemic atherosclerosis: smoking, obesity, hypertension, diabetes, and hyperlipidemia. The 5-year mortality of patients with intermittent claudication approaches 30% to 50%, due chiefly to cardiovascular events. Smoking cessation improves the symptoms of intermittent claudication. REFERENCES 1.Nehler MR, Taylor LM Jr, Moneta GL, Porter JM: Natural history and nonoperative treatment in chronic lower extremity ischemia, in Moore WS (ed): Vascular Surgery: A Comprehensive Review, ed 6. Philadelphia, WB Saunders, 2002, pp 264-276 2.Nehler MR, Wolford H: Natural history and nonoperative treatment of chronic lower extremity ischemia, in Rutherford RB (ed): Vascular Surgery, ed 6. Philadelphia, Elsevier Saunders, 2005, pp 1083-1094 page 480 American College of Surgeons SESAP 13 Category 5 Item 13 A 30-year-old man presents with a brief history of crampy, midabdominal pain. Physical examination is notable for diffuse mild tenderness, but frank signs of peritonitis are absent. Computed tomographic (CT) scan suggests findings consistent with mesenteric venous thrombosis. The initial recommended treatment for symptomatic mesenteric venous thrombosis is (A) venous thrombectomy (B) anticoagulation with heparin (C) tissue plasminogen activator (tPA) via the superior mesenteric artery (D) warfarin (Coumadin) anticoagulation (E) systemic tPA page 481 American College of Surgeons SESAP 13 Critique for Category 5 Item 13 The diagnosis of mesenteric venous thrombosis has been made with increasing frequency during the past several years because of the proliferation of cross-sectional imaging studies. In asymptomatic patients, the treatment of incidentally discovered thrombus in branches of the portal venous system, including the inferior mesenteric and superior mesenteric veins and the splenic veins, must be individualized based on the risk factor present. Hematologic assessment of a hypercoagulable state is recommended. Long-term anticoagulation with warfarin (Coumadin) in asymptomatic patients is not generally recommended. In the symptomatic patient, treatment is clearly indicated. Administration of thrombolytic agents including urokinase and tissue plasminogen activators (tPA) has not been definitively demonstrated to accelerate the lysis of mesenteric venous thrombosis or improve the clinical outcome. Systemic anticoagulation with intravenous heparin and fluid resuscitation are the mainstays of therapy. Surgical exploration is reserved for patients with definitive signs of abdominal catastrophe. Figure 1 shows the cyanotic, edematous, and friable bowel typical of mesenteric venous thrombosis. Figure 2 reveals the bowel opened, demonstrating the venous engorgement of the mucosa. The apparent lethality of mesenteric venous thrombosis has decreased during the last decade due to earlier detection and treatment. Mesenteric venous thrombosis is responsible for less than 10% of clinically significant mesenteric ischemia. A hypercoagulable state can be identified in more than 90% of patients. REFERENCES 1.Hollingshead M, Burke CT, Mauro MA, Weeks SM, et al: Transcatheter thrombolytic therapy for acute mesenteric and portal vein thrombosis. J Vasc Interv Radiol 16:651-661, 2005 2.Morasch MD: Intestinal ischemia caused by venous thrombosis, in Rutherford RB (ed): Vascular Surgery, ed 6. Philadelphia, Elsevier Saunders, 2005, pp 1748-1751 3.Schwartz LB, McKinsey JF, Gewertz BL: Visceral ischemic syndromes, in Moore WS (ed): Vascular Surgery: A Comprehensive Review, ed 6. Philadelphia, WB Saunders, 2002, pp 570-584 page 482 American College of Surgeons SESAP 13 Category 5 Item 14 A 75-year-old man with a ruptured abdominal aortic aneurysm, as shown, is taken urgently to the operating room for repair Which of the following has the most important influence on operative mortality? (A) Distance from patient’s home to hospital (B) Number of co-morbidities (C) Surgeon experience (D) Annual hospital volume of aneurysm repair (E) Site of aortic rupture page 483 American College of Surgeons SESAP 13 Critique for Category 5 Item 14 Ruptured abdominal aortic aneurysms (AAA) are associated with extremely high mortality rates. Large population studies have reported an overall mortality rate of 90% for ruptured AAA, including patients who died before reaching the hospital. The operative mortality for patients who arrive at the hospital alive is approximately 50%, but a number of factors influence the 30-day mortality. A number of patient- and surgeon-specific risk factors have been reported from respective population studies representing the states of Maryland and North Carolina and the province of Ontario. Among patient variables examined, advanced patient age was significantly associated with mortality; however, race, distance from home to hospital, and medical complexity were not significant factors. Among surgeonspecific variables, only surgeon experience with ruptured AAA was significant: surgeons with career experience > 10 ruptured AAA had a significantly lower mortality rate than less experienced surgeons. The annual volume of elective AAA repairs did not influence outcome, nor did hospital volume of ruptured or elective AAA repair. Some technical factors such as aneurysm extension above the renal arteries may be important determinants of outcome, but the site of aneurysm rupture does not have an impact on survival. REFERENCES 1.Dardik A, Burleyson GP, Bowman H, Gordon TA, et al: Surgical repair of ruptured abdominal aortic aneurysms in the state of Maryland: factors influencing outcome among 527 recent cases. J Vasc Surg 28:413-421, 1998 2.Dueck AD, Kucey DS, Johnston KW, Alter D, et al: Survival after ruptured abdominal aortic aneurysm: effect of patient, surgeon, and hospital factors. J Vasc Surg 39:1253-1260, 2004 3.Rutledge R, Oller DW, Meyer AA, Johnson GJ Jr: A statewide, population-based, time-series analysis of the outcome of ruptured abdominal aortic aneurysm. Ann Surg 223:492-505, 1996 page 484 American College of Surgeons SESAP 13 Category 5 Item 15 A 78-year-old man who has the arteriogram shown is being evaluated for endovascular repair of a 6.5-cm aortic aneurysm. To deploy the endograft safely, the left limb of the graft must be extended directly into the left external iliac artery. To prevent an endoleak, the left hypogastric artery should be occluded using coil embolization. The most likely adverse event associated with occlusion of the left hypogastric artery is (A) buttock claudication (B) buttock necrosis (C) spinal cord ischemia (D) ischemic colitis (E) impotence page 485 American College of Surgeons SESAP 13 Critique for Category 5 Item 15 Endovascular aortic aneurysm repair (EVAR) has become a standard treatment option for aortoiliac aneurysms. As devices have improved, endovascular treatment has expanded to include patients with challenging vascular anatomy such as short aneurysm necks and small iliac arteries. Currently, more than half of patients with aortic aneurysms can be expected to be candidates for EVAR. Successful treatment requires that the graft device creates a seal at its proximal and distal ends. The distal end, or landing zone, has been a particular problem for patients with common iliac aneurysms that extend to the bifurcation. In addition to the problem of inadequate seal (type I endoleak), pelvic ischemia has been a concern if the internal iliac artery is occluded. However, increasing experience has shown that interruption of a single internal iliac artery is usually well tolerated. This is usually accomplished by coil occlusion or placement of an occluding device in the internal iliac artery, with placement of the ipsilateral graft limb within the external iliac artery beyond the bifurcation. Most patients remain completely asymptomatic, but up to 25% to 30% of patients develop temporary ipsilateral buttock claudication. Persistent buttock claudication occurs in 10% to 15% of treated patients. Buttock necrosis, ischemic colitis, spinal cord ischemia, and impotence have all been reported, but are rare complications. Although bilateral internal iliac artery occlusion has been reported to be well tolerated in some patients, most surgeons recommend maintaining flow in at least one internal iliac artery. Internal iliac artery bypass has been recommended to reduce the risk of buttock claudication in patients who require bilateral internal iliac artery coil embolization for EVAR. REFERENCES 1.Lyden SP, Sternbach Y, Waldman DL, Green RM: Clinical implications of internal iliac artery embolization in endovascular repair of aortoiliac aneurysms. Ann Vasc Surg 15:539-543, 2001 2.Mehta M, Veith FJ, Ohki T, Cynamon J, et al: Unilateral and bilateral hypogastric artery interruption during aortoiliac aneurysm repair in 154 patients: a relatively innocuous procedure. J Vasc Surg 33:S27-S32, 2001 3.Rhee RY, Muluk SC, Tzeng E, Missig-Carroll N, et al: Can the internal iliac artery be safely covered during endovascular repair of abdominal aortic and iliac artery aneurysms? Ann Vasc Surg 16:2936, 2002 page 486 American College of Surgeons SESAP 13 Category 5 Item 16 Two days after placement of a brachiocephalic fistula for hemodialysis access, a 53-year-old man has extensive edema of the ipsilateral extremity from the hand to the shoulder. Venography confirms a stenosis in the proximal subclavian vein creating an 85% diameter loss. The best management would be (A) chronic oral anticoagulation (B) fistula ligation (C) fistula banding near the arterial anastomosis (D) balloon angioplasty of the subclavian vein stenosis (E) subclavian-jugular venous bypass page 487 American College of Surgeons SESAP 13 Critique for Category 5 Item 16 Pain and swelling developing in an extremity after placement of an arteriovenous access is indicative of venous hypertension. The most common cause is a hemodynamically significant stenosis in the central venous system, usually due to previous central venous catheterization. The most common location of the lesion is the subclavian vein, followed in decreasing order by the innominate vein, axillary vein, and superior vena cava. Long-term oral anticoagulation will not result in symptomatic improvement because it does not alter the hemodynamic characteristics of the stenosis. Symptomatic relief can be gained by ligating the fistula, but this approach sacrifices a functioning access in a patient who may have limited options for alternative access sites. External banding will reduce flow in the fistula, but continued venous hypertension is likely. Correction of the venous hypertension requires treatment of the central vein stenosis. Although endovascular options may be a reasonable alternative, balloon angioplasty of central vein stenoses is associated with early recurrence in most cases. Multiple procedures are often required; long-term relief is attained in less than one third of patients. Open surgical techniques such as subclavian-jugular bypass or jugular turndown (jugular subclavian vein transposition) procedures are the better option, especially in younger patients with reasonable long-term prognosis. Although these operations are more invasive, long-term patency and symptomatic relief are better than with angioplasty. REFERENCES 1.Chandler NM, Mistry BM, Garvin PJ: Surgical bypass for subclavian vein occlusion in hemodialysis patients. J Am Coll Surg 194:416-421, 2002 2.Lumsden AB, MacDonald MJ, Isiklar H, Martin LG, et al: Central venous stenosis in the hemodialysis patient: incidence and efficacy of endovascular treatment. Cardiovasc Surg 5:504-509, 1997 3.Neville RF, Abularrage CJ, White PW, Sidawy AN: Venous hypertension associated with arteriovenous hemodialysis access. Semin Vasc Surg 17:50-56, 2004 4.Sprouse LR II, Lesar CJ, Meier GH III, Parent FN, et al: Percutaneous treatment of symptomatic central venous stenosis. J Vasc Surg 39:578-582, 2004 page 488 American College of Surgeons SESAP 13 Category 5 Item 17 A 78-year-old woman with chronic atrial fibrillation is admitted with a 2-hour history of severe midepigastric abdominal pain that began suddenly. Her abdomen is nontender on physical examination. An abdominal computed tomographic (CT) scan obtained in the emergency department is shown. The next step in management should be (A) mesenteric arteriogram (B) biliary excretion, eg, HIDA, scan (C) systemic urokinase (D) anticoagulation and serial examination (E) exploratory laparotomy page 489 American College of Surgeons SESAP 13 Critique for Category 5 Item 17 Acute mesenteric ischemia typically presents with the sudden onset of severe, unrelenting abdominal pain. In the early stages, abdominal examination is relatively benign. The nonspecific diagnostic findings usually result in delay obtaining a diagnosis. Without prompt treatment, the ischemia typically progresses to infarction, with dire consequences. Embolic occlusion accounts for 25% of all cases of acute mesenteric ischemia, and nearly all of these are cardiogenic in origin. Atrial fibrillation is a common source of emboli; patients with atrial fibrillation who develop acute abdominal pain should be considered at high risk for acute mesenteric ischemia. Acute mesenteric insufficiency is caused by in situ thrombosis of a pre-existing stenosis in the superior mesenteric artery (SMA) in 65% of cases. Nonocclusive mesenteric ischemia accounts for the remaining 10%. Diagnostic tests are often performed in stable patients to confirm the diagnosis and locate the source of mesenteric ischemia. Both invasive arteriography and computed tomography (CT) have been used to study the mesenteric vessels. In the present case, CT demonstrates an embolic occlusion of the SMA at its orifice. Further tests such as mesenteric arteriography or a biliary excretion, eg, HIDA, scan are unnecessary. The patient should undergo exploratory laparotomy without delay: time to reestablishing SMA flow is the most important determinant of outcome and the only one that can be influenced by the surgeon. Anticoagulation and serial abdominal examination will not treat the problem. The onset of peritonitis is associated with a high mortality rate. Lytic therapy may be used to buy time if operation is delayed, but the agent should be delivered through a catheter that is placed directly into the SMA. Administration of systemic urokinase in a patient who will need exploratory laparotomy is contraindicated because of the severe risk of exsanguinating hemorrhage. REFERENCES 1.Chang JB, Stein TA: Mesenteric ischemia: acute and chronic. Ann Vasc Surg 17:323-328, 2003 2.Ritz JP, Germer CT, Buhr HJ: Prognostic factors for mesenteric infarction: multivariate analysis of 187 patients with regard to patient age. Ann Vasc Surg 19:328-334, 2005 page 490 American College of Surgeons SESAP 13 Category 5 Item 18 Five days after an uncomplicated right carotid endarterectomy, a 69-year-old man arrives in the emergency department after the sudden onset of a severe right-sided headache. He is hemodynamically normal and neurologically intact. The next step in management should be (A) administration of intravenous heparin (B) carotid duplex ultrasonography (C) cerebral imaging study (D) carotid arteriogram (E) immediate transport to the operating room for carotid re-exploration page 491 American College of Surgeons SESAP 13 Critique for Category 5 Item 18 Hyperperfusion of the brain is a rare but potentially dangerous complication of carotid endarterectomy (CEA). It has also been described after carotid artery angioplasty and stenting. The cerebral hyperperfusion syndrome is often heralded by severe ipsilateral headache and can progress to seizure activity and cerebral hemorrhage. The overall prevalence of the syndrome after CEA is 0.4% to 7.7%, depending on the definitions used. Hyperperfusion is believed to represent increased cerebral blood flow in a territory with disturbed autoregulation. Proposed risk factors include correction of a very high grade carotid stenosis (especially when the contralateral carotid artery is occluded), previous stroke, poor collateral blood supply, and uncontrolled hypertension. Documentation of increased cerebral blood flow to more than twice baseline may help to identify patients at risk for the syndrome. Cerebral hyperperfusion should be suspected in any patient who develops a severe ipsilateral headache after carotid endarterectomy. A cerebral imaging study should be obtained to detect edema or hemorrhage. Head computed tomographic (CT) scan can be used to diagnose cerebral hemorrhage, but magnetic resonance imaging (MRI) with gadolinium enhancement is particularly sensitive to detect subtle changes associated with hyperperfusion. Because of the risk of cerebral hemorrhage, anticoagulants should not be administered, and antiplatelet agents should be stopped. Hypertension should be carefully controlled. Because the carotid repair is not likely to be the cause of the headache, carotid imaging or operative exploration should not be the next step in this case. REFERENCES 1.Coutts SB, Hill MD, Hu WY: Hyperperfusion syndrome: toward a stricter definition. Neurosurgery 53:1053-1060, 2003 2.Kresowik TF: Complications following carotid endarterectomy and perioperative management, in Rutherford RB (ed): Vascular Surgery, ed 6. Philadelphia, Elsevier Saunders, 2005, pp 2093-2109 page 492 5-cm infrarenal abdominal aortic aneurysm is associated with (A) reduced 30-day morbidity and mortality (B) longer recovery times due to persistent endoleaks (C) lower incidence of colon ischemia (D) fewer re-interventions (E) lower treatment costs page 493 . endovascular repair of a 6.American College of Surgeons SESAP 13 Category 5 Item 19 Compared with open repair. REFERENCES 1. The two trials reported different survival trends over the mid-term. Buth J. Cambria RP.Greenhalgh RM.Prinssen M. 2004 3. the incidence of colon ischemia was similar to that after open repair. et al: Two-year outcomes after conventional or endovascular repair of abdominal aortic aneurysms.Blankensteijn JD.Michaels JA. whereas the perioperative survival advantage in the endograft group was not sustained after the first postoperative year in the DREAM trial. The additional costs of ongoing surveillance to detect graft complications add significantly to the overall expense of endograft repair. However. However. 2005 4. even taking into account the shorter hospital stay and reduced morbidity with endografts. and this complication was associated with high mortality. this did not affect the overall recovery rate. Drury D. patients who underwent endovascular repair had significantly lower morbidity and mortality at 30 days compared with cohorts who underwent open repair. which was faster in the endograft group. the 3% absolute mortality benefit for patients receiving endografts persisted for at least 2 years. In a recent study of more than 700 endovascular aneurysm repairs. 2004 page 494 . Prinssen M. In the EVAR study. J Vasc Surg 41:382-389. These expenses translate into an overall higher cost of endovascular repair compared with open repair. Two randomized studies. van der Ham AC. the EVAR and DREAM trials. Chuang SK. Verhoeven EL. Stoner MC. Powell JT. N Engl J Med 351:1607-1618. However. Thomas SM: Cost-effectiveness of endovascular abdominal aortic aneurysm repair. Patients in the endovascular group had a higher number of re-interventions to treat graft thromosis or endoleaks. As more experience has been gained with endografts. The propensity for endografts to develop complications years after implantation sparks concern that longer term survival may be worse after endovascular repair. In both. Lancet 364:843-848. small bowel ischemia occurred much more commonly after endografts. Cuypers PW. Br J Surg 92:960-967. Kwong GP. the late outcome will not be known until 5-year results are available. The high cost of endografts remains one of the main disadvantages of endovascular repair. compared open versus endovascular repair of AAA. A recent study reported that the cost effectiveness of endovascular repair in patients fit for open repair was not within the acceptable range for a new technique. 2005 2. Brown LC. N Engl J Med 352:2398-2405. 2005 5. surgeons have become better at assessing the benefits and risks of this technique. et al: Early outcomes of endovascular versus open abdominal aortic aneurysm repair in the National Surgical Quality Improvement Program-Private Sector (NSQIP-PS). 30-day operative mortality results: randomised controlled trial. et al: A randomized trial comparing conventional and endovascular repair of abdominal aortic aneurysms.American College of Surgeons SESAP 13 Critique for Category 5 Item 19 Endovascular repair of abdominal aortic aneurysms (AAA) has become routine in most centers. de Jong SE. et al: Comparison of endovascular aneurysm repair with open repair in patients with abdominal aortic aneurysm (EVAR trial 1).Hua HT. Ischemic complications remain a significant problem for patients undergoing endovascular aneurysm repair. The ulcer heals after 6 weeks of compression therapy. The deep venous system is patent. The best long-term management option is (A) continued compression therapy with a fitted stocking (B) ligation of the saphenofemoral junction and saphenous vein stripping (C) subfascial ligation of perforating veins (D) excision of ulcer scar and split-thickness skin graft (E) axillary vein valve transfer page 495 .American College of Surgeons SESAP 13 Category 5 Item 20 A 42-year-old woman presents with a recurrent stasis ulcer on the medial ankle. Venous duplex ultrasonography demonstrates complete valvular incompetence of the ipsilateral saphenous vein. and the valves are competent at all levels. or both. Saphenous reflux is the only abnormality in up to 25% of patients with venous stasis ulcers. Taylor M. Bell PR. In these cases. Br J Surg 86:755-759. 1998 page 496 . et al: Role of superficial venous surgery in the treatment of venous ulceration.Bello M. Vein stripping is the best option in this case.American College of Surgeons SESAP 13 Critique for Category 5 Item 20 Venous stasis ulcers are a significant public health problem in the United States. Subfascial ligation is unnecessary in patients without valvular incompetence of the deep or perforating veins. Thrush AJ. Br J Surg 85:781-784. Deacon J. but it is not appropriate to treat superficial venous insufficiency. et al: Role of saphenous vein surgery in the treatment of venous ulceration. Bell PR.Scriven JM. long-term recurrence of venous insufficiency is more likely if the saphenofemoral junction is ligated without stripping the vein. et al: Surgical correction of isolated superficial venous reflux reduces long-term recurrence rate in chronic venous leg ulcers. but long-term results are not yet known. Hartshorne T. REFERENCES 1. Ligation of the saphenofemoral junction has been associated with ulcer healing.Barwell JR. and the procedure can be performed under local anesthesia. Transfer of an axillary vein segment containing a competent valve may be a good option for a patient with deep venous insufficiency. Although saphenous vein stripping is unnecessary to achieve initial ulcer healing. Hartshorne T. Prevalence data suggest that venous stasis ulcers occur in approximately 1% of the adult population. 1999 3. the deep venous system. Endoluminal saphenous vein ablation using laser or radiofrequency techniques may be an equally good option. and one third remain unhealed. Stasis ulcers result from prolonged venous hypertension from valvular insufficiency in the saphenous venous system. saphenous phlebectomy improves venous function and heals venous ulcers better than compression therapy alone. Ghauri AS. 2000 2. Eur J Vasc Endovasc Surg 20:363-368. Scriven M. He is otherwise in good health and has no contraindications to anticoagulation. Magnetic resonance venography confirms thrombotic occlusion of the left common and external iliac veins. This patient should receive (A) unfractionated heparin only (B) low molecular weight heparin (C) direct thrombin inhibitor (D) catheter-directed thrombolysis (E) systemic thrombolysis page 497 .American College of Surgeons SESAP 13 Category 5 Item 21 A 53-year-old man presents with a 2-day history of pain and swelling in the left leg and thigh. but was largely abandoned after prospective trials failed to show a benefit in reduction of PE or death compared with heparin alone. This technique results in faster clot dissolution with fewer bleeding complications compared with systemic lysis. Meissner MH. and fewer postthrombotic symptoms than patients who were treated with heparin alone. Consequently.Mewissen MW. Wulu JT. Ann Surg 233:752-760. thrombectomy was often incomplete. Cynamon J. The main determinants of postthrombotic syndrome are incomplete venous recanalization and loss of normal venous valvular function. J Vasc Surg 32:130-137.AbuRahma AF. with a major bleeding rate of 11% and a PE rate of 1%. et al: Catheter-directed thrombolysis for ileofemoral deep venous thrombosis improves health-related quality of life. New mechanical endovascular devices have improved the speed and success rate of clot dissolution in patients with iliofemoral DVT. Ng HK: Iliofemoral deep vein thrombosis: conventional therapy versus lysis and percutaneous transluminal angioplasty and stenting. Although this regimen is effective in reducing the risk of pulmonary embolus (PE) and recurrent DVT. REFERENCES 1. oral anticoagulation should be given for at least 6 months. and early recurrence of the thrombosis was commonplace. patients treated with lytic therapy reported better overall physical functioning. Seabrook GR. Once thrombus has been cleared. Radiology 211:3949. Catheter-directed lytic therapy is used to introduce the lytic agent directly into the clot. 1999 page 498 . Multicenter trials have shown an overall success rate in over 80% of treated patients.Comerota AJ. Haughton S. Perkins SE. Surgical thrombectomy showed some initial promise. patients with iliofemoral DVT are at risk for postthrombotic syndrome. which prompted attempts to restore luminal patency in patients with iliofemoral DVT. The continuing problem of severe symptoms in patients with iliofemoral DVT has prompted a renewed interest in lytic therapy to clear the venous thrombus using modern endovascular techniques. 2000 3. thrombectomy is rarely performed except in highly symptomatic patients due to phlegmasia.American College of Surgeons SESAP 13 Critique for Category 5 Item 21 Conventional therapy for deep venous thrombosis (DVT) has been systemic heparin followed by oral anticoagulation for 3 to 6 months. 2001 2. however. less health distress. In a study in which a self-administered health-related quality of life questionnaire was used. et al: Catheter-directed thrombolysis for lower extremity deep venous thrombosis: report of a national multicenter registry. Systemic lytic therapy gained popularity in the 1970s. In addition to improved clot dissolution. Throm RC. Mathias SD. catheter-based lytic therapy appears to improve long-term outcome compared with heparin. Venography demonstrates a stenosis of the left common iliac vein in the area underlying the right common iliac artery.American College of Surgeons SESAP 13 Category 5 Item 22 A 32-year-old man presents with chronic left leg pain and edema. followed by long-term anticoagulation (B) systemic administration of a lytic agent (C) catheter-based lytic therapy (D) placement of a self-expanding stent (E) surgical resection of the obstructing lesion page 499 . The most appropriate treatment would be (A) immediate administration of heparin. Duplex ultrasonography demonstrates continuous flow in the external iliac vein that is suggestive of proximal vein obstruction. catheter-directed lysis has been used to treat venous thrombosis before stent placement. a condition known as May-Thurner syndrome. Patients may present with pain and edema due to venous hypertension before the onset of DVT. Warner MT. with good results.Lamont JP. REFERENCES 1. Open surgical repair is unnecessary because of the superior results obtained with the less invasive endovascular option. Long-term results of chronic anticoagulation are inferior to endovascular therapy. J Vasc Interv Radiol 16:815-822. et al: Prospective evaluation of endoluminal venous stents in the treatment of the May-Thurner syndrome. Lee YS. Pearl GJ. However. Ann Vasc Surg 16:61-64. with 2-year primary patency rates > 90%.Kwak HS. This anatomic arrangement may lead to chronic compression of the vein by the artery. 2002 page 500 . Jin GY. et al: Stents in common iliac vein obstruction with acute ipsilateral deep venous thrombosis: early and late results. lytic therapy is not necessary. Although most cases are asymptomatic. chronic narrowing of the vein is considered a risk factor for deep venous thrombosis (DVT). as in the present scenario. and the iliac vein stenosis represents a significant risk for future venous thrombosis after the anticoagulation is discontinued. Patetsios P. 2005 2. In the absence of DVT. the left common iliac vein is crossed anteriorly by the right common iliac artery.American College of Surgeons SESAP 13 Critique for Category 5 Item 22 In humans. Endovascular therapy with metallic stents is effective in relieving the external compression. Han YM. American College of Surgeons SESAP 13 Category 5 Item 23 Endovascular repair of an abdominal aortic aneurysm has improved outcome over open repair in all of the following EXCEPT (A) graft complications (B) mortality (C) cardiac complications (D) pulmonary complications (E) length of hospital stay page 501 . Decreases in length of hospital stay. pulmonary and cardiac complications. and overall mortality have all been documented. An endograft is composed of the fabric graft and metallic skeleton. Although degeneration of the fabric prosthesis is a factor for both techniques. Rutherford RB: Update on open repair of abdominal aortic aneurysms: the challenges for endovascular repair.Towne JB: Endovascular treatment of abdominal aortic aneurysms.Ricotta JJ: What’s new in vascular surgery. However. J Am Coll Surg 199:946-960.Krupski WC. 2004 2. numerous reports have highlighted the efficacy of endovascular repair over open repair of abdominal aortic aneurysm. data on the durability of these grafts once implanted are limited. With close proximity of the fabric (usually Dacron or PTFE) and metal cage and the sustained wear and tear on the fabric (the heart beats approximately 32 million times per year). the metallic stent in the endograft provides an added area of friction and possible graft-related complications. Am J Surg 189:140-149. 2004 3. graft complications are real.American College of Surgeons SESAP 13 Critique for Category 5 Item 23 Since the initial experience with intraluminal graft implantation in 1991. J Am Coll Surg 198:600-625. REFERENCES 1. 2005 page 502 . Endoleaks also occur when complete exclusion of the native vessel is not achieved. a prosthetic graft will provide a higher patency rate than a radiocephalic fistula page 503 .American College of Surgeons SESAP 13 Category 5 Item 24 Which of the following statements about arteriovenous fistulas (AVFs) for hemodialysis access in patients with end-stage renal disease is TRUE? (A) AVFs should be placed immediately after patients have started dialysis (B) Over 50% of such patients are being dialyzed through AVFs (C) The radiocephalic fistula can be done in over 50% of all patients (D) If an AVF fails to mature properly. secondary operations are rarely successful (E) For a patient with small vessels. A forearm loop graft with prosthetic material. while arteriovenous grafts may last a year or two. Burgmans JP. with the objective of placing AVFs in dialysis patients before they begin receiving dialysis. McVicar JP. will be superior in patients with small vessels. 2005 4. Venous catheters last only a few months at most.Tonnessen BH. Chin AI. Yo TI. et al: Outcomes of upper arm arteriovenous fistulas for maintenance hemodialysis access. Schanzer A. REFERENCES 1. some 240. which has a lower patency rate than a Brescia-Cimino shunt under ideal circumstances.org/ 2. an arteriovenous fistula (AVF) is the best method of access. It takes 3 to 4 months for a fistula to mature. 2005 page 504 .Fitzgerald JT.American College of Surgeons SESAP 13 Critique for Category 5 Item 24 In any given year. J Vasc Surg 42:481-486.fistulafirst. et al: Autogenous radial-cephalic or prosthetic brachialantecubital forearm loop AVF in patients with compromised vessels? A randomized. In prosthetic grafts. At present.000 patients with end-stage renal disease are being treated with maintenance hemodialysis. especially in diabetics. just as with nongraft fistulas. The radiocephalic arteriovenous fistula (Brescia-Cimino shunt) has been the most desired method of acquiring access at the start of dialysis. The Center for Medicare and Medicaid Services (CMS) has announced a Fistula First movement. More than half of patients cannot have this procedure because their vessels are too small or because the cephalic vein is occluded. and often longer. re-operation will often salvage a clotted fistula. Arch Surg 139:201-208. Available at: http://www. Hop WC. and a revision may be necessary.Rooijens PP.Center for Medicaid and Medicare Services. Because it may last for several years. Money SR: Embracing the fistula first national vascular access improvement initiative. the failure rate is high. Achieving a higher rate of first-use AVF requires that patients be identified 6 to 12 months prior to requiring dialysis and referred for creation of a fistula. fewer than 40% of patients receiving hemodialysis have AVFs. Each requires a functioning access to the circulation. J Vasc Surg 42:585-586. multicenter study of the patency of primary hemodialysis access. The Fistula First Breakthrough Initiative. 2004 3. In such patients. He has intermittent tingling in the ring and small fingers. X-rays show soft tissue swelling and no obvious fracture. hyperbaric oxygen (D) carpal tunnel release (E) volar and dorsal forearm fasciotomy page 505 . 16 mm Hg for the dorsal compartment. He is in moderate discomfort. and 18 mm Hg for the mobile wad. Compartment pressures are 20 mm Hg for the volar forearm compartment. The next step should be (A) analgesics and antibiotics with discharge home (B) hospital admission. Two-point discrimination is slightly diminished in the ring and small fingers.American College of Surgeons SESAP 13 Category 5 Item 25 A 32-year-old man presents with swelling of the forearm. serial examinations (C) hospital admission. Hand compartments are soft. as shown. Am J Surg 188:240-245. 2004 page 506 . Emerg Med J 20:565-566. REFERENCES 1.Fields CE. the patient should not be discharged home. and mobile wad (includes brachioradialis. The carpal canal. posterior (dorsal). If the neurologic deficit was present initially. Chaudhury M: An uncommon peripheral nerve injury after penetrating injury of the forearm: the importance of clinical examination. Sensory deficits (paresthesias or numbness) usually precede motor dysfunction. Beredjiklian PK: The significance of the three volar spaces in forearm compartment syndrome: a clinical and cadaveric correlation. Muscles and nerves are especially vulnerable to ischemia and incur irreversible damage if increased pressures are maintained. swollen tense compartments) and may be confirmed by intracompartmental tissue fluid pressures > 30 mm Hg. Schuind F: False aneurysm appearing as delayed ulnar nerve palsy after "minor" penetrating trauma in the forearm.Hargens AR. Pepe MD.American College of Surgeons SESAP 13 Critique for Category 5 Item 25 The photos document an obvious gunshot wound to the forearm. The patient is at risk for compartment syndrome. it is more likely that the peripheral nerve is injured. Latifi R. extensor carpi radialis longus. is a physiologic compartment and should be released when median nerve compression is identified. although open at both ends.Zellweger R. An important aspect in this case is documentation of presenting and subsequent neurologic examinations. 1998 2. J Hand Surg 23:1077-1081. pain with passive extension of the compartment muscles. 2001 6. This patient has signs of ulnar nerve involvement only. Hand Clin 14:371-383. and diagnosis of compartment syndrome.Chan PS. Nicol A. Gelberman RH: Acute compartment syndrome of the forearm. evaluation. Mubarak SJ: Current concepts in the pathophysiology. Hand Clin 14:391-403.Lorea P. Omoshoro-Jones J. Steinberg DR. Surgical release of the compartments is indicated when the patient’s symptoms are consistent with compartment syndrome (pain out of proportion to the injury. but may also have injury to the ulnar nerve related to direct injury or compression from local edema or blast injury. Hyperbaric oxygen is not an acceptable primary treatment for compartment syndrome. et al: An analysis of 124 surgically managed brachial artery injuries. Surg Clin North Am 82:105-114. Hess F. The forearm has three major compartments: anterior (volar). 2002 4. and extensor carpi radialis brevis). Ivatury RR: Brachial and forearm vessel injuries. 1998 5.Botte MJ. J Trauma 51:144-145. With progressive swelling and presence of a peripheral nerve deficit.Sunderamoorthy D. 1998 3. 2003 7. American College of Surgeons SESAP 13 Category 5 Item 26 A 65-year-old diabetic woman has an ulcer on her toe. as shown. Which of the following statements is TRUE? (A) Percutaneous angioplasty has no role in the management of this patient (B) The presence of palpable pulses effectively excludes the diagnosis of significant arterial occlusive disease (C) Arterial bypass will prevent amputation (D) The patency of the vessels in the foot will be greater than in the calf (E) The location and number of arterial stenoses predict primary amputation page 507 . particularly infrapopliteal. in the treatment of ischaemic diabetic foot ulcers: clinical results of a multicentric study of 221 consecutive diabetic subjects. Although useful to categorize extent of disease in patients. 2002 page 508 . 1998 3. Quarantiello A. D’Ayala M. Calia P. Despite this. Diabetes Care 21:625-630. et al: Extensive use of peripheral angioplasty. Caravaggi C. et al: Angiographic evaluation of peripheral arterial occlusive disease and its role as a prognostic determinant for major amputation in diabetic subjects with foot ulcers. Technical failures occurred in 15% of angioplasty and in 25% of bypass procedures.Faglia E. Shireman PK. 2002 2. these scores were not predictive of limb salvage or bypass graft patency.American College of Surgeons SESAP 13 Critique for Category 5 Item 26 A recent study of 104 patients with diabetes mellitus and foot ulcers who were evaluated with arteriography found evidence of arterial stenosis or occlusion in 103 patients. limb salvage was achieved in 78% of patients with 44% requiring amputation of single or multiple digits. Significant stenoses were found in patients with palpable foot pulses and normal ankle brachial indices. Stefanidis D. The most common locations of these stenoses were in the popliteal and infrapopliteal region with the vessels in the foot being relatively spared.Toursarkissian B. Nearly 25% of patients could be treated with percutaneous angioplasty to improve wound healing and 10% required arterial bypass procedures. Mantero M. J Intern Med 252:225-232. J Vasc Surg 35:494-500. Caminiti M. Subsequent studies have evaluated scoring systems assigning values to the number of stenoses in the foot as a means of predicting successful arterial bypass procedures.Faglia E. et al: Angiographic scoring of vascular occlusive disease in the diabetic foot: relevance to bypass graft patency and limb salvage. Favales F. REFERENCES 1. Which of the following statements is TRUE? (A) Anticoagulation is indicated (B) Limb loss is likely without revascularization (C) He is likely to complain of pain in his foot with walking (D) He has a decreased life expectancy (E) Antiplatelet therapy has no role in the medical management of this patient page 509 .American College of Surgeons SESAP 13 Category 5 Item 27 A 57-year-old man has the angiogram shown. a finding typical in patients with peripheral arterial disease (PAD). Such patients most often complain of claudication. Foot pain with ambulation is not a typical presentation because the amount of muscle in the calf is far greater and usually produces symptoms first. and buttock. Patients with symptomatic peripheral arterial disease have twice the risk of mortality from these atherothrombotic disease processes. and recurrent cardiac events among survivors of myocardial infarction. Anticoagulation will not improve walking distance and is not indicated. thigh. inflammation. More ominous. Moss AJ. is the association of claudication and peripheral arterial disease with stroke and myocardial infarction. Eur J Vasc Endovasc Surg 26:1-16. 2003 page 510 . REFERENCES 1. Key features of this pain include its intermittent nature with occurrence during exercise and abatement with rest.American College of Surgeons SESAP 13 Critique of Category 5 Item 27 The angiogram demonstrates occlusion of the superficial femoral artery. Revascularization is only required in one third of patients with claudication and limb loss is relatively rare.Narins CR. Marder VJ. however. Typical locations of pain depend on the location of the arterial stenosis and commonly include the calf. Symptoms occur distal to the stenosis or occlusion as oxygen demand increases with exercise but cannot be supplied. Arch Intern Med 164:440-446. et al: Relationship between intermittent claudication. 2004 2. Consensus statement.Peripheral Arterial Diseases Antiplatelet Consensus Group: Antiplatelet therapy in peripheral arterial disease. thrombosis. Antiplatelet agents are indicated in these patients to reduce cardiovascular mortality and morbidity. Zareba W. American College of Surgeons SESAP 13 Category 5 Item 28 Which of the following statements about the findings shown in this computed tomographic (CT) scan is TRUE? (A) This is the most common site of aneurysmal disease (B) Successful surgical repair eliminates any further enlargement (C) Rupture is more likely to occur than thrombosis (D) Embolization from the aneurysm is a continuous risk (E) There is no role for thrombolytic therapy page 511 . More recently. Surgical intervention requires ligation of the aneurysm and reconstruction with autogenous conduit whenever possible. Unlike AAAs. Martin J. The risk of developing symptoms is approximately 14% per year and includes the risk of complete thrombosis. Thombolytic therapy has been advocated as an adjunct in this setting to identify patent distal vessels. Br J Surg 91:174-177. Many authors recommend surgical intervention when the aneurysm is diagnosed. accounting for more than 70%. Brennan JA.Galland RB. improve small vessel flow. McWilliams RG.Dawson I. Br J Surg 89:1382-1385. Most patients present with symptoms of emoblization (blue toes) or acute limb ischemia. Abdominal aortic aneurysms (AAA) occur more frequently.American College of Surgeons SESAP 13 Critique for Category 5 Item 28 The computed tomographic (CT) scan shows aneurysms of both popliteal arteries (the left is larger than the right). Limb loss in this setting occurs in approximately 30%. and improve subsequent bypass patency. van Bockel JH: Atherosclerotic popliteal aneurysm. 1997 2. et al: Late complications after ligation and bypass for popliteal aneurysm. 2002 3. 80% patency at 5 years is commonly reported. patency of the geniculates resulting in continued aneurysm growth has been reported. endovascular exclusion of the aneurysm with in-line reconstruction with a covered stent graft has been described. Sie RB. Planning revascularization is often complicated by the embolization and thrombosis of normal caliber distal vessels that would have been suitable for bypass. but aneurysms of the popliteal artery are the most frequently occurring peripheral arterial aneurysms. This complication is best treated with exploration from a posterior incision with ligation of the patent vessels from within the aneurysm sac. Even with ligation of the proximal and distal vessel around the aneurysm. Magee TR: Management of popliteal aneurysm. Br J Surg 84:293-299. REFERENCES 1. 2004 page 512 .Kirkpatrick UJ. rupture of an aneurysm in this location is extremely rare. American College of Surgeons SESAP 13 Category 5 Item 29 Which of the following statements about the lesion shown is TRUE? (A) If the patient is otherwise healthy and asymptomatic. heart attack. and death with or without operative intervention (C) Endovascular treatment is associated with improved outcome when compared with operation (D) Successful operative or endovascular intervention eliminates the need for continued medical therapy (E) If the patient has already had a stroke. this lesion should be managed medically (B) The patient is at increased risk of stroke. operative intervention is not indicated page 513 . Mayo Clinic Trial) support endarterectomy to improve subsequent stroke-free survival for all others. carotid stenting may be comparable to endarterectomy both in terms of 30-day morbidity and durability. The only exception would be patients who have had a severely debilitating stroke when long-term survival is unlikely. or who have been deemed poor surgical candidates. CARESS. The evolution of carotid artery stenting has made the timing of intervention more difficult. regardless of the therapy offered. These trials documented a significant risk reduction for subsequent neurologic events. In these patients. Stroke 34:2767-73. Brown RD Jr. plaque embolization (despite cerebral protection devices). Patients who have already had a stroke from such a lesion may also benefit from surgical intervention. REFERENCES 1. 2004 2. Existing data accumulated from large trials (SAPPHIRE. Periprocedural stroke and death rates are similar to the event rate reported for carotid endarterectomy in asymptomatic patients. Miller DA. Antiplatelet therapy is still recommended after successful operation or stenting for its myocardial protective effects as well as to minimize the risk of stroke from the site of intervention. Data from large randomized controlled trials (North American Symptomatic Carotid Endarterectomy Trial. Asymptomatic Carotid Atherosclerosis Study).Goldstein LB: Extracranial carotid artery stenosis. Meyer FB. Veteran Affairs Cooperative Symptomatic Carotid Stenosis Trial.American College of Surgeons SESAP 13 Critique for Category 5 Item 29 The angiogram demonstrates severe stenosis of the right internal and external carotid artery. heart attack.Brott TG. Mayo Clin Proc 79:1197-1208. 2003 page 514 . Veterans Administration Cooperative Study. In three large randomized trials of carotid endarterctomy versus medical management in asymptomatic patients (Mayo Clinic. All studies have clearly documented that patients with atherosclerotic disease of the internal carotid artery are at increased risk of coronary artery disease. Numerous trials have attempted to determine when medical or surgical intervention is indicated in select subgroups of patients. with high carotid bifurcations. myocardial infarction and death were the major adverse events associated with operative intervention in addition to neurologic sequelae. Periprocedural hypotension. stroke. and death. CREST. and myocardial infarction appear to be more likely to occur in those over 80. and the American Heart Association subsequently advocated endarterectomy for good-risk patients. et al: Carotid revascularization for prevention of stroke: carotid endarterectomy and carotid artery stenting. European Carotid Surgery Trial. ARCHeR) suggest that carotid artery stenting can be accomplished with reasonable safety in patients after neck dissection or radiation therapy. Which of the following statements about this condition is TRUE? (A) The prognosis is better than for mesenteric venous thrombosis (B) Mortality is greater than 75% (C) Survival rates have improved over the last 40 years (D) Prognosis is independent of the etiology of the ischemia (E) This disease is usually diagnosed early in its course page 515 .American College of Surgeons SESAP 13 Category 5 Item 30 A 78-year-old man who has a 6-month history of weight loss and postprandial pain develops the abrupt onset of severe abdominal pain. The lateral aortogram shown is obtained. et al: Acute mesenteric ischemia: a clinical review. Levi M. Pain out of proportion to physical findings is classic for mesenteric ischemia. Edmonds HJ.4%.Schoots IG.Oldenburg WA. In most cases. and mortality has remained unchanged in the last 70 years despite application of such adjunctive measures as thrombolytic therapy. Rodenberg TJ. and venous thrombosis in patients with prothrombotic disorders.American College of Surgeons SESAP 13 Critique for Category 5 Item 30 The aortogram demonstrates no filling of the superior mesenteric or inferior mesenteric arteries. 2004 2. mesenteric ischemia may result from acute emboli from cardiac dysrhythmias. In addition to acute arterial thrombosis in the setting of chronic occlusive disease as described in this patient. Lau LL. The patient’s history and abrupt onset of recent symptoms suggest chronic atherosclerotic disease with an episode of acute thrombosis producing severe symptoms. definitive diagnosis with arteriography is delayed by a lack of clinical suspicion for the disease process. Late filling of the celiac artery is seen approximately 2 cm distal to the origin. 2004 page 516 . Patients with nonocclusive ischemia have a mortality rate of 72. Br J Surg 91:17-27. REFERENCES 1. arterial angioplasty. low flow states in patients with congestive heart failure. and stenting in addition to operative exploration. Arch Intern Med 164:1054-1062. Koffeman GI.7% and those with acute arterial thrombosis had the highest reported rate at 77. et al: Systematic review of survival after acute mesenteric ischaemia according to disease aetiology. leading to ongoing bowel ischemia and necrosis. Legemate DA. Quantitative analysis of 45 observational studies of 3692 patients documents improved survival for patients with venous thrombosis compared with arterial emboli (32% versus 54% mortality). Pertinent data include a hemoglobin of 12 g/dL and WBC count of 19. Which of the following statements about his diagnosis and management is TRUE? (A) Treatment will require ostomy formation (B) A negative upper endoscopy eliminates the need for operation (C) Graft excision and extra-anatomic reconstruction should be performed immediately (D) Systemic antibiotics should be started immediately (E) Percutaneous drainage is adequate therapy page 517 . his abdomen is diffusely tender and he is normotensive. The computed tomographic (CT) scan shown is obtained. On physical examination.800/mm3.American College of Surgeons SESAP 13 Category 5 Item 31 A 75-year-old man who had an abdominal aortic aneurysm repair 5 years previously with an aortobifemoral graft presents with malaise and generalized abdominal pain. The period of ischemia to the lower extremeties is actually less with this strategy and patient survival more likely. Most of these infections develop 5 or more years after initial operation and are from fastidious organisms such as Staphylococcus epidermidis.American College of Surgeons SESAP 13 Critique for Category 5 Item 31 Prosthetic graft infections remain the most dreaded of all complications after aortic reconstruction. Although this strategy may be possible in this patient. but ideally should be started after percutaneous sampling of the fluid collection in order to direct therapy. In such patients. followed by laparotomy and excision of all infected graft material. and hematemesis if the graft has eroded into the adjacent bowel. there is a small pocket of air (image B) adjacent to the graft. survivors remain at risk for aortic stump blowout. This may greatly simplify both the debridement and the arterial reconstruction. but might dislodge an already tenuous clot within the lumen of the bowel. Smaller case series have reported antibiotic-impregnated grafts. Percutaneous drainage alone would not be adequate treatment. occurring in 1% to 6% of cases. such as whether the graft in the region of the groins is also infected. complete graft excision with aggressive retroperitoneal debridement and extra-anatomic reconstruction has been recommended. In addition. The low virulence of these organisms has led many to suggest that long-term antibiotics in conjunction with aggressive debridement with in situ reconstruction may be possible. Antibiotic therapy will be required. For normotensive patients who are not actively bleeding. arterial homografts. the presence of air on the CT scan is worrisome for more aggressive infection with polymicrobial or gram-negative organisms. but the serial images clearly demonstrate that the air is extraluminal. leukocytosis with melena. Characteristic computed tomographic (CT) findings include the presence of perigraft air or fluid as well as soft tissue attenuation between the graft and the aortic wall after the immediate perioperative period. Aortoenteric fistulas most frequently occur at the site of the proximal anastomosis of the graft to the aorta and involve the third portion of the duodenum in precisely this location. This strategy has been associated with improved patient survival. A truly negative endoscopic examination (all bowel including the 4 th portion of the duodenum is fully examined and appears normal) is rarely obtained. staged procedures may also be possible. Regardless of the procedure performed. page 518 . Extra-anatomic bypass may be performed first after appropriate antibiotics have been administered and the patient’s condition has stabilized. and autogenous vein grafts harvested from the patient’s own deep femoral system of veins to be acceptable conduits in this setting. It is best performed in the operating room with the surgeon in attendance and ready to operate if needed. Ostomy formation is not necessary in these cases. Characteristic signs and symptoms include generalized malaise. Mortality rates remain at 30% in the immediate postoperative period.These findings are diagnositic of graft infection. Upper endoscopy may confirm this diagnosis. Classically. thrombosis of the extra-anatomoic bypass. This patient’s computed tomographic (CT) scan is remarkable for a large fluid collection in the psoas muscle (image C) and for a fluid collection in the vicinity of the aortic graft (image A). A recent report documented a preoperative falsenegative rate of 47% for endoscopy in patients with aortoenteric fistula diagnosed at laparotomy. Small bowel fistulas in this setting may be closed primarily and excluded from the site of aortic repair with a wrap of omentum or well-vascularized soft tissue. and amputation of the extremities. The bowel appears to lie directly on the graft. fever. a radionuclide-labeled WBC scan may provide additional useful information. 2000 3. et al: Secondary aortoenteric fistula: contemporary outcome with use of extraanatomic bypass and infected graft excision. Calligaro KD. 1995 2. et al: Aortic prosthetic graft infections: radiologic manifestations and implications for management. Jicha DL.Swain TW III. Radiographics 20:977-993. Ehrenfeld WK.Orton DF.Kuestner LM. Saigh JA. LeVeen RF. Vasc Endovasc Surg 38:75-82.American College of Surgeons SESAP 13 REFERENCES 1. J Vasc Surg 21:184-195. Dougherty MD: Management of infected aortic prosthetic grafts. 2004 page 519 . Reilly LM. Culp WC. tributaries. and vein size will not change management (B) Based on its location. for several months. this ulcer is arterial in etiology (C) This is likely the result of a duplication of the greater saphenous vein (D) Therapy should be directed at eliminating venous hypertension (E) Greater saphenous vein stripping will be required to achieve wound healing page 520 . above her medial malleolus.American College of Surgeons SESAP 13 Category 5 Item 32 A 65-year-old woman with a long history of varicose veins has had the ulcer shown. Which of the following statements about her condition is TRUE? (A) Preoperative evaluation to identify sources of venous reflux. Reyna JJ. 1990 2. Arcelus JI. Vascular laboratory testing to rule out deep venous thrombosis should precede any consideration for removal of the superficial venous system At the same time. Dermatol Surg 25:116-120. Harris KA: Management of venous stasis ulcer: long-term follow-up. should be instituted to reduce venous hypertension while a diagnosis is pursued. Granulation tissue is present and there are small areas of accumulated fibrin throughout the ulcer. 1999 page 521 . It is not associated with the development of venous ulceration. Reflux along the entire length of the greater saphenous vein remains an indication for complete excision or ablation.Jamieson WG. Can J Surg 33:222-223. In contrast.American College of Surgeons SESAP 13 Critique for Category 5 Item 32 The ulcer shown is typical in appearance and location for a venous stasis ulcer. These ulcers result from venous hypertension in the subcutaneous space and most commonly occur in a typical “gaiter” distribution at the level of the malleoli. as well as incompetent valves in the greater saphenous vein. Caprini JA. et al: Evaluation of therapeutic compression stockings in the treatment of chronic venous insufficiency. such as an Unna boot or Profore dressing. arterial ulceration most commonly occurs more distal on the dorsum of the foot or at the tips of the toes. Duplication of the greater saphenous vein may occur in up to one third of normal persons at some point along the length of the vein. Common causes of venous hypertension include incompetent perforating veins between the superficial and deep systems. as shown here. Conservative measures to reduce venous hypertension including compression dressings. the greater saphenofemoral junction can be evaluated for the presence of reflux. DeRose G. Elimination of the proximal segment and its tributaries may be all that is required to alleviate venous hypertension and promote wound healing.Motykie GD. REFERENCES 1. American College of Surgeons SESAP 13 Category 5 Item 33 Which of the following statements about common iliac artery aneurysms is TRUE? (A) Surgical repair is not recommended until it is 5 cm (B) Endovascular repair is not recommended (C) They are usually infectious in etiology (D) They frequently rupture when < 3 cm (E) They are most often associated with other aneurysms page 522 . Endovascular treatment options do exist for these aneurysms both in conjunction with repair of an aortic aneurysm or individually.1% to 1. World J Surg 28:797-800. Iliac artery aneurysms are found in 10% to 20% of patients with abdominal aortic aneurysms (AAA). Marfan’s. rupture and mortality rates are high. Matsubara K. Akiyoshi T.5 cm cannot be obtained proximal to its origin to decrease flow to the aneurysm sac. 2004 2.Santilli SM. 2000 page 523 . Lee ES: Expansion rates and outcomes for iliac artery aneurysms. REFERENCES 1.Matsumoto K. cystic necrosis. Most studies of expansion rates and rupture are based on autopsy series In one retrospectively reviewed series. Techniques may include placement of a covered stent and endovascular coiling of an internal iliac artery if a sufficient seal zone of 1. or nonspecific inflammatory conditions. Wernsing SE. et al: Surgical and endovascular procedures for treating isolated iliac artery aneurysms: ten year experience. Because of the difficulty in palpating these aneurysms and their asymptomatic nature. fibrodysplasia. trauma. Watada S. Preservation of the contralateral internal iliac flow and collaterals is important to preserve flow to the pelvis. and no deaths were attributable to aneurysmal disease.American College of Surgeons SESAP 13 Critique for Category 5 Item 33 Isolated common iliac artery aneurysms occur in only 0. medial necrosis.9% of all patients with aneurysmal disease. repair is not recommended for aneurysms < 3 cm. J Vasc Surg 31:114-121. no significant rate of expansion was seen over 4 years in aneurysms < 3 cm. Currently. Iliac artery aneurysms are even more rare due to atypical causes such as infection. American College of Surgeons SESAP 13 Category 5 Item 34 Development of the lower extremity postthrombotic syndrome (stasis. pigmentation. edema. ulceration) (A) is most likely in patients with multi-segment thrombosis (B) is effectively prevented by a 6-month course of low molecular-weight heparin (C) occurs in15% to 20% of all patients with deep vein thrombophlebitis (D) is uncommon after isolated calf vein thrombophlebitis (E) is due entirely to valvular insufficiency page 524 . Janssen MC. the principal causes of the postthrombotic syndrome. analgesics. 2005 page 525 . et al: The development of postthrombotic syndrome in relationship to venous reflux and calf muscle pump dysfunction at 2 years after the onset of deep venous thrombosis. the larger and more proximal the affected vein(s). Strandness DE Jr: Relationship between changes in the deep venous system and the development of the postthrombotic syndrome after an acute episode of lower limb deep vein thrombosis: a one. In addition. The most common inciting factor is a recurrent episode of DVT after resolution of the initial insult. Janssen MC. The immediate goal is to prevent pulmonary embolism. van Asten WN. J Vasc Surg 21:307-312. Anticoagulation is traditionally initiated with heparin (conventional or fractionated) with subsequent conversion to warfarin.Yamaki T. Manzo RA.Haenen JH. elastic compression. Clin Sci 93:7-12. anticoagulants can promote re-canalization of the involved veins. resulting in outflow obstruction and venous hypertension.Saarinen JP. but in general. the longer the therapy. Cameron JD. The mainstays of initial therapy are bedrest. Zeitlin R. the incidence after calf vein involvement is as high as11%. Domonyi K. J Vasc Surg 36:959-964. Wollersheim H. J Vasc Surg 40:1184-1189. Haenen JH. J Am Coll Surg 201:231-238. the occurrence of the syndrome. is an equally important etiologic agent. The third component contributing to chronic venous insufficiency is an ineffective calf muscular pump function.Johnson BF. If the condition is not resolved within several months.to six-year follow-up. 1997 4. McGrath BP: Five-year outcome study of deep vein thrombosis in the lower limbs. Wollersheim H. and anticoagulation. but does not completely prevent. 2002 3. Riha AZ. Permanent obstruction of one or more venous segments. 2004 2. Treatment may reduce. 2002 6. 40% to 60% of patients will develop clinically significant signs and symptoms of the syndrome—chronic leg edema and pain. Although the syndrome is more common after phlebitis of the ileofemoral system. Duration of therapy is somewhat empirical. Nozaki M: Patterns of venous insufficiency after an acute deep vein thrombosis. J Vasc Surg 35:1184-1189. Van't Hof MA. and ulceration. Bergelin RO. Obesity and inherited coagulopathies also place patients at greater risk. with possible avoidance of obstruction and valvular incompetence. dermatitis.American College of Surgeons SESAP 13 Critique for Category 5 Item 34 The postthrombotic syndrome is a highly morbid late complication of deep vein thrombophlebitis (DVT) of the lower extremities. 1995 5. et al: Clinical and haemodynamic sequelae of deep venous thrombosis: retrospective evaluation after 7-13 years. valvular damage is permanent and venous reflux results. The pathophysiology is related to acute dilatation of the major vein beyond the ability of the valve to coapt. REFERENCES 1. Salenius JP: Postthrombotic syndrome after isolated calf deep venous thrombosis: the role of popliteal reflux.Asbeutah AM. Within 5 years of the sentinel episode. After operative embolectomy. the next step in this patient’s evaluation should be (A) magnetic resonance angiogram of the subclavian arteries (B) duplex examination of the carotid arteries (C) echocardiogram (D) cardiac catheterization (E) Adson’s maneuver page 526 .American College of Surgeons SESAP 13 Category 5 Item 35 A 77-year-old woman presents with a cold left hand. Distinguishing between the two is often difficult. Mosby. 2004. ed 6. though duplex evaluation would be a better initial screening test. embolism from an aortic plaque. Quiñones-Baldrich WJ: Acute arterial and graft occlusion. with either valvular heart disease or atrial arrhythmia accounting for most.American College of Surgeons SESAP 13 Critique for Category 5 Item 35 Acute extremity ischemia can be caused by either thrombosis or embolism. Other potential etiologies include aneurysms of the upper extremity arteries with subsequent mural thrombus and embolism. ed 8. Cardiac catheterization was formerly used to evaluate for cardiac structural abnormalities. Echocardiography is an excellent test to search for valvular abnormalities and also to evaluate for thrombosis in any of the cardiac chambers. 2002.Pfeiffer RB III. O’Mara CS: Peripheral arterial embolus. REFERENCES 1. but a history of acute onset that can be pinpointed exactly in time is more consistent with an embolic phenomenon. WB Saunders. Philadelphia. 697-718 2. Peripheral emboli most commonly arise from the heart. but echocardiography has supplanted catheterization for evaluation of patients with suspected embolism.Angle N. Adson’s maneuver is used to detect thoracic outlet obstruction. a duplex examination of the carotid arteries would not be indicated. Because embolism is typically not a sign of chronic arterial disease. or embolism associated with arterial instrumentation. pp 817-820 page 527 . and search for septal defects. in Cameron JL (ed): Current Surgical Therapy. in Moore WS (ed): Vascular Surgery: A Comprehensive Review. Philadelphia. Magnetic resonance angiography (MRA) of the subclavian arteries would be indicated if echocardiography is negative. a 35-year-old man presents with mild dyspnea on exertion. His temperature is 100ºF. His WBC count is 12. except for right lower lobe rales. The next diagnostic test should be (A) bilateral lower extremity duplex examination (B) contrast venography (C) ventilation/perfusion scan (D) computed tomographic (CT) angiography (E) sputum culture page 528 . His chest is clear to auscultation.400/mm3 and D-dimer level is normal. The chest x-ray shows obscuring of the right hemidiaphragm.American College of Surgeons SESAP 13 Category 5 Item 36 Three days after knee arthroscopy. Minimal knee swelling is noted at the arthroscopy site. However. For this reason. et al: Rapid D-dimer testing and pre-test clinical probability in the exclusion of deep venous thrombosis in symptomatic outpatients. et al: Rapid D-dimer test combined a clinical model for deep vein thrombosis. including operation. D-dimer levels should be low in the absence of significant VTE. Although levels are elevated during any thromboembolic event. As a result. REFERENCES 1. When combined with a low clinical suspicion in a patient with a likely alternative diagnosis. making any further diagnostic test unnecessary. and many diagnostic tests are time-consuming. or have a finite complication rate. costly. significant VTE is virtually excluded. serum D-dimer levels have been proposed as a screening tool for suspected VTE. 2001 2. documentation of suspected VTE under such circumstances requires additional diagnostic work-up. 2004 page 529 . Friera A. Validation with ultrasonography and clinical follow-up in 383 patients.Funfsinn N. deep venous thrombosis (DVT) and subsequent pulmonary embolus must be among the differential diagnoses. D-dimer is a degradation product of cross-linked fibrin. Serum D-dimer is a rapid and simple screening test that may be used to eliminate up to 80% of patients without venous thromboembolism (VTE). Caliezi C. many nonthrombotic events. Blood Coagul Fibrinolysis 12:165-170.American College of Surgeons SESAP 13 Critique for Category 5 Item 36 Whenever a patient presents with respiratory symptoms after an operation.Ruiz-Gimenez N. Caballero P. Thromb Haemost 91:1237-1246. Biasiutti FD. may cause transient elevations in D-dimer. DVT will be confirmed in only 20% to 35% of cases after evaluation and testing. the negative predictive values approach 98% to 99%. For this patient with a presumptive diagnosis of pneumonia and a normal D-dimer level. Several studies have documented that a low D-dimer level (the actual cutoff level varies depending on the assay used) has a negative predictive value of 96% to 100%. Korte W. Artieda P. Anticoagulation should be given (A) for 6 weeks (B) for 3 months (C) for 1 year (D) until symptoms resolve (E) for life page 530 .American College of Surgeons SESAP 13 Category 5 Item 37 Following a routine knee arthroscopy. A hypercoagulable evaluation is negative. a healthy 33-year-old man develops an uncomplicated femoral deep venous thrombosis. Circulation 103:2453-2460. Rhedin AS. REFERENCES 1.Agnelli G. 2001 2. et al: Comparison of 3 and 6 months of oral anticoagulant therapy after a first episode of proximal deep vein thrombosis or pulmonary embolism and comparison of 6 and 12 weeks of therapy after isolated calf deep vein thrombosis. et al: Antithrombotic therapy for venous thromboembolic disease: the Seventh ACCP Conference on Antithrombotic and Thrombolytic Therapy.Schulman S. reversible risk factor. as in this patient. Carlsson A. protein C and S levels deficiency. Duhaut P. 1995 page 531 . and known thrombophilia) or idiopathic thrombosis (no known risk factors). heart failure. N Engl J Med 345:165-169.Buller HR. Chabaud S. For patients with DVT isolated to the calf without proximal extension or pulmonary embolism. A number of thrombophilic conditions have been identified. The American College of Chest Physicians Consensus Statement recommends 3 months of treatment for a first-time DVT episode with a known transient.American College of Surgeons SESAP 13 Critique for Category 5 Item 37 The optimal duration of oral anticoagulation for an uncomplicated deep venous thrombosis (DVT) has long been debated. Am J Med 116:816-828. Longer periods of treatment (6 months to 1 year) are recommended for patients with permanent risk factors (obesity. Traditional teaching was that a 3. malignancy.Pinede L. shortening the duration of treatment to as little as a few weeks has been suggested. and factor V Leiden are the most common. Many patients in the latter group may have an undiagnosed hypercoagulable state with estimates ranging from as little as 1% to as high as 30% of patients presenting with DVT. et al: Three months versus one year of oral anticoagulant therapy for idiopathic deep venous thrombosis. Prandoni P. Several large. Lindmarker P. Sanders GD. N Engl J Med 332:1661-1665. Ninet J. immobile status. Anticardiolipin antibody syndrome.Auerbach AD. antithrombin III deficiency. Hambleton J: Cost-effectiveness of testing for hypercoagulability and effects on treatment strategies in patients with deep vein thrombosis. et al: A comparison of six weeks with six months of oral anticoagulant therapy after a first episode of venous thromboembolism. 2004 3. Because there is a defineable complication rate associated with warfarin (Coumadin). Agnelli G. Duration of Anticoagulation Trial Study Group. varicosity. 2004 4. randomized trials have compared various durations of treatment. Hyers TM. Santamaria MG. 6 weeks of treatment might be sufficient. Bagatella P. Chest 126:401S-428S. Hull RD. 2001 5.to 6-month course of therapy was required. American College of Surgeons SESAP 13 Category 5 Item 38 Please view the image. (A) An arteriovenous fistula (AVF) that usually requires transposition (superficialization) of the venous outflow (B) AVF that has patency rate equivalent to the Brescia-Cimino fistula (C) Both (D) Neither page 532 . then select the best answer. 2005 page 533 . Burgmans JP. In a study comparing the three types of fistulas. However. the brachial artery to brachial vein required transposition of the outflow vein (the brachial vein) into the subcutaneous position much more often than either of the others. Chin AI. all had excellent patency rates comparable to the forearm radiocephalic fistula. J Vasc Surg 42:481-486. while the others are upper arm arteriovenous fistulas. the upper arm arteriovenous fistula is clearly superior. probably the most commonly used. Yo TI. The picture shown in item 38 is the normal anatomy of the right antecubital fossa. et al: Outcomes of upper arm arteriovenous fistulas for maintenance hemodialysis access. McVicar JP. When patency is compared with published figures for the patency of the forearm loop prosthetic graft.Rooijens PP. 2004 2. Arch Surg 139:201-208. The picture shown in item 39 is the brachial artery to cephalic vein fistula. REFERENCES 1.Fitzgerald JT. Hop WC. A brachial artery to brachial vein fistula is shown in item 40. et al: Autogenous radial-cephalic or prosthetic brachialantecubital forearm loop AVF in patients with compromised vessels? A randomized. Schanzer A.American College of Surgeons SESAP 13 Critique for Category 5 Item 38 The illustrations depict the right antecubital fossa. multicenter study of the patency of primary hemodialysis access. (A) An arteriovenous fistula (AVF) that usually requires transposition (superficialization) of the venous outflow (B) AVF that has patency rate equivalent to the Brescia-Cimino fistula (C) Both (D) Neither page 534 .American College of Surgeons SESAP 13 Category 5 Item 39 Please view the image. then select the best answer. In a study comparing the three types of fistulas. Yo TI. J Vasc Surg 42:481-486. A brachial artery to brachial vein fistula is shown in item 40. McVicar JP. The picture shown in item 39 is the brachial artery to cephalic vein fistula.American College of Surgeons SESAP 13 Critique for Category 5 Item 39 The illustrations depict the right antecubital fossa. When patency is compared with published figures for the patency of the forearm loop prosthetic graft.Fitzgerald JT. Hop WC. 2004 2. Schanzer A. et al: Outcomes of upper arm arteriovenous fistulas for maintenance hemodialysis access. Arch Surg 139:201-208. probably the most commonly used. et al: Autogenous radial-cephalic or prosthetic brachialantecubital forearm loop AVF in patients with compromised vessels? A randomized. multicenter study of the patency of primary hemodialysis access. The picture shown in item 38 is the normal anatomy of the right antecubital fossa. Chin AI. the upper arm arteriovenous fistula is clearly superior. 2005 page 535 . while the others are upper arm arteriovenous fistulas. Burgmans JP. the brachial artery to brachial vein required transposition of the outflow vein (the brachial vein) into the subcutaneous position much more often than either of the others. However.Rooijens PP. all had excellent patency rates comparable to the forearm radiocephalic fistula. REFERENCES 1. (A) An arteriovenous fistula (AVF) that usually requires transposition (superficialization) of the venous outflow (B) AVF that has patency rate equivalent to the Brescia-Cimino fistula (C) Both (D) Neither page 536 . then select the best answer.American College of Surgeons SESAP 13 Category 5 Item 40 Please view the image. When patency is compared with published figures for the patency of the forearm loop prosthetic graft. the brachial artery to brachial vein required transposition of the outflow vein (the brachial vein) into the subcutaneous position much more often than either of the others. the upper arm arteriovenous fistula is clearly superior. In a study comparing the three types of fistulas. A brachial artery to brachial vein fistula is shown in item 40. Hop WC. 2005 page 537 . multicenter study of the patency of primary hemodialysis access. et al: Outcomes of upper arm arteriovenous fistulas for maintenance hemodialysis access. Burgmans JP.Fitzgerald JT.American College of Surgeons SESAP 13 Critique for Category 5 Item 40 The illustrations depict the right antecubital fossa. Yo TI. while the others are upper arm arteriovenous fistulas. Schanzer A. The picture shown in item 38 is the normal anatomy of the right antecubital fossa. J Vasc Surg 42:481-486. REFERENCES 1. However.Rooijens PP. Arch Surg 139:201-208. The picture shown in item 39 is the brachial artery to cephalic vein fistula. 2004 2. probably the most commonly used. Chin AI. all had excellent patency rates comparable to the forearm radiocephalic fistula. et al: Autogenous radial-cephalic or prosthetic brachialantecubital forearm loop AVF in patients with compromised vessels? A randomized. McVicar JP. Fine-needle aspiration (FNA) reveals papillary cells. At neck exploration. Which of the following gives her a more unfavorable prognosis? (A) Sex (B) Age (C) Primary tumor size (D) Tall cell variant (E) Lymphocytic thyroiditis page 538 . Total thyroidectomy is performed.American College of Surgeons SESAP 13 Category 6 Item 1 A 46-year-old woman has a palpable neck mass. she has a 4-cm tall cell variant papillary cancer with no lymphatic invasion but lymphocytic thyroiditis. Rossi R: An expanded view of risk-group definition in differentiated thyroid carcinoma. Jhiang SM: Long-term impact of initial surgical and medical therapy on papillary and follicular thyroid cancer. REFERENCES 1. The tall cell variant of papillary carcinoma itself has been shown to be more aggressive and have a worse prognosis than the typical papillary thyroid carcinoma.American College of Surgeons SESAP 13 Critique for Category 6 Item 1 A variety of evaluation and scoring systems have been used to determine the risk. Am J Surg Pathol 12:22-27. and cure rate in differentiated thyroid cancer. Thompson NW.Mazzaferri EL. Age less than 50 for a woman and under 40 for a man portends a favorable prognosis. size. recurrence rate. Surgery 104:947-953. Of all the characteristics of this tumor.Cady B. 1988 2. Although lymphatic invasion is associated with a poor prognosis. 1988 3. there is no correlation with risk of recurrence and lymphocytic thyroiditis. and evaluation of metastasis to sites other than lymph nodes in predicting recurrence rate or cure. extent of primary cancer. grade of cancer. as does tumor size less than 5 cm.Johnson TL. Am J Med 97:418-428. 1994 page 539 . Most systems use some combination of age. et al: Prognostic implications of the tall cell variant of papillary thyroid carcinoma. Lloyd RV. Beierwaltes WH. the tall cell variant would be a more unfavorable prognostic factor relative to the other choices. She has been healthy since undergoing lumpectomy for stage I breast cancer 7 years previously.American College of Surgeons SESAP 13 Category 6 Item 2 A 50-year-old woman has a 3-cm right adrenal mass discovered incidentally during an abdominal computed tomographic (CT) scan for epigastric pain. The next step should be (A) measurement of 24-hour urinary cortisol (B) measurement of 24-hour urinary cathecholamine and metabolites (C) CT-guided fine-needle aspiration (FNA) of the mass (D) determination of plasma aldosterone concentration to plasma renin activity ratio (E) operative excision page 540 . because the mass harbors metastatic disease in up to 75% of such patients. Surg Clin North Am 84:755774. Complication rates are around 4%. and breast cancer frequently metastasize to the adrenal. and the plasma aldosterone to renin activity determination helps identify the presence of an aldosteroma. but secondary metastases (19%). Lau J. Excision of solitary adrenal metastases is increasing in frequency and has been shown to prolong survival. Curr Opin Oncol 15:84-90. renal carcinoma. Its frequency can range from as low as 0. 2004 3. and pheochromocytomas (8%) can occur. primary adrenocortical malignancies (10%). FNA should be obtained when the information would change the treatment plan.Mansmann G. Although fine-needle aspiration (FNA) biopsy of adrenal masses is typically discouraged. 2004 2. Pheochromocytoma must be ruled out before performing FNA because it could trigger a potentially fatal hypertensive crisis. The 24-hour urinary cortisol determination would suggest activity related to an adrenal cortex tumor. REFERENCES 1. Lymphoma.American College of Surgeons SESAP 13 Critique for Category 6 Item 2 The clinically silent adrenal mass discovered unexpectedly during abdominal imaging is known as an incidentaloma. et al: The clinically inapparent adrenal mass: update in diagnosis and management. Rothberg M. Available screening tests include 24-hour urinary catecholamine (including metabolites) determination.4 % in patients with a history of cancer.Thompson GB. Operative excision is usually delayed until further determination of the character of the incidentaloma can be obtained. leukemia. Balk E.42% in the normal population to as high as 4. and ovarian carcinoma. which helps detect activity related to a pheochromocytoma. Most of these lesions are benign adenomas (41%). Kebebew E: Laparoscopic adrenalectomy for malignancy. lung. 2003 page 541 . it may be considered in patients with a history of primary extra-adrenal cancer. Other cancers with a predilection for the adrenal include melanoma.Sturgeon C. myelolipomas (9%). Young WF Jr: Adrenal incidentaloma. Endocr Rev 25:309-340. serum electrolytes. BUN.American College of Surgeons SESAP 13 Category 6 Item 3 A 37-year-old hypertensive man has a left-sided 3. an overnight 1-mg dexamethasone suppression test. and creatinine have all been within normal limits.4-cm adrenal mass. The next diagnostic study should be (A) high-dose dexamethasone suppression test (B) plasma catecholamine determination (C) plasma aldosterone concentration to plasma renin activity ratio determination (D) positron-emission tomography (PET) (E) magnetic resonance imaging (MRI) page 542 . Initial studies including a 24hour urine collection for free catecholamine and catecholamine metabolites. parathyroid. REFERENCES 1. and adrenal. Best Pract Res Clin Endocrinol Metab 19:277-292. metastases. et al: The clinically inapparent adrenal mass: update in diagnosis and management.Mansmann G. Positron emission tomography (PET) is most useful in identifying suspected adrenal metastases when computed tomography (CT) and magnetic resonance imaging (MRI) are equivocal. Lau J. Dunn DL. but should not preclude appropriate screening for tumor functioning.Al-Hawary MM. Clark OH: Thyroid. in Brunicardi FC. Francis IR.Lal G. Balk E. and pheochromocytomas. Andersen DK. Billiar TR. Korobkin M: Non-invasive evaluation of the incidentally detected indeterminate adrenal mass. normokalemic patients suggests that the prevalence of primary hyperaldosteronism in this population approaches 12%. MRI is useful in differentiating adenomas. et al (eds): Schwartz's Principles of Surgery. In fact. This screening modality is more effective than simply measuring the potassium level because hypokalemia may not be present early in the course of the disease. Duh QY: Clinical manifestation of aldosteronoma. Rothberg M. 2004 page 543 . New York. McGraw-Hill. 2005 3. Endocr Rev 25:309-340. Surg Clin North Am 84:887-905. An overnight 1-mg dexamethasone suppression test or 24-hour urinary cortisol determination will detect the presence of an active cortical adenoma or carcinoma. 2005. A plasma aldosterone concentration to plasma renin activity ratio (PAC/PRA) determination is useful in identifying patients with aldosteronoma. ed 8. pp 13951470 4. A 24-hour urinary catecholamine and catecholamine metabolite determination aids in identifying a pheochromocytoma. PAC/PRA determination in hypertensive.American College of Surgeons SESAP 13 Critique for Category 6 Item 3 The work-up of a hypertensive patient presenting with a mass of questionable etiology should include screening to determine whether it is a functioning tumor. 2004 2.Al Fehaily M. A previous pregnancy ended with spontaneous abortion at the end the second trimester.American College of Surgeons SESAP 13 Category 6 Item 4 A 35-year-old woman entering her second trimester of pregnancy is hypertensive and has elevated 24-hour urine catecholamines. Which of the following statements is TRUE? (A) She has pre-eclampsia and needs medical management (B) She is a candidate for alpha blockade and adrenalectomy at this time (C) She should be managed to term with alpha blockade and planned vaginal delivery (D) Alpha blockade has an increased complication rate with earlier gestational age (E) The best diagnostic imaging study during pregnancy is a metaiodobenzylguanidine (MIBG) scan page 544 . resection has been associated with good maternal and fetal outcomes.American College of Surgeons SESAP 13 Critique for Category 6 Item 4 An undiagnosed pheochromocytoma in pregnancy may result in a maternal mortality rate of 50% and perhaps a greater fetal mortality rate. Skögseid B (eds): Surgical Endocrinology. pp 247-262 page 545 . This hypertensive patient is beginning her second trimester of pregnancy. This patient is a candidate for alpha blockade and adrenalectomy at this time. Surgery 114:1148-1152. Vaginal delivery is absolutely contraindicated due to the possibility of blood pressure instability and placental abruption. Instituting alpha blockade as soon as the diagnosis has been made reduces fetal mortality regardless of gestational age.Freier DT. Lippincott Williams & Wilkins. sweating. and tachycardia may be associated with preeclampsia. the patient may be treated with alpha blockade until the fetus matures. and her elevated 24-hour urine catecholamines are consistent with the diagnosis of pheochromocytoma. Thompson N: Pheochromocytoma.Gray DK. If a pheochromocytoma is diagnosed during the first or second trimester of pregnancy. Delivery should be via caesarean section under controlled conditions. Pheochromocytoma can be confused with pre-eclampsia in this setting. 1993 2. and the evaluation of a pheochromocytoma may be missed or delayed. The next most effective screening study would be magnetic resonance imaging (MRI). in Doherty GM. Philadelphia. Urine protein is elevated in pre-eclampsia but not with pheochromocytoma. REFERENCES 1. If the diagnosis occurs in the third trimester. Thompson NW: Pheochromocytoma and pregnancy: the epitome of high risk. and an elevated 24-hour urine catecholamine secures the diagnosis of pheochromocytoma. Hypertension. 2001. which provides more detailed anatomy without the radiation exposure of computed tomographic (CT) and metaiodobenzylguanidine (MIBG) scans. has a history of previous fetal demise. resection of the mass would be associated with a recurrence rate of 5% and excellent survival (E) The most common functioning pancreatic lesion in MEN1 is an insulinoma page 546 . Which of the following statements is TRUE? (A) 24-hour urine catecholamine levels should be assessed prior to operation (B) At operation a thorough examination of the duodenum is essential (C) The presence of multiple small hepatic metastases at operation dictates a palliative procedure (D) If no other disease is seen at operation.American College of Surgeons SESAP 13 Category 6 Item 5 A 34-year-old man who is a member of a known multiple endocrine neoplasia 1 (MEN1) kindred presents with a 2-cm lesion in the head of his pancreas and elevated gastrin levels. Moley JF: The multiple endocrine neoplasia syndromes. In some series. however. REFERENCES 1. Philadelphia. 2004. other lesions may be present in the submucosal area of the duodenum as well as in regional lymph nodes. Most series advocate aggressive management of even multicentric tumors with a debulking philosophy. Thompson NW. Given any suspicious findings. mobilization of the duodenum and careful palpation should be part of the intraoperative evaluation. biochemical markers or minimal clinical symptoms recur in over 90% of patients. Recurrence is high. Doherty GM: The surgical management of MEN-1 pancreatoduodenal neuroendocrine disease. Saunders. Insulinoma is the second most common functional tumor. although patients may only exhibit part of the syndrome’s components.Hausman MS Jr. Gauger PG. Although computed tomography (CT) demonstrates a 2-cm lesion in the head of the pancreas. In this patient.Lairmore TC. The finding of multiple small hepatic metastases would not necessarily alter a surgical decision to remove as much functioning tumor as possible. pp 1071-1090 page 547 . The MEN1 trait is transmitted with 100% penetrance. Neuroendocrine tumors of the pancreas and duodenum occur in 30% to 80% of patients.American College of Surgeons SESAP 13 Critique for Category 6 Item 5 Multiple endocrine neoplasia type 1 (MEN1) syndrome is characterized by parathyroid hyperplasia. open duodenotomy or intraoperative endoscopic intubation of the duodenum with transillumination is effective in finding submucosal lesions that should be resected. and adenomas of the anterior pituitary gland. all of the components of MEN1 should be considered prior to operation. Surgery 136:1205-1211. Every effort should be made to locate and excise these tumors at operation. MEN1 patients often have tumors in multicentric locations. but is associated with a much lower recurrence rate of functional activity. ed 17. neuroendocrine tumors of the pancreas and duodenum. At the least. in Townsend CM Jr (ed): Sabiston’s Textbook of Surgery: The Biological Basis of Modern Surgical Practice. 2004 2. A 24-hour urine catecholamine determination would be unnecessary because pheochromocytoma is not part of the syndrome. Gastrinoma is the most common functional neuroendocrine tumor of the pancreas in patients with MEN1. American College of Surgeons SESAP 13 Category 6 Item 6 Which of the following statements about thyroid carcinoma is TRUE? (A) Regional lymph node metastasis is associated with a poorer prognosis for Hurthle cell carcinoma (B) Men have half the risk of developing differentiated thyroid cancer and half the risk of dying from it compared with women (C) Recurrence rates for papillary and follicular cancer are highest among patients aged 30 to 60 years (D) Patients with a goiter for > 5 years have no increased risk of developing thyroid cancer (E) Hurthle cell carcinoma occurs at a younger age than either papillary or follicular carcinoma page 548 . REFERENCES 1. Ghossein RA. An American College of Surgeons Commission on Cancer patient care evaluation study.Hundahl SA. including both the follicular and papillary types. Over the last 30 to 40 years. but have twice the risk of dying from these cancers. et al: Initial results from a prospective cohort study of 5583 cases of thyroid carcinoma treated in the United States during 1996. J Clin Oncol 19:2616-2625. Most large studies demonstrate that regional lymph node metastasis does not significantly alter prognosis or mortality in patients with well-differentiated carcinoma. US and German Thyroid Cancer Study Group.6% of patients. the median age at diagnosis of papillary carcinoma and follicular carcinoma was 44 years and 47 years. thyroid cancer mortality rates have fallen significantly.American College of Surgeons SESAP 13 Critique for Category 6 Item 6 Papillary and follicular thyroid cancers are usually referred to as well-differentiated thyroid cancer. and fewer than 400 cases of well.Mazzaferri EL.Stojadinovic A. 2001 3. J Clin Endocrinol Metab 86:1447-1463. A large cohort study of over 5000 cases in the United States recently published under the auspices of the American College of Surgeons Commission on Cancer demonstrated that a history of goiter (defined as enlargement of the thyroid gland for over 5 years prior to diagnosis) was the strongest risk factor for developing thyroid carcinoma. Hoos A. Urist MJ. particularly in the United States. Hürthle cell carcinoma represents fewer than 5% of all differentiated carcinomas of the thyroid glands. Mazzaferri E. Cancer 89:202-217. Hürthle cell carcinoma occurs in an older population at an age of 61 years. respectively. Men develop papillary or follicular cancer with about half the frequency of women. In this cohort. Kloos RT: Clinical review 128: Current approaches to primary therapy for papillary and follicular thyroid cancer.documented Hürthle cell carcinoma have been reported in the English literature since 1935. mostly due to early diagnosis and effective treatment of well-differentiated thyroid cancer. 2001 page 549 . there is evidence that lymph node metastasis in the Hürthle cell population is associated with poorer prognosis. 2000 2. et al: Hurthle cell carcinoma: a critical histopathologic appraisal. This history was noted in 14. Cady B. However. Cunningham MP. Hürthle cell or oncocytic tumors are rare thyroid neoplasms originating within the follicular cell of the thyroid. Postoperative recurrence rates are especially high during the first 2 decades of life and after age 60. American College of Surgeons SESAP 13 Category 6 Item 7 Which of the following statements about Hashimoto's thyroiditis is NOT true? (A) 24-hour uptake of radioactive iodine 123 confirms the diagnosis (B) It is associated with hyperlipidemia (C) Erythrocyte sedimentation rate (ESR) is normal (D) Ultrasound examination is hypoechogenic (E) Lymphocyte infiltration is seen on pathologic examination page 550 . particularly thymoglobulin and thyroid peroxidase. Thyrotropinsuppressing doses of levothyroxine administered for 6 months decrease the size of the goiter by 30% in some patients. and lymphadenoid goiter. 1996 2. REFERENCES 1. usually enlarged thyroid. High serum thyroid peroxidase antibodies occur in 90% of patients and high anti-thyroglobulin antibodies in 20% to 50%. Hashimoto's thyroiditis is the most common cause of hypothyroidism in the United States. Ultrasound examination demonstrates a diffuse. The disease is also referred to as chronic lymphocytic thyroiditis. lymphoid follicles with germinal centers. Hashimoto's thyroiditis usually presents as a firm nodular symmetric goiter. Because most Americans receive an adequate amount of dietary iodine each day.Pearce EN. The histologic manifestations include lymphocytic infiltration of the thyroid with B and T cells. although 10% of patients present with thyroid atrophy and thus no goiter. Other antibodies block thyrotropin receptors and thyrocytes in Hashimoto's thyroiditis and demonstrate upregulation of the group of tumor necrosis genes (Fas). Whether initiated by a viral infection or simply the manifestation of autoimmunity against intracellular thyroid epithelial proteins. Otolaryngol Clin North Am 29:541-556. 2003 page 551 .Farwell AP. Farwell AP. Some of the antibodies are complement fixing. T cells activate B cells to secrete antibodies to thyroid cells and their contents. Hashimoto's thyroiditis has an 8:1 female over male prevalence. The erythrocyte sedimentation rate is normal. Elevated thyroid antibodies are more common in white Americans than in other ethnic groups and appear to increase with age. although antibodies to other thyroid cell contents have also been elucidated. Replacement therapy with levothyroxine sodium is appropriate and often reverses the hyperlipidemia. N Engl J Med 348:2646-2655. serum thyroglobulin and 24-hour uptake of radioactive iodine 123 are not good diagnostic indicators. Because hyperthyroidism is a component in the early stages of the disease in many patients and throughout the disease in some patients. increasing the risk of atherosclerotic heart disease.American College of Surgeons SESAP 13 Critique for Category 6 Item 7 Hashimoto's thyroiditis is an autoimmune phenomenon of uncertain mechanism. Subclinical hypothyroidism is common at presentation and is often associated with hyperlipidemia. and ultimately fibrosis. Braverman LE: Inflammatory thyroid disorders. resulting in direct cytotoxicity. Braverman LE: Thyroiditis. chronic autoimmune thyroiditis. hypoechogenic. 25 dihydroxy vitamin D) (D) oral phosphate (E) oral magnesium page 552 .American College of Surgeons SESAP 13 Category 6 Item 8 On the third day after total thyroidectomy. The best treatment for this condition would be (A) intravenous calcium gluconate (B) oral calcium carbonate (C) calcitriol (1. a patient develops perioral numbness and tingling of the fingers. it should be autotransplanted. tingling of the fingers. If symptoms are severe. Fabre S. the calcium dose can be increased or oral calcitriol (1. convulsions and opisthotonos may occur. 2002.McHenry CR: Goiter and nontoxic benign thyroid conditions.25 dihydroxyvitamin D facilitates gastrointestinal absorption of calcium and mobilization of calcium from the skeleton. WB Saunders. When a parathyroid gland’s blood supply has been disrupted. Calcium carbonate is usually given in preference to other forms of calcium to bind phosphorus in the gastrointestinal tract. Kakani R. et al: Hypocalcemia following thyroid surgery: incidence and prediction of outcome. pp 1041-1048 2.) If symptoms are moderate. 1998 3. The parathyroid glands are usually supplied by both the superior and inferior thyroid arteries. Ear Nose Throat J 75:612-614. Philadelphia. and their blood supply preserved. The incidence of permanent hypoparathyroidism after thyroidectomy should be less than 2% in experienced hands. Postoperative hypocalcemia will manifest as perioral numbness. Komisar A. et al: The management of post-thyroidectomy hypocalcemia. To avoid this complication. calcium should be supplemented with oral calcium carbonate (500 to 1000 mg 3 times a day. Carnaille B. Kacker A. The parathyroids must be identified.25 dihydroxyvitamin D. REFERENCES 1.Szubin L. trembling of the masseter muscle with stimulation of the facial nerve anterior to the ear (Chvostek’s sign). Combemale F. anxiety. World J Surg 22:718-724.Pattou F.American College of Surgeons SESAP 13 Critique for Category 6 Item 8 Hypoparathyroidism after thyroidectomy is usually secondary to devascularization of the parathyroid glands and can be transient or permanent. intravenous calcium is indicated.25 to 1. If mild symptoms appear. in Bland KI (ed): The Practice of General Surgery. muscle cramps. Treatment with 1. 1996 page 553 . Giving phosphorus or magnesium will result in the excretion of calcium. When left untreated. branches of the inferior thyroid artery must be ligated close to the capsule of the thyroid gland when the thyroid is mobilized. 0. and carpopedal spasm (Trousseau’s sign).0 mg twice daily) should be prescribed. American College of Surgeons SESAP 13 Category 6 Item 9 Characteristics of adrenocortical carcinoma include all of the following EXCEPT (A) size > 6 cm in diameter (B) estrogen production in 30% of cases (C) irregular shape and irregular margins on computed tomography (CT) (D) biochemically active in two thirds of cases (E) rarely bilateral page 554 . Two years later. CT characteristics of malignancy include tumor inhomogeneity. In contrast. A mixed pattern of hormone production is seen in up to 35% of patients. J Clin Endocrinol Metab 91:2027-2037. Functioning adrenocortical neoplasms most commonly produce cortisol. and the ability to perform a complete resection is the strongest predictor of outcome. The rapid onset of Cushing’s syndrome. 2006 2. or aldosterone (2%). remains the single best indicator of malignancy.Kasperlik-Zaluska AA. 1996 page 555 . 1983 3. Evans DB: Adrenal cortical carcinoma. Makowska AM: Impact of adjuvant mitotane on the clinical course of patients with adrenocortical cancer. large adrenal adenomas (> 6 cm) are probably uncommon. Ann Intern Med 98:940-945.Dackiw AP.Copeland PM: The incidentally discovered adrenal mass. often with virilizing features. Cancer 78:1520-1521. as measured on computed tomography (CT) or magnetic resonance imaging (MRI). is characteristic.American College of Surgeons SESAP 13 Critique for Category 6 Item 9 The size of an adrenal mass. but is exceedingly rare. tumors produce androgens (20%). REFERENCES 1. Migdalska BM. 92% (105 of 114) of adrenocortical carcinomas were greater than 6 cm in diameter. irregular shape. 2001 4. Gagel RF. Two thirds of all adrenocortical carcinomas are biochemically active. Primary metachronous adrenocortical carcinoma has been reported in children and adults. Less commonly. Complete surgical resection is the only potentially curative treatment. which is clinically apparent in at least 30% of patients. World J Surg 25:914-926. Lee JE. Fassnacht M: Clinical review: Adrenocortical carcinoma: clinical update. estrogens (10%).Allolio B. although the exact frequency is unknown. In a clinical review of six published series. and irregular margins. followed by a taper of 50%/day over the next 3 days (E) a 100-mg bolus of hydrocortisone at induction of anesthesia. hemodynamic instability (B) a 100-mg bolus of hydrocortisone at induction of anesthesia.American College of Surgeons SESAP 13 Category 6 Item 10 A 66-year-old woman with long-standing rheumatoid arthritis requires emergency surgery for perforated sigmoid colon diverticulitis. prednisone. for years at a dose ranging from 10 to 20 mg/day. ie. followed by continuous infusion of hydrocortisone at a dose of 300 to 400 mg/day for 3 days page 556 . She has been receiving oral corticosteroids. 300 to 400 mg hydrocortisone on the day of operation. eg. then 2 to 3 days of hydrocortisone (150 mg/day in divided doses) (D) a 100-mg bolus of hydrocortisone at induction of anesthesia. The recommended management of perioperative corticosteroids for this patient would be (A) no systemic corticosteroids unless she demonstrates clinical signs of adrenal insufficiency. Her current daily dose is 20 mg. with divided doses of 300 to 400 mg/day (C) a 100-mg bolus of hydrocortisone with induction of anesthesia. 2001 page 557 . there is little evidence that a blunted HPA or adrenal suppression is causally related to any perioperative hemodynamic instability. There are few definitive studies to answer the question of what doses of corticosteroids are needed in these patients. Lesser doses or no supplementation will be sufficient for most patients undergoing an elective operation REFERENCES 1. The traditional stress dose (300 to 400 mg hydrocortisone/day) is excessive. Patients undergoing a major emergency operation as described in this clinical scenario should be treated with hydrocortisone at a dose of 150 mg/day in divided doses for 2 to 3 days. Hall GM: Peri-operative steroid supplementation. Buie WD: Perioperative stress dose steroids: do they make a difference? J Am Coll Surg 193:678-686. Expert opinion suggests the regimens in the table.American College of Surgeons SESAP 13 Critique for Category 6 Item 10 Patients with an intact hypothalamic-pituitary adrenal (HPA) axis undergoing major surgical procedures have increased adrenal secretion of cortisol.Brown CJ. More recent investigators have demonstrated that patients with documented HPA axis suppression can safely undergo surgical procedures without supplemental parenteral corticosteroids. However. patients who have received longterm treatment with exogenous corticosteroids have a blunted HPA axis response to surgical stress. with resumption of preoperative doses of corticosteroids when the patient is stable and/or consuming oral nutrients. A dose taper is unnecessary. Marik P: Hypothalamic-pituitary-adrenal insufficiency. Anaesthesia 53:1091-1104.Zaloga GP.Nicholson G. 1998 3. Recent studies suggest that about 150 mg of hydrocortisone is the maximum dose required. The short halflife of hydrocortisone warrants administration in 3 doses per day or by continuous infusion. 2001 2. Although case reports in the 1950s reported refractory postoperative hypotension in patients with adrenal suppression. Burrin JM. Crit Care Clin 17:25-41. and lower circulating cortisol levels. The patient complains of pain and itching at the site of the rash. The optimal treatment for this rash would be (A) corticosteroid cream (B) radical resection of the tumor (C) neck exploration (D) intravenous antibiotics (E) anticoagulation page 558 . The borders of the rash are blistered and encrusted.American College of Surgeons SESAP 13 Category 6 Item 11 A 49-year-old man has a red rash on his face with areas of central clearing. Duh Q-Y.Thompson GB. abdomen. It develops as erythematous plaques on the face. The rash is pathognomonic and is called necrolytic migratory erythema.American College of Surgeons SESAP 13 Critique for Category 6 Item 11 Glucagonomas are rare neuroendocrine tumors of the pancreas in which patients present with the four Ds: diabetes. plaques coalesce into larger areas. and occasionally mucus membranes. The rash improves after debulking or resection. 2005. deep venous thrombosis. pp 673-690 page 559 . groin. Metastatic disease is common but surgical debulking is appropriate if it can be performed safely. in Clark OH. lower extremities. J Surg Res 120:139-161. Elsevier Saunders. The central area is clear and the borders become blistered and encrusted. Chen H: Pancreatic endocrine tumors. Treatment of glucagonomas is surgical resection. Over several weeks. Philadelphia. Kebebew E (eds): Textbook of Endocrine Surgery. It occurs in 70% of patients at diagnosis and commonly predates other systemic symptoms. dermatitis. Most patients complain of a perioral rash. 2004 2. REFERENCES 1. ed 2. and depression. Young WF: Multiple endocrine neoplasia type I.Mansour JC. Work-up for patients with suspected glucagonoma should include serum markers (chromogranin A and glucagon) and abdominal imaging. American College of Surgeons SESAP 13 Category 6 Item 12 Which of the following statements about laparoscopic adrenalectomy is NOT true? (A) The lateral decubitus position is superior to supine (B) The right adrenal vein is divided at its confluence with the right renal vein (C) This procedure can be used for resection of pheochromocytomas (D) Both right and left adrenal glands can be removed laparoscopically (E) Adrenal cortical cancer should not be removed laparoscopically page 560 . Duh QY: Laparoscopic adrenalectomy—indications and technique. However. Patients need to be re-positioned for a bilateral procedure. Early ligation of the adrenal vein makes it easier to mobilize the gland. Snow and colleagues performed the first successful transabdominal laparoscopic adrenalectomy. and divided using endoscopic scissors. Sierra M. ligated with clips. Two clips are generally left on the vena cava side. Adrenocortical carcinomas are characterized by large adrenal masses with invasion of adjacent structures requiring multi-visceral resection to achieve a negative margin and prevent local recurrence and subsequent cancer-related death. these lesions are usually small and situated deep in the retroperitoneum. several studies have demonstrated the safety and advantages of laparoscopic adrenalectomy. 2003 4.Kercher KW. The anterior transabdominal approach gives a conventional view of the abdominal cavity and allows a bilateral adrenalectomy to be performed without the necessity of re-positioning the patient. Surg Oncol 12:105-123. Although some early reports suggest that selected patients with primary adrenal malignancies can undergo laparoscopic adrenalectomy with acceptable results. Matthews BD. 2005 3. Recent reports have demonstrated that laparoscopic resection of pheochromocytomas can be accomplished safely by experienced surgeons. it requires insertion of additional ports and has been reported to be associated with increased operative times.American College of Surgeons SESAP 13 Critique for Category 6 Item 12 Various open approaches have traditionally been used for adrenal tumors requiring operation.Lal G. The lateral approach is the most widely used method and provides the largest working space because gravity aids retraction of surrounding organs. The adrenals can be removed laparoscopically via a transabdominal (anterior or lateral) or retroperitoneal (lateral or posterior) approach. However. World J Surg 30:893-898. Novitsky YW. Heniford BT: Laparoscopic adrenalectomy for malignancy. the right adrenal vein is identified at its junction with the inferior vena cava (figure). Ann Surg 241:919-926. Kercher KW. et al: Long-term outcome following laparoscopic adrenalectomy for large solid adrenal cortex tumors. Since these initial reports. Park A. Am J Surg 189:405-411. 2005 2. 2006 page 561 . but may make subsequent dissection more difficult due to venous congestion. and the lateral transabdominal approach for laparoscopic adrenalectomy was popularized in 1992. there are too few patients with malignant adrenocortical carcinoma histology in these series to recommend routine use of laparoscopic adrenalectomy for patients with this disease. Ippolito G. and it has become the procedure of choice for the surgical management of many adrenal neoplasms.Cobb WS. Sing RF. making them well suited to a laparoscopic approach. Sebag F. et al: Laparoscopic curative resection of pheochromocytomas. During the procedure.Palazzo FF. REFERENCES 1. A chest x-ray is shown.American College of Surgeons SESAP 13 Category 6 Item 13 A 45-year-old woman has a sticking sensation in her throat during swallowing. During examination she experiences flushing when her arms are raised over her head. She is unable to sleep at night because of dyspnea. The most appropriate treatment for this patient would be (A) video-assisted total thyroidectomy (VAT) (B) total thyroidectomy via right anterior thoracotomy (C) total thyroidectomy via collar incision (D) radioactive iodine ablation (E) thyroxine suppression page 562 . She has a smooth large thyroid gland without nodules. McHenry CR: The clinical presentation and operative management of nodular and diffuse substernal thyroid disease. no previous conventional neck operation) and not for patients with substernal goiter.0-cm skin incision. Eur J Surg 161:725-727.Mack E: Management of patients with substernal goiters. REFERENCES 1. Use of a right thoracotomy has been described only for rare cases in which the goiter involves the posterior mediastinum and should not be used as a standard approach to substernal goiter. Although radioactive iodine can be used to control hyperthyroidism if present. Thompson NW: Rationale for the operative management of substernal goiters. Surg Clin North Am 75:377-394.Allo MD. Traditionally.American College of Surgeons SESAP 13 Critique of Category 6 Item 13 This patient has a large goiter of the thyroid gland with clinical evidence of substernal involvement. thyroid volume < 30 mL. The technique is very similar to conventional surgery.Hedayati N. Jarhult J: High long term recurrence rate after subtotal thyroidectomy for nodular goitre. and radiation thyroiditis may occur soon after administration. The ability to examine a portion of the gland in this patient along with the compressive symptoms and the x-ray image establish that this patient has a cervical goiter with intrathoracic extension and associated compressive symptoms. Multinodular goiter arising within the neck and extending into the substernal position can usually be managed with a total or subtotal thyroidectomy through a transverse collar incision (Kocher) without the need for sternotomy. 2002 3. however. Surgery 94:969-977. it rarely alleviates tracheal deviation and compression caused by a large multinodular goiter even when symptoms of toxicity are relieved.to 2. 1995 page 563 . VAT has been used in some centers. many patients can experience compressive symptoms with less than 50% of the mass within the mediastinum. the term substernal goiter has been used to refer to an enlarged thyroid gland in which more than 50% of the mass is below the thoracic inlet. The mediastinal location of substernal goiter precludes evaluation with fine-needle biopsy. Video-assisted thyroidectomy (VAT) is a gasless procedure performed under endoscopic vision through a single 1. Am Surgeon 68:245-252. Surgical resection is indicated for patients with substernal goiter with symptoms from local compression. Administration of exogenous thyroid hormone to suppress such an enlarged gland will fail to shrink the gland and may also cause thyrotoxicosis.5. Tracheal narrowing and superior vena cava (SVC) syndrome due to extrinsic compression are described complications of retrosternal goiter and are usually associated with positional dyspnea and plethora.Rojdmark J. Neither thyroxin suppression nor radioactive iodine is effective alone. 1995 4. but only with strict eligibility criteria (thyroid nodules < 35 mm. precipitating airway compromise. 1983 2. Fine-needle aspiration (FNA) of the nodule is interpreted as papillary cancer. No lymph nodes are palpalple. Management of this patient should include all of the following EXCEPT (A) total thyroidectomy (B) radioactive iodine ablation (C) suppressive thyroxine (D) modified left radical neck dissection (E) serum thyroglobulin monitoring page 564 .American College of Surgeons SESAP 13 Category 6 Item 14 A 60-year-old man has a 4-cm left thyroid nodule. et al: The results of various modalities of treatment of well differentiated thyroid carcinomas: a retrospective review of 1599 patients. et al: Indeterminate fine-needle aspiration biopsy of the thyroid: identification of subgroups at high risk for invasive carcinoma.Agrawal S: Diagnostic accuracy and role of fine needle aspiration cytology in management of thyroid nodules. J Surg Oncol 58:168-172. prophylactic modified radical neck dissection is not recommended.0 cm. Goepfert H.Tyler DS. total thyroidectomy followed by ablative iodine 131 and TSHsuppressive thyroxine is the treatment of choice. Iodine 131 ablation after thyroidectomy reduces the risk of cancer recurrence versus thyroidectomy alone. in patients with tumors > 1. In this high-risk patient.American College of Surgeons SESAP 13 Critique for Category 6 Item 14 Based on the clinical information available. Lastly. less extensive operation (lobectomy or subtotal thyroidectomy) is associated with an increased risk of cancer recurrence and death compared with total or near-total thyroidectomy.Mazzaferri EL. Most importantly. the preserved thyroid tissue will actively take up iodine 131 and prevent the effective ablation of residual or metastatic thyroid cancer. A total thyroidectomy is optimal in high-risk patients because total removal of the gland allows the patient to receive iodine 131 and suppressive thyroxine postoperatively. this patient most likely has well-differentiated thyroid cancer. The appropriate extent of surgical resection for well-differentiated thyroid cancer has long been controversial. Am J Med 97:418-428. Based on these data. Additionally. the monitoring of serum thyroglobulin as a marker of residual cancer or tumor recurrence is more easily accomplished after a total thyroidectomy. 1992 4. and there is no evidence that removal of grossly normal lymph nodes can prolong the survival of a patient with well-differentiated thyroid cancer. J Clin Endocrinol Metab 75:714-720. 1994 3. Winchester DJ. Fine-needle aspiration (FNA) of a solitary nodule diagnostic for papillary cancer is confirmed in 98% to 100% of cases when final pathology is examined. The extent of lymphadenectomy in this patient with papillary thyroid cancer is another controversial question.0 cm. Surgery 116:10541060. If a partial thyroidectomy is performed. 1994 page 565 . Only 10% of patients with microscopic lymph node metastases actually develop clinically significant recurrent disease. Schultz PN.Samaan NA. Hickey RC. Jhiang SM: Long-term impact of initial surgical and medical therapy on papillary and follicular thyroid cancer. this should be the therapeutic approach for patients with well-differentiated cancers >1. Hickey RC. Caraway NP. REFERENCES 1. 1995 2. Although histologic examinaton of lymph nodes has demonstrated that 30% to 80% of patients with papillary thyroid cancer have lymph node metastases. patients over 50 with 4-cm nodules are at a higher risk of thyroid cancer. American College of Surgeons SESAP 13 Category 6 Item 15 A 15-year-old boy has a rapidly growing 2-cm thyroid nodule and palpable jugular lymph nodes on the left. Elevated serum calcium in this patient is most consistent with (A) familial medullary thyroid cancer (B) multiple endocrine neoplasia type 2A (MEN2A) (C) MEN2B (D) Von Hippel Lindau syndrome (E) MEN1 page 566 . Fine-needle aspiration (FNA) biopsy of the nodule demonstrates calcitonin-positive cells. Angeli A. Newbold K. respectively. 2005 page 567 .Brandi ML. neuroendocrine tumors of the pancreas. The presence of increased serum calcium makes this patient at highest risk of concomitant hyperparthyroidism. MEN1 syndrome is derived from mutation of a different gene (MENIN gene) than MEN2 and is characterized by the concurrence of parathyroid hyperplasia. and hyperparathyroidism. and vascular tumors within the central nervous system REFERENCES 1. Bilezikian JP. Harmer C: Familial medullary thyroid cancer: clinical aspects and prognosis. et al: Guidelines for diagnosis and therapy of MEN type 1 and type 2. MEN2B includes MTC. Later. Eur J Surg Oncol 31:415-419. and adenomas of the anterior pituitary gland. J Clin Endocrinol Metab 86:5658-5671.American College of Surgeons SESAP 13 Critique for Category 6 Item 15 Medullary thyroid carcinoma (MTC) is a rare tumor derived from the parafollicular C cells of the thyroid. Pheochromocytoma and hyperparathyroidism develop in 50% and 20% of patients with MEN2A.Hyer SL. and thus MEN2A rather than MEN2B or FMTC. Three phenotypes are described: multiple endocrine neoplasia type 2A (MEN2A) is characterized by the development of MTC. neuroendocrine tumors of the pancreas or duodenum. accounting for 5% to10% of all thyroid malignancies. Von Hipple Lindau syndrome is caused by inherited mutations of the VHL gene and is associated with pheochromocytomas. These clinical entities result from mutations of the ret proto-oncogene. marfanoid habitus. Although most MTCs are sporadic in origin. and mucosal neuromas. Early observations by Sipple identified an association between inherited MTC and neoplasia of the adrenal glands. associations with mucosal and skeletal abnormalities were described. as many as 20% to 25% arise secondary to inherited genetic mutations. and familial MTC (FMTC) involves inheritance of MTC alone. 2001 2. pheochromocytoma. Gagel RF. American College of Surgeons SESAP 13 Category 6 Item 16 Which of the following statements about relative adrenal insufficiency in volume-resuscitated patients who are in septic shock despite fluid resuscitation and vasopressor therapy is TRUE? (A) It occurs in fewer than 25% of patients (B) It is diagnosed by an increase in cortisol level of > 9 µg/dL in response to corticotrophin (C) It does not affect vasopressor requirements (D) Corticosteroid replacement improves survival (E) Corticosteroid replacement increases the incidence of adverse events page 568 . Arch Surg 140:661-668. In a prospective randomized controlled clinical trial. et al: Meta-analysis: the effect of steroids on survival and shock during sepsis depends on the dose. Patel B. and were randomly assigned to receive either hydrocortisone (50 mg intravenously every 6 hours) and fludrocortisone (50µg tablet once daily) or matching placebos for 7 days. Banks SM. more physiologic. Relative adrenal insufficiency in critically ill patients is defined as failure to increase cortisol level by at least 9 µg/dL in response to corticotrophin.Annane D. Eichacker PQ. N Engl J Med 348:727734. Two possible reasons for this study’s success were the use of relatively low doses of hydrocortisone (200 mg/day) and the addition of mineralocorticoids. 2002 2. et al: Effect of treatment with low doses of hydrocortisone and fludrocortisone on mortality in patients with septic shock.Minneci PC. Ann Intern Med 141:47-56. Larson J. 2005 page 569 . A meta-analysis of clinical trials in septic shock suggests that a 5. JAMA 288:862-871. Several prior studies of corticosteroids for patients with septic shock failed to demonstrate any difference. ie. Deans KJ. Sebille V.Cooper MS. Johnson DJ. Bollaert PE.Rady MY. 300 patients requiring vasopressor therapy for the management of septic shock underwent a corticotrophin stimulation test. Charpentier C. REFERENCES 1.to 7-day course of low-dose. There was no difference in adverse events. Stewart PM: Corticosteroid insufficiency in acutely ill patients. corticosteroids decreases time to shock reversal and improves survival. 2004 4. et al: Cortisol levels and corticosteroid administration fail to predict mortality in critical illness: the confounding effects of organ dysfunction and sex. Those with relative adrenal insufficiency and corticosteroid replacement were more likely to be weaned off vasopressors and had a lower mortality rate.American College of Surgeons SESAP 13 Critique for Category 6 Item 16 Three fourths of patients with septic shock who are poorly responsive to fluid resuscitation and vasopressor therapy are found to have relative adrenal insufficiency. 2003 3. followed by alpha blockade page 570 .American College of Surgeons SESAP 13 Category 6 Item 17 Which of the following statements about the surgical management of primary sporadic pheochromocytoma is TRUE? (A) Preoperative phenoxybenzamine is not necessary (B) Preoperative iodine 123 meta-iodobenzylgaunidine (MIBG) imaging commonly identifies extraadrenal foci of disease (C) Preoperative MIBG imaging reduces postoperative complications (D) Laparoscopic operation is associated with more intraoperative hypotension (E) Most patients require beta blockade. Miskulin J. pp 343-346 page 571 . Although open exploration was once advocated for all pheochromocytomas. 2003 2. et al: Is preoperative iodine 123 metaiodobenzylguanidine scinitigraphy routinely necessary before initial adrenalectomy for pheochromocytoma? Surgery 134:918-922. Lippincott Williams & Wilkins. Endocr Rev 24:539553. Philadelphia. in Khatri VP (ed): Clinical Scenarios in Surgical Oncology. preoperative iodine 123 metaiodobenzylguanidine (MIBG) scanning for primary. Sisson JC. rarely identifies other sites of disease or alters treatment plans/outcomes. sporadic pheochromocytomas has low sensitivity.Norton JA: Management of incidental adrenal lesion.Bravo EL. selective alpha-1 antagonists (such as prazosin) or calcium channel blockers are preferred. Routine alpha blockade with phenoxybenzamine has not been shown to prevent intraoperative hypertension. laparoscopy is now believed to be safe for smaller tumors and may in fact produce less severe intraoperative hypotension. Beta-blockade should never be started before alphablockade because unopposed vasoconstriction may worsen hypertension. Shulkin B. 2006. and the use of drugs that produce profound and long-lasting alpha blockade is no longer believed to be necessary. Although once popular for completion of the metastatic work-up.American College of Surgeons SESAP 13 Critique for Category 6 Item 17 The management of pheochromocytoma has changed dramatically in recent years. and offers no real advantage over magnetic resonance imaging (MRI). When hypertension and/or symptom control is necessary. 2003 3. and many traditional elements of surgical management are no longer believed to be essential. REFERENCES 1. Tagle R: Pheochromocytoma: state-of-the-art and future prospects. Doherty GM. 1 mg/dL. 1. BUN.1 mEq/L. On physical examination he is 5’11”. and PO4. 3. creatinine. weighs 185 lb. Pertinent data include serum sodium. 140 mEq/L. The remainder of his physical examination is unremarkable. His blood pressure has been unresponsive to several medications. 15 mg/dL. and appears healthy. An EKG shows slight left ventricular hypertrophy. serum chloride.American College of Surgeons SESAP 13 Category 6 Item 18 Six months after an uneventful laparoscopic cholecystectomy.0 mEq/L. serum potassium. 4. The next step should be (A) serum aldosterone and renin levels (B) 24-hour urine catecholamine levels (C) plasma metanephrine levels (D) serum calcitonin levels (E) 24-hour urine free cortisol levels page 572 . an otherwise healthy 54-year-old man presents with hypertension. 104 mEq/L. Blood pressure is 180/94 with a pulse of 88. REFERENCES 1. The second part of this question deals with the continued work-up without the findings of a unilateral adrenal mass. Moley JF: The pituitary and adrenal glands. Several studies have suggested that close examination of the ARR may result in higher discovery rates of primary aldosteronism. hypokalemia is considered a virtual prerequisite for pursuing this diagnosis. 2004. Preoperative alpha blockade prior to adrenalectomy. The second is his history of hypertension with hypokalemia. but are unlikely to offer significant aid in diagnosis. This patient does not have any abnormal adrenal CT findings. Plasma metanephrine levels may be helpful in the work-up of pheochromocytoma. Fardella CE. The first. et al: Increased diagnosis of primary aldosteronism. operator-dependent technique. One of the drawbacks of imaging the adrenals in patients with aldosterone excess states is the security of the diagnosis of unilateral disease. However. Unilateral adrenalectomy is associated with excellent results in patients with primary aldosterone producing adenoma of the adrenal gland. In a recent study. Philadelphia.Mulatero P. and a serum aldosterone and renin level would provide the most appropriate information. Serum calcitonin levels are not appropriate because multiple endocrine neoplasia type 2 (MEN2) syndrome is an unlikely diagnosis in this patient. Smaller micronodular bilateral involvement of the adrenal cortex can be missed by both magnetic resonance imaging (MRI) and computed tomography (CT). adrenal vein sampling would be the most reasonable next step. operation is not associated with good results for patients with bilateral hyperplasia. Additionally. an aldosterone excess state is far more likely given this information. in centers from five continents. pp 1023-1070 2. Saunders. ed 17. A 24-hour urine catecholamine level will likely be normal. Primary aldosterone secretion is seen in fewer than 1% of all hypertensive patients. 24 of 58 patients (41%) with normal adrenal CT findings had unilateral aldosterone hypersecretion and were candidates for unilateral laparoscopic adrenalectomy. but can be done safely with good results in experienced centers. is that he recently underwent general anesthesia without complications. or a metastatic work-up are not indicated in this patient. J Clin Endocrinol Metab page 573 . and therefore unrevealing. The question now arises as to whether adrenal vein sampling should be done. with normalization of hypokalemia approaching 90% and improvement in blood pressure in over 80%.American College of Surgeons SESAP 13 Critique of Category 6 Item 18 This patient presents with a history of significant hypertension unresponsive to medical treatment. in Townsend CM Jr (ed): Sabiston Textbook of Surgery: The Biological Basis of Modern Surgical Practice. More recent studies have advocated the use of the plasma aldosterone/plasma renin activity ratio (ARR) as a screening test for hypertensive patients regardless of the potassium levels. In this patient without image localization of unilateral disease. Loh KC. . A urine free cortisol would be appropriate in the setting of cortisol excess. Stowasser M. Although pheochromocytoma is a possibility. As many as 12% of hypertensive patients might be candidates for work-up for primary aldosteronism based on ARR data. including surgically correctable forms.Brunt LM. Two important pieces of data are apparent. Adrenal vein sampling is an invasive. and more subjective. a thyroid ultrasound study. Surgery 136:1227-1235. 2004 page 574 . et al: Role for adrenal venous sampling in primary aldosteronism.Young WF.American College of Surgeons SESAP 13 89:1045-1050. 2004 3. Thompson GB. Grant CS. Stanson AW. 1. Pertinent data include serum sodium. The remainder of his physical examination is unremarkable. An EKG shows slight left ventricular hypertrophy. serum potassium. BUN. 3.1 mg/dL. 140 mEq/L. and appears healthy.0 mEq/L. an otherwise healthy 54-year-old man presents with hypertension. creatinine.1 mEq/L. His blood pressure has been unresponsive to several medications. After appropriate work-up. 4. serum chloride. The next step should be (A) preoperative alpha blockade and adrenalectomy (B) thyroid ultrasound study (C) adrenal vein sampling (D) laparoscopic adrenalectomy (E) endoscopic retrograde cholangiopancreatography (ERCP) page 575 . and PO4. the computed tomographic (CT) scan shown is obtained. Blood pressure is 180/94 with a pulse of 88. weighs 185 lb. 15 mg/dL. On physical examination he is 5’11”. 104 mEq/L.American College of Surgeons SESAP 13 Category 6 Item 19 Six months after an uneventful laparoscopic cholecystectomy. Two important pieces of data are apparent. Philadelphia. Loh KC. REFERENCES 1. and more subjective. adrenal vein sampling would be the most reasonable next step. Fardella CE. operation is not associated with good results for patients with bilateral hyperplasia. including surgically correctable forms. operator-dependent technique. 2004. More recent studies have advocated the use of the plasma aldosterone/plasma renin activity ratio (ARR) as a screening test for hypertensive patients regardless of the potassium levels. Plasma metanephrine levels may be helpful in the work-up of pheochromocytoma. in Townsend CM Jr (ed): Sabiston Textbook of Surgery: The Biological Basis of Modern Surgical Practice. In this patient without image localization of unilateral disease. Several studies have suggested that close examination of the ARR may result in higher discovery rates of primary aldosteronism. As many as 12% of hypertensive patients might be candidates for work-up for primary aldosteronism based on ARR data. or a metastatic work-up are not indicated in this patient. Stowasser M. Moley JF: The pituitary and adrenal glands. The second is his history of hypertension with hypokalemia. J Clin Endocrinol Metab page 576 . However. pp 1023-1070 2. In a recent study. The first. but can be done safely with good results in experienced centers.American College of Surgeons SESAP 13 Critique for Category 6 Item 19 This patient presents with a history of significant hypertension unresponsive to medical treatment. in centers from five continents. Serum calcitonin levels are not appropriate because multiple endocrine neoplasia type 2 (MEN2) syndrome is an unlikely diagnosis in this patient. Adrenal vein sampling is an invasive. Preoperative alpha blockade prior to adrenalectomy. a thyroid ultrasound study. Unilateral adrenalectomy is associated with excellent results in patients with primary aldosterone producing adenoma of the adrenal gland. with normalization of hypokalemia approaching 90% and improvement in blood pressure in over 80%. The question now arises as to whether adrenal vein sampling should be done. One of the drawbacks of imaging the adrenals in patients with aldosterone excess states is the security of the diagnosis of unilateral disease. an aldosterone excess state is far more likely given this information.Mulatero P. hypokalemia is considered a virtual prerequisite for pursuing this diagnosis. and therefore unrevealing. A urine free cortisol would be appropriate in the setting of cortisol excess. et al: Increased diagnosis of primary aldosteronism. Additionally. 24 of 58 patients (41%) with normal adrenal CT findings had unilateral aldosterone hypersecretion and were candidates for unilateral laparoscopic adrenalectomy. ed 17. is that he recently underwent general anesthesia without complications. and a serum aldosterone and renin level would provide the most appropriate information. This patient does not have any abnormal adrenal CT findings. The second part of this question deals with the continued work-up without the findings of a unilateral adrenal mass. . but are unlikely to offer significant aid in diagnosis. Smaller micronodular bilateral involvement of the adrenal cortex can be missed by both magnetic resonance imaging (MRI) and computed tomography (CT).Brunt LM. Although pheochromocytoma is a possibility. Primary aldosterone secretion is seen in fewer than 1% of all hypertensive patients. Saunders. A 24-hour urine catecholamine level will likely be normal. American College of Surgeons SESAP 13 89:1045-1050, 2004 3.Young WF, Stanson AW, Thompson GB, Grant CS, et al: Role for adrenal venous sampling in primary aldosteronism. Surgery 136:1227-1235, 2004 page 577 American College of Surgeons SESAP 13 Category 6 Item 20 A 42-year-old woman is a known member of a multiple endocrine neoplasia type 2A (MEN2A) kindred. She and her mother have both had total thyroidectomy for medullary carcinoma of the thyroid (MCT). She would like her 6-year-old daughter to be tested for possible MCT. The child’s physical examination is normal. Which of the following should be done? (A) Serum calcitonin and carcinoembryonic antigen (CEA) determinations (B) Basal and stimulated serum calcitonin levels (C) Iodine 131 radionuclide scan (D) Annual physical examinations; fine-needle aspiration if a thyroid mass is appreciated, or an ultrasound study at age 12 (E) RET oncogene determination page 578 American College of Surgeons SESAP 13 Critique for Category 6 Item 20 Medullary carcinoma of the thyroid (MCT) was initially described nearly 50 years ago as a tumor of the C cells of the thyroid derived from the neural crest. MCT is associated with a broad spectrum of clinical behavior ranging from virulent to benign. It comprises 5% to 10% of all thyroid cancers and may be classified as sporadic or familial. Sporadic MCT is usually unilateral without a family history. Familial MCT includes the multiple endocrine neoplasia (MEN) type 2A and 2B syndromes. A non MEN carcinoma also occurs in a familial pattern known as familial MCT. Each of these three are inherited as an autosomal dominant lesion. In the MEN2A setting, patients will develop bilateral or multifocal MCT associated with C cell hyperplasia. Germline defects in the RET proto-oncogene are responsible for MEN2A, MEN2B, and familial MCT. Discovery of the genetic alteration responsible for the hereditary forms of MCT allows the detection of the carriers of the gene before calcitonin levels become elevated and long before the carcinoma becomes clinically manifest. In fact, a positive basal or stimulated calcitonin test most likely indicates that the cancer has already developed. The landmark paper by Wells in 1994 described the results of total thyroidectomy and central lymph node dissection for patients as young as 6 years old who had positive genetic testing. Of the patients with normal preoperative calcitonin levels, all had at least C cell hyperplasia and several had microscopic carcinoma. None of the pathologic specimens excised were normal. The mother of a 6-year-old with a clear family history of MEN2A is appropriately concerned about her child carrying the RET oncogene, and testing is appropriate at this age. Serum basal or stimulated calcitonins either at this time or later are not a predictor of the disease. There is no rationale for delaying imaging studies until a later time. When the genetic test is positive, total thyroidectomy with central lymph node dissection is indicated. Screening with other imaging scans such as computed tomography (CT) or positronemission tomography (PET) is probably not appropriate at this time, nor is surveillance. REFERENCES 1.Moley JF, Shervin N: Medullary thyroid carcinoma, in Clark OH, Duh Q-Y, Kebebew E (eds): Textbook of Endocrine Surgery, ed 2. Philadelphia, Elsevier Saunders, 2005, pp 129-141 2.Sanso GE, Domene HM, Garcia R, Pusiol E, et al: Very early detection of RET proto-oncogene mutation is crucial for preventive thyroidectomy in multiple endocrine neoplasia type 2 children: presence of C-cell malignant disease in asymptomatic carriers. Cancer 94:323-330, 2002 3.Skinner MA, Moley JA, Dilley WG, Owzar K, et al: Prophylactic thyroidectomy in multiple endocrine neoplasia type 2A. N Engl J Med 353:1105-1113, 2005 4.Wells SA Jr, Chi DD, Toshima K, Dehner LP, et al: Predictive DNA testing and prophylactic thyroidectomy in patients at risk for multiple endocrine neoplasia type 2A. Ann Surg 220:237-250, 1994 page 579 American College of Surgeons SESAP 13 Category 6 Item 21 A 42-year-old woman is a known member of a multiple endocrine neoplasia type 2A (MEN2A) kindred. She and her mother have both had total thyroidectomy for medullary carcinoma of the thyroid (MCT). She would like her 6-year-old daughter to be tested for possible MCT. The child’s physical examination is normal. Testing for this child yields positive results. The next step should be (A) neck and chest computed tomographic (CT) scan (B) neck and chest positron emission tomographic (PET) scan (C) total thyroidectomy with central lymph node dissection (D) subtotal thyroidectomy with ipislateral lymph node dissection (E) annual surveillance with ultrasound studies until age 12 page 580 American College of Surgeons SESAP 13 Critique for Category 6 Item 21 Medullary carcinoma of the thyroid (MCT) was initially described nearly 50 years ago as a tumor of the C cells of the thyroid derived from the neural crest. MCT is associated with a broad spectrum of clinical behavior ranging from virulent to benign. It comprises 5% to 10% of all thyroid cancers and may be classified as sporadic or familial. Sporadic MCT is usually unilateral without a family history. Familial MCT includes the multiple endocrine neoplasia (MEN) type 2A and 2B syndromes. A non MEN carcinoma also occurs in a familial pattern known as familial MCT. Each of these three are inherited as an autosomal dominant lesion. In the MEN2A setting, patients will develop bilateral or multifocal MCT associated with C cell hyperplasia. Germline defects in the RET proto-oncogene are responsible for MEN2A, MEN2B, and familial MCT. Discovery of the genetic alteration responsible for the hereditary forms of MCT allows the detection of the carriers of the gene before calcitonin levels become elevated and long before the carcinoma becomes clinically manifest. In fact, a positive basal or stimulated calcitonin test most likely indicates that the cancer has already developed. The landmark paper by Wells in 1994 described the results of total thyroidectomy and central lymph node dissection for patients as young as 6 years old who had positive genetic testing. Of the patients with normal preoperative calcitonin levels, all had at least C cell hyperplasia and several had microscopic carcinoma. None of the pathologic specimens excised were normal. The mother of a 6-year-old with a clear family history of MEN2A is appropriately concerned about her child carrying the RET oncogene, and testing is appropriate at this age. Serum basal or stimulated calcitonins either at this time or later are not a predictor of the disease. There is no rationale for delaying imaging studies until a later time. When the genetic test is positive, total thyroidectomy with central lymph node dissection is indicated. Screening with other imaging scans such as computed tomography (CT) or positronemission tomography (PET) is probably not appropriate at this time, nor is surveillance. REFERENCES 1.Moley JF, Shervin N: Medullary thyroid carcinoma, in Clark OH, Duh Q-Y, Kebebew E (eds): Textbook of Endocrine Surgery, ed 2. Philadelphia, Elsevier Saunders, 2005, pp 129-141 2.Sanso GE, Domene HM, Garcia R, Pusiol E, et al: Very early detection of RET proto-oncogene mutation is crucial for preventive thyroidectomy in multiple endocrine neoplasia type 2 children: presence of C-cell malignant disease in asymptomatic carriers. Cancer 94:323-330, 2002 3.Skinner MA, Moley JA, Dilley WG, Owzar K, et al: Prophylactic thyroidectomy in multiple endocrine neoplasia type 2A. N Engl J Med 353:1105-1113, 2005 4.Wells SA Jr, Chi DD, Toshima K, Dehner LP, et al: Predictive DNA testing and prophylactic thyroidectomy in patients at risk for multiple endocrine neoplasia type 2A. Ann Surg 220:237-250, 1994 page 581 American College of Surgeons SESAP 13 Category 6 Item 22 Characterized by gallstones, diabetes, and steatorrhea (A) Glucagonoma (B) Insulinoma (C) Vipoma (D) Gastrinoma (E) Somatostatinoma page 582 American College of Surgeons SESAP 13 Critique for Category 6 Item 22 Somatostatinomas are rare neuroendocrine tumors of D cell origin that secrete excessive amounts of somatostatin. Symptoms of pancreatic somatostatinoma are usually nonspecific, consisting of mild diabetes (inhibition of both insulin and glucagon), cholelithiasis (inhibition of cholecystokinin with reduction in gallbladder contractility), and diarrhea. The diarrhea, with or without steatorrhea, is caused by inhibition of pancreatic enzyme and bicarbonate secretion that leads to impaired intestinal absorption of lipids. In contrast, symptoms associated with duodenal somatostatinomas are usually due to local mass effect rather than the actions of the hormones. The clinical syndrome classically associated with glucagonoma includes necrolytic migratory erythema (NME), cheilitis, diabetes mellitus, anemia, weight loss, venous thrombosis, and neuropsychiatric symptoms. Weight loss and NME are the most prevalent symptoms, occurring in 65% to 70% of patients by the time of diagnosis. NME characteristically begins as erythematous papules or plaques involving the face, perineum, and extremities. The lesions enlarge and coalesce over the next 7 to 14 days, followed by central clearing that leaves bronze-colored, indurated areas centrally, with blistering, crusting, and scaling at the borders. The affected areas are often pruritic and painful. Cheilitis is inflammation of the lips associated with redness and fissures radiating from the corners of the mouth. About 10% of insulinomas are malignant versus 50% of vipomas and 60% of gastrinomas. Most glucagonomas and somatostatinomas are malignant and have metastasized by the time of diagnosis. Vipomas are rare neuroendocrine tumors that secrete vasoactive intestinal polypeptide (VIP). VIP is a 28 amino acid polypeptide that binds to high-affinity receptors on intestinal epithelial cells, leading to activation of cellular adenylate cyclase and cAMP production. This results in net fluid and electrolyte secretions into the lumen, a secretory watery diarrhea, hypokalemia, hypochlorhydria, and alkalosis (WDHHA syndrome). As many as 70% of gastrinomas form in the duodenum; the remainder, with rare exception, arise in the pancreas or less often in lymph nodes adjacent to the pancreas (5%). Duodenal gastrinomas tend to be small and are often multiple. They have less malignant potential than the solitary, sporadic pancreatic gastrinomas. REFERENCES 1.Bloom SR, Yiangou Y, Polak, JM: Vasoactive intestinal peptide secreting tumors. Pathophysiological and clinical correlations. Ann NY Acad Sci 527:518-527, 1988 2.Doherty GM: Rare endocrine tumours of the GI tract. Best Pract Res Clin Gastroenterol 19:807-817, 2005 3.Jensen RT: Endocrine neoplasms of the pancreas, in Yamada T, Alpers DH, Kaplowitz N, Laine L, et al (eds): Textbook of Gastroenterology, ed 4. Philadelphia, Lippincott Williams & Wilkins, 2003, pp 2108-2146 4.Stacpoole PW: The glucagonoma syndrome: clinical features, diagnosis, and treatment. Endocr Rev 2:347-361, 1981 page 583 American College of Surgeons SESAP 13 Category 6 Item 23 Necrolytic migratory erythema (A) Glucagonoma (B) Insulinoma (C) Vipoma (D) Gastrinoma (E) Somatostatinoma page 584 American College of Surgeons SESAP 13 Critique for Category 6 Item 23 Somatostatinomas are rare neuroendocrine tumors of D cell origin that secrete excessive amounts of somatostatin. Symptoms of pancreatic somatostatinoma are usually nonspecific, consisting of mild diabetes (inhibition of both insulin and glucagon), cholelithiasis (inhibition of cholecystokinin with reduction in gallbladder contractility), and diarrhea. The diarrhea, with or without steatorrhea, is caused by inhibition of pancreatic enzyme and bicarbonate secretion that leads to impaired intestinal absorption of lipids. In contrast, symptoms associated with duodenal somatostatinomas are usually due to local mass effect rather than the actions of the hormones. The clinical syndrome classically associated with glucagonoma includes necrolytic migratory erythema (NME), cheilitis, diabetes mellitus, anemia, weight loss, venous thrombosis, and neuropsychiatric symptoms. Weight loss and NME are the most prevalent symptoms, occurring in 65% to 70% of patients by the time of diagnosis. NME characteristically begins as erythematous papules or plaques involving the face, perineum, and extremities. The lesions enlarge and coalesce over the next 7 to 14 days, followed by central clearing that leaves bronze-colored, indurated areas centrally, with blistering, crusting, and scaling at the borders. The affected areas are often pruritic and painful. Cheilitis is inflammation of the lips associated with redness and fissures radiating from the corners of the mouth. About 10% of insulinomas are malignant versus 50% of vipomas and 60% of gastrinomas. Most glucagonomas and somatostatinomas are malignant and have metastasized by the time of diagnosis. Vipomas are rare neuroendocrine tumors that secrete vasoactive intestinal polypeptide (VIP). VIP is a 28 amino acid polypeptide that binds to high-affinity receptors on intestinal epithelial cells, leading to activation of cellular adenylate cyclase and cAMP production. This results in net fluid and electrolyte secretions into the lumen, a secretory watery diarrhea, hypokalemia, hypochlorhydria, and alkalosis (WDHHA syndrome). As many as 70% of gastrinomas form in the duodenum; the remainder, with rare exception, arise in the pancreas or less often in lymph nodes adjacent to the pancreas (5%). Duodenal gastrinomas tend to be small and are often multiple. They have less malignant potential than the solitary, sporadic pancreatic gastrinomas. REFERENCES 1.Bloom SR, Yiangou Y, Polak, JM: Vasoactive intestinal peptide secreting tumors. Pathophysiological and clinical correlations. Ann NY Acad Sci 527:518-527, 1988 2.Doherty GM: Rare endocrine tumours of the GI tract. Best Pract Res Clin Gastroenterol 19:807-817, 2005 3.Jensen RT: Endocrine neoplasms of the pancreas, in Yamada T, Alpers DH, Kaplowitz N, Laine L, et al (eds): Textbook of Gastroenterology, ed 4. Philadelphia, Lippincott Williams & Wilkins, 2003, pp 2108-2146 4.Stacpoole PW: The glucagonoma syndrome: clinical features, diagnosis, and treatment. Endocr Rev 2:347-361, 1981 page 585 American College of Surgeons SESAP 13 Category 6 Item 24 Secretes an amino acid polypeptide that activates cellular adenylate cyclase and cAMP production (A) Glucagonoma (B) Insulinoma (C) Vipoma (D) Gastrinoma (E) Somatostatinoma page 586 American College of Surgeons SESAP 13 Critique for Category 6 Item 24 Somatostatinomas are rare neuroendocrine tumors of D cell origin that secrete excessive amounts of somatostatin. Symptoms of pancreatic somatostatinoma are usually nonspecific, consisting of mild diabetes (inhibition of both insulin and glucagon), cholelithiasis (inhibition of cholecystokinin with reduction in gallbladder contractility), and diarrhea. The diarrhea, with or without steatorrhea, is caused by inhibition of pancreatic enzyme and bicarbonate secretion that leads to impaired intestinal absorption of lipids. In contrast, symptoms associated with duodenal somatostatinomas are usually due to local mass effect rather than the actions of the hormones. The clinical syndrome classically associated with glucagonoma includes necrolytic migratory erythema (NME), cheilitis, diabetes mellitus, anemia, weight loss, venous thrombosis, and neuropsychiatric symptoms. Weight loss and NME are the most prevalent symptoms, occurring in 65% to 70% of patients by the time of diagnosis. NME characteristically begins as erythematous papules or plaques involving the face, perineum, and extremities. The lesions enlarge and coalesce over the next 7 to 14 days, followed by central clearing that leaves bronze-colored, indurated areas centrally, with blistering, crusting, and scaling at the borders. The affected areas are often pruritic and painful. Cheilitis is inflammation of the lips associated with redness and fissures radiating from the corners of the mouth. About 10% of insulinomas are malignant versus 50% of vipomas and 60% of gastrinomas. Most glucagonomas and somatostatinomas are malignant and have metastasized by the time of diagnosis. Vipomas are rare neuroendocrine tumors that secrete vasoactive intestinal polypeptide (VIP). VIP is a 28 amino acid polypeptide that binds to high-affinity receptors on intestinal epithelial cells, leading to activation of cellular adenylate cyclase and cAMP production. This results in net fluid and electrolyte secretions into the lumen, a secretory watery diarrhea, hypokalemia, hypochlorhydria, and alkalosis (WDHHA syndrome). As many as 70% of gastrinomas form in the duodenum; the remainder, with rare exception, arise in the pancreas or less often in lymph nodes adjacent to the pancreas (5%). Duodenal gastrinomas tend to be small and are often multiple. They have less malignant potential than the solitary, sporadic pancreatic gastrinomas. REFERENCES 1.Bloom SR, Yiangou Y, Polak, JM: Vasoactive intestinal peptide secreting tumors. Pathophysiological and clinical correlations. Ann NY Acad Sci 527:518-527, 1988 2.Doherty GM: Rare endocrine tumours of the GI tract. Best Pract Res Clin Gastroenterol 19:807-817, 2005 3.Jensen RT: Endocrine neoplasms of the pancreas, in Yamada T, Alpers DH, Kaplowitz N, Laine L, et al (eds): Textbook of Gastroenterology, ed 4. Philadelphia, Lippincott Williams & Wilkins, 2003, pp 2108-2146 4.Stacpoole PW: The glucagonoma syndrome: clinical features, diagnosis, and treatment. Endocr Rev 2:347-361, 1981 page 587 American College of Surgeons SESAP 13 Category 6 Item 25 Higher frequency in men (A) Aldosteroma (B) Idiopathic hyperaldosteronism (C) Both (D) Neither page 588 American College of Surgeons SESAP 13 Critique for Category 6 Item 25 Primary hyperaldosteronism is defined as hypertension in the presence of elevated plasma aldosterone and suppressed plasma renin activity. Aldosteronoma and idiopathic hyperaldosteronism (IHA) are the two major causes. Aldosteronomas account for approximately 60% of cases and occur more often in women and younger patients. Hypertension and hypokalemia tend to be more severe compared with IHA. Salt loading and postural testing do not change aldosterone levels in patients with aldosteronoma. Computed tomography (CT) will typically reveal a small (< 2 cm) solitary adrenal tumor. Adrenal vein sampling (AVS) will reveal unilateral hypersecretion of aldosterone. This status is reflected by a side-to-side cortisol corrected aldosterone ratio (comparing the two adrenals) > 4. Treatment for aldosteronoma is surgical resection. IHA accounts for approximately 30% of cases of primary hyperaldosteronism. It occurs more often in men and older patients. Hypertension and hypokalemia is less severe than with aldosteronoma. Salt loading leads to a decrease in aldosterone levels, whereas postural testing results in an increase in aldosterone levels in patients with IHA. CT reveals normal-appearing or bilaterally enlarged adrenal glands. AVS will show evidence of bilateral hypersecretion of aldosterone reflected in a side-to-side cortisol corrected aldosterone ratio < 3. Treatment involves medical management with potassiumsparing diuretics. Spironolactone is most effective. Pheochromocytomas typically show evidence of hyperintensity on T2 weighted magnetic resonance imaging. REFERENCES 1.Al Fehaily M, Duh QY: Clinical manifestation of aldosteronoma. Surg Clin North Am 84:887-905, 2004 2.Al-Hawary MM, Francis IR, Korobkin M: Non-invasive evaluation of the incidentally detected indeterminate adrenal mass. Best Pract Res Clin Endocrinol Metab 19:277-292, 2005 3.Mansmann G, Lau J, Balk E, Rothberg M, et al: The clinically inapparent adrenal mass: update in diagnosis and management. Endocr Rev 25:309-340, 2004 4.Thompson GB, Young WF Jr: Adrenal incidentaloma. Curr Opin Oncol 15:84-90, 2003 page 589 American College of Surgeons SESAP 13 Category 6 Item 26 Primary therapy is surgical resection (A) Aldosteroma (B) Idiopathic hyperaldosteronism (C) Both (D) Neither page 590 American College of Surgeons SESAP 13 Critique for Category 6 Item 26 Primary hyperaldosteronism is defined as hypertension in the presence of elevated plasma aldosterone and suppressed plasma renin activity. Aldosteronoma and idiopathic hyperaldosteronism (IHA) are the two major causes. Aldosteronomas account for approximately 60% of cases and occur more often in women and younger patients. Hypertension and hypokalemia tend to be more severe compared with IHA. Salt loading and postural testing do not change aldosterone levels in patients with aldosteronoma. Computed tomography (CT) will typically reveal a small (< 2 cm) solitary adrenal tumor. Adrenal vein sampling (AVS) will reveal unilateral hypersecretion of aldosterone. This status is reflected by a side-to-side cortisol corrected aldosterone ratio (comparing the two adrenals) > 4. Treatment for aldosteronoma is surgical resection. IHA accounts for approximately 30% of cases of primary hyperaldosteronism. It occurs more often in men and older patients. Hypertension and hypokalemia is less severe than with aldosteronoma. Salt loading leads to a decrease in aldosterone levels, whereas postural testing results in an increase in aldosterone levels in patients with IHA. CT reveals normal-appearing or bilaterally enlarged adrenal glands. AVS will show evidence of bilateral hypersecretion of aldosterone reflected in a side-to-side cortisol corrected aldosterone ratio < 3. Treatment involves medical management with potassiumsparing diuretics. Spironolactone is most effective. Pheochromocytomas typically show evidence of hyperintensity on T2 weighted magnetic resonance imaging. REFERENCES 1.Al Fehaily M, Duh QY: Clinical manifestation of aldosteronoma. Surg Clin North Am 84:887-905, 2004 2.Al-Hawary MM, Francis IR, Korobkin M: Non-invasive evaluation of the incidentally detected indeterminate adrenal mass. Best Pract Res Clin Endocrinol Metab 19:277-292, 2005 3.Mansmann G, Lau J, Balk E, Rothberg M, et al: The clinically inapparent adrenal mass: update in diagnosis and management. Endocr Rev 25:309-340, 2004 4.Thompson GB, Young WF Jr: Adrenal incidentaloma. Curr Opin Oncol 15:84-90, 2003 page 591 American College of Surgeons SESAP 13 Category 6 Item 27 Hyperintensity on T2 weighted magnetic resonance imaging (MRI) (A) Aldosteroma (B) Idiopathic hyperaldosteronism (C) Both (D) Neither page 592 American College of Surgeons SESAP 13 Critique for Category 6 Item 27 Primary hyperaldosteronism is defined as hypertension in the presence of elevated plasma aldosterone and suppressed plasma renin activity. Aldosteronoma and idiopathic hyperaldosteronism (IHA) are the two major causes. Aldosteronomas account for approximately 60% of cases and occur more often in women and younger patients. Hypertension and hypokalemia tend to be more severe compared with IHA. Salt loading and postural testing do not change aldosterone levels in patients with aldosteronoma. Computed tomography (CT) will typically reveal a small (< 2 cm) solitary adrenal tumor. Adrenal vein sampling (AVS) will reveal unilateral hypersecretion of aldosterone. This status is reflected by a side-to-side cortisol corrected aldosterone ratio (comparing the two adrenals) > 4. Treatment for aldosteronoma is surgical resection. IHA accounts for approximately 30% of cases of primary hyperaldosteronism. It occurs more often in men and older patients. Hypertension and hypokalemia is less severe than with aldosteronoma. Salt loading leads to a decrease in aldosterone levels, whereas postural testing results in an increase in aldosterone levels in patients with IHA. CT reveals normal-appearing or bilaterally enlarged adrenal glands. AVS will show evidence of bilateral hypersecretion of aldosterone reflected in a side-to-side cortisol corrected aldosterone ratio < 3. Treatment involves medical management with potassiumsparing diuretics. Spironolactone is most effective. Pheochromocytomas typically show evidence of hyperintensity on T2 weighted magnetic resonance imaging. REFERENCES 1.Al Fehaily M, Duh QY: Clinical manifestation of aldosteronoma. Surg Clin North Am 84:887-905, 2004 2.Al-Hawary MM, Francis IR, Korobkin M: Non-invasive evaluation of the incidentally detected indeterminate adrenal mass. Best Pract Res Clin Endocrinol Metab 19:277-292, 2005 3.Mansmann G, Lau J, Balk E, Rothberg M, et al: The clinically inapparent adrenal mass: update in diagnosis and management. Endocr Rev 25:309-340, 2004 4.Thompson GB, Young WF Jr: Adrenal incidentaloma. Curr Opin Oncol 15:84-90, 2003 page 593 American College of Surgeons SESAP 13 Category 6 Item 28 Decrease in aldosterone level with salt loading (A) Aldosteroma (B) Idiopathic hyperaldosteronism (C) Both (D) Neither page 594 American College of Surgeons SESAP 13 Critique for Category 6 Item 28 Primary hyperaldosteronism is defined as hypertension in the presence of elevated plasma aldosterone and suppressed plasma renin activity. Aldosteronoma and idiopathic hyperaldosteronism (IHA) are the two major causes. Aldosteronomas account for approximately 60% of cases and occur more often in women and younger patients. Hypertension and hypokalemia tend to be more severe compared with IHA. Salt loading and postural testing do not change aldosterone levels in patients with aldosteronoma. Computed tomography (CT) will typically reveal a small (< 2 cm) solitary adrenal tumor. Adrenal vein sampling (AVS) will reveal unilateral hypersecretion of aldosterone. This status is reflected by a side-to-side cortisol corrected aldosterone ratio (comparing the two adrenals) > 4. Treatment for aldosteronoma is surgical resection. IHA accounts for approximately 30% of cases of primary hyperaldosteronism. It occurs more often in men and older patients. Hypertension and hypokalemia is less severe than with aldosteronoma. Salt loading leads to a decrease in aldosterone levels, whereas postural testing results in an increase in aldosterone levels in patients with IHA. CT reveals normal-appearing or bilaterally enlarged adrenal glands. AVS will show evidence of bilateral hypersecretion of aldosterone reflected in a side-to-side cortisol corrected aldosterone ratio < 3. Treatment involves medical management with potassiumsparing diuretics. Spironolactone is most effective. Pheochromocytomas typically show evidence of hyperintensity on T2 weighted magnetic resonance imaging. REFERENCES 1.Al Fehaily M, Duh QY: Clinical manifestation of aldosteronoma. Surg Clin North Am 84:887-905, 2004 2.Al-Hawary MM, Francis IR, Korobkin M: Non-invasive evaluation of the incidentally detected indeterminate adrenal mass. Best Pract Res Clin Endocrinol Metab 19:277-292, 2005 3.Mansmann G, Lau J, Balk E, Rothberg M, et al: The clinically inapparent adrenal mass: update in diagnosis and management. Endocr Rev 25:309-340, 2004 4.Thompson GB, Young WF Jr: Adrenal incidentaloma. Curr Opin Oncol 15:84-90, 2003 page 595 American College of Surgeons SESAP 13 Category 7 Item 1 A 45-year-old man is brought to the emergency department after a motor vehicle crash in which his vehicle was struck on the driver side at a high speed by another car. He did not lose consciousness or strike his head and remembers the crash clearly. Two large-bore peripheral intravenous lines are present and he received Ringer’s lactate solution (2 L) before arriving at the hospital. On arrival his Glasgow coma scale (GCS) score is 14. Blood pressure is 60 systolic, pulse is 140, respiratory rate is 24/min. Initial survey shows no neck tenderness, good breath sounds bilaterally, a very distended and tender abdomen with a palpable fracture of the left 10th rib laterally, and no pelvic tenderness or instability on lateral or anterior compression. There are no obvious extremity injuries. An anteroposterior chest x-ray shows normal lung fields and mediastinum. For further evaluation and management, this patient needs (A) diagnostic peritoneal lavage (B) focused assessment with sonography for trauma (FAST) (C) abdominal/pelvic computed tomographic (CT) scan (D) diagnostic laparoscopy (E) laparotomy page 596 American College of Surgeons SESAP 13 Critique for Category 7 Item 1 The management of trauma patients has been standardized according to the Advanced Trauma ® Life Support (ATLS ) program of the American College of Surgeons. A trauma victim will have the best chance for a good outcome if these principles are followed. Using modern diagnostic modalities effectively has resulted in a decrease in negative or unnecessary laparotomies in the case of abdominal trauma. Given a stable patient with abdominal trauma, helpful tests to evaluate whether a significant abdominal injury is present include a diagnostic peritoneal lavage or focused assessment with sonography for trauma (FAST). An abdominal/pelvic computed tomographic (CT) scan is easily available in most centers and will provide detailed information about injuries, especially those to solid organs and the retroperitoneum, that can guide further decision-making. There are, however, some circumstances in which the need for operation is so obvious that additional tests are unnecessary. Absolute indications for laparotomy include blunt abdominal trauma with persistent or significant hypotension despite adequate resuscitation, evidence of peritonitis, hypotension with a penetrating abdominal wound, gunshot wounds traversing the peritoneal cavity, and evisceration. In this case the patient remains in severe shock with hypotension and tachycardia despite the initial fluid challenge and has a very distended and tender abdomen. There are no extremity injuries or thoracic injuries to account for his hypotension. Immediate abdominal exploration to address and fix the problem will avoid unnecessary blood loss by delaying treatment. REFERENCES 1.American College of Surgeons: Abdominal trauma, in Advanced Trauma Life Support Program for Doctors, Student Course Manual, ed 7. Chicago, American College of Surgeons, 2004, pp 131-150 2.Feliciano DV: Abdominal vascular injury, in Moore EE, Feliciano DV, Mattox KL (eds): Trauma, ed 5. New York, McGraw-Hill, 2004, pp 755-777 page 597 American College of Surgeons SESAP 13 Category 7 Item 2 A worker in a nuclear power plant is brought to the emergency department after a radiation leak at the facility. He is estimated to have received a whole body exposure of 10 to 12 Gy. All of the following can be expected to occur EXCEPT (A) pancytopenia (B) severe nausea and vomiting (C) bloody diarrhea (D) collapse of vascular tone with extensive vasodilatation (E) acute renal failure page 598 American College of Surgeons SESAP 13 Critique for Category 7 Item 2 Acute radiation syndromes are characterized by whole body exposures to a relatively large external dose of radiation in a short time. Radioactive materials deposited inside the body seldom produce acute radiation syndromes. The radiation must be of a penetrating type such as gamma rays, neutrons, or high-energy x-rays. The characteristics of the syndromes are secondary to the radiosensitivity of human cell types. Leukocytes, hematopoetic cells, and gastrointestinal epithelium have a high radiosensitivity. Vascular endothelium, pulmonary epithelium, and hepatocytes have a relatively intermediate radiosensitivity. Myocytes and neural cells are relatively radioresistant. Full-body exposure to radiation is well described in three different syndromes. The hematopoetic syndrome (typical exposure 1 to 4 Gy) is characterized by pancytopenia and opportunistic infections. Bleeding with severe thrombocytopenia may also be present. The gastrointestinal syndrome (typical exposure 8 to 10 Gy) is characterized by the hematopoetic syndrome plus severe nausea, vomiting, cramping, and watery diarrhea. The symptoms have an acute onset. Approximately 5 days later the mucosa of the bowel separates and causes additional bloody diarrhea and massive fluid losses, resulting in volume depletion and electrolyte imbalance. This may progress to acute renal failure and death. The neurovascular syndrome (typical exposure 15 to 50 Gy) causes extreme nervousness and confusion, with severe nausea and vomiting. There is often a massive vasodilatory response. Partial recovery usually occurs within 6 hours. Deterioration and death then follow relatively rapidly. Mortality is 100% at 72 hours despite treatment. REFERENCES 1.Centers for Disease Control and Prevention; Emergency Preparedness and Response: Acute radiation syndrome: A fact sheet for physicians. 2005. Available at: www.bt.cdc.gov/radiation/arsphysicianfactsheet.asp 2.Wolf SE, Herndon DN: Burns and radiation injuries, in Moore EE, Feliciano DV, Mattox KL (eds): Trauma, ed 5. New York, McGraw-Hill, 2004, pp 1081-1093 page 599 American College of Surgeons SESAP 13 Category 7 Item 3 A 27-year-old refinery worker is brought to the emergency department after a large natural gas explosion at the plant. The best screening tool for diagnosing a primary blast injury in this patient is (A) chest x-ray (B) otoscopic examination (C) fundoscopic examination (D) focused assessment with sonography for trauma (FAST) (E) chest auscultation page 600 American College of Surgeons SESAP 13 Critique for Category 7 Item 3 Primary blast injury is caused by the shock wave from the explosion. The greatest threat to life is from air embolism, pulmonary contusions, and diffuse parenchymal hemorrhages. These severe injuries are not always apparent on initial evaluation and may not be manifested clinically for 12 to 24 hours. Because physical and radiographic findings are not specific, examining the tympanic membranes for hemorrhage or perforation is a useful screening tool. The eardrum is usually the organ most susceptible to blast effect. A patient without a tympanic membrane rupture is highly unlikely to have significant blast injury to the thorax. The high sensitivity of this examination makes it an ideal screening test for significant blast injury in a mass casualty situation. It is less useful for those who were wearing ear protection. Despite its sensitivity as a screening test, tympanic membrane perforation is not very specific and does not always indicate a significant thorax injury. Patients who sustain an isolated eardrum perforation after an explosion can be safely discharged from the emergency department after chest x-rays and observation for 12 hours. Fundoscopic examination may be useful to identify retinal artery emboli, but this is much rarer than a tympanic membrane rupture. Although chest palpation and chest x-ray are indicated, findings are nonspecific and may not be manifest for several hours. Pulse oximetry may be useful because unexplained low saturation may be the first clinical manifestation of pulmonary injury. Focused assessment with sonography for trauma (FAST) is useful to assess for fluid in the abdominal cavity. In an explosion, solid organ injuries are much more likely to result from secondary blast injury (the patient being thrown against an object with force) or tertiary blast injury (the patient being hit with loose material driven by the blast). Solid organ injury is rare with primary blast injury. REFERENCES 1.Champion HR, Ochsner MG, Bellamy R: Surgical care of victims of conflict, in Moore EE, Feliciano DV, Mattox KL (eds): Trauma, ed 5. New York, McGraw-Hill, 2003, pp 1161-1181 2.Leibovici D, Gofrit ON, Shapira SC: Eardrum perforation in explosion survivors: is it a marker of pulmonary blast injury? Ann Emerg Med 34:168-172, 1999 page 601 American College of Surgeons SESAP 13 Category 7 Item 4 Which of the following statements about lower extremity trauma is TRUE? (A) Replantation of the severed lower extremity should be attempted in most patients if warm ischemia time is < 8 hours (B) The mangled extremity severity system (MESS) score reliably predicts long-term functional outcome (C) Primary amputation should not be undertaken in the setting of intact arterial pulses (D) Current scoring systems of limb injury severity are unreliable in predicting the need for amputation (E) The energy requirement for ambulation with a prosthesis is higher for ankle amputation than for above-knee amputation page 602 American College of Surgeons SESAP 13 Critique for Category 7 Item 4 The treatment of the severely injured extremity is a challenging clinical scenario. The objective of treatment is to restore optimal extremity function while avoiding unnecessary surgical procedures. Accurately identifying the limb that should be primarily amputated from one that should be salvaged or replanted is often impossible. Replantation of the severed limb may be technically feasible in many patients, but should only be attempted in highly selected cases. Significant factors to be considered include the age of the patient; the presence of co-morbidities such as diabetes, smoking, and hypertension; the severity of associated injuries; psychosocial factors; the nature of the injury (sharply severed limb versus avulsion, upper versus lower limb); and the patient’s overall hemodynamic stability. These factors are more important than the time from injury. During the past two decades, a variety of scoring systems have been proposed to guide clinical practice and predict long-term functional outcome after major extremity trauma. The most wellknown scoring system was introduced by Johansen in 1990. The mangled extremity scoring system (MESS) score can be used to classify the severity of extremity trauma based on scoring skeletal and soft tissue damage, vascular injury, presence and severity of shock, and patient age (see table) Although there is considerable debate about the strengths and weaknesses of MESS and other extremity scoring systems, most experts agree that no single scoring system reliably predicts long-term functional outcome in an individual patient. The decision to proceed with primary amputation should be based on the surgeon’s clinical judgment based on training and experience. Primary amputation of the limb with intact pulses may be appropriate in the setting of extensive soft tissue, bony, and neurologic damage, particularly in the presence of major injury to the trunk. In general, the higher the level of amputation, the greater the energy requirement for ambulation. Many elderly patients lack the cardiopulmonary reserve to use a prosthesis even if the amputation heals well. REFERENCES 1.Battiston B, Tos P, Pontini I, Ferrero S: Lower limb replantations: indications and a new scoring system. Microsurgery 22:187-192, 2002 2.Johansen K, Daines M, Howey T, Helfet D, et al: Objective criteria accurately predict amputation following lower extremity trauma. J Trauma 30:568-572, 1990 3.Owings JT, Kennedy JP, Blaisdell FW: Injuries to the extremeties, in Souba WW, Fink MP, Jurkovich GJ, Kaiser LR, et al (eds): ACS Surgery: Principles and Practice 2006. New York, WebMD Inc, Trauma and Thermal Injury, sec 12, chap 7, pp1280-1294 4.Schmidhammer R, Nimmervoll R, Pelinka LE, Huber W, et al: Bilateral lower leg replantation versus prosthetic replacement: long-term outcome of amputation after an occupational railroad accident. J Trauma 57:824-831, 2004 5.Trafton PG, McGough RL: Lower extremity fractures and dislocations, in Moore EE, Feliciano DV, Mattox KL (eds): Trauma, ed 5. New York, McGraw-Hill, 2004, pp 939-968 page 603 pulseless foot after external fixation of a femur fracture page 604 .American College of Surgeons SESAP 13 Category 7 Item 5 Nonoperative management is MOST appropriate for (A) a 6-cm pseudoaneurysm of the common femoral artery after cardiac catheterization (B) intimal flap in the popliteal artery after posterior knee dislocation that is not flow-limiting (C) pulsatile bleeding from a stab wound to the thigh in a hemodynamically stable patient (D) large traumatic arteriovenous fistula between the superficial femoral artery and vein from a low-velocity gunshot wound (E) cool. American College of Surgeons SESAP 13 Critique for Category 7 Item 5 Increasingly, nonoperative management has been successfully applied to selected traumatic vascular injuries. In particular, minor intimal injuries that involve less than 50% of the arterial diameter and are not flow limiting usually heal spontaneously (image 1). On the other hand, most surgeons agree that intervention is appropriate for most arterial injuries, including occlusions resulting in ischemia, persistent hemorrhage, arteriovenous fistula, and large pseudoaneurysms. Arterial spasm (image 2) is frequently observed in arteries adjacent to traumatic soft tissue injuries and typically does not require specific treatment. Most traumatic arteriovenous fistulas (image 3) should be treated either by coil embolization or surgical ligation because of their tendency to enlarge with time. Arterial occlusion (image 4) usually warrants surgical intervention unless the patient’s overall condition precludes surgical intervention or redundant circulation to the affected part is believed to be sufficient to eliminate ischemia. Extravasation of contrast (image 5) during angiography indicates ongoing hemorrhage requiring definitive treatment, either in the form of catheter-based intervention or surgical repair. While small (< 2 cm) posttraumatic, eg, after cardiac catheterization, pseudoaneurysms can be safely observed in anticipation of spontaneous thrombosis, larger pseudoaneurysms require treatment. Current treatment options for iatrogenic pseudoaneurysm include ultrasound-guided compression, percutaneous thrombin injection, and conventional surgical repair. Image 6 demonstrates a large pseudoaneurysm arising from the profunda femoris artery after a stab wound to the groin. One coil has already been placed. REFERENCES 1.Dennis JW, Frykberg ER, Veldenz HC, Huffman S, et al: Validation of nonoperative management of occult vascular injuries and accuracy of physical examination alone in penetrating extremity trauma: 5- to 10-year follow-up. J Trauma 44:243-252, 1998 2.Perry MO, Bongard FS: Vascular trauma, in Moore WS (ed): Vascular Surgery: A Comprehensive Review, ed 6. Philadelphia, WB Saunders, 2002, pp 677-696 3.Rowe VL, Yellin AE, Weaver FA: Vascular injuries of the extremities, in Rutherford RB (ed): Vascular Surgery, ed 6. Philadelphia, Elsevier Saunders, 2005, pp 1044-1057 page 605 American College of Surgeons SESAP 13 Category 7 Item 6 A 40-year-old male unrestrained driver is brought to the emergency department after being ejected from his car during a motor vehicle crash.. Four hours after injury, he is in class IV hemorrhagic shock with a Glasgow coma scale score of 14. Core temperature is 34°C, and he has a base deficit of -18. After primary and secondary survey, his only injury is an isolated severely comminuted open distal femur fracture Focused assessment with sonography for trauma (FAST) is negative, but he has active bleeding from a crush injury to the mid thigh. The patient is taken to the operating room for ongoing arterial and venous bleeding from his thigh wound. Despite ongoing massive resuscitation, he remains in shock and is hypothermic (temperature < 34°C), acidotic (pH < 7.20), and coagulopathic (INR > 1.5). The most appropriate choice in management now would be (A) above-knee amputation of the right lower extremity (B) angiography of the right lower extremity followed by external fixation of the right tibia and femur (C) right below-knee amputation with exploration of the femoral vessels and repair followed by external fixation of both femurs (D) immediate ligation of the superficial femoral artery followed by external fixation of the fractures (E) application of a tourniquet to the right thigh and delayed vascular reconstruction page 606 American College of Surgeons SESAP 13 Critique for Category 7 Item 6 One of the most challenging decisions in the care of an injured patient is whether to attempt salvage of a severely injured limb. Whenever a decision between limb salvage and amputation must be made, the two primary concerns are the systemic consequences of either alternative for the patient, and the likelihood of achieving a functional limb versus the problems associated with limb salvage (time involved, duration of disability, medical risks, socioeconomic costs, number of operations and hospitalizations, etc). The absolute indication for amputation in trauma remains an ischemic limb with unreconstructable vascular injury in a patient with hypothermia, metabolic acidosis, and coagulopathy. Massively crushed muscle and ischemic tissue can release myoglobin and cytokines, leading to a systemic inflammatory response syndrome, resulting in renal failure, adult respiratory distress syndrome, and even death. Prolonged attempts at limb salvage, even though technically possible, may endanger the patient’s life and generally should be avoided. Damage control operation for an extremity injury is appropriate when exsanguination has caused intraoperative metabolic failure; when multisystem injuries have occurred and emergent craniotomy, thoracotomy, or laparotomy is required in addition to vascular repair of the extremity (occlusion of superficial femoral artery from a femur fracture); or when the instability of an open fracture precludes formal repair of the associated vascular injury. In all patients, the decision about whether to undertake immediate or early amputation of a mangled limb should also take into consideration whether it is an upper extremity or lower extremity. When amputation is required, proceeding promptly decreases morbidity and mortality. If a limb is severely injured, it is rare that salvage will completely restore function. The severity and duration of shock, the severity of other injuries (as measured by the injury severity score, ISS), the patient's age, and pre-existing medical conditions are crucial variables. Important features of the extremity injury include duration of ischemia, causative mechanism, fracture pattern, location of vascular injury, neurologic status (particularly anatomic integrity of the tibial nerve), condition of the foot, and extent of any intercalary ischemic zone after revascularization. The patient's occupation and subjective desires also merit consideration. REFERENCES 1.Dirschl DR, Dahners LE: The mangled extremity: when should it be amputated? J Am Acad Orthop Surg 4:182-190, 1996 2.Trafton PG, McGough RL: Lower extremity fractures and dislocations, in Moore EE, Feliciano DV, Mattox KL (eds): Trauma, ed 5. New York, McGraw-Hill, 2004, pp 939-968 page 607 American College of Surgeons SESAP 13 Category 7 Item 7 In the management of traumatic hemothorax, the MOST significant risk factor for the development of empyema is (A) associated pneumothorax with air leak (B) retained hemothorax (C) tube thoracostomy in the emergency department (D) endotracheal intubation (E) multiple rib fractures page 608 American College of Surgeons SESAP 13 Critique for Category 7 Item 7 Thoracic trauma accounts for 25% of mortalities after injury. Many of these injuries can be managed with tube thoracostomy. Retained hemothorax remains the single most important risk factor for development of empyema. Retained hemothorax can be easily cleared by video-assisted thoracoscopy within the first few days after injury. Other less significant risk factors for the development of empyema include emergent (in the emergency department) insertion of the chest tube and insertion by nonsurgeons. Endotracheal intubation and multiple rib fractures are risk factors for development of pneumonia, but are not associated with the development of empyema. REFERENCES 1.Coselli JS, Mattox KL, Beall AC Jr: Reevaluation of early evacuation of clotted hemothorax. Am J Surg 148:786-790, 1984 2.Etoch SW, Bar-Natan MF, Miller FB, Richardson JD: Tube thoracostomy. Factors related to complications. Arch Surg 130:521-525, 1995 3.Shapiro ML, Angood PB: Patient safety, errors, and complications in surgery, in Brunicardi FC, Andersen DK, Billiar TR, Dunn DL, et al (eds): Schwartz’s Principles of Surgery, ed 8. New York, McGraw-Hill, 2005, pp 333-360 page 609 American College of Surgeons SESAP 13 Category 7 Item 8 A 24-year-old man who was the restrained driver in a high-speed motor vehicle crash arrives in the emergency department with a systolic blood pressure of 80 and heart rate of 120. After infusion of 2 L of saline, blood pressure normalizes and the heart rate is 80. His only complaint is of pain in the left upper quadrant of the abdomen. The abdominal computed tomographic (CT) scan shown is obtained. Eighteen hours after admission, systolic blood pressure is 80, heart rate is 120, and hemoglobin is 6 g/dL. The most appropriate management now would be (A) infusion of factor VIIa (B) laparotomy (C) repeat CT scan (D) angiography with embolization (E) transfusion of 2 units of packed RBCs and continued observation page 610 American College of Surgeons SESAP 13 Critique for Category 7 Item 8 Although nonoperative management is possible in many patients with splenic injury, emergency operation is still sometimes necessary to stop life-threatening hemorrhage. Determining which patients require emergency operation and which can be initially managed nonoperatively is sometimes quite difficult. Making the right decision is critically important, and it should never be forgotten that patients with splenic injury can bleed to death. The patient’s hemodynamic stability is of paramount importance in determining whether nonoperative management is appropriate. There is no consensus definition for the somewhat illusory concept of what consitutes hypotension, but systolic blood pressure < 90 mm Hg in an adult is generally considered to be worthy of concern. Prehospital or emergency department hypotension is worrisome, and a high index of suspicion for ongoing hemorrhage should be maintained when either is present. Patients who have been hemodynamically abnormal in the prehospital phase and remain so during their initial stay in the emergency department usually require either a direct trip to the operating room, or more commonly, abdominal ultrasonography or diagnostic peritoneal lavage to help guide the initial decision-making process. Reported success rates for nonoperative management are 95% or higher for pediatric patients and approximately 80% or higher in adults. Nonoperative management is reasonable for patients stable enough to undergo computed tomographic (CT) scanning if they continue to remain stable. In addition to vital signs, hematocrit is another commonly followed parameter. A common practice is to determine a cut-off value, with operation if the hematocrit drops to that level or below. Such an approach works best if there are no associated injuries, because determining the source of the bleeding responsible for the decrease is difficult. The Eastern Association for the Surgery of Trauma (EAST) Multiinstitutional Workgroup reported a failure rate of 10.8% for nonoperative management of blunt splenic injury in adults. Sixty percent of the failures occurred within 24 hours of admission. Inappropriate selection was responsible for failure of nonoperative management in 30% to 40% of patients, ie, hemodynamic instability or initial misinterpretation of diagnostic studies, including active extravasation. Most deaths were from delayed treatment of intra-abdominal injuries. Nonoperative management has been unsuccessful and this patient is in class IV hemorrhagic shock. The most likely source of hemorrhage is the grade III splenic injury visualized on the CT scan. A repeat CT scan will only prolong the time until hemostasis is achieved. Angiography with embolization improves splenic salvage if a pseudoaneursym is demonstrated on the initial CT scan, or a subsequent scan performed within the first 48 hours, but is indicated only in hemodynamically stable patients. Although transfusion alone may be appropriate if the patient is anemic (hemoglobin < 8 g/dL), observation is not appropriate because of the ongoing bleeding and shock. Recombinant factor VII is only indicated in patients with massive hemorrhage and coagulopathy at the time of operative intervention. The table lists CT grading for spleen injuries. REFERENCES 1.Harbrecht, BG, Peitzman AB, Rivera L, Heil B, et al: Contribution of age and gender to outcome of blunt splenic injury in adults: multicenter study of the Eastern Association for the Surgery of Trauma. J Trauma 51:887-895, 2001 page 611 American College of Surgeons SESAP 13 2.Peitzman AB, Harbrecht BG, Rivera L, Heil B, et al: Failure of observation of blunt splenic injury in adults: variability in practice and adverse consequences. J Am Coll Surg 201:179-187, 2005 3.Peitzman AB, Heil B, Rivera L, Federle MB, et al: Blunt splenic injury in adults: multi-institutional study of the Eastern Association for the Surgery of Trauma. J Trauma 49:177-187, 2000 4.Velmahos GC, Chan LS, Kamel E, Murray JA, et al: Nonoperative management of splenic injuries: have we gone too far? Arch Surg 135:674-679, 2000 5.Wisner DH: Injury to the spleen, in Moore EE, Feliciano DV, Mattox KL (eds): Trauma, ed 5. New York, McGraw-Hill, 2004, pp 663-686 page 612 American College of Surgeons SESAP 13 Category 7 Item 9 A 35-year-old man develops a biloma after blunt liver trauma. An initial attempt at computed tomographic (CT)- guided percutaneous drainage of the biloma is unsuccessful. The next step in management of this patient should be (A) exploratory laparotomy with debridement of necrotic parenchyma and omentoplasty (B) a second drain; somatostatin (C) laparoscopy with drainage (D) drainage and endoscopic retrograde cholangiopancreatography (ERCP) with sphincterotomy (E) exploratory laparotomy and hepaticojejunostomy page 613 American College of Surgeons SESAP 13 Critique for Category 7 Item 9 Bilomas are an uncommon complication of hepatic trauma. Intrahepatic biliary ducts are invariably injured along with the hepatic parenchyma at the time of trauma, but most of these injuries heal without sequelae. Bilomas can form weeks to months after the initial trauma as the slow progressive leakage of bile into the traumatized tissue gradually forms a collection. Patients may be asymptomatic or present with right upper quadrant abdominal pain or fullness, fever, or jaundice. Patients with high-grade liver injuries have a higher risk of biloma formation. In a review of 281 patients who developed bilomas after blunt liver injury comparing patients treated nonoperatively (angiographic embolization or observation) with those undergoing operation, all bile leaks occurred in patients with a liver abbreviated injury score (AIS) of > 4. For survivors of more than 4 days, bile leaks developed in 71% of those undergoing operation, 50% of those managed with angiographic embolization, and 17% of those in the observation group. Biliary excretion, eg, HIDA, scanning detected all leaks, leading to earlier drainage procedures. On computed tomography (CT), bilomas appear as round, well-circumscribed collections of lowdensity fluid. Their appearance is similar to that seen with hepatic abscess or resolving hematoma, and differentiating these three entities on CT is difficult. Percutaneous needle aspiration or Tc-99m scintigraphy can confirm the diagnosis. Symptomatic patients can be treated with a combination of percutaneous drain placement and endoscopic retrograde biliary stent placement. Endoscopic retrograde cholangiopancreatography (ERCP) with sphincterotomy and stent insertion reduces the biliary ductal pressure, shortening the time for closure of the biliary leak and resolution of the biloma. Exploration with debridement and omentoplasty is reserved for patients in whom ERCP and sphincterotomy is unsuccessful. Hepaticojejunostomy is used for management of an extrahepatic bile duct injury, but is not indicated in this setting. Laparoscopic insertion of a drain has been described for early management of grade III and IV liver lacerations in hemodynamically stable patients, not for the management of biloma. Somatostatin therapy reduces the incidence of septic morbidity and anastomotic leak after pancreatic resection, but would not be efficacious for the treatment of a biloma. REFERENCES 1.Carrillo EH, Richardson JD: The current management of hepatic trauma. Adv Surg 35:39-59, 2001 2.Carrillo EH, Wohltmann C, Richardson JD, Polk HC Jr: Evolution in the treatment of complex blunt liver injuries. Curr Probl Surg 38:1-60, 2001 3.Christmas AB, Wilson AK, Manning B, Franklin GA, et al: Selective management of blunt hepatic injuries including nonoperative management is a safe and effective strategy. Surgery 138:606-610, 2005 4.Richardson JD, Franklin GA, Lukan JK, Carrillo EH, et al: Evolution in the management of hepatic trauma: a 25-year perspective. Ann Surg 232:324-330, 2000 5.Wahl WL, Brandt MM, Hemmila MR, Arbabi S: Diagnosis and management of bile leaks after blunt liver injury. Surgery 138:742-747, 2005 page 614 American College of Surgeons SESAP 13 Category 7 Item 10 The most appropriate initial management for a posterior hip dislocation would be (A) computed tomography (CT) to evaluate for intra-articular fragments prior to reduction (B) conscious sedation with closed reduction as soon as possible (C) placement of a traction splint (D) bed rest and application of skeletal traction to reduce the dislocation (E) immediate operative reduction page 615 American College of Surgeons SESAP 13 Critique for Category 7 Item 10 Hip dislocation is often caused by a force applied to the femur and is most commonly associated with fractures of the acetabulum or femoral head. Motor vehicles crashes are a common mechanism. Force applied to an abducted hip can result in anterior dislocation, whereas striking the knee on the dashboard with the hip flexed and adducted results in posterior dislocations. Posterior dislocations are often associated with fractures of the posterior wall of the acetabulum. Long-term complications include osteonecrosis, formation of heterotopic ossification, and posttraumatic arthritis. Most sciatic nerve injuries are neurapraxias. Good function can be achieved in 60% to 80% of patients, although full recovery of strength is unusual. Once the hip has been completely dislocated posteriorly, it may appear shortened and internally rotated. Sometimes femoral head or acetabular fractures are associated with this dislocation. Sciatic nerve injuries are present in up to 15% of posterior hip dislocations. Closed reduction is usually accomplished by longitudinal traction, followed by gentle abduction and external rotation. Stability of the reduction should be determined at the time of closed reduction. Open reduction and internal fixation is indicated only if the reduction is unstable and associated with a posterior wall fracture. Computed tomography (CT) provides little useful information prior to reduction. After reduction, postreduction CT scan is indicated to rule out the presence of intra-articular fragments. The timing of reduction influences the incidence of late osteonecrosis of the femoral head, which ranges from 10% to 50% and may occur several years after the injury. Reduction is ideally achieved within the first few hours after injury, as soon as the patient's general status has been adequately assessed. Bed rest and skeletal traction unnecessarily delay immediate reduction of the dislocation. A Thomas or traction splint could not be properly applied given the hip dislocation. REFERENCES 1.Springfield D: Orthopaedics, in Brunicardi FC, Andersen DK, Billiar TR, Dunn DL, et al (eds): Schwartz’s Principles of Surgery, ed 8. New York, McGraw-Hill, 2005, pp 1653-1720 2.Tay BB, Collman WW, Berven S, Fontes R Jr, et al: Orthopedics, in Doherty GM (ed): Current Surgical Diagnosis & Treatment, ed 12. New York, McGraw-Hill, 2006, pp 1097-1208 3.Trafton PG, McGough RL: Lower extremity fractures and dislocations, in Moore EE, Feliciano DV, Mattox KL (eds): Trauma, ed 5. New York, McGraw-Hill, 2004, pp 939-968 page 616 American College of Surgeons SESAP 13 Category 7 Item 11 A 70-year-old man involved in a motor vehicle crash in a rural area arrives at a trauma center 10 hours later. At the trauma center, he can flex but not extend his elbows. He has received only morphine and intravenous fluids since the crash. Which of the following statements about high-dose methylprednisolone administration for treatment of spinal cord injury in this patient is TRUE? (A) Unlikely to improve neurologic function (B) Likely to improve motor function if given for a total of 48 hours (C) Likely to improve sensory function if given for a total of 48 hours (D) Likely to improve motor and sensory function if given for a total of 72 hours (E) Does not result in increased infectious complications if duration is limited to 48 hours page 617 American College of Surgeons SESAP 13 Critique for Category 7 Item 11 Spinal cord injury affects 40 million people worldwide each year. Most are males (male:female ratio 4:1) and are typically between the ages of 20 and 35. The resulting permanent paralysis and major disability are a disaster for the patient and for our healthcare systems. The physical injury is due to contusion, compression, or stretch, but in most cases not transection of the spinal cord. Secondary injury or degeneration is a complex multifactorial process that begins to occur rapidly after the primary mechanical injury. This secondary injury is characterized biochemically by mitochondrial dysfunction, cyclooxygenase, lipoxygenase, and nitric oxide synthase activity, leading to generation of reactive oxygen species and causing cellular damage via lipid peroxidation of cell membranes. The National Acute Spinal Cord Injury Studies (NASCIS) have examined the use of methylprednisolone to lessen secondary injury and improve neurologic recovery after spinal cord injury. Results and conclusions have been controversial because of study design and data analysis. NASCIS I began in 1979 and compared two daily bolus dosing regimens of methylprednisolone, each administered for 10 days after injury. No benefit was seen for the higher dose regimen. No placebo control group was included. The NASCIS II trial showed that a 24-hour regimen of high-dose methylprednisolone (30 mg/kg intravenous bolus followed by 5.4 mg/kg infusion for 23 hours) slightly improved neurologic function below the level of injury in both neurologically complete and incomplete injuries. However, this dosing regimen was only effective when begun within 8 hours of injury, and had a detrimental effect on neurologic recovery if begun after the 8-hour window. This was the only NASCIS trial to include a placebo group. NASCIS III compared this 24-hour regimen from NASCIS II with a similar regimen in which the infusion was given for 48 hours, and also looked at whether starting treatment within 3 hours of injury was more effective than within 3 to 8 hours. The study found that when treatment began within 3 hours, the 24-hour dosing regimen was as effective as the 48-hour regimen. However, when treatment was begun 3 to 8 hours after injury, the 48-hour regimen was superior to the 24-hour regimen. The 48hour regimen also resulted in increases in pneumonia and possibly sepsis, whereas treatment for 24 hours did not. A 72-hour regimen has not been studied. The NASCIS trials have been criticized for aspects of trial design, analysis, and reporting. Although there is no real consensus among neurosurgeons regarding use of methylprednisolone in the treatment of spinal cord injury, the table likely represents the most widely held opinion. REFERENCES 1.Bracken MB: Steroids for acute spinal cord injury. Cochrane Database Syst Rev (3):CD001046, 2002 2.Hall ED, Springer JE: Neuroprotection and acute spinal cord injury: a reappraisal.NeuroRx 1:80-100, 2004 page 618 American College of Surgeons SESAP 13 Category 7 Item 12 After falling from a ladder, a 25-year-old man is admitted with a Glasgow coma scale (GCS) score of 3 and reactive pupils. Head computed tomography (CT) shows diffuse cerebral edema. The patient is in the ICU, intubated and receiving mechanical ventilatory support. A ventriculostomy has been placed and intracranial pressure is 32 mm Hg. Mean arterial pressure (MAP) is 65 mm Hg. Pupils are reactive. Appropriate steps include all of the following EXCEPT (A) chemical paralysis (B) pressors to increase MAP (C) increased analgesia and sedation (D) hyperventilation to PCO (E) administration of mannitol 2 25 torr page 619 American College of Surgeons SESAP 13 Critique for Category 7 Item 12 Increased intracranial pressure (ICP) after traumatic brain injury (TBI) can exacerbate neurologic damage. The fixed volume within the cranial vault contains brain tissue, blood, and cerebrospinal fluid, all relatively noncompressible. Trauma causes the ICP to rise because of bleeding and cerebral edema. Increased ICP damages brain tissue both directly and through decreases in cerebral perfusion pressure (CPP, defined as CPP = MAP - ICP), causing cerebral ischemia. Increased ICP can also lead to one of several herniation syndromes. Severe TBI is indicated by an admission Glasgow coma scale (GCS) score of 8 or less. Initial management involves securing the airway, resuscitation per ATLS® protocols, and head computed tomography (CT) to search for bleeding that might require surgical decompression. Controlling hypotension and hypoxia is doubly important in the setting of severe TBI so as to minimize secondary brain injury. ICP monitoring is usually necessary and the preferred technique is via intraventricular catheter, which also permits some control of ICP via drainage of cerebrospinal fluid. Therapy to decrease ICP (and maximize CPP) should be initiated for ICP of 20 or over. A stepwise approach to treating elevated ICP begins with drainage of CSF if an intraventricular catheter is present. Sedation and analgesia are recommended to decrease the metabolic demands of the nervous system. Propofol is commonly used because it allows rapid withdrawal of sedation for examination, but morphine or fentanyl and lorazepam or midazolam are acceptable alternatives. Chemical paralysis is next, with the downside that it makes further neurologic examination meaningless, although pupillary responses are maintained. Therapy also includes elevating the head of the bed 35 to 40 degrees with the neck in a neutral position. Mannitol has classically been recommended as the next step in control of ICP. The osmotic diuresis caused by mannitol causes serum osmolarity to increase, drawing fluid across the bloodbrain barrier into the circulation. Serum osmolarity should not be allowed to exceed 320 mOsm/L to avoid renal compromise. Hypertonic saline is increasingly believed to be as effective as, or possibly even more effective than, mannitol in increasing serum osmolarity, and its use is associated with increases in intravascular volume, as opposed to the decrease seen with mannitol. Both agents also work to enhance cerebral circulation via decreased blood viscosity. Hyperventilation to PCO 2 35 mm Hg or at least avoidance of hypercapnea is believed to be beneficial in causing mild cerebral vasospasm and thus decreasing the volume that blood occupies in the cranial vault. Although more aggressive hyperventilation is sometimes recommended as an initial and immediately available step to avoid herniation, most authors agree that hyperventilation to levels below 30 mm Hg as a prophylactic measure is dangerous because it may markedly decrease cerebral blood flow and cause cerebral ischemia. Further measures to decrease ICP may include barbiturate coma or even decompressive craniectomy. Throughout all of these manipulations, CPP should be maintained at > 60 mm Hg. Pressors may be required in volume-resuscitated patients with intracranial hypertension and CPP < 60 mm Hg. REFERENCES 1.[No Authors Listed]: The Brain Trauma Foundation. The American Association of Neurological Surgeons. The Joint Section on Neurotrauma and Critical Care. Hyperventilation. J Neurotrauma 17:513-520, 2000 page 620 American College of Surgeons SESAP 13 2.[No Authors Listed]: The Brain Trauma Foundation. The American Association of Neurological Surgeons. The Joint Section on Neurotrauma and Critical Care. Use of mannitol. J Neurotrauma 17:521-525, 2000 3.Vincent JL, Berre J: Primer on medical management of severe brain injury.Crit Care Med 33:13921399, 2005 4.Zwienenberg-Lee M, Kim KD, Muizelaar JP: Injuries to the central nervous system, in Souba WW, Fink MP, Jurkovich GJ, Kaiser LR, et al (eds): ACS Surgery: Principles and Practice 2006. New York, WebMD Inc, Trauma and Thermal Injury, sec 2, chap 7, pp 1145-1161 page 621 American College of Surgeons SESAP 13 Category 7 Item 13 A 62-year-old woman sustained head, chest, and abdominal injuries as an unrestrained passenger in a high-speed motor vehicle crash. She is hypotensive and focused assessment with sonography for trauma (FAST) is abnormal. At laparotomy, a blunt hepatic injury of the right lobe is packed because of active bleeding. Bleeding continues to arise from the retroperitoneum, localized to the right renal hilum. A left kidney is palpable. The patient is intermittently hypotensive despite ongoing blood and crystalloid replacement. The most appropriate management of the renal injury would be (A) right nephrectomy (B) primary vascular repair (C) retroperitoneal packing; renal artery embolization (D) damage control laparotomy (E) vascular repair with autologous vein bypass graft page 622 American College of Surgeons SESAP 13 Critique for Category 7 Item 13 Major renal vascular injuries are uncommon. In a review of 523,870 patients hospitalized for trauma in 1997-1998, 6231 patients (1.2%) had renal injuries (4.89 per 100,000 population), and of these only 4% (n=249) had renal vascular injuries. Surgical management of renal injuries was required in 11% of patients (n=685) and nephrectomy in 7% of patients (n=436). Injury severity score, mechanism, and renal injury severity were independent predictors of nephrectomy. Major renal vascular injuries are frequently associated with poor outcome, including renal dysfunction, posttraumatic renovascular hypertension, and renal failure. As described by the AAST Organ Injury Severity Scales (table), grade V renal injury includes completely shattered kidney or avulsion of renal hilum that devascularizes kidney. A multicenter retrospective review of grade IV/V renal injuries (n=89) documented that patients with blunt injuries were 2.29 times more likely to have a poor outcome compared with those who had penetrating injuries. Similarly, patients with grade V injuries were 2.2 times more likely to have a poor outcome. Patients with blunt major vascular injuries (grade V) are likely to have associated major parenchymal disruption contributing to poor function of the revascularized kidney, and these patients may be best served by immediate nephrectomy, provided that there is a functioning contralateral kidney (see algorithm). The major indication for nephrectomy in a patient with a major vascular renal injury is persistent shock despite ongoing fluid resuscitation or grade V renal injury in a normotensive patient. REFERENCES 1.Baverstock R, Simons R, McLoughlin M: Severe blunt renal trauma: A 7-year retrospective review from a provincial trauma centre. Can J Urol 8:1372-1376, 2001 2.Heyns CF: Renal trauma: indications for imaging and surgical exploration. BJU Int 93:1165-1170, 2004 3.Knudson MM, Harrison PB, Hoyt DB, Shatz DV, et al: Outcome after major renovascular injuries: a Western Trauma Association multicenter report. J Trauma 49:1116-1122, 2000 4.Santucci RA, Wessells H, Bartsch G, Descotes J, et al: Evaluation and management of renal injuries: consensus statement of the renal trauma subcommittee. BJU Int 93:937-954, 2004 5.Wessells H, Suh D, Porter JR, Rivara F, et al: Renal injury and operative management in the United States: results of a population-based study. J Trauma 54:423-430, 2003 page 623 American College of Surgeons SESAP 13 Category 7 Item 14 A 27-year-old woman sustains a blunt carotid injury. Carotid angiography demonstrates a carotid dissection beginning in the bulb and extending to the petrous portion. A small residual lumen is present. The optimal treatment would be (A) carotid artery stent placement (B) systemic anticoagulation with unfractionated heparin (C) carotid exploration (D) external carotid-internal carotid (ECIC) bypass (E) low molecular weight heparin (30 mg/kg/day) page 624 American College of Surgeons SESAP 13 Critique for Category 7 Item 14 Blunt carotid artery injuries are potentially devastating injuries due to neurologic complications. Early detection and treatment remain the goals of management. Most of these injuries occur secondary to hyperextension/rotation injuries sustained in high-impact motor vehicle crashes. A much smaller percentage are caused by a direct blow to the neck. Five different mechanisms for the development of traumatic carotid arery dissection have been described: direct trauma to the artery, hyperextension-rotation of the head with stretching of supra-aortic vessels, blunt intra-oral trauma, basal skull or mandibular fractures, and combined chest-head injuries with carotid stretching. The most common mechanism is a direct blow to the head with hyperextension-rotation of the neck, which causes stretching of the internal carotid artery over the transverse processes of the first and second cervical vertebrae. Carotid artery dissection typically causes headache or neck ache, followed after hours to days by focal motor or sensory deficits. Undiagnosed blunt carotid artery injuries may have a delayed presentation varying from several days to over 1 year. The identification of asymptomatic lesions is critical because of the high incidence of subsequent associated neurologic deficits. Although carotid four-vessel angiography remains the gold standard, other less invasive diagnostic imaging modalities are also used. Early identification leading to early treatment significantly reduces stroke rates. Treatment options for patients with traumatic carotid artery dissection include observation, antithrombotic therapy, open surgical repair of the affected carotid artery, and endovascular carotid artery treatment. The mainstay of treatment for most blunt carotid injuries is systemic anticoagulation with intravenous unfractionated heparin, which has been shown to prevent progression of injury to a higher injury grade, to reduce the number of strokes, and to prevent neurologic deterioration. Low molecular weight heparin in a dose range used for venous thromboembolism prophylaxis would not be adequate anticoagulation therapy. Antiplatelet therapy alone (aspirin and/or clopidogrel) has been used for patients with contraindications to systemic anticoagulation. Deteriorating or fluctuating neurologic symptoms might be an indication for surgical or endovascular intervention. ECIC bypass is a surgical treatment option that is not indicated in this case. Follow-up diagnostic testing in 7 to 10 days has been recommended because prior studies have reported that 65% of grade I injuries and 51% of grade II injuries evolved, requiring a change in treatment. Overall mortality resulting from posttraumatic carotid artery dissection varies from 5% to 40% with neurologic morbidity from 12% to 80%, with bilateral lesions being the most severe. Recanalization occurs in 75% of stenoses and in 25% to 40% of occlusions. Despite medical treatment, cases with transient carotid stenosis or occlusion had annual rates of 0.3% for ipsilateral carotid territory stroke and of 0.6% for any stroke. In those patients with permanent carotid stenosis or occlusion, annual rates of 0.7% for ipsilateral carotid territory stroke and 1.4% for any stroke have been reported. These results question the rationale of surgical or endovascular revascularization in patients with traumatic carotid artery dissection. REFERENCES 1.Cothren CC, Moore EE, Ray CE Jr, Ciesla DJ, et al: Carotid artery stents for blunt cerebrovascular page 625 American College of Surgeons SESAP 13 injury: risks exceed benefits. Arch Surg 140:480-485, 2005 2.Cothren CC, Moore EE, Biffl WL, Ciesla DJ, et al: Anticoagulation is the gold standard therapy for blunt carotid injuries to reduce stroke rate. Arch Surg 139:540-545, 2004 3.Kerby JD, May AK, Gomez CR, Rue LW III: Treatment of bilateral blunt carotid injury using percutaneous angioplasty and stenting: case report and review of the literature. J Trauma 49:784787, 2000 4.Singh RR, Barry MC, Ireland A, Bouchier Hayes D: Current diagnosis and management of blunt internal carotid artery injury. Eur J Vasc Endovasc Surg 27:577-584, 2004 5.Vishteh AG, Marciano FF, David CA, Schievink WI, et al: Long-term graft patency rates and clinical outcomes after revascularization for symptomatic traumatic internal carotid artery dissection. Neurosurgery 43:761-767, 1998 page 626 American College of Surgeons SESAP 13 Category 7 Item 15 A 68-year-old man sustained multiple injuries in a motor vehicle crash, including traumatic brain injury with multiple small frontal contusions and intraparenchymal hematomas noted on head computed tomographic (CT) scan. He has been receiving long-term anticoagulation with warfarin for chronic atrial fibrillation and prior stroke. His admission INR is 2.8. The most rapid method of reversing the anticoagulation effect would be intravenous administration of (A) recombinant factor VIIa (B) fresh frozen plasma (C) vitamin K (D) cryoprecipitate (E) thrombin concentrate page 627 American College of Surgeons SESAP 13 Critique for Category 7 Item 15 An increasing number of patients are receiving anticoagulants for conditions such as atrial fibrillation, valvular disease, and thromboembolic disease, which are more prevalent in the aging population. Patients receiving pre-injury anticoagulation therapy have a four- to fivefold higher risk of mortality from intracranial head injuries than patients without pre-injury anticoagulation, and this can occur even in patients who are initially asymptomatic with seemingly minor head trauma. A prospective cohort study reported a 48% mortality rate in warfarin anticoagulated trauma patients sustaining intracranial hemorrhage (ICH) compared with a 10% mortality rate in nonanticoagulated patients. ICH progressed in 40% of patients with a resultant 65% mortality rate. Prompt reversal of the pre-injury anticoagulation is necessary. Most trauma surgeons have used fresh frozen plasma (FFP) for reversal of anticoagulation in trauma. In a retrospective review evaluating the efficacy of FFP in reversing the anticoagulant effect in patients with warfarinassociated intracranial hemorrhage, median time to normalization of coagulation parameters was 32 hours in patients given FFP and vitamin K compared with 8.8 hours for those receiving recombinant factor VIIa (rFVIIa). Aggressive institutional protocols for reversal of anticoagulation with FFP and vitamin K have been shown to significantly reduce the anticoagulation reversal total time to 3.5 hours, but are a labor-intensive endeavor requiring the cooperation of many different departments. Several units of FFP take longer to infuse than a single bolus of rFVIIa. FFP requires checking for blood compatibility, requires thawing (30 minutes), and can induce hypervolemia compared with the rapid-onset action of rFVIIa, which may allow faster and safer correction of coagulopathy in traumatic brain injury. In a recent prospective study of 399 patients with intracerebral hemorrhage diagnosed by computed tomography (CT), patients were randomly assigned to receive placebo or rFVIIa to reduce hematoma expansion. Treatment with rFVIIA within 4 hours after the onset of intracerebral hemorrhage limited the growth of the hematoma, reduced mortality, and improved functional outcomes at 90 days. Prompt reversal of anticoagulation in patients with evidence of traumatic brain injury by CT scan imaging is warranted. Cryoprecipitate and thrombin concentrate are not indicated for warfarin reversal. REFERENCES 1.Brody DL, Aiyagari V, Shackleford AM, Diringer MN: Use of recombinant factor VIIa in patients with warfarin-associated intracranial hemorrhage. Neurocrit Care 2:263-267, 2005 2.Cohen DB, Rinker C, Wilberger JE: Traumatic brain injury in anticoagulated patients. J Trauma 60:553557, 2006 3.Coimbra R, Hoyt DB, Anjaria DJ, Potenza BM, et al: Reversal of anticoagulation in trauma: a North American survey on clinical practices among trauma surgeons. J Trauma 59:375-382, 2005 4.Ivascu FA, Howells GA, Junn FS, Bair HA, et al: Rapid warfarin reversal in anticoagulated patients with traumatic intracranial hemorrhage reduces hemorrhage progression and mortality. J Trauma 59:1131-1137, 2005 5.Karni A, Holtzman R, Bass T, Zorman G, et al: Traumatic head injury in the anticoagulated elderly patient: a lethal combination. Am Surg 67:1098-1100, 2001 page 628 American College of Surgeons SESAP 13 6.Lavoie A, Ratte S, Clas D, Demers J, et al: Preinjury warfarin use among elderly patients with closed head injuries in a trauma center. J Trauma 56:802-807, 2004 7.Mayer SA, Brun NC, Begtrup K, Broderick J, et al: Recombinant activated factor VIIa intracerebral hemorrhage trial investigators. Recombinant activated factor VII for acute intracerebral hemorrhage. N Engl J Med 352:777-785, 2005 page 629 American College of Surgeons SESAP 13 Category 7 Item 16 A 25-year-old woman was ejected from a vehicle in a high-speed crash. She is intubated and unresponsive, with a cervical collar in place. Her Glasgow coma scale (GCS) score is 3. She has multiple lower extremity fractures, right pulmonary contusion, and right humerus fracture. The diagnostic procedure of choice for radiologic clearance of the cervical spine for possible injury in this patient would be (A) lateral cervical spine x-ray (B) 5-view cervical spine x-rays (C) 3-view cervical spine x-rays (D) computed tomographic (CT) scan of occiput to T1 (E) magnetic resonance imaging (MRI) of cervical spine page 630 American College of Surgeons SESAP 13 Critique for Category 7 Item 16 Clearance of the cervical spine in patients with altered mental status or distracting injuries can be difficult. In a series of over 1000 patients, five-view (odontoid, anteroposterior, lateral, and oblique) plain films of the cervical spine for initial evaluation of blunt trauma patients with altered mental status failed to diagnose 52.3% of cervical spine fractures identified by computed tomographic (CT) scan of the occiput to T1, failed to diagnose 5 patients with unstable cervical fractures, and failed to identify 93.3% of patients with occipital condyle fractures. Additional studies have documented the superiority of CT scan imaging of the cervical spine with a sensitivity of 99% and specificity of 100%. The patient with altered mental status should undergo CT scan of the cervical spine from occiput to T1 in addition to head CT scan. Patients with focal neurologic signs, evidence of cord or disc injury, and patients requiring preoperative spinal cord assessment should undergo magnetic resonance imaging (MRI). The risk of relying solely on CT scan to clear the cervical spine is the possibility of ligamentous injury, which may be missed by both plain x-rays and CT scans. Evaluation for ligamentous injuries in patients with no evidence of injury on cervical spine CT scan should be accomplished by careful physical examination, supervised flexion/extension films under fluoroscopy by an experienced physician or radiologist, or MR imaging. Some studies have documented that MR imaging is superior to helical CT for screening of ligamentous cervical injuries, but all studies agree that helical CT is the most sensitive, specific, and cost-effective modality for screening the cervical spine for bony injuries. REFERENCES 1.Diaz JJ Jr, Aulino JM, Collier B, Roman C, et al: The early work-up for isolated ligamentous injury of the cervical spine: does computed tomography scan have a role? J Trauma 59:897-903, 2005 2.Diaz JJ Jr, Gillman C, Morris JA Jr, May AK, et al: Are five-view plain films of the cervical spine unreliable? A prospective evaluation in blunt trauma patients with altered mental status. J Trauma 55:658-663, 2003 3.Richards PJ: Cervical spine clearance: a review. Injury 36:248-269, 2005 4.Sanchez B, Waxman K, Jones T, Conner S, et al: Cervical spine clearance in blunt trauma: evaluation of a computed tomography-based protocol. J Trauma 59:179-183, 2005 5.Schuster R, Waxman K, Sanchez B, Becerra S, et al: Magnetic resonance imaging is not needed to clear cervical spines in blunt trauma patients with normal computed tomographic results and no motor deficits. Arch Surg 140:762-766, 2005 page 631 American College of Surgeons SESAP 13 Category 7 Item 17 Each of the following is believed to be an indication to screen for a blunt carotid injury EXCEPT (A) basilar skull fracture through the foramen lacerum (B) diffuse axonal injury (C) focal neurologic deficit not explained by head computed tomographic (CT) scan (D) direct blow to the neck (E) LeFort I facial fracture page 632 . The signs and symptoms of blunt carotid artery injury include arterial hemorrhage. Offner PJ. Horner syndrome. Ryu RK.Cothren CC. Ann Surg 228:462-470. near-hanging with anoxic brain injury.Biffl WL. J Trauma 51:279-285. Blunt carotid injuries are associated with a high risk of ischemic neurologic complications. blunt carotid injuries are now diagnosed in approximately 1% of patients who sustain blunt trauma. expanding cervical hematoma. Arch Surg 139:540-545. 2004 3. Fabian TC. Ciesla DJ. diffuse axonal injury. et al: Anticoagulation is the gold standard therapy for blunt carotid injuries to reduce stroke rate. et al: The unrecognized epidemic of blunt carotid arterial injuries: early diagnosis improves neurologic outcome. and direct blows to the neck are considered to be risk factors for blunt carotid injury. Increased recognition is the result of aggressive screening programs at large trauma centers.Miller PR. REFERENCES 1. Timmons S. Cerebral arteriography is considered to be the gold standard for screening patients at risk for blunt carotid injuries. basilar skull fracture with carotid canal involvement. Moore EE. Le Fort I fractures have not been associated with blunt carotid injuries. rapid identification is extremely important because appropriate treatment appears to prevent stroke. Le Fort II and III fractures. Moore EE. 2001 page 633 . 1998 2. and focal neurologic deficit not explained by head computed tomographic (CT) findings.American College of Surgeons SESAP 13 Critique for Category 7 Item 17 Once considered very rare. Bee TK. et al: Blunt cerebrovascular injuries: diagnosis and treatment. Biffl WL. He was found unconscious by his wife and transported to the emergency department. alert. and a cutaneous burn to his right thigh of approximately 5 % total body surface area. evidence of thermal injury to the left arm. a 35-year-old man is thrown 10 ft off a ladder after coming in contact with a 7200-volt power line. and neurologically intact. He is currently awake.American College of Surgeons SESAP 13 Category 7 Item 18 While hanging Christmas lights on his house. an exit wound on his right knee. He has a charred area on his left hand. The LEAST appropriate intervention in the emergency department would be (A) endotracheal intubation (B) measurement of compartment pressures (C) head computed tomographic (CT) scan (D) measurement of serum creatinine phosphokinase levels (E) escharotomy of the left arm page 634 . In one series. the time of contact. and traumatic brain and intra-abdominal injuries. lowvoltage injuries (< 1000 V). 2006 page 635 . If compartment pressures are elevated or trend upward. is related to the resistance of the contact point. cardiac and pulmonary contusions. Electrical injuries. and the type of current. the amperage involved. but then should decrease. explaining the magnitude of damage seen in the distal extremities. producing elevated tissue pressures and further muscle necrosis. Once the electricity overcomes the resistance of skin. Relying on sequential observation of clinical signs and symptoms such as pain. Gibran NS: Practice guidelines for the management of electrical injuries. Patients with electrical injuries can sustain such strong tetanic contractions that compression fractures may occur without external trauma. escharotomies and prompt fasciotomies should be performed.Arnoldo B. and lightning strikes. A carpal tunnel release is appropriate for forearm compartment swelling. Serial measurements of serum creatinine kinase (CK) and measurements of compartment pressures are essential. The areas of greatest electrical resistance in the body are skin and bone. All compartments in the lower leg and in the forearm should be released. following Ohm’s law. as is occasional release of the thigh and upper arm compartments. The patient with an electrical injury is at significant risk of developing compartment syndrome. occurring after significant tissue damage. Cardiac markers with serial EKGs should be obtained if myocardial injury is suspected. The pathophysiology of electrical burns. This destruction is magnified when the area of small cross section contains a highly resistive structure such as an axial bone.American College of Surgeons SESAP 13 Critique for Category 7 Item 18 Electrical injuries may be divided into four categories: high-voltage injuries (> 1000 V). Any continued elevation requires re-evaluation of the patient for further muscle necrosis. Electrical burns frequently have associated trauma. Serum CK will continue to rise for 24 to 48 hours. The tissue destruction is highest when the current enters an area with a small cross-sectional diameter and tends to be dissipated when a larger surface is crossed. The damaged or necrotic muscle swells within the overlying fascia. The electrical burn patient will often have normal-appearing skin with muscular destruction beneath. REFERENCES 1. arc injuries. It is imperative to ascertain an accurate history of the accident to establish whether the patient has been thrown or fallen from a height. particularly along muscular origins and insertions of bone. the body acts as a volume conductor of heat. These are all late phenomena of increased compartment pressure. Klein M. An EKG should be obtained in the emergency department because myocardial injury can occur similar to myocardial contusion. loss of distal sensation. are often associated with other forms of trauma. J Burn Care Res 27:439-447. particularly high-voltage electrical injuries. or diminution of pulses is inadequate. 60% of high-voltage electrical injuries were associated with other significant injuries. The acute care of the electrical burn patient requires an initial assessment of cardiac status because of the potentially life-threatening arrhythmias that can result. in which no current passes through the body. Ventricular fibrillation is the most common cause of death at the scene and occurs soon after the electrical injury. including long bone fractures. Extensive trauma evaluation of these patients is essential. pp 455-460 page 636 . Bryant CA. Kowalske K. in Achauer BM. Russell RC (eds): Plastic Surgery Indications. Morgan RF. J Burn Care Rehabil 25:479-484. et al: Injuries due to commercial electric current. Barret JP: Electrical injuries. Louis. Eriksson E. 2001. in Herndon D (ed): Total Burn Care. 1984 6. Kenney JG. St. ed 2. WB Saunders.McCauley RL. Helm PA. et al: Electrical injuries: a 20-year review. Operations. Masterson TS. Hunt JL: Electrical injuries.Purdue GF. Mason AD Jr. London. Pruitt BA Jr: The pathophysiology of acute electrical injuries. 2004 3. Vander Kolk C.Arnoldo BD. pp 375-385 5. Mosby. 2000.American College of Surgeons SESAP 13 2.Hunt JL. and Outcomes. Purdue GF.Nichter LS. J Trauma 16:335-340. 1976 4. J Burn Care Res 5:124-137. and hemodynamically stable.American College of Surgeons SESAP 13 Category 7 Item 19 A security guard sustains a left thoracoabdominal stab wound. The most appropriate management would be (A) observation (B) ultrasonography (C) laparotomy (D) thoracoscopy (E) laparoscopy page 637 . talking. On examination the abdomen is soft and nontender. He is alert. Because of the pressure gradients across the diaphragm.McQuay N JR. Garcia-Roca R. 2005 2. McKenney MG. Coln CE. Expectant management is problematic because diaphragmatic injuries can be asymptomatic for an indefinite period. although right diaphragmatic injuries can be associated with serious intra-abdominal injuries. etc) could reliably detect a diaphragm injury.Soffer D. The patient would likely have to be repositioned. 2003 3.Friese RS. et al: A prospective evaluation of ultrasonography for the diagnosis of penetrating torso injury. including major hepatic injuries and biliary-pleural fistulas. J Trauma 56:953-957. 2004 page 638 . Gentilello LM: Laparoscopy is sufficient to exclude occult diaphragm injury after penetrating abdominal trauma. No conventional diagnostic study (CT scan. with the left side being potentially more problematic. diagnostic peritoneal lavage.American College of Surgeons SESAP 13 Critique for Category 7 Item 19 The thoracoabdominal region. plain x-ray. ultrasound. Cohn S. these injuries are associated with herniation and possible strangulation of abdominal viscera. REFERENCES 1. routine laparotomy was the best option for any patient sustaining a penetrating thoracoabdominal injury. Although advocated by some because of the relative ease of evaluating the diaphragm. and draped again prior to laparotomy if abdominal exploration of a penetrating diaphragmatic injury is required. thoracoscopy often requires placing the patient in the decubitus position. prepped. defined as the area between the nipple and the costal margin. Britt LD: Laparoscopy in the evaluation of penetrating thoracoabdominal trauma. was once considered the ultimate blind spot in penetrating thoracoabdominal injuries. Until the introduction of diagnostic laparoscopy. This is particularly true for left-side injuries. J Trauma 58:789-792. Am Surg 69:788-791. American College of Surgeons SESAP 13 Category 7 Item 20 A 20-year-old man with a stab wound to the abdomen is hemodynamically normal. The most appropriate management would be (A) distal pancreatectomy and splenectomy (B) distal pancreatectomy and splenic preservation (C) wide drainage of the injury with closed suction drains (D) jejunal anastomosis to proximal pancreatic segment (E) jejunal anastomosis to both the proximal and distal pancreatic segment page 639 . he has a near-total pancreatic transection just to the left of the superior mesenteric artery. He has no other associated injuries. At exploration. but abdominal examination is consistent with peritonitis. coagulopathic. Jejunal anastomosis to either the proximal segment of the pancreas or both proximal and distal pancreatic segments is too time-consuming and. et al: Pancreatic trauma: a simplified management guideline. and the pancreatic injury is isolated or present with only minor associated injuries. ed 5. would have the associated risk of anastomotic leakage. most importantly. REFERENCES 1. Feliciano DV. 2004. pp 725 2. Lyden SP. and environmental factors are not optimal.American College of Surgeons SESAP 13 Critique for Category 7 Item 20 The most appropriate management for near-total pancreatic transection would be distal pancreatectomy with splenic preservation if possible. or hypothermic. Mattox KL (eds): Trauma. 1997 page 640 . splenectomy should be performed. New York.Jurkovich GJ. If the patient’s hemodynamic status and/or comorbidities. in Moore EE.Patton JH Jr. Splenic salvage should be considered if the patient is completely hemodynamically normal and normothermic. Croce MA. Wide drainage of the injury with closed suction could serve as a temporizing measure if the patient was hemodynamically abnormal. Pritchard FE. J Trauma 43:234-239. McGraw-Hill. Bulger EM: Duodenum and pancreas. which of the following does NOT influence the development of intra-abdominal infection? (A) Fecal contamination (B) Use of a diverting colostomy (C) Blood transfusion > 4 units / 24 hours (D) Shock at admission (E) Delay in therapy > 6 hours page 641 .American College of Surgeons SESAP 13 Category 7 Item 21 In the treatment of penetrating colon injuries. Predictors of complications included severe fecal contamination and transfusion of more than 4 units of blood in the first 24 hours. REFERENCES 1. The incidence of abdominal complications was not affected by diversion versus primary anastomosis. About one third were treated with diversion.Hudolin T.Adesanya AA. Ordonez C. Br J Surg 92:643-647. 2004 2. Ekanem EE: A ten-year study of penetrating injuries of the colon. J Trauma 50:765-775. a delay of more than 6 hours in taking the patient to the operating room. 297 patients were treated with either diversion or primary anastomosis at the surgeon’s option. There was no difference in either the complication rate or the mortality rate. Murray JA. Hudolin I: The role of primary repair for colonic injuries in wartime. Other predictors reported by others include shock on admission. A study of 60 patients from Nigeria showed that patients undergoing primary repair fared no worse than those who received a colostomy.American College of Surgeons SESAP 13 Critique for Category 7 Item 21 In a prospective multi-institution study under the auspices of the American Association for the Surgery of Trauma. Subsequent closure of the colostomy was associated with a morbidity of 21% and a mortality of 5%.Demetriades D. severe associated injuries). Neither of these last two reported series was randomized. Chan L. and a penetrating abdominal trauma index > 25 (ie. 2005 page 642 . Dis Colon Rectum 47:2169-2177. and two thirds with primary anastomosis. 2001 3. et al: Penetrating colon injuries requiring resection: diversion or primary anastomosis? An AAST prospective multicenter study. 122 patients had primary repair and 137 had a colostomy. In a series of 259 patients with colon injuries in the setting of the Bosnia-Herzegovina conflict. She is 7 weeks pregnant.American College of Surgeons SESAP 13 Category 7 Item 22 A 16-year-old girl is brought to the emergency department with facial trauma. and a seatbelt sign. an obvious chest contusion. Which of the following statements is TRUE? (A) She is in the her first trimester and is at no increased risk for fetal demise (B) Radiation risk from trauma x-rays is insignificant in pregnant patients (C) She is likely to have a more severe injury than a woman who is not pregnant (D) Maternal systolic blood pressure < 90 is a prognostic indicator for decreased fetal survival (E) Maternal heart rate > 120 is a prognostic indicator for decreased fetal survival page 643 . J Am Coll Surg 200:49-56. abnormal fetal heart rate. 2001 2. which were on average 20. maternal death.9% were found to be pregnant. Bieneik EJ III. Demarest GB. Fildes J. Schermer CR. In one study of female trauma patients of childbearing age. Independent risk factors for fetal loss after trauma have been identified to be injury severity score (ISS) >15. Fetal mortality in this group was significantly higher (100%) than in women with recognized pregnancies. In a survey of the American College of Surgeons National Trauma Bank. and Glasgow coma scale (GCS) score = 8. pregnant patients who present after blunt trauma with any of the identified risk factors warrant monitoring for at least 24 hours. Given these data. Pregnant female trauma patients tend to be younger.5 weeks gestation (25%. Lazar AA.0005).Curet MJ. 2.American College of Surgeons SESAP 13 Critique for Category 7 Item 22 Trauma is the leading cause of maternal death during pregnancy. et al: Predictors of outcome in trauma during pregnancy: identification of patients who can be monitored for less than 6 hours. 2005 page 644 . Napolitano LM. and more likely to be African American or Hispanic than nonpregnant female trauma patients.9 weeks gestation. REFERENCES 1. abbreviated injury score (AIS) 3 in head. These authors concluded that rapid pregnancy tests should be considered in all female trauma victims of child-bearing age to promote reduced fetal radiation exposure and potential fetal mortality in patients with newly diagnosed incidental pregnancy. abdomen. averaging 6. J Trauma 49:1824. less severely injured. 20% of injured pregnant patients tested positive for drugs or alcohol. et al: Profile of mothers at risk: an analysis of injury and pregnancy loss in 1.Bochicchio GV. A separate retrospective review of data from a single center concluded that risk factors significantly predictive of fetal death include ejections. Because radiation exposure during a trauma evaluation commonly exceeds 5 rads. lack of restraints. and pedestrian collisions significantly predicted contractions or preterm labor. motorcycle and pedestrian collisions. 2000 3.Ikossi DG. Morabito D. 9% of these were newly diagnosed incidental pregnancies. Gestational age >35 weeks. Haan J.195 trauma patients. early recognition that a woman is pregnant is imperative. thorax. Champion H. or lower extremities. and ISS > 9. p=0. assaults. et al: Incidental pregnancy in trauma patients. J Am Coll Surg 192:566-569. Those without these risk factors can safely be monitored for 6 hours after trauma before discharge. and approximately 30% in motor vehicle crashes were not using seatbelts. maternal tachycardia. American College of Surgeons SESAP 13 Category 7 Item 23 Which of the following statements about the current management of liver trauma is NOT true? (A) Nonoperative management is appropriate for blunt liver trauma (B) Operative management of blunt hepatic injury is indicated for hemodynamic abnormality and associated organ injury (C) Patients with blunt grade V hepatic injury usually require operation (D) Angiographic embolization is useful as an adjunct in both operative and nonoperative management (E) Anatomic lobar resection is not required for most injuries page 645 . Khan A: Nonoperative management of penetrating liver trauma. Latifi R. et al: Penetrating torso trauma: triple-contrast helical CT in peritoneal violation and organ injury — a prospective study in 200 patients.Malhotra AK. 2003 2. Although the failure rate of nonoperative management is increased in higher grade injuries. 2004 page 646 . Mirvis SE. Fabian TC. J Trauma 54:925-929. 2000 3. Embolization may also be used to improve the chances for operative success when bleeding cannot be completely controlled. In spite of continued improvement in nonoperative management. Killeen KL. and without other intra-abdominal injuries necessitating laparotomy.Pal KM. Surg Clin North Am 84:437-450.Trunkey DD: Hepatic trauma: contemporary management. most grade IV and V injuries may still be successfully managed nonoperatively. which was successful in all cases. 2004 4. et al: Multiplicity of solid organ injury: influence on management and outcomes after blunt abdominal trauma. Lobar resection is rarely required for blunt hepatic trauma. with a relatively low failure rate.Shanmuganathan K. Up to 90% of hepatic injuries can be managed nonoperatively at most institutions. Chiu WC. In a study comparing patients with isolated spleen or liver injuries versus those with combined liver and spleen injuries. Injury 31:199-201. Radiology 231:775-784. The chief criterion is that patients be hemodynamically normal. Angiography and embolization may be valuable in controlling arteriovenous fistulas and ongoing bleeding in a patient stable enough for the procedure. patients who present with hemodynamic abnormalities or who decompensate after arrival despite adequate fluid administration need prompt operation. one third of patients with high-grade liver and spleen injuries qualified for nonoperative management. REFERENCES 1.American College of Surgeons SESAP 13 Critique for Category 7 Item 23 Most blunt hepatic injuries are preferentially managed nonoperatively. Ivatury RR. the patient’s blood pressure decreases to 88/50. Bladder pressure is 30 mm Hg. and grade II liver laceration bleeding is easily controlled with suture ligatures. and is taken to the operating room. The abdomen is closed primarily and the patient is transferred to the ICU.American College of Surgeons SESAP 13 Category 7 Item 24 A 33-year-old man is hypotensive after a motor vehicle crash. central venous pressure is 11 mm Hg. Resuscitation continues. Six hours later. Hemoglobin is 10 g/dL. and peak airway pressures have increased from 32 to 50 cm H 2 0. and he becomes oliguric. Splenectomy and left nephrectomy are performed for severe hemorrhage. His abdomen is distended and firm. requiring several liters of crystalloid as well as blood and platelets. his pulse increases to 120. The next step should be (A) increasing the rate of fluid resuscitation (B) placement of a pulmonary artery catheter (C) changing the ventilator mode to pressure-control ventilation (D) inotropic therapy (E) immediate laparotomy page 647 . The only satisfactory treatment in this circumstance is immediate laparotomy to decompress the abdomen. Using pressure-control ventilation is extremely dangerous in the patient with abdominal compartment syndrome. Curr Opin Crit Care 9:133-136. because the physiologic problem is decreased venous return from the lower body. nor is initiation of inotropic therapy. and peak airway pressures increase because of the increasing subdiaphragmatic pressure. Curr Probl Surg 41:815-876. Increasing fluid resuscitation is clearly counterproductive. Abdominal compartment syndrome may occur after massive trauma if the abdomen is primarily closed.McNelis J. but central venous pressure may be normal or high normal. Brasel KJ: Management of the patient with an open abdomen: techniques in temporary and definitive closure.Rutherford EJ. Placement of a pulmonary artery catheter is not particularly helpful. because the intra-abdominal pressure may completely overcome the set pressure of ventilation. 2003 2. not decreased intravascular volume. as it will merely exacerbate the underlying problem. Decreases in urine output may be out of proportion to the decrease in systolic blood pressure because of the effects of the increased abdominal pressure on the kidneys and ureters.American College of Surgeons SESAP 13 Critique for Category 7 Item 24 This patient has abdominal compartment syndrome. 2004 page 648 . and immediate laparotomy is required. Simms HH: Abdominal compartment syndrome: clinical manifestations and predictive factors. especially if fluid administration after operation is extensive. followed by temporary abdominal closure. REFERENCES 1. and a catastrophic decrease in tidal volumes and minute ventilation may ensue. Marini CP. Skeete DA. Vital signs are similar to hypovolemia. He has bilateral breath sounds. The most appropriate next step would be (A) laparotomy (B) diagnostic peritoneal lavage (DPL) (C) admission for observation (D) angiography (E) laparoscopy page 649 . At the scene.American College of Surgeons SESAP 13 Category 7 Item 25 A 25-year-old man is taken to the emergency department after falling 15 feet onto his right side. systolic blood pressure was 60 mm Hg. On arrival he is alert. The computed tomographic (CT) scan shown is obtained. with a blood pressure of 110 systolic after receiving 2 L of crystalloids. His vital signs have remained normal since receiving the crystalloids. reduces the need for blood transfusions and incidence of intra-abdominal sepsis. with a ruptured intraparenchymal hematoma with active bleeding. Bahramipour P. Spain DA. resuscitation with crystalloid (2 L) should be the initial step. Abdominal computed tomography (CT) shows a grade IV liver injury. Myers RA. Niles SE. Holcomb JB. and with evidence of hemorrhage < 15% of total blood volume. Nonoperative management of blunt liver injuries is recommended for all patients with liver injuries and normal hemodynamics. 1999 2. Lavery RF. Wohltmann CD. Barone A. 2004 3. Moncure M. high morbidity. with greater than 90% success. et al: Angiographic embolization for liver injuries: low mortality. Rodriguez A. Laparoscopy may benefit this patient later in his course if he develops significant hemoperitoneum or biliary ascites. et al: The accuracy of diagnostic laparoscopy in trauma patients: a prospective. J Trauma 55:1077-1082. Nonoperative management can be attempted in approximately 75% of patients with grade IV injuries.Duane TM.American College of Surgeons SESAP 13 Critique for Category 7 Item 25 This patient’s vital signs are normal. Early angiographic intervention. Moore JB. J Trauma 46:619-622. However. controlled study. 1998 4. Diagnostic peritoneal lavage will confirm the presence of blood.Carrillo EH.Kozar RA. 2005 5. Bochicchio GV.Mohr AM. and should be attempted only when such a team is available. Management of high-grade liver injuries is a multidisciplinary effort. REFERENCES 1. based on CT scan results and hemodynamics. J Trauma 59:1066-1071. Even with early intervention. followed by transfusion of packed RBCs if hemorrhage continues. J Trauma 57:494-500. Como JJ. The need for operative therapy may be predicted by initial platelet count (< 200. Int Surg 83:294-298. but has no role in initial management once the diagnosis has been made. et al: Interventional techniques are useful adjuncts in nonoperative management of hepatic injuries. 2003 page 650 . the complication rate and need for subsequent invasive intervention is as high as 50%. Scalea TM: Reevaluating the management and outcomes of severe blunt liver injury. Most series using this method of management include very few injuries of higher grade.Elliot DC.000/dL) and fluid needs during the first 4 hours (> 3 L). CT shows evidence of intraperitoneal blood in addition to the grade IV liver injury. subsequent operative therapy is required in 25% of cases. but will not add any information helpful in the management of this patient. Schmieg RE. et al: Complications of nonoperative management of high-grade blunt hepatic injuries. American College of Surgeons SESAP 13 Category 7 Item 26 A 19-year-old man is brought to the emergency department with a gunshot wound in the right anterior axillary line 2 cm above the nipple and a second wound in the left back 2 cm lateral to and 2 cm above the tip of the scapula. The next step should be (A) angiography (B) chest computed tomography (CT) (C) thoracoscopy (D) bronchoscopy (E) observation and repeat chest x-ray in 4 hours page 651 . He has mild chest pain. A chest x-ray shows a right pleural effusion and no missiles. pulse is 88. and blood pressure is 132/70. but is not needed in patients with gunshot wounds. Meier DE. and bronchoscopy. 2002 4. Berne TV. and risk of occult great vessel injury 20%. Repeat chest x-ray will demonstrate a delayed hemothorax or pneumothorax in 10% of patients with stab wounds. In addition to a right tube thoracostomy. 1998 2. May AK. Reilly PM. Thoracoscopy may be necessary to evacuate the hemothorax.Nagy KK. However. J Trauma 22:291-294. J Trauma 49:689-695. Schwab CW. Asensio JA. J Trauma 45:446-456. 2000 3. but this would be done only after injury to other mediastinal structures has been evaluated. Joseph KT. Roberts RR. Aurbakken CM.Weigelt JA.Hanpeter DE. one or more of these adjunctive tests may still be necessary for missile trajectory injuries in close proximity to a mediastinal structure. et al: Determining anatomic injury with computed tomography in selected torso gunshot wounds. Although this can be done with a combination of aortography.American College of Surgeons SESAP 13 Critique for Category 7 Item 26 This patient has a transmediastinal gunshot wound. chest computed tomography (CT) is a more efficient method of evaluation. REFERENCES 1. et al: Trans-mediastinal gunshot wounds: are “stable” patients really stable? World J Surg 26:1247-1250. he must be evaluated for injury to major mediastinal structures. of esophageal injury is 15% to 30%. and eliminates the need for these more invasive tests in 50% to 75% of patients with missile trajectory injuries. et al: Helical computed tomographic scan in the evaluation of mediastinal gunshot wounds. esophagoscopy. His normal initial hemodynamics do not exclude significant injury: his risk of occult cardiac injury is 5% to 10%.Grossman MD. esophagram. 1982 page 652 . Smith RF. Demetriades D. chest CT may not be able to exclude injury. Thal ER: Management of asymptomatic patients following stab wounds to the chest. American College of Surgeons SESAP 13 Category 7 Item 27 Which of the following patients requires immediate operation? (A) 23-year-old woman thrown from a horse who has a subarachnoid hemorrhage and is hypoxic (PO2 56 torr on 100% oxygen) and has the computed tomographic (CT) scan shown (B) 45-year-old asymptomatic left-handed man with a periclavicular stab wound and the arteriogram shown (C) 52-year-old man with back pain (pulse 82 and blood pressure 140/68) who fell 20 feet 12 hours ago and has the CT shown (D) 65-year-old man involved in a frontal motor vehicle crash (pulse 74 and blood pressure 132/82) with the CT shown (E) 34-year-old woman with a pelvic fracture (pulse 54 and blood pressure 110/52) and the CT shown page 653 . Gomez H. Revascularization might be possible within the first 2 hours after injury. Operative repair of this injury is associated with significant morbidity. Even this combination of factors is associated with a sensitivity of only 75% and a specificity of 79%. Davis KA. 1999 3. Systemic heparin is contraindicated in the presence of a significant subarachnoid hemorrhage. Patient (B) has an injury to the proximal subclavian artery. The best predictor of hollow viscus injury is the presence of free fluid without concomitant solid organ injury. 2004 5. Patient (E) has active contrast extravasation from mesenteric vessels. 2003 4. Arbabi S. Any indication of blunt intestinal injury mandates operative treatment. et al: Delayed repair for blunt thoracic aortic injury: is it really equivalent to early repair. would be less desirable. Rollet G. More important is her degree of hypoxemia and inability to tolerate single-lung ventilation. J Vasc Surg 40:873-879. revascularization should not be attempted. with a significant pulmonary contusion impairing her oxygenation. J Am Coll Surg 188:290-295. 2004 page 654 . Luchette FA.Amabile P. Patient (C) has a nonperfused left kidney. et al: Penetrating injuries to the subclavian and axillary vessels. Patient (D) has a grade IV splenic injury with active extravasation. where the stent would need to cross the mobile shoulder joint. This patient has two such injuries. Knudson MM. EAST Multi-Institutional Hollow Viscus Injury Research Group: Current diagnostic approaches lack sensitivity in the diagnosis of perforated blunt small bowel injury: analysis from 275. An endovascular approach to the more distal subclavian artery or proximal axillary artery. This is present in a minority of blunt intestinal injuries. Biffl W. The presence of active extravasation does not mandate operative treatment. but it offers an alternative in patients with other injuries who are unable to tolerate operative treatment. and bowel wall thickening. Brandt MM. Chahwan S. making endovascular repair a better option than operative repair. J Trauma 56:13-23. Watts DD.Haan JM. Because this patient was injured 12 hours previously and has irreversible ischemic damage to the kidney. Free intraperitoneal extravasation is more often associated with the need for operative treatment. because delayed diagnosis and treatment significantly increases morbidity and mortality. J Trauma 56:542-547.557 trauma admissions from the EAST multi-institutional HVI trial. Whether endovascular treatment is equivalent to operative repair of traumatic aortic injury is unclear.Hemmila MR. 2004 2. which would be required for operative repair.Demetriades D. using either an endovascular or operative approach depending on the injury. and it can be successfully treated with endovascular stenting. although heparin-bonded circuits can be used if cardiopulmonary bypass is necessary for repair. et al: Surgical versus endovascular treatment of traumatic thoracic aortic rupture.Fakhry SM. Rowe SA. Collart F. The images (1 and 2) depict completed stent deployment. pneumoperitoneum. REFERENCES 1. the decision should be based on hemodynamic status and the presence of other injuries. et al: Splenic embolization revisited: a multicenter review. Peng R.American College of Surgeons SESAP 13 Critique for Category 7 Item 27 Patient (A) has an aortic injury. Gariboldi V. J Trauma 54:295-306. 2001 7. Gray DK. Bartsch G. et al: Implications of the “contrast blush” finding on computed tomographic scan of the spleen in trauma.Omert LA. et al: Evaluation and management of renal injures: consensus statement of the renal trauma subcommittee. 2004 page 655 . 2004 8. Salyer D. Descotes J. Stewart TC. BJU Int 93:937-954. Lawlor DK. Wessells H.Santucci RA. J Trauma 56:565-570.Ott MC. Dunham CM.American College of Surgeons SESAP 13 6. Porter J. J Trauma 51:272-278. et al: Management of blunt thoracic aortic injuries: endovascular stents versus open repair. He is normotensive on arrival and PaO 2 is 65 torr on room air. Which of the following statements is NOT true? (A) His outcome will be improved if an epidural catheter is used for pain control (B) His outcome will be improved if he is admitted to the ICU and a pulmonary artery catheter is placed (C) He is most likely to die within the first 48 hours of admission (D) He is twice as likely to die as a 45-year-old with similar injuries (E) An abdominal computed tomographic (CT) scan should be obtained page 656 .American College of Surgeons SESAP 13 Category 7 Item 28 An 82-year-old man has 6 rib fractures on the lower left chest after falling 12 feet. et al: Effect on outcome of early intensive management of geriatric trauma patients. However. Simon HM. Marvin RG. J Trauma 50:684-688. Mortality is twice as high in elderly patients with rib fractures compared with younger patients.McKinley BA. Br J Surg 89:1319-1322.Shweiki E. Death is most likely to occur between days 7 and 30 due to multisystem organ failure. Pneumonia contributes to mortality. Cocanour CS. 2000 5. 2002 4. 2001 page 657 . Duncan AO. Marquez A. Use of an epidural catheter to improve pain control decreases both pneumonia and time receiving mechanical ventilatory support. altered mental status. Klena J.Scalea TM. Alo K. particularly in patients with isolated thoracic trauma. Indeck M: Assessing the true risk of abdominal solid organ injury in hospitalized rib fracture patients. patients with low rib fractures should be evaluated for intra-abdominal injury using abdominal computed tomography (CT). although their survival can be improved with early. Jurkovich GJ: Rib fractures in the elderly.Demetriades D. Arneson MA. aggressive monitoring using a pulmonary artery catheter. REFERENCES 1. and increases 19% for each rib fractured. 2004 2.Bulger EM. Arch Surg 135:688-695. J Trauma 30:129-136. Even with this aggressive approach. Klotz P. The elderly do not recover as well from injury. mortality is significantly higher than in younger patients. Mock CN. most elderly patients with rib fractures are not candidates for epidural catheter placement because of spine fractures. Velmahos G. et al: Geriatric blunt multiple trauma: improved survival with early invasive monitoring. Surgery 136:426-430. J Trauma 48:10401046. Jurkovich GJ: Epidural analgesia improves outcome after multiple rib fractures. Lower rib fractures are associated with a significantly increased risk of splenic injury on the left and liver injury on the right. Wood GC.Bulger EM. Atweh NA. Pneumonia is caused by a combination of underlying lung injury and poor respiratory mechanics secondary to pain. 1990 6. et al: Blunt trauma resuscitation: the old can respond. Even without abdominal symptoms.American College of Surgeons SESAP 13 Critique for Category 7 Item 28 Elderly patients comprise a growing segment of the population and a growing segment of those hospitalized for traumatic injury. pre-existing spine disease. Edwards T. 2000 3. Karaiskakis M. or coagulopathy. American College of Surgeons SESAP 13 Category 7 Item 29 Which of the following retroperitoneal hematomas should always be explored in the hemodynamically normal trauma patient? (A) Zone I blunt injury (B) Zone I penetrating injury (C) Zone II blunt injury (D) Zone II penetrating injury (E) Zone III blunt injury page 658 . and the pancreas. Meyer A.Falcone RA Jr.Baniel J.Goins WA. REFERENCES 1.Wessells H. Associated injuries usually dictate exploration in patients with moderate or severe hematomas. In blunt trauma. et al: Retroperitoneal hematoma after blunt trauma. It resolves spontaneously in 76% to 87% of patients and can be adequately treated with a combination of stenting and percutaneous drainage. McAninch JW. Schein M: The management of penetrating trauma to the urinary tract.Santucci RA. Brathwaite CE. 1994 2. exploration is recommended even though it increases the potential for renal loss. Zone III retroperitoneal hematomas are also known as pelvic hematomas. J Urol 157:24-27. 1990 4. No patient with only a mild hematoma needs exploration. 2004 6. the retroperitoneal hematoma should be explored at that time. J Am Coll Surg 178:417-425. the duodenum. Rodriguez A. penetrating zone II retroperitoneal hematomas can also be observed. If adequate preoperative imaging is not available. et al: Zone I retroperitoneal hematoma identified by computed tomography scan as an indicator of significant abdominal injury. hematomas can be classified as mild (fat stranding). Expanding pelvic hematomas should be explored if encountered at laparotomy. et al: Evaluation and management of renal injures: consensus statement of the renal trauma subcommittee. Surgery 126:608-615. Penetrating injuries should be explored to rule out major vascular injury. these are most often due to renal injury and rarely require exploration unless they are rapidly expanding. and exploration can decompress a tamponade effect. although hemorrhage may be best controlled using a combination of pelvic packing and angioembolization. Ann Surg 211:109-123.Feliciano DV: Management of traumatic retroperitoneal hematoma. or severe (organ displacement due to hematoma). In patients with adequate imaging. Wessells H. The presence of urinary extravasation does not mandate exploration. Luchette FA. moderate (obliteration of fat). if laparotomy is required. the decision to explore a retroperitoneal hematoma is based on the presence or absence of specific findings on preoperative imaging. Surg Gynecol Obstet 174:281-290. Tiao G. 1992 5. 1997 page 659 . even if exploration is required for associated injuries.American College of Surgeons SESAP 13 Critique for Category 7 Item 29 Zone I retroperitoneal hematomas are also known as central hematomas. Zone II retroperitoneal hematomas are also known as lateral hematomas. BJU Int 93:937-954. Lewis J. Bruce J: Criteria for nonoperative treatment of significant penetrating renal lacerations. When due to penetrating trauma. When due to blunt trauma. Based on abdominal computed tomography (CT). Descotes J. All patients with shock should be explored. Bartsch G. Choe KA. significantly exacerbating blood loss. they should be explored to exclude injury to major vascular structures. 1999 3. Blunt injuries are usually due to pelvic fractures. The next step in management should be (A) repeat abdominal CT with oral contrast (B) laparotomy (C) focused assessment with sonography for trauma (FAST) (D) admission with serial abdominal examinations (E) diagnostic peritoneal lavage page 660 . The computed tomographic (CT) scan shown is obtained. and the remainder of his abdominal examination is normal.American College of Surgeons SESAP 13 Category 7 Item 30 A 45-year-old man who was wearing a seatbelt has a pulse of 100 and blood pressure of 130/90 after a motor vehicle crash. He is appropriately tender over the seatbelt mark. The appearance of the seatbelt mark is shown. his relative risk of small bowel injury is increased by a factor of 2. REFERENCES 1. Watts DD. Although wearing his seatbelt reduces his risk of mortality significantly. Up to 15% of patients with hollow viscus injury have no suggestive signs on CT. Thomason M. 2003 2. The decision is more difficult in the absence of any of these signs. Mashiko K. 2004 page 661 . Lee CH. Meredith W. Oller D. Luchette FA.Stafford RE.Stuhlfaut JW. P’eng FK: Prospective comparison of diagnostic peritoneal lavage. Adding oral contrast only increases the time from injury to diagnosis. Radiology 233:689-694. et al: The spectrum of abdominal injuries associated with the use of seat belts. Because DPL doesn’t evaluate retroperitoneal structures. thickened bowel. it may be negative if done within 3 hours of injury.557 trauma admissions from the EAST multi-institutional HVI trial. Given the significant association of a seatbelt mark with hollow viscus injury. Arch Surg 134:622-627. 1991 5. et al: New diagnostic peritoneal lavage criteria for diagnosis of intestinal injury. J Trauma 54:295-306. not the accuracy of CT in diagnosing hollow viscus injury. This combination has the highest sensitivity (75%) and specificity (79%) for the detection of bowel injury. McGonigal MD.4. Ultrasonography may be limited by pneumoperitoneum. J Trauma 44:991-997. and ultrasonography for the diagnosis of blunt abdominal trauma. et al: Blunt abdominal trauma: performance of CT without oral contrast material. J Trauma 35:267-270. His computed tomographic (CT) scan shows free abdominal fluid.Otomo Y. Kato K.Rutledge R. both commonly associated with an abdominal seatbelt mark. and pneumoperitoneum. Ultrasonography is similarly not indicated with the clear need for laparotomy. Although DPL can be used to diagnose hollow viscus injury. Lucey BC. Soto JA. but the risk of false negative is high in cases with only minimal fluid. it is also unhelpful in diagnosing pancreatic and duodenal injury.Fakhry SM. any one of these CT findings would be enough to prompt laparotomy. Johnson TJ: Oral contrast solution and computed tomography for blunt abdominal trauma: a randomized study.Liu M. J Trauma 31:820-826. EAST Multi-Institutional Hollow Viscus Injury Research Group: Current diagnostic approaches lack sensitivity in the diagnosis of perforated blunt small bowel injury: analysis from 275. 1993 3.American College of Surgeons SESAP 13 Critique for Category 7 Item 30 This patient has an abdominal seatbelt mark. and is not helpful in the evaluation of retroperitoneal structures. 1999 6. Ulrich A. Weigelt JA. computed tomographic scanning. Diagnostic peritoneal lavage (DPL) is not indicated in this patient whose CT clearly indicates the need for laparotomy. Henmi H. It may be helpful if the injury is significant enough to have produced a large amount of fluid. 1998 4. His initial chest x-ray is normal. All of the following studies may be indicated EXCEPT (A) repeat chest x-ray in 4 to 6 hours (B) focused assessment with sonography for trauma (FAST) (C) diagnostic peritoneal lavage (D) chest computed tomography (E) laparoscopy page 662 .American College of Surgeons SESAP 13 Category 7 Item 31 A 23-year-old man has a stab wound 1 cm below and 2 cm medial to the left nipple. He is alert and breathing comfortably. His pulse is 94 and blood pressure is 115/70. Occult diaphragmatic injury is present in approximately 7%. local wound exploration and/or diagnostic peritoneal lavage. is not sensitive enough to evaluate for diaphragm injury. 2003 4. If laparoscopy is not performed to evaluate for potential diaphragm injury. J Trauma 53:1135-1138. Papagiannopoulos KA. et al: FAST (focused assessment with sonography in trauma) accurate for cardiac and intraperitoneal injury in penetrating anterior chest trauma. J Trauma 55:646-650. Because this patient is at risk for abdominal injury.American College of Surgeons SESAP 13 Critique for Category 7 Item 31 The location of this patient’s stab wound places him at risk for injury to the lung. Pretorius J. 2004 page 663 . Chest computed tomography (CT) is not necessary to evaluate for hemopneumothorax. Physical examination alone may miss any or all of these injuries. Ann Thorac Surg 68:2119-2122. Roberts R. prompting the need for a delayed inspiratory/expiratory film.Ordog GJ. Marx JA. Shoemaker W: Asymptomatic stab wounds of the chest. 1994 5. Options include admission with serial abdominal examination. Hemopneumothorax is not apparent on admission in 10% of cases. heart. Balasubramanium S. J Ultrasound Med 23:467-472. Nagy K. or abdominal computed tomography. Evaluation for cardiac injury can be done with either sonography or a subxiphoid pericardial window. J Trauma 36:680-684.Leppaniemi A. the accuracy of sonography is greater than 95%. 2002 2. the abdomen must be evaluated in another manner. and abdomen. diaphragm. and is not specific enough to evaluate for cardiac injury. and can be evaluated with either laparoscopy or thoracoscopy. Wasserberger J. Van Rooyen T.Tayal VS. 1999 3. Tomaszewski CA. laparoscopy would be able to evaluate both diaphragmatic penetration and peritoneal penetration. et al: Current evaluation of cardiac stab wounds. Beatty MA. et al: Prospective evaluation of the sensitivity of physical examination in chest trauma. Brakenridge S. Haapiainen R: Occult diaphragmatic injuries caused by stab wounds.Harris DG. REFERENCES 1.Bokhari F. American College of Surgeons SESAP 13 Category 7 Item 32 Which of the following statements about imaging strategies to evaluate the spine after injury is TRUE? (A) Computed tomography (CT) is less accurate than plain x-rays for the thoracolumbar spine (B) CT is less accurate than plain x-rays for the cervical spine (C) Initial screening for thoracolumbar spine injuries should be done with plain x-rays (D) Initial screening for cervical injury in obtunded patients should be done with plain x-rays (E) CT is more accurate than flexion-extension films of the cervical spine page 664 . et al: Computed tomographic scanning reduces cost and time of complete spine evaluation. Eber CD.Freedman I. reconstructing the same images to evaluate for spinal column injury allows these injuries to be identified or eliminated much earlier in the patient’s overall evaluation. Yeom K. specificity from 72% to 89%. CT is also more accurate than plain x-rays for the cervical spine. J Am Coll Surg 200:160-165. CT is more accurate than plain x-rays for diagnosis of thoracolumbar injury.American College of Surgeons SESAP 13 Critique for Category 7 Item 32 The increased use of computed tomography (CT) for evaluation of the chest and abdomen has changed the recommendations for use of plain x-rays to evaluate suspected spinal injury. the best strategy for evaluating the cervical spine is physical examination. J Trauma 58:1183-8. versus a sensitivity of 95% to 97% and a specificity of 93% to 100% for CT. Patients without midline cervical tenderness who have normal range of motion do not require any radiographic imaging. Hinrichs C. et al: Prospective validation of computed tomographic screening of the thoracolumbar spine in trauma. van Gelderen D. 2005 4. et al: Cervical spine assessment in the unconscious trauma patient: a major trauma service’s experience with passive flexion-extension radiography. Moore DE. 2003 5. REFERENCES 1. Visvikis G. 2004 2. 2005 3. et al: Radiographic cervical spine evaluation in the alert asymptomatic blunt trauma victim: much ado about nothing. Tatevossian R. with a greater sensitivity. A strategy based on plain x-rays means additional static images are required in at least 10% of patients. J Trauma 40:768774. In awake. Morris JA Jr. and positive and negative predictive value. and are much more costly because they involve additional personnel time to perform the examination. and another significant percentage will require CT to clarify areas of possible injury. Cooper DJ.Velmahos GC. Because the images are already being obtained to evaluate for intrathoracic and intra-abdominal injury. Wahl WL. Kazerooni E. Theodorou D. Dittus RS. Even though CT is approximately three times as costly as a 3-view static series. Sensitivity of plain x-rays varies from 44% to 84%.Brandt MM. J Trauma 55:228-235. particularly those already undergoing CT of other body regions. Passive flexion/extension films are not as accurate as CT. 1996 page 665 . Fitzgerald M. it is probably more cost-effective in most patients. J Trauma 56:1022-1028. et al: Cervical spine evaluation in urban trauma centers: lowering institutional costs and complications through helical CT scan.Hauser CJ.Grogan EL. Belzberg H. specificity. CT is also better able to distinguish acute fracture from chronic fractures or degenerative disease. alert patients without distracting injury. American College of Surgeons SESAP 13 Category 7 Item 33 Which of the following statements about airbags and seatbelts is TRUE? (A) More than half of the states have a primary seatbelt enforcement law (B) Seatbelt use in states with a primary enforcement law is 5% higher than in states without a primary enforcement law (C) Airbags alone reduce driver mortality from motor vehicle crashes by 30% (D) Seatbelt use has increased 10% in the last 10 years (E) Seatbelts reduce driver mortality from motor vehicle crashes by 45% page 666 . Seatbelts are much more effective in reducing mortality than airbags. In contrast. 21 states had such a law.National Highway Traffic Safety Administration. REFERENCES 1.Knudson MM. from just under 60% in 1995 to over 80% in 2005.gov/people/injury/SafetyBelt/OPIPT_FinalRpt_07-17-03.dot. et al: Surgeons and injury prevention: what you don’t know can hurt you! J Am Coll Surg 193:119-124. July 2003. 2001 2. Use is highest in states with primary enforcement laws. use is approximately 13% higher in states with primary enforcement laws. Available at: http://www. Straus EM. both because of effectiveness and the magnitude of patients injured in motor vehicle crashes each year. drivers can be cited for not wearing seatbelts only if they are stopped for another offense. rear-impact crashes. and rollover crashes. They are most effective in frontal crashes. As of July 2004. airbags reduce the risk of fatality by only 15%. Hammond JS. with an average reduction in fatality risk of 45%.nhtsa. Initiatives to address safety belt use.html page 667 . In states with secondary laws. Primary enforcement works. where drivers can be stopped by law enforcement personnel solely for not wearing their seatbelts. Seatbelt use has increased dramatically in the last 10 years.American College of Surgeons SESAP 13 Critique for Category 7 Item 33 Use of restraints is one of the most important and effective methods to reduce the burden of injury in the United States. Vassar MJ. American College of Surgeons SESAP 13 Category 7 Item 34 Which of the following is NOT necessary to determine brain death? (A) Loss of response to painful stimuli (B) Absence of brainstem reflexes (C) Apnea (D) Temperature > 36°C (E) Study confirming absence of cerebral blood flow page 668 . Government Printing Office. in President's Commission on Ethical Problems in Medicine and References Biomedical and Behavioral Research. and body temperature at least 36°C. they are not the current standard for declaration of brain death. These studies can be used to confirm brain death. Washington.” Therefore. page 669 . the moment of death was considered to be the time when a person ceased. Electroencephalography (EEG) is not sufficient to establish brain death. Report of the Ad Hoc Committee of the Harvard Medical School to Examine the Definition of Brain Death. but are insufficient to act as the sole criterion for determining brain death. or … irreversible cessation of all functions of the entire brain. including the brain stem. movement. REFERENCES 1. JAMA 205:337-440. Traditionally. communication. and Ethical Issues in Definition of Death.Defining Death: A Report on the Medical. and did not resume. 1968 2. the accepted medical standards for clinical diagnosis of brain death are: no voluntary or involuntary movement except spinal reflexes. This cardiorespiratory criterion presumes the integrating function of the brain stem. 1981 2.American College of Surgeons SESAP 13 Critique for Category 7 Item 34 Declaring death is one of the legal duties of physicians. apnea in the presence of elevated PCO exclusion of a toxic or drug etiology. is dead. DC.[No Authors Listed]: A definition of irreversible coma. Legal. a “brain death” criteria was developed in 1968 and further defined in 1981 as: “An individual who has sustained either… irreversible cessation of circulatory or respiratory function. Because mechanical ventilatory support allows the body to be oxygenated in the absence of brain stem function. and breathing. and although radionuclide cerebral blood flow studies are promising. There are no wounds on the injured extremity. There is no evidence of perfusion to his foot. Which of the following statements is TRUE? (A) Limb salvage will be mainly determined by the presence or absence of arterial injury (B) Perfusion to the limb cannot be restored until the fractures are stabilized (C) Anticoagulation is contraindicated (D) The risk of amputation increases with concomitant venous injury (E) Angiography is required prior to treatment page 670 .American College of Surgeons SESAP 13 Category 7 Item 35 A 36-year-old man who was hit by a car has the x-ray shown. He has no other associated injuries. Yellin AE: Difficult peripheral vascular injuries. A 10-year retrospective series of popliteal artery trauma found a nearly 31% rate of amputation for limbs presenting without distal pulses.Weaver FA. concomitant venous injury. et al: Popliteal artery trauma. temporary shunting with systemic anticoagulation using intravenous tubing from the normal proximal vessel to the normal distal vessel will restore perfusion while stabilization occurs. These intimal injuries are prone to thrombosis. Ann Surg 225:518-527. 1997 2.Melton SM.American College of Surgeons SESAP 13 Critique for Category 7 Item 35 This patient’s x-ray shows a tibial plateau fracture of the extremity. 1996 page 671 . Hyperextension of the knee causes ligamentous injuries. temporary arterial shunting with anticoagulation may be useful to maintain perfusion of the distal extremity. Stabilization is required prior to definitive bypass to estimate the appropriate length for the bypass and to prevent kinking and/or occlusion of the new graft during subsequent orthopaedic manipulation. Pritchard FE. a high mangled extremity score. although benefit must be weighed against risk of bleeding if other operative procedures are needed. Patton JH Jr. The initial maneuver to restore perfusion to the foot is to stabilize the leg. Surg Clin North Am 76: 843-859. Croce MA. Posterior knee dislocation is the most likely explanation for the lack of perfusion to the foot in this scenario. Preoperative arteriography should not be performed if it delays re-perfusion. When the ischemic interval is already prolonged. and may frequently stretch the artery to such a degree that an intimal tear results. Prompt re-perfusion of the extremity is clearly the priority in these patients. If time to operative exploration is prolonged. Papanicolaou G. Systemic anticoagulation and intraoperative thrombolysis improves limb salvage. Logistic regression analysis identified blunt injury. and the need for heparin and urokinase therapy as independent predictors of limb loss. Thrombolytic therapy may also be administered intraoperatively in an attempt to identify vessels suitable for bypass and to re-perfuse thrombosed distal vessels. REFERENCES 1. Venous thrombosis may also occur in this setting. Quite often arteriography is not required because computed tomographic (CT) arteriography can demonstrate the lack of perfusion to the distal vessels. Which of the following statements is TRUE? (A) Sweating.American College of Surgeons SESAP 13 Category 7 Item 36 An 18-year-old man has no neurologic function below the level of injury after falling from a cliff. and bradycardia may be frequent symptoms after discharge (B) The external fixator placed for an associated tibial fracture can be safely removed without anesthesia (C) Autonomic dysreflexia will not occur because of the level of the spinal cord injury (D) Hypotension is commonly seen with autonomic dysreflexia (E) Consistent management of bowel and bladder function will eliminate the risk of autonomic dysreflexia page 672 . He regains no neurologic function after stabilization. The x-ray and magnetic resonance imaging (MRI) scan shown are obtained. nausea. this type of injury is associated with a period of neurogenic shock characterized by systemic hypotension and persistent bradycardia. with one episode occurring within the first week after injury. flushing. Acutely.Blackmer J: Rehabilitation medicine: 1. REFERENCES 1. These episodes are characterized by volatility of systemic blood pressure and periods of hypertension. Furlan JC. piloerection. nasal congestion. menstruation. and death if left untreated. With injuries at T6 or higher. and medications. Fehlings MG: Autonomic dysreflexia in acute spinal cord injury: an under-recognized clinical entity. the afferent impulse generates a generalized sympathetic response. 2003 2.7%. When autonomic dysreflexia occurs. Phosphodiesterase inhibitors (silfenadil) are commonly used in male spinal cord injury patients for the treatment of impotence. the offending stimulus should be identified and removed as rapidly as possible. trauma. In these patients. medication with a rapid onset and short duration of action should be given promptly. Autonomic dysreflexia will occur in 20% to 70% of patients with complete spinal cord injuries at or above the T6 level.Krassioukov AV. there is no intact descending inhibitory pathway to inhibit the hypertensive response. Noxious and nonnoxious stimuli below the level of injury may provoke such episodes. but other remaining causes may include pressure sores. which frequently ends the episode. With these stimuli. A recent retrospective review of patients with acute spinal cord injury documented the development of autonomic dysreflexia within the first month after injury in 5. Removal of orthopaedic hardware could precipitate such an episode. CMAJ 169:931-935.American College of Surgeons SESAP 13 Critique for Category 7 Item 36 This patient’s x-ray and magnetic resonance imaging (MRI) scan demonstrate complete transection of the spinal cord at the C6 level with associated bony deformity. Epidural anesthesia would minimize the pulmonary complications in this patient and prevent the painful stimulus from triggering an episode. In addition. The excessive parasympathetic response above the level of injury results in the vasodilation. In the case of persistent hypertension. This state may persist for days to weeks. and the combination of nitrates and silfenadil may result in profound hypotension. seizures. 2003 page 673 . sweating. J Neurotrauma 20:707-716. Autonomic dysrefexia. causing vasoconstriction below the level of the spinal cord injury. and may cause intracranial hemorrhage. Established bowel and bladder routines will eliminate distention as a possible trigger. and headache that alert the patient to the episode. the splanchnic circulation is affected and provides the volume of blood necessary to produce such profound hemodynamic changes. there is marked reduction or absence of reflex activity below the level of the injury (spinal shock). American College of Surgeons SESAP 13 Category 7 Item 37 Injury to the eye and adnexa (A) occurs in 1% to 3% of major trauma patients (ISS > 15) (B) is not diminished by seatbelt use among patients involved in motor vehicle crashes (C) is increased among survivors of motor vehicle crashes who wear eyeglasses (D) is sustained by 10% of patients whose airbags deploy during a motor vehicle crash (E) is vision-threatening in children when accompanied by orbital floor fractures page 674 . Since the passage of seatbelt legislation. Woods CD. 2005 3. Initial examiners may lack the necessary expertise.Lehto KS. Hill DA: Eye injuries in patients with major trauma. Kress TA.4%.Kuhn F.5 times if the vehicle is not airbag equipped. motor vehicle related blinding eye injuries are estimated to have been reduced by 50% to 75%. et al: Airbag-induced eye injuries: a report of 25 cases. Janik JS: Spectrum of ocular injuries in children with major trauma. breathing.Poon A. Witherspoon CD: Eye injury and the air bag. J Trauma 46:494-499. In one large series of major trauma patients (6313 patients. and orbital floor fractures in particular. Meaningful examination of the eye may be difficult or impossible in the unconscious patient with maxillofacial trauma.5% had ocular trauma. Eyewear may alter the pattern of injury. the rate of ocular trauma increases 2. but these patients are at greatest risk for visual impairment or loss. Curr Opin Ophthalmol 6:38-44. Paul BK. Bain L. REFERENCES 1.Duma SM. and circulation. Although anecdotal reports of airbag-related ocular injuries have appeared within the last decade. Porta DJ. 1999 6.Garcia TA. J Trauma 41:114-119. Champion HR: Ocular trauma among major trauma victims in a regional trauma center. Early ophthalmologic consultation should be considered in patients with these injuries. 1993 page 675 . Tervo TM: Do motor vehicle airbags increase risk of ocular injuries in adults? Ophthalmology 110:1082-1088. but does not appear to increase its incidence. J Trauma 59:169-174. 2003 5. Morris R. J Trauma 34:223-226. Furthermore. orbital wall fracture. and a contusion of the eye and/or adnexa compared with children with ISS < 15. Maxillofacial injuries. Children with an ocular injury sustained during major trauma are more likely to have a basilar skull fracture. 1995 4. 1996 2. the actual risk of vision-threatening injuries is 0.American College of Surgeons SESAP 13 Critique for Category 7 Item 37 Initial assessment and treatment after major trauma (injury severity score >15) focuses on the airway. ISS > 15) from a level I trauma center.Sastry SM. McGetrick BA. Sulander PO. 13. McCluskey PJ. are associated with an increased risk of vision-threatening ocular injury in both adults and children. American College of Surgeons SESAP 13 Category 7 Item 38 Multi-institutional studies of current treatment for blunt injury to the spleen demonstrate that nonoperative management (A) is associated with a mortality risk > 30% in hemodynamically abnormal patients (B) is not appropriate unless the patient is in a level I or II trauma center (C) is appropriate for children but not for adults (D) is not amenable to management by guidelines (E) most commonly fails at 48 to 96 hours after injury page 676 . Arch Surg 140:563-568. Can J Surg 45:358-362. REFERENCES 1. Postsplenectomy sepsis was described in 1973. ISS > 25. 2005 4. Heil B.American College of Surgeons SESAP 13 Critique for Category 7 Item 38 Most patients. Nonoperative management fails in 8% to10% of patients in several reported series. blood pressure < 100 systolic). no data exist to demonstrate that nonoperative management should not be pursued in smaller hospitals. In one large multi-institutional study. The APSA Trauma Committee. 2000 6. Various authors are concerned that the pendulum has swung so far in the direction of nonoperative treatment that clinicians are not paying sufficient attention to abnormal clinical findings. Newly developed evidence-based guidelines.Peitzman AB. Harbrecht BG. 2005 3. 2003 2. Scalea TM. J Trauma 55:317-322.Stylianos S: Evidence-based guidelines for resource utilization in children with isolated spleen or liver injury. combined with the clinical judgment of experienced surgeons. J Pediatr Surg 35:164-169. Postuma R. et al: Failure of observation of blunt splenic injury in adults: variability in practice and adverse consequences. Kramer M. death was the outcome in 37% of patients whose operation was delayed despite persistently abnormal and unstable hemodynamics (pulse > 100. higher grade of splenic injury.Peitzman AB. triggering the shift toward observational management. et al: Blunt splenic injury in adults: Multi-institutional Study of the Eastern Association for the Surgery of Trauma.Haan J. most frequently in the first 24 hours after admission. The authors suggested this high failure rate was secondary to poor patient selection. Rivera L.McIntyre LK. Some studies indicate that failure rates may be higher in level III or IV trauma centers compared with level I or II. Heil B. degree of hemoperitoneum. and the proliferation of computed tomographic (CT) scanners made accurate diagnosis and nonoperative treatment possible. personnel. persistent abdominal pain and/or signs of peritonitis. both children and adults. Wiseman N: Blunt splenic injuries in a Canadian pediatric population: the need for a management guideline. and contrast blush from the injured spleen. have been shown to decrease resource utilization and hasten return to normal activities. J Am Coll Surg 201:179-187. Several studies have documented considerable variability in management from region to region and institution to institution. provided the necessary equipment. Other suggested predictors of failure include age > 55 years. Rivera L. Ilahi ON. and operating room accessibility are in place. Federle MB. 2000 5. Trends toward avoidance of invasive procedures provided a further impetus. Jurkovich GJ: Failure of nonoperative management of splenic injuries: causes and consequences. et al: Protocol-driven nonoperative management in patients with splenic trauma and minimal associated injury decreases length of stay. 2002 page 677 . However. Schiff M.Zabolotny B. Hancock BJ. who sustain blunt injury of the spleen can safely be managed without an operation. J Trauma 49:177-189. American College of Surgeons SESAP 13 Category 7 Item 39 A 36-year-old man is brought to the emergency department after falling off of an all-terrain vehicle (ATV). An intracranial pressure (ICP) monitor is placed. He has a Glasgow coma scale (GCS) score of 5 and decerebrate posturing. He is hemodynamically normal. and ICP is elevated Which of the following is NOT a standard early intervention for controlling ICP to maintain appropriate cerebral perfusion pressure? (A) Increase ventilator rate (B) Increase inspired oxygen content (C) Provide mannitol therapy (D) Elevate the head of the bed (E) Infuse hypertonic saline solution page 678 . and computed tomography (CT) shows diffuse axonal injury and cerebral edema. in Pappas TN. Purcell GP (eds): Unbound Surgery. An acute rise in ICP can be treated by hyperventilation with increased ventilatory rate.Zwienenberg LM. although serum osmolarity and urine output must be monitored carefully. Adequacy of cerebral perfusion pressure is the ultimate indicator and predictor of outcome. in Souba WW. Muizelaar JP: Injuries to the central nervous system. Kim KD. Although adequate oxygenation is extremely important to predicting outcome in closed head injury. Trauma and Thermal Injury. Cerebral perfusion pressure is the mean arterial pressure minus intracranial pressure (ICP). Mannitol and hypertonic saline are both effective in reducing intracranial pressure. Fink MP. sec 7. increasing oxygen content does not play any role in controlling intracranial pressure or cerebral perfusion pressure. Several studies have now shown that maintaining a cerebral perfusion pressure of 60 mm Hg is sufficient to assure optimal perfusion. available at www. Haglund MM: Head trauma. 2004. Standard early maneuvers to control intracranial pressure include elevation of the head of the bed and appropriate volume resuscitation.com page 679 . although the long-term effects of cerebral vasoconstriction can be detrimental. Web MD. pp 1145-1161 2. Jurkovich GJ. New York. REFERENCES 1.unboundsurgery. chap 2.Little KM. et al: ACS Surgery: Principles and Practice 2006.American College of Surgeons SESAP 13 Critique for Category 7 Item 39 The patient described has an elevated intracranial pressure and as a consequence diminished cerebral perfusion pressure after a brain injury. but becomes hypotensive on her return to the emergency department. She is initially hemodynamically normal.American College of Surgeons SESAP 13 Category 7 Item 40 A patient admitted after blunt abdominal trauma has the computed tomographic (CT) scan shown. The next step should be (A) focused assessment with sonography for trauma (FAST) (B) repeat CT scan (C) transfer to the ICU for observation (D) exploratory laparotomy (E) angiography page 680 . and responds to transfusion of 2 units of blood and crystalloid resuscitation. physical examination.Ochsner MG: Factors of failure for nonoperative management of blunt liver and splenic injuries.Carrillo EH. A blush associated with a solid organ injury is indicative of ongoing bleeding in that organ.American College of Surgeons SESAP 13 Critique for Category 7 Item 40 The abdominal computed tomographic (CT) scan shows evidence of a hepatic parenchymal injury with associated extravasation of contrast or blush. monitoring hematocrit and vital signs. Curr Probl Surg 38:1-60. Polk HC Jr: Evolution in the treatment of complex blunt liver injuries. 2001 page 681 . The density of the contrast is similar to that seen in the aorta and is consistent with a potential hepatic arterial injury. eg. 2001 2. with further intervention at any time if the patient develops either acute instability that cannot be corrected or hemodynamic abnormality that requires transfusion to regain stability. repeat CT and focused assessment with sonography for trauma (FAST) are not indicated. It is not unreasonable to allow other services to proceed with operative management of injuries if the patient is hemodynamically normal. World J Surg 25:1393-1396. Because the diagnosis has already been made in this case. A patient who requires blood intermittently but does not have consistently abnormal vital signs is a candidate for early angiography for selective embolization. A hemodynamically normal patient with a blush should be admitted to the ICU for close observation. REFERENCES 1. whereas a patient who is acutely unstable and receiving ongoing blood transfusion should undergo exploratory laparotomy. this procedure should be terminated to allow emergent exploration for management of the liver injury. Close observation is required if this is to be allowed. Wohltmann C. Richardson JD. If the patient becomes unstable in the operating room while having another procedure. On arrival. She had no signs of life when the paramedics arrived. She was found against a tree 20 yards ahead of the vehicle.American College of Surgeons SESAP 13 Category 7 Item 41 A 20-year-old woman was an unrestrained passenger in a high-speed motor vehicle crash. establishing central venous access (E) immediate operation without further work-up page 682 . The best course of action now would be (A) immediate ED thoracotomy through left chest (B) immediate ED thoracotomy through right chest (C) declaration of death (D) continued external CPR. but develops a pulseless rhythm during CPR in the field and en route to the emergency department. she loses her cardiac rhythm. Jacobson LE. None of the lesser measures are indicated. pp 1188-1206 page 683 . 2002 3. 1992 2. et al (eds): ACS Surgery: Principles and Practice 2006. WebMD Inc. New York.Ladd AP. Fink MP. Trauma and Thermal Injury. Bourguert CC: Emergency room resuscitative thoracotomy: when is it indicated? J Trauma 33:714-721. REFERENCES 1. Vanek VW. Kaiser LR. in Souba WW.Boyd M. chap 7. Am Surg 68:421-424. Broadie TA. Jurkovich GJ.American College of Surgeons SESAP 13 Critique for Category 7 Item 41 Multiple retrospective studies and meta-analyses have concluded that survival is essentially zero in patients who have blunt trauma and no signs of life in the field or in the emergency department. et al: Emergency room thoracotomy: updated guidelines for a level 1 trauma center. Gomez GA. Meredith JW: Injuries to the chest. sec 5.Kincaid EH. Emergency department thoracotomy is potentially dangerous to the personnel conducting it from injuries from sharps and possible viral transmission. It will certainly be futile in this case. the patient has a stable airway and is hemodynamically normal. midaxillary line. The next step in management should be (A) chest computed tomography (CT) (B) angiography of the great vessels (C) flexible bronchoscopy (D) esophagogram (E) transfer to a trauma center page 684 . The exit is the left thorax in the fifth intercostal space. A chest x-ray suggests a widened mediastinum with pneumomediastinum. After placement of bilateral chest tubes and evacuation of 250 mL of blood. The entrance wound is at the fourth intercostal space in the midaxillary line on the right.American College of Surgeons SESAP 13 Category 7 Item 42 A 29-year-old man sustains a gunshot wound while hunting and is brought to a small community hospital with limited surgical support. studies to document the presence of a suspected esophageal injury should not delay transfer to a trauma center.Asensio JA. When a community hospital lacks the appropriate resources to provide rapid definitive treatment.Smakman N. et al: Factors affecting outcome in penetrating oesophageal trauma. Because esophageal injuries may be particularly subtle. et al: Penetrating esophageal injuries: multicenter study of the American Association for the Surgery of Trauma. REFERENCES 1. Br J Surg 91:1513-1519. J Trauma 50:289-296. Nicol AJ. 2004 page 685 . Brooks A. the index of suspicion must be high and evidence actively sought. Forno W. and injury to the tracheobronchial tree often results in large air leaks and an inability to inflate the lung. 2001 2. Morbidity and mortality outcomes after esophageal injury are directly related to delays in both diagnosis and treatment. Walther G. a gunshot wound that traverses both thoracic cavities raises concern for damage to mediastinal structures.American College of Surgeons SESAP 13 Critique for Category 7 Item 42 Despite hemodynamic normality. Injury to the great vessels may result in a widened mediastinum and/or cardiac tamponade. MacKersie R. Chahwan S. The focused assessment with sonography for trauma (FAST) of the abdomen shown is performed. and O2 saturation of 99% (FiO 2 1. good breath sounds bilaterally. The remaining views were normal. The next step in management should be (A) chest computed tomography (CT) (B) pericardial window (C) median sternotomy (D) abdominal CT (E) laparotomy page 686 . he has ecchymosis over the sternum.American College of Surgeons SESAP 13 Category 7 Item 43 A 12-year-boy was intubated in the field for respiratory distress after a high-speed bicycle collision. On arrival. He remains hypotensive with systolic blood pressure in the 90s despite ongoing resuscitation.0). although the patient may reach the hospital alive if the injury is not full thickness and the tamponade stops the bleeding. Filho CS. Blunt cardiac rupture is frequently fatal. J Trauma 48:971-974. However.Rezende Neto JB. Chest computed tomography (CT) can be used in a stable patient with an equivocal ultrasound. Schirmer TP. REFERENCES 1. 2001 page 687 . Sideman MJ. He is demonstrating shock despite ongoing resuscitation. these techniques are not therapeutic and will not eliminate the need for median sternotomy. However. J Trauma 50:746-749. and laparotomy would not be indicated. Median sternotomy should be the next step in the management of this trauma patient. et al: Blunt hemopericardium detected by surgeonperformed sonography. and the most likely diagnosis is cardiac tamponade. 2000 2. Abrantes WL: Blunt traumatic rupture of the heart in a child: case report and review of the literature. Pericardiocentesis and pericardial window are diagnostic procedures to identify a patient with a pericardial tamponade and will only temporarily relieve hemodynamic instability. but would be contraindicated in an unstable patient.Carrillo EH. as would abdominal CT. Focused assessment with sonography for trauma (FAST) shows fluid in the pericardial space and no further diagnostic steps are needed. Wallace JM. Diniz HO. The odds of significant intra-abdominal bleeding in a patient with a normal FAST are very low.American College of Surgeons SESAP 13 Critique for Category 7 Item 43 This patient’s imaging studies show a large hemopericardium. the patient’s condition will deteriorate if the underlying cause of the tamponade physiology is not corrected. and he withdraws to painful stimulus.American College of Surgeons SESAP 13 Category 7 Item 44 A 23-year-old man was placed in spine precautions and intubated in the field for decreased mental status after falling 20 feet from a ladder. he does not open his eyes. On arrival. The next step in management should be (A) hyperventilation to arterial PCO2 of 25 torr (B) mannitol. pupils are 4 mm and reactive bilaterally. Head computed tomography (CT) reveals a small subarachnoid hemorrhage and blood in the left lateral ventricle. his vitals signs are normal. 4 mg/kg intravenously (D) serial examination and repeat head CT in 12 hours (E) intracranial pressure monitoring page 688 . 1 g/kg intravenously (C) dexamethasone. reduced length of stay. but by convention a score of 1 is assigned. J Trauma 50:657-664. 2004 2. The severity of head injury is assessed by the Glasgow coma scale (GCS) score. intubated = T). et al: Management of brain-injured patients by an evidence-based medicine protocol improves outcomes and decreases hospital charges. et al: The impact on outcomes in a community hospital setting of using the AANS traumatic brain injury guidelines.Palmer S. V1. Assessing the verbal component in an intubated patient is difficult. Prophylactic hyperventilation. 2001 page 689 .American College of Surgeons SESAP 13 Critique for Category 7 Item 44 This patient has a significant head injury. M4. Incorporation of these guidelines into the care of the neurotrauma patient results in improved outcome. The range of possible scores is 3 (worst) to 15 (best). mannitol. and possibly reduced costs. Bader MK. placement of an ICP catheter should be the next step in management. and corticosteroids have not been shown to improve outcomes and may be detrimental. REFERENCES 1. The three parameters comprising the score are outlined in the table: This patient’s GCS score is 6T (E1. This patient’s severe head injury (GCS < 8) and his abnormal head computed tomographic (CT) scan qualify him for intracranial pressure (ICP) monitoring. Americans Associations for Neurologic Surgeons. Trask AL. Waller MA. Qureshi A. J Trauma 56:492-499. Although serial examination and repeat head CT are appropriate.Fakhry SM. The management of these patients has been outlined in a comprehensive review of available data by the American Association of Neurological Surgeons. Palmer J. Watts DD. a restrained passenger has the chest x-ray shown. The next step in management should be (A) diagnostic peritoneal lavage (B) laparoscopy (C) laparotomy (D) tube thoracostomy (E) thoracotomy page 690 .American College of Surgeons SESAP 13 Category 7 Item 45 Following a side impact motor vehicle crash. et al: Blunt diaphragm injuries.Voeller GR. Laparotomy should be the operative approach because up to 80% of these patients have associated intra-abdominal injuries. Without the nasogastric tube being above the diaphragm. REFERENCES 1. Thoracotomy has been advocated for late repair of ruptured diaphragm (weeks to months later) because significant intrathoracic adhesions may develop. abdominal insufflation with laparoscopy may cause a tension pneumothorax. 1990 page 691 . Gakidis J. et al: Traumatic rupture of the diaphragm: experience with 65 patients. this could be interpreted as pneumothorax. Kudsk K. Diagnostic peritoneal lavage is unnecessary because it will not alter management. Paraskevopoulos J. Laparoscopy and thorascopy may not provide adequate visualization of other injuries. A five-year experience. Fabian TC. This patient’s chest x-ray is diagnostic of a ruptured diaphragm. Reisser JR. Potaris K.American College of Surgeons SESAP 13 Critique for Category 7 Item 45 Diaphragmatic rupture occurs in up to 5% of patients after major blunt traum. 2003 2. Am Surg 56:28-31. approximately 70% of these injuries involve the left hemidiaphragm.Mihos P. In blunt trauma. Injury 34:169-172. The diagnosis may be difficult because chest x-rays often show some obscuring of the hemidiaphragm that is interpreted as a hemothorax or pulmonary contusion. His abdomen is soft and not distended. and has a 2-cm laceration just lateral to umbilicus. the MOST appropriate maneuver would be (A) diagnostic laparoscopy in the emergency department (B) laparotomy (C) focused assessment with sonography for trauma (FAST) (D) discharge (E) observation for 12 hours page 692 .American College of Surgeons SESAP 13 Category 7 Item 46 A young man presents 1 hour after sustaining a stab wound to the abdomen. Of the following. is hemodynamically normal. He is not intoxicated. Majercik S.Tsikitis V. Haapiainen R: Diagnostic laparoscopy in abdominal stab wounds: a prospective. would be acceptable. In a review of 232 patients with anterior or lateral abdominal stab wounds. hypotension. a prospective study of 177 patients with penetrating torso trauma demonstrated that a negative FAST rarely altered the management of these patients. and a specificity of 98% for injury. This has led to a more selective approach to patient management. McKenney MG.Soffer D. Harrington DT. Cohn S. or active bleeding) should be observed for a minimum of 12 hours. Laparotomy was previously considered to be mandatory for such patients. diagnostic laparoscopy in the operating room added little to the evaluation of these patients.Alzamel HA. 2004 4. REFERENCES 1. Diagnostic laparoscopy in the emergency department offers no further advantage. Biffl WL. As an alternative. et al: Selective clinical management of anterior abdominal stab wounds. Patients without hard signs of intra-abdomoinal injury (peritonitis. 2004 page 693 . Observation for 6 hours or less may miss up to 50% of injuries. local wound exploration with discharge if the wound does not penetrate the fascia. J Trauma 56:953—959. 2005 2.Leppaniemi A.American College of Surgeons SESAP 13 Critique for Category 7 Item 46 This patient is asymptomatic after an anterior abdominal stab wound. Am J Surg 188:807—812. but two thirds were nontherapeutic and associated with a finite complication rate. or diagnostic peritoneal lavage or observation if it does. Although focused assessment with sonography for trauma (FAST) has an overall accuracy of 85%. Garcia-Roca R. et al: A prospective evaluation of ultrasonography for the diagnosis of penetrating torso injury. 2003 3. evisceration. Cohn SM: When is it safe to discharge asymptomatic patients with abdominal stab wounds? J Trauma 58:523—525. a sensitivity of 48%. randomized study. J Trauma 55:636-645. anastomosis. Treatment of the colon injury should be (A) debridement and primary repair (B) primary repair with proximal diversion (C) resection. end-colostomy.American College of Surgeons SESAP 13 Category 7 Item 47 During laparotomy for a gunshot wound to the abdomen. a patient is found to have 4 holes in close proximity in the small bowel and a 4-cm laceration to the anterior surface of the sigmoid colon located 5 cm above the peritoneal reflection. and Hartmann’s pouch (E) resection. end-colostomy. The patient is hemodynamically normal after minimal resuscitation and there is no fecal spillage. and mucus fistula (D) resection. and exteriorization page 694 . Garcia VF.Cayton CG. Ann Surg 235:775-781. Several published reports document that primary repair even with resection and anastomosis is feasible in many patients with destructive colon lesions. Magnotti LJ. Croce MA. Given the significant complication rate associated with subsequent colostomy closure. Ivatury RR: Patient management guidelines for penetrating intraperitoneal colon injuries. REFERENCES 1.pdf 2. Fabian TC. Ordonez C. The management of destructive colon wounds is somewhat more controversial. East Practice Parameter Workgroup for Penetrating Colon Injury Management. J Trauma 50:765-775.Demetriades D. et al: Penetrating colon injuries requiring resection: diversion or primary anastomosis? An AAST prospective multicenter study. Nondestructive injuries involving less than 50% of the circumference of the bowel and without devascularization may be safely repaired primarily. Chan L. Colon injuries may be broadly classified into 2 groups: nondestructive and destructive. Murray JA. Fabian TC. http://east. A number of studies have recently suggested that a single-stage procedure with primary repair may be the preferred treatment for many colon injuries. 2002 page 695 . the method of colon management did not influence the incidence of colon-related abdominal complications regardless of the presence of risk factors.org/tpg/chap4. resection and primary anastomosis should be considered for all patients with destructive colon injuries. Because colon diversion was associated with a worse quality of life and required a second operation for closure. This patient has a nondestructive injury to the sigmoid colon and the appropriate treatment would be debridement and primary repair.Miller PR. Destructive injuries require resection for control of the injury. some authors have questioned whether the concern for a high anastomatic failure rate and related complications was overestimated. et al: Improving outcomes following penetrating colon wounds: application of a clinical pathway.American College of Surgeons SESAP 13 Critique for Category 7 Item 47 Penetrating sigmoid colon injuries in the civilian population were previously treated with exteriorization of the wound (uncommon) or proximal diverting colostomy (common). In these reports. 2001 3. In the operating room. The next step in management of this patient should be (A) Roux-en-Y drainage of the pancreatic injury (B) closed suction drainage (C) pyloric exclusion and duodenal repair with lateral duodenostomy (D) distal pancreatectomy (E) pancreaticoduodenectomy (Whipple procedure) page 696 .American College of Surgeons SESAP 13 Category 7 Item 48 A 21-year-old man sustains severe blunt abdominal trauma in a high-speed motor vehicle crash. the patient has uncontrolled bleeding from behind the pancreas and complete transection of the second portion of the duodenum. The abdominal computed tomographic (CT) scan shown is obtained. pancreaticoduodenectomy is the only procedure indicated. 2003 2. These patients typically also sustain massive and unreconstructable injury to the head of the pancreas involving the main pancreatic duct. Issekutz A. Closed suction drainage. In this setting. uncontrollable retropancreatic hemorrhage from associated vascular injuries to the portal vein. or laceration exceeds 50% of the pancreatic parenchyma on the basis of CT scanning. If transection of the pancreas is complete. Hsu YP. pyloric exclusion.Lin BC. Chen RJ. REFERENCES 1. major duct injury should be suspected and operation scheduled immediately. and/or unreconstructable injuries to the first or second portions of the duodenum. J Trauma 56:774-778. 2003 page 697 . superior mesenteric artery. but highly lethal. and/or superior mesenteric vein.Asensio JA. 2004 3. Petrone P. and distal pancreatectomy would not be appropriate because of the severity of the pancreaticoduodenal injury and ongoing hemorrhage. Complex pancreaticoduodenal injuries requiring pancreaticoduodenectomy are uncommon. Kuncir E. the intrapancreatic portion of the distal common bile duct with devitalization of its blood supply. et al: Pancreaticoduodenectomy: a rare procedure for the management of complex pancreaticoduodenal injuries. J Am Coll Surg 197:937-942. Roldan G.Olah A. et al: Management of blunt major pancreatic injury. Haulik L. Roux-en-Y drainage of the pancreatic injury alone is inadequate. Current management guidelines for assessment of pancreaticoduodenal trauma include abdominal CT scanning for visualization of the pancreas and evaluation of injury severity. Fang JF. Owing to the severity of the injury and the uncontrolled bleeding reported behind the pancreas.American College of Surgeons SESAP 13 Critique for Category 7 Item 48 The abdominal computed tomographic (CT) scan shows a severe injury to the pancreaticoduodenal region. Dis Surg 20:408-414. Makay R: Pancreatic transection from blunt abdominal trauma: early versus delayed diagnosis and surgical management. The most common indication for performance of a pancreaticoduodenectomy is massive. American College of Surgeons SESAP 13 Category 7 Item 49 All of the following may be acute complications of crush injury and rhabdomyolysis EXCEPT (A) hypovolemia (B) hypercalcemia (C) hyperkalemia (D) acidemia (E) hyperphosphatemia page 698 . often due to the initial injury but also in part because of the increase in capillary permeability after re-perfusion. The systemic effects of crush injury and rhabdomyolysis are numerous. profound hyperkalemia leads to sudden death from cardiac arrhythmias on reperfusion. Fenoldopam. REFERENCES 1. Bicarbonate administration has also been advocated to alkalinize the urine and prevent precipitation of myoglobin.Huerta-Alardin AL. Clinical experience with fenoldopam in rhabdomyolysis is limited. Myoglobin released from injured muscle cells is filtered in the kidney and can precipitate in the tubules. Corcoran T. increases renal blood flow and prompts natriuresis. Occasionally. Myoglobin precipitation is more likely if the urine pH is acidic and is directly toxic to the renal tubules. but clinical studies have not confirmed an advantage over vigorous fluid resuscitation. In addition. This also serves to dilute the myoglobin load in the renal tubules. Direct release of potassium from injured muscle and a shift of intracellular potassium into the extracellular space in response to acidosis both contribute to hyperkalemia. preventing occlusion. Hyperphosphatemia and hyperkalemia frequently occur during rhabdomyolysis as both phosphate and potassium are released from injured cells. Markos F. Snow HM. Crit Care 9:158-169. Evans K. leading to obstruction. Both of these have theoretical benefits. Mullins RJ: Rhabdomyolysis and myoglobinuric renal failure in trauma and surgical page 699 . Hypovolemia occurs in many patients. 2004 2. Varon J. a variety of vasoconstrictors may be released that further decrease renal perfusion. 2003 4. which leads to significant tissue edema.American College of Surgeons SESAP 13 Critique for Category 7 Item 49 Crush injury is a term used to describe systemic effects of muscle injury caused by significant direct trauma or prolonged compression of muscle. which is most commonly quantified by measuring serum creatine kinase (CK).Slater MS. Mannitol has frequently been used to promote an osmotic diuresis to flush the tubules. a highly selective dopamine 1 receptor agonist. but animal data suggest it worsens renal injury. The crush and re-perfusion injury lead to sequestration of a large volume of plasma in the muscle with resultant hypovolemia and decreased renal perfusion (pre-renal azotemia). Chan L. Muscle breakdown and necrosis (rhabdomyolysis) produces release of intracellular muscle contents into the systemic circulation. Eur J Anaesthesiol 20:711-718. Acute renal failure is the most profound and significant complication of rhabdomyolysis. In cases of prolonged compression. et al: Preventing renal failure in patients with rhabdomyolysis: do bicarbonate and mannitol make a difference? J Trauma 56:1191-1196. et al: Effects of fenoldopam on renal blood flow and its function in a canine model of rhabdomyolysis. Rhee P. occurring in up to 35% of cases.Murray C. The etiology is multifactorial. the direct muscle trauma is further exacerbated by the re-perfusion injury that occurs once the victim is freed.Brown CV. The mainstay of prophylaxis of renal failure is prompt and adequate restoration of intravascular volume. 2005 3. The re-perfusion injury leads to derangements in calcium homeostasis with an increase in intracellular calcium and accompanying decrease in serum calcium levels. Loop diuretics (furosemide) have the theoretical disadvantage of acidifying the urine and may also lead to decreased intravascular volume. Marik PE: Bench-to-bedside review: Rhabdomyolysis—an overview for clinicians. American College of Surgeons SESAP 13 patients: a review. J Am Coll Surg 186:693-716. 1998 page 700 . the most important component of treatment to avoid renal failure is (A) mannitol (B) furosemide (C) bicarbonate (D) volume replacement (E) fenoldopam page 701 .American College of Surgeons SESAP 13 Category 7 Item 50 Following crush injury and rhabdomyolysis. In addition. In cases of prolonged compression. but clinical studies have not confirmed an advantage over vigorous fluid resuscitation. Mullins RJ: Rhabdomyolysis and myoglobinuric renal failure in trauma and surgical page 702 . REFERENCES 1. which is most commonly quantified by measuring serum creatine kinase (CK). The etiology is multifactorial. Hyperphosphatemia and hyperkalemia frequently occur during rhabdomyolysis as both phosphate and potassium are released from injured cells. a highly selective dopamine 1 receptor agonist.Slater MS. Myoglobin precipitation is more likely if the urine pH is acidic and is directly toxic to the renal tubules.Huerta-Alardin AL. Corcoran T. Clinical experience with fenoldopam in rhabdomyolysis is limited. preventing occlusion. profound hyperkalemia leads to sudden death from cardiac arrhythmias on reperfusion. Rhee P.Brown CV. Fenoldopam. Occasionally. This also serves to dilute the myoglobin load in the renal tubules. 2004 2. Bicarbonate administration has also been advocated to alkalinize the urine and prevent precipitation of myoglobin. Chan L. Varon J.Murray C. Direct release of potassium from injured muscle and a shift of intracellular potassium into the extracellular space in response to acidosis both contribute to hyperkalemia. Hypovolemia occurs in many patients. et al: Effects of fenoldopam on renal blood flow and its function in a canine model of rhabdomyolysis. The mainstay of prophylaxis of renal failure is prompt and adequate restoration of intravascular volume. the direct muscle trauma is further exacerbated by the re-perfusion injury that occurs once the victim is freed. Mannitol has frequently been used to promote an osmotic diuresis to flush the tubules.American College of Surgeons SESAP 13 Critique for Category 7 Item 50 Crush injury is a term used to describe systemic effects of muscle injury caused by significant direct trauma or prolonged compression of muscle. Evans K. but animal data suggest it worsens renal injury. which leads to significant tissue edema. The crush and re-perfusion injury lead to sequestration of a large volume of plasma in the muscle with resultant hypovolemia and decreased renal perfusion (pre-renal azotemia). Crit Care 9:158-169. Acute renal failure is the most profound and significant complication of rhabdomyolysis. leading to obstruction. occurring in up to 35% of cases. The systemic effects of crush injury and rhabdomyolysis are numerous. Muscle breakdown and necrosis (rhabdomyolysis) produces release of intracellular muscle contents into the systemic circulation. often due to the initial injury but also in part because of the increase in capillary permeability after re-perfusion. increases renal blood flow and prompts natriuresis. 2003 4. Loop diuretics (furosemide) have the theoretical disadvantage of acidifying the urine and may also lead to decreased intravascular volume. et al: Preventing renal failure in patients with rhabdomyolysis: do bicarbonate and mannitol make a difference? J Trauma 56:1191-1196. The re-perfusion injury leads to derangements in calcium homeostasis with an increase in intracellular calcium and accompanying decrease in serum calcium levels. a variety of vasoconstrictors may be released that further decrease renal perfusion. Eur J Anaesthesiol 20:711-718. Markos F. 2005 3. Myoglobin released from injured muscle cells is filtered in the kidney and can precipitate in the tubules. Snow HM. Marik PE: Bench-to-bedside review: Rhabdomyolysis—an overview for clinicians. Both of these have theoretical benefits. American College of Surgeons SESAP 13 patients: a review. 1998 page 703 . J Am Coll Surg 186:693-716. American College of Surgeons SESAP 13 Category 7 Item 51 56-year-old man with a zone I stab wound to the right neck and evidence of innominate artery injury on angiography (A) Median sternotomy (B) Left anterolateral thoracotomy (C) Right anterolateral thoracotomy (D) Left posterolateral thoracotomy (E) Right posterolateral thoracotomy page 704 . including the hilar pulmonary artery vein and bronchus. and intrathoracic vena cava can also be accessed through this incision. et al (eds): Thoracic Surgery. The incision chosen can greatly influence the ability to expose and repair the injury. innominate vein. Mattox KL (eds): Trauma. Ginsberg RJ. pp 571-581 2. 2002. the mediastinum. This may be minimized with a double lumen tube. Posterolateral thoracotomy requires the patient to be re-positioned in the lateral position. pp 18581863 page 705 . It can also be problematic in a contaminated wound. It is a good choice in a patient who is hemodynamically unstable with uncontrolled bleeding from an unknown source. Mattox KL: Penetrating trauma. ed 2. esophagus. It is a poor choice for the descending aorta. and the right chest. Soltero E. or lung hilum. New York. It can also be extended across the sternum to allow greater exposure to the heart. This incision needs proper equipment and some expertise. REFERENCES 1. ed 5. which has the potential to allow blood and secretions to enter the noninjured side. Churchill Livingstone. McGraw-Hill. including the ascending aorta and its transverse arch. 2004. Feliciano DV. A right anterolateral thoracotomy is rarely used. A median sternotomy provides excellent exposure to the heart and the proximal great vessels. A left posterolateral thoracotomy allows much greater exposure to the left pulmonary hilum.Mattox KL. Cooper JD. A left anterior thoracotomy provides reasonable exposure to the heart and the descending aorta for clamping purposes and open cardiac massage. in Moore EE. proximal right subclavian vein. New York. Wall MJ Jr. Placing the patient in a lateral position may exacerbate hemodynamic instability in hypovolemic patients. A right posterolateral thoracotomy is indicated in a patient with possible right hilar injuries to the lung and also gives good exposure to the esophagus in the right chest. Deslauriers J. The proximal pulmonary arteries. It also allows the ideal exposure for the descending aorta.Wall MJ Jr. LeMaire SA: Injury to the thoracic great vessels. including the most common areas for a traumatic rupture of the aorta. in Pearson FG.American College of Surgeons SESAP 13 Critique for Category 7 Item 51 The choice of thoracic incision for trauma repair is based on the most likely anatomic injury and the patient’s physiologic status. American College of Surgeons SESAP 13 Category 7 Item 52 34-year-old man with widened mediastinum and aortic tear at the ligamentum teres after a highspeed motor vehicle crash (A) Median sternotomy (B) Left anterolateral thoracotomy (C) Right anterolateral thoracotomy (D) Left posterolateral thoracotomy (E) Right posterolateral thoracotomy page 706 . the mediastinum. Feliciano DV. Posterolateral thoracotomy requires the patient to be re-positioned in the lateral position. esophagus. ed 5. It is a poor choice for the descending aorta. It also allows the ideal exposure for the descending aorta. Deslauriers J.American College of Surgeons SESAP 13 Critique for Category 7 Item 52 The choice of thoracic incision for trauma repair is based on the most likely anatomic injury and the patient’s physiologic status. LeMaire SA: Injury to the thoracic great vessels. The proximal pulmonary arteries. New York. pp 571-581 2. A left anterior thoracotomy provides reasonable exposure to the heart and the descending aorta for clamping purposes and open cardiac massage. It can also be problematic in a contaminated wound. innominate vein. A left posterolateral thoracotomy allows much greater exposure to the left pulmonary hilum. Mattox KL (eds): Trauma. 2002.Mattox KL. A right posterolateral thoracotomy is indicated in a patient with possible right hilar injuries to the lung and also gives good exposure to the esophagus in the right chest. et al (eds): Thoracic Surgery. and the right chest. and intrathoracic vena cava can also be accessed through this incision. REFERENCES 1. This incision needs proper equipment and some expertise. Mattox KL: Penetrating trauma. 2004. A right anterolateral thoracotomy is rarely used. A median sternotomy provides excellent exposure to the heart and the proximal great vessels. pp 18581863 page 707 . including the hilar pulmonary artery vein and bronchus. Ginsberg RJ. Wall MJ Jr. The incision chosen can greatly influence the ability to expose and repair the injury.Wall MJ Jr. Placing the patient in a lateral position may exacerbate hemodynamic instability in hypovolemic patients. Churchill Livingstone. Cooper JD. It can also be extended across the sternum to allow greater exposure to the heart. New York. including the most common areas for a traumatic rupture of the aorta. ed 2. or lung hilum. Soltero E. This may be minimized with a double lumen tube. proximal right subclavian vein. including the ascending aorta and its transverse arch. in Moore EE. It is a good choice in a patient who is hemodynamically unstable with uncontrolled bleeding from an unknown source. which has the potential to allow blood and secretions to enter the noninjured side. McGraw-Hill. in Pearson FG. American College of Surgeons SESAP 13 Category 7 Item 53 22-year-old man with a transmediastinal gunshot wound to the chest (at the 5 th ICS level) who is hemodynamically unstable (A) Median sternotomy (B) Left anterolateral thoracotomy (C) Right anterolateral thoracotomy (D) Left posterolateral thoracotomy (E) Right posterolateral thoracotomy page 708 . It is a poor choice for the descending aorta. Placing the patient in a lateral position may exacerbate hemodynamic instability in hypovolemic patients.Mattox KL. ed 5. New York. Soltero E. Feliciano DV.Wall MJ Jr. It also allows the ideal exposure for the descending aorta. 2002. McGraw-Hill. in Moore EE. which has the potential to allow blood and secretions to enter the noninjured side. It is a good choice in a patient who is hemodynamically unstable with uncontrolled bleeding from an unknown source. A left posterolateral thoracotomy allows much greater exposure to the left pulmonary hilum. Ginsberg RJ. A right anterolateral thoracotomy is rarely used. A left anterior thoracotomy provides reasonable exposure to the heart and the descending aorta for clamping purposes and open cardiac massage. A median sternotomy provides excellent exposure to the heart and the proximal great vessels. innominate vein. and intrathoracic vena cava can also be accessed through this incision. This incision needs proper equipment and some expertise. This may be minimized with a double lumen tube. LeMaire SA: Injury to the thoracic great vessels. proximal right subclavian vein. Wall MJ Jr. ed 2. including the most common areas for a traumatic rupture of the aorta.American College of Surgeons SESAP 13 Critique for Category 7 Item 53 The choice of thoracic incision for trauma repair is based on the most likely anatomic injury and the patient’s physiologic status. Posterolateral thoracotomy requires the patient to be re-positioned in the lateral position. It can also be extended across the sternum to allow greater exposure to the heart. esophagus. including the ascending aorta and its transverse arch. The proximal pulmonary arteries. Churchill Livingstone. The incision chosen can greatly influence the ability to expose and repair the injury. REFERENCES 1. or lung hilum. Deslauriers J. A right posterolateral thoracotomy is indicated in a patient with possible right hilar injuries to the lung and also gives good exposure to the esophagus in the right chest. including the hilar pulmonary artery vein and bronchus. 2004. It can also be problematic in a contaminated wound. pp 571-581 2. in Pearson FG. the mediastinum. New York. Mattox KL (eds): Trauma. et al (eds): Thoracic Surgery. Mattox KL: Penetrating trauma. pp 18581863 page 709 . Cooper JD. and the right chest. American College of Surgeons SESAP 13 Category 7 Item 54 44-year-old woman with iatrogenic perforation of the mid thoracic esophagus (A) Median sternotomy (B) Left anterolateral thoracotomy (C) Right anterolateral thoracotomy (D) Left posterolateral thoracotomy (E) Right posterolateral thoracotomy page 710 . Cooper JD. proximal right subclavian vein. It is a poor choice for the descending aorta. including the most common areas for a traumatic rupture of the aorta.American College of Surgeons SESAP 13 Critique for Category 7 Item 54 The choice of thoracic incision for trauma repair is based on the most likely anatomic injury and the patient’s physiologic status. and the right chest. REFERENCES 1. A left posterolateral thoracotomy allows much greater exposure to the left pulmonary hilum. It also allows the ideal exposure for the descending aorta. Feliciano DV. This may be minimized with a double lumen tube. A left anterior thoracotomy provides reasonable exposure to the heart and the descending aorta for clamping purposes and open cardiac massage. The incision chosen can greatly influence the ability to expose and repair the injury. esophagus. A median sternotomy provides excellent exposure to the heart and the proximal great vessels. LeMaire SA: Injury to the thoracic great vessels. Mattox KL: Penetrating trauma. ed 2. including the hilar pulmonary artery vein and bronchus. It is a good choice in a patient who is hemodynamically unstable with uncontrolled bleeding from an unknown source. 2002. A right anterolateral thoracotomy is rarely used. New York. innominate vein. ed 5. pp 571-581 2. Posterolateral thoracotomy requires the patient to be re-positioned in the lateral position. Ginsberg RJ. A right posterolateral thoracotomy is indicated in a patient with possible right hilar injuries to the lung and also gives good exposure to the esophagus in the right chest. Soltero E. et al (eds): Thoracic Surgery. New York. including the ascending aorta and its transverse arch. 2004. This incision needs proper equipment and some expertise. McGraw-Hill.Mattox KL. in Moore EE. or lung hilum. The proximal pulmonary arteries. which has the potential to allow blood and secretions to enter the noninjured side. Churchill Livingstone. pp 18581863 page 711 . Deslauriers J. Wall MJ Jr. in Pearson FG. Mattox KL (eds): Trauma. the mediastinum. Placing the patient in a lateral position may exacerbate hemodynamic instability in hypovolemic patients. and intrathoracic vena cava can also be accessed through this incision.Wall MJ Jr. It can also be problematic in a contaminated wound. It can also be extended across the sternum to allow greater exposure to the heart. American College of Surgeons SESAP 13 Category 7 Item 55 Common in household use (A) Alternating current (B) Direct current (C) Both (D) Neither page 712 . the damage sustained is more directly related to the current. Dickson WA: Electrical injury and the frequency of cardiac complications.Miller BK. At low voltages. J Trauma 26:166-167. causing asystole and respiratory arrest through a direct effect on the respiratory center. Cataracts develop in approximately 5% of patients with electrical injury in proximity to the head and neck. 2002 3. direct current is used when higher voltage is needed. REFERENCES 1. Monshizadeh R. Lightning is direct current with extremely high voltage. Although observation for delayed dysrhythmias has been common. and can occur up to 2 years after injury. and can occur as a result of low or high voltage. or the voltage divided by the resistance. Alternating current of low voltage is used in most homes and offices. patients with a normal initial electrocardiogram and no history of heart disease will not develop a delayed dysrhythmia and do not need to be monitored. Extremely high voltage is more likely to result in asystole. Hunt JL: Electrocardiogrpahic monitoring after electrical injury: necessity or luxury.American College of Surgeons SESAP 13 Critique for Category 7 Item 55 Electrical current is either alternating. Although electrical injuries are commonly classified by voltage. Burns 23:576-578. CLAO J 28:224-227.Purdue GF. 1997 2. 1986 page 713 . et al: Ocular manifestations of electrical injury: a case report and review of the literature. alternating current is more damaging than direct current because it results in tetanic contractions and thus prolonged contact. but more commonly they occur after a month. although high voltage can be generated with alternating current and direct current is occasionally low voltage.Koumbourlis AC: Electrical injuries. Tabandeh H. and were first described in someone struck by lightning. They can develop with both types of current at any voltage.Arrowsmith J. In severe injury close to the eye they may occur within hours. Ongoing eye examinations are required after electrical injury. Cardiac injury can occur as a result of direct injury to the myocardium or due to rhythm disturbance. Goldstein MH. 2002 4. In general. lower voltages produce ventricular fibrillation or other rhythm disturbances that are less serious. Usgaocar RP. with electrons flowing in one direction only. or direct. Crit Care Med 30:S424-S430. with electrons flowing back and forth. American College of Surgeons SESAP 13 Category 7 Item 56 Lightning (A) Alternating current (B) Direct current (C) Both (D) Neither page 714 . They can develop with both types of current at any voltage. the damage sustained is more directly related to the current. or direct. Alternating current of low voltage is used in most homes and offices. Lightning is direct current with extremely high voltage. Goldstein MH. Cardiac injury can occur as a result of direct injury to the myocardium or due to rhythm disturbance. Although observation for delayed dysrhythmias has been common. Burns 23:576-578. 2002 4. and were first described in someone struck by lightning.Purdue GF. REFERENCES 1. patients with a normal initial electrocardiogram and no history of heart disease will not develop a delayed dysrhythmia and do not need to be monitored. direct current is used when higher voltage is needed. In general.Arrowsmith J. causing asystole and respiratory arrest through a direct effect on the respiratory center. Extremely high voltage is more likely to result in asystole. Dickson WA: Electrical injury and the frequency of cardiac complications.Koumbourlis AC: Electrical injuries.American College of Surgeons SESAP 13 Critique for Category 7 Item 56 Electrical current is either alternating. Tabandeh H. 1986 page 715 . At low voltages. J Trauma 26:166-167. lower voltages produce ventricular fibrillation or other rhythm disturbances that are less serious. et al: Ocular manifestations of electrical injury: a case report and review of the literature. but more commonly they occur after a month. Crit Care Med 30:S424-S430. CLAO J 28:224-227. Monshizadeh R. and can occur up to 2 years after injury. alternating current is more damaging than direct current because it results in tetanic contractions and thus prolonged contact. although high voltage can be generated with alternating current and direct current is occasionally low voltage. or the voltage divided by the resistance. 1997 2. with electrons flowing back and forth. with electrons flowing in one direction only. 2002 3. Hunt JL: Electrocardiogrpahic monitoring after electrical injury: necessity or luxury. Although electrical injuries are commonly classified by voltage. and can occur as a result of low or high voltage. Cataracts develop in approximately 5% of patients with electrical injury in proximity to the head and neck. Ongoing eye examinations are required after electrical injury. In severe injury close to the eye they may occur within hours.Miller BK. Usgaocar RP. American College of Surgeons SESAP 13 Category 7 Item 57 Delayed dysrhythmias common (A) Alternating current (B) Direct current (C) Both (D) Neither page 716 . with electrons flowing back and forth.American College of Surgeons SESAP 13 Critique for Category 7 Item 57 Electrical current is either alternating.Arrowsmith J. and can occur as a result of low or high voltage. causing asystole and respiratory arrest through a direct effect on the respiratory center. Lightning is direct current with extremely high voltage. Tabandeh H. Hunt JL: Electrocardiogrpahic monitoring after electrical injury: necessity or luxury. Cardiac injury can occur as a result of direct injury to the myocardium or due to rhythm disturbance. Dickson WA: Electrical injury and the frequency of cardiac complications. Burns 23:576-578. Goldstein MH. although high voltage can be generated with alternating current and direct current is occasionally low voltage. direct current is used when higher voltage is needed. alternating current is more damaging than direct current because it results in tetanic contractions and thus prolonged contact. but more commonly they occur after a month. and can occur up to 2 years after injury. lower voltages produce ventricular fibrillation or other rhythm disturbances that are less serious. or direct. the damage sustained is more directly related to the current. et al: Ocular manifestations of electrical injury: a case report and review of the literature. Ongoing eye examinations are required after electrical injury. Usgaocar RP. Alternating current of low voltage is used in most homes and offices. and were first described in someone struck by lightning. Cataracts develop in approximately 5% of patients with electrical injury in proximity to the head and neck. Crit Care Med 30:S424-S430. In severe injury close to the eye they may occur within hours. At low voltages. REFERENCES 1. They can develop with both types of current at any voltage.Miller BK. CLAO J 28:224-227. or the voltage divided by the resistance. 2002 3. J Trauma 26:166-167. 1997 2. with electrons flowing in one direction only.Purdue GF. In general. Extremely high voltage is more likely to result in asystole. 2002 4. patients with a normal initial electrocardiogram and no history of heart disease will not develop a delayed dysrhythmia and do not need to be monitored. Monshizadeh R. Although observation for delayed dysrhythmias has been common.Koumbourlis AC: Electrical injuries. 1986 page 717 . Although electrical injuries are commonly classified by voltage. American College of Surgeons SESAP 13 Category 7 Item 58 Cataracts (A) Alternating current (B) Direct current (C) Both (D) Neither page 718 . or the voltage divided by the resistance. Cardiac injury can occur as a result of direct injury to the myocardium or due to rhythm disturbance. 1986 page 719 . Hunt JL: Electrocardiogrpahic monitoring after electrical injury: necessity or luxury. Extremely high voltage is more likely to result in asystole. Usgaocar RP. They can develop with both types of current at any voltage. CLAO J 28:224-227. Tabandeh H. Ongoing eye examinations are required after electrical injury. direct current is used when higher voltage is needed. and were first described in someone struck by lightning. Burns 23:576-578.Arrowsmith J. Alternating current of low voltage is used in most homes and offices. J Trauma 26:166-167. In severe injury close to the eye they may occur within hours. Monshizadeh R. lower voltages produce ventricular fibrillation or other rhythm disturbances that are less serious. and can occur up to 2 years after injury. and can occur as a result of low or high voltage. 1997 2. Cataracts develop in approximately 5% of patients with electrical injury in proximity to the head and neck. 2002 4. the damage sustained is more directly related to the current. with electrons flowing in one direction only. alternating current is more damaging than direct current because it results in tetanic contractions and thus prolonged contact. et al: Ocular manifestations of electrical injury: a case report and review of the literature. with electrons flowing back and forth. although high voltage can be generated with alternating current and direct current is occasionally low voltage. Goldstein MH.Koumbourlis AC: Electrical injuries. Dickson WA: Electrical injury and the frequency of cardiac complications.Purdue GF. patients with a normal initial electrocardiogram and no history of heart disease will not develop a delayed dysrhythmia and do not need to be monitored. Although electrical injuries are commonly classified by voltage. REFERENCES 1. At low voltages. but more commonly they occur after a month. causing asystole and respiratory arrest through a direct effect on the respiratory center. 2002 3. In general.Miller BK. Although observation for delayed dysrhythmias has been common. Crit Care Med 30:S424-S430. or direct. Lightning is direct current with extremely high voltage.American College of Surgeons SESAP 13 Critique for Category 7 Item 58 Electrical current is either alternating. but not in burn patients (C) frequently occurs in a patchy distribution (D) is caused by overgrowth of the bowel with Bacteroides fragilis (E) is associated with low mortality if broad-spectrum antibiotics are administered at the first sign of abdominal distention page 720 .American College of Surgeons SESAP 13 Category 8 Item 1 Nonocclusive bowel necrosis (NOBN) associated with early enteral nutrition (A) typically occurs within 24 hours of initiating tube feeds (B) has been reported in critically ill trauma patients. Schunn CD. which causes progressive distention that could impair mucosal perfusion.5%).Marvin RG. but this is not true for every case. McKinley BA. and in burn patients. EN is less expensive than total parenteral nutrition (TPN). Clinical findings may include tachycardia. J Am Coll Surg 180:410-416. REFERENCES 1. Daly JM: Small bowel necrosis associated with postoperative jejunal tube feeding. Systemic hypoperfusion is commonly believed to predispose to intestinal necrosis. If nonocclusive bowel necrosis associated with enteral nutrition is suspected (sepsis syndrome with or without gastrointestinal specific symptoms such as distention.Kowal-Vern A. McGill V. fever. If perforation is suspected based on clinical grounds or radiographic findings. and significantly reduces septic morbidity. and leukocytosis (much like bacterial sepsis). Signs specific to the gastrointestinal tract are not consistent or may occur late. No specific clinical parameters reliably diagnose this entity before bowel necrosis occurs. exploratory laparotomy is indicated. or abdominal tenderness). Several interconnected theories attempt to explain how enteral nutrition contributes to the pathogenesis of NOBN. Onset typically occurs in the second week of tube feeds after a period of enteral nutrition tolerance. Arch Surg 132:440-443. the absorption of intraluminal nutrients may deleteriously increase energy demands in metabolically stressed enterocytes.3% to 8. McQuiggan M. Cocanour CS. ileus. Broad-spectrum antibiotics do not change the outcome. Second.American College of Surgeons SESAP 13 Critique for Category 8 Item 1 Nonocclusive bowel necrosis (NOBN) associated with early enteral nutrition (EN) has been reported in critically ill trauma patients. It is a rare but lethal complication. resulting in ischemic injury. EN administered in patients with ileus may allow bacterial overgrowth. EN may generate intraluminal toxins that cause direct mucosal injury or initiate destructive submucosal inflammation. and postoperative patients. Gamelli RL: Ischemic necrotic bowel disease in thermal injury. Clinical studies have demonstrated that EN is feasible after abdominal operation or major trauma. 1997 2. Am J Surg 179:7-12. NOBN is extremely unusual in unfed patients who survive resuscitation from hemorrhagic shock. Early EN has become standard ICU practice. Additionally. Third. et al: Nonocclusive bowel necrosis occurring in critically ill trauma patients receiving enteral nutrition manifests no reliable clinical signs for early detection. NOBN frequently occurs in a patchy distribution with intervening nonnecrotic bowel (similar to neonatal necrotizing enterocolitis). burn patients. leading to selective mucosal ischemia. Specific infectious agents have not been shown to be associated with NOBN despite the observation that significant bacterial overgrowth is likely. tube feeds should be stopped and TPN initiated. First. 1995 page 721 . 2000 3. NOBN is a devastating complication in a small subset of these patients (0. Just before his discharge from the ICU 10 days later. Which of the following is associated with the GREATEST risk of developing a pressure ulcer? (A) Emergency admission to the ICU (B) Severity of APACHE score (C) Number of days without nutrition (D) Development of multisystem failure in the ICU (E) Number of days in bed page 722 . a type II sacral decubitus ulcer is noted.American College of Surgeons SESAP 13 Category 8 Item 2 A 45-year-old man is admitted to the ICU after an exploratory laparotomy for a gunshot wound to the abdomen. chronic illness. poor tissue perfusion. Patients in the ICU can be at high risk for pressure ulcer formation. Pressure ulcer classification is based on depth of tissue destruction: type I ulcers have intact skin. Barie PS: Factors influencing the development of decubitus ulcers in critically ill surgical patients. 2001 page 723 . These devices reduce the risk of ulcer formation when compared with standard mattresses. days in bed. REFERENCES 1. patients who develop a pressure ulcer within 6 weeks of hospitalization have a threefold increase in mortality rate compared with those who do not. impaired wound healing. Among these factors. and type IV ulcers involve extensive destruction with full-thickness skin breakdown and involvement of bone. type III ulcers feature deep cratering with full-thickness skin breakdown but intact underlying fascia. convincing data supporting their use are limited. A recent retrospective study reported that an ICU stay > 7 days significantly increased the incidence of ulcer formation. Risk factors for ulcer formation include immobility. Multivariate analysis of these patients showed that independent risk factors included emergency admission. none is superior to another. conveying a 36-fold increased risk. Hydo LJ. malnutrition. Although nutritional supplementation and passive positioning are believed to help decrease the incidence of pressure ulcers.Thomas DR: Issues and dilemmas in the prevention and treatment of pressure ulcers: a review. Crit Care Med 29:1678-1682. and days without nutrition. muscle. trauma. and older age. Pressure-relieving devices designed to decrease pressure at the tissue interfaces are the most effective method for preventing pressure ulcers. J Gerontol A Biol Sci Med Sci 56:M328-M340. In fact.Eachempati SR. elderly age. depressed consciousness. APACHE III scores do not correlate with decubitus ulcer formation.American College of Surgeons SESAP 13 Critique for Category 8 Item 2 Decubitus pressure ulcers can produce significant morbidity and mortality in hospitalized patients. type II ulcers feature shallow cratering with partial-thickness skin breakdown. 2001 2. nonblanchable erythema. emergency admission to the ICU correlated the most strongly with decubitus ulcer formation. inadequate nursing care. and supporting structures. The catheter is removed and the tip is cultured. Blood cultures are positive for methicillin-sensitive S aureus at 48 hours. The most appropriate treatment now would be to (A) discontinue antibiotics and reculture if the patient again develops a fever (B) add gentamicin and continue antibiotics for 7 days (C) continue with vancomycin for 7 days (D) switch antibiotics to cephalexin for 7 days (E) continue nafcillin for 14 days page 724 .1°F. The catheter site is red with some tenderness. The patient is currently afebrile. Peripheral blood cultures are obtained. The patient begins to receive nafcillin.American College of Surgeons SESAP 13 Category 8 Item 3 A 35-year-old patient who has a right subclavian venous catheter for total parenteral nutrition (TPN) spikes a fever to 102. The tip grows > 15 colonies of Staphylococcus aureus at 24 hours. the catheter should be removed and its tip quantitatively cultured. or has purulent drainage. a 10. painful. Philadelphia. the diagnosis is a true catheter-related infection. Sherertz RJ. Raad II. ed 8. et al: Guidelines for the management of intravascular catheter-related infections.American College of Surgeons SESAP 13 Critique for Category 8 Item 3 The diagnosis of catheter-related bloodstream infection is made by examination of the catheter insertion site. a diagnosis of catheter colonization is made. 2001 page 725 . in Cameron JL (ed): Current Surgical Therapy. For coagulase-negative Staphylococcus. If the catheter tip and the blood cultures are positive for the same organisms. Antimicrobial therapy is not necessarily required because removal of the catheter is considered therapeutic if the patient has no other signs of infection. If the catheter tip is positive and the peripheral blood cultures are negative. REFERENCES 1. If both cultures are positive for coagulasepositive Staphylococcus. pp 1096-1100 2.to 14-day course is recommended to reduce the incidence of secondary infections such as epidural abscesses.Cocanour CS: Catheter sepsis in the intensive care unit. Clin Infect Dis 32:1249-1272. and culture of peripherally drawn blood. If the insertion site is erythematous. Farr BM. 2004. a 5. Intravenous antibiotics are indicated.to 10-day course is recommended. Mosby. culture of the catheter tip itself.Mermel LA. American College of Surgeons SESAP 13 Category 8 Item 4 Inverse-ratio mechanical ventilation (A) is more physiologic than conventional volume ventilation (B) cannot be used for permissive hypercapnia (C) can result in significant auto-PEEP (D) requires less sedation (E) improves venous return to the heart page 726 . This may have a detrimental effect on venous return to the heart.Bidani A. Johnson JL. In this situation. Zwischenberger JB: Mechanical ventilatory assistance. Tuazon DM. and can be achieved with inverse ratio ventilation. High inverse ratios do not allow adequate expiratory time. As a result. Churchill Livingstone. Inverseratio mechanical ventilation is the technique of inverting this ratio to make inspiration time longer than expiration time. Increased sedation and chemical paralysis may be needed to improve patient compliance. 2002. New York. Deslauriers J. 2004. patients may start stacking their breaths. The increased time for positive-pressure inspiration increases the positive pressure in the thoracic cavity. particularly in a hypovolemic patient. Greene KE: Respiratory insufficiency. resulting in autoPEEP. Mattox KL (eds): Trauma. pp 1295-1309 page 727 .Offner PJ. Inverting the ratio changes the patient’s normal ventilatory cycle. Alpard SK. in Moore EE. Feliciano DV. ed 5. Cooper JD.American College of Surgeons SESAP 13 Critique of Category 8 Item 4 Physiologic respirations typically have an inspiration to expiration ratio of 1:2 or greater. REFERENCES 1. ed 2. et al (eds): Thoracic Surgery. in Pearson FG. which can cause anxiety or discomfort in some patients. Permissive hypercapnia is used to reduce barotrauma. each inspiration occurs before expiration is complete. New York. Ginsberg RJ. McGraw-Hill. pp 155-162 2. American College of Surgeons SESAP 13 Category 8 Item 5 Which of the following statements about stress ulceration in the ICU is TRUE? (A) The incidence of bleeding in patients receiving prophylaxis for stress ulceration is 5% to 7% (B) The most common indication for initiation of stress ulceration prophylaxis is shock/hypotension (C) Sucralfate prophylaxis is associated with an increased incidence of stress-related mucosal bleeding compared with H2 receptor blocker prophylaxis (D) Sucralfate prophylaxis is associated with a lower incidence of nosocomial pneumonia compared with H2 receptor blocker prophylaxis (E) Gastric pH > 6 is necessary to maintain effective stress ulcer prophylaxis page 728 . Steinberg KP: Stress-related mucosal disease in the critically ill patient: risk factors and strategies to prevent stress-related bleeding in the intensive care unit. When ranitidine was compared with sucralfate in another recent large randomized trial. A recent randomized comparison of 1200 ICU patients reported a statistically increased incidence of stress-related mucosal bleeding in the sucralfate arm (3. 2002 page 729 . Rebuck JA. The overall incidence of bleeding in patients receiving prophylaxis for stress ulceration is 2. no significant differences in rates of pneumonia were found between the two groups.5% to 3%. sucralfate. 2004 2. sepsis.Daley RJ. used in 12% of patients. Rogers FB: Prevention of stress ulceration: current trends in critical care. Welage LS. The gastric pH necessary to maintain effective stress ulcer prophylaxis is 4 or above. The pharmacologic agents most commonly used for stress ulcer prophylaxis include histamine (H2) receptor blockers.8%). followed by shock/hypotension. Crit Care Med 32:2008-2013. compared with ranitidine (1. used in 23% of patients. used in 64% of patients. REFERENCES 1. and the proton pump inhibitors (PPIs). and head injury/major neurologic insult. The most common indication for initiation of stress ulceration prophylaxis is respiratory failure.7%). Crit Care Med 30:S362-S364. Smaller trials comparing H2 receptor antagonists with proton pump inhibitors for stress ulcer prophylaxis and incidence of nosocomial pneumonia in the critical care setting have typically found no difference with regard to these outcome measures.American College of Surgeons SESAP 13 Critique for Category 8 Item 5 Stress ulceration continues to be a problem in the ICU. American College of Surgeons SESAP 13 Category 8 Item 6 Low molecular weight heparin (LMWH) compared with unfractionated intravenous heparin (UIH) for the treatment of pulmonary embolism is associated with a (A) higher incidence of recurrent symptomatic venous thromboembolism at the end of treatment (B) higher incidence of all-cause mortality (C) lower incidence of major bleeding complications (D) higher incidence of recurrent symptomatic venous thromboembolism at 3 months (E) lower risk of heparin-induced thrombocytopenia page 730 . There was also no significant difference in the incidence of major bleeding complications. Olsson CG. incidence of recurrent symptomatic venous thromboembolism at the end of treatment and at 3 months follow-up. Eikelboom JW: Low-molecular-weight heparin compared with intravenous unfractionated heparin for treatment of pulmonary embolism: a meta-analysis of randomized. nor any difference in the incidence of allcause mortality at 1 and 3 months. controlled trials. In terms of thromboembolic recurrences and major bleeding complications. Abildgaard U. McQuillan A. However.Quinlan DJ. and its use as first-line anticoagulant therapy for pulmonary embolism is supported. LMWH is as effective and safe as IUH for the initial treatment of pulmonary embolism. REFERENCES 1. 2004 page 731 . The results did not differ regardless of whether patients presented initially with symptomatic pulmonary embolism. or possibly a lower.Merli G.American College of Surgeons SESAP 13 Critique for Category 8 Item 6 Many clinicians continue to believe that dose-adjusted intravenous unfractionated heparin (IUH) is safer and more effective than low molecular weight heparin (LMWH) for treatment of pulmonary embolism. Ann Intern Med 140:175-183. several recent randomized trials comparing the two have shown that LMWH has at least the same. et al: Subcutaneous enoxaparin once or twice daily compared with intravenous unfractionated heparin for treatment of venous thromboembolic disease. Ann Intern Med 134:191-202. Spiro TE. 2001 2. Its superior convenience and lower risk for heparininduced thrombocytopenia make it an attractive alternative to IUH for treatment of pulmonary embolism. American College of Surgeons SESAP 13 Category 8 Item 7 Which of the following statements about central venous catheter infection in the ICU is TRUE? (A) Hydrophobic catheters can reduce the incidence of infection (B) Subclavian and internal jugular vein catheters have similar rates of infection (C) A single positive culture of blood collected through a central venous catheter is an indication for removal of the catheter (D) Semiquantitative cultures are more sensitive than quantitative cultures of the catheter tip for diagnosing catheter infection (E) Catheter infection is best established by comparing the number of colony-forming units (CFU) in blood drawn through the catheter to the number in peripheral blood page 732 . The cutoff point differed between the various methods used for catheter culture: > 15 colony-forming units (CFU) for the semiquantitative method or at least a 1000 CFU for the quantitative methods. one through the suspected central venous catheter and one through peripheral venipuncture. This is particularly likely if there is no other apparent source of infection. and iatrogenic factors. Soon after insertion.American College of Surgeons SESAP 13 Critique for Category 8 Item 7 Microbial colonization and catheter-related bloodstream infections result from the interaction of microbial factors. attached to the catheter surface. The use of paired quantitative blood cultures allows the diagnosis of catheter-related bloodstream infection without the removal of the catheter. The hydrophobic nature of central venous catheters typically increases their predisposition toward infection. Based on this approach. The nonspecific manifestations of central venous catheter infection include fever. increased cost. and occasionally hypotension. The time required for the culture to become positive is a useful tool in the quest for diagnosis of catheterrelated bloodstream infection. including hydrophobicity and exopolysaccharide production. Catheterrelated bloodstream infection is suggested if the number of CFU isolated from blood collected through the central venous catheter is at least 5 times the number of CFU isolated from the peripherally collected blood sample. leading to the formation of a microbial biofilm that enables the microorganisms to attach to the catheter surface. A single positive blood culture collected through the catheter may indicate nothing more than universal colonization of the catheter with microbes. including fibrin and fibronectin. where they are resistant to antimicrobial agents. The hydrophobic characteristics of these organisms are key factors in their attachment to the catheter polymer surface. are isolated from one or more blood cultures. catheter material. two blood samples are collected simultaneously. A recent meta-analysis showed that quantitative catheter cultures are more sensitive than semiquantitative cultures in establishing the diagnosis of catheter-related bloodstream infection. thrombin sheets rich in host-derived protein adhesins such as fibrinogen and fibronectin form around the internal/external surface of the catheter. and polyethylene. The organisms become embedded in the biofilm. and thrombogenicity. leading to unnecessary antimicrobial therapy and wasteful catheter removal. A catheter-related bloodstream infection was formerly confirmed by catheter removal and culture with either a semiquantitative or quantitative catheter/culture method. surface charges. This last sign is most often associated with bloodstream infections caused by gram-negative organisms or Candida. including polyvinylchloride. Microorganisms then undergo phenotypic changes. host factors. chills. The diagnosis of catheter-related bloodstream infection should be suspected if common skin organisms. resulting in a release of enzymes that produce an extracellular slime or exopolysaccharide. This time is proportionately page 733 . Microbial organisms typically migrate from the skin insertion site or the hub of the catheter and attach themselves to the catheter polymers. such as administration of total parenteral nutrition (TPN). such as protein adhesins. Infection was confirmed if the same organism isolated from the blood cultures was retrieved in significant numbers from the catheter tip culture in the absence of another apparent source of infection. The diagnosis of central venous catheter infection can be problematic. These proteins become the receptors to which various organisms such as Staphylococcus aureus. including Staphylococcus aureus or Candida. Staphylococcus epidermidis. including hydrophobicity. The incidence of catheter-related bloodstream infections is often overestimated. and inconvenience for patients. and Candida albicans attach themselves. silicone. 2002 page 734 . Infect Control Hosp Epidemiol 25:675-677. Angeloro P. Barboza L. Kluger DM. femoral vein catheterization is associated with a high risk for deep venous thrombosis. Raad II: Intravascular catheter-related infections: a preventable challenge in the critically ill. The use of 2% aqueous chlorhexidine solution is associated with an 84% lower rate of catheter-related bloodstream infection compared with a solution of 10% povidone iodine and 70% alcohol. 2004 3. The difficulty immobilizing a catheter inserted into the internal jugular vein. The risk for infection of central venous catheters is significantly decreased if the site of catheterization is the subclavian vein rather than internal jugular or femoral vein.American College of Surgeons SESAP 13 related to the microbial inoculum in the blood sample. Medicine 81:466-479. REFERENCES 1. Semin Respir Infect 15:264-271. Catheter-related bloodstream infection is suspected if the blood collected through a central venous catheter becomes positive at least 2 hours earlier than a concurrent culture of peripheral venous blood.Gnass SA.Safdar N. Bilicich D.Chatzinikolaou I. Applying topical antimicrobial or antiseptic agents to the skin insertion site can also reduce infection rates by preventing the skin fold from colonizing the catheter. noncuffed central venous catheters: implications for preventive strategies. in combination with proximity to oropharyngeal secretions. 2000 2. et al: Prevention of central venous catheter-related bloodstream infections using non-technologic strategies. In addition. seems to be responsible for the increased risk for infection of central venous catheters placed in the internal jugular vein. Using maximal sterile barriers during catheterization significantly reduces the incidence of catheter-related bloodstream infections. Maki DG: A review of risk factors for catheter-related bloodstream infection caused by percutaneously inserted. and serum calcium. Pertinent data include serum sodium. serum chloride. his course is complicated by respiratory failure. serum potassium.4 mEq/L. 12 mEq/L. 4 mEq/L. He then develops recurrent episodes of bradycardia complicated by asystole requiring cardiopulmonary resuscitation. Four weeks after admission.American College of Surgeons SESAP 13 Category 8 Item 8 A 28-year-old man sustains severe blunt trauma to the torso after a high-speed motor vehicle crash. In the ICU. serum phosphate. 134 mEq/L. 109 mEq/L. 2. serum magnesium. liver failure. he has recovered from renal failure.6 mEq/L. Definitive treatment of his bradycardia/asystole requires (A) hydration (B) loop diuresis (C) atropine (D) external pacing (E) biphosphonate therapy page 735 . 4. and acute oliguric renal failure requiring dialysis. 19 mEq/L. HCO3. coagulopathy. history of bacteremia. Burchard KW. have had at least one episode of hypotension and shock. Querusio L. treatment with total parenteral nutrition (TPN) > 21 days. and external pacing. Hypercalcemia-induced bradycardia is typically refractory to hydration. when symptomatic hypercalcemia manifests as bradycardia. 2005 page 736 . REFERENCES 1. A syndrome of tertiary hyperparathyroidism that occurs after recovery from multiple organ failure with acute oliguric renal failure has recently been described. Ann Surg 202:512-518. Gann DS: Hypercalcemia in critically ill surgical patients. among other conditions. 1985 2. which follows even into the resolution phase of this illness. need for dialysis. an ICU stay > 36 days. Exposure of these patients to events that induce low concentrations of ionized calcium stimulates parathormone secretion. loop diuretics. transfusion requirement > 24 units. Ledgerwood AM. This ultimately results in autonomous secretion of parathyroid hormone. which must be repeated until the patient’s organ system function has returned to normal. Bradycardia can be severe.Forster J.Jeffries CC. sometimes leading to asystole requiring cardiopulmonary resuscitation. Definitive treatment of this syndrome requires biphosphonate therapy. Patients who develop it typically have a higher incidence of renal failure. Am J Surg 189:369-372. Lucas CE: Life-threatening tertiary hyperparathyroidism in the critically ill. and a treatment course of antibiotic therapy > 7 days.American College of Surgeons SESAP 13 Critique for Category 8 Item 8 Tertiary hyperparathyroidism most commonly occurs in patients who have recovered from renal failure after renal transplantation. Approximately 15% of critically ill surgical patients develop hypercalcemia. atropine. American College of Surgeons SESAP 13 Category 8 Item 9 A 53-year-old woman undergoes coronary artery bypass grafting and mitral valve replacement with a St. Jude mechanical prosthesis. Warfarin (Coumadin) therapy is initiated postoperatively, but on postoperative day 5 she has the acute onset of right calf pain, swelling, and tenderness. Duplex scanning demonstrates acute deep venous thrombosis of the right femoral-popiliteal veins. Intravenous unfractionated heparin therapy is begun. On postoperative day 9, she has the acute onset of shortness of breath, hypoxemia, and chest pain. Computed tomographic (CT) angiography demonstrates multiple, bilateral pulmonary emboli. In addition, her platelet count has fallen to 50,000. Definitive management of this problem should be (A) discontinuing unfractionated heparin (B) discontinuing unfractionated heparin and beginning low molecular weight heparin (C) discontinuing unfractionated heparin and beginning argatroban (D) continuing unfractionated heparin and placing an inferior vena cava filter (E) continuing unfractionated heparin and immediate platelet transfusion page 737 American College of Surgeons SESAP 13 Critique for Category 8 Item 9 This clinical scenario describes a case of heparin-induced thrombocytopenia (HIT), a potentially catastrophic generalized thrombotic disorder triggered by heparin therapy. HIT typically begins 4 to 14 days after initiation of therapy with intravenous unfractionated heparin, although it can be associated with any type of heparin, given at any dose and by any route. For patients receiving heparin therapy, the risk of developing HIT is 2% to 5%. The diagnosis should be suspected whenever the platelet count is less than 150,000, or decreased 50% or more from baseline. Thrombocytopenia is typically moderately severe with platelet counts ranging from 50,000 to 75,000. Thrombotic complications are responsible for the severe morbidity and mortality associated with HIT. One half to two thirds of patients with HIT may have thromboembolic complications. HIT has two major subtypes. Type 1 is associated with a mild thrombocytopenia, usually occurs within 4 days of starting therapy, is not immune mediated, appears to be caused by a direct agglutinating effect of heparin on platelets, is not associated with thrombosis, and resolves despite the continuation of heparin therapy. HIT type 2 is associated with severe thrombocytopenia and/or a significant fall in the baseline platelet count, usually occurring 4 to 14 days after heparin therapy is initiated. It is immune mediated and may be associated with both venous and arterial thrombosis. Type 2 HIT is generally associated with intravenous infusion of unfractionated heparin, but has also been reported with subcutaneous low-dose heparin, heparin flushes, heparin-coated catheters, and low molecular weight heparins (LMWH). With type 2 HIT, heparin exposure stimulates the release of platelet factor 4 (PF4), which is a heparin-neutralizing protein found in the alpha granules of platelets. Heparin and PF4 then form a complex that stimulates the production of immunoglobulin G (IgG) antibodies. The heparin, PF4, and IgG immune complexes bind to the Fc receptors on the platelet surfaces, resulting in platelet activation, aggregation, release of prothrombotic, platelet derived micro particles, and the late development of thrombocytopenia. Treatment of HIT requires immediate discontinuation of unfractionated heparin and the beginning of treatment with a direct thrombin inhibitor such as argatroban, bivalirudin, or lepirudin. These agents inhibit the formation of fibrin-bound thrombin, preventing thrombus extension and growth. They also produce a more predictable anticoagulant response because they do not bind to plasma proteins and are not neutralized by PF4. A number of prospective studies have demonstrated decreased mortality, limb amputation, and thrombotic complications when thrombin inhibitors are used to treat HIT. Platelet transfusion is contraindicated because it can promote thrombosis in the setting of HIT. Multiple studies have shown that merely discontinuing heparin with no subsequent anticoagulation therapy is unacceptable. Up to 50% of patients who merely have their unfractionated heparin discontinued will suffer a subsequent thrombotic event. LMWH is contraindicated for treatment of HIT because it cross-reacts with the antibodies that cause HIT, thereby worsening the syndrome. Warfarin is contraindicated as acute treatment in this setting because it can paradoxically worsen the thrombosis and cause venous gangrene. The heparinoid danaparoid sodium has been used successfully in some patients, but danaparoid cross-reacts with approximately 10% of HIT antibodies and can cause progression of the syndrome. Placement of an inferior vena cava filter does not prevent HIT complications occurring proximal to the filter, or in the arterial system, and is not indicated in this setting. Institution of immediate fibrinolytic therapy would be contraindicated in this patient who has recently undergone coronary artery bypass grafting and mitral valve replacement. page 738 American College of Surgeons SESAP 13 REFERENCES 1.Kondo LM, Wittkowsky AK, Wiggins BS: Argatroban for prevention and treatment of thromboembolism in heparin-induced thrombocytopenia. Ann Pharmacother 35:440-451, 2001 2.Lewis BE, Wallis DE, Leya F, Hursting MJ, et al: Argatroban anticoagulation in patients with heparininduced thrombocytopenia. Arch Intern Med 163:1849-1856, 2003 3.Warkentin TE, Greinacher A: Heparin-induced thrombocytopenia: recognition, treatment, and prevention: the Seventh ACCP Conference on Antithrombotic and Thrombolytic Therapy. Chest 126:311S-337S, 2004 page 739 American College of Surgeons SESAP 13 Category 8 Item 10 Each of the following measures decreases the incidence of ventilator-associated pneumonia in surgical patients EXCEPT (A) elevation of the head of the bed to 45 degrees (B) aspiration of subglottic secretions (C) frequent ventilator circuit changes (D) selective digestive tract decontamination (E) use of oscillating beds page 740 American College of Surgeons SESAP 13 Critique for Category 8 Item 10 A number of prospective, randomized clinical trials have evaluated physical and positional strategies for the prevention of ventilator-associated pneumonia (VAP). Prevention strategies examined include the route of endotracheal intubation; a systematic search for maxillary sinusitis; frequency of ventilator circuit changes; airway humidification; endotracheal suctioning systems; subglottic secretion drainage; chest physiotherapy; elevation of the head of the bed, and oscillating beds; and timing of tracheostomy. A review of these factors permits a number of recommendations about the relative value of such strategies. Orotracheal intubation is associated with a lower incidence of VAP compared with nasotracheal intubation, and should be used when possible. Although a systematic search for maxillary sinusitis in patients intubated nasotracheally may decrease the incidence of VAP, this practice cannot be supported for patients intubated orotracheally. The frequency of ventilator circuit changes does not influence the incidence of VAP. Less frequent changes are not associated with harm, and more frequent changes are associated with increased cost. A new circuit should be used for each patient and should be changed if it becomes soiled. Heat and moisture exchangers may be associated with a slightly decreased incidence of VAP compared with heated humidifiers. They should be used in patients who have no contraindication such as hemoptysis or requirement of high-frequency jet ventilation, and should be changed weekly. Closed endotracheal suction systems, changed for each patient and as clinically indicated, are useful in preventing VAP, but scheduled daily changes and nonscheduled changes in closed suctioning systems have no impact on the incidence of VAP. Subglottic secretion drainage is associated with a decreased incidence of VAP, especially early-onset VAP, and is recommended. As yet, there is no definitive evidence that chest physiotherapy decreases the incidence of VAP, and evidence to correlate the incidence of VAP with early versus late tracheostomy is insufficient. Kinetic or oscillating beds are associated with a decreased incidence of VAP and should be considered for at-risk patients. Positioning patients at 45 degrees from horizontal is also associated with a decreased incidence and should be used in the absence of any contraindication. Sucralfate does not influence the incidence compared with placebo. Based on evidence from 10 meta-analyses, selective digestive decontamination is associated with a decreased incidence. However, these therapies should be time-limited to prevent the emergence of resistant organisms. REFERENCES 1.Collard HR, Saint S, Matthay MA: Prevention of ventilator-associated pneumonia: an evidence-based systematic review. Ann Intern Med 138:494-501, 2003 2.Dodek P, Keenan S, Cook D, Heyland D, et al: Evidence-based clinical practice guideline for the prevention of ventilator-associated pneumonia. Ann Intern Med 141:305-313, 2004 page 741 American College of Surgeons SESAP 13 Category 8 Item 11 Which of the following statements about alcohol withdrawal syndrome in the patient with multiple injuries is TRUE? (A) Dilantin (phenytoin) should be administered prior to benzodiazepines to reduce the risk of seizures (B) Beta blockers are effective in preventing the progression to delirium tremens (C) Intravenous alcohol is superior to benzodiazepines for prevention of the syndrome (D) Propofol (Diprivan) may be used as an alternative to benzodiazepines for treatment of established delirium tremens (E) Prophylaxis regimens that include clonidine in addition to benzodiazepines reduce the incidence of cardiac complications page 742 American College of Surgeons SESAP 13 Critique of Category 8 Item 11 There are four stages of alcohol withdrawal. The first stage is autonomic hyperactivity, in which clinical symptoms appear within hours of the cessation of alcohol and peak within 24 to 48 hours. There is usually a clear sensorium, but this is often accompanied by tremulousness, sweating, anxiety or agitation, insomnia, nausea, and vomiting. This stage is secondary to sympathetic outflow, documented by increased circulating catecholamine levels in the urine, serum, and the cerebrospinal fluid. Approximately 25% of patients progress to a more severe second stage characterized by hallucinations. The third stage is neuronal excitation, which is accompanied by seizure activity in up to 10% of patients. The seizures are usually single, short, and generalized tonic-clonic. The fourth stage is delirium tremens. Occurring in up to 5% of patients, delirium tremens usually begins at 48 to 72 hours, but may be delayed up to 4 to 5 days. Approximately 30% of alcoholics with seizures progress to this stage. Delirium tremens is characterized by disorientation, confusion, impaired attention, pronounced autonomic hyperactivity, and visual and auditory hallucinations. If left untreated, mortality rates may reach 15%. Mortality is usually caused by cardiovascular or respiratory collapse. Benzodiazepines are the mainstay of therapy. Beta blockers may be useful to control tachycardia, but they do not prevent or treat delirium tremens. The role of intravenous alcohol in the prevention and treatment of alcohol withdrawal syndrome is controversial. Intravenous alcohol appears to be equivalent but not superior to benzodiazepines for preventing the syndrome. Propofol (Diprivan) has gained support as an alternative to benzodiazepines for treatment of delirium tremens. The addition of clonidine to prophylaxis regimens does not reduce the incidence of cardiac complications. There is no evidence that phenytoin should be administered prior to benzodiazepines. REFERENCES 1.Al-Sanouri I, Dikin M, Soubani AO: Critical care aspects of alcohol abuse. South Med J 98:372-381, 2005 2.Gentilello LM: Alcohol and drugs, in Moore EE, Feliciano DV, Mattox KL (eds): Trauma, ed 5. New York, McGraw-Hill, 2004, pp 1007-1020 3.Schmidt GA: Special problems in the ICU, in Hall JB, Schmidt GA, Wood LDH (eds): Principles of Critical Care, ed 3. New York, McGraw-Hill, 2005, pp 1465-1472 page 743 American College of Surgeons SESAP 13 Category 8 Item 12 In critically ill septic trauma patients, transfusion of packed RBCs (A) reduces ventilation/perfusion mismatch (B) reduces the need for erythropoietin (C) increases septic morbidity and mortality (D) is indicated for a hemoglobin of 8 g/dL (E) reduces length of stay in the ICU page 744 American College of Surgeons SESAP 13 Critique for Category 8 Item 12 Many clinicians believe that the practice of transfusing packed RBCs in critically ill septic trauma patients deserves more scrutiny, given an increased awareness of bloodborne pathogens, and evidence that allogeneic packed RBCs may increase susceptibility to infection, thereby increasing morbidity and mortality. In a 1999 Canadian Clinical Trials Group randomized controlled trial comparing transfusion at a trigger of 7 g/dL of hemoglobin versus 10 g/dL of hemoglobin, a trigger of 7 g/dL was at least as safe as a trigger of 10 g/dL, as measured using the primary endpoint of 30-day mortality of all causes and several other secondary endpoints. In several subgroups (patients under 55 years and patients with APACHE II scores < 20), the lower transfusion trigger was associated with a statistically significant mortality benefit. Using a modified Delphi method and consensus conference, the participants concluded that in the absence of extenuating circumstances and following resolution of tissue hypoperfusion, RBC transfusion should be targeted to maintain hemoglobin at 7 g/dL. Erythropoietin is not recommended as a specific treatment for sepsis-associated anemia. Transfusion is not associated with a reduction in ventilation/perfusion mismatch or reducing the length of ICU stay. REFERENCES 1.Hebert PC, McDonald BJ, Tinmouth A: Clinical consequences of anemia and red cell transfusion in the critically ill. Crit Care Clin 20:225-235, 2004 2.Hebert PC, Wells G, Blajchman MA, Marshall J, et al: A multicenter, randomized, controlled clinical trial of transfusion requirements in critical care. Transfusion Requirements in Critical Care Investigators, Canadian Critical Care Trials Group. N Engl J Med 340:409-417, 1999 3.McIntyre L, Hebert PC, Wells G, Fergusson D, et al: Is a restrictive transfusion strategy safe for resuscitated and critically ill trauma patients? J Trauma 57:563-568, 2004 4.Napolitano LM, Corwin HL: Efficacy of red blood cell transfusion in the critically ill. Crit Care Clin 20:255-268, 2004 5.Napolitano LM: Current status of blood component therapy in surgical critical care. Curr Opin Crit Care 10:311-317, 2004 6.Zimmerman JL: Use of blood products in sepsis: an evidence-based review. Crit Care Med 32:S542S547, 2004 page 745 American College of Surgeons SESAP 13 Category 8 Item 13 Which of the following statements about activated factor VII is TRUE? (A) Causes systemic hypotension (B) First-line therapy for coagulopathy of trauma (C) Thrombotic complications do not occur (D) Effective in acidotic patients (E) Effective for reversal of warfarin anticoagulation page 746 American College of Surgeons SESAP 13 Critique of Category 8 Item 13 Recombinant activated factor VII (rVIIa) received FDA approval in 1999 for treatment of bleeding in patients with hemophilia A or B. In pharmacologic doses, rVIIa binds with tissue factor (TF), a cellbound complex exposed to blood in the setting of injury. The TF-VIIa complex activates factors X and IX, leading to production of small amounts of thrombin. This thrombin accelerates the coagulation process by activating platelets, factor V, and factor VIII. Factor VIIIa then combines with factor IX to generate more factor X, greatly increasing thrombin production. rVIIa may also enhance thrombin generation on the platelet surface via an interaction with factor X, independent of TF. rVIIa is believed to have its greatest effect at sites of injury where TF is exposed and where platelets are activated, and does not appear to systemically activate coagulation. Thrombotic complications have been reported, however, and caution is warranted in patients with atherosclerotic disease. Off-label use of rVIIa has been increasing. The most accepted use to date is for emergency reversal of warfarin (vitamin K antagonist) anticoagulation, particularly in patients with intracerebral hemorrhage. Trauma patients with ongoing life-threatening bleeding due to coagulopathy have been shown retrospectively to benefit from rVIIa administration. Most algorithms suggest that bleeding amenable to surgical or angiographic control be stopped prior to administration of rVIIa; conventional prothrombotic therapy be underway; pH be corrected if possible; and rVIIa therapy be withheld from patients in extremis. There is in vitro evidence that a decrease in pH from 7.4 to 7.0 reduces rVIIa activity by over 90%. Off-label uses are also reported in settings of perioperative hemorrhage, platelet-related bleeding disorders, bone marrow transplantation, hepatic dysfunction, hepatic transplantation, neonatal and pediatric coagulopathy, and other coagulation factor deficiencies. Hypertension is an adverse reaction in up to 10% of patients receiving rVIIa. REFERENCES 1.Berletta JF, Ahrens CL, Tyburski JG, Wilson RF: A review of recombinant factor VII for refractory bleeding in nonhemophilic trauma patients. J Trauma 58:646-651, 2005 2.O’Connell KA, Wood JJ, Wise RP, Lozier JN, et al: Thromboembolic adverse events after use of recombinant human coagulation factor VIIa. JAMA 295:293-298, 2006 3.Ratko TA: UHC high-impact technology brief: off-label use of recombinant activated factor VII (NovoSeven). Oakbrook, IL: UHC Clinical Practice Advancement Center, University HealthSystem Consortium, 2003 4.Recombinant human factor VIIa: Drug information. Select Drug Information from Lexi-Comp Online. Copyright (1978-present) Lexi-Comp, Inc. in Up-to-Date Online 13.3 page 747 American College of Surgeons SESAP 13 Category 8 Item 14 All of the following are effective in decreasing mortality in severe sepsis EXCEPT (A) achieving central venous oxygen saturation of 70% within 6 hours of presentation (early goaldirected therapy) (B) activated protein C (rhAPC) administration (C) achieving supranormal oxygen delivery (> 600 mL/min/m2) (D) intravenous corticosteroids for ACTH nonresponders (E) tight glucose control (80 to110 mg/dL) page 748 American College of Surgeons SESAP 13 Critique for Category 8 Item 14 Systemic inflammatory response syndrome (SIRS) is defined as two or more of the following: temperature > 38°C or < 36°C, pulse > 90 beats/min, respiratory rate > 20, or PaCO2 < 32 mm Hg. Sepsis is SIRS in the setting of documented infection, and severe sepsis is sepsis accompanied by organ dysfunction or signs of hypoperfusion (lactic acidosis, oliguria, or altered mental status). When hypotension is also present, the patient is considered to be in septic shock. Approximately 750,000 cases of sepsis occur per year in the United States, and the mortality rate of severe sepsis is 30% to 50%. In 1995, severe sepsis accounted for 9.3% of all deaths. While research has focused on understanding the mechanisms underlying the cellular and systemic responses to infection, a “magic bullet” approach to halting this complex cascade of inflammatory mediators has been difficult to attain. These mediators of inflammation activate coagulation and cause diffuse endovascular injury, which may cause organ failure in severe sepsis. Activated protein C (APC) is an endogenous protein that promotes fibrinolysis and inhibits thrombosis and inflammation. Levels of endogenous protein C are reduced in septic patients. Exogenously administrated APC reduces mortality in patients with severe sepsis and organ failure from 31% to 25%. APC predisposes to severe bleeding, and must be used with caution perioperatively and in patients at risk for bleeding. Other more generalized approaches to the treatment of severe sepsis are also beneficial. Early goal-directed resuscitation involves aggressive correction of central venous PO2 within the first 6 hours after presentation using protocol-driven administration of intravenous fluids, pressors, and transfusion of blood. In a study of patients with severe sepsis and septic shock, mortality was reduced from 47% to 31%. Patients with severe sepsis and septic shock shown to have adrenal insufficiency had a reduction in mortality from 63% to 53% when given a regimen of hydrocortisone and fludrocortisone. In patients admitted to a surgical ICU, tight blood glucose control (80 to 120 mg/dL) reduced mortality from 8.0% to 4.6%, mostly due to a reduction in deaths from multiple organ failure with a proven septic focus. All of these reductions in mortality were statistically significant. Supranormal oxygen delivery (oxygen delivery > 600 mL/min) does not reduce mortality in patients with severe sepsis and organ failure, and may in some cases increase the incidence of abdominal compartment syndrome. REFERENCES 1.Annane D, Sebille V, Charpentier C, Bollaert PE, et al: Effect of treatment with low doses of hydrocortisone and fludrocortisone on mortality in patients with septic shock. JAMA 288:862-871, 2002 2.Bernard GR, Vincent JL, Laterre PF, LaRosa SP: Efficacy and safety of recombinant human activated protein C for severe sepsis. N Engl J Med 344:699-709, 2001 3.Dellinger RP, Carlet JM, Masur H, Gerlach H, et al: Surviving Sepsis Campaign guidelines for management of severe sepsis and septic shock. Crit Care Med 32:858-873, 2004 4.Kern JW, Shoemaker WC: Meta-analysis of hemodynamic optimization in high-risk patients. Crit Care Med 30:1686-1692, 2002 5.Rivers E, Nguyen B, Havstad S, Ressler J, et al: Early goal-directed therapy in the treatment of severe sepsis and septic shock. N Engl J Med 345:1368-1377, 2001 page 749 American College of Surgeons SESAP 13 6.van den Berghe G, Wouters P, Weekers F, Verwaest C, et al: Intensive insulin therapy in the critically ill patients. N Engl J Med 345:1359-1367, 2001 page 750 American College of Surgeons SESAP 13 Category 8 Item 15 A 50-year-old man is admitted to the surgical ICU after aortoiliac reconstruction. On admission, his blood pressure is 80/40, heart rate is 130/minute, and urine output is 10 mL in the last hour. His abdomen is soft and nondistended. His last hematocrit in the operating room was 35%, and central venous pressure was 2 mm Hg. A decision is made to administer a fluid bolus intravenously. What type of fluid should be used for volume resuscitation in this patient? (A) Isotonic crystalloid (normal saline, Ringer’s lactate solution) (B) Hypertonic saline (C) 5% albumin (D) Hydroxyethyl starch (E) Whole blood page 751 American College of Surgeons SESAP 13 Critique for Category 8 Item 15 Isotonic crystalloid administration is adequate for fluid resuscitation in the patient described in this question. A large multicenter randomized, double-blind trial (SAFE Study: Saline vs Albumin Fluid Evaluation) comparing the effect of fluid resuscitation with albumin (5% human albumin) or normal saline (0.9% sodium chloride) on mortality in a heterogeneous population of 6997 ICU patients conducted by the Australian and New Zealand Intensive Care Society Clinical Trials Group identified no differences in mortality or organ failure. Rates of secondary outcomes — survival time, organ dysfunction, the duration of mechanical ventilatory support, the duration of renal replacement therapy, and the length of stay in the ICU and hospital — were also similar. An additional prospective randomized study comparing albumin and hydroxyethyl starch for hypoalbuminemic critically ill patients confirmed that raising colloid osmotic pressure with albumin administration was not associated with improved clinical outcome. Given the lack of differences in outcome and the increased risks associated with albumin administration in terms of cost and exposure to human infectious agents, isotonic crystalloid is recommended. Blood transfusion is also not indicated, based on the results of the Transfusion in Critical Care (TRICC) report documenting that a restrictive strategy of red cell transfusion (transfusion for hemoglobin < 7 g/dL) was as effective as a liberal strategy (hemoglobin < 10 g/dL) in 838 critically ill patients. There is no published evidence to suggest that hypertonic saline is superior to isotonic crystalloid in randomized human clinical trials. REFERENCES 1.Finfer S, Bellomo R, Boyce N, French J, et al: A comparison of albumin and saline for fluid resuscitation in the intensive care unit. N Engl J Med 350:2247-2256, 2004 2.Hebert PC, Wells G, Blajchman MA, Marshall J, et al: A multicenter, randomized, controlled clinical trial of transfusion requirements in critical care. Transfusion Requirements in Critical Care Investigators, Canadian Critical Care Trials Group. N Engl J Med 340:409-417, 1999 3.Veneman TF, Oude Nijhuis J, Woittiez AJ: Human albumin and starch administration in critically ill patients: a prospective randomized clinical trial. Wien Klin Wochenschr 116:305-309, 2004 page 752 American College of Surgeons SESAP 13 Category 8 Item 16 The strongest independent risk factor for stress-related mucosal damage and gastrointestinal bleeding in critically ill patients is (A) glucocorticoid administration (B) mechanical ventilatory support for 48 or more hours (C) hypotension (D) renal failure (E) organ transplantation page 753 American College of Surgeons SESAP 13 Critique for Category 8 Item 16 Stress-related mucosal disease (SRMD) includes a continuum of conditions, ranging from superficial mucosal damage to stress ulcers, caused by gastrointestinal mucosal ischemia, and is most commonly seen in critically ill patients in the ICU. Strong independent risk factors for SRMD and stressinduced gastrointestinal bleeding include respiratory failure (defined by mechanical ventilatory support for at least 48 hours) and coagulopathy (defined as a platelet count of < 50,000 mm3, and an international normalized ratio of >1.5, or a partial thromboplastin time of >2 times the control value). In a prospective observational study, the frequency of bleeding was 3.7% if one or both of these risk factors were present and 0.1%if neither factor was present. Simple regression analysis showed that sepsis, renal insufficiency, hepatic failure, enteral feeding, and the use of glucocorticoids, heparin, and warfarin were risk factors for bleeding; multiple regression analysis showed that only mechanical ventilatory support and coagulopathy were independently predictive of clinically important bleeding (see the table). These high-risk patients should receive stress ulcer prophylaxis. REFERENCES 1.Cook DJ, Fuller HD, Guyatt GH, Marshall JC, et al: Risk factors for gastrointestinal bleeding in critically ill patients. Canadian Critical Care Trials Group. N Engl J Med 330:377-381, 1994 2.Metz DC: Preventing the gastrointestinal consequences of stress-related mucosal disease. Curr Med Res Opin 21:11-18, 2005 3.[No Authors Listed]: ASHP Therapeutic Guidelines on Stress Ulcer Prophylaxis. ASHP Commission on Therapeutics and approved by the ASHP Board of Directors on November 14, 1998. Am J Health Syst Pharm 56:347-379, 1999 4.Spirt MJ: Stress-related mucosal disease: risk factors and prophylactic therapy. Clin Ther 26:197213, 2004 page 754 American College of Surgeons SESAP 13 Category 8 Item 17 A femorotibial bypass is planned for a 70-year-old man with rest pain. The patient has a long history of stable angina, with risk factors of diabetes mellitus, heavy smoking, hypertension, and hypercholesterolemia. Coronary catheterization demonstrates stenoses in 3 vessels and a left ventricular ejection fraction of 45%. The next step should be (A) coronary artery bypass with balanced general anesthesia (B) coronary angioplasty and placement of drug-eluding stents (C) thallium stress test to determine the amount of myocardium at risk (D) femorotibial bypass with perioperative beta blockade (E) below-knee amputation under regional anesthesia page 755 American College of Surgeons SESAP 13 Critique for Category 8 Item 17 Coronary artery disease (CAD) is the leading cause of operative deaths in vascular surgery, and it is the most common cause of late mortality in patients with peripheral arterial disease (PAD). The high prevalence of CAD among vascular patients has been recognized for many years, but operative mortality rates remain far below rates that would be predicted on the basis of coronary angiography. Attempts to predict patients at risk for cardiac events have included risk indices, preoperative screening programs, and cardiac stress tests. Although these tests are generally accurate for predicting patients at low risk, the best course of action for patients at moderate or high risk has been controversial. Patients with PAD have significantly higher morbidity after coronary artery bypass graft (CABG) than patients without PAD. Furthermore, it is not clear that the combined morbidity and mortality of coronary artery revascularization before elective major vascular surgery is different from morbidity and mortality of vascular operations without coronary artery revascularization. In a randomized, multicenter Veterans Affairs trial evaluating the benefit of coronary artery revascularization before elective major vascular surgery, 1190 of 5859 patients scheduled for vascular operations were deemed to have sufficient cardiac risk to warrant preoperative coronary angiography. Of these patients, 510 were randomly assigned to preliminary coronary artery revascularization (CABG or percutaneous catheter-directed therapies) or no coronary artery revascularization. Patients assigned to coronary artery revascularization had a significantly longer time from assignment to vascular surgery (18 vs 54 days), but there were no differences in the incidence of perioperative myocardial infarction, operative deaths, or late deaths between the two groups. These data show that coronary artery revascularization before a vascular operation would not reduce the incidence of perioperative or late cardiac complications in the present clinical scenario. CABG or percutaneous intervention should be reserved for a vascular patient with unstable angina or advanced coronary artery disease in whom a long-term survival benefit has been proved. Because coronary revascularization is not indicated before a vascular operation, routine thallium stress testing is not necessary. The best course of action is to proceed with the femoropopliteal bypass. Because revascularization is likely to be successful, primary amputation is not required in this case. Careful perioperative management is critical to ensure the best outcome for patients with cardiac disease undergoing noncardiac surgery. In addition to controlling blood pressure and providing optimal intravascular volume, beta blockers should be instituted during the preoperative period. In randomized trials comparing beta blockers with placebo, high-risk patients receiving beta blockers had a reduced rate of perioperative cardiac events and an improved 6-month survival. Beta blockers may also reduce the risk of myocardial infarction and death in high-risk patients. These agents should be started as soon as possible before operation only in high-risk patients, and the dose should be titrated to achieve a resting heart rate of 50 to 60 beats/minute. Benefit is not well-established in low- and moderate-risk patients. REFERENCES 1.Krupski WC, Nehler MR, Whitehill TA: Preoperative cardiac risk management. Cardiovasc Surg 10:415-420, 2002 2.McFalls EO, Ward HB, Moritz TE, Goldman S, et al: Coronary-artery revascularization before elective major vascular surgery. N Engl J Med 351:2795-2804, 2004 page 756 American College of Surgeons SESAP 13 Category 8 Item 18 A 21-year-old man with a history of alcohol abuse extricated unconscious from his apartment has a 45% total body surface area (TBSA) third-degree burn. He is intubated on admission, and bronchoscopy reveals soot, edema, and erythema throughout his trachea and bronchi. On postburn day 2, his ventilatory status deteriorates. All of the following may be contributing factors EXCEPT (A) tracheal and bronchial casts (B) bronchiolar fibrosis (C) bronchial mucus plugs (D) alveolar edema (E) reduced surfactant page 757 American College of Surgeons SESAP 13 Critique for Category 8 Item 18 The finding of inhalation injury is the most important prognostic factor for mortality after burn injury, regardless of total body surface area (TBSA) or the patient’s age. After significant smoke inhalation, the ciliary function of the respiratory lining ceases. The epithelial cells immediately separate from the basement membrane and intercellular bonds are disrupted. As with edema in the burn wound, the local injury to the mucosa of the airway initiates the cascade of acute inflammation, allowing plasma proteins to leak from the pulmonary microvasculature into the interstitial space. Edema develops and is further exacerbated by as much as a tenfold increase in blood flow to the lungs and tracheobronchial tree. Although serious pulmonary injury may exist without endoscopically apparent changes, initial bronchoscopy may demonstrate evidence of edema and hyperemia of the airways. Exudate formation occurs with the protein leak that can increase the interstitial and intra-alveolar oncotic pressures, resulting in alveolar flooding that further impedes gas exchange. These exudates congeal with neutrophils and necrotic cellular debris to form fibrin casts, partially or fully obstructing the airways and causing bronchoconstriction. A repeat diagnostic/therapeutic bronchoscopy on days 3 to 5 may reveal these casts. The cellular debris and excessive mucus are a natural medium for bacterial growth. Pneumonia is a common cause of infection in patients with inhalation injury. Release of the proteolytic enzymes and oxidants initiated by the acute inflammatory response further destroys the lung parenchyma and increases the microvascular permeability, further propagating edema formation and decreasing gas exchange. Surfactant is diminished, resulting in alveolar collapse and a decrease in lung compliance. Permanent airway damage with fibrosis, airway stenosis, and granulomatous scar formation is a late finding of inhalation injury. REFERENCES 1.Clark WR Jr, Nieman GF: Smoke inhalation. Burns Incl Therm Inj 14:473-494, 1988 2.Herndon DN, Traber DL: Pulmonary circulation and burns and trauma. J Trauma 30:S41-S44, 1990 3.Herndon DN, Thompson PB, Traber DL: Pulmonary injury in burned patients. Crit Care Clin 1:79-96, 1985 3.Ruddy RM: Smoke inhalation injury. Pediatr Clin North Am 41:317-336, 1994 4.Thompson PB, Herndon DN, Traber DL, Abston S: Effect on mortality of inhalation injury. J Trauma 26:163-165, 1986 page 758 American College of Surgeons SESAP 13 Category 8 Item 19 Which of the following is an effect of red wine that may be involved in helping prevent ischemic heart disease? (A) Up-regulation of cellular adhesion molecules (ICAM-1, VCAM-1) (B) Binding of toxic products from cigarette smoke (C) Inhibition of inducible nitric oxide synthase (iNOS), a known inflammatory mediator (D) Promotion of low-density lipoprotein (LDL) oxidation (E) Inhibition of platelet aggregation page 759 American College of Surgeons SESAP 13 Critique for Category 8 Item 19 The French paradox describes the co-existence in France of a low mortality from ischemic heart disease with a high amount of saturated fats in the diet. First reported by Renaud and de Lorgeril in 1992, this effect was attributed to the anti-platelet-aggregation effect of alcohol. Although it was clear that ingestion of red wine was more likely to be preventive than ingestion of white wine or other forms of alcohol, no explanation was offered. It now appears that the effects of red wine are due primarily to the effects of polyphenols, components of red wine such as flavonol monomers, flavonols such as quercitin, and anthocyanins. These compounds are present in red wines to the amount of 200 mg/glass versus 30 mg/glass of white wine. The polyphenols have multiple effects that may be beneficial. They inhibit cellular adhesion molecules, such as ICAM-1 and VCAM-1. However, they do not appear to have an effect on the products of cigarette smoke. They up-regulate endothelial nitric oxide synthase (eNOS), thereby inducing vascular relaxation. Low-density lipoprotein (LDL) oxidation, believed to be involved in atherogenesis, is inhibited by polyphenols. High-density lipoprotein (HDL) cholesterol is also increased as an effect of polyphenols. Finally, polyphenols inhibit platelet aggregation. In this, its effect may be synergistic with that of alcohol, which also tends to inhibit platelet aggregation. REFERENCES 1.Cordova AC, Jackson LS, Berke-Schlessel DW, Sumpio BE: The cardiovascular protective effect of red wine. J Am Coll Surg 200:428-439, 2005 2.Renaud S, de Lorgeril M: Wine, alcohol, platelets, and the French paradox for coronary heart disease. Lancet 339:1523-1526, 1992 page 760 American College of Surgeons SESAP 13 Category 8 Item 20 Which of the following statements about the use of pulmonary artery catheters in high-risk surgical patients is TRUE? (A) The incidence of renal failure is lower (B) Perioperative mortality is lower (C) Long-term (12-month) mortality is lower (D) The length of stay is not affected (E) The amount of fluid administration is lower page 761 American College of Surgeons SESAP 13 Critique for Category 8 Item 20 During the 1980s and 1990s, use of the flow-directed pulmonary artery catheter became nearly routine in high-risk surgical patients, yet a nagging question remained about their use. They provided data that gave clinicians a much clearer picture of the state of the cardiovascular system. Despite this, better information did not seem to translate into superior outcomes for patients, and the use of the catheter itself caused some complications. An observational study of 5735 patients concluded that there was actually an increase in mortality with the use of pulmonary artery catheters. Two subsequent prospective randomized studies did not show an increase in either short-term or long-term mortality. Both showed no benefit and one found a higher rate of pulmonary embolism from the use of the catheters. Whereas the use of pulmonary artery catheters might in theory permit fluid administration to be titrated very precisely, a study from the UK found that the amount of fluids given to adult ICU patients with pulmonary artery catheters was significantly greater during the first 24 hours than in patients without catheters, and the incidence of renal failure was higher in the group with pulmonary artery catheters. They also noted thrombocytopenia in association with pulmonary artery catheters, a finding reported by others as well. The length of stay, either in the ICU or in the hospital, was not affected. Some authors suggest that clinicians simply don’t know how to use pulmonary artery catheters properly: if the information obtained were to be standardized and the response to it codified into a management protocol, these catheters may yet be found to be of benefit. But this highly optimistic view is not supported by data. Perhaps some subgroups of patients, under well-devised management protocols, may show benefits from the use of a pulmonary artery catheter, but even that is somewhat speculative on the basis of the available evidence. REFERENCES 1.Connors AF Jr, Speroff T, Dawson NV, Thomas C, et al: The effectiveness of right heart catheterization in the initial care of critically ill patients. SUPPORT Investigators. JAMA 276:889-897, 1996 2.Rhodes A, Cusack RJ, Newman PJ, Grounds RM, et al: A randomized, controlled trial of the pulmonary artery catheter in critically ill patients. Intensive Care Med 28:256-264, 2002 3.Sandham JD, Hull RD, Brant RF, Knox L, et al: A randomized, controlled trial of the use of pulmonaryartery catheters in high-risk surgical patients. N Engl J Med 348:5-14, 2003 4.Shah MR, Hasselblad V, Stevenson LW, Binanay C, et al: Impact of the pulmonary artery catheter in critically ill patients: meta-analysis of randomized clinical trials. JAMA 294:1664-1670, 2005 page 762 American College of Surgeons SESAP 13 Category 8 Item 21 Which of the following is the most effective practice for reducing central venous catheter-related bacteremia? (A) Changing the catheter over a guidewire every 3 days (B) Using betadine for skin preparation (C) Using antibiotic-impregnated catheters (D) Using a systems-based approach to catheter insertion (E) Using prophylactic antibiotics for skin flora page 763 Perl TM. the rate of catheter-related infections in the ICU decreased over 4 years from 11. Scheduled changing of central venous catheters over a guidewire is associated with an increase in septic events. diagnosis. 2003 3. Semin Respir Crit Care Med 24:23-36.3/1000 catheter days to 0/1000 catheter days.Berenholtz SM. 2005 page 764 .Maki DG. 2004 2. et al: Effect of a second-generation venous catheter impregnated with chlorhexidine and silver sulfadiazine on central catheter-related infections: a randomized. Hobson D. Pronovost PJ. REFERENCES 1. Lisco SJ. creating a catheter insertion cart. Chlorhexidine for skin cleansing at the time of catheter insertion decreases the incidence of central venous catheter infection compared with betadine. Crit Care Med 32:2014-2020. controlled trial. and implementing a checklist to ensure adherence to evidence-based guidelines). Lipsett PA. Crnich CJ: Line sepsis in the ICU: prevention. Prophylactic antibiotics at the time of insertion are not indicated for this clean procedure. The use of antibiotic-impregnated catheters is not associated with a decrease in central venous catheter infection and bacteremia. et al: Eliminating catheter-related bloodstream infections in the intensive care unit.American College of Surgeons SESAP 13 Critique for Category 8 Item 21 Catheter-related bloodstream infections are most effectively reduced by following a systems-based approach for catheter insertion.Rupp ME. Lipsett PA. Ann Intern Med 143:570-580. asking providers daily whether catheters could be removed. and management. In one quality improvement study that used a systems-based approach (based on interventions including staff education. Over the past 24 hours. She is receiving dopamine (10 ug/kg/min) with no change in her hemodynamic pressures. 18 mmol/L. her systolic blood pressure has ranged from 80 to 100 mm Hg. sepsis.9 mg/dL. potassium. 110 mmol/L.American College of Surgeons SESAP 13 Category 8 Item 22 A 37-year-old woman remains in the ICU 2 weeks after a complicated distal pancreatic resection followed by pancreatic leak. Laboratory values are as follows: serum sodium. BUN. pulse is 110 beats/min. chloride. and the acute respiratory distress syndrome (ARDS). intra-abdominal abscess. she has had a temperature > 38°C. 130 mmol/L. 5. 1.3 mmol/L. 35 mg/dL. The most appropriate next step would be (A) ACTH (Cortrosyn) stimulation test (B) measurement of TSH level (C) adminstration of methylprednisolone (30 mg/kg) (D) administration of calcium chloride (E) administration of activated protein C page 765 . CO2. Central venous pressure is 10 cm H2O and pulmonary capillary wedge pressure is 23 cm H2O. and creatinine. For the last 12 hours. REFERENCES 1. postural hypotension. Evidence supports the use of supplemental corticosteroids in ICU patients with established septic shock. or patients with maximal adrenal cortex stimulation by endogenous ACTH. Secretion of the thyroid hormones T4 (thyroxine) and T3 (triiodothyronine) is regulated by pituitary thyrotropin (TSH). and often fever. In the largest randomized. Activated protein C inhibits factors Va and VIIIa.American College of Surgeons SESAP 13 Critique for Category 8 Item 22 Acute adrenal insufficiency is life-threatening. which involves measurement of serum cortisol before and after 30 to 60 minutes of intravenous or intramuscular injection of 250 µg 1-24 ACTH. Bollaert PE. exogenous hormone administration usually does not evoke any further increase in serum cortisol. It is indicated in the management of sepsis in adult patients. hyperkalemia (65%). Stewart PM: Corticosteroid insufficiency in acutely ill patients. Mineralocorticoid deficiency accounts for hyponatremia (90% of patients with primary adrenal insufficiency). In healthy persons. resulting in low blood pressure. JAMA 288:862-871. and salt craving (15%). Three randomized. this challenge leads to a physiologic increase in serum cortisol to peak concentrations > 500 nmol/L. et al: Effect of treatment with low doses of hydrocortisone and fludrocortisone on mortality in patients with septic shock. 2003 page 766 . The impaired ability of the adrenal cortex to respond to ACTH is readily demonstrated by the standard short corticotropin test. Thyroid function tests are used in a variety of clinical settings to screen thyroid function. Septic patients who remain hypotensive while receiving vasopressors may be treated empiricly with intravenous hydrocortisone (50 mg every 6 hours) until their stimulation test results return. 2003 3. limiting thrombotic effects. Mineralocorticoid deficiency. 2002 2. vomiting. In patients with primary adrenal insufficiency. especially in those with biochemical evidence of functional hypoadrenalism. placebo-controlled trial.Annane D. Sebille V. and sometimes even in prerenal failure. which is present only in primary adrenal insufficiency. controlled trials of hydrocortisone replacement in patients with septic shock have shown improvement in hemodynamics and a reduction in the need for vasopressor therapy. N Engl J Med 348:727734. Patients typically present with severe hypotension or hypovolemic shock. and are sometimes misdiagnosed as having an acute abdomen. acute abdominal pain.Cooper MS. Charpentier C. leads to dehydration and hypovolemia. Allolio B: Adrenal insufficiency. 300 medical and surgical patients treated with hydrocortisone (200 mg/day) and fludrocortisone (50 mg once daily for 7 days) had significantly reduced mortality and duration of vasopressor therapy. and monitor the treatment of hyperthyroidism. assess the adequacy of levothyroxine therapy.Arlt W. Calcium chloride is indicated in patients with hypocalcemia and after multiple blood transfusions. Lancet 361:1881-1893. clindamycin. repeat abdominal examination. She is hypotensive and has diffuse abdominal tenderness. The next step should be (A) immediate exploratory laparotomy with abdominal colectomy and ileostomy (B) colonoscopy for colonic decompression (C) gastrointestinal tract decompression with a nasogastric tube.American College of Surgeons SESAP 13 Category 8 Item 23 A 79-year-old previously active woman is admitted with sepsis and necrotizing fasciitis from a small wound on her leg that developed after working in her garden 5 days previously. On day 11 she has decreased mental status with a temperature of 40°C and a WBC count of 35. The abdominal x-ray shown is obtained. and gentamicin. and xray in 4 to 6 hours (D) barium enema (E) neostigmine administration page 767 . The wound is widely debrided and treated with a 10-day course of penicillin.000/mm3. Saunders MD. Angriman I. but would be contraindicated with signs of peritonitis. and excellent hand-washing with soap and water (alcohol-based cleaners are inadequate). Intensive Care Med 29:1030. and judicious antibiotic use are key for preventing its spread. A latex agglutination assay is neither sensitive nor specific.Hurley BW. Vitale A. Operation may be required in up to 70% of cases and may result in a small number of related complications and deaths. Ruffolo C. 2002 4. C difficile infection has a wide spectrum. Owens RC Jr. Aliment Pharmacol Ther 22:917-925. severe sepsis and multiple organ dysfunction syndrome. Severe sepsis and multiple organ dysfunction syndrome secondary to C difficile infection is associated with a high mortality rate. 2005 5. Thompson A. 2005 2. Killgore GE. This potentially lethal complication is probably related to Clostridium difficile pseudomembranous colitis in this case. The diagnosis of C difficile infection can be made based on an enzyme immunoassay that rapidly detects toxin A and B but is less sensitive than a cytotoxin assay. Hickey C. and requires surgical intervention. REFERENCES 1. but idiopathic inflammatory bowel disease is an alternative cause.Dobson G. Nguyen CC: The spectrum of pseudomembranous enterocolitis and antibiotic-associated diarrhea. Breakdown of gut barrier function may facilitate progression to multiorgan failure.D’Amico C. or C difficile infection. It is very contagious. 2005 page 768 . Neostigmine has been described as a nonsurgical therapy for colonic pseudo-obstruction. Kimmey MB: Systematic review: acute colonic pseudo-obstruction. 2003 3. N Engl J Med 353:2433-2441. Trinder J: Clostridium difficile colitis causing toxic megacolon. Recent reports suggest that a previously uncommon C difficile strain has emerged with antibiotic resistance and accelerated toxin A and B production.McDonald LC. Digestion 72:146—149. et al: An epidemic. especially in the elderly. toxic megacolon. Arch Intern Med 162:2177—2184. Toxic megacolon is characterized by total or segmental nonobstructive colonic dilatation of at least 6 cm associated with systemic toxicity. toxin gene-variant strain of Clostridium difficile. enteric precautions. varying from asymptomatic to fulminant toxic megacolon. which only detects toxin B and takes longer.American College of Surgeons SESAP 13 Critique for Category 8 Item 23 The patient has likely developed toxic megacolon with peritonitis. making it more virulent. Colonoscopy and barium enema are contraindicated in this patient. et al: Early surgery for the treatment of toxic megacolon. Which of the following statements about a mass casualty disaster of this magnitude is TRUE? (A) Patients with any chance for survival should be resuscitated regardless of injury severity (B) Transfer of critically injured patients to trauma facilities away from the site constitutes an EMTALA violation (C) Military emergency medical response teams should be the first medical teams to be mobilized (D) The governor or state health official has the authority to declare a state of emergency to qualify for national assistance (E) Most patients are likely to be triaged at the site by pre-hospital providers and transported by ambulance to a regional medical center page 769 . and burn injuries.American College of Surgeons SESAP 13 Category 8 Item 24 An explosion at an oil refinery with 2500 employees has resulted in an unknown number of critically injured patients with blunt trauma. penetrating trauma. Mintz Y. Belzberg H.Repine TB. In situations when there are large numbers of injured patients and limited resources. 2004 4.Almogy G. 2004 3. Emergency medical providers may be better prepared to make these decisions if the medical community reviews the ethics of triage and proactively establishes guidelines to be used in extraordinary circumstances. Since 2001. transcontinental transport of burn patients may facilitate optimal care of patients with these injuries. 2005 page 770 . Lisagor P. REFERENCES 1. Cohen DJ: The dynamics and ethics of triage: rationing care in hard times. Our current high expectations for excellent outcomes and survival rates do not necessarily correlate with the realities during a disaster or during war. secondary triage may be necessary to regional trauma and burn centers. Multiple reports on disasters over the past 15 years suggest that first responders are more likely to be bystanders and that many more patients are transported to nearby hospitals in private vehicles than by pre-hospital providers. A state governor or health official. et al: Suicide bombing attacks: update and modifications to the protocol.Frykberg ER: Principles of mass casualty management following terrorist disasters. as demonstrated in US military experience in Iraq and in subsequent US civilian practice. patients who might ordinarily survive may consume too many resources to warrant aggressive resuscitation when many more patients with moderate injuries could benefit from scarce supplies. Gibran N. 2005 6. Ann Surg 239:295-303. et al: Plenary session II: American Burn Association disaster readiness plan. Gibran NS. Successful disaster management must involve distribution of patients beyond the nearest hospital center to maximally treat as many injured patients as possible without overwhelming the local center personnel and resources. Barillo DJ.ABA Board of Trustees. J Burn Care Rehabil 26:102-106. Jordan M: Defining the ratio of outcomes to resources for triage of burn patients in mass casualties. Ideally primary triage at the disaster site will distribute patients to nearby local hospitals in a sequential manner. Ann Surg 239:319-321. J Burn Care Rehabil 26:183-191. Americans have become increasingly aware of disaster preparedness and planning has improved somewhat. supplies. Therefore. the assistant secretary of defense for health affairs. This will lead to difficult life and death triage decisions. Pikarsky AK. 2005 2. 2005 5. and the National Transportation Safety Board are all able to activate the national disaster medical system. and equipment might need to be rationed. EMTALA governs hospital-hospital transfer and does not apply to disaster sites.American College of Surgeons SESAP 13 Critique for Category 8 Item 24 This scenario represents an industrial mass-casualty disaster that could rapidly overwhelm local and national medical resources. Mozingo DW. Such an event will likely result in many patients with burn injuries. Involvement of military triage is a last resort after FEMA resources are exhausted. J Burn Care Rehabil 26:478-482. who can be easily transported long distances.Jordan MH. Milit Med 170:505-509. personnel. Committee on Organization and Delivery of Burn Care: Disaster management and the ABA Plan.Saffle JR. American College of Surgeons SESAP 13 Category 8 Item 25 Which of the following statements about positive end-expiratory pressure (PEEP) in acute respiratory distress syndrome (ARDS) is NOT true? (A) Can lower pulmonary capillary flow (B) Can cause hypotension (C) Decreases functional residual capacity of the lung (D) May increase the measured pulmonary artery wedge pressure (E) May decrease lung water page 771 . Eur Respir J Suppl 42:2s—9s.[No Authors Listed]: Ventilation with lower tidal volumes as compared with traditional tidal volumes for acute lung injury and the acute respiratory distress syndrome. alveolar pressure partially impinges on capillary blood flow. pulmonary capillary blood flow is essentially unchanged throughout the respiratory cycle. Minerva Anestesiol 71:265-72. In zone 3. In zone 1. alveolar pressure effectively occludes the pulmonary capillary bed. Increasing PEEP will therefore shift more of the lung to zones 1 and 2. The Acute Respiratory Distress Syndrome Network. REFERENCES 1. 2000 2. Left atrial pressure measurement using pulmonary artery catheters may be artificially increased by transmural pressure. resulting in PEEP. N Engl J Med 342:1301-1308.Ricard JD. more of the lung is shifted toward West zones 1 and 2 from zones 2 and 3. and may result in hypotension. The effect of PEEP is to recruit additional alveolar units.Villar J: The use of positive end-expiratory pressure in the management of the acute respiratory distress syndrome. Increased intra-alveolar pressure decreases pulmonary alveolar water by decreasing the pressure gradient from the pulmonary capillary bed to the alveoli. increasing the functional residual capacity of the lung. and the effect is most pronounced at peak inspiration in a patient receiving ventilatory support with PEEP. At high levels of PEEP. Dreyfuss D. In zone 2. This shift is accentuated if the filling pressures—and therefore pulmonary capillary pressures—are lower. Saumon G: Ventilator-induced lung injury. hence increasing lung compliance and decreasing positive inspiratory pressure as well as increasing the diminished functional residual capacity characteristic of acute respiratory distress syndrome (ARDS). 2003 3. 2005 page 772 .American College of Surgeons SESAP 13 Critique for Category 8 Item 25 Positive end-expiratory pressure (PEEP) tends to stent open terminal bronchioles and alveoli. American College of Surgeons SESAP 13 Category 8 Item 26 A typical infecting organism in tertiary peritonitis is (A) Escherichia coli (B) Candida (C) Proteus sp (D) Pneumococcus (E) Mycobacteria page 773 . high APACHE scores. Tertiary peritonitis is associated with a high incidence of organ failure. REFERENCES 1. Bruining HA: Future directions in the management of tertiary peritonitis. Proteus is usually a urinary tract pathogen and mycobacteria infects the peritoneum after primary lung infection. World J Surg 22:158-163. and tertiary peritonitis may be more a marker for mortality than a direct cause. Rotstein OD. Their presence implies a degree of immunocompromise.American College of Surgeons SESAP 13 Critique for Category 8 Item 26 Tertiary peritonitis occurs after apparently successful treatment of secondary bacterial peritonitis. 2002 2.Nathens AB. Marshall JC: Tertiary peritonitis: clinical features of a complex nosocomial infection. Pneumococcus is rarely associated with peritonitis. Escherichia coli is associated with secondary peritonitis.Buijk SE. Typical organisms include Candida and Enterococcus sp and Staphylococcus epidermidis. 1998 page 774 . Surgical and antimicrobial therapy both have poor outcomes. and a high mortality rate. Intensive Care Med 28:1024-1029. The infecting organism or organisms are generally not pathogenic under normal circumstances. 4 units/hour) in septic shock (A) raises systolic and mean arterial blood pressure (B) increases cardiac output (C) decreases urine output (D) causes more cardiac complications than norepinephrine (E) causes tachycardia page 775 .American College of Surgeons SESAP 13 Category 8 Item 27 Low-dose intravenous vasopressin (2. Systolic and mean arterial blood pressures are increased. likely due to the increase in glomerular filtration rate as a result of the increase in blood pressure. even this low dose of vasopressin. but prospective randomized clinical trials are ongoing and are not yet published. Curr Opin Crit Care 11:413-417. 2005 4.Sharma VK.Holmes CL: Vasoactive drugs in the intensive care unit. 2005 2. Cullinane DC. Clinical evidence as to its safety and efficacy is growing.Hall LG. Pharmacotherapy 24:1002-1012. International Sepsis Forum: The International Sepsis Forum’s controversies in sepsis: my initial vasopressor agent in septic shock is norepinephrine rather than dopamine. may cause an ADHlike effect with a resultant decrease in urine output. Rarely. Oyen LJ. Meinders AE: Vasopressin: physiology and clinical use in patients with vasodilatory shock: a review. and the pulse rate usually remains the same or shows a slight decrease. et al: Fixed-dose vasopressin compared with titrated dopamine and norepinephrine as initial vasopressor therapy for septic shock. SCCM. however. REFERENCES 1. Neth J Med 63:4-13. 2003 page 776 . while the same dosage administered to normal subjects has no appreciable effect.American College of Surgeons SESAP 13 Critique for Category 8 Item 27 Low-dose intravenous vasopressin (2. The effect on urine output is usually salutary.4 units/hour) has a number of physiologic effects in patients with vasodilatory shock. Dellinger RP. Taner CB.den Ouden DT. 2004 3. Retrospective studies to date have not shown a significant increase in cardiac complications compared with norepinephrine or dopamine. cardiac output may show a mild decrease. Crit Care 7:3-5. He has hypertension well-controlled with an angiotensin-converting enzyme (ACE) inhibitor. He does not smoke and is able to walk 2 flights of stairs without difficulty.American College of Surgeons SESAP 13 Category 8 Item 28 A 50-year-old man is scheduled for low anterior resection. Preoperative evaluation and management should include (A) dobutamine stress echocardiography (B) perioperative beta blockade (C) preoperative pulmonary artery catheter placement (D) myocardial perfusion scan (E) none of the above page 777 . Ann Intern Med 138:506-511. The perioperative cardiac risk in these patients is 9% to 32%.6%. The perioperative cardiac risk in these patients is 2. Additional noninvasive testing is also suggested in patients with three or more Revised Cardiac Risk Criteria. Myocardial perfusion imaging has a low positive predictive value and does not provide prognostic value beyond clinical risk stratification. the consistent results in high-risk patients suggest that 2 to 9 patients must be treated to prevent a single event of perioperative ischemia. Perioperative use of pulmonary artery catheters does not improve outcome in these patients. target heart rate is 50 to 60 beats/min or to development of symptoms. or cardiac death. dobutamine stress echocardiography may be helpful. diabetes mellitus requiring insulin.6% with perioperative beta blockade. In these highrisk patients. different lengths of treatment. REFERENCES 1.4%. Suggested duration for administration of beta blockade is 7 to 30 days perioperatively. and chronic renal insufficiency with a creatinine > 2. JAMA 287:1435-1444.0 mg/dL) who do not have contraindications should also be given perioperative beta blockade. The perioperative cardiac risk in patients without any of these criteria is 0. suprainguinal vascular or intrathoracic operation. and continue at least throughout the hospital stay with a potential benefit for continuing up to 30 days. 2002 2. Intervention to treat coronary ischemia is suggested in these patients. which can be lowered to 0. Postoperatively. Goldman L: beta-Blockers and reduction of cardiac events in noncardiac surgery: scientific review. Clinical risk stratification can be based on either the modified Mangano criteria or the Revised Cardiac Risk Index Criteria.Auerbach AD. current smoker.2% to 6. infarction. In patients with ischemia identified on noninvasive testing. This cannot be lowered by beta blockade.Grayburn PA. Although studies investigating the effectiveness of beta blockade have used different doses.8% to1. 2003 page 778 .2% in patients without ischemia who are given perioperative beta blockade. dobutamine stress echocardiography has a low positive predictive value and patients are often subjected to further unnecessary testing and operative delay. and diabetes mellitus not requiring insulin therapy) who do not have contraindications should be given perioperative beta blockade. Patients with a single Revised Cardiac Risk Index Criteria (intraperitoneal. beta blockade should start as soon as possible. ischemic heart disease. In low. augmented with noninvasive testing in select high-risk patients. and perioperative beta blockade is not indicated for such patients. the perioperative cardiac risk remains 6. cerebrovascular disease.American College of Surgeons SESAP 13 Critique for Category 8 Item 28 There is growing evidence that perioperative cardiac morbidity in major noncardiac operations can be lowered by perioperative administration of beta blockers. Patients with two of the modified Mangano criteria (age 65 or older. and evaluated different outcomes. hypertension.to medium-risk patients. Hillis LD: Cardiac events in patients undergoing noncardiac surgery: shifting the paradigm from noninvasive risk stratification to therapy. Patients with poor functional status may benefit from additional noninvasive testing.5% to16% even with perioperative beta blockade. as results are correlated with perioperative cardiac risk.4% to 1. This has shifted the perioperative cardiac evaluation and management paradigm from risk stratification with noninvasive testing to clinical risk stratification and treatment. serum cholesterol > 240 mg/dL. which can be decreased to 0. 2005 page 779 .McGory ML.American College of Surgeons SESAP 13 3. Ko CY: A meta-analysis of perioperative beta blockade: what is the actual risk reduction? Surgery 138:171-179. Maggard MA. American College of Surgeons SESAP 13 Category 8 Item 29 A 54-year-old woman who has been receiving total parenteral nutrition (TPN) for 3 months is admitted to the ICU after takedown of her enterocutaneous fistula. and on postoperative day 3 her morning serum glucose is 250 mg/dL. Which of the following statements is TRUE? (A) Her hyperglycemia may be the result of selenium deficiency (B) Her T-cells will not function normally (C) Her hyperglycemia may be the result of a dextrose infusion rate of 3 mg/kg/min (D) An insulin drip should be started to keep her blood glucose < 110 mg/dL (E) Her risk of postoperative infection is increased only if she is diabetic page 780 . TPN is re-started on postoperative day 2. including catecholamines. part 1. is manifested by myalgias and cardiomyopathy. 2003 4. IA. Dubuque. Am J Health Syst Pharm 61:1938-1949. pp 61-78 3. 2001 page 781 . making the provision of parenteral nutrition a careful balancing act. Chromium potentiates the effect of insulin through the poorly characterized glucose tolerance factor. Intensive Care Med 29:642-645. Rationale. is the primary reason for the hyperglycemia seen postoperatively and in other states of physiologic stress. and strict control aims to maintain glucose levels < 110 mg/dL. as well as the cytokines interleukin-1.Btaiche IF. The rate of dextrose infusion is important. The benefit comes from a reduction in the mortality from multiple organ failure secondary to sepsis. Balancing nutrition with glycemic control has led to increased use of intravenous insulin therapy in many ICUs. Verwaest C. 2002. This contrasts with rates of up to 5 mg/kg/min that are well-tolerated in nonstressed patients. Supplementation of hyperglycemic patients without chromium deficiency is of no proven benefit. et al (eds): Nutritional Considerations in the Intensive Care Unit: Science. in Shikora SA. et al: Intensive insulin therapy in the critically ill patient. Khalidi N: Metabolic complications of parenteral nutrition in adults. Schwaitzberg SB. Kendall/Hunt. hyperglycemia leads to inhibition of glucose-6-phosphate dehydrogenase. Hyperglycemia is often defined as glucose > 200 mg/dL. another trace element. it also decreases both infectious complications and overall mortality in critically ill nondiabetic patients undergoing noncardiac operations. American Society for Parenteral & Enteral Nutrition. The recommended daily requirement of chromium is 10 to 15 µg. and most standard enteral and parenteral formulations contain adequate amounts of chromium. 2004 2.van den Berghe G. glucagon. At the cellular level.Mason JB: Vitamins and trace elements in the critically ill patient. Nielsen SE. Weekers F. and tumor necrosis factor alpha. REFERENCES 1. N Engl J Med 345:1359-1367. but not hyperglycemia. Rask-Madsen J: High glucose impairs superoxide production from isolated blood neutrophils. This increase in endogenous glucose. and growth hormone. Martindale RG. cortisol. because hyperglycemia develops in 49% of those in whom the infusion rate is > 5 mg/kg/min but not in any patients in whom the infusion rate is kept < 3 mg/kg/min. Deficiency of selenium. Although strict glucose control was initially shown to benefit diabetic patients undergoing coronary artery bypass grafting. The decrease in infectious complications seen when strict glucose control is maintained is clinically relevant. resulting in diminished levels of superoxide produced by activated neutrophils.American College of Surgeons SESAP 13 Critique for Category 8 Item 29 Hyperglycemia can occur in patients receiving long-term total parenteral nutrition (TPN) as a result of chromium deficiency if this trace element is not supplemented. endogenous glucose production is the result of several counterregulatory hormones. Wouters P. and Practice. coupled with insulin resistance.Perner A. interleukin-6. These patients have a much lower tolerance of dextrose infusion than nonstressed patients. This reduces the bactericidal activity of these polymorphonuclear leukocytes but not T-cells. In the postoperative state. Cultures are obtained. Which of the following statements is TRUE? (A) Antibiotic therapy should be withheld until culture results are known (B) He should be given antibiotics active against methicillin-resistant Staphylococcus aureus (MRSA) (C) Bronchoscopy is required for diagnosis (D) Antibiotic treatment should continue for 2 weeks (E) He should receive drotrecogin alfa (activated) (Xigris) page 782 . pulse is 82. The chest x-ray shown is obtained. He developed a left pneumothorax after placement of a central venous line 18 hours previously. and respiratory rate is 32.500/mm3 with 8% bands.American College of Surgeons SESAP 13 Category 8 Item 30 A 44-year-old diabetic man who has been receiving ventilatory support for 12 days develops a temperature of 102°F and purulent sputum. His blood pressure is 130/82. He has received a 10-day course of intravenous antibiotics for peritonitis after repair of a blunt small bowel injury. His WBC count is 11. Laterre PF. LaRosa SP. Garg R. To benefit from this drug. leukocytosis persists to a similar degree (11to 12. pulse > 90. et al: Comparison of 8 vs 15 days of antibiotic therapy for ventilator-associated pneumonia in adults: a randomized trial. Guertin MC.8% to 24.4° F.Abraham E. however. and Xigris is not recommended for use within 96 hours of surgery. et al: Protected specimen brush or bronchoalveolar page 783 . quantitative results are both more sensitive and more specific than nonquantitative specimens. tailored for the local flora in the particular ICU and including coverage for resistant gram-negative species. Esposito TJ. 2001 3. is high. This increase in bleeding is not insignificant in surgical patients. Longer treatment regimens do not decrease mortality. and purulent sputum. Clinical criteria alone. If the bacteriologic agent is a gram-negative rod (most often Pseudomonas). the APACHE II score must be over 25 to receive the benefit in terms of improved survival. treatment should be continued for 8 days. those treated for 8 days will have more recurrent infections than those treated 15 days.0% to 3. designed to treat sepsis. with equivalent mortality. bronchoscopic-guided protected specimen brush. or bronchoscopic alveolar lavage. 2005 5. Vincent JL. 2005 2. previous antibiotic therapy. Litalien C. it can be stopped if the culture is negative. culture results are not usually available for 24 to 48 hours. leukocytosis. Eckert MJ. However. Gram stain can be helpful in guiding early therapy if positive.Chastre J.000 or >10% immature forms. N Engl J Med 353:1332-1341. Quantitation is much more important than the method used to obtain the culture. should be included. blind protected specimen brush. JAMA 290:2588-2598. Laterre PF. Furthermore.Bernard GR.5%. fever. and coverage active against MRSA. Empiric therapy should be initiated as soon as pneumonia is suspected and the culture is obtained.de Jaeger A. The diagnosis is best established using quantitative culture of either an endotracheal aspirate. and prolonged mechanical ventilatory support. specifically healthcare-related MRSA. particularly in surgical patients. with evidence of dysfunction in at least one organ system. Empiric therapy should be aimed at bacteria normally found in late-onset ventilator-associated pneumonia. Levy H.000/mm3). Once the diagnosis of pneumonia is confirmed. and delaying appropriate antibiotic treatment increases mortality. REFERENCES 1.7%.Davis KA. respiratory rate > 20 or PCO2 < 32. WBC count > 12. patients must have a known or suspected infection and three of the following criteria: temperature > 100. Reed RL II. with an increase in the risk of bleeding from 2. 2003 4. et al: Ventilator-associated pneumonia in injured patients: do you trust your Gram’s stain? J Trauma 58:462-467. Lacroix J. Mortality is decreased from 30. Wolff M. et al: Drotrecogin alfa (activated) for adults with severe sepsis and a low risk of death. Fagon JY. but a negative Gram stain does not reliably exclude pneumonia. will overdiagnose pneumonia. This patient is also at high risk for methicillin-resistant Staphylococcus aureus (MRSA) because of his diabetes mellitus. who has had a prolonged period of ventilation now with a radiographic infiltrate. At either 8 or 15 days. Chevret S. This patient is not a candidate for drotrecogin alfa (Xigris). N Engl J Med 344:699-709.American College of Surgeons SESAP 13 Critique of Category 8 Item 30 The suspicion for ventilator-associated pneumonia in this man. et al: Efficacy and safety of recombinant human activated protein C for severe sepsis. Cainzos M. Am J Surg 187:134-145. 2003 8. Chest 122:262-268. 1999 6. Fraser VJ.Iregui M.Solomkin JS. 2002 7.American College of Surgeons SESAP 13 lavage to diagnose bacterial nosocomial pneumonia in ventilated adults: a meta-analysis. Dellinger EP. Ward S. et al: Clinical importance of delays in the initiation of appropriate antibiotic treatment for ventilator-associated pneumonia.Napolitano LM: Hospital-acquired and ventilator-associated pneumonia: what’s new in diagnosis and treatment? Am J Surg 186:4S-14S. 2004 page 784 . Crit Care Med 27:2548-2560. et al: A consensus statement on empiric therapy for suspected gram-positive infections in surgical patients. Sherman G. Bjornson HS. exceeds 99% (D) Elevation in D-dimer requires empiric anticoagulation (E) The best initial screening test is ventilation-perfusion radionuclide scanning page 785 . ie. ruling out clinically significant pulmonary embolism.American College of Surgeons SESAP 13 Category 8 Item 31 Which of the following statements about the postoperative evaluation of hospitalized patients with clinical signs of pulmonary embolism is TRUE? (A) Chest computed tomography (CT) has excellent sensitivity for isolated peripheral pulmonary embolism (B) Interpretation of pulmonary angiography in these patients is characterized by excellent interobserver agreement (C) The negative predictive value for spiral CT. Although any CT technique may miss isolated peripheral emboli. D-dimer levels will be elevated in postoperative patients regardless of the presence of thromboembolism. REFERENCES 1. Kipfmueller F. 2005 2. Henry JW. et al: Clinical validity of a negative computed tomography scan in patients with suspected pulmonary embolism: a systematic review. neither empiric anticoagulation nor confirmatory pulmonary angiogram is indicated.Quiroz R. 1999 page 786 . There are currently few indications for ventilation-perfusion scans or pulmonary angiography in these patients. if a patient with suspected venous thromboembolism or pulmonary embolism has a negative chest CT.American College of Surgeons SESAP 13 Critique for Category 8 Item 31 Spiral chest computed tomography (CT) has become the diagnostic test of choice in patients with suspected pulmonary embolus. the sensitivity of detecting clinically significant pulmonary emboli approaches 100%. Kucher N. JAMA 293:2012-2017. Radiology 210:689-691. In other words. Several studies have demonstrated that the diagnosis of pulmonary embolism after hospital discharge is rare in patients with negative pulmonary CT scans.Stein PD. Gottschalk A: Reassessment of pulmonary angiography for the diagnosis of pulmonary embolism: relation of interpreter agreement to the order of the involved pulmonary arterial branch. Zou KH. The inter-observer variation in interpretation for conventional pulmonary angiography or ventilationperfusion scan far exceeds that reported for CT scan. Evaluation reveals a normal size right kidney and a small (7 cm) left kidney.3 g/dL.American College of Surgeons SESAP 13 Category 8 Item 32 An 86-year-old hypertensive man has a creatinine of 2. An arteriogram is planned. Which of the following statements about management is TRUE? (A) Pre-procedure hydration with intravenous normal saline is contraindicated because of the risk of congestive heart failure (B) Pre-procedure hydration with intravenous sodium bicarbonate rather than normal saline is more effective at preventing contrast-induced nephropathy (C) An increase in serum creatinine of 25% is expected and generally not clinically significant after contrast arteriography (D) The risk of renal failure from intra-arterial contrast is too high in this patient and alternative imaging should be ordered (E) If the BUN/creatinine ratio is > 20 prior to the procedure. the risk of contrast-induced nephropathy is low page 787 . Nasr S.5 g/dL as being clnically significant after administration of contrast. Holleman JH. A single creatinine value should not be used as a contraindication to the examination prior to evaluation after hydration. but no absolute value is recognized as a contraindication for the use of contrast. Nephron Clin Pract 93:C29-C34.7% of patients receiving arterial contrast after pre-procedure hydration with sodium bicarbonate versus saline. a randomized blinded study also documented a reduction in the incidence of acute renal failure from 13. JAMA 291:2328-2334.Chabova V. Aggarwal K.2 µmol/L or 0. et al: Outcomes of atherosclerotic renal artery stenosis managed without revascularization.American College of Surgeons SESAP 13 Critique of Category 8 Item 32 Contrast-induced nephropathy is one of the leading causes of acute renal failure in the hospital setting.6% to 1. Preprocedure hydration with normal saline is possible to prevent contrast-induced nephropathy even in patients with cardiac dysfunction. or hospital discharge would most often be delayed as part of routine management. Most patients with these elevations will not require hemodialysis. Dopaminergic agonists such as dopamine or fenoldapam have been recommended to preserve renal blood flow during the phase of contrast-induced renal artery vasoconstriction. Burgess WP. In a study of patients undergoing cardiac catheterization. et al: A randomized prospective trial to assess the role of saline hydration on the development of contrast nephrotoxicity.Merten GJ.Trivedi HS. Low osmolar contrast agents and hemofiltration have been successful in reducing the incidence of acute renal failure but are expensive. Tuttle KR: Atherosclerotic renal artery stenosis: current status and future directions. A complete review of the patient’s medication is required to decide whether any will have a negative impact on hydration and contrast use. et al: Prevention of contrast-induced nephropathy with sodium bicarbonate: a randomized controlled trial. no patient developed cardiac failure or respiratory distress during pre-procedure hydration. 2004 page 788 . N-acetylcysteine (Mucomyst) has also been recommended for its free radical scavenger properties. Elevations in baseline serum creatinine have been associated with a higher incidence of contrast-induced nephropathy. with published rates ranging from 0 to 90%. Mayo Clin Proc 75:437-444. 2003 4. McKusick MA. 2000 2. 2004 3. If the risk of acute renal failure outweighs the potential benefits from using contrast. Most recently. Gray LV. No randomized clinical trial has shown any of these methods to reduce the incidence of acute renal failure. but operation. further interventions. Schirger A. and hemofiltration is also associated with significant morbidity. Moore H. Curr Opin Nephrol Hypertens 13:613-621. REFERENCES 1. Most authors define a creatinine elevation > 44. A BUN/ creatinine ratio > 20 implies that the patient is not hydrated and would be at increased risk of renal dysfunction from contrast.Zalunardo N. Several treatment strategies have been investigated in an attempt to avoid this complication. Stanson AW. alternative methods of imaging should be pursued. The computed tomographic (CT) scan shown is obtained. Management has included fluid resuscitation and sigmoidoscopy. Urine output has been decreasing over the last 3 hours. and a stool sample has been sent for C difficile toxin assay and culture. he has profuse diarrhea and a heart rate of 130/min. After a 5-day course of metronidazole. Sigmoidoscopy showed no pseudomembranes. Which of the following statements is TRUE? (A) This patient has no criteria consistent with a diagnosis of fulminant C difficile colitis (B) Fulminant C difficile colitis is commonly associated with leukopenia (C) A history of C difficile confers immunity for recurrent infection with C difficile (D) Toxin assay may be negative in > 10% of patients with fulminant C difficile colitis (E) Medical and surgical therapy for fulminant C difficile colitis are associated with similar mortality rates page 789 .American College of Surgeons SESAP 13 Category 8 Item 33 Three weeks after elective sigmoid resection for cancer. a 70-year-old man has Clostridium difficile colitis. Ann Surg 235:363-372. REFERENCES 1. Prior infection with C difficile confers no immunity. Sirio CA. Therapy should be instituted prior to a positive toxin assay. Harbrecht BG. The external appearance of the bowel may be relatively benign. as seen in this patient’s computed tomographic (CT) scan. The procedure of choice is total abdominal colectomy. pericolonic stranding. severe abdominal pain. Subgroups of patients have also been identified who are incompletely treated and continue to have persistent symptoms. but intraoperative colonoscopy will confirm either pseudomembranes or diffuse patchy erythema and inflammation. Dis Colon Rectum 47:1620-1626. as in this patient. Most patients respond within 3 to 5 days of treatment. et al: Fulminant Clostridium difficile: an underappreciated and increasing cause of death and complications. Virgo KS. Lee P. 2002 2. Failure to respond may indicate the development of fulminant colitis with systemic signs of infection. Continued medical management of fulminant colitis is plagued by the continued risk of colon perforation and death. stool toxin assay was positive only in 82%. One series reported that 75% of patients requiring colectomy for C difficile colitis had documented prior infection. et al: Outcome after colectomy for Clostridium difficile colitis. and only 54% had pseudomembranes on colonoscopy. Mazuski JE. including fever and leukocytosis. Boujoukas AJ. Patients frequently have profuse watery diarrhea and present with massively distended or toxic megacolon. Low serum albumin and continued antibiotic therapy have been associated with increased rates of relapse. Surgical intervention at this point is associated with a mortality of 30% to 80%. 2004 page 790 . Relapse rates of 20% after therapy have also been reported. and ascites suggest toxic colitis. Common CT findings of bowel wall thickening.American College of Surgeons SESAP 13 Critique for Category 8 Item 33 The incidence of infections with Clostridium difficile is increasing and mortality remains high at 13%. and peritoneal signs on abdominal examination. In a retrospective review of VA patients with C difficile colitis.Longo WE.Dallal RM. A chest x-ray and computed tomographic (CT) scan are obtained. Four days postoperatively he has an increased oxygen requirement. 8) in the ICU. The hematoma is evacuated during the first 24 hours after injury.American College of Surgeons SESAP 13 Category 8 Item 34 A 23-year-old man has a subdural hematoma after a motor vehicle crash. He remains intubated and comatose (Glasgow coma scale score. Which of the following statements is TRUE? (A) He needs broad-spectrum antibiotics for ventilator-associated pneumonia (B) Chest tube placement will decrease his oxygen requirement (C) Systemic anticoagulation is contraindicated (D) Ventilation in the prone position will decrease his oxygen requirement (E) An IVC filter will decrease the subsequent risk of deep venous thrombosis page 791 . as shown. 2001 2. Heniford BT: Bedside insertion of inferior vena cava filters in the intensive care unit. There is no pneumothorax on the chest x-ray requiring chest tube placement or pneumonic process requiring antibiotics. REFERENCES 1. The CT also demonstrates significant filling defects consistent with a saddle pulmonary embolus in the pulmonary arteries. Systemic anticoagulation is the treatment of choice. Jacobs DG. but will do nothing to prevent the further development of deep venous thrombosis in the extremities. Fibrotic changes consistent with ARDS are not seen.Streiff MB: Vena caval filters: a review for intensive care specialists. which is not uncommon in ICU patients receiving mechanical ventilatory support. J Am Coll Surg 192:570-576.American College of Surgeons SESAP 13 Critique of Category 8 Item 34 This patient’s computed tomographic (CT) scan shows basilar atelectasis. J Intensive Care Med 18:5979. and prone positioning will most likely not help this patient.Sing RF. 2003 page 792 . Inferior vena caval filter placement will prevent subsequent pulmonary emboli. but contraindicated in this patient because of his closed head injury. The most likely etiology is (A) hypochloremia (B) hypoglycemia (C) hypocalcemia (D) hypomagnesemia (E) hypophosphatemia page 793 .American College of Surgeons SESAP 13 Category 8 Item 35 A 62-year-old man has hypokalemia refractory to potassium supplementation after hemiglossectomy. magnesium. pp 10411076 4. Magnesium depletion has been implicated in cardiac arrhythmias. 1996 3.Pollak MR. Lobo SS. ed 7. Philadelphia.Agus ZS: Hypomagnesemia. Am J Emerg Med 21:444-447. McGory R: Magnesium repletion and its effect on potassium homeostasis in critically ill adults: results of a double-blind. et al: Development of ionized hypomagnesemia is associated with higher mortality rates. Mercan D. but correction of the serum calcium or phosphate would have no effect on hypokalemia. 1999 2. In patients with reduced renal function. Hyperglycemia might increase potassium loss by reducing intracellular water and potassium and promoting an osmotic diuresis. REFERENCES 1. intravenous replacement is highly effective in correcting the deficiency. 2004. the rate of repletion should be reduced by 25% to 50%.Hamill-Ruth RJ. diarrhea.Stalnikowicz R: The significance of routine serum magnesium determination in the ED. inadequate nutrition. Hypocalcemia and hypophosphatemia are commonly found in association with hypomagnesemia. A simple regimen for nonemergent magnesium repletion is to administer 64 mEq (8 g) of magnesium sulfate over 24 hours and half of that dose (32 mEq) daily for the next 2 to 6 days. This relationship can be due to underlying disorders that cause loss of magnesium and potassium such as vomiting. Crit Care Med 24:38-45. controlled trial. Crit Care Med 31:1082-1087. and aminoglycoside use. J Am Soc Nephrol 10:1616-1622. In hospitalized patients. It is relatively refractory to isolated potassium replacement in the face of concomitant magnesium depletion. diuretic therapy. 2003 page 794 . Repletion of the other cations and anions listed has no effect on correcting hypokalemia. Significant improvement in potassium homeostasis occurs in critically ill patients when serum magnesium levels are maintained at high-normal levels. 2003 5. inexpensive. randomized. Saunders. Hypoglycemia would not. Hypokalemia occurs in 40% to 60% of hypomagnesemic patients and is even more common in critically ill patients. in Brenner BM (ed): Brenner & Rector’s: The Kidney.Soliman HM. and is well tolerated. Intracellular magnesium repletion occurs much more slowly than repletion of serum levels. and phosphate metabolism. and extended therapy is required. Yu ASL: Clinical disturbances of calcium. Melot C.American College of Surgeons SESAP 13 Critique for Category 8 Item 35 The plasma magnesium concentration is often not measured as part of routine screening or monitoring. Renal potassium loss is also increased in hypomagnesemic patients. especially when accompanied by hypokalemia. American College of Surgeons SESAP 13 Category 8 Item 36 All of the following are beneficial after acute myocardial infarction EXCEPT (A) beta blockers (B) angiotensin-converting enzyme (ACE) inhibitors (C) aspirin (D) thrombolysis (E) calcium channel blockers page 795 . Fibrinolytic Therapy Trialists' (FTT) Collaborative Group. Exner DV. One randomized clinical trial found limited evidence that for patients with left ventricular dysfunction. Hampton JR. prompt thrombolytic treatment (within 6 hours and perhaps up to 12 hours and longer after the onset of symptoms) reduced mortality compared with placebo. However. randomized clinical trials comparing different types of thrombolytic agents with each other found no significant difference in mortality. nifedipine given in the first few days after acute myocardial infarction may increase mortality compared with placebo. and stroke at 1 month compared with placebo. REFERENCES 1. Lancet 343:311322. The evidence suggests that prompt thrombolytic treatment is far more important than the particular thrombolytic agent used. one systematic review reported that aspirin reduces mortality. Borkowf CB.[No Authors Listed]: Collaborative overview of randomised trials of antiplatelet therapy—I: prevention of death. One nonsystematic review found that thrombolytic treatment increased the risk of stroke or major bleeding compared with control. re-infarction. Collins R. Mason J. Peto R. 1986 6. Banks DC.[No Authors Listed]: Indications for fibrinolytic therapy in suspected acute myocardial infarction: collaborative overview of early mortality and major morbidity results from all randomised trials of more than 1000 patients. one systematic review found that angiotensin converting enzyme (ACE) inhibitors reduce mortality after 6 weeks compared with placebo.American College of Surgeons SESAP 13 Critique for Category 8 Item 36 Two systematic reviews and one subsequent randomized clinical trial have found that â blockers reduce mortality compared with no â blockers. et al: Beta blockade during and after myocardial infarction: an overview of the randomized trials. et al: beta Blockade after myocardial infarction: systematic review and meta regression analysis. 1994 5.Wilcox RG.Freemantle N. 1999 2. 1999 3. et al: Trial of early nifedipine in acute myocardial infarction: the Trent study. and stroke by prolonged antiplatelet therapy in various categories of patients. et al: Effect of angiotensin converting enzyme inhibition on sudden cardiac death in patients following acute myocardial infarction. Geller NL. Young P. 1994 4. Br Med J 293:1204-1208. For patients with acute myocardial infarction. Birkhead JS. 1985 page 796 . Antiplatelet Trialists' Collaboration. A meta-analysis of randomized clinical trials. In one nonsystematic review of large randomized clinical trials in patients with acute myocardial infarction and ST segment elevation or bundle branch block on their initial EKG. Prog Cardiovasc Dis 27:335-371. J Am Coll Cardiol 33:598-604. Lewis J.Domanski MJ. myocardial infarction. A strategy of rapid thrombolysis in a broad population is likely to have the greatest impact on mortality. Cleland J. For patients treated within 14 days of acute myocardial infarction.Yusuf S. BMJ 318:1730-1737. There is no evidence that calcium channel blockers reduce mortality compared with placebo. BMJ 308:81-106. admitted with back pain and 2-mm inferior wall ST-depression on EKG (D) 48-year-old woman scheduled for sigmoid resection for a diverticular stricture who has aortic stenosis and dyspnea on walking 100 ft (E) 78-year-old man scheduled for transurethral resection of the prostate (TURP) who has controlled hypertension and left ventricular hypertrophy on EKG page 797 .American College of Surgeons SESAP 13 Category 8 Item 37 Preoperative invasive cardiac testing is MOST appropriate for a (A) 62-year-old man scheduled for laparoscopic incisional hernia repair who has been asymptomatic since coronary artery bypass 1 year ago (B) 57-year-old woman scheduled for lumpectomy with sentinel node biopsy who takes warfarin (Coumadin) for rate-controlled atrial fibrillation (C) 62-year-old man who has a 5-cm abdominal aortic aneurysm. often have insufficient cardiac reserve to withstand a major surgical stress. and symptomatic valvular heart disease are known markers of high cardiac risk. she has a low to intermediate overall cardiac risk because she is undergoing a relatively low risk procedure. Chaitman BR.Mukherjee D. Other serious co-morbidities such as diabetes. thoracic. Recent myocardial infarction (within 1 month of operation). aortic valve replacement prior to elective operation. extensive procedure with large fluid requirements). significant arrhythmias (high-grade atrioventricular block and supraventricular arrhythmias with uncontrolled ventricular response). If coronary stenting is elected for an identified lesion. and chronic renal and pulmonary disease may further complicate perioperative outcomes.Eagle KA. the emergency nature of his illness (ruptured aortic aneurysm) makes cardiac testing both inappropriate and impractical. unstable angina. Silverman DG: Assessment and management of patients with ischemic heart disease. Patients who are at low risk. based on clinical and functional status and proposed low-risk surgery. decompenstated heart failure. Eagle KA: Perioperative cardiac assessment for noncardiac surgery: eight steps to the best possible outcome. Patients with limited exercise capacity. eg. 2002 3. For patients (A) and (E). Circulation 107:2771-2774. shortness of breath walking a flight of stairs or short distances.Akhtar S. et al: ACC/AHA guideline update for perioperative cardiovascular evaluation for noncardiac surgery—executive summary a report of the American College of Cardiology/American Heart Association Task Force on Practice Guidelines (Committee to Update the 1996 Guidelines on Perioperative Cardiovascular Evaluation for Noncardiac Surgery). Although patient (C ) has a high cardiac risk. Berger PB. head and neck. Attention to fluids and rhythm control will be important to her care. Circulation 105:1257-1267. Although patient (B) has a controlled arrhythmia. ranging from minimal risk (endoscopy and superficial biopsies) to intermediate risk (intraperitoneal. coronary angiography. 2003 page 798 . Calkins H. because these conditions may be exacerbated under anesthesia or during the perioperative period. This patient would benefit from invasive cardiac testing and. Crit Care Med 32:S126-S136. perioperative -blockade and careful fluid management would be an appropriate management strategy. REFERENCES 1. Surgical procedures have been stratified for their relative cardiac risk. and the physiologic impact of the specific operation. orthopaedic. A history of stable or more moderate cardiac disease should still not be ignored. Patients with intermediate risk may be treated empirically with blockade and/or be considered for noninvasive testing. potentially. Patient (D) has symptomatic valvular heart disease with a severely limited functional status and is undergoing an elective abdominal procedure with a potential for large fluid shifts. Patients with high-risk clinical features and poor functional status undergoing high-risk procedures may benefit from preoperative invasive cardiac testing. the patient’s functional status. who have an intermediate cardiac risk.American College of Surgeons SESAP 13 Critique for Category 8 Item 37 The preoperative evaluation of cardiac risk for a patient undergoing noncardiac surgery centers on an assessment of underlying cardiac conditions. eg. major vascular. 2004 2. stroke. the primary surgical procedure should be delayed to allow for stent endothelization and a full course of antiplatelet therapy to minimize acute stent thrombosis. do not require further evaluation. urologic surgery) to high risk (emergent. American College of Surgeons SESAP 13 Category 8 Item 38 Which of the following is NOT a risk factor for ventilator-associated pneumonia? (A) Age > 60 years (B) Head trauma (C) Re-intubation (D) Frequent changes in ventilator tubing (E) Prior antibiotic exposure page 799 . Ann Intern Med 138:494-501. Because contamination of ventilator circuits is universal and has no clinical implication. Heyland D. et al: Evidence-based clinical practice guideline for the prevention of ventilator-associated pneumonia. 2003 2. burns. and aerosolization of bacteria is possible REFERENCES 1.Dodek P. aspiration of colonized secretions is believed to be responsible for the increased incidence of acquired pathogenic bacterial pneumonias. acute respiratory distress syndrome.Kollef MH: Prevention of hospital-associated pneumonia and ventilator-associated pneumonia. improper manipulation of the circuits may allow condensate from the warm humidified air that ventilates patients to precipitate. In turn. ventilator circuit change interval does not affect the incidence of ventilator-associated pneumonia (VAP). neurologic conditions. Clinical risk factors include chronic obstructive pulmonary disease. Prior exposure to antibiotics alters normal oropharyngeal flora and a prolonged preoperative hospitalization exposes patients to an environment harboring resistant bacteria. Keenan S. Matthay MA: Prevention of ventilator-associated pneumonia: an evidence-based systematic review. and witnessed aspirations.American College of Surgeons SESAP 13 Critique for Category 8 Item 38 Patients over age 60 who require prolonged (> 48 hours) mechanical ventilatory support are at high risk of ventilator-associated pneumonia (VAP). Ann Intern Med 141:305-313. 2004 page 800 . Surgeons can influence the incidence of VAP in their patients by altering their practice strategies. Patients requiring re-intubation or the use of paralytic agents are also at higher risk. Cook D. However. 2004 3. Saint S. Crit Care Med 32:1396-1405.Collard HR. American College of Surgeons SESAP 13 Category 8 Item 39 Which of the following statements about upper extremity deep venous thrombosis is TRUE? (A) It does not pose a risk for pulmonary embolus (B) Duplex is as sensitive for diagnosis as in lower extremity thrombosis (C) Treatment of thrombosis associated with a central line requires only removal of the catheter (D) Lean. young patients with a central line are at highest risk (E) Long-term anticoagulation is not required for treatment page 801 . Because central lines constitute a primary risk factor. Semin Vasc Med 1:105-110.Mustafa S. The strongest independent factor in the development of an upper extremity DVT is the presence of a central venous catheter. REFERENCES 1. and postthrombotic venous insufficiency of the arm and hand. Chest 123:1953-1956. limitation of venous access. Systemic or local infusion of thrombolytic agents has been successfully used. for which operation. Otten TR. Unlike lower extremity DVT. but controlled trials are lacking. placement of a superior vena cava filter may be warranted as a means of preventing symptomatic pulmonary embolism.Bernardi E. Tapson VF. Circulation 110:1605-1611. 2001 2. lean patients who will have prolonged placement of a central line should receive prophylaxis with low-dose heparin or low-dose warfarin. 2002 4. Goldhaber SZ: Upper-extremity deep vein thrombosis. Giorlami B. advanced age. superior vena cava syndrome. upper extremity DVT occurs more frequently in young patients with lean body mass. Therapeutic administration of unfractionated or fractionated low molecular weight heparin followed by oral anticoagulation for 3 months is a wellsupported treatment for upper extremity DVT and should be considered the treatment regimen of choice. Marchiori A.American College of Surgeons SESAP 13 Critique for Category 8 Item 39 Although previously believed to be self-limiting. Kucher N. Goldhaber SZ: Upper-extremity deep vein thrombosis: a prospective registry of 592 patients. Duplex ultrasonography is less sensitive and specific as a diagnostic modality for upper extremity DVT compared with lower extremity DVT. diagnosis and management. The young lean patient with a central line is at particular risk for upper extremity DVT and should be considered for DVT prophylaxis. and obesity pose independent risks. Stein PD. at least in part due to shadowing interference of the clavicle and sternum. Piccioli A. Patel KC. 2003 page 802 . The optimal management of upper extremity DVT remains controversial. et al: Upper extremity deep vein thrombosis: risk factors. but is associated with additional risk in surgical or trauma patients. deep venous thrombosis (DVT) of the axillary or subclavian vein(s) may be accompanied by potentially serious complications. Circulation 106:1874-1880. Many different therapeutic strategies have been used.Joffe HV. When anticoagulation or thrombolytic therapy is contraindicated. including pulmonary embolism.Joffe HV. 2004 3. et al: Upper extremity deep venous thrombosis. American College of Surgeons SESAP 13 Category 8 Item 40 Which of the following has been shown to reduce the risk of mortality for patients with severe sepsis? (A) Activated protein C (B) High-dose corticosteroids (C) Anti-endotoxin antibody (D) Erythropoetin (E) Total parenteral nutrition page 803 . at best high-dose corticosteroids (up to 600 mg/kg of hydrocortisone or equivalent per day) have no effect and may actually be detrimental. use has been somewhat limited in surgical patients but may be considered for those whose relative bleeding risk is acceptable. Because patients receiving activated protein C have a higher incidence of serious bleeding (3. et al: Novel therapies for sepsis: a review. no benefit has been documented. Corticosteroids have been used in multiple trials and the effect seems to be dose dependent. Several recent studies suggest that low-dose corticosteroids (100 to 200 mg/day) may be beneficial. N Engl J Med 344:699-709.0%).5% vs 2. 2001 2. et al: Efficacy and safety of recombinant human activated protein C for severe sepsis.Deans KJ. REFERENCES 1. Vincent JL. LaRosa FP. The benefit was greatest in patients whose APACHE II scores were > 25. J Trauma 58:867-874. 2005 page 804 . In a study of almost 1700 patients with severe sepsis and failure of at least one organ. Although results have been variable. administration of activated protein C administered within 12 to 24 hours of initial presentation and sustained for 96 hours was associated with a 6% absolute reduction in mortality (from 31% to 25%).American College of Surgeons SESAP 13 Critique for Category 8 Item 40 Activated protein C (drotrecogin alfa) is the only drug approved by the FDA for the reduction of mortality in patients with severe sepsis. Haley M. Erythropoetin has not been tested in the management of sepsis. Eichacker PQ. Multiple studies of total parenteral nutrition (TPN) in a variety of settings have not shown a benefit in survival.Bernard GR. Despite early encouraging data for anti-endotoxin therapy in several studies. Laterre PF. Natanson C. One hour later her oxygen saturation drops to 89% despite an FiO2 of 1. The chest x-ray shown is obtained. The next step in management should be (A) chest computed tomography (CT) (B) tube thoracostomy (C) video-assisted thoracic surgery (VATS) (D) laparotomy (E) intravenous heparin bolus and chest CT angiography page 805 .0.American College of Surgeons SESAP 13 Category 8 Item 41 A young woman undergoes intubation and placement of a central line after severe head injury. 2005 3. This sign of an anterior pneumothorax can be appreciated in up to 24% of otherwise occult pneumothoraces. given that a left hemidiaphragm is clearly visible on this chest x-ray. This patient’s chest x-ray demonstrates a deep sulcus sign. Episodes of hypoxia can significantly impair outcome after traumatic brain injury and must be investigated and treated aggressively. REFERENCES 1. 2002 2. especially in view of the abnormal chest x-ray. although it has been used in patients whose air leak persists 3 to 5 days after injury. Laupland KB. McQuatt A. A pulmonary embolus within the first hour of admission would not be a likely explanation for her hypoxia.Andrews PJ. Protetch J. J Emerg Med 26:231-232. and hypoperfusion. there is no evidence for this diagnosis. J Trauma 54:312—319.American College of Surgeons SESAP 13 Critique for Category 8 Item 41 This patient has a pneumothorax after insertion of a central venous line resulting in hypoxia.Jeremitsky E. a lucency of the lateral costophrenic angle with an abnormally deepened lateral costophrenic angle that has a sharp. Although laparotomy would be appropriate if a ruptured diaphragm were suspected. Omert L.Ball CG. Dunham CM. hypoxia. Although an anterior pneumothorax would be readily apparent on computed tomography (CT). A chest tube should be inserted immediately to correct her hypoxia. and are termed occult. Often these are anterior pneumothoraces where the lung collapses from anterior to posterior rather than the usual lateral to medial. Fox DL: Factors related to the failure of radiographic recognition of occult posttraumatic pneumothoraces. Up to 50% of pneumothoraces may be missed by supine portable chest x-ray after trauma. Statham PF. and no pleural strip is seen on a supine chest x-ray. Kirkpatrick AW. 2004 4.Henderson SO. However. There is no acute indication for video-assisted thoracic surgery (VATS) in this patient. Sleeman DH. and insertion of a chest tube is the immediate need. Shoenberger JM: Anterior pneumothorax and a negative chest X-ray in trauma. et al: Predicting recovery in patients suffering from traumatic brain injury by using admission variables and physiological data: a comparison between decision tree analysis and logistic regression. Am J Surg 189:541-546. et al: Harbingers of poor outcome the day after severe brain injury: hypothermia. many of these x-rays have subtle findings to suggest a pneumothorax. J Neurosurg 97:326-336. angular appearance. Systemic anticoagulation without a clear indication would be contraindicated with severe traumatic brain injury. 2003 page 806 . this patient is hypoxic with an abnormal chest x-ray strongly suggestive of a pneumothorax. American College of Surgeons SESAP 13 Category 8 Item 42 All of the following are involved in control of aldosterone secretion EXCEPT (A) angiotensin (B) dopamine (C) cortisol (D) serum potassium (E) serum sodium page 807 . Lever AF. J Endocrinol Invest 18:495-511.Gordon RD: Primary aldosteronism. Renin is released from the juxtaglomerular apparatus of the kidney in response to a decrease in vascular volume such as occurs with hemorrhage. Renin released into the bloodstream hydrolyzes a circulating protein substrate derived from the liver to produce the decapeptide angiotensin 1. et al: Control of aldosterone secretion. angiotensin 2. REFERENCES 1. ACTH alone is not sufficient to restore the full secretory capacity of the zona glomerulosa. Brown JJ. 1979 2. Aldosterone secretion is also controlled directly by the concentration of serum potassium. but cortisol has no effect on aldosterone secretion. Although ACTH can stimulate aldosterone. In the anephric patient.American College of Surgeons SESAP 13 Critique for Category 8 Item 42 The primary control mechanism for aldosterone secretion is the renin-angiotensin system. 1995 page 808 . a negative sodium balance. The peptide is then further cleaved by a converting enzyme in the lung to form an octopeptide. whereas a decrease results in a lowering of the aldosterone secretion rate. in the absence of the renin-angiotensin system. Clin Sci 56:389399. Studies have shown that dopamine may directly inhibit the secretion of aldosterone. Mason PA. An increase in serum potassium results in the stimulation of aldosterone secretion. or dehydration.Fraser R. Cortisol production is also stimulated by ACTH. Angiotensin 2 is a trophic hormone to the adrenal zona glomerulosa and stimulates secretion of aldosterone. serum potassium appears to be the primary mechanism for control of aldosterone secretion. and progressive weight loss.American College of Surgeons SESAP 13 Category 8 Item 43 A 58-year-old man with a long-standing pyloric channel ulcer that has not responded to therapy presents with a 5-day history of nausea and vomiting. His blood pressure is 95/60 and pulse rate is 110/minute. All of the following metabolic abnormalities are expected EXCEPT (A) elevated pH (B) low serum sodium (C) low serum chloride (D) low serum potassium (E) decreased ionized calcium page 809 . alkalosis will resolve. central venous pressure and arterial blood gas. REFERENCES 1. and allow conservation of chloride and excretion of bicarbonate. Overzealous replacement should be avoided and when hypernatremia is present. 1973 page 810 . Because extracellular fluid deficits accompany this condition. and multiplying it by one half of estimated total body water (0. and if necessary. the volume of replacement must be judged according to serum and urine electrolytes. The dose of hydrogen cation should be calculated by determining base excess (or increase in bicarbonate above 25 mEq/L). J Am Soc Nephrol 11:369-375. Am Rev Respir Dis 107:836-841. and significant loss of chloride in the vomitus is coupled with hydrogen ion loss. which decreases the concentration of ionized calcium (although total calcium concentration remains constant) and may lead to tetany. this bicarbonate is lost in compensatory fashion in the urine. The key to treatment of the metabolic abnormalities is giving isotonic saline. Luke RG. and Ringer’s lactate is not a good choice for fluid resuscitation in a patient with hypochloremic hyponatremic alkalosis.1N hydrochloric acid is safe if administered through a central venous line. it becomes urgently conserved at the renal tubule with bicarbonate at the expense of the hydrogen and potassium cations. more rapid treatment such as the administration of acid may be needed. a hypertonic solution should not be used. Rome L: Adaptations to chloride-depletion alkalosis. Most cases are associated with duodenal or pyloric channel ulceration. Gastric juice is chloride rich. half the sodium deficit should be replaced in the first 24 hours. with gastric ulceration accounting for only 5% of cases. 2000 2. The immediate hazards to the patient with an obstructed stomach are dehydration. the calculated deficits. With correction of the salt deficit. Infusion of 0. One half of this calculated amount is then administered over 24 hours. As sodium is lost both in the vomitus and in the urine accompanying the bicarbonate. judicious potassium supplementation is started.3 x weight). Early on. Gifford JD.American College of Surgeons SESAP 13 Critique for Category 8 Item 43 Gastric outlet obstruction is the least frequent ulcer complication. In the severely depleted patient. which will expand the extracellular fluid volume. Once adequate renal function is demonstrated. increase renal blood flow. and acid-base imbalance. Am J Physiol 261:R771-R781. Compensation for the alkalosis falters as a paradoxical acid urine is produced.Shear L. electrolyte disturbances. Brandman IS: Hypoxia and hypercapnia caused by respiratory compensation for metabolic alkalosis. This promotes intracellular acidosis and extracellular alkalosis. 1991 3. The classic metabolic derangement is hypochloremic hypokalemic metabolic alkalosis. The lactate in Ringer’s lactate solution is converted to bicarbonate. Hypokalemia is also accentuated and potassium is replaced at the intracellular level by hydrogen cation.Galla JH: Metabolic alkalosis. The serum becomes hypochloremic with the anion balance maintained by rising bicarbonate. When clinically significant hypoventilation develops as compensation for metabolic alkalosis.Galla JH. American College of Surgeons SESAP 13 Category 8 Item 44 A 58-year-old man with a long-standing pyloric channel ulcer that has not responded to therapy presents with a 5-day history of nausea and vomiting. His blood pressure is 95/60 and pulse rate is 110/minute. Initial resuscitation should include potassium chloride and (A) Ringer’s lactate solution (B) normal saline (C) half-normal saline (D) hydrochloric acid (E) hypertonic saline page 811 . and progressive weight loss. Gastric juice is chloride rich. central venous pressure and arterial blood gas. and acid-base imbalance. and significant loss of chloride in the vomitus is coupled with hydrogen ion loss. The classic metabolic derangement is hypochloremic hypokalemic metabolic alkalosis. it becomes urgently conserved at the renal tubule with bicarbonate at the expense of the hydrogen and potassium cations. the calculated deficits. Rome L: Adaptations to chloride-depletion alkalosis. This promotes intracellular acidosis and extracellular alkalosis. 1991 3. Compensation for the alkalosis falters as a paradoxical acid urine is produced.Shear L. increase renal blood flow. As sodium is lost both in the vomitus and in the urine accompanying the bicarbonate. Luke RG. Most cases are associated with duodenal or pyloric channel ulceration. In the severely depleted patient. Infusion of 0. judicious potassium supplementation is started.American College of Surgeons SESAP 13 Critique for Category 8 Item 44 Gastric outlet obstruction is the least frequent ulcer complication. When clinically significant hypoventilation develops as compensation for metabolic alkalosis.3 x weight). with gastric ulceration accounting for only 5% of cases. 2000 2. and allow conservation of chloride and excretion of bicarbonate. Because extracellular fluid deficits accompany this condition. The immediate hazards to the patient with an obstructed stomach are dehydration. Overzealous replacement should be avoided and when hypernatremia is present. One half of this calculated amount is then administered over 24 hours. J Am Soc Nephrol 11:369-375. and Ringer’s lactate is not a good choice for fluid resuscitation in a patient with hypochloremic hyponatremic alkalosis. and multiplying it by one half of estimated total body water (0. The dose of hydrogen cation should be calculated by determining base excess (or increase in bicarbonate above 25 mEq/L). more rapid treatment such as the administration of acid may be needed. electrolyte disturbances. The lactate in Ringer’s lactate solution is converted to bicarbonate. Brandman IS: Hypoxia and hypercapnia caused by respiratory compensation for metabolic alkalosis. Am J Physiol 261:R771-R781.1N hydrochloric acid is safe if administered through a central venous line. Gifford JD. Early on. which will expand the extracellular fluid volume. With correction of the salt deficit. Am Rev Respir Dis 107:836-841.Galla JH. which decreases the concentration of ionized calcium (although total calcium concentration remains constant) and may lead to tetany. Once adequate renal function is demonstrated. and if necessary. alkalosis will resolve. The serum becomes hypochloremic with the anion balance maintained by rising bicarbonate.Galla JH: Metabolic alkalosis. the volume of replacement must be judged according to serum and urine electrolytes. half the sodium deficit should be replaced in the first 24 hours. Hypokalemia is also accentuated and potassium is replaced at the intracellular level by hydrogen cation. REFERENCES 1. a hypertonic solution should not be used. 1973 page 812 . The key to treatment of the metabolic abnormalities is giving isotonic saline. this bicarbonate is lost in compensatory fashion in the urine. American College of Surgeons SESAP 13 Category 8 Item 45 Paralysis commonly used (A) Pressure regulated volume control (PRVC) (B) Pressure controlled inverse ratio ventilation (PC-IRV) (C) Pressure support ventilation (PSV) (D) Airway pressure release ventilation (APRV) (E) Continuous positive airway pressure (CPAP) page 813 . Selecting a support mode requires an understanding of the indications. Critical Care. It is quite comfortable and tolerated by the awake alert intubated patient. PC-IRV is indicated for severe acute lung injury. in Souba WW. auto-PEEP is common. benefits. Kaiser LR. Nathens AB: Mechanical ventilation. CPAP is a fixed pressure delivered throughout the respiratory phase and is commonly used in the nonintubated patient to improve ventilatory function. Pressure regulated volume control (PRVC) is commonly used as a mainstay of ventilator management in patients with pulmonary failure. Pressure support ventilation (PSV) is a pre-set pressure delivery that allows variation in spontaneous tidal volume. Fink MP. PRVC achieves a pre-set tidal volume by continuously evaluating the tidal volume and adjusting the pressure accordingly to maintain proper tidal volume. www. Pressure controlled inverse ratio ventilation (PC-IRV) increases the inspiratory phase dramatically. which is intermittently released for short periods to allow lung emptying. Jurkovich GJ. Hyzy RC: Conventional mechanical ventilation.Bhan U.Sena MJ.American College of Surgeons SESAP 13 Critique for Category 8 Item 45 A variety of mechanical ventilatory support modes are now available. APRV allows spontaneous respirations thereby limiting the need for neuromuscular blocking agents.utdol. Paralysis may be necessary to achieve this unnatural sequence of the inspiratory to expiratory phase. WebMD Inc.com 2. New York. chap 8. and risks of each mode. pp 1390-1407 page 814 . REFERENCES 1. Available at: Up To Date. Airway pressure release ventilation (APRV) is a new modality that uses high continuous positive airway pressure (CPAP). sec 5. et al (eds): ACS Surgery: Principles and Practice 2006. Because of the shortened expiratory time. American College of Surgeons SESAP 13 Category 8 Item 46 Noninvasive mode (A) Pressure regulated volume control (PRVC) (B) Pressure controlled inverse ratio ventilation (PC-IRV) (C) Pressure support ventilation (PSV) (D) Airway pressure release ventilation (APRV) (E) Continuous positive airway pressure (CPAP) page 815 . Nathens AB: Mechanical ventilation. Pressure controlled inverse ratio ventilation (PC-IRV) increases the inspiratory phase dramatically. pp 1390-1407 page 816 . Available at: Up To Date.com 2. PC-IRV is indicated for severe acute lung injury.Bhan U. Hyzy RC: Conventional mechanical ventilation. CPAP is a fixed pressure delivered throughout the respiratory phase and is commonly used in the nonintubated patient to improve ventilatory function. Paralysis may be necessary to achieve this unnatural sequence of the inspiratory to expiratory phase. It is quite comfortable and tolerated by the awake alert intubated patient. Kaiser LR. PRVC achieves a pre-set tidal volume by continuously evaluating the tidal volume and adjusting the pressure accordingly to maintain proper tidal volume. et al (eds): ACS Surgery: Principles and Practice 2006. Pressure support ventilation (PSV) is a pre-set pressure delivery that allows variation in spontaneous tidal volume. Critical Care. Because of the shortened expiratory time. sec 5. benefits. www.utdol. chap 8. REFERENCES 1. and risks of each mode.American College of Surgeons SESAP 13 Critique for Category 8 Item 46 A variety of mechanical ventilatory support modes are now available. Airway pressure release ventilation (APRV) is a new modality that uses high continuous positive airway pressure (CPAP). Pressure regulated volume control (PRVC) is commonly used as a mainstay of ventilator management in patients with pulmonary failure. Fink MP. New York. Jurkovich GJ. Selecting a support mode requires an understanding of the indications.Sena MJ. in Souba WW. APRV allows spontaneous respirations thereby limiting the need for neuromuscular blocking agents. which is intermittently released for short periods to allow lung emptying. auto-PEEP is common. WebMD Inc. American College of Surgeons SESAP 13 Category 8 Item 47 Auto PEEP common (A) Pressure regulated volume control (PRVC) (B) Pressure controlled inverse ratio ventilation (PC-IRV) (C) Pressure support ventilation (PSV) (D) Airway pressure release ventilation (APRV) (E) Continuous positive airway pressure (CPAP) page 817 . Pressure controlled inverse ratio ventilation (PC-IRV) increases the inspiratory phase dramatically. et al (eds): ACS Surgery: Principles and Practice 2006. Selecting a support mode requires an understanding of the indications. Pressure support ventilation (PSV) is a pre-set pressure delivery that allows variation in spontaneous tidal volume.utdol. Because of the shortened expiratory time. Jurkovich GJ. Hyzy RC: Conventional mechanical ventilation. Airway pressure release ventilation (APRV) is a new modality that uses high continuous positive airway pressure (CPAP). REFERENCES 1.American College of Surgeons SESAP 13 Critique for Category 8 Item 47 A variety of mechanical ventilatory support modes are now available. pp 1390-1407 page 818 . which is intermittently released for short periods to allow lung emptying. auto-PEEP is common. chap 8. benefits. It is quite comfortable and tolerated by the awake alert intubated patient. APRV allows spontaneous respirations thereby limiting the need for neuromuscular blocking agents. sec 5. WebMD Inc. www. Kaiser LR. Available at: Up To Date.Bhan U. PC-IRV is indicated for severe acute lung injury.com 2. Paralysis may be necessary to achieve this unnatural sequence of the inspiratory to expiratory phase. Pressure regulated volume control (PRVC) is commonly used as a mainstay of ventilator management in patients with pulmonary failure. Nathens AB: Mechanical ventilation. and risks of each mode. CPAP is a fixed pressure delivered throughout the respiratory phase and is commonly used in the nonintubated patient to improve ventilatory function. PRVC achieves a pre-set tidal volume by continuously evaluating the tidal volume and adjusting the pressure accordingly to maintain proper tidal volume. in Souba WW. Critical Care.Sena MJ. New York. Fink MP. American College of Surgeons SESAP 13 Category 8 Item 48 57-year-old hypertensive man who develops torsades de pointes 4 days after 4-vessel coronary artery bypass graft (CABG) for severe coronary artery disease associated with recent anterior myocardial infarct (A) Hyperkalemia (B) Hypokalemia (C) Hypermagnesemia (D) Hypomagnesemia (E) Hypocalcemia page 819 . Magnesium administration is sometimes used as a therapy for premature labor. pp 1087-1091 page 820 . Oka T. It can be seen after neck surgery. Lippincott Williams & Wilkins. Emmett M: Fluids and electrolytes.Majercik S. 2002. Hypocalcemia has also been associated with laryngeal spasm. as illustrated. where injury to the gland occurs. 2004. Hypomagnesemia is often characterized by hyperreflexia and seizures. Philadelphia. Capote LR (eds): The Physiologic Basis of Surgery. and carpal pedal spasm. Hypocalcemia is associated with paresthesias. Cioffi WG: Fluid and electrolyte therapy. para-oral tingling.Fenves AZ. and abdominal pain are sometimes also present. Hypokalemia produces flat T-waves and is often associated with a marked ileus. in O’Leary JP. The patient often has prolonged QT intervals. ed 8. Torsades de pointes is a ventricular tachycardia that is associated with a long QT interval. ed 3. It is also associated with the serious cardiac arrhythmia of torsades de pointes.American College of Surgeons SESAP 13 Critique for Category 8 Item 48 Hyperkalemia is associated with renal failure. pp 84-88 2. in Cameron JL (ed): Current Surgical Therapy. REFERENCES 1. Mosby. peaked T-waves. Hypermagnesia is associated with flaccid paralysis and hyporeflexia. vomiting. Nausea. widened QRS complexes. New Orleans. and in severe cases. American College of Surgeons SESAP 13 Category 8 Item 49 22-year-old prima gravida undergoing therapy for premature labor who is hyporeflexive and unable to move (A) Hyperkalemia (B) Hypokalemia (C) Hypermagnesemia (D) Hypomagnesemia (E) Hypocalcemia page 821 . Lippincott Williams & Wilkins. Hypocalcemia has also been associated with laryngeal spasm. Oka T. widened QRS complexes. ed 3. and carpal pedal spasm. The patient often has prolonged QT intervals. Hypokalemia produces flat T-waves and is often associated with a marked ileus. pp 84-88 2. para-oral tingling.Majercik S. Hypomagnesemia is often characterized by hyperreflexia and seizures. 2004. Hypermagnesia is associated with flaccid paralysis and hyporeflexia. It is also associated with the serious cardiac arrhythmia of torsades de pointes. where injury to the gland occurs. Philadelphia. pp 1087-1091 page 822 . in Cameron JL (ed): Current Surgical Therapy. Hypocalcemia is associated with paresthesias. vomiting.Fenves AZ. peaked T-waves. 2002. and in severe cases. and abdominal pain are sometimes also present. Emmett M: Fluids and electrolytes. Cioffi WG: Fluid and electrolyte therapy. Capote LR (eds): The Physiologic Basis of Surgery. Nausea. New Orleans. Mosby. as illustrated.American College of Surgeons SESAP 13 Critique for Category 8 Item 49 Hyperkalemia is associated with renal failure. It can be seen after neck surgery. Magnesium administration is sometimes used as a therapy for premature labor. REFERENCES 1. in O’Leary JP. ed 8. Torsades de pointes is a ventricular tachycardia that is associated with a long QT interval. hypertensive female smoker who has decreased urine output and evidence of peaked T-waves on EKG 2 days after suprarenal aortic aneurysm repair (A) Hyperkalemia (B) Hypokalemia (C) Hypermagnesemia (D) Hypomagnesemia (E) Hypocalcemia page 823 .American College of Surgeons SESAP 13 Category 8 Item 50 67-year-old diabetic. American College of Surgeons SESAP 13 Critique for Category 8 Item 50 Hyperkalemia is associated with renal failure. ed 8. 2002. Hypocalcemia is associated with paresthesias. Hypermagnesia is associated with flaccid paralysis and hyporeflexia. Cioffi WG: Fluid and electrolyte therapy. Hypocalcemia has also been associated with laryngeal spasm. The patient often has prolonged QT intervals. Magnesium administration is sometimes used as a therapy for premature labor.Fenves AZ. widened QRS complexes. and carpal pedal spasm. Hypomagnesemia is often characterized by hyperreflexia and seizures. pp 1087-1091 page 824 . Emmett M: Fluids and electrolytes. pp 84-88 2. as illustrated. and abdominal pain are sometimes also present. Philadelphia. para-oral tingling. where injury to the gland occurs. Torsades de pointes is a ventricular tachycardia that is associated with a long QT interval. 2004. Capote LR (eds): The Physiologic Basis of Surgery. in Cameron JL (ed): Current Surgical Therapy. New Orleans. It can be seen after neck surgery. Hypokalemia produces flat T-waves and is often associated with a marked ileus. Nausea. vomiting. Oka T. It is also associated with the serious cardiac arrhythmia of torsades de pointes. Lippincott Williams & Wilkins. and in severe cases. ed 3. Mosby.Majercik S. in O’Leary JP. peaked T-waves. REFERENCES 1. American College of Surgeons SESAP 13 Category 8 Item 51 Blocks glycoprotein IIb/IIIa receptor (A) Aspirin (B) Clopidogrel (Plavix) (C) Abciximab (Reopro) (D) Cilostazol (Pletal) (E) Ibuprofen page 825 . These data show that clopidogrel should be discontinued at least 5 to 7 days before elective operations. and severe thrombocytopenia occurs in 1% to 2% of patients. then platelet transfusions may be required. The antiplatelet effects usually disappear within 12 hours after therapy is stopped. bleeding complications were much more frequent in the clopidogrel plus aspirin group. Platelet counts are expected to return to normal over several days. Abciximab (Reopro) is one of a new family of antiplatelet agents that inhibit the platelet glycoprotein IIb/IIIa receptor. Bleeding complications are significantly increased in patients who receive abciximab. In several prospective studies of clopidogrel plus aspirin versus aspirin alone for patients with cardiovascular disease. This agent is used to improve walking distance in patients with intermittent claudication. These drugs selectively inhibit ADP receptor-mediated platelet aggregation. all bleeding times returned to normal levels within 96 hours. these agents are not used as antiplatelet agents in the United States. or both. reoperation. but they increase the risk of bleeding in surgical patients. patients who receive clopidogrel within 5 days of operation have a high risk of major bleeding requiring transfusions. and side effects. Clopidogrel is the preferred agent because ticlopidine can cause thrombocytopenic purpura and neutropenia. bleeding is a reported side effect. page 826 . which will allow the agents to be discontinued during the perioperative period to minimize the bleeding risk. These data support cessation of aspirin 5 days before operation. However. Infusions of these agents reduce the risk of procedure-related vessel occlusion after percutaneous angioplasty. or death from all cardiovascular causes. Cilostazol (Pletal) is a reversible phosphodiesterase III inhibitor that allows for increased availability of cAMP. In a study using sensitive measures of platelet function. Furthermore. Because conventional doses of NSAIDs are not adequate to block platelet aggregation in vivo. Clopidogrel (Plavix) and ticlopidine (Ticlid) are thienopyridenes with strong platelet inhibitory properties. Aspirin is a relatively weak antiplatelet agent because it blocks only one pathway by which platelet activation and aggregation occur. These drugs differ in their mechanisms of action. Aspirin blocks the conversion of arachidonic acid to thromboxane by permanently inactivating the cyclo-oxygenase activity of the prostaglandin synthase-1 (COX-1) and prostaglandin synthase-2 (COX-2). All platelet effects are reversed within 24 hours of drug cessation. The effect lasts for the lifetime of the platelet. If an emergency operation is required sooner. and all platelet tests were normal within 144 hours after aspirin was stopped. Nonsteriodal anti-inflammatory drugs (NSAIDs) inhibit thromboxane-dependent platelet function by reversibly inhibiting COX-1. clopidogrel plus aspirin was more effective in reducing the combined risk incidence of stroke.American College of Surgeons SESAP 13 Critique for Category 8 Item 51 Antiplatelet agents are widely used to reduce adverse vascular events. These drugs represent the most potent and expensive antiplatelet agents and are used in patients at high risk for adverse coronary events who undergo intervention within 1 to 2 days of hospitalization. myocardial infarction. leading to vasodilation and platelet inhibition. It is a relatively weak inhibitor of platelet aggregation. Surgeons should be aware of the duration of effect for commonly used antiplatelet agents. duration of effect. Understanding the mechanism of action of these agents will help guide therapy when bleeding does occur. and cilostazol should be discontinued at least 5 to 7 days before elective operation. However. and coadministration with aspirin or warfarin does not lead to significant changes in coagulation parameters. 2001 page 827 . Manco-Johnson MJ: Brief communication: duration of platelet dysfunction after a 7-day course of ibuprofen. Jacobson L.Lange RA. McGreal GT. Ryan DA. Hillis LD: Antiplatelet therapy for ischemic heart disease.Patrono C. Ann Intern Med 142:506-509. J Am Coll Surg 200:564-573. 2004 5. Mohler ER III: Drug treatment of intermittent claudication.Goldenberg NA. and side effects. et al: Platelet-active drugs: the relationships among dose. Chest 119:39S-63S. Crowe BH. Coller B.Cahill RA. effectiveness. Dalen JE.Jacoby D. N Engl J Med 350:277-280. 2005 3. 2004 4. et al: Duration of increased bleeding tendency after cessation of aspirin therapy. Drugs 64:1657-1670.American College of Surgeons SESAP 13 REFERENCES 1. Fitzgerald GA. 2005 2. American College of Surgeons SESAP 13 Category 8 Item 52 Inhibits platelet phosphodiesterase (A) Aspirin (B) Clopidogrel (Plavix) (C) Abciximab (Reopro) (D) Cilostazol (Pletal) (E) Ibuprofen page 828 . or both. these agents are not used as antiplatelet agents in the United States. but they increase the risk of bleeding in surgical patients. Abciximab (Reopro) is one of a new family of antiplatelet agents that inhibit the platelet glycoprotein IIb/IIIa receptor. The antiplatelet effects usually disappear within 12 hours after therapy is stopped. However. leading to vasodilation and platelet inhibition. and all platelet tests were normal within 144 hours after aspirin was stopped. Aspirin blocks the conversion of arachidonic acid to thromboxane by permanently inactivating the cyclo-oxygenase activity of the prostaglandin synthase-1 (COX-1) and prostaglandin synthase-2 (COX-2). These data show that clopidogrel should be discontinued at least 5 to 7 days before elective operations. Clopidogrel (Plavix) and ticlopidine (Ticlid) are thienopyridenes with strong platelet inhibitory properties. bleeding is a reported side effect. then platelet transfusions may be required. myocardial infarction. These drugs represent the most potent and expensive antiplatelet agents and are used in patients at high risk for adverse coronary events who undergo intervention within 1 to 2 days of hospitalization. and side effects. Cilostazol (Pletal) is a reversible phosphodiesterase III inhibitor that allows for increased availability of cAMP. Platelet counts are expected to return to normal over several days. These drugs selectively inhibit ADP receptor-mediated platelet aggregation. However. reoperation. and coadministration with aspirin or warfarin does not lead to significant changes in coagulation parameters. In several prospective studies of clopidogrel plus aspirin versus aspirin alone for patients with cardiovascular disease. clopidogrel plus aspirin was more effective in reducing the combined risk incidence of stroke. Infusions of these agents reduce the risk of procedure-related vessel occlusion after percutaneous angioplasty. Aspirin is a relatively weak antiplatelet agent because it blocks only one pathway by which platelet activation and aggregation occur. All platelet effects are reversed within 24 hours of drug cessation. These drugs differ in their mechanisms of action. The effect lasts for the lifetime of the platelet. and severe thrombocytopenia occurs in 1% to 2% of patients. duration of effect.American College of Surgeons SESAP 13 Critique of Category 8 Item 52 Antiplatelet agents are widely used to reduce adverse vascular events. Nonsteriodal anti-inflammatory drugs (NSAIDs) inhibit thromboxane-dependent platelet function by reversibly inhibiting COX-1. or death from all cardiovascular causes. Bleeding complications are significantly increased in patients who receive abciximab. all bleeding times returned to normal levels within 96 hours. Surgeons should be aware of the duration of effect for commonly used antiplatelet agents. If an emergency operation is required sooner. and cilostazol should be discontinued at least 5 to 7 days before elective operation. In a study using sensitive measures of platelet function. Clopidogrel is the preferred agent because ticlopidine can cause thrombocytopenic purpura and neutropenia. Understanding the mechanism of action of these agents will help guide therapy when bleeding does occur. It is a relatively weak inhibitor of platelet aggregation. Because conventional doses of NSAIDs are not adequate to block platelet aggregation in vivo. page 829 . patients who receive clopidogrel within 5 days of operation have a high risk of major bleeding requiring transfusions. bleeding complications were much more frequent in the clopidogrel plus aspirin group. These data support cessation of aspirin 5 days before operation. which will allow the agents to be discontinued during the perioperative period to minimize the bleeding risk. Furthermore. This agent is used to improve walking distance in patients with intermittent claudication. Jacoby D. 2001 page 830 .American College of Surgeons SESAP 13 REFERENCES 1. 2004 5. Dalen JE. et al: Platelet-active drugs: the relationships among dose. Mohler ER III: Drug treatment of intermittent claudication.Cahill RA. Ann Intern Med 142:506-509. Crowe BH. N Engl J Med 350:277-280. J Am Coll Surg 200:564-573. 2004 4.Patrono C. 2005 3. McGreal GT. Fitzgerald GA. Hillis LD: Antiplatelet therapy for ischemic heart disease. et al: Duration of increased bleeding tendency after cessation of aspirin therapy. Manco-Johnson MJ: Brief communication: duration of platelet dysfunction after a 7-day course of ibuprofen. Jacobson L. and side effects. Coller B. effectiveness. 2005 2. Drugs 64:1657-1670.Lange RA.Goldenberg NA. Ryan DA. Chest 119:39S-63S. American College of Surgeons SESAP 13 Category 8 Item 53 Permanent COX-1 and COX-2 inhibitor (A) Aspirin (B) Clopidogrel (Plavix) (C) Abciximab (Reopro) (D) Cilostazol (Pletal) (E) Ibuprofen page 831 . leading to vasodilation and platelet inhibition. These drugs selectively inhibit ADP receptor-mediated platelet aggregation. and all platelet tests were normal within 144 hours after aspirin was stopped. The effect lasts for the lifetime of the platelet. Furthermore. Because conventional doses of NSAIDs are not adequate to block platelet aggregation in vivo. which will allow the agents to be discontinued during the perioperative period to minimize the bleeding risk. Surgeons should be aware of the duration of effect for commonly used antiplatelet agents. reoperation. Abciximab (Reopro) is one of a new family of antiplatelet agents that inhibit the platelet glycoprotein IIb/IIIa receptor. or both. and cilostazol should be discontinued at least 5 to 7 days before elective operation. Nonsteriodal anti-inflammatory drugs (NSAIDs) inhibit thromboxane-dependent platelet function by reversibly inhibiting COX-1. Infusions of these agents reduce the risk of procedure-related vessel occlusion after percutaneous angioplasty. These drugs represent the most potent and expensive antiplatelet agents and are used in patients at high risk for adverse coronary events who undergo intervention within 1 to 2 days of hospitalization. these agents are not used as antiplatelet agents in the United States. bleeding is a reported side effect. and severe thrombocytopenia occurs in 1% to 2% of patients. page 832 . or death from all cardiovascular causes. Bleeding complications are significantly increased in patients who receive abciximab. all bleeding times returned to normal levels within 96 hours. Cilostazol (Pletal) is a reversible phosphodiesterase III inhibitor that allows for increased availability of cAMP. However. This agent is used to improve walking distance in patients with intermittent claudication. These data support cessation of aspirin 5 days before operation. However. patients who receive clopidogrel within 5 days of operation have a high risk of major bleeding requiring transfusions. These drugs differ in their mechanisms of action. bleeding complications were much more frequent in the clopidogrel plus aspirin group. Clopidogrel (Plavix) and ticlopidine (Ticlid) are thienopyridenes with strong platelet inhibitory properties. Platelet counts are expected to return to normal over several days. clopidogrel plus aspirin was more effective in reducing the combined risk incidence of stroke.American College of Surgeons SESAP 13 Critique of Category 8 Item 53 Antiplatelet agents are widely used to reduce adverse vascular events. In several prospective studies of clopidogrel plus aspirin versus aspirin alone for patients with cardiovascular disease. It is a relatively weak inhibitor of platelet aggregation. All platelet effects are reversed within 24 hours of drug cessation. Aspirin blocks the conversion of arachidonic acid to thromboxane by permanently inactivating the cyclo-oxygenase activity of the prostaglandin synthase-1 (COX-1) and prostaglandin synthase-2 (COX-2). The antiplatelet effects usually disappear within 12 hours after therapy is stopped. If an emergency operation is required sooner. and side effects. Aspirin is a relatively weak antiplatelet agent because it blocks only one pathway by which platelet activation and aggregation occur. myocardial infarction. These data show that clopidogrel should be discontinued at least 5 to 7 days before elective operations. but they increase the risk of bleeding in surgical patients. Clopidogrel is the preferred agent because ticlopidine can cause thrombocytopenic purpura and neutropenia. then platelet transfusions may be required. and coadministration with aspirin or warfarin does not lead to significant changes in coagulation parameters. In a study using sensitive measures of platelet function. Understanding the mechanism of action of these agents will help guide therapy when bleeding does occur. duration of effect. 2004 5. et al: Duration of increased bleeding tendency after cessation of aspirin therapy. 2001 page 833 . and side effects. J Am Coll Surg 200:564-573. Hillis LD: Antiplatelet therapy for ischemic heart disease. Fitzgerald GA. Ryan DA. Dalen JE. Chest 119:39S-63S.Goldenberg NA. 2005 3.Lange RA.Jacoby D. effectiveness. 2004 4. Ann Intern Med 142:506-509. N Engl J Med 350:277-280.Cahill RA.Patrono C. Drugs 64:1657-1670. et al: Platelet-active drugs: the relationships among dose. Mohler ER III: Drug treatment of intermittent claudication. Coller B.American College of Surgeons SESAP 13 REFERENCES 1. Jacobson L. Manco-Johnson MJ: Brief communication: duration of platelet dysfunction after a 7-day course of ibuprofen. 2005 2. Crowe BH. McGreal GT. American College of Surgeons SESAP 13 Category 8 Item 54 Blocks platelet ADP receptor (A) Aspirin (B) Clopidogrel (Plavix) (C) Abciximab (Reopro) (D) Cilostazol (Pletal) (E) Ibuprofen page 834 . clopidogrel plus aspirin was more effective in reducing the combined risk incidence of stroke. or both. However. These data support cessation of aspirin 5 days before operation. The antiplatelet effects usually disappear within 12 hours after therapy is stopped. leading to vasodilation and platelet inhibition. Abciximab (Reopro) is one of a new family of antiplatelet agents that inhibit the platelet glycoprotein IIb/IIIa receptor. but they increase the risk of bleeding in surgical patients. Bleeding complications are significantly increased in patients who receive abciximab. Clopidogrel (Plavix) and ticlopidine (Ticlid) are thienopyridenes with strong platelet inhibitory properties. In a study using sensitive measures of platelet function. This agent is used to improve walking distance in patients with intermittent claudication. It is a relatively weak inhibitor of platelet aggregation. Clopidogrel is the preferred agent because ticlopidine can cause thrombocytopenic purpura and neutropenia. then platelet transfusions may be required. In several prospective studies of clopidogrel plus aspirin versus aspirin alone for patients with cardiovascular disease. Infusions of these agents reduce the risk of procedure-related vessel occlusion after percutaneous angioplasty. reoperation. or death from all cardiovascular causes. However. and side effects. Surgeons should be aware of the duration of effect for commonly used antiplatelet agents. Aspirin is a relatively weak antiplatelet agent because it blocks only one pathway by which platelet activation and aggregation occur. and severe thrombocytopenia occurs in 1% to 2% of patients. myocardial infarction. These drugs differ in their mechanisms of action. patients who receive clopidogrel within 5 days of operation have a high risk of major bleeding requiring transfusions. duration of effect. Furthermore. These data show that clopidogrel should be discontinued at least 5 to 7 days before elective operations. All platelet effects are reversed within 24 hours of drug cessation. These drugs selectively inhibit ADP receptor-mediated platelet aggregation. and cilostazol should be discontinued at least 5 to 7 days before elective operation. Nonsteriodal anti-inflammatory drugs (NSAIDs) inhibit thromboxane-dependent platelet function by reversibly inhibiting COX-1. these agents are not used as antiplatelet agents in the United States. Because conventional doses of NSAIDs are not adequate to block platelet aggregation in vivo. which will allow the agents to be discontinued during the perioperative period to minimize the bleeding risk. Cilostazol (Pletal) is a reversible phosphodiesterase III inhibitor that allows for increased availability of cAMP. The effect lasts for the lifetime of the platelet. Platelet counts are expected to return to normal over several days. These drugs represent the most potent and expensive antiplatelet agents and are used in patients at high risk for adverse coronary events who undergo intervention within 1 to 2 days of hospitalization. If an emergency operation is required sooner. bleeding is a reported side effect. bleeding complications were much more frequent in the clopidogrel plus aspirin group.American College of Surgeons SESAP 13 Critique for Category 8 Item 54 Antiplatelet agents are widely used to reduce adverse vascular events. and all platelet tests were normal within 144 hours after aspirin was stopped. Understanding the mechanism of action of these agents will help guide therapy when bleeding does occur. page 835 . and coadministration with aspirin or warfarin does not lead to significant changes in coagulation parameters. all bleeding times returned to normal levels within 96 hours. Aspirin blocks the conversion of arachidonic acid to thromboxane by permanently inactivating the cyclo-oxygenase activity of the prostaglandin synthase-1 (COX-1) and prostaglandin synthase-2 (COX-2). Lange RA. N Engl J Med 350:277-280. Ann Intern Med 142:506-509. Coller B. 2004 5. Dalen JE. effectiveness. 2001 page 836 . 2005 3. Manco-Johnson MJ: Brief communication: duration of platelet dysfunction after a 7-day course of ibuprofen. Drugs 64:1657-1670. Mohler ER III: Drug treatment of intermittent claudication. 2005 2. Crowe BH. et al: Duration of increased bleeding tendency after cessation of aspirin therapy.American College of Surgeons SESAP 13 REFERENCES 1. 2004 4. J Am Coll Surg 200:564-573. Fitzgerald GA. Jacobson L.Cahill RA. and side effects. Ryan DA. Chest 119:39S-63S. et al: Platelet-active drugs: the relationships among dose.Patrono C.Goldenberg NA. McGreal GT.Jacoby D. Hillis LD: Antiplatelet therapy for ischemic heart disease. American College of Surgeons SESAP 13 Category 8 Item 55 Primary action on factor Xa (A) Fondaparinux (B) Low molecular weight heparin (LMWH) (C) Both (D) Neither page 837 . in Rutherford RB (ed): Vascular Surgery. Neither drug is a direct thrombin inhibitor.[No Authors Listed]: Fondaparinux (Arixtra). the inactivation of factor IIa requires an additional 16-unit saccharide chain to produce a conformational change.Meissner MH: Antithrombotic therapy. 2002 page 838 . ed 6. 2005. Although the incidence of heparin-induced thrombocytopenia appears to be lower with low molecular weight heparin than with unfractionated heparin. WB Saunders. a synthetic pentasaccharide. bind to the heparin-binding site of antithrombin III. a new anticoagulant. pp 511-529 2. Philadelphia. increasing its antifactor Xa activity. Because both fondaparinux and low molecular weight heparin bind to the smaller site and do not confer the conformational change. they are ineffective against factor IIa. The activated partial thromboplastin time (aPTT) is not useful for monitoring either because it measures factor IIa. Because fondaparinux does not bind to platelet factor IV. REFERENCES 1. dosages should be adjusted in patients with renal failure. Med Lett Drugs Ther 44:43-44.American College of Surgeons SESAP 13 Critique for Category 8 Item 55 Low molecular weight heparin (LMWH) and fondaparinux. it does not cause immune-mediated thrombocytopenias. Because both medications are cleared by the kidneys. However. Antithrombin III inactivates both factor Xa and factor IIa. it has been reported. American College of Surgeons SESAP 13 Category 8 Item 56 Monitored by activated partial thromboplastin time (aPTT) (A) Fondaparinux (B) Low molecular weight heparin (LMWH) (C) Both (D) Neither page 839 . Meissner MH: Antithrombotic therapy. it has been reported. pp 511-529 2. Philadelphia. it does not cause immune-mediated thrombocytopenias. Neither drug is a direct thrombin inhibitor. Antithrombin III inactivates both factor Xa and factor IIa. they are ineffective against factor IIa. a synthetic pentasaccharide. Because both fondaparinux and low molecular weight heparin bind to the smaller site and do not confer the conformational change. The activated partial thromboplastin time (aPTT) is not useful for monitoring either because it measures factor IIa. increasing its antifactor Xa activity. a new anticoagulant.[No Authors Listed]: Fondaparinux (Arixtra). the inactivation of factor IIa requires an additional 16-unit saccharide chain to produce a conformational change. 2002 page 840 . REFERENCES 1. in Rutherford RB (ed): Vascular Surgery. 2005. WB Saunders. ed 6. bind to the heparin-binding site of antithrombin III. Med Lett Drugs Ther 44:43-44.American College of Surgeons SESAP 13 Critique for Category 8 Item 56 Low molecular weight heparin (LMWH) and fondaparinux. dosages should be adjusted in patients with renal failure. Although the incidence of heparin-induced thrombocytopenia appears to be lower with low molecular weight heparin than with unfractionated heparin. Because fondaparinux does not bind to platelet factor IV. However. Because both medications are cleared by the kidneys. American College of Surgeons SESAP 13 Category 8 Item 57 Mechanism of action via antithrombin III (A) Fondaparinux (B) Low molecular weight heparin (LMWH) (C) Both (D) Neither page 841 . [No Authors Listed]: Fondaparinux (Arixtra). The activated partial thromboplastin time (aPTT) is not useful for monitoring either because it measures factor IIa. Because both fondaparinux and low molecular weight heparin bind to the smaller site and do not confer the conformational change. 2005. However. the inactivation of factor IIa requires an additional 16-unit saccharide chain to produce a conformational change. Because fondaparinux does not bind to platelet factor IV. Neither drug is a direct thrombin inhibitor. it does not cause immune-mediated thrombocytopenias. dosages should be adjusted in patients with renal failure. Because both medications are cleared by the kidneys.Meissner MH: Antithrombotic therapy. bind to the heparin-binding site of antithrombin III. Although the incidence of heparin-induced thrombocytopenia appears to be lower with low molecular weight heparin than with unfractionated heparin. it has been reported.American College of Surgeons SESAP 13 Critique for Category 8 Item 57 Low molecular weight heparin (LMWH) and fondaparinux. a synthetic pentasaccharide. increasing its antifactor Xa activity. pp 511-529 2. WB Saunders. ed 6. Antithrombin III inactivates both factor Xa and factor IIa. a new anticoagulant. Med Lett Drugs Ther 44:43-44. they are ineffective against factor IIa. in Rutherford RB (ed): Vascular Surgery. 2002 page 842 . Philadelphia. REFERENCES 1. American College of Surgeons SESAP 13 Category 8 Item 58 Pulmonary capillary wedge pressure 25 (A) Abdominal compartment syndrome (B) Biventricular heart failure (C) Both (D) Neither page 843 . 2003. REFERENCES 1.Sosin MD. causing increased intrathoracic pressure. and decreasing urine output can mimic biventricular heart failure. in Irwin RS. Volume resuscitation can restore cardiac output and ameliorate some effects of abdominal compartment syndrome. Increased abdominal pressure decreases diaphragmatic excursion with inspiration. ed 5. These changes are myriad and affect many organ systems. The obvious difference in the two clinical syndromes is the presence of elevated intraabdominal pressure in abdominal compartment syndrome. WB Saunders.American College of Surgeons SESAP 13 Critique for Category 8 Item 58 Abdominal compartment syndrome is the cascade of physiologic changes that occurs in the presence of increased and abnormal pressure in the abdominal cavity. Abraham E. apparently elevated central pressures.Saggi BH. Kochanek P (eds): Textbook of Critical Care. Rippe JM (eds): Irwin and Rippe’s Intensive Care Medicine. it will further decrease preload and exacerbate cardiac and renal dysfunction in abdominal compartment syndrome. Cerebral perfusion pressure (intracranial pressure minus mean arterial pressure) is decreased because elevated intrathoracic pressure causes intracranial pressure to rise and mean arterial pressure to fall because of decreased cardiac output. Decreased cardiac output and increased intraabdominal pressure drop renal blood flow and urine output. and elevated airway pressures are the result of direct pulmonary compression (or decreased compliance). Ivatury RR. but may exacerbate increased abdominal pressure. Philadelphia. Other systems are also affected in abdominal compartment sydrome. 2005. This combination of decreased cardiac output. in Fink MP. Bloomfield GL: Abdominal compartment syndrome. While diuresis is a mainstay of therapy for biventricular cardiac failure. pp 813-821 3. hypercarbia. J Trauma 45:597-609. McCaffree DR: Pulmonary artery catheters. Hypoxia. ed 5. The increased visceral pressure decreases perfusion of intra-abdominal and retroperitoneal organs.Voyce SJ. Lippincott Wiliams & Wilkins. Cardiac output drops secondary to direct cardiac compression decreasing contractility. pp 45-66 page 844 . Lip GYH: Severe heart failure. The elevated intra-abdominal hypertension also directly reduces cortical blood flow and renal vein flow. Sugerman HJ. Philadelphia. Although preload is decreased. central perfusion pressures can be spuriously elevated because of increased intrathoracic pressure. 1998 2. leading to intestinal and hepatic ischemia as well as decreased urine output. Preload is decreased due to decreasing return of blood related to elevated intrathoracic pressure. Vincent JL. and systemic afterload increasing because of pressure on intra-abdominal organs. American College of Surgeons SESAP 13 Category 8 Item 59 Preload decreased (A) Abdominal compartment syndrome (B) Biventricular heart failure (C) Both (D) Neither page 845 . McCaffree DR: Pulmonary artery catheters. ed 5. The increased visceral pressure decreases perfusion of intra-abdominal and retroperitoneal organs. causing increased intrathoracic pressure. While diuresis is a mainstay of therapy for biventricular cardiac failure. apparently elevated central pressures. in Fink MP. Increased abdominal pressure decreases diaphragmatic excursion with inspiration. but may exacerbate increased abdominal pressure. Although preload is decreased. J Trauma 45:597-609. it will further decrease preload and exacerbate cardiac and renal dysfunction in abdominal compartment syndrome. Preload is decreased due to decreasing return of blood related to elevated intrathoracic pressure. Philadelphia. leading to intestinal and hepatic ischemia as well as decreased urine output. 1998 2. Other systems are also affected in abdominal compartment sydrome. Cardiac output drops secondary to direct cardiac compression decreasing contractility. REFERENCES 1. Philadelphia.Sosin MD. Kochanek P (eds): Textbook of Critical Care. Rippe JM (eds): Irwin and Rippe’s Intensive Care Medicine. pp 813-821 3. Lip GYH: Severe heart failure. pp 45-66 page 846 . Abraham E. Sugerman HJ. ed 5. and systemic afterload increasing because of pressure on intra-abdominal organs. Lippincott Wiliams & Wilkins.Voyce SJ. hypercarbia. and decreasing urine output can mimic biventricular heart failure. Ivatury RR. Cerebral perfusion pressure (intracranial pressure minus mean arterial pressure) is decreased because elevated intrathoracic pressure causes intracranial pressure to rise and mean arterial pressure to fall because of decreased cardiac output. WB Saunders. This combination of decreased cardiac output. Decreased cardiac output and increased intraabdominal pressure drop renal blood flow and urine output. and elevated airway pressures are the result of direct pulmonary compression (or decreased compliance).Saggi BH. The obvious difference in the two clinical syndromes is the presence of elevated intraabdominal pressure in abdominal compartment syndrome. Vincent JL. Hypoxia. central perfusion pressures can be spuriously elevated because of increased intrathoracic pressure. Bloomfield GL: Abdominal compartment syndrome. Volume resuscitation can restore cardiac output and ameliorate some effects of abdominal compartment syndrome. 2003. The elevated intra-abdominal hypertension also directly reduces cortical blood flow and renal vein flow. in Irwin RS. These changes are myriad and affect many organ systems. 2005.American College of Surgeons SESAP 13 Critique for Category 8 Item 59 Abdominal compartment syndrome is the cascade of physiologic changes that occurs in the presence of increased and abnormal pressure in the abdominal cavity. American College of Surgeons SESAP 13 Category 8 Item 60 Initial treatment includes diuresis (A) Abdominal compartment syndrome (B) Biventricular heart failure (C) Both (D) Neither page 847 . Vincent JL. Philadelphia. Bloomfield GL: Abdominal compartment syndrome. pp 45-66 page 848 . ed 5. leading to intestinal and hepatic ischemia as well as decreased urine output. and decreasing urine output can mimic biventricular heart failure. McCaffree DR: Pulmonary artery catheters. Other systems are also affected in abdominal compartment sydrome. and elevated airway pressures are the result of direct pulmonary compression (or decreased compliance). pp 813-821 3. Abraham E. hypercarbia. While diuresis is a mainstay of therapy for biventricular cardiac failure. Although preload is decreased. Rippe JM (eds): Irwin and Rippe’s Intensive Care Medicine. Hypoxia. but may exacerbate increased abdominal pressure. ed 5. Lip GYH: Severe heart failure. Philadelphia. 1998 2. 2003. Preload is decreased due to decreasing return of blood related to elevated intrathoracic pressure. REFERENCES 1. it will further decrease preload and exacerbate cardiac and renal dysfunction in abdominal compartment syndrome.Voyce SJ. Lippincott Wiliams & Wilkins.Saggi BH. and systemic afterload increasing because of pressure on intra-abdominal organs. These changes are myriad and affect many organ systems. WB Saunders. 2005. Volume resuscitation can restore cardiac output and ameliorate some effects of abdominal compartment syndrome. causing increased intrathoracic pressure. Increased abdominal pressure decreases diaphragmatic excursion with inspiration. Decreased cardiac output and increased intraabdominal pressure drop renal blood flow and urine output.Sosin MD. in Fink MP.American College of Surgeons SESAP 13 Critique for Category 8 Item 60 Abdominal compartment syndrome is the cascade of physiologic changes that occurs in the presence of increased and abnormal pressure in the abdominal cavity. The obvious difference in the two clinical syndromes is the presence of elevated intraabdominal pressure in abdominal compartment syndrome. The elevated intra-abdominal hypertension also directly reduces cortical blood flow and renal vein flow. central perfusion pressures can be spuriously elevated because of increased intrathoracic pressure. The increased visceral pressure decreases perfusion of intra-abdominal and retroperitoneal organs. Cardiac output drops secondary to direct cardiac compression decreasing contractility. apparently elevated central pressures. Ivatury RR. in Irwin RS. Cerebral perfusion pressure (intracranial pressure minus mean arterial pressure) is decreased because elevated intrathoracic pressure causes intracranial pressure to rise and mean arterial pressure to fall because of decreased cardiac output. This combination of decreased cardiac output. Kochanek P (eds): Textbook of Critical Care. Sugerman HJ. J Trauma 45:597-609. American College of Surgeons SESAP 13 Category 8 Item 61 Associated with improved mortality in pediatric acute respiratory distress syndrome (ARDS) (A) Inhaled nitric oxide (B) Intratracheal exogenous surfactant (C) Both (D) Neither page 849 . American College of Surgeons SESAP 13 Critique for Category 8 Item 61 Acute lung injury (ALI. short-term physiologic improvements may be crucial for patient survival in severe cases for whom refractory hypoxemia or pulmonary hypertension are major clinical problems. In contrast. Bohn D: Inhaled nitric oxide for acute hypoxic respiratory failure in children and adults: A meta-analysis. 2003 3. et al: Low-dose inhaled nitric oxide in patients with acute lung injury: a randomized controlled trial. Although current cumulative evidence from clinical trials suggests that nitric oxide has no place in the routine therapy of patients with ALI or ARDS. Markovitz BP. Inhaled nitric oxide improves oxygenation in ALI and ARDS.Sokol J. Jacobs Se. although duration of ventilator therapy and length of ICU or hospital stay were not significantly reduced. JAMA 293:470-476. Bauman LA. exogenous surfactant therapy is standard of care in neonates with ARDS. Despite evidence that patients with ALI and ARDS have pulmonary surfactant dysfunction. In these limited situations. Caldwell E. 2004 4. resulting in improved right ventricular function. children. as a component of a multimodal approach that includes other strategies such as high-frequency oscillation and prone positioning. REFERENCES 1. Straube RC. 2004 5. 2005 2. The incidence of ALI is 79/100. et al: Incidence and outcomes of acute lung injury. N Engl J Med 353:1685-1693. Early surfactant administration in neonates at risk for or presenting with ARDS is associated with significantly improved outcomes. et al: Effect of exogenous surfactant (calfactant) in pediatric acute lung injury: a randomized controlled trial. Anesth Analg 97:989-998. Peabody E. and adolescents documented that calfactant acutely improved oxygenation and significantly decreased mortality. nitric oxide may have a role as salvage therapy. Zimmerman JL. defined as PaO2/FiO2 ratio < 300) and the acute respiratory distress syndrome (ARDS. 2005 page 850 . Thomas NJ.5%.Taylor RW. A recent study with a natural lung surfactant (calfactant) containing high levels of surfactant-specific protein B in 153 infants.Rubenfeld GD. Dellinger RP.000 person-years with an in-hospital mortality rate of 38. but has no effect on the duration of ventilatory support or mortality. Inhaled nitric oxide is a selective pulmonary vasodilator that acts on the endothelial surface of the lung to produce regional vasodilaton in ventilated lung units. Inhaled nitric oxide and intratracheal administration of exogenous surfactant have been used in patients with ALI and ARDS. defined as PaO2/FiO2 ratio < 200) are critical illness syndromes consisting of acute hypoxemic respiratory failure with bilateral pulmonary infiltrates that are not attributed to left atrial hypertension. Soll RF: Early surfactant administration with brief ventilation vs selective surfactant and continued mechanical ventilation for preterm infants with or at risk for respiratory distress syndrome.Stevens TP. JAMA 291:1603-1609. several exogenous surfactant preparations have not been successful in adult clinical trials. Blennow M. Weaver J. Cochrane Database Syst Rev (3):CD003063.Willson DF. American College of Surgeons SESAP 13 Category 8 Item 62 Significant increase in PaO2 in adult ARDS (A) Inhaled nitric oxide (B) Intratracheal exogenous surfactant (C) Both (D) Neither page 851 . 2005 page 852 . nitric oxide may have a role as salvage therapy. 2005 2. Peabody E. Inhaled nitric oxide improves oxygenation in ALI and ARDS. Jacobs Se.Stevens TP. Cochrane Database Syst Rev (3):CD003063.000 person-years with an in-hospital mortality rate of 38. Weaver J. Markovitz BP. short-term physiologic improvements may be crucial for patient survival in severe cases for whom refractory hypoxemia or pulmonary hypertension are major clinical problems. et al: Effect of exogenous surfactant (calfactant) in pediatric acute lung injury: a randomized controlled trial. et al: Incidence and outcomes of acute lung injury. as a component of a multimodal approach that includes other strategies such as high-frequency oscillation and prone positioning. et al: Low-dose inhaled nitric oxide in patients with acute lung injury: a randomized controlled trial. Inhaled nitric oxide and intratracheal administration of exogenous surfactant have been used in patients with ALI and ARDS. exogenous surfactant therapy is standard of care in neonates with ARDS. Anesth Analg 97:989-998. Thomas NJ. 2003 3. 2004 5. resulting in improved right ventricular function. Straube RC.Rubenfeld GD. N Engl J Med 353:1685-1693. Soll RF: Early surfactant administration with brief ventilation vs selective surfactant and continued mechanical ventilation for preterm infants with or at risk for respiratory distress syndrome. JAMA 291:1603-1609. but has no effect on the duration of ventilatory support or mortality. Dellinger RP. Blennow M. Zimmerman JL. In contrast.5%. In these limited situations. and adolescents documented that calfactant acutely improved oxygenation and significantly decreased mortality.Willson DF. The incidence of ALI is 79/100. A recent study with a natural lung surfactant (calfactant) containing high levels of surfactant-specific protein B in 153 infants. although duration of ventilator therapy and length of ICU or hospital stay were not significantly reduced. several exogenous surfactant preparations have not been successful in adult clinical trials. defined as PaO2/FiO2 ratio < 200) are critical illness syndromes consisting of acute hypoxemic respiratory failure with bilateral pulmonary infiltrates that are not attributed to left atrial hypertension. Although current cumulative evidence from clinical trials suggests that nitric oxide has no place in the routine therapy of patients with ALI or ARDS. Inhaled nitric oxide is a selective pulmonary vasodilator that acts on the endothelial surface of the lung to produce regional vasodilaton in ventilated lung units.Taylor RW. defined as PaO2/FiO2 ratio < 300) and the acute respiratory distress syndrome (ARDS. Bauman LA.Sokol J. JAMA 293:470-476. Bohn D: Inhaled nitric oxide for acute hypoxic respiratory failure in children and adults: A meta-analysis. Caldwell E. children. Despite evidence that patients with ALI and ARDS have pulmonary surfactant dysfunction. Early surfactant administration in neonates at risk for or presenting with ARDS is associated with significantly improved outcomes.American College of Surgeons SESAP 13 Critique for Category 8 Item 62 Acute lung injury (ALI. REFERENCES 1. 2004 4. American College of Surgeons SESAP 13 Category 8 Item 63 No effect on mortality in adult ARDS (A) Inhaled nitric oxide (B) Intratracheal exogenous surfactant (C) Both (D) Neither page 853 . Bohn D: Inhaled nitric oxide for acute hypoxic respiratory failure in children and adults: A meta-analysis. JAMA 293:470-476. et al: Incidence and outcomes of acute lung injury. et al: Effect of exogenous surfactant (calfactant) in pediatric acute lung injury: a randomized controlled trial. children. 2004 4. Thomas NJ. Markovitz BP. Jacobs Se. Inhaled nitric oxide improves oxygenation in ALI and ARDS. Peabody E. but has no effect on the duration of ventilatory support or mortality.Stevens TP. exogenous surfactant therapy is standard of care in neonates with ARDS. N Engl J Med 353:1685-1693.000 person-years with an in-hospital mortality rate of 38. Although current cumulative evidence from clinical trials suggests that nitric oxide has no place in the routine therapy of patients with ALI or ARDS. although duration of ventilator therapy and length of ICU or hospital stay were not significantly reduced.5%. Caldwell E. 2005 page 854 . REFERENCES 1. A recent study with a natural lung surfactant (calfactant) containing high levels of surfactant-specific protein B in 153 infants. nitric oxide may have a role as salvage therapy. et al: Low-dose inhaled nitric oxide in patients with acute lung injury: a randomized controlled trial. JAMA 291:1603-1609. Inhaled nitric oxide and intratracheal administration of exogenous surfactant have been used in patients with ALI and ARDS. Anesth Analg 97:989-998. 2004 5.Willson DF. Cochrane Database Syst Rev (3):CD003063. short-term physiologic improvements may be crucial for patient survival in severe cases for whom refractory hypoxemia or pulmonary hypertension are major clinical problems. 2003 3. resulting in improved right ventricular function. Dellinger RP. several exogenous surfactant preparations have not been successful in adult clinical trials. Soll RF: Early surfactant administration with brief ventilation vs selective surfactant and continued mechanical ventilation for preterm infants with or at risk for respiratory distress syndrome.Rubenfeld GD. Inhaled nitric oxide is a selective pulmonary vasodilator that acts on the endothelial surface of the lung to produce regional vasodilaton in ventilated lung units. and adolescents documented that calfactant acutely improved oxygenation and significantly decreased mortality. 2005 2.Sokol J. defined as PaO2/FiO2 ratio < 200) are critical illness syndromes consisting of acute hypoxemic respiratory failure with bilateral pulmonary infiltrates that are not attributed to left atrial hypertension. Blennow M. Zimmerman JL. Early surfactant administration in neonates at risk for or presenting with ARDS is associated with significantly improved outcomes. as a component of a multimodal approach that includes other strategies such as high-frequency oscillation and prone positioning.Taylor RW. In contrast. The incidence of ALI is 79/100. Bauman LA. In these limited situations. Straube RC. defined as PaO2/FiO2 ratio < 300) and the acute respiratory distress syndrome (ARDS. Weaver J. Despite evidence that patients with ALI and ARDS have pulmonary surfactant dysfunction.American College of Surgeons SESAP 13 Critique of Category 8 Item 63 Acute lung injury (ALI. American College of Surgeons SESAP 13 Category 8 Item 64 Inhibit(s) cell wall synthesis (A) Liposomal amphotericin B (B) Caspofungin (C) Both (D) Neither page 855 . a glucan synthesis inhibitor.Walsh TJ. Am J Health Syst Pharm 59:33-41. Caspofungin. thus interfering with the synthesis of the fungal cell wall. which results in an increase in membrane permeability. Jarvis B: Caspofungin. Maertens JA. N Engl J Med 351:1391-1402. 2001 3. Drugs 61:1121-1129. It is a semisynthetic echinocandin that inhibits the synthesis of B-(1. Both liposomal amphotericin B and caspofungin are effective as empiric antifungal therapy for patients with persistent fever and associated neutropenia. is the first in a new class of antifungal agents that interfere with fungal cell wall synthesis. Donowitz GR.Kam LW. Conventional amphotericin B binds to ergosterol on the fungal cell wall. 3)D-glucan.Keating GM. REFERENCES 1. and subsequent cell death. 2004 page 856 . et al: Caspofungin versus liposomal amphotericin B for empirical antifungal therapy in patients with persistent fever and neutropenia. Teppler H. Lin JD: Management of systemic candidal infections in the intensive care unit. leakage of cellular components. 2002 2. The lipid-based formulations of amphotericin B were developed to decrease nephrotoxicity and enhance drug delivery.American College of Surgeons SESAP 13 Critique for Category 8 Item 64 Antimicrobial therapeutic options in the management of systemic fungal infections have been limited. American College of Surgeons SESAP 13 Category 8 Item 65 Bind(s) to ergosterol on the cell wall and increase(s) membrane permeability (A) Liposomal amphotericin B (B) Caspofungin (C) Both (D) Neither page 857 . Walsh TJ. It is a semisynthetic echinocandin that inhibits the synthesis of B-(1. et al: Caspofungin versus liposomal amphotericin B for empirical antifungal therapy in patients with persistent fever and neutropenia. Am J Health Syst Pharm 59:33-41. Teppler H. and subsequent cell death. 2002 2. 2001 3. The lipid-based formulations of amphotericin B were developed to decrease nephrotoxicity and enhance drug delivery. 2004 page 858 . Jarvis B: Caspofungin.Kam LW. a glucan synthesis inhibitor. 3)D-glucan.American College of Surgeons SESAP 13 Critique for Category 8 Item 65 Antimicrobial therapeutic options in the management of systemic fungal infections have been limited.Keating GM. Conventional amphotericin B binds to ergosterol on the fungal cell wall. Lin JD: Management of systemic candidal infections in the intensive care unit. REFERENCES 1. Both liposomal amphotericin B and caspofungin are effective as empiric antifungal therapy for patients with persistent fever and associated neutropenia. Maertens JA. Drugs 61:1121-1129. thus interfering with the synthesis of the fungal cell wall. which results in an increase in membrane permeability. N Engl J Med 351:1391-1402. leakage of cellular components. is the first in a new class of antifungal agents that interfere with fungal cell wall synthesis. Donowitz GR. Caspofungin. American College of Surgeons SESAP 13 Category 8 Item 66 Effective as an empirical antifungal therapy in patients with persistent fever and neutropenia (A) Liposomal amphotericin B (B) Caspofungin (C) Both (D) Neither page 859 . leakage of cellular components. 2004 page 860 . Teppler H. Both liposomal amphotericin B and caspofungin are effective as empiric antifungal therapy for patients with persistent fever and associated neutropenia.Walsh TJ. a glucan synthesis inhibitor.Keating GM.Kam LW. thus interfering with the synthesis of the fungal cell wall. 2002 2. Maertens JA. which results in an increase in membrane permeability.American College of Surgeons SESAP 13 Critique for Category 8 Item 66 Antimicrobial therapeutic options in the management of systemic fungal infections have been limited. It is a semisynthetic echinocandin that inhibits the synthesis of B-(1. is the first in a new class of antifungal agents that interfere with fungal cell wall synthesis. Jarvis B: Caspofungin. Lin JD: Management of systemic candidal infections in the intensive care unit. Caspofungin. Drugs 61:1121-1129. 2001 3. N Engl J Med 351:1391-1402. Conventional amphotericin B binds to ergosterol on the fungal cell wall. 3)D-glucan. Donowitz GR. et al: Caspofungin versus liposomal amphotericin B for empirical antifungal therapy in patients with persistent fever and neutropenia. The lipid-based formulations of amphotericin B were developed to decrease nephrotoxicity and enhance drug delivery. and subsequent cell death. REFERENCES 1. Am J Health Syst Pharm 59:33-41. American College of Surgeons SESAP 13 Category 8 Item 67 Preoperative use decreases morbidity in severely malnourished patients (A) Enteral nutrition (B) Parenteral nutrition (C) Both (D) Neither page 861 . nor with preoperative supplementation for < 5 days. Mortality declines from 12% to 3% to 6%.McClave SA. The easiest method for assessing nutritional risk defines severe malnutrition as an albumin < 2. The recommended length of preoperative enteral supplementation is at least 10 days. In severely malnourished patients. In unselected patients. preoperative parenteral nutrition decreases morbidity by approximately 10%.Howard L. No mortality benefit is seen with preoperative parenteral nutrition.5 mg/dL or unintentional weight loss > 20%. particularly those with pancreatitis.Marik PE. nutritional supplementation is of little benefit. and parenteral supplementation actually increases infectious complications. or preservation of gutassociated lymphoid tissue (GALT) with a resultant decrease in bacterial translocation compared with parenteral nutrition. 1991 page 862 . Kichian K. Among patients with acute pancreatitis who have been adequately resuscitated.Gramlich L. Nutrition 20:843-848. enteral nutrition decreases overall infectious complications. The optimal time for preoperative supplementation is unknown. This benefit is not seen in patients with minimal or moderate malnutrition. Spain DA: Preoperative issues in clinical nutrition. Some patients may tolerate gastric feeding. jejunal feeding may be required. Snider HL.American College of Surgeons SESAP 13 Critique for Category 8 Item 67 Nutritional supplementation can be provided enterally or parenterally. N Engl J Med 325:525-532. 2003 3. Achieving caloric goals may be more difficult with the enteral route. number of surgical interventions required. Chest 115:64S-70S. Ashley C: Nutrition in the perioperative patient. but the benefits outweigh the risk. Decreased infections and decreased cost make enteral rather than parenteral nutrition the route of choice when possible. 2004 2. This may be due to fewer episodes of hyperglycemia seen in patients with enteral nutrition. REFERENCES 1. 2004 4. et al: Does enteral nutrition compared to parenteral nutrition result in better outcomes in critically ill adult patients? A systematic review of the literature. but at least 7 days is recommended. Zaloga GP: Meta-analysis of parenteral nutrition versus enteral nutrition in patients with acute pancreatitis. In critically ill patients.or postoperatively. in others. Overall mortality is equivalent for both methods of nutritional supplementation in critically ill patients. Annu Rev Nutr 23:263-282. The Veterans Affairs Total Parenteral Nutrition Cooperative Study Group. enteral nutrition via a jejunal tube decreases infectious complications. and in elective cases supplied pre. Pinilla J. Parenteral nutrition remains associated with increased infectious complications in this population. because many critically ill or postoperative patients have compromised tolerance. and morbidity is reduced by 20% to 30%. underlining the importance of nutritional risk assessment in patients undergoing elective operations. BMJ 328:1407. Rodych NJ. and length of hospital stay. whereas preoperative enteral nutrition can provide both a morbidity and mortality benefit in patients with severe malnutrition.[No Authors Listed]: Perioperative total parenteral nutrition in surgical patients. 1999 5. American College of Surgeons SESAP 13 Category 8 Item 68 Decrease(s) infectious complications in critically ill patients (A) Enteral nutrition (B) Parenteral nutrition (C) Both (D) Neither page 863 . In severely malnourished patients. 1991 page 864 . Mortality declines from 12% to 3% to 6%. Achieving caloric goals may be more difficult with the enteral route. Zaloga GP: Meta-analysis of parenteral nutrition versus enteral nutrition in patients with acute pancreatitis. et al: Does enteral nutrition compared to parenteral nutrition result in better outcomes in critically ill adult patients? A systematic review of the literature.Marik PE. preoperative parenteral nutrition decreases morbidity by approximately 10%.or postoperatively. enteral nutrition decreases overall infectious complications. This benefit is not seen in patients with minimal or moderate malnutrition.McClave SA. Parenteral nutrition remains associated with increased infectious complications in this population. particularly those with pancreatitis. No mortality benefit is seen with preoperative parenteral nutrition. Some patients may tolerate gastric feeding. In unselected patients. whereas preoperative enteral nutrition can provide both a morbidity and mortality benefit in patients with severe malnutrition. REFERENCES 1. because many critically ill or postoperative patients have compromised tolerance. BMJ 328:1407. nor with preoperative supplementation for < 5 days. but at least 7 days is recommended. and length of hospital stay. Snider HL. number of surgical interventions required. The optimal time for preoperative supplementation is unknown. The recommended length of preoperative enteral supplementation is at least 10 days. Among patients with acute pancreatitis who have been adequately resuscitated. N Engl J Med 325:525-532. underlining the importance of nutritional risk assessment in patients undergoing elective operations. Decreased infections and decreased cost make enteral rather than parenteral nutrition the route of choice when possible. in others. and in elective cases supplied pre. 2003 3. enteral nutrition via a jejunal tube decreases infectious complications. 2004 4. nutritional supplementation is of little benefit.[No Authors Listed]: Perioperative total parenteral nutrition in surgical patients. Spain DA: Preoperative issues in clinical nutrition. 2004 2. jejunal feeding may be required. Annu Rev Nutr 23:263-282. but the benefits outweigh the risk. Overall mortality is equivalent for both methods of nutritional supplementation in critically ill patients. or preservation of gutassociated lymphoid tissue (GALT) with a resultant decrease in bacterial translocation compared with parenteral nutrition. Ashley C: Nutrition in the perioperative patient.Howard L. The easiest method for assessing nutritional risk defines severe malnutrition as an albumin < 2. Nutrition 20:843-848. This may be due to fewer episodes of hyperglycemia seen in patients with enteral nutrition. In critically ill patients. Chest 115:64S-70S. The Veterans Affairs Total Parenteral Nutrition Cooperative Study Group. and parenteral supplementation actually increases infectious complications. Pinilla J. Rodych NJ. and morbidity is reduced by 20% to 30%. Kichian K. 1999 5.Gramlich L.American College of Surgeons SESAP 13 Critique for Category 8 Item 68 Nutritional supplementation can be provided enterally or parenterally.5 mg/dL or unintentional weight loss > 20%. American College of Surgeons SESAP 13 Category 8 Item 69 Routine postoperative use after major cancer resections decreases mortality (A) Enteral nutrition (B) Parenteral nutrition (C) Both (D) Neither page 865 . nor with preoperative supplementation for < 5 days. jejunal feeding may be required.Howard L. Nutrition 20:843-848. and in elective cases supplied pre. 1999 5. This may be due to fewer episodes of hyperglycemia seen in patients with enteral nutrition. In unselected patients. The optimal time for preoperative supplementation is unknown. No mortality benefit is seen with preoperative parenteral nutrition. Among patients with acute pancreatitis who have been adequately resuscitated. Overall mortality is equivalent for both methods of nutritional supplementation in critically ill patients. Pinilla J. whereas preoperative enteral nutrition can provide both a morbidity and mortality benefit in patients with severe malnutrition. et al: Does enteral nutrition compared to parenteral nutrition result in better outcomes in critically ill adult patients? A systematic review of the literature. 1991 page 866 . This benefit is not seen in patients with minimal or moderate malnutrition. Achieving caloric goals may be more difficult with the enteral route. particularly those with pancreatitis.or postoperatively. underlining the importance of nutritional risk assessment in patients undergoing elective operations. Decreased infections and decreased cost make enteral rather than parenteral nutrition the route of choice when possible. number of surgical interventions required. but at least 7 days is recommended. Spain DA: Preoperative issues in clinical nutrition.American College of Surgeons SESAP 13 Critique for Category 8 Item 69 Nutritional supplementation can be provided enterally or parenterally.[No Authors Listed]: Perioperative total parenteral nutrition in surgical patients. because many critically ill or postoperative patients have compromised tolerance. preoperative parenteral nutrition decreases morbidity by approximately 10%. nutritional supplementation is of little benefit.Gramlich L. enteral nutrition via a jejunal tube decreases infectious complications. Some patients may tolerate gastric feeding. Ashley C: Nutrition in the perioperative patient. and length of hospital stay. 2003 3. Chest 115:64S-70S. In severely malnourished patients. The Veterans Affairs Total Parenteral Nutrition Cooperative Study Group. N Engl J Med 325:525-532. Mortality declines from 12% to 3% to 6%. Annu Rev Nutr 23:263-282. The easiest method for assessing nutritional risk defines severe malnutrition as an albumin < 2. REFERENCES 1. enteral nutrition decreases overall infectious complications. BMJ 328:1407. In critically ill patients. The recommended length of preoperative enteral supplementation is at least 10 days. Kichian K.Marik PE. 2004 2. or preservation of gutassociated lymphoid tissue (GALT) with a resultant decrease in bacterial translocation compared with parenteral nutrition. Rodych NJ. and morbidity is reduced by 20% to 30%. 2004 4. but the benefits outweigh the risk.McClave SA. in others.5 mg/dL or unintentional weight loss > 20%. Parenteral nutrition remains associated with increased infectious complications in this population. and parenteral supplementation actually increases infectious complications. Zaloga GP: Meta-analysis of parenteral nutrition versus enteral nutrition in patients with acute pancreatitis. Snider HL. American College of Surgeons SESAP 13 Category 8 Item 70 Positive end-expiratory pressure (PEEP) (A) Primarily affects PaO2 (B) Primarily affects PaCO2 (C) Both (D) Neither page 867 . in Souba WW. PaO2 is altered by manipulating FiO2. Changing the I/E ratio allows inspired oxygen to remain in contact with the alveoli for a longer period. New York. Fink MP. et al (eds): ACS Surgery: Principles and Practice 2006. With more gas exchange units.American College of Surgeons SESAP 13 Critique for Category 8 Item 70 At its most basic. Nathens AB: Mechanical ventilation.Sena MJ. REFERENCES 1. chap 8. pp 1390-1407 page 868 . Kaiser LR. sec 5. Critical Care. Increasing PEEP will lead to a recruitment of alveoli. positive end-expiratory pressure (PEEP). WebMD Inc. PaCO2 will not be affected by this change. Increasing the FiO2 should increase the PaO2 and have no effect on the PaCO2. Ve is a product of respiratory rate and tidal volume. Jurkovich GJ. Arterial oxygenation is time dependent and increasing this time will increase the amount of oxygen in the bloodstream. the arterial oxygenation level will increase. Manipulation of either the tidal volume or the respiratory rate on the ventilator will lead to a change in the PaCO2. or the inspiratory to expiratory (I/E) ratio. PaCO2 is affected by the minute ventilation (Ve). the ventilator controls PaCO2 and PaO2. This also should have minimal effect on the PaCO2. Neither of these changes should have an effect on the PaO2. American College of Surgeons SESAP 13 Category 8 Item 71 Tidal volume (A) Primarily affects PaO2 (B) Primarily affects PaCO2 (C) Both (D) Neither page 869 . PaO2 is altered by manipulating FiO2. New York. With more gas exchange units. This also should have minimal effect on the PaCO2. REFERENCES 1. WebMD Inc. PaCO2 is affected by the minute ventilation (Ve). PaCO2 will not be affected by this change. Arterial oxygenation is time dependent and increasing this time will increase the amount of oxygen in the bloodstream. Ve is a product of respiratory rate and tidal volume. Neither of these changes should have an effect on the PaO2. et al (eds): ACS Surgery: Principles and Practice 2006. Jurkovich GJ. Critical Care. the ventilator controls PaCO2 and PaO2.Sena MJ. sec 5. Nathens AB: Mechanical ventilation. in Souba WW. pp 1390-1407 page 870 . positive end-expiratory pressure (PEEP). Increasing PEEP will lead to a recruitment of alveoli. Increasing the FiO2 should increase the PaO2 and have no effect on the PaCO2. Manipulation of either the tidal volume or the respiratory rate on the ventilator will lead to a change in the PaCO2.American College of Surgeons SESAP 13 Critique for Category 8 Item 71 At its most basic. the arterial oxygenation level will increase. chap 8. Kaiser LR. Fink MP. Changing the I/E ratio allows inspired oxygen to remain in contact with the alveoli for a longer period. or the inspiratory to expiratory (I/E) ratio. American College of Surgeons SESAP 13 Category 8 Item 72 Respiratory rate (A) Primarily affects PaO2 (B) Primarily affects PaCO2 (C) Both (D) Neither page 871 . WebMD Inc. Fink MP. in Souba WW. Neither of these changes should have an effect on the PaO2. PaO2 is altered by manipulating FiO2. This also should have minimal effect on the PaCO2. Arterial oxygenation is time dependent and increasing this time will increase the amount of oxygen in the bloodstream.Sena MJ. New York. chap 8. positive end-expiratory pressure (PEEP).American College of Surgeons SESAP 13 Critique for Category 8 Item 72 At its most basic. PaCO2 will not be affected by this change. Kaiser LR. Ve is a product of respiratory rate and tidal volume. REFERENCES 1. et al (eds): ACS Surgery: Principles and Practice 2006. the arterial oxygenation level will increase. Changing the I/E ratio allows inspired oxygen to remain in contact with the alveoli for a longer period. With more gas exchange units. or the inspiratory to expiratory (I/E) ratio. Jurkovich GJ. Nathens AB: Mechanical ventilation. Increasing PEEP will lead to a recruitment of alveoli. the ventilator controls PaCO2 and PaO2. Manipulation of either the tidal volume or the respiratory rate on the ventilator will lead to a change in the PaCO2. Critical Care. pp 1390-1407 page 872 . PaCO2 is affected by the minute ventilation (Ve). sec 5. Increasing the FiO2 should increase the PaO2 and have no effect on the PaCO2. American College of Surgeons SESAP 13 Category 8 Item 73 Hyponatremia (A) Cerebral salt wasting (B) Syndrome of inappropriate antidiuretic hormone secretion (SIADH) (C) Both (D) Neither page 873 . The potassium level is normal. or low serum osmolality. Gottfried ON. brain stem herniation.American College of Surgeons SESAP 13 Critique for Category 8 Item 73 Hyponatremia is a common electrolyte abnormality seen after traumatic brain injury and/or neurosurgical procedures. Regardless of the exact etiology. inappropriately concentrated urine compared with serum osmolality. Severe or rapidly evolving hyponatremia can lead to seizures.Cole CD. Hyponatremia may be associated with high. The primary treatment of CSW is sodium and volume replacement. Couldwell WT: Hyponatremia in the neurosurgical patient: diagnosis and management. the kidneys will correct this “water overload. These patients are hyponatremic. thyroid. or renal dysfunction. In cases of SIADH. muscle cramps. respiratory arrest. disorientation. and the absence of peripheral edema or dehydration with no evidence of adrenal. The process likely involves disruption of neural input into the kidney and/or the release of some circulating natriuretic factor. volume contracted. the primary problem is increased renal losses of sodium. while their BUN is decreased. 2000 2. and depressed reflexes. Distinguishing between these two conditions may be difficult in some cases because the clinical presentations are similar. Primary treatment is water restriction. REFERENCES 1. both cerebral salt wasting (CSW) and the syndrome of inappropriate antidiuretic hormone secretion (SIADH) are potential causes of hyponatremia. 2003 page 874 .” Among patients with recent trauma or neurosurgical procedure. Hypoantremia is defined as a serum sodium concentration < 136 mmol/L. Neurologic injury may stimulate the sympathetic release of BNP from the heart. N Engl J Med 342:1581-1589. The diagnosis of SIADH is based on low serum sodium. The pathophysiology of CSW is not well characterized. and death.Adrogue HJ. such as brain natriuretic protein (BNP). With appropriate volume resuscitation. lethargy. The physiologic stimuli for secretion of ADH are hyperosmolality or volume contraction. These patients usually have a normal volume status as well as hematocrit. Most patients who develop hyponatremia are inadequately resuscitated and have initiated antidiuretic hormone (ADH) as a mechanism of water retention and secondary volume expansion. excessive secretion of ADH occurs as a result of injury and persists despite low serum osmolality. restlessness. nausea. The most common symptoms of hyponatremia are headache. vomiting. Because the seriousness of hyponatremia depends on the rate of change as well as the degree of hyponatremia. permanent brain damage. high urinary sodium level. 2004 3. but most commonly it is hypoosmolar and is due to water retention rather than active salt loss. coma.Palmer BF: Hyponatremia in patients with central nervous system disease: SIADH versus CSW. Liu JK. although most patients with a serum sodium > 125 mmol/L will be asymptomatic unless the change in serum sodium has been precipitous. BUN and hematocrit are typically elevated while potassium levels are normal or increased. Trends Endocrinol Metab 14:182-187. and have increased urine sodium losses. normal. The primary differentiation is the patient’s volume status. complications can also arise if correction is too rapid. Neurosurg Focus 16:E9. Madias NE: Hyponatremia. Some studies have suggested the brain may directly release BNP in response to injury as a possible mechanism to limit intracranial pressure increases. American College of Surgeons SESAP 13 Category 8 Item 74 Increased extracellular fluid volume (A) Cerebral salt wasting (B) Syndrome of inappropriate antidiuretic hormone secretion (SIADH) (C) Both (D) Neither page 875 . Distinguishing between these two conditions may be difficult in some cases because the clinical presentations are similar.Palmer BF: Hyponatremia in patients with central nervous system disease: SIADH versus CSW. The physiologic stimuli for secretion of ADH are hyperosmolality or volume contraction. Hyponatremia may be associated with high. restlessness. and the absence of peripheral edema or dehydration with no evidence of adrenal. Madias NE: Hyponatremia. the kidneys will correct this “water overload. The pathophysiology of CSW is not well characterized.” Among patients with recent trauma or neurosurgical procedure. The primary treatment of CSW is sodium and volume replacement. brain stem herniation. REFERENCES 1. Couldwell WT: Hyponatremia in the neurosurgical patient: diagnosis and management. In cases of SIADH. normal. muscle cramps. Neurosurg Focus 16:E9. such as brain natriuretic protein (BNP).Adrogue HJ. N Engl J Med 342:1581-1589. the primary problem is increased renal losses of sodium. With appropriate volume resuscitation. although most patients with a serum sodium > 125 mmol/L will be asymptomatic unless the change in serum sodium has been precipitous. and have increased urine sodium losses. vomiting. or renal dysfunction. Gottfried ON. disorientation. coma. Neurologic injury may stimulate the sympathetic release of BNP from the heart.Cole CD. and depressed reflexes. while their BUN is decreased. BUN and hematocrit are typically elevated while potassium levels are normal or increased. or low serum osmolality. Primary treatment is water restriction. These patients usually have a normal volume status as well as hematocrit. Regardless of the exact etiology. thyroid. respiratory arrest. Most patients who develop hyponatremia are inadequately resuscitated and have initiated antidiuretic hormone (ADH) as a mechanism of water retention and secondary volume expansion. nausea. permanent brain damage. 2000 2. complications can also arise if correction is too rapid. The diagnosis of SIADH is based on low serum sodium. These patients are hyponatremic. 2004 3. Liu JK. excessive secretion of ADH occurs as a result of injury and persists despite low serum osmolality. Trends Endocrinol Metab 14:182-187. Some studies have suggested the brain may directly release BNP in response to injury as a possible mechanism to limit intracranial pressure increases. but most commonly it is hypoosmolar and is due to water retention rather than active salt loss. and death. The most common symptoms of hyponatremia are headache. both cerebral salt wasting (CSW) and the syndrome of inappropriate antidiuretic hormone secretion (SIADH) are potential causes of hyponatremia. The process likely involves disruption of neural input into the kidney and/or the release of some circulating natriuretic factor. 2003 page 876 . Severe or rapidly evolving hyponatremia can lead to seizures. The potassium level is normal. The primary differentiation is the patient’s volume status. Hypoantremia is defined as a serum sodium concentration < 136 mmol/L. high urinary sodium level. inappropriately concentrated urine compared with serum osmolality. volume contracted.American College of Surgeons SESAP 13 Critique for Category 8 Item 74 Hyponatremia is a common electrolyte abnormality seen after traumatic brain injury and/or neurosurgical procedures. lethargy. Because the seriousness of hyponatremia depends on the rate of change as well as the degree of hyponatremia. American College of Surgeons SESAP 13 Category 8 Item 75 Increased BUN (A) Cerebral salt wasting (B) Syndrome of inappropriate antidiuretic hormone secretion (SIADH) (C) Both (D) Neither page 877 . permanent brain damage. but most commonly it is hypoosmolar and is due to water retention rather than active salt loss. Gottfried ON. muscle cramps. N Engl J Med 342:1581-1589. 2004 3. nausea. Because the seriousness of hyponatremia depends on the rate of change as well as the degree of hyponatremia. the kidneys will correct this “water overload.Adrogue HJ. Hyponatremia may be associated with high. Hypoantremia is defined as a serum sodium concentration < 136 mmol/L. The physiologic stimuli for secretion of ADH are hyperosmolality or volume contraction. and depressed reflexes. Severe or rapidly evolving hyponatremia can lead to seizures. and have increased urine sodium losses.American College of Surgeons SESAP 13 Critique for Category 8 Item 75 Hyponatremia is a common electrolyte abnormality seen after traumatic brain injury and/or neurosurgical procedures. lethargy. excessive secretion of ADH occurs as a result of injury and persists despite low serum osmolality. 2000 2. both cerebral salt wasting (CSW) and the syndrome of inappropriate antidiuretic hormone secretion (SIADH) are potential causes of hyponatremia. The primary treatment of CSW is sodium and volume replacement. The process likely involves disruption of neural input into the kidney and/or the release of some circulating natriuretic factor. Neurosurg Focus 16:E9. Distinguishing between these two conditions may be difficult in some cases because the clinical presentations are similar. brain stem herniation. The potassium level is normal. The diagnosis of SIADH is based on low serum sodium. Madias NE: Hyponatremia. coma. Regardless of the exact etiology.” Among patients with recent trauma or neurosurgical procedure.Palmer BF: Hyponatremia in patients with central nervous system disease: SIADH versus CSW. Trends Endocrinol Metab 14:182-187. REFERENCES 1. Some studies have suggested the brain may directly release BNP in response to injury as a possible mechanism to limit intracranial pressure increases. Neurologic injury may stimulate the sympathetic release of BNP from the heart. and death. such as brain natriuretic protein (BNP). The primary differentiation is the patient’s volume status. The most common symptoms of hyponatremia are headache.Cole CD. Primary treatment is water restriction. With appropriate volume resuscitation. high urinary sodium level. thyroid. These patients are hyponatremic. Couldwell WT: Hyponatremia in the neurosurgical patient: diagnosis and management. vomiting. or renal dysfunction. 2003 page 878 . disorientation. volume contracted. while their BUN is decreased. although most patients with a serum sodium > 125 mmol/L will be asymptomatic unless the change in serum sodium has been precipitous. BUN and hematocrit are typically elevated while potassium levels are normal or increased. restlessness. or low serum osmolality. inappropriately concentrated urine compared with serum osmolality. These patients usually have a normal volume status as well as hematocrit. complications can also arise if correction is too rapid. normal. In cases of SIADH. Liu JK. the primary problem is increased renal losses of sodium. The pathophysiology of CSW is not well characterized. Most patients who develop hyponatremia are inadequately resuscitated and have initiated antidiuretic hormone (ADH) as a mechanism of water retention and secondary volume expansion. and the absence of peripheral edema or dehydration with no evidence of adrenal. respiratory arrest. American College of Surgeons SESAP 13 Category 9 Item 1 Three months after deceased donor kidney transplant. The patient has been treated for acute rejection within the past 2 weeks.4 to 2. diarrhea. a 50-year-old man comes to the emergency department with malaise.0 mg/dL over the past two weeks. Physical examination reveals diffuse epigastric tenderness. Which of the following statements is TRUE? (A) Renal graft magnetic resonance angiography (MRA) should be obtained (B) Colonoscopy is indicated (C) Upper gastrointestinal endoscopy should be diagnostic (D) The prednisone dose should be increased (E) He should receive Helicobacter pylori therapy as an outpatient page 879 . The serum creatinine has risen from 1. and the prednisone dose is being tapered. abdominal pain. and nausea. et al: Clinical and immunologic aspects of cytomegalovirus infection in solid organ transplant recipients. van Leeuwen EM. and C virus. Because this patient’s presentation does not strongly suggest CMV colitis. colonoscopy would not be indicated as a primary diagnostic test. hepatitis A. 2002 3. A high index of suspicion for CMV disease is indicated in all transplant recipients in this time period. This patient’s constitutional symptoms and abdominal complaints should raise concern for CMV disease. CMV disease can be manifested as a constitutional malaise with fever. 2004 2. The risk for symptomatic CMV infection rises significantly in seronegative recipients of seropositive organs. van Lier RA.Chiu LM. the first test would be a graft ultrasound. In this patient serology for CMV should be obtained and an upper gastrointestinal endoscopy with biopsy should provide diagnostic material.Rowshani AT. pneumonitis. Domagala BM. REFERENCES 1. the incidence remains over 50%. retinitis or colitis. even if the treating physician does not know the organ and recipient CMV status at the time of presentation. Prog Transplant 14:114-129. The rise in serum creatinine in this patient may be multifactorial. and HIV. gastritis. Transplantation 79:381-386. and human herpes virus-6 (HHV-6). but also in patients treated with antilymphocyte therapy for rejection management. Treating the patient’s symptoms presuming benign ulcer disease would not be sufficient.American College of Surgeons SESAP 13 Critique for Category 9 Item 1 The major opportunistic infections seen in the early posttransplant period (1 to 6 months) include herpes family viruses cytomegalovirus (CMV). If a mechanical etiology is being considered. Park JM: Management of opportunistic infections in solid-organ transplantation. Epstein-Barr virus (EBV). B. Bemelman FJ. 2005 page 880 .Fishman JA: Overview: fungal infections in the transplant patient. Magnetic resonance angiography (MRA) would not contribute to this patient’s evaluation at this point. but treating for rejection without an evaluation for infection would be incorrect. Although CMV prophylaxis has significantly reduced the incidence of CMV disease in the highest risk patients. Transpl Infect Dis 3:3-11. Nine years previously she had a deceased donor renal transplant for end-stage renal disease. The most likely cause of the abnormality is (A) Hodgkin's lymphoma (B) B-cell lymphoma (C) T-cell lymphoma (D) gastrointestinal stromal tumor (GIST) (E) plasmacytoma page 881 . She has left-sided abdominal pain. Colonoscopy is normal.American College of Surgeons SESAP 13 Category 9 Item 2 A 32-year-old woman presents 6 weeks after an uncomplicated laparoscopic cholecystectomy for typical biliary colic symptoms. The abdominal computed tomographic (CT) scan shown is obtained. Dorent R. 1995 4. Br J Haematol 127:429-432.Leblond V.com 3. Available at: Up To Date. et al: T-cell post-transplantation lymphoproliferative disorders after cardiac transplantation: a single institutional experience. Mancini DM.utdol. Curr Probl Surg 25:367-472. Gastrointestinal stromal tumors and plasmacytomas do not predominate in transplant recipients. the incidence is up to 93% in transplant recipients. Tsao L. They may be monoclonal or polyclonal Management of these tumors is complicated by the need to balance the immunosuppressive needs of the recipient organ with treatment of the tumor. and the intensity and duration of induction immunosuppression. Davi F. The computed tomographic (CT) scan shows a diffuse mass arising from the proximal small bowel mesentery. 2004 2. The most likely cause of this abnormality is a non-Hodgkin’s B-cell lymphoma. REFERENCES 1.Friedberg JW. Factors associated with an increased incidence of B-cell lymphoproliferative disorders in the posttransplant patient include recipient negative donor positive EBV. Eisen HJ.Penn I: Cancers complicating organ transplantation. and up to 89% of these are B-cell lymphomas. Sutton L. J Clin Oncol 13:961-968. www. Starzl TE: The diagnosis and treatment of posttransplant lymphoproliferative disorders. Makowka L. 1988 5. Addonizio LJ. Most patients with monoclonal malignancies can be treated with intensive chemotherapy (CHOP).Nalesnik MA. Overall survival rates range from 25% to 35%. et al: Lymphoproliferative disorders after organ transplantation: a report of 24 cases observed in a single center. While non-Hodgkin’s lymphoma accounts for up to 65% of all lymphomas in the general population. Studies have shown an increased risk with OKT3 as well as tacrolimus (FK506). Brennan DC: Lymphoproliferative disorders following solid organ transplantation.Draoua HY. Epstein-Barr virus (EBV)-associated lymphoproliferations are noted in most of these B-cell lymphomas. 1990 page 882 . which led to partial small bowel obstruction.American College of Surgeons SESAP 13 Critique for Category 9 Item 2 Lymphoproliferative disorders are a significant consequence of solid organ transplantation. N Engl J Med 323:1767-1769. American College of Surgeons SESAP 13 Category 9 Item 3 Which of the following is an absolute contraindication for liver transplantation? (A) Autoimmune hepatitis (B) Hepatitis C infection (C) Synchronous extrahepatic malignancy (D) Hepatitis B infection (E) Fulminant hepatic failure page 883 . REFERENCES 1. the general indications for liver transplantation are outlined in the table. in Morris PJ. Wood WC (eds): Oxford Textbook of Surgery. in Fink MP. ed 5. Ko DSC. Fung JJ: Liver transplantation recipients. Bollinger RR. 2001. New York. which may be modified by regional variation. in Norton JA. et al (eds): Surgery: Basic Science and Clinical Evidence. The second table lists contraindications to liver transplantation. Although there is regional variation. Vincent J-L. 2005.American College of Surgeons SESAP 13 Critique for Category 9 Item 3 Liver transplantation is a recognized treatment option for end-stage liver disease for several etiologies. pp 685-707 2. Barie PS.Kramer DJ. Cosimi AB: Liver transplantation. Oxford University Press. Chang AE. New York. ed 2. Elsevier Saunders. pp 1986-2000 page 884 .Harland RC: Pancreas transplantation. Mazariegos GV. Kochanek PM (eds): Textbook of Critical Care. Abraham E. 2000. Springer Verlag.Feng S. pp 1463-1473 3. Philadelphia. American College of Surgeons SESAP 13 Category 9 Item 4 Following intestinal transplantation for short bowel syndrome as a sequela of necrotizing enterocolitis. survival of the intestinal graft would be significantly prolonged by (A) antilymphocyte antibody induction (B) bone marrow infusion (C) calcineurin inhibitor administration (D) leptin administration (E) administration of the Rho-kinase inhibitor. Y-27632 page 885 . pp 1473-1499 2. et al (eds): Surgery: Basic Science and Clinical Evidence. 2001 page 886 . 2001. REFERENCES 1. No data support the specific efficacy of bone marrow infusion or Rho-Kinase inhibitor with intestinal transplantation. Chang AE. However.Reynolds M: What’s new in pediatric surgery. Both of these agents are calcineurin inhibitors.Hanto DW. Leptin is not used in small bowel transplantation. Bollinger RR. Barie PS.American College of Surgeons SESAP 13 Critique for Category 9 Item 4 Intestinal transplantation was initially performed under cyclosporine immunosuppression. Springer Verlag. New York. it has been documented that immunologic results are improved with the use of tacrolimus (FK-506). Antilymphocyte antibody induction has not been associated with any improvement in immunologic results. Whiting JF. Valente JF: Transplantation of the liver and intestine. Virtually all recent reports have emphasized the use of this class of immunosuppressive drug in conjunction with corticosteroids. J Am Coll Surg 193:66-72. in Norton JA. American College of Surgeons SESAP 13 Category 9 Item 5 Liver transplantation is contraindicated for a (A) 56-year-old man 4 days after orthotopic liver transplant who has acute hepatic artery thrombosis and for whom arterial thrombectomy was unsuccessful (B) 48-year-old man with a history of alcoholic cirrhosis who has been abstinent for 5 years (C) 68-year-old woman with nonalcoholic cirrhosis related to fatty liver disease (D) 52-year-old man with cirrhosis who has just undergone a wedge resection of a 5-mm focus of hepatocellular carcinoma involving the middle lobe of the right lung (E) 32-year-old woman in the ICU with variceal bleeding who has portal hypertension due to hepatitis C-induced cirrhosis page 887 . Settmacher U. but 6444 transplants were performed in 2005. The number of liver transplants performed annually from 1998 to 2002 in the US increased slightly from 4516 to 5329. et al: Selection of patients with hepatocellular carcinoma for liver transplantation.United Network for Organ Sharing. www. Transplantation 71:1189-1204.Stange BJ.Sutcliffe R.org. et al: Hepatic artery thrombosis after adult liver transplantation. The presence of extrahepatic disease is a contraindication to proceeding with liver transplantation. Nuessler NC. Although seniors with far-advanced end-stage liver disease are high-risk for liver transplantation when compared with younger patients.American College of Surgeons SESAP 13 Critique for Category 9 Item 5 Liver transplantation is the standard treatment for end-stage liver disease. 2006 page 888 . ~1% to 4% risk per year in patients with hepatitis C. and advanced age alone should not exclude a patient from liver transplant. Liver transplantation can be safely performed in patients with small tumors (< 5 cm) in the setting of cirrhosis. Liver Transpl 10:957-967. Patients older than 60 are undergoing transplantation with increasing frequency. Olthoff KM. Frost AE. 2001 4. there was no difference in survival. 2004 2. Patients with a history of alcoholism or alcohol abuse are considered for transplantation after careful assessment by a psychiatrist or addiction specialist. mechanisms organized by the United Network of Organ Sharing (UNOS) facilitate early re-transplantation.Keswani RN. 2003 3. The length of abstinence required for listing varies.unos.Steinman TI. Ahmed A. et al: Guidelines for the referral and management of patients eligible for solid organ transplantation. Portmann B. When arterial revascularization cannot be successfully accomplished in a patient with early HAT. Glanemann M. but HAT can be a devastating problem that requires re-transplantation in nearly half the patients due to biliary complications or deteriorating liver graft function. Keeffe EB: Older age and liver transplantation: a review. accessed June 28. Br J Surg 93:11-18. reflecting increased educational efforts about organ donation. When comparing patients who receive transplants for alcoholic cirrhosis and nonalcoholic liver disease. Rela M. REFERENCES 1. Liver Transpl 9:612-620. All patients with cirrhosis are at higher risk for developing hepatocellular carcinoma. 2006 5. The incidence of hepatic artery thrombosis (HAT) after liver transplantation ranges from 2% to 9% in various series. Becker BN. Hepatitis C infection is currently the most common condition resulting in liver transplantation. most seniors who survive liver transplantation have full or only minimally limited functional status. Maguire D. and 1-year survival rates of 85% to 90% are typical at most centers. The most likely cause is (A) hyperparathyroidism (B) Paget’s disease (C) posttransplant lymphoproliferative disease (D) corticosteroids (E) osteomyelitis page 889 .American College of Surgeons SESAP 13 Category 9 Item 6 Ten years after a renal transplant. The patient has developed hip pain. Drug therapy includes tacrolimus and corticosteroids. and the x-ray shown is obtained. a patient continues to have stable renal function. mediastinum. the risk of proximal femur fractures is only marginally increased. and -1. which is based on the measurement of bone mineral density (BMD). et al: Assessment of fracture risk. 1999 3. liver. J Bone Miner Res 14:1700-1707. 2005 2. While severe primary hyperparathyroidism (HPT) is clearly associated with osteitis fibrosa cystica (brown tumor). Osteoporos Int 16:581-589. solitary lesions were seen in the lungs. A number of clinical risk factors provide additional information on fracture risk. lung. bone marrow. and pelvic fractures. or nonspecific symptoms such as tonsillitis (particularly children) and weight loss. REFERENCES 1. asymptomatic primary HPT adversely affects the skeleton. Melton LJ III. smoking. the lymph nodes and liver were most commonly affected. De Laet C. and rheumatoid arthritis. The remaining patients had solitary lesions at extra-nodal sites including kidney.Taylor AL. Johansson H. a prior fragility fracture. distal forearm. Crit Rev Oncol Hematol 56:155-167. systemic corticosteroids. and skin. Wermers RA. bowel. Crowson CS. bowel. Primary HPT is associated with an increased risk of vertebral. The independent contribution of these risk factors can be integrated by the calculation of fracture probability with or without the use of BMD. 2005 page 890 .Kanis JA. rib. In patients presenting with multiple lesions. Current US guidelines suggest that intervention should be offered to patients shown to have a T-score of -2. mediastinum. including age. it remains uncertain whether mild.American College of Surgeons SESAP 13 Critique for Category 9 Item 6 The x-ray is consistent with the diagnosis of osteoporosis. and central nervous system (CNS). Primary hyperparathyroidism (HPT) is a complex endocrinopathy involving calcium metabolism and a potent hormone made by the parathyroid glands. and CNS. Marcus R. excess alcohol intake. The Canadian PTLD Survey Group analyzed 90 cases of PTLD occurring in 4283 solid organ transplant recipients followed over a nine-year period. The diagnosis is confirmed by inappropriately elevated parathyroid hormone levels accompanied by a high normal or elevated serum calcium. More rarely. Bradley JA: Post-transplant lymphoproliferative disorders (PTLD) after solid organ transplantation. Other sites included kidney. tonsil. Approximately two thirds of patients presented with disease localized to a single site. lymphadenopathy (30%).0 SD in the absence of significant risk factors. Most patients present with fever (50%). a parental history of hip fracture. The clinical presentation of posttransplant lymphoproliferative disease (PTLD) is highly variable. Borgstrom F.5 SD in the presence of risk factors. of which only a quarter were within the lymph nodes. Treatment may be offered to patients identified as having a fracture probability over an intervention threshold. et al: Primary hyperparathyroidism and the risk of fracture: a population-based study. tonsils.Khosla S. and trimethoprim-sulfa. corticosteroids. Two weeks after transplant. The most likely cause of these symptoms is (A) tacrolimus toxicity (B) corticosteroid toxicity (C) mycophenolic acid toxicity (D) rejection (E) meningitis page 891 .American College of Surgeons SESAP 13 Category 9 Item 7 After receiving a renal transplant. mycophenolate mofetil. a patient is started on a regimen of tacrolimus. she develops seizures. Sollinger HW: Mycophenolates in transplantation. Taylor RS. but potentially increases the risk of patients developing posttransplant diabetes. After the first month there is an increased risk of atypical infections. The neurologic complications are believed to be related to tacrolimus’s effect on magnesium wasting. Mycophenolate mofetil is a potent immunosuppressant that inhibits B. REFERENCES 1. Clin Transplant 18:485-492.American College of Surgeons SESAP 13 Critique for Category 9 Item 7 Cyclosporine is an immunosuppressant introduced in organ transplantation more than two decades ago.Maes BD. Both substances have similar intrinsic nephrotoxic potential. Outcomes. Chapman JR.Webster A. 2004 2. Woodroffe RC. including 1-year serum creatinine levels and graft and patient survival. Cochrane Database Syst Rev (4):CD003961.and T-cell proliferation by blocking production of guanosine nucleotides required for DNA synthesis. Vanrenterghem YF: Cyclosporine: advantages versus disadvantages vis-à-vis tacrolimus. 2005 page 892 . Transplant Proc 36:40S-49S. et al: Tacrolimus versus cyclosporin as primary immunosuppression for kidney transplant recipients. The risk of infectious complications for a patient receiving long-term immunosuppression during the first month posttransplant is similar to that of an immunocompetent postoperative patient. 2004 3. and neurologic complications including tremors and seizures. Tacrolimus is superior to cyclosporine in improving graft survival and preventing acute rejection after kidney transplantation. It is commonly associated with bone marrow suppression and gastrointestinal side effects. are excellent after kidney transplantation with these drugs. and tacrolimus (FK-506) was introduced more than one decade ago. gastrointestinal side effects. The next step should be (A) plasmapheresis (B) splenectomy (C) methotrexate (D) increased corticosteroid dose (E) increased intravenous immunoglobulin therapy page 893 .000/mm3.American College of Surgeons SESAP 13 Category 9 Item 8 A 52-year-old man who has immune thrombocytopenic purpura (ITP) presents with a 1-week history of uncontrollable epistaxis and generalized petechiae.000/mm3.5 mg/kg/day for 5 days) followed by intravenous immunoglobulin therapy (1 g/kg/day for 3 days). He receives prednisone (1. His admission platelet count is 8. His platelet count remains below 10. ITP is usually discovered incidentally. Plasmapheresis and methotrexate have no demonstrated role for the treatment of ITP. Such infections are often fulminant. Additional tests. Response rates range from 50% to over 75% and most patients respond in the first 3 weeks of treatment. lymphoproliferative disorders (chronic lymphocytic leukemia. Pnu-Immune). Petechiae or ecchymoses develop spontaneously when counts are between 10. immunodeficiency states (IgA deficiency and common variable hypogammaglobulinemia). and patients with counts below 10. and examination of the peripheral blood smear. treatment with oral prednisone is required at the time of presentation. The optimum timing of vaccine given postoperatively is unknown and recommendations vary widely. immunization against S pneumoniae using the 23-valent pneumococcal polysaccharide vaccine (Pneumovax. help exclude other causes. and lymphoma). The goal of splenectomy is to reduce clearance of particulate antigens. Vaccines are ideally given 2 weeks preoperatively. Splenectomy is often recommended within 3 to 6 months if more than 10 to 20 mg of prednisone/day is required to maintain a platelet count above 30. is recommended for all patients having a splenectomy. This is commonly done by a history. tolerance of corticosteroids. Decisions about splenectomy depend on the severity of the disease.000/mm3. ITP is mediated by autoantibodies. Most patients have antibodies against several platelet-surface glycoproteins when the disease becomes clinically evident.000 and 30. the antiphospholipid syndrome. because the immune response to the vaccines is believed to be better when the spleen is intact. but it still should be given.000/mm3 may note excessive bruising with minor trauma.000/mm3 are at risk for internal bleeding. ITP occurs most commonly in patients aged 18 and 40 and is two to three times as common among women as men. which covers ~73% of strains causing OPSI. diminish response to new antigens. but the factors that initiate autoantibody production are unknown. complete blood count. Asplenic persons are at risk for overwhelming postsplenectomy sepsis (OPSI) caused by bacteria and occasionally protozoa.American College of Surgeons SESAP 13 Critique for Category 9 Item 8 Immune thrombocytopenic purpura (ITP) is an autoimmune disorder characterized by a low platelet count and mucocutaneous bleeding. and therapy with drugs such as heparin and quinidine. Those with platelet counts of 30.000/mm3 . A booster dose is page 894 . The clinical diagnosis of ITP is made by confirming the presence of isolated thrombocytopenia and by excluding concurrent causes of thrombocytopenia. In patients with counts below 10.000/mm3. with high mortality. physical examination. including CT scanning and liver biopsy. Sustained remission is infrequent and the cost of intravenous immune globulin is considerable. and decrease levels of tuftsin and properdin. infection with HIV and hepatitis C virus. Secondary forms of the disease occur in association with systemic lupus erythematosus.000 to 50. Intravenous immune globulin is used to treat internal bleeding when the platelet count remains below 5000/mm3 despite corticosteroid treatment or when extensive or progressive purpura is present. large granular lymphocytic leukemia. impair phagocytosis of bacteria. A response is seen in 80% of cases. Because of the preeminent role of pneumococcal infection in OPSI. and the patient's preferences. In patients with platelet counts > 50. but do not have a primary role in the management of ITP. The diagnosis of ITP remains one of exclusion. Increasing doses of corticosteroids and intravenous immune globulin will not improve response rates.000/mm3. Park AE. if necessary. Annual influenza vaccination is also recommended for asplenic persons. N Engl J Med 346:995-1008. the increased severity of disease suggests that the vaccine should be used. Dunn DL. All vaccines can be given together. 2005. 1997 4.[No Authors Listed]: Diagnosis and treatment of idiopathic thrombocytopenic purpura: recommendations of the American Society of Hematology. 2001 3. Wall RA: Prevention and management of infections in patients without a spleen. Although the role of meningococci in OPSI is not clearly established. Andersen DK. McKinlay R: Spleen.Cines DB. pp 1297-1316 page 895 . and is recommended. a single dose of H influenzae type b protein conjugate vaccine is believed to be immunogenic in asplenic patients. 2002 2.American College of Surgeons SESAP 13 recommended at 3 to10 years. ed 8. Blanchette VS: Immune thrombocytopenic purpura. REFERENCES 1. The American Society of Hematology ITP Practice Guideline Panel. et al (eds): Schwartz's Principles of Surgery. McGraw-Hill.Davidson RN. in Brunicardi FC. Clin Microbiol Infect 7:657-660. Billiar TR. Although the role of H influenzae type b in OPSI is unclear. Ann Intern Med 126:319-326. New York. 03% (D) The increasing number of kidney transplants is secondary to increasing numbers of living donors (E) Direct payment from a patient waiting for an organ transplant to a potential donor for the purpose of donation is illegal in the US page 896 .American College of Surgeons SESAP 13 Category 9 Item 9 Which of the following statements about organ donation is NOT true? (A) The United Network of Organ Sharing (UNOS) publicly opposes persons who solicit deceased organ donation as a means for achieving their own transplant (B) A nationwide commercial website matching prospective living donors and recipients is illegal in the US (C) The short-term mortality risk for a living kidney donor is 0. and through other advertising. organ donation has been fertile territory for both extraordinary compassion and complex ethical questions. spouse. OPTN is establishing quality criteria and guidelines for living donor kidney and liver programs. including 62. but doubts about the safety of donation by living persons and the integrity of the allocation process can jeopardize public trust. The OPTN takes an active role in transplantation from living donors. Yet the demand for organs remains far greater than the supply. OPTN/UNOS later announced its opposition to the solicitation of organs from deceased donors who had no personal or family bond with the patient as possibly undermining the allocation system and preventing the best medical use of the organ.American College of Surgeons SESAP 13 Critique for Category 9 Item 9 During the past 50 years. including medical criteria for who can donate and individual transplant institutions' protocols for live unrelated donation. Reasonable payments are permitted for the costs of organ procurement and storage and for the expenses incurred by living donors in travel. Soliciting organs from a living or dead donor is not unlawful. housing. defined as the allocation of an organ to a recipient named by those authorized to make the donation.000 solid organ transplantations in the United States. The networks share the same board of directors and are collectively known as OPTN/UNOS. OPTN/UNOS will provide comprehensive resource information to support prospective live donors. The responsibilities include developing guidelines about the public solicitation of organ donors.05%. there were about 27. Despite improvements in minimally invasive approaches to perform the donor procedure. In 2004.500 patients awaiting kidneys. the Department of Health and Human Services directed it to develop voluntary-allocation guidelines for organ donations from living donors that are made to an anonymous pool. and the outcome from a friend. In 2004. and other voluntary policies and guidelines to promote the safety and efficacy of living donor transplantation for the donor and the recipient. the rules for donation and the fair allocation of organs have struggled to keep pace. Programs must have staff with the requisite training and expertise to evaluate donors fully. Federal regulations authorize directed donation. it will not create a Web site similar to MatchingDonors. Although public solicitation of organs has been a factor in very few transplantations. and the subversion of the standards for donation. not to specific patients. The increase in number of kidney transplants during the past decade has been driven by the increase in living renal donors. OPTN/UNOS suggests that anyone can be a live kidney donor who is medically and psychosocially suitable. however. remains a major surgical procedure with attendant risks and complications. or anonymous donor is just as good as that from a parent or child. there is still a risk of mortality < 0. The United Network for Organ Sharing (UNOS) operates the Organ Procurement and Transplantation Network (OPTN) under contract with the federal government.com. nearly 1600 more than in 2003. to ensure page 897 . although the National Organ Transplant Act of 1984 prohibits the transfer of organs for use in transplantation if the transfer affects interstate commerce. the transplantation of kidneys from living donors is increasingly common. and lost wages. the inequitable allocation of organs. As transplantation has become safer and outcomes have improved. A passionate controversy has developed about the public solicitation of organs over the Internet. It is no longer the case that one has to be HLA-matched.000 people on waiting lists. Greater public awareness of the organ shortage could lead to more donations. Specifically. Solicitation arouses concern about the potential for financial exploitation. Transplantation of a kidney from a living donor. and later announced that it would not participate in efforts to solicit living donors for specific transplant candidates. There are currently about 100. on billboards. Ingelfinger JR: Risks and benefits to the living donor.Truog RD: The ethics of organ donation by living donors. there might be a registry for altruistic kidney and liver donors. which lists volunteer marrow donors. 2005 page 898 . 2005 3. N Engl J Med 353:444-446. The OPTN is working toward requiring that transplantation centers report within 72 hours the death of a living donor or the donor's need for organ transplantation. OPTN/UNOS will develop a nationwide mechanism for allocating organs from living donors who have not directed their donation to specific persons. Finally. In addition.American College of Surgeons SESAP 13 that donations are voluntary. The principles will be similar to those for allocating organs from cadaveric donors. REFERENCES 1. such as the National Marrow Donor Program. N Engl J Med 353:447-449. better data about the outcomes of living donors should be forthcoming. Eventually.Steinbrook R: Public solicitation of organ donors. and to perform living donor surgery. N Engl J Med 353:441-444. 2005 2. American College of Surgeons SESAP 13 Category 9 Item 10 Posttransplant lymphoproliferative disorder (A) is usually seen 1 year or more after transplant (B) is most common in liver transplant recipients (C) is related to herpes virus infection (D) can be treated with rituximab (E) requires excisional biopsy for diagnosis page 899 . and enlargement of the tonsils and cervical nodes. with fever. It is most common in patients with small bowel (7% to 11%) and lung (2% to 8%) transplants. and early presentation mimics infectious mononucleosis. 2000 3. et al: Treatment of posttransplant lymphoproliferative disease with rituximab: the remission. Maintenance of graft function during treatment with rituximab makes this an attractive option. Arch Intern Med 163:1997-2004. patients presenting after one year have more extranodal involvement. Stevens SJ. Monoclonal gammopathy is present in 71% of patients with the disease and only 28% of patients without the disease.American College of Surgeons SESAP 13 Critique for Category 9 Item 10 Posttransplant lymphoproliferative disorder (PTLD) is a lesion associated with the Epstein-Barr virus that occurs in the setting of immunosuppression after transplantation. 2003 2. REFERENCES 1. Middeldorp JM. Impaired cytotoxic T-cell response leads to an uncontrolled B cell proliferation. due to the high amount of lymphoid tissue present in these grafts. has shown promise in treating this disease.Swinnen LJ: Diagnosis and treatment of transplant-related lymphoma. Biselli M. Patients with a higher degree of immunosuppression may present with a more diffusely infiltrative process involving multiple organs. In contrast. The gold standard for diagnosis is histopathology or cytology. Transplantation 73:100-104. and the complication. This leads to regression of PTLD in 25% to 50% of patients.Andreone P. the relapse. The CD20 antigen is widely expressed on B lymphocytes. Lorenzini S. an anti-CD 20 antibody. examining tissue from either an excisional biopsy or from fine-needle aspiration. 2002 page 900 . with gastrointestinal involvement common. The syndrome’s variable presentation makes diagnosis difficult. but in other transplants reduction of immunosuppression puts the patient’s life at risk. Ann Oncol 1:45-48. Rituximab. and is most successful in patients who develop PTLD within the first year. et al: Posttransplantation lymphopoliferative disorders. and other serologic and diagnostic tests have similar accuracy. although CD20 negative B lymphocytes responsible for PTLD do occur. Primary treatment is reduction of immunosuppression if possible. and the response rate with rituximab is approximately 65%. night sweats.Verschuuren EA. The incidence in renal transplant patients is approximately 1%. Gramenzi A. The transplanted organ is affected in up to 20% of cases. This may be the best option for renal transplant patients with PTLD. Incidence is highest within the first year after transplantation. van Imhoff GW. 5 mg/dL. The next step in management should be (A) subtotal parathyroidectomy (B) an increase in corticosteroid dose (C) hydration and diuresis (D) calcitonin injections (E) phosphate binders page 901 . Her serum calcium is 11. and osteoporosis.American College of Surgeons SESAP 13 Category 9 Item 11 A 37-year-old woman with end-stage renal disease had a cadaveric renal transplant 18 months previously. She now presents with fatigue. bone pain.0 mg/dL. She tolerated her transplantation well and has been receiving a stable regimen of tacrolimus and corticosteroids with a creatinine of 1. Starling JR. Secondary hyperparathyroidism can often be controlled with adequate calcium replacement and phosphate binders as well as vitamin D replacement. eucalcemia will not be achieved with the forearm graft alone. Surgical treatment is recommended for all patients with tertiary hyperparathyroidism. Because the parathyroid remnant is left in its natural position and is living on its own vascular supply.American College of Surgeons SESAP 13 Critique for Category 9 Item 11 Tertiary hyperparathyroidism occurs in 2% to 5% of patients after renal transplantation. the chance of recurrent disease is low. In a significant number of patients undergoing total parathyroidectomy and autotransplantation. Most patients with end-stage renal disease develop secondary hyperparathyroidism while undergoing dialysis. In several large series.Nichol PF. 2002 page 902 . REFERENCES 1. the incidence of hypoparathyroidism is very low. In most patients. subtotal parathyroidectomy was associated with a 99% long-term cure rate with minimal morbidity.Haustein SV. Klovning JJ. Hydration and diuresis are options for patients who have severe hypercalcemia (calcium > 13 mg/dL). Surgery 138:1066-1071. Chen H: The role of intraoperative parathyroid hormone testing in patients with tertiary hyperparathyroidism after renal transplantation.5 glands) or total parathyroidectomy with forearm parathyroid graft auto-transplantation. 2005 2. but kidney transplantation is the best treatment. Ann Surg 235:673-680. These are temporizing measures only. and osteoporosis. Starling JR. Corticosteroids and calcitonin have no role in the management of tertiary hyperparathyroidism. Mack E. The 2% to 5% of patients who develop tertiary hyperparathyroidism present with elevated calcium and elevated parathyroid hormone levels. et al: Long-term follow-up of patients with tertiary hyperparathyroidism treated by resection of a single or double adenoma. As long as renal functions remain stable. The definitive treatment can be either subtotal parathyroidectomy (resection of 3. secondary hyperparathyroidism resolves within 1 year of transplantation. They often present with fatigue. Mack E. bone pain. American College of Surgeons SESAP 13 Category 9 Item 12 Which of the following statements about acute rejection is NOT true? (A) It is T-cell mediated (B) It is related to organ-host human leukocyte antigen disparity (C) Treatment can save the grafted organ in 90% to 95% of cases (D) It does not occur with living related donors (E) It is associated with an increased risk of chronic rejection page 903 . These rearrangements occur as immature T cells develop in the thymus. 1999 3. The TCR recognizes peptide antigens in association with the major histocompatibility complex (MHC). and the serum creatinine level at 6 months after transplantation as independent prognostic variables predicting long-term graft failure after kidney transplantation from living related donors. Acute rejection most commonly occurs between 1 and 6 weeks after transplantation. but may occur at any time. the number of acute rejection episodes. Parrott NR. 10% to 40% of transplant recipients experience acute rejection. N Engl J Med 313:337-42. Transpl Int 11:S5-S9. As acute rejection occurs. Ortho Multicenter Transplant Study Group. Currently available drugs such as high-dose corticosteroids can arrest acute rejection in virtually all cases. This can be seen in biopsy specimens of the graft. Roberts I.Ashraf S. At a cellular and molecular level. The antigenic specificity of the T lymphocyte is conferred by the α and β-chains of the T-cell receptor complex (TCR). activated T lymphocytes infiltrate the allograft and begin to destroy their target cells. The present success of organ transplantation has been made possible by significant advances in the prevention and treatment of acute rejection. Dyer P. Patients receiving kidney transplants from living related donors are not protected against acute rejection.[No Authors Listed]: A randomized clinical trial of OKT3 monoclonal antibody for acute rejection of cadaveric renal transplants. 1998 2. Acute cellular rejection results from the activation of recipient T lymphocytes by graftderived antigens. In humans these cell-surface proteins are termed human leukocyte antigens (HLA).Benichou G: Direct and indirect antigen recognition: the pathways to allograft immune rejection. 1985 page 904 . A multivariate analysis of several recent studies identified the etiology of the recipient’s renal failure. Front Biosci 4:D476-D480. the basis for allograft rejection is now reasonably clear. despite optimal baseline immunosuppression. and most clinical immunosuppression is directed toward its prevention or treatment.American College of Surgeons SESAP 13 Critique for Category 9 Item 12 Acute cellular rejection of solid organ transplants is the form of early rejection most commonly encountered in transplantation. However. REFERENCES 1. et al: Clinical response and temporal patterns of acute cellular rejection: relationship to chronic transplant nephropathy. American College of Surgeons SESAP 13 Category 9 Item 13 Has largely replaced azathioprine in transplant patients (A) Cyclosporine (B) Tacrolimus (C) Mycophenolate mofetil (D) Sirolimus (E) Daclizumab page 905 . including posttransplant diabetes mellitus.[No Authors Listed]: A comparison of tacrolimus (FK 506) and cyclosporine for immunosupression in liver transplantation. Whereas the risk for nephropathy is similar. inhibits purine synthesis. Sirolimus is a macrolide antibiotic. and hyperlipidemia. the risk for hyperlipidemia and hypertension is less. and the risk for diabetes is greater than with cyclosporine. daclizumab has not been associated with an increased risk of infection or lymphoproliferative disease. Daclizumab is a monoclonal antibody against CD 25.Halloran PF: Immunosuppressive drugs for kidney transplantation. Mycophenolate mofetil. mycophenolate has largely replaced azathioprine in transplant patients. that has been combined with cyclosporine and tacrolimus to decrease the risk of rejection. 2004 2. REFERENCES 1. However. ripamycin. Because of its safety profile and ease of use (dose monitoring is unnecessary). hypertension. with current recommendations including measurement of peak (2 hours after dose) drug levels. nephropathy. it has been suggested that this agent can potentiate cyclosporine or tacrolimus nephrotoxicity. The US Multicenter FK506 Liver Study Group. like azathioprine.American College of Surgeons SESAP 13 Critique for Category 9 Item 13 The side effect profile of cyclosporine. 1994 page 906 . or the interleukin 2 receptor alpha chain. Unlike virtually all other classes of drugs used in transplantation. N Engl J Med 331:1110-1115. while those with hypertension or hyperlipidemia might be better treated with tacrolimus. Older or obese patients with an increased risk for diabetes should be treated with cyclosporine. Tacrolimus has a similar mode of action and side effect profile as cyclosporine. Transplant patients with low risk for rejection can be treated with sirolimus as a single agent 2 to 4 months after transplantation. N Engl J Med 351:2715-2729. warrants close dose monitoring. American College of Surgeons SESAP 13 Category 9 Item 14 Less likely to cause hyperlipidemia. hypertension. and cosmetic problems than other drugs in class (A) Cyclosporine (B) Tacrolimus (C) Mycophenolate mofetil (D) Sirolimus (E) Daclizumab page 907 . Whereas the risk for nephropathy is similar. including posttransplant diabetes mellitus. Because of its safety profile and ease of use (dose monitoring is unnecessary). Sirolimus is a macrolide antibiotic. it has been suggested that this agent can potentiate cyclosporine or tacrolimus nephrotoxicity. that has been combined with cyclosporine and tacrolimus to decrease the risk of rejection. while those with hypertension or hyperlipidemia might be better treated with tacrolimus. hypertension. inhibits purine synthesis. ripamycin. Transplant patients with low risk for rejection can be treated with sirolimus as a single agent 2 to 4 months after transplantation. mycophenolate has largely replaced azathioprine in transplant patients. 1994 page 908 . Daclizumab is a monoclonal antibody against CD 25. or the interleukin 2 receptor alpha chain. However. and hyperlipidemia. and the risk for diabetes is greater than with cyclosporine. 2004 2. The US Multicenter FK506 Liver Study Group. like azathioprine. daclizumab has not been associated with an increased risk of infection or lymphoproliferative disease. Unlike virtually all other classes of drugs used in transplantation. the risk for hyperlipidemia and hypertension is less. Older or obese patients with an increased risk for diabetes should be treated with cyclosporine. REFERENCES 1.American College of Surgeons SESAP 13 Critique for Category 9 Item 14 The side effect profile of cyclosporine.Halloran PF: Immunosuppressive drugs for kidney transplantation.[No Authors Listed]: A comparison of tacrolimus (FK 506) and cyclosporine for immunosupression in liver transplantation. N Engl J Med 351:2715-2729. Tacrolimus has a similar mode of action and side effect profile as cyclosporine. N Engl J Med 331:1110-1115. warrants close dose monitoring. nephropathy. with current recommendations including measurement of peak (2 hours after dose) drug levels. Mycophenolate mofetil. American College of Surgeons SESAP 13 Category 9 Item 15 Little or no increased risk of posttransplant infection or lymphoproliferative disease (A) Cyclosporine (B) Tacrolimus (C) Mycophenolate mofetil (D) Sirolimus (E) Daclizumab page 909 . Transplant patients with low risk for rejection can be treated with sirolimus as a single agent 2 to 4 months after transplantation. nephropathy. it has been suggested that this agent can potentiate cyclosporine or tacrolimus nephrotoxicity. Daclizumab is a monoclonal antibody against CD 25. Tacrolimus has a similar mode of action and side effect profile as cyclosporine. inhibits purine synthesis. ripamycin. while those with hypertension or hyperlipidemia might be better treated with tacrolimus. Sirolimus is a macrolide antibiotic. the risk for hyperlipidemia and hypertension is less. 1994 page 910 . Unlike virtually all other classes of drugs used in transplantation. N Engl J Med 331:1110-1115. The US Multicenter FK506 Liver Study Group. However. or the interleukin 2 receptor alpha chain.[No Authors Listed]: A comparison of tacrolimus (FK 506) and cyclosporine for immunosupression in liver transplantation. REFERENCES 1. hypertension. with current recommendations including measurement of peak (2 hours after dose) drug levels.American College of Surgeons SESAP 13 Critique for Category 9 Item 15 The side effect profile of cyclosporine. Whereas the risk for nephropathy is similar. daclizumab has not been associated with an increased risk of infection or lymphoproliferative disease. Because of its safety profile and ease of use (dose monitoring is unnecessary). N Engl J Med 351:2715-2729. like azathioprine. 2004 2. Mycophenolate mofetil. and hyperlipidemia. that has been combined with cyclosporine and tacrolimus to decrease the risk of rejection. including posttransplant diabetes mellitus. mycophenolate has largely replaced azathioprine in transplant patients.Halloran PF: Immunosuppressive drugs for kidney transplantation. and the risk for diabetes is greater than with cyclosporine. Older or obese patients with an increased risk for diabetes should be treated with cyclosporine. warrants close dose monitoring. American College of Surgeons SESAP 13 Category 9 Item 16 Laparoscopic donor nephrectomy (A) Increase in the pool of kidneys for transplant (B) Improved renal graft survival (C) Both (D) Neither page 911 . single center historically controlled series show no significant improvement in function in the resulting grafts compared with open graft retrieval. as well as the increased use of laparoscopic donor nephrectomy.Barry JM: Current status of renal transplantation. Urol Clin North Am 28:677-686. and cross-match positive donors. Use of these measures has correlated with a 50% increase in available renal grafts in the US over the last decade. Important strategies to increase living donor transplants include transplants from genetically unrelated donors. 2005 page 912 . REFERENCES 1. Urology 66:43-46. 2005 2. Outcomes from living. Golconda M. McBride MA.Troppmann C. ABO incompatible donors. Conlin M.American College of Surgeons SESAP 13 Critique for Category 9 Item 16 Increasing the number of living donor kidney transplants is an important strategy to overcome the insufficient supply of deceased-donor kidneys for renal transplantation. Am J Transplant 5:175-182. Norman D: Strategies to increase living donor kidney transplants. and pediatric registry data actually suggest that laparoscopically retrieved grafts may have inferior function. biologically unrelated donors are significantly better than the results from deceased donor renal transplants.Barry JM. 2001 3. Patient evaluations and outcomes. Although laparoscopic donor nephrectomy appears to have made living renal donation more acceptable. Perez RV: Laparoscopic live donor nephrectomy: a risk factor for delayed function and rejection in pediatric kidney recipients? A UNOS analysis. while ABO-incompatible transplants yield results inferior to ABOcompatible transplants. Baker TJ. American College of Surgeons SESAP 13 Category 9 Item 17 ABO-incompatible transplants (A) Increase in the pool of kidneys for transplant (B) Improved renal graft survival (C) Both (D) Neither page 913 . while ABO-incompatible transplants yield results inferior to ABOcompatible transplants. Outcomes from living. and cross-match positive donors.Troppmann C. Am J Transplant 5:175-182. Urology 66:43-46. as well as the increased use of laparoscopic donor nephrectomy.Barry JM: Current status of renal transplantation. Urol Clin North Am 28:677-686. 2005 2. Use of these measures has correlated with a 50% increase in available renal grafts in the US over the last decade. Although laparoscopic donor nephrectomy appears to have made living renal donation more acceptable. and pediatric registry data actually suggest that laparoscopically retrieved grafts may have inferior function. 2005 page 914 .Barry JM. Norman D: Strategies to increase living donor kidney transplants. Perez RV: Laparoscopic live donor nephrectomy: a risk factor for delayed function and rejection in pediatric kidney recipients? A UNOS analysis. Important strategies to increase living donor transplants include transplants from genetically unrelated donors.American College of Surgeons SESAP 13 Critique for Category 9 Item 17 Increasing the number of living donor kidney transplants is an important strategy to overcome the insufficient supply of deceased-donor kidneys for renal transplantation. Baker TJ. McBride MA. ABO incompatible donors. biologically unrelated donors are significantly better than the results from deceased donor renal transplants. 2001 3. single center historically controlled series show no significant improvement in function in the resulting grafts compared with open graft retrieval. Patient evaluations and outcomes. Golconda M. REFERENCES 1. Conlin M. biologically unrelated donors versus deceased donor transplants (A) Increase in the pool of kidneys for transplant (B) Improved renal graft survival (C) Both (D) Neither page 915 .American College of Surgeons SESAP 13 Category 9 Item 18 Living. Patient evaluations and outcomes. Urology 66:43-46. and cross-match positive donors.Barry JM: Current status of renal transplantation.Troppmann C. Conlin M. 2005 page 916 . Baker TJ. while ABO-incompatible transplants yield results inferior to ABOcompatible transplants. single center historically controlled series show no significant improvement in function in the resulting grafts compared with open graft retrieval. Am J Transplant 5:175-182. Golconda M. and pediatric registry data actually suggest that laparoscopically retrieved grafts may have inferior function. biologically unrelated donors are significantly better than the results from deceased donor renal transplants. Outcomes from living. Perez RV: Laparoscopic live donor nephrectomy: a risk factor for delayed function and rejection in pediatric kidney recipients? A UNOS analysis. ABO incompatible donors. as well as the increased use of laparoscopic donor nephrectomy. Important strategies to increase living donor transplants include transplants from genetically unrelated donors. McBride MA. Although laparoscopic donor nephrectomy appears to have made living renal donation more acceptable. 2005 2. Urol Clin North Am 28:677-686.Barry JM. Norman D: Strategies to increase living donor kidney transplants. REFERENCES 1. 2001 3. Use of these measures has correlated with a 50% increase in available renal grafts in the US over the last decade.American College of Surgeons SESAP 13 Critique for Category 9 Item 18 Increasing the number of living donor kidney transplants is an important strategy to overcome the insufficient supply of deceased-donor kidneys for renal transplantation. American College of Surgeons SESAP 13 Category 10 Item 1 Idiopathic hypertropic pyloric stenosis is associated with all of the following EXCEPT (A) monozygotic twin (B) family history (C) white race (D) male sex (E) urban setting page 917 . com/ped/topic1103. Monozygotic twins also have a significantly increased risk. independent of race. 2006. Pyloric stenosis is two times more common in whites than Hispanics or African Americans. The typical infant with pyloric stenosis presents with nonbilious projectile vomiting. Available at: http://www. Available at: Up to Date. Endom EE: Infantile hypertrophic pyloric stenosis. antral polyps. www. gastric duplication.com 3. There is a twofold increase among children whose father had pyloric stenosis. especially in patients outside the typical age range. and hypochloremic metabolic alkalosis (with delayed diagnosis).Olive AP. likely secondary to increased awareness and ease of evaluation with abdominal ultrasound examination.American College of Surgeons SESAP 13 Critique for Category 10 Item 1 Idiopathic hypertropic pyloric stenosis is the most common cause of intestinal obstruction in the pediatric population. dehydration. include malrotation with or without midgut volvulus. Pyloric Stenosis. and focal alveolar hyperplasia. Wales PW. 2005 page 918 . Wajja A. Other considerations.htm 2.emedicine. There is a 4:1 male to female ratio.To T. A recent Canadian study showed an increased association with a rural setting. REFERENCES 1. Arch Pediatr Adolesc Med 159:520-525. and language of origin. socioeconomic status.utod. Langer JC: Population demographic indicators associated with incidence of pyloric stenosis.Nazer H: Pediatrics. and the association is dramatically increased if the child’s mother had pyloric stenosis. The age at presentation has decreased over the last 20 years from a mean age of 5 weeks to 3 weeks. eMedicine website. Her most recent hemoglobin is 4 mg/dL. On review of systems she reports occasional headaches and epigastric pain. Indications for splenectomy in this child would include (A) hemoglobin < 6 mg/dL (B) transfusion requirement (C) presence of cholelithiasis (D) reticulocyte > 10 % (E) bilirubin > 2 mg/dL page 919 . Her parents report that she became ill after having flushed cheeks for several days. Blood smear suggests spherocytosis. Warm and cold antibody testing is negative.American College of Surgeons SESAP 13 Category 10 Item 2 A 12-year-old girl is admitted with anemia and splenomegaly. but timing is an important consideration because of the risk of overwhelming postsplenectomy infection (OPSI). and splenomegaly. and reticulocyte counts (see table). More recently. 2004 2. splenectomy should be delayed until after the age of 6. Mild leukopenia and thrombocytopenia may also be present. The virus causes temporary red cell aplasia. In such cases. new guidelines. and has settled into a stable state.Bolton-Maggs PH: The diagnosis and management of hereditary spherocytosis. HS can be divided into mild.Bolton-Maggs PH: Hereditary spherocytosis. cholecystectomy with splenectomy should be considered in patients with HS and asymptomatic cholelithiasis that develops within the first decade. REFERENCES 1. and all patients should have preoperative vaccination against encapsulated organisms at least 2 weeks prior to operation. Mild disease does not usually require surgical treatment. The presence of symptomatic gallstones. which manifests the underlying hemolysis due to HS. Additionally. If possible. evidence of spherocytosis on blood smear. Initial transfusion support may be required. The severity of disease can be a useful guide. and documentation of RBC osmotic fragility. the eosin-5maleimide (EMA) binding test has shown promise as an effective means of diagnosing HS that is less time-consuming than checking for osmotic fragility. Severity should not be determined until the patient has recovered from any acute presentation. By age 13. should prompt concomitant cholecystectomy with splenectomy. like parvovirus infection. It is the most common hemolytic disorder in Northern Europe. Splenectomy is curative. diagnosis often occurs after parvovirus infection. patients with moderate and severe HS should undergo splenectomy. and severe states based on the degree of anemia. Traditionally. 2000 page 920 . The destabilized red cell membrane gives it a characteristic spherical appearance and substantially decreases its lifespan. Patients are also prone to the development of pigment stones within the gallbladder. however. up to 94% of patients with HS will develop cholelithiasis. moderate. bilirubin levels.American College of Surgeons SESAP 13 Critique for Category 10 Item 2 Hereditary spherocytosis (HS) is a heterogeneous genetic disorder resulting in chronic hemolysis secondary to destabilization of the red cell membrane. diagnosis of HS was made based on family history. The ensuing chronic hemolysis with increased serum bilirubin levels results in the characteristic features of the disorder: anemia. Mild disease can often go undiagnosed for many years. jaundice. but recovery occurs once the infection runs its course. having a prevalence of 1 in 2000. Baillieres Best Pract Res Clin Haematol 13:327-342. In general. Arch Dis Child 89:809-812. The laparoscopic approach should be considered in patients requiring operation. and hemoglobin levels drop substantially. Which of the following would be most accurate in making the diagnosis? (A) Abdominal ultrasound study (B) Upper gastrointestinal contrast study (C) Abdominal and pelvic computed tomographic (CT) scan (D) Abdominal x-rays (E) Barium enema page 921 . The infant had been doing well at home until today. when she became lethargic and began vomiting greenish material in a projectile fashion.American College of Surgeons SESAP 13 Category 10 Item 3 A 3-week-old infant girl is brought to the emergency department with lethargy and vomiting. small bowel gas may be confined to the right side.American College of Surgeons SESAP 13 Critique for Category 10 Item 3 Malrotation includes a number of midgut congenital abnormalities resulting from improper rotation and fixation during gestation and occurs at a rate of 1 in 500 live births. or Z-shaped course of the distal duodenum and proximal jejunum. The mesenteric base is broadened by sharply dividing the adhesive bands connecting the cecum. McGraw-Hill.Prasil P. and immediate diagnostic imaging is indicated for any infant with bilious vomiting to determine whether malrotation is present. False-negative examinations occur in up to 20% of patients with malrotation. but this finding is neither sensitive nor specific.Strouse PJ: Disorders of intestinal rotation and fixation (“malrotation”). Although abdominal plain films. This altered development typically results in the creation of a narrowed mesenteric base capable of causing midgut volvulus. if present. In one study. Occasionally. computed tomography (CT). Andersen DK. et al (eds): Schwartz’s Principles of Surgery. CT may also show an abnormal relationship between the SMA and SMV. New York. and abdominal wall. BE was classically used to diagnose malrotation through the demonstration of abnormal positioning of the cecum. Newman K. Pediatr Radiol 34:837-851. Dunn DL. An UGI remains the study of choice for diagnosis of malrotation. 2004 page 922 . 2000 3. and may demonstrate malpositioned bowel. prompting further work-up. Midgut volvulus is a life-threatening condition that requires prompt diagnosis and operation. ed 8. Nguyen LT. Volvulus typically reveals a downward spiraling of the distal duodenum and proximal jejunum ending in a bird’s-beak narrowing. These findings are often noted incidentally on work-up of older infants and occasionally adults for other complaints. corkscrew. colon. All of the small intestine is left on the right side of the abdomen and all of the colon on the right side. however. and right-sided location of the proximal jejunum. Shaw KS. Plain films rarely are suggestive of malrotation. since the cecum will be positioned in the left upper. Ford HR: Pediatric surgery. Bilious vomiting is the hallmark presentation in newborns. 2004. Billiar TR. pp 14711518 2. a spiral. Inversion of the relationship between the superior mesenteric artery (SMA) and superior mesenteric vein (SMV) on ultrasonography is suggestive of malrotation. and barium enema (BE) can demonstrate findings suggestive of malrotation. Imaging findings that indicate malrotation include abnormal positioning of the duodenojejunal junction. et al: Should malrotation in children be treated differently according to age? J Pediatr Surg 35:756-758. in Brunicardi FC. it correctly diagnosed 94% of cases. up to 80% of cases occur in the newborn period. ultrasonography. Although malrotation carries a lifetime risk of developing volvulus.Hackam DJ. and a Ladd procedure. an upper gastrointestinal contrast study is the preferred modality. Operative therapy involves counterclockwise untwisting of a volvulus. Flageole H. Appendectomy completes the operation. because the bowel gas pattern often appears normal. REFERENCES 1. and dark. gelatinous stool.American College of Surgeons SESAP 13 Category 10 Item 4 A 14-month-old girl presents with a 1-day history of lethargy. emesis. She has a moderately distended abdomen without focal tenderness on examination. The most likely diagnosis is (A) intestinal malrotation with midgut volvulus (B) distal intestinal web (C) Hirschsprung’s disease (D) intussusception (E) foreign body ingestion with mechanical obstruction page 923 . The plain films shown are obtained. a pathologic lead point such as Meckel’s diverticulum. Intussusception is diagnosed by ultrasound or contrast enema and may be treated successfully in at least 50% of cases with careful hydrostatic reduction using air or contrast under fluoroscopic guidance. the contrast enema will be negative for intussusception and the cecum may be abnormally positioned. Failure of hydrostatic reduction is generally managed with operative exploration and manual reduction of the intussusception. REFERENCES 1. Navarro O: Intussusception. 2004.American College of Surgeons SESAP 13 Critique for Category 10 Item 4 The most common cause of small bowel obstruction in children 6 months to 2 years of age is intussusception. although a subset of patients with some movement of the intussusceptum on initial study might have a repeat attempt at hydrostatic reduction 2-4 hours after the first attempt. malrotation with volvulus may be confirmed with an upper gastrointestinal contrast study. In this setting. ed 17. particularly in neonates and children under 12 months of age. In older children and adults presenting with intussusception.Daneman A. most commonly the ileum into the cecum. polyp. Part 1: a review of diagnostic approaches. The cause in this age group is unknown. Philadelphia. The differential diagnosis includes intestinal malrotation with midgut volvulus. Pediatr Radiol 33:79-85. Infants and children with peritonitis secondary to suspected intussusception should be taken to the operating room after fluid resuscitation without attempts at contrast enema.Warner BW: Pediatric surgery. pp 2097-2134 page 924 . 2003 2. Distal intestinal webs and Hirschsprung’s disease typically become symptomatic at birth. although lymphoid hypertrophy is usually observed. or telescoping of the bowel onto itself. in Townsend CM Jr (ed): Sabiston Textbook of Surgery: The Biological Basis of Modern Surgical Practice. or metastases is more commonly seen. Saunders. Foreign body ingestion causing mechanical intestinal obstruction is rare in this age group. American College of Surgeons SESAP 13 Category 10 Item 5 Neonatal Hirschsprung’s disease is associated with all of the following EXCEPT (A) failure to pass meconium in the first 24 hours of life (B) dilated proximal bowel on prenatal ultrasound study (C) feeding intolerance (D) higher incidence with trisomy 21 (E) male predominance page 925 . and failure to pass meconium within the first 24 hours of life due to the distal aganglionic segment of bowel. abdominal distention. REFERENCES 1. Amsterdam. intestinal atresia. obstructive plain films despite a patent gastrointestinal tract. the disease is characterized by feeding difficulty. Hirschsprung’s disease is typically diagnosed during the first few days of life. et al: Diagnosing Hirschsprung’s disease: increasing the odds of a positive rectal biopsy result. Harwood Academic Publishers. 2003 2. or meconium ileus secondary to cystic fibrosis. Levitt MA. A transanal rectal suction biopsy of the distal rectal mucosa and submucosa can be performed at the bedside.Meier-Ruge W: Histological diagnosis and differential diagnosis. Zafar MS. J Pediatr Surg 38:412-416. Diagnosis is generally made by contrast enema demonstrating a transition point between the contracted distal bowel and the normal dilated proximal intestine. 2004 page 926 . There is a notable male predominance and a higher incidence of Hirschsprung’s disease in children with trisomy 21. pp 252-264 3. Aganglionosis causes muscular contraction of the involved segment. Zallen GS. in Holschneider AM. Puri P (eds): Hirschsprung's Disease and Allied Disorders. J Pediatr Surg 39:1449-1453.American College of Surgeons SESAP 13 Critique for Category 10 Item 5 Hirschsprung’s disease is defined as a congenital absence of ganglion cells in the myenteric and submucosal plexuses of the intestinal tract. causing the normal proximal bowel to become dilated secondary to a functional obstruction. most commonly ending in the sigmoid colon. Fetal ultrasound study has not proven useful in the prenatal diagnosis of Hirschsprung’s disease.Lewis NA. Clinically.Swenson O: Hirschsprung's disease—a complicated therapeutic problem: some thoughts and solutions based on data and personal experience over 56 years. 2000. Dilated proximal bowel on prenatal ultrasonography is consistent with imperforate anus. and the absence of submucosal ganglion cells with nerve fiber hypertrophy will confirm the diagnosis. The aganglionic segment involves the distal rectum and extends proximally for variable distance. He has a palpable midepigastric mass. and progressive postprandial nonbilious emesis. The most likely diagnosis is (A) duodenal atresia (B) annular pancreas (C) intestinal duplication cyst (D) neuroblastoma (E) hypertrophic pyloric stenosis page 927 .American College of Surgeons SESAP 13 Category 10 Item 6 A 6-week-old infant presents with failure to thrive. dehydration. Lippincott Williams & Wilkins. reflecting gaseous dilatation of the stomach and proximal duodenum. Neuroblastomas generally arise in the adrenal gland or paravertebral sympathetic chain ganglia and may present as a palpable abdominal mass. because hypochloremic. ed 4. Duodenal atresia and annular pancreas typically present with feeding intolerance in the first few hours to days of life. Van Der Zee J. Ann Surg 240:774-778. metabolic alkalosis is common and requires adequate fluid resuscitation with saline prior to pyloromyotomy. however.Sato TT. Lillemoe KD. 2004 2. failure to thrive. The pyloromyotomy may be performed either open or via a laparoscopic approach.to 2-month-old infant is characteristic of hypertrophic pyloric stenosis. Philadelphia. 2006. Doherty GM. pp 1878-1944 page 928 . A hallmark of duodenal atresia is a double-bubble sign on plain abdominal film.Hall NJ. In this setting. Intestinal duplications are rare and typically difficult to palpate given the propensity to be cystic. Maier RV. et al (eds): Greenfield’s Surgery: Scientific Principles and Practice. and a palpable midepigastric mass in a 1. the only further diagnostic work-up required is serum electrolyte determination.American College of Surgeons SESAP 13 Critique for Category 10 Item 6 Progressive postprandial nonbilious emesis. REFERENCES 1. Tan HL Pierro A: Meta-analysis of laparoscopic versus open pyloromyotomy. hypokalemic. the tumors do not typically cause symptoms of gastric outlet obstruction. Both duodenal atresia and annular pancreas are treated by duodenoduodenostomy. dehydration. Oldham KT: Pediatric abdomen. in Mulholland MW. American College of Surgeons SESAP 13 Category 10 Item 7 The diagnosis for the neonate shown is (A) imperforate anus (B) pilonidal cyst (C) myelomeningocele (D) uterine prolapse (E) sacrococcygeal teratoma page 929 . Most neonatal SCTs are benign. Tandon RK. Philadelphia. diagnostic imaging with either helical computed tomographic (CT) scan or magnetic resonance imaging (MRI) is extremely useful in preoperative planning for definitive resection. Skinner MA (eds): Principles and Practice of Pediatric Surgery. Complete resection is the surgical goal. REFERENCES 1. 2002 page 930 . Pediatr Surg Int 18:384-387. Neonatal SCT is more common in females and typically presents as a presacral mass. Lippincott Williams & Wilkins. Malignant elements such as endodermal sinus tumors or embyronal carcinoma are more common in older children.Wakhlu A. This lesion is located in the posterior lumbar vertebral bodies and does not descend to the sacrum.American College of Surgeons SESAP 13 Critique for Category 10 Item 7 This newborn has a sacrococcygeal teratoma (SCT). Wakhlu AK: Sacrococcygeal teratoma. 2005. although some infants may have small epithelial pilonidal clefts or blind-ending tracts. Because some tumors will have dramatic intrapelvic and intra-abdominal extension.Skinner MA: Germ cell tumors. Imperforate anus is characterized by either an absent anal opening or a fistula to the perineum or vagina and is not associated with a sacral mass. Foglia RP. These tumors are comprised of the three embryonic germ layers and the tissue is foreign to the anatomic site. pp 637-648 2. Uterine prolapse is not a neonatal condition and is most commonly observed in multiparous women with laxity of the pelvic musculature. which may require a combined abdominal and presacral approach with early vascular control of the middle sacral artery. Misra S. Myelomeningocele is an incomplete meningeal and dermal covering of the spinal cord in association with a spinal skeletal defect. The coccyx must be resected because it contains tumor elements. ed 2. Follow-up for both benign and malignant SCT is essential because of the risk for local recurrence. Colombani PM. which may be either benign or malignant. in Oldham KT. Pilonidal cystic disease is extremely unlikely in the neonate. It has continued to increase in size over the last 4 months.American College of Surgeons SESAP 13 Category 10 Item 8 The lesion shown appeared in the child shown when she was 2 months old. The most appropriate therapy would be (A) multiple pulse dye laser treatments (B) excision and flap closure (C) local corticosteroid injection (D) observation (E) subcutaneous injection of interferon alfa 2 page 931 . Although hemangiomas may present in a number of ways. Esterly NB. but not in lesions of significant volume because of its shallow depth of penetration. while smaller. occurring in 5% to 10% of children 1-year-old and younger. N Engl J Med 341:173-181. flatter lesions may disappear completely. J Pediatr 128:329-335. Enjolras O. Immediate operation is indicated for ulceration and recurrent bleeding. Because the extent of operation and reconstruction required for the patient pictured would leave significant deformity. at that time. Alfa interferon injection is also effective but has multiple toxicities and should be reserved for life-threatening situations such as airway hemorrhage. 1997 2. obstruction of visual axis. Frieden IJ: Hemangiomas in children. 1996 3. if any. Pulsed dye laser with particular affinity for the pigment hemoglobin is useful in superficial vascular malformations. operation should be performed before the child reaches school age. Most hemangiomas will involute significantly if not completely by age 5. Chang CJ. Plast Reconstr Surg 99:1301-1308. Vander Kam.Drolet BA. waiting for involution and dealing with the surgical deformity. VM: Management of hemangioma of infancy: review of 245 patients. Mulliken JB: Congenital hemangioma: evidence of accelerated involution. which is the most common soft tissue tumor of childhood. Larger lesions may leave residual excess skin and stroma causing some deformity. Operation is effective therapy for hemangioma.Boon LM. REFERENCES 1. and other serious complications. but rarely resolve the problem completely and may result in atrophy of the overlying skin structures and further deformity. losing volume and color. parents typically note a small red freckle or nothing at birth followed by a relatively rapid growth that usually plateaus over several months. Local or systemic corticosteroids are somewhat effective.American College of Surgeons SESAP 13 Critique for Category 10 Item 8 This child has a hemangioma. observation would be the most appropriate therapy.Achauer BM. 1999 page 932 . but should be weighed against observation and waiting for involution. If involution does not occur. The most likely diagnosis is (A) epididymidis (B) testicular torsion (C) torsion of an appendix testis (D) orchitis (E) testicular tumor page 933 . now associated with mild hemi scrotal swelling and tenderness.American College of Surgeons SESAP 13 Category 10 Item 9 An 11-year-old boy presents with a 2-day history of increasing right scrotal pain. pp 1093-1124 3. there is local swelling and often an associated scrotal hydrocele. There may be considerable delay before the child is brought to the emergency department. generally because the symptoms do not subside. in Grosfeld JL. and more susceptible to torsion. Appendixes of the testis and epididymis are often found at elective inguinal hernia repair and can be removed. Because many are mullerian duct remnants. Because the small appendix infarcts. as the mullerian remnants become swollen and top heavy. O’Neill JA Jr. AJR Am J Roentgenol 184:1287-1292. Fonkalsrud EW.Baldisserotto M. torsion. Treatment consists of analgesics. 2002. If the blue dot sign is not present. Dora MD: Color Doppler sonography of normal and torsed testicular appendages in children.Hutson JM: Undescended testis. making them more prone to torsion and infarction. ed 4. pp 1193-1214 page 934 . A careful examination may reveal a firm. If there is fluid in the hemiscrotum. pea-sized nodule in the upper scrotum attached to the epididymis or testis. the peak incidence for the clinical presentation of a torsed appendix is just before puberty. 2005 2. transillumination may reveal a blue dot sign. Kramer SA (eds): Clinical Pediatric Urology. King LR. Testicular torsion generally causes significant pain that prompts an immediate need for medical attention. Philadelphia. Pertence AP.American College of Surgeons SESAP 13 Critique for Category 10 Item 9 Mullerian and mesonephric vestiges are common on both the testis and epididymis. Orchitis and testicular tumors are relatively rare at this age. REFERENCES 1. and varicocele. London. de Souza JC. Martin Dunitz Ltd. 2006. Epididymitis is unusual in prepubertal boys.Barthold JS. exquisitely tender. it is safest to obtain a Doppler ultrasound of the testis to make sure that the testis is not torsed. as the most common etiology is a sexually transmitted disease. although it usually is of gradual onset and symptoms are often relatively mild. coinciding with a physiologic estrogen surge. The natural history is resolution of the signs and symptoms within 7 to 14 days of the event. Many of these appendices are pedunculated. Pain is the common presenting symptom. ed 6. in Belman AB. Kass EJ: Abnormalities of the penis and scrotum. Coran AG (eds): Pediatric Surgery. Mosby. American College of Surgeons SESAP 13 Category 10 Item 10 A 1-day old infant has repetitive bilious emesis on initial feeding attempts. An x-ray is obtained. The most likely diagnosis is (A) duodenal atresia (B) jejunoilieal atresia (C) meconium ileus (D) meconium plug syndrome (E) Hirschsprung’s disease page 935 . as shown. which is usually due to cystic fibrosis. The etiology of jejunal and ileal atresias. is a developmental vascular accident to the intestine. which is most consistent with the diagnosis of atresia or meconium ileus. Philadelphia. characterized by a double-bubble sign.Klein MD: Congenital defects of the abdominal wall. Coran AG (eds): Pediatric Surgery. Mosby. Fonkalsrud EW. in Grosfeld JL. but there are often air fluid levels with a soap bubble appearance. ed 6.American College of Surgeons SESAP 13 Critique for Category 10 Item 10 The x-ray shows a complete intestinal obstruction in the upper-to-mid small bowel. Hirschsprung’s disease is associated with colonic atresia. Philadelphia. pp 1157-1171 3. Coran AG (eds): Pediatric Surgery. in Grosfeld JL. O’Neill JA Jr. Meconium ileus. O’Neill JA Jr. and there may be extra-intestinal calcifications if the bowel is perforated.Ziegler MM: Meconium ileus. REFERENCES 1. causes a more distal intestinal obstruction due to inspissated meconium. Hirschsprung’s disease causes a functional intestinal obstruction due to the absence of ganglion cells in the myenteric plexus. Meconium plug syndrome is a condition that causes a distal intestinal obstruction in the colon due to inspissated stool. there is an unused colon. Fonkalsrud EW. ed 6. pp 1289-1303 page 936 . ed 6. It may be isolated or associated with cystic fibrosis or Hirschsprung’s disease. The condition is less likely to be associated with other congenital anomalies. O’Neill JA Jr. 2006. 2006. Duodenal atresia. in Grosfeld JL. has an association with both Down’s syndrome and congenital heart disease. Mosby. pp 1269-1287 2. but in either case. 2006. Mosby. In this case. Philadelphia. Occasionally. the x-ray would reflect a more distal obstruction.Grosfeld JL: Jejunoileal atresia and stenosis. The rectosigmoid colon is the most commonly involved site. which is the most common condition exemplified by this radiographic appearance. A contrast colon study is usually done in infants with suspected intestinal obstruction to aid in the diagnosis. Fonkalsrud EW. Coran AG (eds): Pediatric Surgery. American College of Surgeons SESAP 13 Category 10 Item 11 Which of the following stimuli in hospitalized infants MOST affects subsequent pain response? (A) Circumcision with analgesia (B) Major neonatal surgery with postoperative analgesia (C) Repeated heel lancing for blood tests (D) Immunizations (E) Venipuncture page 937 . JAMA 290:2464-2469.Acharya AB. Bustani PC. Infants who have repeated heel lancing have behavioral and other effects that outlast the painful stimulus by hours or days. 1998 2. Gerstmann DR: Treatment of neonatal pain without a gold standard: the case for caregiving interventions and sucrose administration. 1997 page 938 . et al: Randomised controlled trial of eutectic mixture of local anesthetics cream for venepuncture in healthy preterm infants. Those who had circumcision with analgesia are far less likely to have a noxious response to later immunizations. and whether analgesia was administered. has little effect on subsequent pain response in infancy.Howard RF: Current status of pain management in children. Neonatal Netw 23:33-45. 2004 4. REFERENCES 1. with effective administration of postoperative analgesia.Taddio A. 2003 3. Lancet 349:599-603. Long-term differences have been noticed in subsequent pain behavior between infants who had circumcision as neonates with or without analgesia. Katz J. Phillips JD. The most important determinants of the long-term outcome of infant pain include timing. Major surgery in the neonatal period. Ilersich A.American College of Surgeons SESAP 13 Critique for Category 10 Item 11 The experience of pain in early life may lead to long-term consequences. Latta TM. Venipuncture is a much less common method of obtaining routine blood work in infants than heel lancing.Prince WL. Koren G: Effect of neonatal circumcision on pain response during subsequent routine vaccination. degree of injury. Arch Dis Child Fetal Neonatal Ed 78:F138-F142. Taub NA. Horns KM. American College of Surgeons SESAP 13 Category 10 Item 12 Which of the following medical conditions has NOT been implicated as a reason for routine circumcision? (A) HIV transmission (B) Urinary tract infection (C) Chlamydia infection (D) Cervical carcinoma (E) Penile carcinoma page 939 . Hutson JM: Circumcision: a surgeon’s perspective. which results in a reduced risk of cervical cancer in the partners of these men. Bosch FX. there is good concordance for an association between many of the ulcerative sexually transmitted diseases including chancroid. et al: Male circumcision. For chlamydial. Although male circumcision is practiced in many parts of Africa. and gonorrhea.Adelman WP. 2004 6. The International Agency for Research on Cancer has performed several large case-control studies of cervical cancer in different countries. N Engl J Med 346:1105-1112. REFERENCES 1. there is wide regional variation. and testicular self-examination. thus providing an entry site as well as route for future transmission of the virus. penile human papillomavirus infection. the evidence for effectiveness of circumcision is inconclusive. circumcision. or other types of urethritis. and cervical cancer in female partners. Munoz N. AIDS 14:2361-2370. estimated at < 1%. Sex Transm Infect 74:368-373. 1998 5.Weiss HA. 2004 2. Several studies link both penile intraepithelial neoplasia and penile carcinoma with the uncircumcised state. whereas there is no scientific evidence that improved hygiene reduces the risk for penile carcinoma among uncircumcised men.Short RV: Male circumcision: a scientific perspective. Joffe A: Controversies in male adolescent health: varicocele. Hayes RJ: Male circumcision and risk of HIV infection in sub-Saharan Africa: a systematic review and meta-analysis.Castellsagué X.American College of Surgeons SESAP 13 Critique for Category 10 Item 12 Much of the controversy surrounding newborn circumcision relates to the immediate risks and benefits of an elective procedure weighed against potential long-term benefits. non-gonococcal.Moses S. 2004 4. 2000 page 940 . genital herpes. J Med Ethics 30:241. Although there is some inconsistency among studies. Ronald AR: Male circumcision: assessment of health benefits and risks. There is evidence that newborn circumcision decreases the subsequent risk of urinary tract infection from 7:1000 to < 2:1000. J Med Ethics 30:238-240. syphilis. and the lack of circumcision. Curr Opin Pediatr 16:363-367. Bailey RC. Male circumcision is associated with a reduced risk of penile human papilloma virus infection. male circumcision is associated with a significantly reduced risk of HIV infection among men. Meijer CJ. Quigley MA. The HIV virus can bind to receptors on the inner layer of the foreskin. In sub-Saharan Africa. 2002 3. The risks of circumcision in the neonatal period are low. American College of Surgeons SESAP 13 Category 10 Item 13 Primary sclerosing cholangitis in children (A) is associated with a viral infection (B) has a high rate of treatment success using ursodeoxycholic acid alone or with immunosuppressive therapy (C) is usually associated with inflammatory bowel disease (D) is usually associated with autoimmune hepatitis (E) is cured by liver transplantation page 941 . Three patients developed recurrence of PSC in their allograft at 2.and sex-matched US population. REFERENCES 1. Lindor KD.Pall H. Hepatology 38:210-217. In a recent longitudinal-cohort study to determine the long-term outcome of children with PSC. Perrault J. survival was significantly shorter in children with PSC.Feldstein AE. Whether PSC occurring in childhood is the same disease process as PSC developing in adulthood is uncertain. Compared with an age. 81% had concomitant inflammatory bowel disease. El-Youssif M. Autoimmune hepatitis overlapping with PSC was present in 35%. Many went on to require liver transplantation for end-stage PSC. 2003 2. 2005 page 942 . and 6 years after transplant. 3.3. Jonas MM: Pediatric hepatobiliary disease.8. alone or in combination with immunosuppressive medications. characterized by fibro-obliterative inflammation of the biliary tract.American College of Surgeons SESAP 13 Critique for Category 10 Item 13 Primary sclerosing cholangitis (PSC) is a slowly progressive cholestatic liver disease of unknown etiology. et al: Primary sclerosing cholangitis in children: a long-term follow-up study. Curr Opin Gastroenterol 21:344-347. A positive but transient clinical and/or biochemical response occurred under therapy with ursodeoxycholic acid. as pictured. There is no history of fever and the WBC count is normal.American College of Surgeons SESAP 13 Category 10 Item 14 A 5-month-old infant presents with a 2-day history of perianal swelling and erythema. The most appropriate first step in management would be (A) anoscopy (B) incision and drainage (C) admission and intravenous antibiotics (D) rectal ultrasonography (E) colon contrast study page 943 . or an immune deficiency. the fistula connects to an anal crypt and can be identified by injecting dilute hydrogen peroxide or methylene blue. Diagnostic tests are unnecessary.Serour F. Protracted courses are not indicated.Oh JT. Antibiotics may be considered for patients undergoing drainage of a perianal abscess. The condition is not related to other disorders such as Crohn’s or Hirschsprung’s disease. Although many pediatric surgeons favor operative management. Several hypotheses for the etiology have been proposed. Han SJ. 2001 2. including an infected abnormal anal gland or anal fissure. particularly in small infants who are at great risk for bacteremia. and entrapment of migratory cells from a congenital sinus or the hindgut. Most infants with this disorder are not systemically ill at presentation. et al: Fistula-in-ano in infants: is nonoperative management effective? J Pediatr Surg 36:1367-1369. REFERENCES 1. although the pathogenesis is not fully understood.American College of Surgeons SESAP 13 Critique for Category 10 Item 14 Fistula-in-ano is a relatively common condition in infants. 2005 page 944 . The condition is more common in male infants. and the fistula tracts are superficial to the sphincter complex. Choi SH. This is a congenital and isolated condition. Han A. Treatment of the initial abscess is by incision and drainage or aspiration. and that the condition is self-limited. there are studies suggesting that nonoperative management is safe and effective. and it is rare to see a complex fistula or recurrence after surgical management. Typically. A lacrimal duct probe can then be inserted and the cautery used to open the tract and obliterate the small sinus tract. dermal infection. Somekh E. A persistent fistula is managed operatively with fistulectomy and cryptotomy. Gorenstein A: Perianal abscess and fistula-in-ano in infants: a different entity? Dis Colon Rectum 48:359-364. American College of Surgeons SESAP 13 Category 10 Item 15 A 5-month-old infant presents with a 2-day history of perianal swelling and erythema. the infant has intermittent swelling and purulent drainage from the same location. There is no history of fever and the WBC count is normal. The diagnosis is (A) Crohn’s disease (B) congenital fistula-in-ano (C) recurrent perirectal abscess (D) immunodeficiency disorder (E) Hirschsprung’s disease page 945 . At age 7 months. as pictured. Oh JT. Typically. Gorenstein A: Perianal abscess and fistula-in-ano in infants: a different entity? Dis Colon Rectum 48:359-364. dermal infection. including an infected abnormal anal gland or anal fissure. Several hypotheses for the etiology have been proposed. et al: Fistula-in-ano in infants: is nonoperative management effective? J Pediatr Surg 36:1367-1369.Serour F. The condition is more common in male infants. Most infants with this disorder are not systemically ill at presentation. A lacrimal duct probe can then be inserted and the cautery used to open the tract and obliterate the small sinus tract. Han SJ. although the pathogenesis is not fully understood. Antibiotics may be considered for patients undergoing drainage of a perianal abscess. and that the condition is self-limited. or an immune deficiency. and the fistula tracts are superficial to the sphincter complex. the fistula connects to an anal crypt and can be identified by injecting dilute hydrogen peroxide or methylene blue. This is a congenital and isolated condition. A persistent fistula is managed operatively with fistulectomy and cryptotomy. 2005 page 946 . Treatment of the initial abscess is by incision and drainage or aspiration. Diagnostic tests are unnecessary. particularly in small infants who are at great risk for bacteremia.American College of Surgeons SESAP 13 Critique for Category 10 Item 15 Fistula-in-ano is a relatively common condition in infants. and entrapment of migratory cells from a congenital sinus or the hindgut. there are studies suggesting that nonoperative management is safe and effective. Choi SH. The condition is not related to other disorders such as Crohn’s or Hirschsprung’s disease. Han A. Protracted courses are not indicated. Although many pediatric surgeons favor operative management. REFERENCES 1. Somekh E. 2001 2. and it is rare to see a complex fistula or recurrence after surgical management. American College of Surgeons SESAP 13 Category 10 Item 16 Nonbilious emesis (A) Duodenal atresia (B) Pyloric stenosis (C) Jejunal-ileal atresia (D) Meconium ileus (E) Imperforate anus page 947 . Seventy percent of infants with anorectal malformations have associated anomalies. et al (eds): Greenfield's Surgery: Scientific Principles and Practice. The Christmas tree or apple-peel deformity is a rare variant where the superior mesenteric artery fails to form and the entire small bowel receives its blood supply from the ileocolic vessel. obstructed duodenum—the classic double-bubble. Saunders. 2006. Cardiac lesions. progressing to projectile. Tracheo-esophageal fistula. Anorectal malformations. REFERENCES 1. Intestinal atresias occur most commonly in the distal ileum. Doherty GM. Oldham KT: Pediatric abdomen. in Mulholland MW. infants present with abdominal distention. Polyhydramnios indicates the potential for this anomaly. Hypertrophic pyloric stenosis usually begins subtly as a feeding disorder. bilious vomiting.American College of Surgeons SESAP 13 Critique for Category 10 Item 16 Intestinal obstruction may occur from a variety of causes in the first month of life. as shown in the table. identified by the acronym VACTERL (or VATER): Vertebral dysplasia or agenesis. pp 1878-1944 2. Plain abdominal x-rays show an air-fluid level in the stomach. with spitting-up and vomiting. which can be diagnosed by maternal ultrasound. in Townsend CM Jr (ed): Sabiston Textbook of Surgery: The Biological Basis of Modern Surgical Practice. ed 4. depending on the location of the obstruction. Lillemoe KD. Maier RV. They may vomit bile-stained or nonbilious fluid. Philadelphia. Lippincott Williams & Wilkins. Renal or lower urogenital anomalies. and another smaller air-fluid level in the dilated. and Limb abnormalities. ed 17.Sato TT. Infants with duodenal atresia have a scaphoid abdomen. and 90% are complete (10% may have a stenosis or semiperforated web). and failure to pass meconium. Physical examination will reveal an olive-sized tumor in the right upper quadrant. Pyloromyotomy is the treatment of choice. because delay may be life-threatening. Postpartum.Warner BW: Pediatric surgery. Philadelphia. Atresias of the jejunum and ileum are classified into four categories. are unable to tolerate any oral intake. pp 2097-2134 page 948 . and are rarely discharged after birth unless they have a web variant with a perforated septum and patent GI tract. nonbilious emesis. 2004. Prompt diagnosis and treatment is vital. Full development of the typical clinical syndrome usually takes 3 to 6weeks. American College of Surgeons SESAP 13 Category 10 Item 17 Double bubble (A) Duodenal atresia (B) Pyloric stenosis (C) Jejunal-ileal atresia (D) Meconium ileus (E) Imperforate anus page 949 . Anorectal malformations. The Christmas tree or apple-peel deformity is a rare variant where the superior mesenteric artery fails to form and the entire small bowel receives its blood supply from the ileocolic vessel. infants present with abdominal distention. Philadelphia. They may vomit bile-stained or nonbilious fluid. 2004. with spitting-up and vomiting.Warner BW: Pediatric surgery. nonbilious emesis. Saunders. depending on the location of the obstruction. progressing to projectile. and Limb abnormalities. Pyloromyotomy is the treatment of choice. Physical examination will reveal an olive-sized tumor in the right upper quadrant. as shown in the table. Polyhydramnios indicates the potential for this anomaly. in Townsend CM Jr (ed): Sabiston Textbook of Surgery: The Biological Basis of Modern Surgical Practice. Hypertrophic pyloric stenosis usually begins subtly as a feeding disorder. Cardiac lesions. in Mulholland MW. which can be diagnosed by maternal ultrasound. et al (eds): Greenfield's Surgery: Scientific Principles and Practice. Philadelphia. Lippincott Williams & Wilkins. and failure to pass meconium. Seventy percent of infants with anorectal malformations have associated anomalies. Maier RV. Tracheo-esophageal fistula. obstructed duodenum—the classic double-bubble. Atresias of the jejunum and ileum are classified into four categories. and 90% are complete (10% may have a stenosis or semiperforated web). 2006. ed 17. pp 1878-1944 2. Doherty GM. Prompt diagnosis and treatment is vital. are unable to tolerate any oral intake. because delay may be life-threatening. ed 4.Sato TT. Infants with duodenal atresia have a scaphoid abdomen. and are rarely discharged after birth unless they have a web variant with a perforated septum and patent GI tract.American College of Surgeons SESAP 13 Critique for Category 10 Item 17 Intestinal obstruction may occur from a variety of causes in the first month of life. Lillemoe KD. identified by the acronym VACTERL (or VATER): Vertebral dysplasia or agenesis. pp 2097-2134 page 950 . Intestinal atresias occur most commonly in the distal ileum. Full development of the typical clinical syndrome usually takes 3 to 6weeks. Postpartum. Renal or lower urogenital anomalies. bilious vomiting. and another smaller air-fluid level in the dilated. Oldham KT: Pediatric abdomen. REFERENCES 1. Plain abdominal x-rays show an air-fluid level in the stomach. American College of Surgeons SESAP 13 Category 10 Item 18 VACTERL association (A) Duodenal atresia (B) Pyloric stenosis (C) Jejunal-ileal atresia (D) Meconium ileus (E) Imperforate anus page 951 . pp 1878-1944 2. are unable to tolerate any oral intake. bilious vomiting. Atresias of the jejunum and ileum are classified into four categories. Tracheo-esophageal fistula. and failure to pass meconium. REFERENCES 1. progressing to projectile. pp 2097-2134 page 952 . Physical examination will reveal an olive-sized tumor in the right upper quadrant. Renal or lower urogenital anomalies. as shown in the table. in Mulholland MW. 2004. They may vomit bile-stained or nonbilious fluid. Anorectal malformations. ed 17. because delay may be life-threatening. Saunders. 2006. Hypertrophic pyloric stenosis usually begins subtly as a feeding disorder. Prompt diagnosis and treatment is vital.Warner BW: Pediatric surgery. Oldham KT: Pediatric abdomen. and Limb abnormalities. and another smaller air-fluid level in the dilated. ed 4. Pyloromyotomy is the treatment of choice. identified by the acronym VACTERL (or VATER): Vertebral dysplasia or agenesis. Polyhydramnios indicates the potential for this anomaly. Cardiac lesions. Seventy percent of infants with anorectal malformations have associated anomalies. Postpartum. in Townsend CM Jr (ed): Sabiston Textbook of Surgery: The Biological Basis of Modern Surgical Practice. et al (eds): Greenfield's Surgery: Scientific Principles and Practice. which can be diagnosed by maternal ultrasound.Sato TT. Philadelphia. Lippincott Williams & Wilkins. depending on the location of the obstruction. Maier RV. Philadelphia. Full development of the typical clinical syndrome usually takes 3 to 6weeks. infants present with abdominal distention. Lillemoe KD. nonbilious emesis. and are rarely discharged after birth unless they have a web variant with a perforated septum and patent GI tract.American College of Surgeons SESAP 13 Critique for Category 10 Item 18 Intestinal obstruction may occur from a variety of causes in the first month of life. Intestinal atresias occur most commonly in the distal ileum. Infants with duodenal atresia have a scaphoid abdomen. and 90% are complete (10% may have a stenosis or semiperforated web). obstructed duodenum—the classic double-bubble. Plain abdominal x-rays show an air-fluid level in the stomach. Doherty GM. The Christmas tree or apple-peel deformity is a rare variant where the superior mesenteric artery fails to form and the entire small bowel receives its blood supply from the ileocolic vessel. with spitting-up and vomiting. American College of Surgeons SESAP 13 Category 10 Item 19 Christmas tree deformity (A) Duodenal atresia (B) Pyloric stenosis (C) Jejunal-ileal atresia (D) Meconium ileus (E) Imperforate anus page 953 . and 90% are complete (10% may have a stenosis or semiperforated web). identified by the acronym VACTERL (or VATER): Vertebral dysplasia or agenesis. in Mulholland MW. progressing to projectile. which can be diagnosed by maternal ultrasound.American College of Surgeons SESAP 13 Critique for Category 10 Item 19 Intestinal obstruction may occur from a variety of causes in the first month of life. Tracheo-esophageal fistula. Lippincott Williams & Wilkins. Renal or lower urogenital anomalies. ed 17. infants present with abdominal distention. Anorectal malformations. Hypertrophic pyloric stenosis usually begins subtly as a feeding disorder. Doherty GM. and are rarely discharged after birth unless they have a web variant with a perforated septum and patent GI tract. Cardiac lesions. and Limb abnormalities. et al (eds): Greenfield's Surgery: Scientific Principles and Practice. and another smaller air-fluid level in the dilated. and failure to pass meconium. They may vomit bile-stained or nonbilious fluid.Sato TT. Philadelphia. Lillemoe KD. REFERENCES 1. Oldham KT: Pediatric abdomen. Full development of the typical clinical syndrome usually takes 3 to 6weeks. Infants with duodenal atresia have a scaphoid abdomen. Pyloromyotomy is the treatment of choice. bilious vomiting. pp 1878-1944 2. Intestinal atresias occur most commonly in the distal ileum. because delay may be life-threatening. with spitting-up and vomiting. in Townsend CM Jr (ed): Sabiston Textbook of Surgery: The Biological Basis of Modern Surgical Practice. Saunders. Plain abdominal x-rays show an air-fluid level in the stomach. as shown in the table.Warner BW: Pediatric surgery. nonbilious emesis. obstructed duodenum—the classic double-bubble. Physical examination will reveal an olive-sized tumor in the right upper quadrant. Prompt diagnosis and treatment is vital. Philadelphia. Atresias of the jejunum and ileum are classified into four categories. Seventy percent of infants with anorectal malformations have associated anomalies. pp 2097-2134 page 954 . 2004. depending on the location of the obstruction. are unable to tolerate any oral intake. ed 4. Maier RV. Postpartum. 2006. Polyhydramnios indicates the potential for this anomaly. The Christmas tree or apple-peel deformity is a rare variant where the superior mesenteric artery fails to form and the entire small bowel receives its blood supply from the ileocolic vessel. American College of Surgeons SESAP 13 Category 10 Item 20 Chromosomal abnormalities (A) Figure A (B) Figure B (C) Both (D) Neither page 955 . American College of Surgeons SESAP 13 Critique for Category 10 Item 20 Associated anomalies are much more common with omphalocele than with gastroschisis. Lippincott Williams & Wilkins. Lillemoe KD. 2006. Maier RV. 2003 2. et al (eds): Greenfield's Surgery: Scientific Principles and Practice.Langer JC: Abdominal wall defects. These infants are typically born to younger mothers and do not have other associated anomalies. pp 1878-1944 page 956 . infants with omphalocele have a higher incidence of chromosomal abnormalities and other conditions such as Beckwith-Wiedemann syndrome (macroglossia. hyperinsulinism. Infants with omphalocele are at moderate to high risk for anomalies of the skeleton.Sato TT. ed 4. World J Surg 27:117-124. reflecting the more global abnormality of embryogenesis and abdominal wall development in omphalocele compared with the simple mechanical defect in gastroschisis. Infants with gastroschisis are at higher risk (approximately 8% to 15%) for associated intestinal atresia or stenosis given the mechanical compression of the intestine through the right-sided abdominal wall defect. omphalocele). Oldham KT: Pediatric abdomen. Approximately 50% to 60% of infants with omphalocele will have at least one associated congenital anomaly. REFERENCES 1. nervous system. Philadelphia. in Mulholland MW. Some element of intestinal nonrotation is generally seen with both omphalocele and gastroschisis due to the extrusion of the intestine from the peritoneal cavity during fetal development. genitourinary system. gastrointestinal tract. In addition. Doherty GM. and cardiopulmonary system. American College of Surgeons SESAP 13 Category 10 Item 21 Younger maternal age (A) Figure A (B) Figure B (C) Both (D) Neither page 957 . and cardiopulmonary system. These infants are typically born to younger mothers and do not have other associated anomalies. hyperinsulinism. in Mulholland MW. Some element of intestinal nonrotation is generally seen with both omphalocele and gastroschisis due to the extrusion of the intestine from the peritoneal cavity during fetal development. World J Surg 27:117-124.Langer JC: Abdominal wall defects. infants with omphalocele have a higher incidence of chromosomal abnormalities and other conditions such as Beckwith-Wiedemann syndrome (macroglossia. 2003 2. pp 1878-1944 page 958 . Infants with gastroschisis are at higher risk (approximately 8% to 15%) for associated intestinal atresia or stenosis given the mechanical compression of the intestine through the right-sided abdominal wall defect. Doherty GM. 2006. gastrointestinal tract. Maier RV. Approximately 50% to 60% of infants with omphalocele will have at least one associated congenital anomaly. Oldham KT: Pediatric abdomen. Philadelphia. genitourinary system. Infants with omphalocele are at moderate to high risk for anomalies of the skeleton. Lippincott Williams & Wilkins. In addition. ed 4.American College of Surgeons SESAP 13 Critique for Category 10 Item 21 Associated anomalies are much more common with omphalocele than with gastroschisis. et al (eds): Greenfield's Surgery: Scientific Principles and Practice. reflecting the more global abnormality of embryogenesis and abdominal wall development in omphalocele compared with the simple mechanical defect in gastroschisis. REFERENCES 1.Sato TT. nervous system. Lillemoe KD. omphalocele). American College of Surgeons SESAP 13 Category 10 Item 22 Intestinal atresia (A) Figure A (B) Figure B (C) Both (D) Neither page 959 . reflecting the more global abnormality of embryogenesis and abdominal wall development in omphalocele compared with the simple mechanical defect in gastroschisis. Infants with gastroschisis are at higher risk (approximately 8% to 15%) for associated intestinal atresia or stenosis given the mechanical compression of the intestine through the right-sided abdominal wall defect. genitourinary system. Lippincott Williams & Wilkins. nervous system. Lillemoe KD. Infants with omphalocele are at moderate to high risk for anomalies of the skeleton. 2003 2. Some element of intestinal nonrotation is generally seen with both omphalocele and gastroschisis due to the extrusion of the intestine from the peritoneal cavity during fetal development. Philadelphia. World J Surg 27:117-124. in Mulholland MW. hyperinsulinism. and cardiopulmonary system. infants with omphalocele have a higher incidence of chromosomal abnormalities and other conditions such as Beckwith-Wiedemann syndrome (macroglossia. gastrointestinal tract. Maier RV. 2006. Approximately 50% to 60% of infants with omphalocele will have at least one associated congenital anomaly. pp 1878-1944 page 960 . et al (eds): Greenfield's Surgery: Scientific Principles and Practice. In addition. Oldham KT: Pediatric abdomen. omphalocele).Langer JC: Abdominal wall defects. ed 4. REFERENCES 1. These infants are typically born to younger mothers and do not have other associated anomalies.Sato TT. Doherty GM.American College of Surgeons SESAP 13 Critique for Category 10 Item 22 Associated anomalies are much more common with omphalocele than with gastroschisis. American College of Surgeons SESAP 13 Category 10 Item 23 Retrosternal (A) Bochdalek hernia (B) Morgagni hernia (C) Both (D) Neither page 961 . 2006. When conventional resuscitative measures are not successful. Greenburg AG (eds): Nyhus & Condon’s Hernia. Philadelphia. Kim P. J Pediatr Surg 38:1563-1568. including passage of a nasogastric tube to decompress the stomach. Lelli JL Jr. Lippincott. premature return of the intestines to the abdominal cavity interferes with closure of the canal. in Mulholland MW. Barnhart DC. Lippincott Williams & Wilkins. et al (eds): Greenfield's Surgery: Scientific Principles and Practice. a large proportion of bowel enters the thoracic cavity. Dillon PW: Pediatric chest. Doherty GM. Philadelphia. Saunders.Cilley RE. The combination of pulmonary hypoplasia. Because of the potential for incarceration and strangulation. Surgeons are gaining increasing experience repairing both types of lesions with minimally invasive techniques. and is less likely to be associated with a neonatal emergency. ed 4. 2004. pulmonary hypertension. The defect is less common than Bochdalek hernia. and intestinal rotational disorders creates the potential for neonatal respiratory distress. located posterolaterally usually on the left. results from failure of closure of the pleuroperitoneal canal at 8 to 9 weeks of fetal development. pp 467-478 page 962 . Maier RV. Morgagni’s hernia is located anteromedial and retrosternal. Williams & Wilkins. Surgical correction may then be considered. ed 5. and results from failure of fusion between the septum transversum and the sternum or anterior ribs. once discovered it should be repaired. Deans KJ. ed 17. in Townsend CM Jr (ed): Sabiston Textbook of Surgery: The Biological Basis of Modern Surgical Practice. Greenfeld J. extracorporeal membrane oxygenation (ECMO) may allow the pulmonary vascular bed and lungs to mature. Both lesions are preferentially approached from the abdomen. pp 2097-2134 5. et al: Early experience with minimally invasive repair of congenital diaphragmatic hernias: results and lessons learned. and mechanical ventilatory support. Newborns with immediate respiratory distress require aggressive resuscitation. pp 1848-1877 3. 2002. Barotrauma is a significant risk. Lillemoe KD. As a result.Minneci PC.Warner BW: Pediatric surgery. Ann Thorac Surg 77:1956-1959. Ashcraft KW: Congenital diaphragmatic hernia. Mathisen DJ: Foramen of Morgagni hernia: changes in diagnosis and treatment. Philadelphia. preventing proper development of the lung. tracheal intubation. Bochdalek hernia. In theory. 2003 2.Woods RK. It is more often discovered in older children or adults. either as a result of symptoms or as an incidental finding. REFERENCES 1. 2004 4.Arca MJ.American College of Surgeons SESAP 13 Critique for Category 10 Item 23 Bochdalek and Morgagni hernias are both congenital diaphragmatic hernias. in Fitzgibbons RJ Jr. American College of Surgeons SESAP 13 Category 10 Item 24 Posteromedial (A) Bochdalek hernia (B) Morgagni hernia (C) Both (D) Neither page 963 . pp 2097-2134 5. once discovered it should be repaired. 2004 4. REFERENCES 1. pp 1848-1877 3. Surgeons are gaining increasing experience repairing both types of lesions with minimally invasive techniques. and is less likely to be associated with a neonatal emergency. Lillemoe KD. Morgagni’s hernia is located anteromedial and retrosternal. and mechanical ventilatory support. Williams & Wilkins. Lelli JL Jr. Mathisen DJ: Foramen of Morgagni hernia: changes in diagnosis and treatment. Philadelphia. Both lesions are preferentially approached from the abdomen. Philadelphia. 2003 2. Ashcraft KW: Congenital diaphragmatic hernia. and intestinal rotational disorders creates the potential for neonatal respiratory distress. Ann Thorac Surg 77:1956-1959. extracorporeal membrane oxygenation (ECMO) may allow the pulmonary vascular bed and lungs to mature. Kim P. premature return of the intestines to the abdominal cavity interferes with closure of the canal. results from failure of closure of the pleuroperitoneal canal at 8 to 9 weeks of fetal development. et al (eds): Greenfield's Surgery: Scientific Principles and Practice. a large proportion of bowel enters the thoracic cavity. Lippincott. tracheal intubation. pulmonary hypertension. and results from failure of fusion between the septum transversum and the sternum or anterior ribs. either as a result of symptoms or as an incidental finding. 2004. J Pediatr Surg 38:1563-1568. in Townsend CM Jr (ed): Sabiston Textbook of Surgery: The Biological Basis of Modern Surgical Practice. Dillon PW: Pediatric chest. The combination of pulmonary hypoplasia. pp 467-478 page 964 . in Fitzgibbons RJ Jr. ed 17. Lippincott Williams & Wilkins. Saunders. Newborns with immediate respiratory distress require aggressive resuscitation. Greenburg AG (eds): Nyhus & Condon’s Hernia. In theory. Barotrauma is a significant risk. Philadelphia. ed 4. As a result. including passage of a nasogastric tube to decompress the stomach. Because of the potential for incarceration and strangulation.Arca MJ.American College of Surgeons SESAP 13 Critique for Category 10 Item 24 Bochdalek and Morgagni hernias are both congenital diaphragmatic hernias. Doherty GM.Minneci PC. in Mulholland MW.Cilley RE. et al: Early experience with minimally invasive repair of congenital diaphragmatic hernias: results and lessons learned. Surgical correction may then be considered. 2006. preventing proper development of the lung. located posterolaterally usually on the left.Woods RK. The defect is less common than Bochdalek hernia. When conventional resuscitative measures are not successful. 2002.Warner BW: Pediatric surgery. Barnhart DC. Greenfeld J. Deans KJ. Bochdalek hernia. Maier RV. ed 5. It is more often discovered in older children or adults. American College of Surgeons SESAP 13 Category 10 Item 25 Extracorporeal membrane oxygenation (ECMO) (A) Bochdalek hernia (B) Morgagni hernia (C) Both (D) Neither page 965 . either as a result of symptoms or as an incidental finding. Morgagni’s hernia is located anteromedial and retrosternal. Greenfeld J. pp 2097-2134 5. preventing proper development of the lung. REFERENCES 1. ed 17. As a result. Philadelphia. Lelli JL Jr. Kim P. Maier RV. extracorporeal membrane oxygenation (ECMO) may allow the pulmonary vascular bed and lungs to mature. including passage of a nasogastric tube to decompress the stomach. in Mulholland MW. In theory. premature return of the intestines to the abdominal cavity interferes with closure of the canal.Arca MJ. Philadelphia. Barotrauma is a significant risk. pulmonary hypertension. tracheal intubation. and intestinal rotational disorders creates the potential for neonatal respiratory distress. located posterolaterally usually on the left. It is more often discovered in older children or adults.Cilley RE. ed 4. once discovered it should be repaired. and results from failure of fusion between the septum transversum and the sternum or anterior ribs. Because of the potential for incarceration and strangulation. et al (eds): Greenfield's Surgery: Scientific Principles and Practice. Philadelphia.Warner BW: Pediatric surgery. pp 1848-1877 3.Woods RK. Barnhart DC. Mathisen DJ: Foramen of Morgagni hernia: changes in diagnosis and treatment. Lippincott. When conventional resuscitative measures are not successful. Lillemoe KD. Ann Thorac Surg 77:1956-1959. pp 467-478 page 966 . ed 5. Bochdalek hernia. Lippincott Williams & Wilkins. in Fitzgibbons RJ Jr. Deans KJ. Both lesions are preferentially approached from the abdomen. and mechanical ventilatory support. The defect is less common than Bochdalek hernia. Dillon PW: Pediatric chest. et al: Early experience with minimally invasive repair of congenital diaphragmatic hernias: results and lessons learned. The combination of pulmonary hypoplasia. 2004 4. Newborns with immediate respiratory distress require aggressive resuscitation. 2002. 2004. in Townsend CM Jr (ed): Sabiston Textbook of Surgery: The Biological Basis of Modern Surgical Practice. Greenburg AG (eds): Nyhus & Condon’s Hernia.Minneci PC. Saunders. a large proportion of bowel enters the thoracic cavity. and is less likely to be associated with a neonatal emergency. J Pediatr Surg 38:1563-1568. Williams & Wilkins. Ashcraft KW: Congenital diaphragmatic hernia. 2006. Surgical correction may then be considered. Doherty GM.American College of Surgeons SESAP 13 Critique of Category 10 Item 25 Bochdalek and Morgagni hernias are both congenital diaphragmatic hernias. 2003 2. Surgeons are gaining increasing experience repairing both types of lesions with minimally invasive techniques. results from failure of closure of the pleuroperitoneal canal at 8 to 9 weeks of fetal development. American College of Surgeons SESAP 13 Category 11 Item 1 A 70-year-old man presents with dysphagia with intermittent regurgitation of mucoid material. He has lost 35 lb since the onset of his symptoms 3 months ago. The barium swallow shown is obtained. All of the following are pertinent to his work-up EXCEPT (A) esophagoscopy (B) 24-hour pH monitoring (C) esophageal manometry (D) serum albumin (E) chest x-ray page 967 . Br J Surg 88:629-642.Thomas ML.Rosati R. and when dysmotility is present. Bona S. a distal esophageal myotomy is part of the treatment. Bonavina L. Fosh BG. REFERENCES 1. Ann Surg 235:346-354. and esophagoscopy is indicated to exclude these lesions. et al: Physiologic basis for the treatment of epiphrenic diverticulum. and fundoplication through laparoscopy: a new option to treat epiphrenic esophageal diverticula? Ann Surg 227:174178. et al: Diverticulectomy. Patients often lose considerable weight and total parenteral nutrition (TPN) or enteral feeding may be indicated if malnutrition is severe. In a few patients. DeMeester TR. myotomy. dilatation should accompany the diverticulectomy. Diverticulectomy or diverticulopexy is indicated in symptomatic patients. If a stricture or web is present. et al: Oesophageal diverticula. Fumagalli U. Lord RV. If dysmotility exists. diverticulectomy alone will suffice. Aspiration pneumonia may occur in a significant number of patients. Anthony AA.Nehra D. Theisen J. 1998 3.American College of Surgeons SESAP 13 Critique for Category 11 Item 1 This patient presents with a mid-esophageal-epigastric diverticulum. but further therapy is predicated on the manometric and endoscopic findings. 2002 2. distal myotomy is indicated. This can also be caused by distal esophageal web stricture or cancer. Finch JG. 2001 page 968 . These lesions are often associated with esophageal dysmotility. he has decreased breath sounds on the right side.American College of Surgeons SESAP 13 Category 11 Item 2 A 35-year-old airline pilot has acute shortness of breath. On physical examination. Chest x-ray reveals pneumothorax and a chest tube is placed. The most appropriate management now would be (A) serial chest x-rays (B) chemical pleurodesis (C) axillary thoracotomy (D) video-assisted thoracic surgery (VATS) (E) posterolateral thoracotomy page 969 . Treatment should include stapled apical bullectomy with parietal pleural mechanical abrasion of the upper half of the hemithorax. They define clinical stability as a respiratory rate < 24 breaths/min. VATS had a statistically significant lower pneumothorax recurrence rate compared with pleural drainage. heart rate 60 to120 beats/min.American College of Surgeons SESAP 13 Critique for Category 11 Item 2 Spontaneous pneumothoraces can be divided into primary and secondary subtypes. the ACCP recommends special consideration for patients involved in high-risk activities (such as SCUBA and flying) during which a recurrent spontaneous pneumothorax could prove fatal. persistent air leak. Chest 119:590-602. et al: Results of thoracoscopic pleural abrasion for primary spontaneous pneumothorax. In the Delphi consensus statement on therapy for spontaneous pneumothorax. REFERENCES 1. Ong TH. A secondary spontaneous pneumothorax (SSP) occurs in patients with known underlying lung disease. Respir Med 98:579-590. Galetta D. A patient with vital signs outside these parameters is considered unstable. HIV. Surg Endosc 18:466-471.Baumann MH. 2004 page 970 . atelectasis. the American College of Chest Physicians (ACCP) differentiates small from large pneumothoraces based on the apexto-cupola distance on chest x-ray (cut off at 3 cm).Sedrakyan A. Koh MS. Chest x-ray verification of PSP resolution after tube thoracostomy is the first stage in removing a chest tube. Stern JB. A systematic review of six randomized trials comparing VATS with conventional methods of treatment for pneumothorax (all types) revealed reduced pain and shorter hospital stay for VATS. Heffner JE. Additionally. A primary pneumothorax (PSP) occurs in patients with seemingly normal lung anatomy and lack of a predisposing underlying condition. Even though therapy to prevent further PSP is typically reserved for a first recurrence. et al: Management of spontaneous pneumothorax: an American College of Chest Physicians Delphi consensus statement. and extrapleural hematoma. Complications include intraoperative and postoperative bleeding. Debrosse D. chest wall infection. Therapeutic intervention after the first episode of PSP is acceptable for such patients. Treasure T: Video assisted thoracic surgery for treatment of pneumothorax and lung resections: systematic review of randomised clinical trials.Devanand A. 2001 2. A comprehensive overview for treatment is shown (see abridged outline). The ACCP considers video-assisted thoracic surgery (VATS) the preferred approach.Gossot D. Lewsey J. Light R. BMJ 329:1008. Its complication profile was comparable to open thoracotomy. eg. 2004 3. normal blood pressure. et al: Simple aspiration versus chest-tube insertion in the management of primary spontaneous pneumothorax: a systematic review. van der Meulen J. pleural effusion. 2004 4. Low SY. VATS has a higher success rate than pleurodesis in preventing recurrence. and oxygen saturation > 90 %. Strange C. Recurrence in the ipsilateral or contralateral lung has been seen with both types. Appropriate management would include intravenous antibiotics and (A) needle drainage (B) ultrasound-guided pigtail catheter (C) bedside thoracostomy (D) video-assisted thoracic surgery (VATS) decortication (E) open decortication page 971 . Chest x-ray reveals opacification of the left diaphragm with a large effusion.American College of Surgeons SESAP 13 Category 11 Item 3 A 43-year-old woman has a 2-day history of fever. and cough productive of yellow sputum. On examination she has decreased breath sounds at the left base. Diagnostic thoracentesis yields frank pus with gram-negative bacteria on gram stain. Computed tomography (CT) shows a well-circumscribed pleural collection without loculations. shortness of breath. chap 7. 2000 2. et al: Medical and surgical treatment of parapneumonic effusions: an evidence-based guideline. with worse prognosis for the elderly. in Souba WW. Mortality can be up to 20%. Trauma and Thermal Injury. Heffner J. Thoracentesis or pigtail catheters would not adequately drain an empyema. Surg Clin North Am 82:643-671. thoracic cavity surgery. Even though most patients with a thoracic empyema will eventually require operative or fibrinolytic therapy for adequate drainage. REFERENCES 1. Sundaresan S: Thoracic empyema.Colice GL. thoracic trauma.Kincaid EH. patients with co-existing cardiopulmonary or renal disease. 2002 3. Drainage of any collection should be based on its stage and complexity (see algorithm). but chest computed tomography (CT) can be helpful in characterizing the empyema and guiding interventional treatment. Initial imaging involves chest x-ray. The development of a thoracic empyema progresses through various stages (see table). and spreading infection (adjacent structures. Chest 118:1158-1171. idiopathic.de Hoyos A. Close observation is recommended to ensure complete drainage after chest tube placement. and empyema with gram-negative rods and multiple pathogens. those with hospital-acquired empyema. Curtis A. WebMD. the causes of empyema include parapneumonic effusions (usually due to pneumonia). Meredith JW: Injuries to the chest. Fink MP. Kaiser LR. Appropriate antibiotic coverage should be initiated for any parapneumonic effusion or empyema. pp 1188-1206 page 972 . subdiaphragmatic). New York.American College of Surgeons SESAP 13 Critique for Category 11 Item 3 In descending order of frequency. Deslauriers J. Jurkovich GJ. et al (eds): ACS Surgery: Principles and Practice 2006. the American College of Chest Physicians (ACCP) recognizes that some uncomplicated (no loculations) early empyemas will resolve with only tube thoracostomy. sec 5. American College of Surgeons SESAP 13 Category 11 Item 4 A 35-year-old woman has the chest x-ray shown during a pre-employment physical. these lesions are malignant in 25% of cases (D) Most are commonly located in a paravertebral sulcus (E) Magnetic resonance imaging (MRI) is reserved for cases with an intraspinal component page 973 . The chest computed tomographic (CT) scan shown is subsequently obtained. Which of the following statements about the lesion identified is NOT true? (A) Most affected adults are asymptomatic (B) Most commonly occur in the upper half of the mediastinum (C) In adults. and may arise from any thoracic nerve. pp 17321738 2. 2002. They comprise 25% to 35% of all mediastinal neoplasms and are more common in children. Philadelphia. Mosby. neurofibromas are usually nonencapsulated and homogeneous. Both tumors occasionally become large and produce benign pressure erosion that can be manifest by deformities of ribs. The incidence of malignant degeneration of a benign neurofibroma is roughly 5% in patients with neurofibromatosis. usually occurring in the upper half. such as schwannomas (neurilemmomas) and neurofibromas. pp 333-334 page 974 . Cooper JD.Collins BJ. originating from the sympathetic chain or from one of the rami of an intercostal nerve. et al (eds): Thoracic Surgery. Approximately 10% proliferate through an intervertebral foramen into the spinal canal. In contrast.American College of Surgeons SESAP 13 Critique for Category 11 Item 4 The chest x-ray and computed tomographic (CT) scan demonstrate the classic appearance of a neurogenic tumor of the posterior mediastinum. This specific tumor type is known as a dumbbell tumor based on its characteristic shape. Yang SC: Posterior mediastinal masses. Nonetheless. with the perineurium encapsulating a heterogeneous mass that frequently contains areas of cystic degeneration. although paresthesias from nerve compression or intraspinal tumor extension may occur. Peripheral nerve tumors. Deslauriers J. Neurogenic tumors are the most common masses encountered in the posterior mediastinum. Schwannomas represent 75% of nerve sheath tumors and consist of cells proliferating in the endoneurium. Churchill Livingstone. are usually benign. REFERENCES 1. in Pearson FG. are the most common types of mediastinal neurogenic tumors. Ginsberg RJ. thoracic vertebrae. Philadelphia. ed 2. or neural foramina. These tumors typically arise in the paravertebral sulcus. even though the risk of malignancy is small except in patients with neurofibromatosis or a history of radiation exposure. Both have a peak incidence in the 20s and 30s.Bousamra M: Neurogenic tumors of the mediastinum. in Yang SC. Any suspicious posterior mediastinal mass should be evaluated by chest CT and magnetic resonance imaging (MRI) if any intraspinal extension is suspected. Cameron DE (eds): Current Therapy in Thoracic and Cardiovascular Surgery. Treatment of dumbbell tumors requires a combined neurosurgical and thoracic approach. Neurogenic neoplasms are usually asymptomatic and are benign in 75% of children and in more than 90% of adults. 2004. The patient with a posterior mediastinal schwannoma or neurofibroma is usually asymptomatic. most authors recommend removal. pulse 110/minute. Proper management of this injury would be (A) controlled removal of foreign object in the emergency department with patient in lateral decubitus position and strict spinal precautions (B) immediate tracheobronchoscopy and esophagoscopy followed by controlled removal of foreign object if no injuries are demonstrated (C) emergent left posterolateral thoracotomy (D) emergent left anterior thoracotomy (E) emergent median sternotomy and left clavicular incision page 975 . and respiratory rate 24/minute.American College of Surgeons SESAP 13 Category 11 Item 5 A 38-year-old man sustains the penetrating injury shown. Vital signs are blood pressure 90/60. Chahwan S. Tracheobronchoscopy and esophagoscopy are not indicated as initial procedures in this setting. 2003 page 976 . Guske PJ.Demetriades D. Lujan HJ. et al: Penetrating injuries of the subclavian artery. This approach allows adequate exposure for proximal and distal vascular control to allow removal of the penetrating foreign object under the most controlled circumstances. Asensio JA: Subclavian and axillary vascular injuries. Gomez H. None of the other management possibilities listed would be appropriate for this injury. 2001 2. Removal of foreign penetrating objects in proximity to major vascular structures is rarely indicated in the emergency department. Such penetrating injuries and their involvement of adjacent structures within the mediastinum can best be assessed by direct inspection in the operating room. REFERENCES 1. Am J Surg 185:580-584. Koffron AJ. Based on the location and severity of the injury.American College of Surgeons SESAP 13 Critique for Category 11 Item 5 The illustration demonstrates a penetrating injury to the left superior hemithorax. 1999 3. An emergent left posterolateral or anterior thoracotomy would not allow for adequate exposure and control of the potential vascular or central visceral structures involved.Lin PH. Peng R. et al: Penetrating injuries to the subclavian and axillary vessels.Demetriades D. Surg Clin North Am 81:13571373. J Am Coll Surg 188:290-295. An emergent median sternotomy and left clavicular incision is required. the aortic arch and great vessels are the structures most likely to be injured. Which of the following statements about management of this lesion is TRUE? (A) They are usually malignant (B) Surgical excision alone is curative (C) Surgical excision should be followed by radiotherapy (D) Surgical excision should be followed by radiotherapy and chemotherapy (E) Surgical excision is reserved for patients who have a residual mass lesion after completing 4 cycles of chemotherapy page 977 . Alphafetoprotein and beta-HCG levels are not elevated. mesodermal.American College of Surgeons SESAP 13 Category 11 Item 6 A 55-year-old man with a chronic cough has the chest x-ray shown. Core needle biopsy demonstrates tissue of endodermal. and ectodermal origin. The chest computed tomographic (CT) scan shown is obtained. Jones DR: Anterior mediastinal masses. Teratomas are the most common germ cell tumors of the mediastinum. Resection is typically accomplished through a median sternotomy because of the usual anterior location of these lesions. et al (eds): Thoracic Surgery. Philadelphia. pp 1711-1723 page 978 . Complete resection of a benign mediastinal teratoma is curative. REFERENCES 1. ed 2. in Yang SC. and serum markers are diagnostic of a teratoma. Patients with benign teratomas are marker negative. Ginsberg RJ. Philadelphia. Deslauriers J. pp 327-329 2. Cooper JD. comprising two thirds of such tumors in adults. core needle biopsy. Cameron DE (eds): Current Therapy in Thoracic and Cardiovascular Surgery. The most commonly encountered mesenchymal elements are cartilage and fat. There is no role for adjuvant chemotherapy or radiotherapy for treatment of these lesions. 2002. including hair and teeth. 2004. computed tomographic (CT) scan. Benign teratomas are frequently asymptomatic and are discovered on a chest x-ray or CT scan performed for unrelated reasons. and the most common epithelial components are squamous and glandular epithelium.Wright CD: Germ cell tumors of the mediastinum.American College of Surgeons SESAP 13 Critique for Category 11 Item 6 This patient’s chest x-ray. although they are more common in patients under age 50. These tumors can occur in any age group. The diagnosis can be made clinically based on the CT scan findings. Churchill Livingstone. Mature teratomas are characterized by tissues representing at least two of the three germinal layers. The clinical presentation of teratomas depends on their histology. Teratomas typically have well-defined margins that are smooth and lobulated. Mosby. These cystic tumors may have an unusual expression of the ectodermal component. in Pearson FG. 120/minute. pulse rate. 95/55. Physical examination demonstrates severe ecchymosis and a hematoma at the right clavicular head. The computed tomographic (CT) scan with 3D reconstruction shown is obtained. He has a palpable pulse in the left wrist and no pulse on the right.American College of Surgeons SESAP 13 Category 11 Item 7 A 24-year-old man is brought to the emergency department after a high-speed motor vehicle crash. Vital signs are blood pressure. His Glasgow coma scale (GCS) score is 15. 28/minute. and respiratory rate. The best management would be (A) innominate artery ligation (B) right carotid to subclavian artery bypass with polytetrafluoroethylene (PTFE) (C) Dacron graft from aorta to distal innominate artery (D) separate saphenous vein grafts from aorta to right carotid artery and right subclavian artery (E) transposed superficial femoral artery graft from aorta to innominate artery page 979 . Cardiopulmonary bypass.Stover S. Clinical signs such as differences in blood pressure between the arms and widening of the mediastinum on chest x-ray are highly suggestive of trauma to the thoracic aortic great vessels. The placement of a right carotid to subclavian artery bypass with a polytetrafluoroethylene (PTFE) graft would not effectively restore forward flow into the carotid and subclavian arteries and is not indicated. with the innominate artery being the most frequently injured vessel. Bass TL: Blunt innominate artery injury. dyspnea. Proximal injuries are best approached with innominate artery ligation at the arch with pledgetted sutures and a Dacron bypass graft placed from the ascending aorta to the distal innominate artery beyond the level of the injury. 2002 2. REFERENCES 1. with cervical extension as required. Innominate artery ligation alone is not recommended because it does not ensure adequate flow into the right carotid and subclavian arteries. Once the diagnosis is established. Ratnatunga C: Traumatic innominate artery transection. which may rupture. may be necessary depending on the extent of the injury. 2001 page 980 . Lottenberg L. The saphenous vein graft is not optimal in this setting because of its smaller size and inability to adequately provide the increased flow demands required of major arterial branches off the aortic arch. Most patients can undergo revascularization of the innominate artery with simple vascular surgical techniques. (See Image2. Holtzman RB. Use of the superficial femoral artery placed from the aorta to the innominate artery is not indicated in this setting because it introduces undue complexity and predisposes the patient to lower extremity ischemia. The presenting symptoms of innominate artery injury (Image 1) typically include chest pain. and cough. A chest computed tomographic (CT) scan may contribute to the diagnosis and can rule out major trauma of the aorta and great vessels. Am Surg 67:757-759.American College of Surgeons SESAP 13 Critique for Category 11 Item 7 Blunt chest trauma is a common cause of injury to the aortic arch branches. Image 3 and Image 4.Anastasiadis K. including use of hypothermia or continuous cerebral retrograde perfusion. Reliably excellent surgical outcomes can be achieved in experienced hands. A high index of suspicion in chest deceleration injuries is required to make an accurate diagnosis and avoid the complications of thromboembolism and/or pseudoaneurysm rupture. including prosthetic graft replacement.) Operative caveats include placing the distal clamp on the innominate artery just proximal to the carotid subclavian bifurcation to allow retrograde perfusion of the carotid artery. as delay may contribute to the subsequent development of a pseudoaneurysm. Channon KM. J Cardiovasc Surg 43:697-700. immediate operation is recommended. The most common surgical approach is through a full median sternotomy. or distal thromboembolism. The most likely diagnosis is (A) intralobar sequestration (B) congenital cystic adenomatoid malformation (C) congenital lobar emphysema (D) bronchogenic cyst (E) esophageal duplication cyst page 981 .American College of Surgeons SESAP 13 Category 11 Item 8 An 8-year-old boy presents with recurrent left lower lobe pneumonia. The computed tomographic (CT) scan shown is obtained. Extralobar sequestration is associated with other anomalies such as diaphragmatic hernia and congenital heart disease. Lobectomy is curative and may be required on an urgent basis if respiratory compromise occurs due to hyperinflation of the involved lobe. intraparenchymal epithelial-lined lesions differentiated by the epithelial lining of the cyst.Zach MS. Resection is indicated to prevent infection and neoplastic transformation. 2003 2. Crombleholme TM: Management of congenital lung lesions. and in the case of bronchogenic cyst.American College of Surgeons SESAP 13 Critique for Category 11 Item 8 The computed tomographic (CT) scan demonstrates a cystic lesion of the left lower lobe with an aberrant systemic arterial vessel approximately 70% the diameter of the aorta. Eber E: Adult outcome of congenital lower respiratory tract malformations. Bronchogenic cysts and esophageal duplication cysts are usually solitary. This congenital pulmonary airway malformation is found in the lung parenchyma and is in the left lower lobe in 60% to 70% of cases. Thorax 56:6572. Extralobar sequestrations are surrounded by separate visceral pleura and are also more commonly found on the left side. typically in the left upper lobe. particularly rhabdomyosarcoma. resection is indicated to prevent progressive symptoms related to enlargement or infection. REFERENCES 1. Semin Pediatr Surg 12:10-16. causing hyperinflation and mediastinal shift. Congenital lobar emphysema is characterized by progressive air-trapping. Both lesions can occur in the mediastinum without distinct bronchial or esophageal communication. This is an intralobar sequestration. Infants generally become symptomatic in the first few days to weeks of life.Adzick NS. Flake AW. the presence of cartilage. Should these lesions persist after delivery. Congenital cystic adenomatoid malformations are also pulmonary airway lesions that have normal communication with the bronchial and pulmonary vasculature. Malignancy has been reported to occur in these lesions. 2001 page 982 . There is typically abnormal or absent bronchial communication and an aberrant systemic arterial supply that must be identified and controlled to prevent hemorrhage. American College of Surgeons SESAP 13 Category 11 Item 9 Which of the following statements about lung cancer is TRUE? (A) Most common type of noncutaneous cancer in men and women (B) Most common cause of cancer death in men and women (C) Deaths reduced by 25% in smokers undergoing annual screening with chest x-ray and sputum cytology (D) Metastatic disease to regional or distant sites already present in one fourth of patients at diagnosis (E) Overall 5-year survival rate 25% page 983 . Goodman PC. et al: Cancer statistics. Sullivan DC: Clinical practice. after prostate cancer in men and breast cancer in women. Lung cancer screening.Jemal A. three fourths of patients with lung cancer already have metastatic disease at the time of diagnosis. an equal number of subjects in both groups died from their cancer. but data are not yet available to determine whether CT screening reduces overall mortality from lung cancer.Mulshine JL. Murray T. CA Cancer J Clin 54:8-29. The 5year survival for patients with lung cancer is about 15%. Screening for lung cancer in high-risk patients has been disappointing. Randomized trials in the 1950s and 1970s showed no significant reduction in deaths from lung cancer using a screening program of chest x-rays and analysis of sputum cytology in high-risk smokers. 2005 3. N Engl J Med 343:1627-1633. Early studies suggest that CT screening may increase the percentage of lung cancers diagnosed as stage 1. N Engl J Med 352:2714-2720. However. More recently.American College of Surgeons SESAP 13 Critique for Category 11 Item 9 Lung cancer is the second most common type of noncutaneous cancer in the United States. accounting for 30% of all cancer deaths in the US. REFERENCES 1. Tiwari RC.Patz EF Jr. 2004 2. use of higher-resolution computed tomographic (CT) scanning has rejuvenated interest in screening for lung cancer. 2000 page 984 . 2004. Ghafoor A. Bepler G: Screening for lung cancer. Although a higher percentage of screened patients underwent surgical resection compared with controls. lung cancer is by far the leading cause of death from cancer in both men and women. Which of the following is the BEST predictor of the risk of pulmonary complications after lung resection for this patient? (A) Age > 60 years (B) History of lung resection (C) Predicted postoperative FEV 1 < 40% (D) Preoperative PaCO 2 > 45 torr (E) Preoperative PaO 2 < 60 torr page 985 . He has previously had wedge resection of his left lung for a penetrating injury.American College of Surgeons SESAP 13 Category 11 Item 10 A 65-year-old man develops a 1-cm nodule in the right lung 2 years after sigmoid resection for colon cancer. but patients younger than 70 years old are not considered to be at higher risk on the basis of age alone. REFERENCES 1. Bolliger CT: Functional evaluation before lung resection.Reilly JJ Jr: Evidence-based preoperative evaluation of candidates for thoracotomy. and height. Clin Chest Med 23:159-172. Advanced age is an independent risk factor for complications after lung resection. Of these. the FEV1 is the most predictive of outcome. 2001 page 986 . In general. assessment of pulmonary function before lung resection is appropriate to help predict whether the patient will be left with sufficient pulmonary reserve.Slinger PD. Most patients who present with surgical lung pathology have underlying chronic obstructive pulmonary disease (COPD). 1999 2. A ppo FEV1 < 40% identifies a patient at high risk of postoperative respiratory complications after lung resection. 2002 3. Diacon AH. but it does not represent an increased risk for complications per se. Johnston MR: Preoperative assessment for pulmonary resection. Although traditional risk factors such as PaO2 < 60 mm Hg and PaCO2 > 45 mm Hg are not good predictors of postoperative complications. they remain important warning indicators that should prompt further evaluation. A history of lung resection should be considered in calculating the ppo FEV1. and it is useful to express results in terms of the predicted postoperative FEV1 (ppo FEV1) based on the residual lung parenchyma and corrected for age. Chest 116:474S-476S.American College of Surgeons SESAP 13 Critique for Category 11 Item 10 Respiratory complications are the major cause of perioperative morbidity and mortality in thoracic surgery patients. Anesthesiol Clin North Am 19:411-433. tests of respiratory mechanics and lung volume correlate most strongly with outcome after lung resection.Schuurmans MM. Because lung resection invariably leads to a reduction in overall pulmonary function. sex. The appearance of the wound is pictured. and ipsilateral latissimusdorsi musculocutaneous flap (C) Debridement. antibiotic therapy.American College of Surgeons SESAP 13 Category 11 Item 11 A 75-year-old woman presents 16 years after modified radical mastectomy and adjuvant radiotherapy for T2 N1 infiltrating ductal carcinoma of the breast. Biopsy shows no malignancy. and vacuum-assisted closure (D) Resection of full-thickness radiated chest wall and rectus abdominis musculocutaneous flap (E) Resection of exposed ribs and omental transposition flap page 987 . Which of the following is most likely to result in a healed wound? (A) Ten treatments of 100% oxygen at 3 atmospheres pressure (B) Resection of exposed ribs. reconstruction with synthetic mesh. Clin Plast Surg 20:473-483. Placement of synthetic mesh is not necessary and increases the risk of subsequent infection in this chronically infected field. or more likely recurrent carcinoma of the breast. is indicated. and the internal mammary nodes. 1984 2. 1989 3. the contralateral internal mammary vessels and superior epigastric vessels will supply the rectus abdominis musculocutaneous flap. Although the blood supply to the ipsilateral rectus abdominis flap is likely to be removed with a chest wall resection. King RC: Risks and benefits of using Marlex mesh in chest wall reconstruction. Ann Plast Surg 31:303-306.Kindwall EP: Hyperbaric oxygen’s effect on radiation necrosis.American College of Surgeons SESAP 13 Critique for Category 11 Item 11 This patient has osteoradionecrosis of the chest wall.Kroll SS. inadequate resection will increase the risk of failure. and vacuum-assisted closure is appropriate. is the optimal treatment. but is not adequate to resolve the necrosis and infection of the exposed rib and does not address the surrounding damaged tissue. 1993 4. Postradiation malignancy of the chest wall is a well-recognized occurrence and this patient’s interval from operation fits the diagnosis. Ipsilateral latissimus dorsi musculocutaneous flap might be effective. they are unlikely to allow healing of a wound of this magnitude. the axilla. Bostwick J: Rectus abdominis muscle-musculocutaneous flap in chest-wall reconstruction. Surg Clin North Am 69:1007-1027. Walsh G. Localized areas of osteoradionecrosis and the juxta-axillary and upper outer quadrant or upper inner quadrants of the chest wall were previously common because of the cumulative toxicity of overlapping fields of radiation therapy used after mastectomy to treat the chest wall. so biopsy to rule out sarcoma of the chest wall.Coleman JJ III.Arnold PG. but the risk of inadequate blood supply is high because previous axillary dissection and high-dose radiotherapy may have damaged the thoracodorsal vessels. 1993 page 988 . the supraclavicular nodes. the dominant blood supply to the flap. Hyperbaric oxygen would increase the oxygen tension in adjacent vascularized tissue. Resection of the radiation-damaged area. Resection of the exposed rib would be inadequate because the entire radiated area is at risk of progressive necrosis. Experience with 100 consecutive patients. Ann Surg 199:725-732. Although debridement and antibiotic therapy are indicated. Although the omental flap is a good choice for reconstruction. REFERENCES 1. Pairolero PC: Chest wall reconstruction. surgical therapy must address the entire area of injury. including skin and adjacent muscle and rib. Ryan B. American College of Surgeons SESAP 13 Category 11 Item 12 A 40-year-old thin smoker presents with acute shortness of breath and was found to have a rightsided pneumothorax. Computed tomographic (CT) scan shows a large bleb in the right upper lobe. The best management now would be (A) continued chest tube suction and observation (B) chest tube suction with hyperbaric oxygen treatment (C) talc powder instillation through the chest tube (D) resection of the bleb (E) formal pulmonary lobectomy of affected lung page 989 . A chest tube produces immediate relief of symptoms. He had a similar episode 12 months ago. Noguchi Y. Moriyama S: Video-assisted thoracoscopic surgery for primary spontaneous pneumothorax: evaluation of indications and long-term outcome compared with conservative treatment and open thoracotomy. Clin Evid 13:1884-1890. formal lobectomy is usually not necessary. usually with some type of tube thoracostomy. Conservative management is appropriate for patients with first-time pneumothoraces. 40% to 50% of pneumothoraces will recur. the risk of a third pneumothorax is over 60%. Resection of blebs reduces the risk of recurrent pneumothoraces to under 10%. When the air leak resolves.Wakai A: Spontaneous pneumothorax. the chest tube can be placed to water seal. These tend to be local at the apex of the superior segments of the lung. Initial management should be evacuation of the pneumothorax. or the instillation of a sclerotic agent such as talc. At this point. Early operation reduces hospital stay and decreases the risk of recurrence. The typical patient is a young. Shimbo T. There is no consensus on how many days constitute a prolonged air leak. Most surgeons would agree that more than 10 days constitutes a prolonged air leak. the tube can be removed. many would advocate surgical intervention.Morimoto T. while conservative management is ultimately successful in most patients and 50% will never have another pneumothorax. 2004 2.Sawada S. 2005 3. After initial conservative management. or more commonly now. Although conservative management is possible in the setting of a recurrent pneumothorax. Patients present with the acute onset of chest pain and dyspnea. Poudrage. if the lung does not collapse and the patient remains asymptomatic. Resection can be done through an open thoracotomy. videoassisted thoracoscopic surgery (VATS). Watanabe Y. 2005 page 990 . Most spontaneous pneumothoraces are related to pulmonary blebs. tall. et al: Effects of timing of thoracoscopic surgery for primary spontaneous pneumothorax on prognosis and costs. REFERENCES 1. Koyama H. is not indicated in an initial episode of spontaneous pneumothorax and is inadequate therapy in the presence of a large air leak. The decision on what to do with a patient with a prolonged air leak is more difficult. while less than 5 days is not enough time. spontaneous pneumothorax is not uncommon. Hyperbaric oxygen or administration of high FiO2 may help resolve pneumothoraces without persistent air leak. thin male with a history of smoking. however. Am J Surg 187:767-774.American College of Surgeons SESAP 13 Critique for Category 11 Item 12 Although most pneumothoraces are caused by some type of trauma. This is not appropriate treatment for a recurrent pneumothorax. Chest 127:2226-2230. Because blebs are usually localized. The computed tomographic (CT) scan shown is obtained.American College of Surgeons SESAP 13 Category 11 Item 13 The 11-year-old boy shown in the photo complains of easy fatiguability and worsening shortness of breath during soccer practice. The best management for this problem would be (A) bilevel positive-pressure ventilation (B) resection of deformed costal cartilages and chest wall reconstruction (C) sternal osteotomy (D) insertion of steel struts (E) close observation page 991 . The bar is used to elevate the deformed sternum and costal cartilages to the desired position.Nuss D. World J Surg 27:502-508. et al: Experience and modification update for the minimally invasive Nuss technique for pectus excavatum repair in 303 patients.American College of Surgeons SESAP 13 Critique for Category 11 Item 13 A minimally invasive technique for definitive repair of pectus excavatum was reported in 1998 for young children (mean age of under 5). REFERENCES 1.Fonkalsrud EW: Current management of pectus excavatum. Katz ME: A 10-year review of a minimally invasive technique for the correction of pectus excavatum. Results comparing the Nuss procedure to open procedures show that the morbidity is less with the minimally invasive procedure and the results equivalent. The Nuss procedure avoids the major resective procedures with cartilage resection and sternal osteotomy by placing a carefully preformed convex steel bar under the sternum through bilateral thoracic incisions. Lawson ML. Goretsky MJ. 2003 3. which facilitates the placement of the bar. 2002 2. Thoracoscopy is used to visualize the passage of the clamp behind the sternum. 1998 page 992 . Croitoru DP. Kelly RE Jr.Croitoru DP. J Pediatr Surg 37:437-445. The best results are seen when done in children under 15. Observation or bilevel positivepressure ventilation has no role in management of symptomatic pectus excavatum. Kelly RE Jr. Symptomatic patients were previously treated with an open procedure to correct the pectus deformity. This minimally invasive procedure is effective for management of pectus excavatum. J Pediatr Surg 33:545-552. and malaise. Bronchial washings are negative for malignancy. Bronchoscopy is performed. but the infiltrate has remained. The most appropriate treatment for most patients with this lesion would be (A) bronchotomy of the right upper lobe bronchus and removal of the lesion (B) sleeve resection of the right upper lobe bronchus and adjacent main bronchus (C) right upper lobectomy (D) endobronchial removal of the lesion (E) radiofrequency ablation of the lesion by the endobronchial approach page 993 . as shown. Biopsy of the lesion shows normal mucosa and cartilaginous material. fever. He has received 3 courses of antibiotics for presumed right upper lobe pneumonia. with no evidence of malignancy. The computed tomographic (CT) scan shown is obtained.American College of Surgeons SESAP 13 Category 11 Item 14 A 45-year-old smoker has a 4-month history of a right upper lobe infiltrate. Symptoms include chronic cough with production of purulent sputum. and lobectomy or segmentectomy to remove the diseased lung is the procedure of choice. and is in an unfavorable location in the upper lobe bronchus. 2003 page 994 .Fink G. but this tumor is too large (> 50% of the diameter of the bronchus). or by bronchotomy and tumor enucleation. sleeve resection should be considered. Chest 119:1647-1651. Krelbaum T. The therapeutic issue is whether it should be resected locally. Most series have noted that most patients with endobronchial tumors are treated with lobectomy or even pneumonectomy. Endobronchial removal may be indicated in some cases. Yellin A. This is usually a benign lesion. et al: Pulmonary carcinoid: presentation. As described. this patient’s computed tomographic (CT) scan shows chronic pneumonia in the lobe distal to the lesion. diagnosis. In the presence of normal distal lung. However.Scott WJ: Surgical treatment of other bronchial tumors. it would appear to be a hamartoma. although like other tumors in the tracheobronchial tree it has the potential for at least local recurrence. or whether it should be removed by lobectomy.American College of Surgeons SESAP 13 Critique for Category 11 Item 14 This lesion is benign. and outcome in 142 cases in Israel and review of 640 cases from the literature. Chest Surg Clin North Am 13:111-128. Bendayan D. 2001 2. REFERENCES 1. Radiofrequency ablation has not been reported for this type of tumor. by sleeve resection. and T3) (C) The response is good or excellent in nearly all patients (D) TTS has little or no effect on lower extremity hyperhydrosis (E) Compensatory sweating in truncal areas is uncommon page 995 .American College of Surgeons SESAP 13 Category 11 Item 15 Which of the following statements about thoracoscopic thoracic sympathectomy (TTS) for upper extremity hyperhydrosis is TRUE? (A) TTS is best performed only after a stellate ganglion injection (B) TTS must include the lower half of the stellate ganglion (T1. T2. Yazbek G. Therapy includes topical antiperspirants and oral medications (anticholinergics. Both palmar and axillary hyperhydrosis are relieved. Kauffman P. de Campos JR. REFERENCES 1. In contrast to sympathectomy for the arm pain of reflex sympathetic dystrophy. and T3 appears to produce no better results than individual sympathectomy of T2. 2005 2. Gustatory sweating is also common. Arch Surg 140:676-680. 2005 page 996 . Wolosker N. At least one series found that concurrent plantar hyperhydrosis was relieved in half of the patients. Excessive is difficult to define. or axillary sweating sufficient to interfere with normal wearing of clothing. and lumbar region is common and can sometimes be severe after this procedure. tranquilizers. Zacherl J. but is unlikely to be severe. occurring in 20% to 90% of cases in published series. and may not even be predictive of a good result. Bischof G: Effect of endoscopic thoracic sympathetic block on plantar hyperhidrosis. et al: Palmar hyperhidrosis — which is the best level of denervation using video-assisted thoracoscopic sympathectomy: T2 or T3 ganglion? J Vasc Surg 42:281-285. but other series have not yet confirmed this.Neumayer C. probably reflecting differing criteria. T3. but most patients complain of palmar sweating sufficient to have adverse social effects. Sympathectomy of the stellate ganglion of T1. Dorsal sympathectomy is indicated for patients who fail to respond to other modalities. groin. a trial of stellate ganglion block is not necessary. occurring in a third of patients or more. Thoracoscopy has made dorsal sympathectomy both simple and effective. Panhofer P.American College of Surgeons SESAP 13 Critique for Category 11 Item 15 Hyperhydrosis is defined as excessive sweating. NSAIDs). The optimum technique of the operation is still unclear. Compensatory sweating of the abdomen. with a response rate of 95% to100%. T2. or T4. American College of Surgeons SESAP 13 Category 11 Item 16 Intrapleural instillation of streptokinase in patients with early empyema (< 7 days) (A) improves mortality from empyema (B) lowers the rate of surgical intervention (C) decreases the length of hospital stay (D) improves the radiographic outcome (E) has no significant effect on outcome page 997 . Although this study appears large and definitive. but there have been few adequate studies. REFERENCES 1. Am J Respir Crit Care Med 170:49-53. ie. and supportive care.Lee YC: Ongoing search for effective intrapleural therapy for empyema: is streptokinase the answer? Am J Respir Crit Care Med 170:1-2. the streptokinase group exhibited a high level of antistreptokinase antibody. Van de Wal BW.Maskell NA. In a recent study from South Africa comparing 22 patients with and 22 without streptokinase. length of stay. Hedley EL. Davies CW. incidence of surgical intervention (also 16% and 14%). the streptokinase group had only 3 patients referred for operation (15%). N Engl J Med 352:865-874. 2004 2. No difference was found in mortality (16% in streptokinase group vs 14% in control group). Unsurprisingly. This rate of thoracotomy is unusually high for patients with early empyemas. lung volume measurements. N Engl J Med 352:926-928. or bleeding tendency after surgical intervention. 2004 4.Heffner JE: Multicenter trials of treatment for empyema — after all these years.000 units twice a day for 3 days) through the chest tube versus standard treatment. 2005 3. while the control group had 10 patients. et al: Intrapleural streptokinase for empyema and complicated parapneumonic effusions. Nunn AJ. Nonetheless. Theron J. Schuurmans MM. an important limitation is that the tubes used were small (12 Fr) by North American standards. However. mortality and outcome were no different. radiographic assessment of outcome. antibiotics. but the streptokinase group required operation much less often. tube thoracostomy. et al: UK Controlled trial of intrapleural streptokinase for pleural infection. surgical intervention as required.Diacon AH. or 45%.American College of Surgeons SESAP 13 Critique for Category 11 Item 16 Thrombolytic agents have been strongly advocated and widely used to prevent loculation and avoid surgical intervention in patients with empyema. there is ongoing enthusiasm for using thrombolytic agents in patients with complex postpneumonic effusions and with early empyema. in whom the therapy might be expected to work better than in patients with established empyema (> 7 days). patients were randomly assigned to receive streptokinase (250. 2005 page 998 . In a multicenter study of 454 patients in the United Kingdom. and computed tomographic findings (B) Thoracoscopy with talc insufflation produces more improvement than talc slurry instillation through a chest tube (C) Malignancies with the best response to talc pleurodesis are breast and lung primaries (D) Talc pleurodesis is associated with a low incidence of adverse effects (E) Larger doses of talc are more likely to produce pulmonary toxicity page 999 . bloody pleural effusion.American College of Surgeons SESAP 13 Category 11 Item 17 Which of the following statements about malignant pleural effusion is TRUE? (A) Diagnostic criteria include symptom duration. Chest 127:1017-1022.Ferrer J.Dresler CM. Herndon JE II. absence of fever. In a recent multi-center study. Therapeutic insufflation of talc powder elicits fibrosis and may prevent recurrence of the effusion. thoracoscopy is often advocated to establish a diagnosis. no difference was found in the overall response rate between talc insufflated at thoracoscopy and talc slurry instilled in a chest tube. Patients with lung or breast primary tumors had higher success with thoracoscopy than with instillation of talc slurry.American College of Surgeons SESAP 13 Critique for Category 11 Item 17 When a patient has a persistent or recurrent pleural effusion. et al: Predictors of pleural malignancy in patients with pleural effusion undergoing thoracoscopy. REFERENCES 1. and chest computed tomographic (CT) findings suggesting parenchymal or pleural malignancy. Talc appears able to migrate into the lungs and it may thereby trigger the acute respiratory distress syndrome. Chest 127:909-915. The 30-day outcome was successful in both groups (78% in thoracoscopy and 71% in slurry). Both groups receiving talc had a significant 30-day mortality rate: 6% in the slurry group and 8% in the thoracoscopy group. Roldan J. Teixidor J. In an analysis of 93 patients. variables in a multi-variate model found to predict the presence of malignancy included symptoms more than 1 month. The recurrence rate was high by either technique. Talc cannot be considered a nontoxic therapy. rising to 50% by 150 days among the survivors in both groups. These criteria can be used to assess the need for patients to have diagnostic thoracoscopy. bloody pleural fluid. et al: Phase III intergroup study of talc poudrage vs talc slurry sclerosis for malignant pleural effusion. Talc-induced respiratory failure is not dependent on the dose used. The mechanism by which talc produces respiratory failure is not known. and pleural fluid cytology proves nondiagnostic. the incidence of respiratory failure in the two groups was 4% and 8%. and has been reported with doses as low as 2 g. Olak J. More concerning. Using all of these criteria resulted in a 95% correct classification. 2005 2. Pallisa E. 2005 page 1000 . Richards WG. The next diagnostic test should be (A) pulmonary angiogram (B) computed tomographic (CT) scan (C) ventilation-perfusion scan (D) D-dimer assay (E) ultrasound study of the lower extremity page 1001 . The chest x-ray shown is obtained.American College of Surgeons SESAP 13 Category 11 Item 18 A 64-year-old man has shortness of breath and documented hypoxemia 2 days after a right hemicolectomy. It is highly accurate for larger emboli. Laboratory studies. and feasibility must be considered. so an electrocardiogram may be appropriate. Helical chest computed tomography has emerged as an extremely useful test for the diagnosis of pulmonary embolism. cancer. although it is now reserved for the subgroup of patients in whom the diagnosis can’t be made by other. Zou KH. 2001 4. positive predictive value. 2005 page 1002 . and the negative predictive value of a negative or low probability ventilationperfusion scan is 76% to 88%. Manzato F. pulmonary embolism must be excluded. REFERENCES 1. This patient’s chest x-ray shows some atelectasis or consolidation in the left lower lung. Clin Exp Med 1:161-164. and in general this test is much less helpful in the in-patient setting. It has been the gold standard for the diagnosis. Veraldi GF. but suffers from lack of accuracy. However. Most patients do not have a definitive or high-probability scan. it may be helpful if negative.Fedullo PF. 2005 3.Quiroz R. are also appropriate. trauma. although sensitivity may be as low as 66% for subsegmental emboli. In these settings. Noguchi Y. Ultrasound study of the lower extremities is positive in 50% of patients with known embolism. It has a negative predictive value of 99%.Hayashino Y. but cannot reliably exclude pulmonary embolism. et al: Clinical validity of a negative computed tomography scan in patients with suspected pulmonary embolism: a systematic review. including arterial blood gases and cardiac enzymes. Radiology 234:740-748. Although it is relatively noninvasive.American College of Surgeons SESAP 13 Critique for Category 11 Item 18 Shortness of breath and hypoxemia two days after a major abdominal operation has many possible causes. Ventilation-perfusion scan is noninvasive. Cardiac disease can manifest as pulmonary symptoms. It is helpful if positive. equivalent to pulmonary angiogram. D-dimer has been suggested as a reliable diagnostic tool for pulmonary embolism in the outpatient setting. postoperative patients. The negative predictive value of angiography is 95% to100%. Fraccaroli M. particularly in patients with known coronary artery disease or diabetes. inflammatory states. Fukui T: Ventilation-perfusion scanning and helical CT in suspected pulmonary embolism: meta-analysis of diagnostic performance. and the elderly. In the absence of pneumonia or a cardiac etiology. negative predictive value. JAMA 293:2012-2017. Kucher N. In determining which test is most appropriate. it cannot be used in patients with significant renal dysfunction because of the need for intravenous contrast. it is also elevated in many conditions associated with an increased risk of thromboembolic events: pregnancy. a nonspecific finding unlikely to be helpful in explaining significant shortness of breath. Kipfmueller F.Lippi G. less invasive means. A careful history and physical examination should elicit signs and symptoms suggestive of pneumonia. Its positive predictive value is equivalent to a high probability ventilation-perfusion scan. The evaluation of suspected pulmonary embolism. 2003 2. Goto M. Tapson VF: Clinical practice. but an elevated D-dimer is not. N Engl J Med 349:1247-1256. et al: Variation of plasma D-dimer following surgery: implications for prediction of postoperative venous thromboembolism. and it is more sensitive than pulmonary angiography for segmental and subsegmental emboli. It is elevated in nearly all patients with thromboembolism. This is particularly true in patients like this one who have an abnormal chest x-ray. pulse is 110. Her electrocardiogram is normal. The chest computed tomographic (CT) scan shown is obtained. Appropriate treatment would be (A) systemic heparin therapy (B) local thrombolytic therapy (C) systemic thrombolytic therapy (D) percutaneous mechanical thrombectomy (E) placement of an inferior vena caval filter page 1003 .American College of Surgeons SESAP 13 Category 11 Item 19 A 45-year-old woman has right-sided chest pain and a nonproductive cough 7 days after a modified radical mastectomy. Her blood pressure is 120/70. and respiratory rate is 24. Murata S. or an S in lead I and Q waves in lead III on electrocardiogram.Wan S. without circulatory compromise or right ventricular strain this patient does not need operative thromboembolectomy. not high enough to consider thrombolytic therapy. N Engl J Med 347:1143-1150. her response to anticoagulation. Using this scoring system. Mechanical thrombectomy removes 30% to 80% of total clot burden. and the duration of her risk for recurrent thromboembolic events. Agnelli G. Eikelboom JW: Thrombolysis compared with heparin for the initial treatment of pulmonary embolism: a meta-analysis of the randomized controlled trials. The decision to place an inferior vena cava filter should be made based on evidence of lower extremity clot. pulmonary hypertension on right heart catheterization. Heinrich F. Circulation 110:744-749. 2005 4. has also been used to determine the need for thrombolytics. A combination of clot burden and reduction of flow as demonstrated by angiography. Without these criteria. et al: Heparin plus alteplase compared with heparin alone in patients with submassive pulmonary embolism. If right ventricular strain were present in this patient. Flow does not appear to be completely obstructed to either lung. inverted T waves in leads V1. or right ventricular dysfunction and pulmonary artery hypertension on echocardiography. catheter thrombectomy would be an option. Nakazawa K. 2004 page 1004 . Because angiography is now rarely used to make the diagnosis of pulmonary embolism. operative thromboembolectomy is used very infrequently for the treatment of acute pulmonary embolus. Koolen MG. J Nippon Med Sch 72:74-84. This option uses local and systemic heparin therapy in addition to local thrombolytic therapy.Tajima H. thrombolytic therapy has a significant survival advantage over systemic heparin therapy alone. Baarslag HJ. Cardiovasc Intervent Radiol 26:246-250. et al: Mechanical thrombectomy for early treatment of massive pulmonary embolism. However. Miller scores are now infrequently used. Thrombolytic therapy would also be relatively contraindicated in this patient who has had a recent operation. In patients with right ventricular strain.American College of Surgeons SESAP 13 Critique for Category 11 Item 19 Computed tomography (CT) shows a pulmonary embolus with extension into both the right and left main pulmonary arteries. Quinlan DJ.Reekers JA. With the availability of thrombolysis and mechanical catheter thrombectomy. the presence of clot in the right and left pulmonary arteries would score 16— without limitation of flow. called the Miller score. Evidence of right ventricular strain can include a new right bundle branch block (complete or incomplete). Heusel G. thrombolytic therapy does not provide a survival advantage. et al: Recent advances in interventional radiology for acute massive pulmonary thromboembolism. and V3. 2003 3. 2002 2. REFERENCES 1. V2.Konstantinides S. Van Delden O. Indications for local or systemic thrombolytic therapy include circulatory collapse and right ventricular strain. which immediately improves cardiac output and PaO2. Kumazaki T. Geibel A. hypotension.American College of Surgeons SESAP 13 Category 11 Item 20 A 45-year-old otherwise healthy patient has fever. and respiratory failure 72 hours after endoscopic dilation of a benign distal esophageal stricture. The most appropriate step now would be (A) broad-spectrum antibiotics and observation (B) esophageal resection (C) thoracotomy with closure of the perforation in layers (D) repeat endoscopy (E) gastrograffin swallow page 1005 . The chest computed tomographic (CT) scan with intravenous contrast shown is obtained. 1996 4. a contrast esophagogram with aqueous contrast can be obtained. Mansour KA: Esophageal perforation: emphasis on management. because these factors control the degree of contamination of the soft tissues and pleural space. makes primary repair less successful. however the false negative rate of this study is ~20%. A patient reporting any of these symptoms after instrumentation of the esophagus should be considered to have a perforation until proven otherwise. relief of distal esophageal obstruction if needed. and can be performed if the patient’s clinical status allows. Repeat endoscopy is not recommended and may further the extent of injury and spillage of enteric contents into the mediastinum. odynophagia. The symptoms are generally severe but may be nonspecific and include chest and abdominal pain. Wein BB.Goldstein LA. which is often diseased and contributed to the perforation in the first place. buttressing these repairs with pleura. and establishment of antibiotic and nutritional therapy. dyspnea. Kiss J. These patients should receive broad-spectrum antibiotics. In a critically ill patient. If the clinical diagnosis remains in question. pericardium. This approach reliably removes the defect and the surrounding esophagus. 1997 3. but necessitates a delayed reconstructive procedure. Iatrogenic injury is the most common cause of perforation. Ann Thorac Surg 65:1433-1436. the more likely that simple repair will fail. A high index of suspicion is important for early recognition of injury. Antibiotic therapy alone is insufficient. retrospective analysis supports esophageal resection in patients with established sepsis from a perforated esophagus. and fever. Primary repair is indicated when the defect is small and identified early. drainage of the mediastinum. the segmental blood supply of the esophagus is often damaged or insufficient to heal a large defect or one that has required surgical mobilization of the esophagus. Sziranyi E: The role of esophagectomy in the management of esophageal perforations. The patient’s clinical status (clinical sepsis) generally reflects the extent of injury and time from the injury to treatment. The surgical treatment of a patient in extremis should not be delayed for confirmatory studies. debridement of necrotic and infected tissue.Bufkin BL. REFERENCES 1.Altorjay A. Radiology 202:683-686. The repaired tissue must be viable and under no tension when repaired. which is lethal unless treated early and effectively. The larger the defect and the longer the time from injury. immediate reconstruction of the esophagus after resection has been performed with good results. as in this case. Thompson WR: Esophageal perforations: a 15 year experience. Additionally.Buecker A. In some centers. Voros A. or a vascularized intercostal muscle flap is recommended. demonstrating air in the mediastinum and bilateral pleural effusions. 1998 2. Ann Thorac Surg 61:1447-1451. Chest x-rays are diagnostic in only 15% of patients and may be normal in 10% of patients with perforation. Am J Surg 143:495page 1006 . resection coupled with a protective proximal diverting stoma with distal drainage of the stomach can be performed quickly. This patient’s computed tomographic (CT) scan is diagnostic. Miller JI Jr. Delayed recognition. Neuerburg JM. The spectum of surgical care for these injuries includes: closure of the perforation.American College of Surgeons SESAP 13 Critique for Category 11 Item 20 Rupture or laceration of the esophagus results in a chemical and infectious mediastinitis. Guenther RW: Esophageal perforation: comparison of use of aqueous and barium-containing contrast media. Although no prospective data are available. org/sections/clinicalresources/clinicalcases/article-7. Ginsberg RJ: Esophageal perforation: a continuing challenge. 1992 6. Available at: http://www. Sukumar MS: Necrotizing esophagitis: the black esophagus. Enestvedt CK.ctsnet. 1982 5.Tripathy U. Ann Thorac Surg 53:534543. Komanapalli CB.Jones WG II.html page 1007 . 2005. The Cardiothoracic Network Surgery Network website.American College of Surgeons SESAP 13 503. and halitosis.000/mm3. a 56-year-old man has a worsening metabolic acidosis associated with decreasing urine output. hemodynamic instability.American College of Surgeons SESAP 13 Category 11 Item 21 Thirty-six hours after transhiatal esophagectomy with pyloroplasty and cervical anastomosis for a T2N0 adenocarcinoma of the gastroesophageal junction. Thin gray drainage is noted at the inferior aspect of the cervical wound. Hematocrit is 28% and WBC count is 18. Nasogastric tube output is dark brown and guaiac positive. The next step in management should be (A) barium swallow (B) chest and abdominal computed tomography (CT) (C) gastric debridement with cervical diversion (D) drainage of cervical wound infection (E) gastric resection with colon interposition page 1008 . Orringer MB.Urschel JD.Urschel JD: Esophagogastric anastomotic leaks: the importance of gastric ischemia and therapeutic applications of gastric conditioning. Barium swallow or computed tomography (CT) of the abdomen and/or chest may confirm the presence of a leak but may also appear unremarkable. but only delays therapy in the face of evidence for an anastomotic leak. Colonic interposition may be required for eventual reconstruction. Iannettoni MD. Whyte RI. J Invest Surg 11:245-250. and nonbilious. though mortality is greater than 90% even with prompt surgical management. Yee J: Challenges in reversing esophageal discontinuity operations.American College of Surgeons SESAP 13 Critique for Category 11 Item 21 The clinical scenario of hemodynamic instability. Thurer R. followed by exploration of the neck with visualization of the conduit. foul-smelling output from the nasogastric tube strongly suggests that the gastric conduit is necrotic. Early anastomotic leak after transhiatal esophagectomy is most often due to ischemia of the gastric conduit. Dis Esophagus 16:6065. 2003 2. The mediastinum and pleural spaces should be drained and nutritional support provided. 2003 4. Orringer MB: Catastrophic complications of the cervical esophagogastric anastomosis. delaying definitive management in this setting. Med Sci Monit 9:RA173-RA180. Confirmation of a necrotic stomach requires cervical diversion of the esophagus and abdominal exploration with removal of the necrotic stomach. 2003 5. Ashiku S. but should be delayed until the patient has fully recovered. 1998 page 1009 . but complete conduit loss is a relatively uncommon complication.Urschel JD. 1995 3. Sellke FW: Salvage or planned esophagectomy after chemoradiation therapy for locally advanced esophageal cancer—a review. metabolic acidosis. Early recognition and treatment can be life-saving.Iannettoni MD. Ann Thorac Surg 76:989-994.Barkley C. Definitive management requires rapid resuscitation and hemodynamic support. Sellke FW: Complications of salvage esophagectomy. J Thorac Cardiovasc Surg 110:1493-1500. Endoscopy may be a useful adjunct to the evaluation of conduit viability. Local wound care would not be adequate treatment. REFERENCES 1. American College of Surgeons SESAP 13 Category 11 Item 22 The 24-year-old man pictured has symptoms of a bronchopleural fistula. he was treated for an empyema that developed after bacterial pneumonia. Twelve months previously. Initial treatment of an empyema should include all of the following EXCEPT (A) antibiotic therapy (B) decortication (C) closed tube thoracostomy (D) thoracoscopy (E) recruitment of atelectatic lung page 1010 . there is often enough of a remnant of latissimus or serratus to obliterate the defect. combinations of Pneumococcus. parenchymal lung infection. preventing re-expansion of the compressed lung and harboring persistent bacteria. Small fistulas may be addressed through the cavity. Billiar TR. Antibiotics tailored to the specific organism are a necessary part of treatment. thoracotomy and decortication is curative. McGraw-Hill. and gram-negative organisms are prevalent. mediastinum. other Streptococci. pp 545-610 page 1011 . lung. or secondary to super-infection of a pleural effusion from cardiac or other origins. Ann Surg 211:656-662. in Brunicardi FC. REFERENCES 1. Dunn DL. When present. The omentum pedicled on the gastroepiploic vessels is also useful. Removal of any obstruction to expansion of the lung and closure of the bronchopleural fistula if present are also necessary. Muscle flaps or musculocutaneous flaps bring well-vascularized tissue and bulk into the defect to help control infection and close the dead space. An account of their use in the management of 100 consecutive patients. the pectoralis major. Common to the process is bacterial growth within the fluid comprising the empyema. New York. Treatment is oriented toward removal or containment of the infectious process. ed 8. latissimus dorsi. Pairolero PC: Intrathoracic muscle flaps. Dead space must be obliterated. If there is no bronchopleural fistula and the underlying lung is capable of fully reexpanding. surgical complication. 2005. Thoracentesis alone is successful in less than 10% of cases. If the fistula is large.Arnold PG. and culture-specific antibiotic therapy directed toward the offending organism is critical. 1990 2.Maddaus MA. Although Streptococcus pneumoniae was formerly the most common organism seen. Andersen DK. Staphylococcus. either by collapsing the chest wall down to the lung surface by rib resection or thoracoplasty and/or by filling the defect with a musculocutaneous flap. thoracotomy with isolation and ligation or stapling of the bronchus is indicated. Depending on the site of the fistula and/or defect. et al (eds): Schwartz's Principles of Surgery.American College of Surgeons SESAP 13 Critique for Category 11 Item 22 Empyema is a purulent collection confined to the pleural space. and compression of the lung with restriction of normal aerated lung to expand to fill the entire pleural space. Luketich JD: Chest wall. or serratus anterior may be used and brought intrathoracic by mobilization on their axial blood supply. but used alone are unlikely to be curative. Closed-suction drainage is effective when systemic sepsis has been controlled and the lung expands after drainage. the pleural collection tends to form a rind. Even in the presence of a previous posterolateral thoracotomy. and pleura. Flap closure is unlikely to be successful unless the infected rind is removed and the bronchopleural fistula closed. An empyema may be converted to open drainage with or without rib resection. Possible etiologies include trauma. the bronchopleural fistula must be closed. he was treated for an empyema that developed after bacterial pneumonia. His current problem is most likely to be definitively treated by (A) 6 weeks of antibiotic therapy (B) pneumonectomy (C) musculocutaneous flap and 6 weeks of antibiotics (D) endobronchial fibrin glue (E) closure of bronchopleural fistula and obliteration of dead space page 1012 . Twelve months previously.American College of Surgeons SESAP 13 Category 11 Item 23 The 24-year-old man pictured has symptoms of a bronchopleural fistula. Maddaus MA. and culture-specific antibiotic therapy directed toward the offending organism is critical. latissimus dorsi. 1990 2. parenchymal lung infection. Andersen DK. Common to the process is bacterial growth within the fluid comprising the empyema. Luketich JD: Chest wall.American College of Surgeons SESAP 13 Critique for Category 11 Item 23 Empyema is a purulent collection confined to the pleural space. Treatment is oriented toward removal or containment of the infectious process. et al (eds): Schwartz's Principles of Surgery. New York. Depending on the site of the fistula and/or defect. Dead space must be obliterated. the pectoralis major. the pleural collection tends to form a rind. The omentum pedicled on the gastroepiploic vessels is also useful. Flap closure is unlikely to be successful unless the infected rind is removed and the bronchopleural fistula closed. and gram-negative organisms are prevalent. Antibiotics tailored to the specific organism are a necessary part of treatment. preventing re-expansion of the compressed lung and harboring persistent bacteria. Billiar TR. Pairolero PC: Intrathoracic muscle flaps. Thoracentesis alone is successful in less than 10% of cases. ed 8. An account of their use in the management of 100 consecutive patients. mediastinum. or secondary to super-infection of a pleural effusion from cardiac or other origins. Staphylococcus. and pleura. When present. and compression of the lung with restriction of normal aerated lung to expand to fill the entire pleural space. REFERENCES 1. Even in the presence of a previous posterolateral thoracotomy. Removal of any obstruction to expansion of the lung and closure of the bronchopleural fistula if present are also necessary. lung. or serratus anterior may be used and brought intrathoracic by mobilization on their axial blood supply. Dunn DL. 2005. thoracotomy and decortication is curative. but used alone are unlikely to be curative. Possible etiologies include trauma. in Brunicardi FC. either by collapsing the chest wall down to the lung surface by rib resection or thoracoplasty and/or by filling the defect with a musculocutaneous flap. pp 545-610 page 1013 . combinations of Pneumococcus. Although Streptococcus pneumoniae was formerly the most common organism seen. If the fistula is large. An empyema may be converted to open drainage with or without rib resection. Closed-suction drainage is effective when systemic sepsis has been controlled and the lung expands after drainage.Arnold PG. Small fistulas may be addressed through the cavity. there is often enough of a remnant of latissimus or serratus to obliterate the defect. other Streptococci. Ann Surg 211:656-662. If there is no bronchopleural fistula and the underlying lung is capable of fully reexpanding. the bronchopleural fistula must be closed. McGraw-Hill. Muscle flaps or musculocutaneous flaps bring well-vascularized tissue and bulk into the defect to help control infection and close the dead space. surgical complication. thoracotomy with isolation and ligation or stapling of the bronchus is indicated. Her chest x-ray is also shown.American College of Surgeons SESAP 13 Category 11 Item 24 The patient pictured is scheduled to undergo thyroidectomy for presumed multinodular goiter. The most likely life-threatening postoperative complication she is at risk for would be (A) hypoparathyroidism (B) unilateral recurrent nerve injury (C) bilateral recurrent nerve injury (D) tracheomalacia (E) esophageal perforation page 1014 . 1995 page 1015 . With respect to multinodular goiter. Injection of a paralyzed cord with silicone is sometimes used to medialize the cord and improve speech. Even with bilateral recurrent nerve injury. If it is soft and deformed. and is unlikely to be lifethreatening. the constant pressure on the trachea leads to softness of the tracheal rings. 2005 2. and outcome. the cord stops at the paramedian position.American College of Surgeons SESAP 13 Critique for Category 11 Item 24 This patient has a large multinodular goiter. Chest 127:984-1005. although it may be difficult to determine whether this is due to tracheomalacia. morbidity. Ernst A: Tracheomalacia and tracheobronchomalacia in children and adults: an in-depth review. not in the midline. or concomitant pulmonary disease. Patients may or may not be symptomatic. Preoperative evaluation should include laryngoscopy to evaluate vocal cord function and pulmonary function testing to differentiate pulmonary parenchymal disease from airway obstruction. If the trachea collapses with spontaneous breaths. approximately a third of the airway remains patent. Tracheomalacia is a weakness of the trachea. J Surg Oncol 60:207-212. the presence of tracheomalacia is confirmed. Permanent hypoparathyroidism may be difficult to treat but is unlikely to result in death. Am Surg 60:586-591. The endotracheal tube is pulled back superior to the level of the deformity. Shaha AR: Substernal goiter. Waltz DA. REFERENCES 1. but is not appropriate for airway problems.McHenry CR. Piotrowski JJ: Thyroidectomy in patients with marked thyroid enlargement: airway management. After completion of the thyroidectomy. 1994 3. Hypocalcemia may occur relatively acutely because of damage to the parathyroids or their blood supply. Symptoms are usually dyspnea. If recognized. vocal cord paresis or paralysis secondary to recurrent laryngeal nerve involvement. Most patients with large goiter will not have tracheomalacia and can be extubated without difficulty. and tracheostomy should be performed. with possible leakage into the pleural space. the balloon of the endotracheal tube should be deflated and the patient allowed to breath spontaneously.Newman E. There is no indication for injection of any nerve with silicone. and the trachea cannot maintain its shape. the trachea must be evaluated intraoperatively.Carden KA. Boiselle PM. When the recurrent laryngeal nerve is damaged. Injury to the esophagus in the neck or even upper mediastinum is unlikely to cause the problems caused by injury in the thorax. Although the patient is at risk for all the listed complications. frequently due to reduction and/or atrophy of the longitudinal elastic fibers or the pars membranacea or impaired integrity of the cartilageous rings. which is usually enough to allow inspiration and expiration. tracheomalacia would be most life threatening. leading to a softer airway more prone to collapse. However. The trachea is observed directly with a bronchoscope. and the chest x-ray demonstrates narrowing of the trachea by the goiter. tracheomalacia should be suspected. infusions of calcium are indicated. Tracheomalacia should be suspected in all patients with large multinodular goiters. The recommended management for this condition would be (A) routine intravenous calcium infusion (B) extubation while the patient is awake in the PACU or ICU (C) injection of vocal cords with silicone (D) tracheostomy (E) routine esophagography page 1016 . Her chest x-ray is also shown.American College of Surgeons SESAP 13 Category 11 Item 25 The patient pictured is scheduled to undergo thyroidectomy for presumed multinodular goiter. Newman E. When the recurrent laryngeal nerve is damaged. However.American College of Surgeons SESAP 13 Critique for Category 11 Item 25 This patient has a large multinodular goiter. with possible leakage into the pleural space. REFERENCES 1. Tracheomalacia is a weakness of the trachea. and the trachea cannot maintain its shape. and tracheostomy should be performed. Tracheomalacia should be suspected in all patients with large multinodular goiters.Carden KA. vocal cord paresis or paralysis secondary to recurrent laryngeal nerve involvement. infusions of calcium are indicated. Hypocalcemia may occur relatively acutely because of damage to the parathyroids or their blood supply. the cord stops at the paramedian position. but is not appropriate for airway problems. Am Surg 60:586-591. If recognized. although it may be difficult to determine whether this is due to tracheomalacia. The endotracheal tube is pulled back superior to the level of the deformity.McHenry CR. tracheomalacia would be most life threatening. Ernst A: Tracheomalacia and tracheobronchomalacia in children and adults: an in-depth review. the trachea must be evaluated intraoperatively. Preoperative evaluation should include laryngoscopy to evaluate vocal cord function and pulmonary function testing to differentiate pulmonary parenchymal disease from airway obstruction. Shaha AR: Substernal goiter. 1994 3. and the chest x-ray demonstrates narrowing of the trachea by the goiter. With respect to multinodular goiter. 1995 page 1017 . If the trachea collapses with spontaneous breaths. Most patients with large goiter will not have tracheomalacia and can be extubated without difficulty. There is no indication for injection of any nerve with silicone. After completion of the thyroidectomy. not in the midline. the balloon of the endotracheal tube should be deflated and the patient allowed to breath spontaneously. Permanent hypoparathyroidism may be difficult to treat but is unlikely to result in death. Waltz DA. and outcome. tracheomalacia should be suspected. Piotrowski JJ: Thyroidectomy in patients with marked thyroid enlargement: airway management. J Surg Oncol 60:207-212. Injury to the esophagus in the neck or even upper mediastinum is unlikely to cause the problems caused by injury in the thorax. frequently due to reduction and/or atrophy of the longitudinal elastic fibers or the pars membranacea or impaired integrity of the cartilageous rings. which is usually enough to allow inspiration and expiration. The trachea is observed directly with a bronchoscope. approximately a third of the airway remains patent. Symptoms are usually dyspnea. Even with bilateral recurrent nerve injury. or concomitant pulmonary disease. If it is soft and deformed. Although the patient is at risk for all the listed complications. leading to a softer airway more prone to collapse. morbidity. and is unlikely to be lifethreatening. Patients may or may not be symptomatic. the presence of tracheomalacia is confirmed. the constant pressure on the trachea leads to softness of the tracheal rings. Injection of a paralyzed cord with silicone is sometimes used to medialize the cord and improve speech. Boiselle PM. 2005 2. Chest 127:984-1005. and sex-adjusted incidence is increasing (B) It is commonly associated with indwelling vascular devices (C) The age at diagnosis is decreasing (D) Congenital heart disease is the most common predisposing cardiac condition (E) It rarely occurs in patients with a normal heart page 1018 .American College of Surgeons SESAP 13 Category 11 Item 26 Which of the following statements about infective endocarditis in the United States is TRUE? (A) The age. multicenter international cohort of adults with infective endocarditis from 39 medical centers in 16 countries.Tleyjeh IM. 2005 page 1019 .Quagliarello V: Infective endocarditis: global. mitral valve prolapse (MVP) is now believed to be the most common predisposing cardiac condition in patients with IE. persistent bacteremia. In the last 5 years of the study. including methicillin-resistant S aureus (MRSA). is currently seen in developing countries where rheumatic heart disease is still prevalent. it remains a persistent and serious clinical condition in local communities and around the globe. S aureus was the most common pathogen identified. an intravascular device is the presumed source of bacteremia. The global emergence of health care contact as a critical risk factor for S aureus IE has now been established. JAMA 293:30613062. age. et al: Temporal trends in infective endocarditis: a population-based study in Olmsted County. and to have higher rates of complications such as stroke. In a population-based epidemiologic study in Minnesota evaluating IE episodes in a single county. the Middle East. Minnesota. with a mean age of 67. Health care-associated infection was the most common form of S aureus endocarditis. systemic embolization. In a recent prospective. and future perspectives. The difference in mortality between MRSA. have chronic illnesses. In developed countries. IE continues to be a disease of older persons. although most cases are still seen in patients with normal hearts. REFERENCES 1. regional. et al: Staphylococcus aureus endocarditis: a consequence of medical progress. most IE cases occurred in patients with no known underlying heart disease. Patients with IE due to S aureus were more likely than patients with IE due to other pathogens to be female. JAMA 293:3022-3028. have also increased. Murad HS. Steckelberg JM. Anavekar NS. There was a declining trend in proportion with rheumatic heart disease and an increasing trend in prosthetic valve and MVP proportions over time. Endocarditis due to MRSA was more common in the United States and Brazil than in Europe. and death. Taken together. 2005 3. or Australia/New Zealand. intravenous catheters for outpatient infusion therapy. JAMA 293:3012-3021. these findings indicate that S aureus IE is increasingly a consequence of medical progress. 2005 2. With increasing use of intravascular devices.4 years in 1995 to 2000. yet more than 60% of these infections were acquired outside the hospital.Fowler VG Jr. Infective endocarditis (IE) caused by rheumatic carditis.and MSSA-infected subjects did not achieve statistical significance. Miro JM. S aureus IE has also been associated with immunosuppression. with viridan group streptococci as the most common causative pathogen. opportunities for patients to become colonized and infected with Staphylococcus aureus. In a significant proportion of patients.American College of Surgeons SESAP 13 Critique for Category 11 Item 26 Although endocarditis is relatively uncommon. prosthetic valves. and patient placement in long-term care facilities. Hoen B.and sex-adjusted incidence of IE did not change significantly over time. Cabell CH. American College of Surgeons SESAP 13 Category 11 Item 27 Which of the following statements about Staphylococcus aureus infective endocarditis (IE) is TRUE? (A) Most patients are male (B) Most patients with health-care associated S aureus IE acquire the infection in the hospital (C) The risk of morbidity and mortality is the same as with non S aureus IE (D) Methicillin resistance does not affect mortality rates (E) Persistent bacteremia is a distinct characteristic in all cases page 1020 . Cabell CH. S aureus was the most common pathogen identified.American College of Surgeons SESAP 13 Critique for Category 11 Item 27 Although endocarditis is relatively uncommon. opportunities for patients to become colonized and infected with Staphylococcus aureus. Health care-associated infection was the most common form of S aureus endocarditis. and death. The difference in mortality between MRSA. In a significant proportion of patients.and sex-adjusted incidence of IE did not change significantly over time. although most cases are still seen in patients with normal hearts. or Australia/New Zealand. these findings indicate that S aureus IE is increasingly a consequence of medical progress. have also increased.Tleyjeh IM. There was a declining trend in proportion with rheumatic heart disease and an increasing trend in prosthetic valve and MVP proportions over time. REFERENCES 1. With increasing use of intravascular devices. mitral valve prolapse (MVP) is now believed to be the most common predisposing cardiac condition in patients with IE. Patients with IE due to S aureus were more likely than patients with IE due to other pathogens to be female. it remains a persistent and serious clinical condition in local communities and around the globe. including methicillin-resistant S aureus (MRSA). an intravascular device is the presumed source of bacteremia. most IE cases occurred in patients with no known underlying heart disease. In developed countries. Miro JM. systemic embolization. In the last 5 years of the study. Taken together.4 years in 1995 to 2000. and to have higher rates of complications such as stroke. have chronic illnesses. JAMA 293:3012-3021. persistent bacteremia. multicenter international cohort of adults with infective endocarditis from 39 medical centers in 16 countries. The global emergence of health care contact as a critical risk factor for S aureus IE has now been established. Minnesota. 2005 2. Anavekar NS. and future perspectives.Fowler VG Jr. Infective endocarditis (IE) caused by rheumatic carditis. Hoen B. 2005 3. with viridan group streptococci as the most common causative pathogen. age. regional. is currently seen in developing countries where rheumatic heart disease is still prevalent. 2005 page 1021 . S aureus IE has also been associated with immunosuppression. In a population-based epidemiologic study in Minnesota evaluating IE episodes in a single county. yet more than 60% of these infections were acquired outside the hospital.Quagliarello V: Infective endocarditis: global. with a mean age of 67. IE continues to be a disease of older persons. the Middle East. In a recent prospective. Murad HS. et al: Staphylococcus aureus endocarditis: a consequence of medical progress. Endocarditis due to MRSA was more common in the United States and Brazil than in Europe. Steckelberg JM. JAMA 293:3022-3028. intravenous catheters for outpatient infusion therapy. JAMA 293:30613062.and MSSA-infected subjects did not achieve statistical significance. prosthetic valves. and patient placement in long-term care facilities. et al: Temporal trends in infective endocarditis: a population-based study in Olmsted County. American College of Surgeons SESAP 13 Category 11 Item 28 Symptoms due to compression of adjacent structures (A) Figure A (B) Figure B (C) Both (D) Neither page 1022 . primarily for diagnostic purposes. McElvein RB: Percutaneous fine needle aspiration in the diagnosis and management of mediastinal cysts in adults. In addition. In contradistinction to bronchogenic cysts. Surgical excision is recommended in all patients with congenital foregut cysts to provide definitive pathologic diagnosis. they are not associated with malignant degeneration. These cysts may develop in the mediastinum or within the lung parenchyma. They most commonly occur at the cardiophrenic angles. 1988 page 1023 . a lesion showing these characteristics can be managed with simple aspiration or observation. although superimposed infection can lead to erosion into the airway. they appear as unilocular cysts at the right cardiophrenic angle with a smooth border and a Hounsfield number of almost 0 by CT scan. these benign cysts can cause compression of the adjacent airway or esophagus. ed 2. Deslauriers J. Bronchoscopic aspiration and mediastinoscopic or percutaneous drainage are therapeutic alternatives in high-risk surgical patients. infectious complications have been reported with simple computed tomographic (CT)-guided needle drainage alone.Nath PH. however. South Med J 81:1225-1228. They are lined with respiratory epithelium and contain a milky white or brown mucoid material.American College of Surgeons SESAP 13 Critique for Category 11 Item 28 Congenital foregut cysts account for 15% to 20% of mediastinal masses.Cangemi V. the antecedent of the esophagus and tracheobronchial tree. Pleuro-pericardial cysts are the second most common mediastinal cysts. comprising 60% of all cysts. REFERENCES 1. which is not recommended as initial treatment. Cooper JD. et al: Pericardial cysts of the mediastinum. pp 1647-1654 2. but most often they originate near the carina. 2002. Polettini E. these techniques do not retrieve cyst wall tissue for analysis and recurrence is likely. unilocular lesions that occur equally in men and women. They rarely cause symptoms secondary to compression. although therapeutic excision is performed on occasion. This form of needle aspiration should be reserved for patients unable to tolerate a surgical procedure.Allen MS: Cysts and duplications in adults. Ginsberg RJ. Sanders C. occurring within the middle mediastinum. Direct communication with the tracheobronchial tree is rare. Malignant degeneration is extremely rare. with the goal of short-term palliation of symptoms only. Surgical excision is performed occasionally. Gualdi G. Holley HC. Most cysts are asymptomatic because of their small size and failure to compress surrounding structures. Bronchogenic cysts are the most common congenital foregut cysts. There are important differences in the clinical characteristics of the various types of foregut cysts that direct their management. However. Classically. Churchill Livingstone. These masses have a characteristic radiographic appearance. They originate from sequestrations of the ventral foregut. to prevent future complications. or can become infected with resultant mediastinitis. and to alleviate symptoms. J Cardiovasc Surg 40:909-913. et al (eds): Thoracic Surgery. 1999 3. in Pearson FG. Volpino P. They are spherical. these lesions are typically not prone to infection. The remaining 8% are found in other sites on the pericardium. Owing to their lack of communication with the tracheobronchial tree or esophagus. Philadelphia. homogeneous. With increasing size. As is the case with foregut cysts. These cysts arise from failure of the mesenchymal lacunae to fuse normally to form the pericardial sac. They make up one third of all primary cysts of the mediastinum. 70% on the right and 22% on the left. Minami M. Results in 20 consecutive surgically treated cases.Sirivella S. Nesbitt JC: Middle mediastinal masses. 1985 5. Zikria EA. Cameron DE (eds): Current Therapy in Thoracic and Cardiovascular Surgery. J Thorac Cardiovasc Surg 90:776-782. 2003 6. A unique clinical problem. in Yang SC.American College of Surgeons SESAP 13 4. Ohta M. 2000 7. Mosby. Miller WH. Rice TW: Subcarinal foregut cysts.Zikri MA. J Cardiovasc Surg 41:137141. Philadelphia. Miyoshi S. et al: Clinical spectrum of mediastinal cysts. et al: Foregut cysts of the mediastinum. Chest 124:125132.Takeda S.Zorn GL III. Ford WB. 2004. pp 330-332 page 1024 . American College of Surgeons SESAP 13 Category 11 Item 29 Initial treatment CT-guided needle drainage of fluid (A) Figure A (B) Figure B (C) Both (D) Neither page 1025 . McElvein RB: Percutaneous fine needle aspiration in the diagnosis and management of mediastinal cysts in adults. Malignant degeneration is extremely rare. Surgical excision is performed occasionally.Cangemi V. Most cysts are asymptomatic because of their small size and failure to compress surrounding structures. Classically.American College of Surgeons SESAP 13 Critique for Category 11 Item 29 Congenital foregut cysts account for 15% to 20% of mediastinal masses. et al: Pericardial cysts of the mediastinum. J Cardiovasc Surg 40:909-913. However. These cysts arise from failure of the mesenchymal lacunae to fuse normally to form the pericardial sac. These masses have a characteristic radiographic appearance.Allen MS: Cysts and duplications in adults. Surgical excision is recommended in all patients with congenital foregut cysts to provide definitive pathologic diagnosis. There are important differences in the clinical characteristics of the various types of foregut cysts that direct their management. The remaining 8% are found in other sites on the pericardium. occurring within the middle mediastinum. the antecedent of the esophagus and tracheobronchial tree. This form of needle aspiration should be reserved for patients unable to tolerate a surgical procedure. Holley HC. in Pearson FG. primarily for diagnostic purposes. They originate from sequestrations of the ventral foregut. Ginsberg RJ. a lesion showing these characteristics can be managed with simple aspiration or observation. With increasing size. 1988 page 1026 . they are not associated with malignant degeneration. infectious complications have been reported with simple computed tomographic (CT)-guided needle drainage alone. Cooper JD. They are lined with respiratory epithelium and contain a milky white or brown mucoid material. Sanders C. In contradistinction to bronchogenic cysts. these lesions are typically not prone to infection. Bronchogenic cysts are the most common congenital foregut cysts. or can become infected with resultant mediastinitis. with the goal of short-term palliation of symptoms only. Philadelphia. 2002. Pleuro-pericardial cysts are the second most common mediastinal cysts. Gualdi G. Owing to their lack of communication with the tracheobronchial tree or esophagus. Polettini E. South Med J 81:1225-1228. comprising 60% of all cysts. They most commonly occur at the cardiophrenic angles. to prevent future complications. They are spherical. 1999 3. These cysts may develop in the mediastinum or within the lung parenchyma. In addition. et al (eds): Thoracic Surgery. ed 2. homogeneous. they appear as unilocular cysts at the right cardiophrenic angle with a smooth border and a Hounsfield number of almost 0 by CT scan. Bronchoscopic aspiration and mediastinoscopic or percutaneous drainage are therapeutic alternatives in high-risk surgical patients. however. although superimposed infection can lead to erosion into the airway. unilocular lesions that occur equally in men and women. which is not recommended as initial treatment.Nath PH. Deslauriers J. They make up one third of all primary cysts of the mediastinum. although therapeutic excision is performed on occasion. these benign cysts can cause compression of the adjacent airway or esophagus. Direct communication with the tracheobronchial tree is rare. They rarely cause symptoms secondary to compression. pp 1647-1654 2. Churchill Livingstone. As is the case with foregut cysts. and to alleviate symptoms. REFERENCES 1. Volpino P. 70% on the right and 22% on the left. these techniques do not retrieve cyst wall tissue for analysis and recurrence is likely. but most often they originate near the carina. Philadelphia. 2000 7. Chest 124:125132. Minami M.Zorn GL III. Ford WB. Cameron DE (eds): Current Therapy in Thoracic and Cardiovascular Surgery. pp 330-332 page 1027 . 1985 5. Nesbitt JC: Middle mediastinal masses.Zikri MA. et al: Foregut cysts of the mediastinum. Zikria EA. Results in 20 consecutive surgically treated cases. A unique clinical problem.Takeda S. 2004. Mosby. Rice TW: Subcarinal foregut cysts. Ohta M. 2003 6. in Yang SC. Miyoshi S. J Cardiovasc Surg 41:137141. J Thorac Cardiovasc Surg 90:776-782.Sirivella S.American College of Surgeons SESAP 13 4. Miller WH. et al: Clinical spectrum of mediastinal cysts. American College of Surgeons SESAP 13 Category 11 Item 30 Prone to infection (A) Figure A (B) Figure B (C) Both (D) Neither page 1028 . The remaining 8% are found in other sites on the pericardium. these benign cysts can cause compression of the adjacent airway or esophagus. They make up one third of all primary cysts of the mediastinum. There are important differences in the clinical characteristics of the various types of foregut cysts that direct their management. These cysts arise from failure of the mesenchymal lacunae to fuse normally to form the pericardial sac. They originate from sequestrations of the ventral foregut. They are lined with respiratory epithelium and contain a milky white or brown mucoid material. In addition. 1988 page 1029 . infectious complications have been reported with simple computed tomographic (CT)-guided needle drainage alone. however. Ginsberg RJ. Gualdi G. et al (eds): Thoracic Surgery. these lesions are typically not prone to infection. primarily for diagnostic purposes. Classically. Sanders C. although superimposed infection can lead to erosion into the airway. McElvein RB: Percutaneous fine needle aspiration in the diagnosis and management of mediastinal cysts in adults. They rarely cause symptoms secondary to compression.Allen MS: Cysts and duplications in adults. Volpino P. to prevent future complications. Owing to their lack of communication with the tracheobronchial tree or esophagus. Holley HC. They most commonly occur at the cardiophrenic angles.American College of Surgeons SESAP 13 Critique for Category 11 Item 30 Congenital foregut cysts account for 15% to 20% of mediastinal masses. Deslauriers J. With increasing size. or can become infected with resultant mediastinitis. Most cysts are asymptomatic because of their small size and failure to compress surrounding structures. these techniques do not retrieve cyst wall tissue for analysis and recurrence is likely. Philadelphia. the antecedent of the esophagus and tracheobronchial tree. Pleuro-pericardial cysts are the second most common mediastinal cysts. This form of needle aspiration should be reserved for patients unable to tolerate a surgical procedure. 1999 3. comprising 60% of all cysts. with the goal of short-term palliation of symptoms only. Cooper JD. in Pearson FG. Bronchoscopic aspiration and mediastinoscopic or percutaneous drainage are therapeutic alternatives in high-risk surgical patients. but most often they originate near the carina. Surgical excision is performed occasionally. They are spherical. Bronchogenic cysts are the most common congenital foregut cysts. As is the case with foregut cysts. However. In contradistinction to bronchogenic cysts. These masses have a characteristic radiographic appearance. although therapeutic excision is performed on occasion. unilocular lesions that occur equally in men and women. and to alleviate symptoms. Polettini E. Malignant degeneration is extremely rare. they are not associated with malignant degeneration. occurring within the middle mediastinum. et al: Pericardial cysts of the mediastinum. Direct communication with the tracheobronchial tree is rare. which is not recommended as initial treatment. a lesion showing these characteristics can be managed with simple aspiration or observation. they appear as unilocular cysts at the right cardiophrenic angle with a smooth border and a Hounsfield number of almost 0 by CT scan. Churchill Livingstone. REFERENCES 1.Cangemi V.Nath PH. ed 2. South Med J 81:1225-1228. pp 1647-1654 2. J Cardiovasc Surg 40:909-913. Surgical excision is recommended in all patients with congenital foregut cysts to provide definitive pathologic diagnosis. homogeneous. 70% on the right and 22% on the left. 2002. These cysts may develop in the mediastinum or within the lung parenchyma. Minami M. 1985 5. 2003 6.Zikri MA. 2004. Philadelphia.Takeda S. Mosby. 2000 7. Cameron DE (eds): Current Therapy in Thoracic and Cardiovascular Surgery. Ohta M. A unique clinical problem. Miller WH. Miyoshi S. et al: Foregut cysts of the mediastinum. in Yang SC. pp 330-332 page 1030 .Sirivella S.Zorn GL III. Zikria EA.American College of Surgeons SESAP 13 4. Ford WB. et al: Clinical spectrum of mediastinal cysts. J Cardiovasc Surg 41:137141. Rice TW: Subcarinal foregut cysts. Nesbitt JC: Middle mediastinal masses. J Thorac Cardiovasc Surg 90:776-782. Chest 124:125132. Results in 20 consecutive surgically treated cases. Laparoscopic biopsy of one of the lesions with frozen-section analysis reveals metastatic ovarian adenocarcinoma.American College of Surgeons SESAP 13 Category 12 Item 1 At laparoscopy for acute appendicitis. a 50-year-old woman has a right ovarian mass with omental and peritoneal lesions consistent with carcinomatosis and no appendicitis. The most important factor in this patient's long-term survival is (A) chemoresistance assay profiles from the biopsy material (B) the degree of cytoreduction achieved (C) tumor grade (D) number of peritoneal sites involved with tumor (E) size of the metastatic deposits page 1031 . This patient has only undergone laparoscopy. Brady MF. et al: Survival effect of maximal cytoreductive surgery for advanced ovarian carcinoma during the platinum era: a meta-analysis. Neither tumor grade nor in vitro chemoresistance of the tumor is as important as the completeness of cytoreduction. Tomacruz RS. Penalver M. and referral to a subspecialist should be considered. Armstrong DK. 2005 3. bilateral salpingo-oophorectomy (BSO). REFERENCES 1. Averette HE.Bristow RE.Rose PG. N Engl J Med 351:2489-2497. Arvas M. Ortac F. The impact of physician's specialty on patients' survival. et al: National survey of ovarian carcinoma. The surgeon must assess his/her ability to perform the appropriate operation from both a diagnostic and therapeutic standpoint. Nerenstone S. Part V. 1995 4. not the initial size or number of the metastatic deposits.Gungor M. Clarke-Pearson D. and resection of all peritoneal lesions if feasible. total abdominal hysterectomy (TAH). omentectomy. 1993 5.American College of Surgeons SESAP 13 Critique for Category 12 Item 1 The primary determinant of survival in ovarian cancer is the degree of cytoreduction. chemoresistence assays are most commonly used in patients with recurrent or refractory disease. Gynecol Oncol 97:74-79. Trimble EL. Hoskins W. Ann Med 27:95-100. The degree of cytoreduction should be noted because the presence of residual disease adversely affects prognosis and may direct further therapy. Kosebay D. J Clin Oncol 20:1248-1259. Cancer 72:3663-3670. The appropriate surgical procedure would be peritoneal washings. Some data suggest a survival improvement in patients treated by gynecologic oncologists. et al: Secondary surgical cytoreduction for advanced ovarian carcinoma. 2004 page 1032 . Because essentially all women with advanced stage ovarian carcinomas will receive platinum-based systemic chemotherapy as first-line therapy. 2002 2. The completeness of the initial operation is critical in reducing morbidity and improving survival. et al: The role of secondary cytoreductive surgery for recurrent ovarian cancer.Hogberg T: Primary surgery in ovarian cancer: current opinions.Nguyen HN. American College of Surgeons SESAP 13 Category 12 Item 2 For a patient suspected of having pelvic inflammatory disease. and chronic pelvic pain (E) Adnexal tenderness is more commonly bilateral page 1033 . ectopic pregnancy. which of the following is NOT true? (A) Tubo-ovarian abscesses of 4 cm frequently require surgical drainage (B) The use of laparoscopy for diagnosis has increased (C) Most patients initially present to their primary care physicians (D) Long-term sequelae include infertility. cervical motion tenderness.American College of Surgeons SESAP 13 Critique for Category 12 Item 2 Pelvic inflammatory disease (PID) primarily affects young sexually active reproductive-age women. Both are polymicrobial with a predominance of anaerobic organisms. In most cases these infections arise from the sexually transmitted pathogens Chlamydia trachomatis and Neisseria gonorrhoeae. ovary.to 6-cm masses require surgical drainage versus 70% of those with masses 10 cm or larger. elevated sedimentation rate or C-reactive protein. 2003 page 1034 . Treatment goals should be directed not only at the acute inflammatory condition. mucopurulent cervicitis. and the presence of a tubo-ovarian abscess. Long-term sequelae of PID include infertility. the larger proportion of women with mild disease may not be identified.These patients may present with right upper quadrant pain and mildly elevated liver function tests that can be mistaken for liver or gallbladder disease. REFERENCES 1. Recent studies from the Centers for Disease Control (CDC) using discharge and outpatient database information estimate that approximately 780. or presence of inflammatory mass on pelvic sonography. documentation of cervical infection with C trachomatis or N gonorrhoeae. Wiesenfeld HC: Pelvic inflammatory disease: new diagnostic criteria and treatment. If inflammatory masses are identified at the time of laparoscopy. Obstet Gynecol Clin North Am 30:777-793. but also at prevention of these long-term sequelae of risks. Currently. Other supporting clinical presentations include oral temperature above 38. It is difficult to determine whether the actual incidence has decreased or has been altered as a result of treatment shifting to the outpatient setting. For patients presenting with acute severe peritonitis or in patients who appear ill with an unclear diagnosis. The finding of a TOA necessitates a hospital admission for intravenous antibiotic therapy and close monitoring for signs of rupture or unfavorable response to antibiotics.3°C. the patient can undergo culture and drainage for diagnostic confirmation. This is actually less than the 1 million annual cases estimated in the United States during the 1970s and 1980s. The size of the TOA is proportional to the need for surgical intervention: 15% of women with 4. a complication of PID that is characterized by an inflammatory mass involving the fallopian tube. One such example is a tubo-ovarian abscess (TOA). inability to exclude a surgical emergency such as appendicitis. While it is unusual to recover N gonorrhoeae and C trachomatis from the abscess. and bilateral adnexal tenderness on pelvic examination. thus decreasing emergency department visits and hospital admissions. and adjacent structures. Patients typically present clinically with the triad of lower abdominal tenderness. severe illness manifesting with high fever or peritonitis (only present in one third of patients). Fitz-Hugh-Curtis syndrome (ascending perihepatitis) is characterized by inflammation and adhesion formation that involve the liver capsule. ectopic pregnancy. failure to respond to outpatient oral antibiotic therapy. these organisms play a role in inciting the ascent of the various microorganisms into the upper genital tract. inability to tolerate oral intake. In 2002 the CDC criteria for hospitalization of patients with PID included: pregnancy. The microbiology of TOAs is similar to PID infections. If these presentations are used alone. most patients initially present to their primary care physicians.000 cases of acute PID are diagnosed annually. and chronic pelvic pain.Beigi RH. diagnostic laparoscopy is appropriate. Warburton F. 2003 page 1035 .Simms I.American College of Surgeons SESAP 13 2. Sex Transm Infect 79:491-494. Westrom L: Diagnosis of pelvic inflammatory disease: time for a rethink. as shown. nausea. A plain abdominal x-ray is obtained. Two years ago she had widespread ovarian cancer treated with a debulking operation that included a right hemicolectomy and postoperative chemotherapy.American College of Surgeons SESAP 13 Category 12 Item 3 A 60-year-old woman has a 2-day history of increasing abdominal distention. Which of the following statements is TRUE? (A) Bowel obstruction is rare in ovarian cancer (B) Medical management with octreotide is not as effective as operation for symptom relief (C) Operation provides no survival advantage over medical management with octreotide (D) Proximal colostomy is required (E) Stenting can be as effective as operation in patients with distal stricture page 1036 . She began a second course of chemotherapy a month ago for presumed recurrent disease and appears to have distal colonic obstruction. and vomiting. She has passed only some gas and a small amount of fluid. Pothuri B. Theodosopoulos T. In one study. 2004 3. Gerardi M. Carter J: Management of upper gastrointestinal obstruction in advanced ovarian cancer with intraluminal stents. One retrospective study describing the management of 20 patients using octreotide and 27 patients who had surgical exploration found that both octreotide and operation controlled immediate symptoms. Frigerio L. 2004 4. Surg Endosc 18:421-426. Barakat RR.Mangili G. 2004 page 1037 . Stents have been less successful for management of upper gastrointestinal obstruction. and only 3 of those had successful palliation for 60 days or longer. Gynecol Oncol 95:739-741. Franchi M. Management is both unsatisfactory and controversial. Long-term survival was better in the surgical group. There has been considerable interest in the use of stents. REFERENCES 1.American College of Surgeons SESAP 13 Critique for Category 12 Item 3 Bowel obstruction is the most common complication in patients with ovarian cancer. In another study of 10 patients undergoing surgical exploration. et al: Stenting or stoma creation for patients with inoperable malignant colonic obstructions? Results of a study and costeffectiveness analysis. et al: Reoperation for palliation of recurrent malignant bowel obstruction in ovarian carcinoma.Xinopoulos D. but the perioperative mortality was 22%. Aletti G. et al: Palliative care for intestinal obstruction in recurrent ovarian cancer: a multivariate analysis. Int J Gynecol Cancer 15:830-835. Meyer L. Tsamakidis K. self-expanding metallic stents for colon obstruction were successful in 14 of 15 patients.Rao A. 2005 2. Dimitroulopoulos D. proximal colostomy. Gynecol Oncol 95:193-195. Land R. and were believed to be superior to the comparison therapy. surgical correction was possible for only 5. especially for obstruction of the large bowel in the pelvis. The most appropriate diagnostic test would be (A) cervical cultures. She is uncertain of the timing of her last menses.American College of Surgeons SESAP 13 Category 12 Item 4 A 28-year-old woman with a history of repeated episodes of pelvic inflammatory disease presents with intermittent vaginal spotting associated with crampy pelvic pain. WBC count (B) serum and urine β-HCG levels (C) computed tomographic (CT) scan (D) transabdominal ultrasound study of the uterus and ovaries (E) laparoscopy page 1038 . Abdominal computed tomographic (CT) scan is unlikely to be diagnostic. hyperosmolar glucose solution. Transabdominal ultrasonography is not as sensitive as transvaginal ultrasonography in ectopic pregnancy. This might be related to the most common risk factor for ectopic pregnancy. Kucera E.American College of Surgeons SESAP 13 Critique for Category 12 Item 4 Ectopic pregnancy has increased in incidence from 0. Jirecek S. particularly at an early stage of ectopic pregnancy. decreasing the chance for successful uterine implantation of the embryo. In patients with a β-HCG <1000 that is falling. including systemic methotrexate. 2005 2.Lehner R. and NaCl. Multiple episodes of pelvic inflammatory disease (PID) damage the ciliary action in the fallopian tubes. and of the potential impact of the various options on future fertility. symptoms are more typical of ectopic pregnancy. noninvasive options for terminating the pregnancy. et al: Ectopic pregnancy. 2000 page 1039 .5% to up to 2% of pregnancies over the last 30 years. Egarter C. and may mistake a pseudogestational sac for a true gestational sac.Farquhar CM: Ectopic pregnancy. REFERENCES 1. local injection of the gestational sac with prostaglandins. In any case. and this should be the focus of her work-up considering the potentially life-threatening complications that may ensue. While a current episode of PID is a possible diagnosis in this patient. There are other. a tubal pregnancy not in imminent danger of rupture allows for a discussion of treatment options with the patient. prior inflammation of the fallopian tubes. Lancet 366:583-591. and observation may be all that is required. Its reliability is perhaps 70% versus 90% for transvaginal ultrasonography. the tubal pregnancy is likely resorbing. Arch Gynecol Obstet 263:87—92. Exploratory laparoscopy may be excessively invasive when the patient has not yet demonstrated signs of rupture. and palpitations. The most appropriate therapy for this patient would be (A) carbimazole (B) propylthiouracil (PTU) (C) radioactive iodine treatment (D) total thyroidectomy (E) subtotal thyroidectomy page 1040 . tremors.01 IU. She also has positive anti-TSH receptor antibodies.American College of Surgeons SESAP 13 Category 12 Item 5 A 23-year-old woman in her second trimester of pregnancy has symptoms of heat intolerance. Laboratory studies reveal a suppressed serum thyroid-stimulating hormone (TSH) level of 0. J Surg Res 133:207-214. operation. in Falk SA (ed): Thyroid Disease: Endocrinology.Schussler-Fiorenza CM. 1997. in Clark OH. when radioactive iodine would be an option. The two definitive therapies are radioactive iodine and operation. Duh Q-Y. Definitive therapy could occur after delivery of the fetus. Treatment options include antithyroid drugs such as carbimazole and propylthiouracil (PTU). Chen H: The surgical management of Graves’ disease. Kebebew E (eds): Textbook of Endocrine Surgery. REFERENCES 1. Perrier ND: Grave’s and Plummer’s diseases: medical and surgical management. Elsevier Saunders. and Radiotherapy. Surgery. who is in her second trimester. pp 253-296 3. antithyroid drugs are used to control the hyperthyroidism and rarely result in long-term cure.Mechanick JI. In general. then a surgical approach could be taken during the third trimester. Nuclear Medicine. Carbimazole is not indicated due to potential fetal toxicity. pp 54-67 2. If her hyperthyroidism is difficult to control. 2005. ed 2.Boger MS. Lippincott-Raven. Philadelphia. Bruns CM. or radioactive iodine. 2006 page 1041 . PTU is preferred in pregnancy and lactation due to higher protein binding. as evidenced by the suppressed thyroid-stimulating hormone (TSH) level and the positive anti-TSH receptor antibodies. usually total thyroidectomy. ed 2.American College of Surgeons SESAP 13 Critique for Category 12 Item 5 The patient in this scenario has hyperthyroidism due to Graves’ disease during pregnancy. Davies TF: Medical management of hyperthyroidism: theoretical and practical aspects. Philadelphia. For this patient. the best course of action would be to administer PTU to control her hyperthyroidism. American College of Surgeons SESAP 13 Category 12 Item 6 An otherwise healthy 45-year-old woman has acute right lower quadrant pain. A normal appendix and innumerable blue-colored lesions involving the ovaries. Biopsy identifies these as endometrial tissue. uterus. The most appropriate therapy would be (A) appendectomy and surgical removal of the implants (B) hysterectomy and bilateral salpingo-oophorectomy (C) appendectomy (D) right hemicolectomy (E) laser ablation of the implants page 1042 . and most of the peritoneal surfaces in the pelvis are identified through a right lower quadrant incision. a complex disease with several different therapeutic approaches.American College of Surgeons SESAP 13 Critique for Category 12 Item 6 Unsuspected findings during exploration through a right lower quadrant incision present a difficult intraoperative surgical dilemna. Endometriomas are usually resistant to medical management. refractory. 1990 2. or extensive disease. Therapy includes medical management with progestins. as in this case. Surg Gynecol Obstet 171:95-98. with definitive management deferred to an elective setting if possible. danazol.Marcoux S. In a young patient with a high likelihood of recurrent abdominal complaints. Pain or a mass resulting from cyclic enlargement of an implant from endometriosis is termed an endometrioma. These will ameliorate the pain. or gonadotropin-releasing hormone (GnRH) analogs. the appendix should be removed now to avoid confusion in the future. and the patient’s age and desire for pregnancy. or resection via laparotomy or laparoscopy is usually needed to remove the endometriomas. the extent and location of disease. Canadian Collaborative Group on Endometriosis. Maheux R.Fisher KS. and ablation. may even require hysterectomy and bilateral salpingo-oophorectomy or resection of small or large bowel in an effort to remove the burden of endometriomas within the pelvis. but will not enhance fertility. N Engl J Med 337:217-222. This patient will likely require the complete spectrum of therapy for this disorder. In this case. REFERENCES 1. The treatment of endometriosis depends on the severity of symptoms. A recent randomized clinical trial of women with mild to moderate endometriosis demonstrated increased fecundity in women who received laparoscopic resection or ablation. Berube S: Laparoscopic surgery in infertile women with minimal or mild endometriosis. commonly using a laser. This patient has severe pelvic endometriosis. applied in a stepwise fashion with appropriate counseling. Patients with previously unidentified problems that are not an immediate threat should undergo diagnostic or staging procedures. Ross DS: Guidelines for therapeutic decision in incidental appendectomy. given the presence of a right lower quadrant incision. Patients with recurrent. 1997 page 1043 . biopsy has clearly identified this patient’s problem as symptomatic endometriosis occupying the peritoneal surface of the pelvis. American College of Surgeons SESAP 13 Category 12 Item 7 Which of the following statements about cervical cancer is TRUE? (A) Screening with annual Pap tests should begin at age 25 for average-risk women (B) Screening with annual Pap smears should continue until age 60 (C) Screening of average-risk patients may be done every other year if the liquid-based Pap test is used (D) Women who test positive for human papilloma virus require both earlier and more frequent cervical cytologic screening (E) A single Pap smear has a sensitivity of 85% for detecting cervical intraepithelial neoplasia page 1044 . Moscicki AB. CA Cancer J Clin 52:342-362. PAP smears have a sensitivity of only 50% to 60% for detecting cervical intraepithelial neoplasia. the putative causative agent for cervical cancer. Cokkinides V. Much of this reduction has been attributed to widespread screening with Pap smears.Saslow D. Runowicz CD. Solomon D. however. REFERENCES 1. Beginning at age 30. the established recommendations for screening do not change for patients known to be infected with the HPV virus. 2004.American College of Surgeons SESAP 13 Critique for Category 12 Item 7 Both the incidence and the mortality of cervical cancer have decreased by 67% in the past decade. The American Cancer Society currently recommends that cervical cancer screening with annual Pap smears begin 3 years after the first experience with vaginal intercourse. may become an important tool in screening for this malignancy. women of average risk can be screened every 3 years. Curr Opin Obstet Gynecol 15:473-477. 2003 2.Smith RA. Testing for human papilloma virus (HPV). but may be done every 2 years if the more sensitive liquid-based Pap test is used. Screening should be done yearly with standard Pap tests. Screening should continue until the age of 70. 2004 page 1045 . Currently. American Cancer Society: American Cancer Society guidelines for the early detection of cancer. but no later than 21 years of age. et al: American Cancer Society guideline for the early detection of cervical neoplasia and cancer. Freeman-Wang T: Human papilloma virus and cervical screening. CA Cancer J Clin 54:41-52.Damasus-Awatai G. Eyre HJ. 2002 3. She has had multiple episodes of pelvic inflammatory disease and had mild pelvic pain 2 weeks ago associated with vaginal discharge. This patient is LEAST likely to have (A) pericholecystic inflammation/gallbladder wall thickening without stones on ultrasound study (B) perihepatic inflammation/violin strings on exploration (C) associated chlamydial infections (D) contrast enhancement along liver edge on computed tomography (CT) (E) a normal risk of ectopic pregnancy page 1046 .American College of Surgeons SESAP 13 Category 12 Item 8 A 28-year-old woman presents with a 1-day history of right upper quadrant pain. patients present with sharp right upper quadrant pain and pleuritic pain radiating to shoulder. Holt K: Gynecologic infections. chronic abdominal pain. and ectopic pregnancies. 2003 page 1047 . Pelvic inflammatory disease affects 11% of women and is associated with increases in infertility. Chlamydia is the most common causative organism. in the chronic phase. Diagnosis is based on high clinical suspicion and can be difficult to make. malaise. 2004 2. Exploration can reveal the characteristic violin strings or stringy inflammatory adhesions of the liver to the abdominal wall. fever. REFERENCES 1. It can also be accompanied by vaginal discharge. Mignon F. adhesions usually lead to chronic dull right upper quadrant pain. Radiographic studies can also be normal. FHC syndrome can mimic acute cholecystitis. and contrast enhancement along the liver edge. Computed tomographic (CT) findings can consist of pericholecystic inflammation. and elevated WBC counts and sedimentation rates. Gagnon JH: Fitz-Hugh-Curtis syndrome caused by Chlamydia trachomatis: atypical CT findings. pneumonia.Mesurolle B. hepatitis. mild gallbladder wall thickening.American College of Surgeons SESAP 13 Critique for Category 12 Item 8 Fitz-Hugh-Curtis (FHC) syndrome is a complication of pelvic inflammatory disease in sexually active women characterized by perihepatitis and right upper quadrant pain. and pelvic inflammatory disease. Work-up consists of cervical cultures for Chlamydia trachomatis or Neisseria gonorrhoeae. Emerg Med Clin North Am 21:631-648. In the acute phase of FHC syndrome. Ultrasonography may reveal mild gallbladder wall thickening in the absence of gallstones. AJR Am J Roentgenol 182:822-824.Zeger W. American College of Surgeons SESAP 13 Category 12 Item 9 Which of the following statements about appendicitis during pregnancy is NOT true? (A) Laparoscopic and open appendectomy have similar risks in the third trimester (B) Appendicitis in the third trimester should be treated with intravenous antibiotics and postpartum interval appendectomy (C) The risk of appendiceal perforation prior to diagnosis is increased in the third trimester (D) Postoperative management during the third trimester should include uterine and fetal monitoring (E) The risk of fetal loss during appendectomy is increased in the first trimester page 1048 . Appendicitis symptoms can be attributed to pre-term labor or to the fullness of the expanding uterus. During the first trimester. Lishner M. REFERENCES 1. et al: Pregnancy outcome following non-obstetric surgical intervention. regardless of the approach used. The risks and benefits of intervention for these emergencies must consider both mother and fetus. Nonoperative management of appendicitis is not indicated. Railton C. Appendicitis late in pregnancy should be managed surgically. Because there is an increased incidence of preterm labor after operation in the third trimester. Oren D. but it can be performed safely.Rollins MD. 2004 page 1049 . postoperative management should involve admission to a unit that provides uterine and fetal monitoring. Price RR: Laparoscopy for appendicitis and cholelithiasis during pregnancy: a new standard of care. although they are uncommon. general anesthesia for either technique leads to an increased risk of fetal loss.American College of Surgeons SESAP 13 Critique for Category 12 Item 9 Surgical emergencies do occur during pregnancy. For this reason there is an increased incidence of appendiceal perforation. As pregnancy continues. Chan KJ. Surg Endosc 18:237-241. there is an increased risk for misdiagnosing appendicitis. Am J Surg 190:467-473. The technique does have to be altered somewhat secondary to the gravid uterus. The risks and complications are identical to those of an open appendectomy. intervention for appendicitis should not be delayed during the first trimester. Laparoscopic appendectomy can be performed late in pregnancy.Cohen-Kerem R. Appendicitis during pregnancy can be managed by either open or laparoscopic technique. 2005 2. However. American College of Surgeons SESAP 13 Category 12 Item 10 Which of the following causes of acute abdomen is more common during pregnancy than in the nonpregnant state? (A) Adnexal torsion (B) Appendicitis (C) Bowel obstruction (D) Cholecystitis (E) Pancreatitis page 1050 . Diagnosis is a challenge. In select studies. but are notoriously unreliable signs in pregnancy. Surgical treatment has traditionally been used sparingly for pregnant patients. nasogastric suction. Prompt operation is indicated. REFERENCES 1. the surgical management of cholecystitis by open or laparoscopic cholecystectomy has been shown to be associated with better pregnancy outcomes. but Murphy’s sign is seen less commonly.American College of Surgeons SESAP 13 Critique for Category 12 Item 10 Adnexal torsion is one of the few causes of acute abdomen that is more common in pregnant than in nonpregnant women. Clin Obstet Gynecol 45:553561. particularly in the second and third trimesters. fever. It is seen most often during the third trimester. Adhesions are the cause of obstruction in 70% of cases. as fetal loss may reach 36% with appendiceal perforation. Ultrasonography is helpful in the diagnosis.Stone K: Acute abdominal emergencies associated with pregnancy. Pancreatitis is rare in pregnancy. Immediate open or laparoscopic surgical detorsion is indicated. This conservative approach has recently been challenged. Cholelithiasis is the most frequent cause of pancreatitis during pregnancy. with fluid therapy. occurring in 1 of every 1500 deliveries. Patients usually present with sudden lateralized lower quadrant pain. Bowel obstruction in pregnancy occurs in 1 of 2500 to 3500 deliveries. Surgical intervention is not usually required. Nausea. Symptoms are comparable to nonpregnant patients. Clin Obstet Gynecol 45:405-413. Treatment is similar to nonpregnant patients. Delay in diagnosis results in maternal and fetal mortality. correction of fluid and electrolytic abnormalities. Ultrasonography is helpful in the diagnosis. Volvulus is the second most common cause of bowel obstruction in pregnancy. vomiting. Symptoms and management are similar to that of nonpregnant patients. and timely operation if medical treatment is unsuccessful. Obstruction is seen most often with first pregnancies and during the last trimester of pregnancy. 2002 2.000 deliveries. its incidence is not increased during pregnancy. Right lower quadrant pain is the most consistent symptom. Although appendicitis is the most common cause of acute abdomen in pregnancy. as ovarian viability is at stake. Cholecystitis is the second most frequent surgical condition in pregnancy. and leukocytosis may be present. Cholelithiasis is the cause in 90% of cases. 2002 page 1051 .Sharp HT: The acute abdomen during pregnancy. It is seen in 1 of 1600 to 10. American College of Surgeons SESAP 13 Category 13 Item 1 Which of the following statements about seminoma is TRUE? (A) It is the most common cancer in American men (B) It often secretes alpha-fetoprotein (C) Cisplatin-based chemotherapy is commonly used (D) It is radiosensitive (E) Survival rates < 10% are expected for patients with advanced disease page 1052 . Lippincott Williams & Wilkins. pp 2072-2095 2. Pienta KJ: Testicular cancer. Stage III and IV seminomas require cisplatin-based adjuvant chemotherapy Nonseminomatous tumors are less radiosensitive. Burnett AL. Curr Opin Oncol 16:253-256. nonseminoma. Philadelphia. Small stage II tumors (< 5 cm) are treated with RPLND followed by chemotherapy if cancer is discovered in the pathologic specimen. but it accounts for only approximately 1% of all cancers in American men. Stage I tumors are confined to the testis. This approach establishes a definitive diagnosis and initial treatment. 2004 page 1053 . Germ cell tumors are characterized as seminoma. Seminomas are the most common type of testicular tumors. or both. Sertoli and Leydig cells. and mixed. abdomen. but may secrete bHCG in up to 10% of cases. chemotherapy. Seminomas are highly radiosensitive. stage III tumors have spread beyond retroperitoneal nodes. Five-year survival rates greater than 75% for advanced disease and greater than 90% for lower-stage disease are reported. additional chemotherapy is given. Lillemoe KD. Endodermal sinus cancer or a yolk sac tumor secretes alpha-fetoprotein (AFP). and stage IV indicates extralymphatic metastasis. stage II tumors have spread to regional nodes. surgical exploration consisting of orchiectomy through an inguinal incision is indicated. They require either retroperitoneal lymph node dissection (RPLND). ed 4.Chen DY. Pure seminoma does not secrete AFP. et al (eds): Greenfield's Surgery: Scientific Principles and Practice.American College of Surgeons SESAP 13 Critique for Category 13 Item 1 Testis cancer is the most common cancer in men aged 15 to 34 years. and choriocarcinoma secretes beta-human chorionic gonadotropin (bHCG). Sertoli or Leydig cell tumors. Tumors derived from the testes can originate from either germ cells or supportive cells. If physical examination and ultrasonography indicate suspicion of a testicular mass. Maier RV. in Mulholland MW. ie. Bulky stage II and III tumors are often treated initially with chemotherapy. Stage I and II seminomas are treated with adjuvant radiation therapy. Rodriguez R. Nonseminomatous tumors may secrete marker proteins that aid in preoperative and postoperative assessment. then RPLND if a residual retroperitoneal mass is identified on postchemotherapy CT and tumor markers have normalized. limited chemotherapy. Jarrett TW: Male genitourinary anatomy and physiology. and pelvis. Sex cord stromal tumors. Stage I tumors may be treated with close surveillance alone. Metastatic testis cancer is treatable. Doherty GM. If residual cancer is found in the pathology specimen. ie. are much less common than germ cell tumors. 2006.MacVicar GR. REFERENCES 1. or RPLND. Staging is then accomplished via computed tomography (CT) of the chest. American College of Surgeons SESAP 13 Category 13 Item 2 Iatrogenic injury of the left ureter with electrocautery at the pelvic brim is best managed with (A) primary repair over a stent (B) ureteroneocystostomy (C) debridement and primary repair over a stent (D) psoas hitch (E) anastomosis to the uninjured right ureter page 1054 . REFERENCES 1. Money SR: Prevention and management of ureteral injuries during aortic surgery. 2002 2. Int Urogynecol J Pelvic Floor Dysfunct 13:18-21.Carley ME. McIntire D.American College of Surgeons SESAP 13 Critique for Category 13 Item 2 Transection of the ureter is a dreaded complication of pelvic surgery. risk factors and morbidity of unintended bladder or ureter injury during hysterectomy. Electrocautery increases the potential for thermal tissue destruction beyond the point of transection and mandates debridement of potentially involved tissues rather than primary repair. More complicated repair procedures such as ureteroneocystostomy. 2001 page 1055 . Am J Obstet Gynecol 188:1273-1277. Morrow J. Carley JM.Chan JK. psoas hitch of the bladder. 2003 3. Schaffer J: Incidence. Manetta A: Prevention of ureteral injuries in gynecologic surgery.York JW. The algorithm depicts the clinical management of ureteral injury. and anastomosis of the injured ureter to the contralateral ureter are unnecessary in this setting and can be associated with significant added risk. A tension-free anastomosis is important and some mobilization of the ureteral remnants from the retroperitoneum may be necessary. The intraoperative placement of a double-J stent is useful in protecting the anastomosis and provides access for transcystoscopic therapy. Semin Vasc Surg 14:266-274. The next step in management should be (A) biopsy of the lesion (B) total nephrectomy (C) partial nephrectomy (D) total nephroureterectomy (E) total nephroureterectomy with aortic lymphadenectomy page 1056 . a 45-year-old woman is found to have a 1.5-cm vascular mass in the right kidney.American College of Surgeons SESAP 13 Category 13 Item 3 During computed tomography (CT) for blunt abdominal trauma. Follw up MRI suggests renal cell carcinoma. Mayo Clin Proc 75:1236-1242. Weaver AL. J Urol 174:57-60.Israel GM. 2005 3. Urol Clin North Am 30:499-514. Torres VE. Biopsy of a characteristic lesion such as this is unnecessary and would not alter the ultimate treatment recommendation. Bosniak MA: Renal imaging for diagnosis and staging of renal cell carcinoma. Calvi P. but the risk of a positive margin remains a potential limitation. Some authors suggest enucleation of technically favorable lesions. In patients with tumors smaller than 4 cm. A satisfactory margin appears to be at least 3 mm. 2004 page 1057 . Introini C. 2003 2. et al: Progression and long-term survival after simple enucleation for the elective treatment of renal cell carcinoma: experience in 107 patients. Blute ML. contemporary series definitively demonstrate 5-year cancer-specific survival of 95% or higher. Identification of a vascular mass in the kidney should raise the concern of carcinoma.American College of Surgeons SESAP 13 Critique for Category 13 Item 3 Renal cell carcinoma is the most common renal neoplasm. Minervini A. representing ~80% of all renal tumors. Serni S. REFERENCES 1. et al: Matched comparison of radical nephrectomy vs nephron-sparing surgery in patients with unilateral renal cell carcinoma and a normal contralateral kidney. 2000 4.Puppo P. comparable with the survival of patients undergoing radical surgery for similar sized lesions.Lapini A. Eur Urol 46:477-481. Masieri L. Naselli A: Long term results of excision of small renal cancer surrounded by a minimal layer of grossly normal parenchyma: review of 94 cases. Early stage renal cell cancer should be treated with nephron-sparing surgery such as partial nephrectomy.Lau WK. Placement of a ureteral stent in this patient prior to sigmoid colectomy would decrease the (A) overall operative cost (B) likelihood of causing a ureteral injury (C) likelihood of missing a ureteral injury (D) overall operative morbidity (E) operative blood loss page 1058 . She has had 2 cesarean sections and a total abdominal hysterectomy. a 54-year-old woman requires percutaneous drainage of a pericolonic abscess. One year previously she was hospitalized for uncomplicated diverticulitis.American College of Surgeons SESAP 13 Category 13 Item 4 Shortly after being hospitalized for complicated diverticulitis. One study estimated that it increased operative costs by $1500 per patient. Prophylactic stent placement does help to identify a ureteral injury intraoperatively. selective stent placement in patients with preoperative risk factors for iatrogenic ureteral trauma would seem prudent. Armenakas N. and devascularization. it should be considered in patients with large pelvic masses or a history of prior pelvic operation.3%. Prophylactic ureteral stent placement prior to operation in patients at high risk for injury is also controversial. Processes that lead to distortion or scarring of the pelvic anatomy increase the risk of ureteral injury. Gynecologic procedures account for almost three quarters of such injuries. Although the effectiveness of preoperative ureteral imaging remains controversial. Bose TM: Urinary tract injury in pelvic surgery. transfusion rate. infection. or endometriosis. mobilization of the ureter. It is associated with greater intraoperative blood loss. and hospital length of stay.Brandes S. peritonealization of the pelvic floor. partial transection.Aronson MP. Because delay in diagnosing a ureteral injury increases morbidity (and overall cost). division of the lateral ligaments of the rectum. REFERENCES 1. Ureteral ligation is the most common type of injury. the distal third of the ureter is involved. followed by ureteral kinking by suture. crush injury. operating time. irradiation. Coburn M. It does not prevent or decrease the rate of ureteral injury.5% of all pelvic operations.American College of Surgeons SESAP 13 Critique for Category 13 Item 4 Iatrogenic trauma remains the leading cause of ureteral injury. Ureteral injury is estimated to complicate approximately 0. 2004 3. Clin Obstet Gynecol 45:428-438. 2000 page 1059 . The prevalence of ureteral injury after colorectal procedures is estimated to be 0. BJU Int 86:313-317. or control of profuse bleeding. Most often. BJU Int 94:277-289.Preston JM: Iatrogenic ureteric injury: common medicolegal pitfalls. McAninch J: Diagnosis and management of ureteric injury: an evidence-based analysis. Injury during these cases typically occurs during ligation of the inferior mesenteric artery. 2002 2. nor does it appear to have an affect on operative blood loss or overall morbidity. complete avulsion or transection. American College of Surgeons SESAP 13 Category 13 Item 5 Affect(s) fertility (A) Varicocele (B) Spermatocele (C) Both (D) Neither page 1060 . Alsikafi NF: Spermatocele. which almost invariably is a left-sided process. Mosby.Glick PL. leading to dilation and incompetence of valves within the pampiniform plexus. 2006. which can lead to testicular atrophy and may result in decreased spermatogenesis. Indications for surgical correction of varicoceles are retarded growth of the left testes in a young man or testicular atrophy on the left side. REFERENCES 1.American College of Surgeons SESAP 13 Critique for Category 13 Item 5 Varicoceles and spermatoceles are two of the most common benign nonacute scrotal pathologies in adults. The left spermatic vein (gonadal) is one of the longest veins in the body and enters the left renal vein at a perpendicular angle. Philadelphia. Up To Date website.com 3. which are rare. pp 1172-1192 page 1061 . This in turn leads to venous congestion and elevated temperature in the hemi-scrotum. eMedicine website. Fonkalsrud EW. Varicoceles are dilation of the pampiniform plexus. ie.utdol. Most varicoceles do not require surgical intervention. O’Neill JA Jr. ed 6. Rarely. Available at: www. Available at: emedicine. Intravascular pressure of the left renal vein is increased in most men because of being compressed between the aorta and the superior mesenteric artery as it traverses to the aorta. Coran AG (eds): Pediatric Surgery. Surgical correction is indicated and only for size or symptoms. The major distinction is that varicoceles may effect fertility while spermatoceles do not. varicoceles may be bilateral. Spermatoceles have no effect on fertility. A man with a varicocele who has been deemed infertile is also a candidate for surgical correction of the varicocele. cancer Spermatoceles are randomly found and do not have a predilection for one side or the other. but a unilateral right-sided varicocele is extremely uncommon and should alert the physician to a retroperitoneal cause of right gonadal vein obstruction.Eyre RC: Evaluation of nonacute scrotal pathology in adults. This phenomenon causes increased pressure in the gonadal vein. Boulanger SC: Inguinal hernias and hydroceles. in Grosfeld JL. 2006.com 2. American College of Surgeons SESAP 13 Category 13 Item 6 Predominantly left-sided (A) Varicocele (B) Spermatocele (C) Both (D) Neither page 1062 . Alsikafi NF: Spermatocele. varicoceles may be bilateral. O’Neill JA Jr. Up To Date website.American College of Surgeons SESAP 13 Critique for Category 13 Item 6 Varicoceles and spermatoceles are two of the most common benign nonacute scrotal pathologies in adults. Rarely. This phenomenon causes increased pressure in the gonadal vein. in Grosfeld JL.com 2. 2006. Most varicoceles do not require surgical intervention. Available at: www. Surgical correction is indicated and only for size or symptoms. leading to dilation and incompetence of valves within the pampiniform plexus. ie. Coran AG (eds): Pediatric Surgery. Available at: emedicine.com 3.Glick PL. Indications for surgical correction of varicoceles are retarded growth of the left testes in a young man or testicular atrophy on the left side. cancer Spermatoceles are randomly found and do not have a predilection for one side or the other. A man with a varicocele who has been deemed infertile is also a candidate for surgical correction of the varicocele. Boulanger SC: Inguinal hernias and hydroceles. Philadelphia. The major distinction is that varicoceles may effect fertility while spermatoceles do not. Spermatoceles have no effect on fertility. Fonkalsrud EW. Intravascular pressure of the left renal vein is increased in most men because of being compressed between the aorta and the superior mesenteric artery as it traverses to the aorta. The left spermatic vein (gonadal) is one of the longest veins in the body and enters the left renal vein at a perpendicular angle. which almost invariably is a left-sided process. This in turn leads to venous congestion and elevated temperature in the hemi-scrotum. 2006. ed 6. REFERENCES 1. Mosby. Varicoceles are dilation of the pampiniform plexus. which are rare. which can lead to testicular atrophy and may result in decreased spermatogenesis. but a unilateral right-sided varicocele is extremely uncommon and should alert the physician to a retroperitoneal cause of right gonadal vein obstruction. pp 1172-1192 page 1063 .Eyre RC: Evaluation of nonacute scrotal pathology in adults. eMedicine website.utdol. American College of Surgeons SESAP 13 Category 13 Item 7 Require(s) surgical correction (A) Varicocele (B) Spermatocele (C) Both (D) Neither page 1064 . 2006.Glick PL. Available at: emedicine. Philadelphia. Up To Date website. varicoceles may be bilateral.American College of Surgeons SESAP 13 Critique for Category 13 Item 7 Varicoceles and spermatoceles are two of the most common benign nonacute scrotal pathologies in adults. Boulanger SC: Inguinal hernias and hydroceles. but a unilateral right-sided varicocele is extremely uncommon and should alert the physician to a retroperitoneal cause of right gonadal vein obstruction. leading to dilation and incompetence of valves within the pampiniform plexus. eMedicine website.Eyre RC: Evaluation of nonacute scrotal pathology in adults. A man with a varicocele who has been deemed infertile is also a candidate for surgical correction of the varicocele. which are rare. The major distinction is that varicoceles may effect fertility while spermatoceles do not.com 2.com 3. Available at: www. Spermatoceles have no effect on fertility. O’Neill JA Jr. REFERENCES 1. Indications for surgical correction of varicoceles are retarded growth of the left testes in a young man or testicular atrophy on the left side. Intravascular pressure of the left renal vein is increased in most men because of being compressed between the aorta and the superior mesenteric artery as it traverses to the aorta. This in turn leads to venous congestion and elevated temperature in the hemi-scrotum. which almost invariably is a left-sided process.utdol. ed 6. cancer Spermatoceles are randomly found and do not have a predilection for one side or the other. Rarely. 2006. Most varicoceles do not require surgical intervention. The left spermatic vein (gonadal) is one of the longest veins in the body and enters the left renal vein at a perpendicular angle. Fonkalsrud EW.Alsikafi NF: Spermatocele. Mosby. which can lead to testicular atrophy and may result in decreased spermatogenesis. Surgical correction is indicated and only for size or symptoms. in Grosfeld JL. pp 1172-1192 page 1065 . This phenomenon causes increased pressure in the gonadal vein. Varicoceles are dilation of the pampiniform plexus. Coran AG (eds): Pediatric Surgery. ie. American College of Surgeons SESAP 13 Category 13 Item 8 Responsible for ejaculatory function (A) Sympathetic pelvic nerves (B) Parasympathetic pelvic nerves (C) Both (D) Neither page 1066 . American College of Surgeons SESAP 13 Critique for Category 13 Item 8 Autonomic nerve damage and its associated sexual dysfunction has long been recognized as a potential complication of rectal surgery. and seminal vesicles. The sympathetic fibers originate from the spinal cord levels T12L2 and form a plexus around the aorta. Philadelphia. Lippincott Williams & Wilkins. and are the third potential injury point during division of the lateral stalks. medial to the ureter. prostate. Nerve injury can result from direct nerve division or ligation. ANZ J Surg 74:248-259.Wexner SD. and fourth sacral nerves join the nervi erigentes to form the inferior hypogastric plexus. ed 5.Keating JP: Sexual function after rectal excision. permanent impotence occurs in fewer than 2% of patients with benign disease after restorative proctocolectomy. and the parasympathetic fibers. which are responsible for ejaculatory function. 2004 2. The two components of the autonomic pathway in males are the sympathetic fibers. and colon. or adjuvant radiation therapy. The fibers then coalesce to form the left and right hypogastric nerves as they course over the sacral promontory and pass behind the fascia propria of the mesorectum. impotence rates higher than 40% have been reported with increasing patient age and operations for malignancies of the lower rectum such as abdominoperineal resection. which are responsible for erectile function. passing posteriorly to the inferior mesenteric artery. Careful anatomic dissection in the retroperitoneum and pelvis with preservation of the autonomic fibers provides the best chance at avoiding such injuries and preserving sexual function. REFERENCES 1. These fibers then pass anteromedially between the rectum. In experienced hands. it is at this point that the parasympathetic fibers from the second. However. pp 1-30 page 1067 . Finally. thermal injury from electrosurgical devices. traction or crush injury. third. in Corman ML (ed): Colon and Rectal Surgery. becoming at this level the second potential injury point. Third. Jorge JMN: Anatomy and embryology of the anus. the hypogastric nerves pass inferolaterally along the pelvic sidewall. Four autonomic nerve injury points must be avoided during left colon and rectal surgery. 2005. which constitutes the first potential injury point. rectum. American College of Surgeons SESAP 13 Category 13 Item 9 Nervi eregentes (A) Sympathetic pelvic nerves (B) Parasympathetic pelvic nerves (C) Both (D) Neither page 1068 . In experienced hands. impotence rates higher than 40% have been reported with increasing patient age and operations for malignancies of the lower rectum such as abdominoperineal resection. The two components of the autonomic pathway in males are the sympathetic fibers. and seminal vesicles.Wexner SD. Nerve injury can result from direct nerve division or ligation. medial to the ureter. However. Careful anatomic dissection in the retroperitoneum and pelvis with preservation of the autonomic fibers provides the best chance at avoiding such injuries and preserving sexual function. 2004 2. which are responsible for ejaculatory function. which are responsible for erectile function. the hypogastric nerves pass inferolaterally along the pelvic sidewall. Jorge JMN: Anatomy and embryology of the anus. passing posteriorly to the inferior mesenteric artery. or adjuvant radiation therapy. The sympathetic fibers originate from the spinal cord levels T12L2 and form a plexus around the aorta. and fourth sacral nerves join the nervi erigentes to form the inferior hypogastric plexus. These fibers then pass anteromedially between the rectum. in Corman ML (ed): Colon and Rectal Surgery. and colon. which constitutes the first potential injury point. The fibers then coalesce to form the left and right hypogastric nerves as they course over the sacral promontory and pass behind the fascia propria of the mesorectum. becoming at this level the second potential injury point. 2005.American College of Surgeons SESAP 13 Critique for Category 13 Item 9 Autonomic nerve damage and its associated sexual dysfunction has long been recognized as a potential complication of rectal surgery. and are the third potential injury point during division of the lateral stalks. ed 5. and the parasympathetic fibers.Keating JP: Sexual function after rectal excision. ANZ J Surg 74:248-259. prostate. Philadelphia. permanent impotence occurs in fewer than 2% of patients with benign disease after restorative proctocolectomy. Lippincott Williams & Wilkins. Third. pp 1-30 page 1069 . Finally. third. it is at this point that the parasympathetic fibers from the second. Four autonomic nerve injury points must be avoided during left colon and rectal surgery. thermal injury from electrosurgical devices. rectum. REFERENCES 1. traction or crush injury. American College of Surgeons SESAP 13 Category 13 Item 10 At risk of injury during ligation of inferior mesenteric artery (A) Sympathetic pelvic nerves (B) Parasympathetic pelvic nerves (C) Both (D) Neither page 1070 . or adjuvant radiation therapy. The fibers then coalesce to form the left and right hypogastric nerves as they course over the sacral promontory and pass behind the fascia propria of the mesorectum. 2005. Philadelphia. the hypogastric nerves pass inferolaterally along the pelvic sidewall. and the parasympathetic fibers. impotence rates higher than 40% have been reported with increasing patient age and operations for malignancies of the lower rectum such as abdominoperineal resection. Jorge JMN: Anatomy and embryology of the anus. it is at this point that the parasympathetic fibers from the second. Four autonomic nerve injury points must be avoided during left colon and rectal surgery. and colon. The two components of the autonomic pathway in males are the sympathetic fibers. In experienced hands. pp 1-30 page 1071 . which are responsible for ejaculatory function. prostate. which are responsible for erectile function. which constitutes the first potential injury point. Lippincott Williams & Wilkins. becoming at this level the second potential injury point.Keating JP: Sexual function after rectal excision. However. Nerve injury can result from direct nerve division or ligation.Wexner SD. REFERENCES 1. Third. thermal injury from electrosurgical devices. Careful anatomic dissection in the retroperitoneum and pelvis with preservation of the autonomic fibers provides the best chance at avoiding such injuries and preserving sexual function. permanent impotence occurs in fewer than 2% of patients with benign disease after restorative proctocolectomy. and fourth sacral nerves join the nervi erigentes to form the inferior hypogastric plexus. These fibers then pass anteromedially between the rectum. and are the third potential injury point during division of the lateral stalks.American College of Surgeons SESAP 13 Critique for Category 13 Item 10 Autonomic nerve damage and its associated sexual dysfunction has long been recognized as a potential complication of rectal surgery. and seminal vesicles. in Corman ML (ed): Colon and Rectal Surgery. traction or crush injury. Finally. 2004 2. rectum. medial to the ureter. third. passing posteriorly to the inferior mesenteric artery. The sympathetic fibers originate from the spinal cord levels T12L2 and form a plexus around the aorta. ed 5. ANZ J Surg 74:248-259. American College of Surgeons SESAP 13 Category 14 Item 1 The lesions shown are identified on initial exploration for a laparoscopic cholecystectomy for symptomatic gallstones. and further operation is not appropriate page 1072 . Two years previously. the patient had laparotomy for resection of a stage I sigmoid colon cancer. Which of the following statements is TRUE? (A) The lesions are granulomas and can be ignored (B) The procedure should be converted to an open exploration for adequate staging (C) Surgical peritonectomy is indicated (D) Laparoscopic cholecystectomy with biopsy of the lesions should be performed (E) The patient’s life expectancy is less than 3 months. Further cancer-related surgical therapy would not be appropriate at this point because the patient has not been fully staged for other metastatic disease. Laparotomy would be of no diagnostic or therapeutic benefit.Verwaal VJ. Brooks AD. Witkamp A. REFERENCES 1. Sharma S. Boot H. Saltz LB. et al: Long-term survival of peritoneal carcinomatosis of colorectal origin. van Ruth S.Bloemendaal AL. Ann Surg Oncol 12:65-71. et al: Surgical debulking and intraperitoneal chemotherapy for established peritoneal metastases from colon and appendix cancer. Ann Surg Oncol 8:787-795. Verwaal VJ. 2005 page 1073 . may improve survival and should be considered after appropriate staging. Eur J Surg Oncol 31:1145-1151. 2005 2. et al: Conventional surgery and systemic chemotherapy for peritoneal carcinomatosis of colorectal origin: a prospective study.American College of Surgeons SESAP 13 Critique for Category 14 Item 1 In this patient with a history of adenocarcinoma of the colon. removal of the patient’s symptomatic gallbladder is still appropriate. studding of the peritoneum should raise concern for peritoneal carcinomatosis. van Ruth S. Surgical therapy directed at isolated peritoneal disease. Because the median survival of patients with isolated peritoneal carcinomatosis from metastatic colorectal cancer is 21 months. Boot H.Culliford AT IV. 2001 3. including cytoreduction and intraperitoneal hyperthermic chemotherapy. Biopsy of the lesions should be obtained to establish the diagnosis. which of the following statements is NOT true? (A) Screening positron emission tomography (PET) for liver resection candidates is associated with improved 3.and 5-year survival compared with historical controls (B) Chest x-ray is a satisfactory test to rule out lung metastases (C) Radiofrequency ablation is equivalent to resection as therapy for solitary metastases < 5 cm in diameter (D) Tumor debulking does not improve the efficacy of chemotherapy (E) Bilobar metastases are not a contraindication to resection page 1074 .American College of Surgeons SESAP 13 Category 14 Item 2 When considering treatment of a liver metastasis from colorectal cancer. Vauthey JN. 2004 2. margins less than 1. Ann Surg 239:818-825. Linehan DC. Dougherty E. Ellis V. Drebin JA.Pawlik TM. Chest CTs are too sensitive for general screening of patients with liver metastases because of the high rate of detection of irrelevant lesions. Ann Surg 240:438-447. Radiofrequency ablation is not an equivalent substitute for resection and should be considered only in patients who are not satisfactory candidates for laparoscopic or open liver resection.and 5-year survivals compared with historic controls who were not staged with PET. Bodniewicz J. Conlon K. REFERENCES 1. however. and combined resection/ablation for colorectal liver metastases. et al: Recurrence and outcomes following hepatic resection.0 cm that are histologically negative are still associated with a survival benefit. Histologically negative margins and complete resection of detectable disease must be achieved in order to demonstrate a survival benefit from liver resection. 2005 page 1075 . Dehdashti F.0 cm are sought so as to include any potential risk of satellite lesions. Staging of metastatic colorectal cancer should include a chest x-ray or chest computed tomographic (CT) scan to rule out the presence of lung metastases.American College of Surgeons SESAP 13 Critique for Category 14 Item 2 The management of patients with hepatic metastases of colorectal cancer remains centered on surgical resection of patients with appropriately staged disease. Scoggins CR. Cancer 91:1121-1128. An additional contraindication is liver metastases that cannot be treated with an R0 outcome (no residual disease). Zorzi D. et al: A clinical scoring system predicts the yield of diagnostic laparoscopy in patients with potentially resectable hepatic colorectal metastases. Positron emission tomography (PET) is proving to be a useful tool in the staging of patients with liver metastases of colorectal cancer. Clear contraindications to liverdirected therapy of any kind include all extrahepatic metastatic disease that is not amenable to surgical therapy. et al: Five-year survival after resection of hepatic metastases from colorectal cancer in patients screened by positron emission tomography with F-18 fluorodeoxyglucose (FDG-PET). Abdalla EK. 2004 3.Jarnagin WR.Fernandez FG. A variety of different prognostic tools have been used to predict survival benefit from liver resection in metastatic colorectal cancer. Ellis LM. 2001 4. radiofrequency ablation. Screening PET eliminates patients who will not benefit from resection. Neither the presence of bilobar metastases nor more than three lesions is a clear contraindication to resection. et al: Effect of surgical margin status on survival and site of recurrence after hepatic resection for colorectal metastases. Patients screened for extrahepatic disease with PET have improved 3.Abdalla EK. Ann Surg 241:715-722. Margins greater than 1. American College of Surgeons SESAP 13 Category 14 Item 3 Which of the following statements about esophagectomy is TRUE? (A) Operative mortality is 5% (B) Respiratory failure is more common after a transthoracic approach than with a transhiatal approach (C) Wound dehiscence is increased with a transhiatal approach compared with a transthoracic approach (D) Neoadjuvant chemotherapy reduces perioperative mortality (E) Hospital volume has no effect on mortality rate page 1076 . Bull D. diabetes mellitus.Bailey SH. Individual surgeon experience with either or both of these procedures is typically limited. The two procedures had equivalent incidences of respiratory failure.American College of Surgeons SESAP 13 Critique for Category 14 Item 3 Transthoracic and transhiatal esophagectomy are used for resection of carcinoma of the esophagus as well as for benign esophageal disease. The largest reported series to date is from the Department of Veteran Affairs National Surgical Quality Improvement Program. Ann Thorac Surg 75:217-222.Rentz J. sepsis. Because of its wider margin of resection. J Thorac Cardiovasc Surg 125:1114-1120. infection. Rentz JJ. Wound dehiscence was more common in patients undergoing transhiatal esophagectomy than after transthoracic esophagectomy. Harpole D. anastomotic complications. Intraoperative risk factors for mortality following esophagectomy included the need for transfusion and longer operative time. ascites. which was used to prospectively analyze surgical outcomes from a cross-section of surgical services performing transthoracic and transhiatal esophagectomy. BUN > 40 mg/dL. alkaline phosphatase level > 125units/L. diabetes. et al: Transthoracic versus transhiatal esophagectomy: a prospective study of 945 patients. REFERENCES 1. 2003 2. Preoperative factors affecting morbidity included increasing age. Overall 30-day mortality for esophagectomy approached 10% and was equivalent for transthoracic and transhiatal approaches. renal failure. decreased functional status. and decreased functional status. These studies specifically addressed risk factors for morbidity and mortality following esophagectomy and directly compared outcomes following transthoracic versus transhiatal esophagectomy. alcohol abuse. dyspnea. Large national databases are one of the few means of assessing a nationwide cross-sectional experience with these procedures. alkaline phosphatase level > 125 units/L. and increasing age. and mediastinitis. Harpole DH. Bailey S. Whether this affects long-term survival is still being debated. transthoracic esophagectomy is typically associated with an increased yield of lymph nodes. Bull DA. et al: Outcomes after esophagectomy: a ten-year prospective cohort. Hospital volume does appear to affect mortality rates. chronic obstructive pulmonary disease. The overall morbidity after esophagectomy approaches 50%. 2003 page 1077 . although whether this is due to individual surgeon variables or hospital-specific variables is still unclear. Preoperative predictors of mortality for both procedures included neoadjuvant therapy. bleeding. lower serum albumin concentration. 0 cm page 1078 .5 cm (C) 1.American College of Surgeons SESAP 13 Category 14 Item 4 Initial biopsy of a 0.0 cm (D) 2.7-mm thick mole on the thigh reveals melanoma with positive margins.0 cm (E) 5. Which of the following is the recommended margin for re-excision? (A) No re-excision is necessary if margins are negative (B) 0. Fine-needle aspirate or lymph node biopsy can also be considered at the time of excisional biopsy. and Clark level. The pathology report should include a minimum Breslow thickness. 2004. and the incidence is rising dramatically.to 3-mm margins. ulceration status. REFERENCES 1. including ulceration. in National Comprehensive Cancer Network: Clinical Practice Guidelines in Oncology-v.2006.Urist MM. Saunders. Accessed 11/2005 2. Median age at diagnosis is 45 to 55 years. Coit DG. and extranodal soft tissue extension of tumor. The incision should be oriented longitudinally to the involved extremity to facilitate later excision if necessary. et al: Melanoma. Philadelphia. melanoma is increasing faster than any other cancer and in women. clinically apparent nodal disease (as opposed to histologically positive but clinically negative). Patients with clinically positive nodes on presentation should undergo lymphadenectomy in addition to wide local excision of the primary tumor once excisional biopsy reveals melanoma. Patients with positive sentinel nodes should undergo completion lymphadenectomy of the affected nodal area. and 2% to 5% present with distant metastases. Each year approximately 7000 to 8000 patients die of melanoma. vertical growth phase. Mortality is directly linked to tumor thickness at biopsy. ed 17. Dilawari RA. Definitive treatment of the biopsy site includes re-excision with appropriate margins.000 patients each year. Biopsy of a suspicious lesion should be performed as a full-thickness excision with 1. pp 781-801 page 1079 . Patients diagnosed with melanoma should undergo complete cutaneous examination as well as a thorough history and physical examination. faster than any cancer except lung cancer. any thickness. Full-thickness punch or incisional biopsy is acceptable in sites not amenable to excisional biopsy. because additional positive nodes are found in 10% to 30%. These recommendations are based on a large trial suggesting that for 1. Daud A. 10% to 13% have regional disease (positive lymph nodes) at presentation. Most patients (82% to 85%) present with localized disease. in Townsend CM Jr (ed): Sabiston Textbook of Surgery: The Biological Basis of Modern Surgical Practice. should also prompt consideration of sentinel node biopsy. For melanomas <1 mm thick.1. Clark level. positive deep margins or adverse histologic features.American College of Surgeons SESAP 13 Critique for Category 14 Item 4 Melanoma is diagnosed in approximately 60.Houghton AN. Breslow thickness. Soong SJ: Melanoma and cutaneous malignancies. or extensive regression. Tumor ulceration is also a prognostic indicator in those who have regional disease.to 4-mm thick melanomas. Further testing depends on tumor characteristics and thickness. as well as deep and peripheral margin information. as are tumor burden (number of positive nodes). with special attention to the nodal basins to which the biopsied tumor is likely to have spread. In men. 4-cm excison margins offered no improvement in survival over 2-cm excisional margins. Sentinel node biopsy should be performed for patients with melanomas >1 mm thick or with Clark level IV. and presence of ulceration are major prognostic factors in those who present with localized disease. portal lymphadenectomy. port sites. port sites.American College of Surgeons SESAP 13 Category 14 Item 5 A laparoscopic cholecystectomy is performed in a 55-year-old woman. Pathology reveals gallbladder adenocarcinoma confined to the mucosal lamina propria. Appropriate management includes (A) observation and surveillance follow-up only (B) excision of gallbladder fossa and laparoscopic port sites (C) excision of gallbladder fossa. excision of common bile duct (E) irradiation of the gallbladder bed page 1080 . portal lymphadenectomy (D) excision of gallbladder fossa. Muratore A. T4) represent advanced disease. Radiation treatment is not warranted for this T1 lesion.Varshney S. Five-year survival rates range from 83% to 100%. This offers a five-year survival rate of 60% to 80%. excision of the medial hepatic segments with regional lymphadenectomy with or without excision of the extrahepatic bile duct is recommended. Polastri R. Eur J Surg Oncol 26:438-443. most authors recommend consideration of port site excision and extended cholecystectomy with wedge hepatectomy of the gallbladder fossa and regional lymphadenectomy. Patients must be followed-up closely for the potential development of port site metastases or locoregional recurrence.American College of Surgeons SESAP 13 Critique for Category 14 Item 5 Long-term survival in patients with gallbladder carcinoma is generally limited to those with isolated. In the setting of an incidentally discovered gallbladder carcinoma confined to the mucosal lamina propria (stage T1a) following laparoscopic cholecystectomy. Eur J Surg Oncol 28:4-10. Capussotti L: Radical surgery for gallbladder cancer: current options. Butturini G. In the absence of extensive nodal disease or distant metastases. Lesions that extend beyond the serosa of the gallbladder (T3. Gupta R: Incidental carcinoma of the gallbladder. extended surgical resection offers the only possibility of long-term survival. incidentally discovered disease confined to the mucosa. For tumors invading the muscularis of the gallbladder (T1b). 2002 page 1081 . 2000 2. no further surgical treatment is required. For T2 lesions invading perimuscular connective tissue not beyond serosa or liver. REFERENCES 1. Magnetic resonance imaging (MRI) shows a 4-cm neoplasm arising in the body of the rectus femoris muscle. The mass has enlarged over the course of 2 months and is painful.American College of Surgeons SESAP 13 Category 14 Item 6 A 56-year-old man presents with a mass in his mid-anterior thigh. A diagnosis of malignant fibrous histiocytoma is made by percutaneous biopsy Which of the following statements is TRUE? (A) Preoperative combined chemoradiotherapy improves outcome (B) Muscle compartment resection is required (C) Tumor grade is more important than size in staging (D) Induction chemotherapy with ifosamide and vincristine increases survival (E) Postoperative adjuvant radiotherapy to 60 Gy is appropriate page 1082 . Five-year survival rates are in the range of 86% for stage I.McCarter MD. size. Turcotte R. Sarcoma is one of the few tumors in which grade is a significant part of the AJCC staging system. Enucleation or mere excision of the visible disease results in a 90% local recurrence rate and a 30% ultimate local failure rate. and 10% to 20% for stage IV. 52% for stage III. it is believed to be equivalent but not superior to size in importance. as well as distant or nodal metastases. Grade. Preoperative combined chemoradiotherapy is associated with an increased incidence of wound complications and the necessity to close wounds with secondary measures such as musculocutaneous flaps. 2005 2.to 3-cm margin and postoperative adjuvant radiotherapy gives local control over 90% with the lowest incidence of wound complications. which is 90% to 95% accurate and is associated with a decreased complication rate compared with incisional biopsy. Adequate excision with a 2. is the recommended technique to establish a tissue diagnosis. Because sarcomas grow both centrifugally in a spherical fashion and along tissue planes. REFERENCES 1.American College of Surgeons SESAP 13 Critique for Category 14 Item 6 Combined therapy with either preoperative or postoperative radiotherapy and surgical resection with a 2. 2002 3. a pseudocapsule is formed that requires wide excision. Thomas JM: Soft-tissue sarcomas in adults. 72% for stage II.Clark MA. and depth of penetration with respect to the deep layer of the investing fascia.O’Sullivan B. Judson I. N Engl J Med 353:701711. Jaques DP. Fisher C. Surg Oncol Clin North Am 11:11-22. Lancet 359:2235-2241. 2002 page 1083 . are components of this staging system. The usual preoperative dose is 50 to 55 Gy.to 3-cm clear margin on final pathology provides equivalent control to amputation. Amputation is usually reserved for recurrence after limb salvage procedures or for tumors that have destroyed critical anatomic and/or functional structures that cannot be replaced. Percutaneous core needle biopsy. 60 to 65 Gy is recommended postoperatively. it has not improved survival for localized tumors and is usually reserved for metastatic disease. Davis AM. Although chemotherapy is useful in Ewing’s sarcoma and rhabdomyosarcoma (particularly in children). Brennan MF: Randomized clinical trials in soft tissue sarcoma. Total functional muscle compartment resection is not necessary unless required to obtain adequate clear margins. Bell R. et al: Preoperative versus postoperative radiotherapy in soft-tissue sarcoma of the limbs: a randomised trial. American College of Surgeons SESAP 13 Category 14 Item 7 Principles of treatment of gastrointestinal stromal tumors (GIST) include all of the following EXCEPT (A) segmental resection with negative margins (B) resection of involved tissues including adjacent organs (C) en bloc lymphadenectomy (D) tyrosine kinase inhibitor (imatinib. Gleevec) in patients with residual disease (E) multitargeted tyrosine kinase inhibitor (Sunitinib) in patients refractory or intolerant to Gleevec page 1084 . N Engl J Med 347:472-480. Segmental resection of the stomach or intestine should be performed with the goal of achieving negative resection margins. Prognosis is influenced by the completeness of resection. a tyrosine kinase called c-Kit. clinical benefit for GIST patients with advanced or unresectable disease. They can often be lifted away from surrounding structures. Posner M. Sunitinib. J Clin Oncol 24:2325-2331.Demetri GD. Imitanib is effective in tumors with the mutant isoforms of KIT as well as some GISTs that contain mutant isoforms of PDGFRa.American College of Surgeons SESAP 13 Critique for Category 14 Item 7 The goal of treatment of all gastrointestinal stromal tumors (GIST) is complete gross resection with an intact pseudocapsule. Wider resection of uninvolved tissue is of no additional benefit. which can be detected by immunohistochemistry. The malignant cells in most GISTs overexpress the product of the KIT proto-oncogene. Emerging data suggest that selection of clones with secondary mutations in c-Kit is the most common mechanism of imitanib resistance. This agent is not yet commercially available in the United States. Routine lymphadenectomy is unnecessary because nodal metastases are rare.Raut CP. there was no known effective single-agent or combination chemotherapy regimen for unresectable or metastatic GIST. Prior to 2000. Lewis JJ. Desai J. rapid. GISTs often project extraluminally from the stomach or small intestine and displace rather than invade adjacent organs. Recent data indicate that molecularly targeted therapy with the tyrosine kinase inhibitor imitanib (Gleevec) can produce dramatic.DeMatteo RP. et al: Two hundred gastrointestinal stromal tumors: recurrence patterns and prognostic factors for survival. Early reports suggest efficacy for the multitargeted tyrosine kinase inhibitor. Ann Surg 231:51-58. Van den Abbeele AD. Although they may appear ominous on CT. Morgan JA. von Mehren M. Leung D. Possible methods of overcoming imitanib resistance include the use of more powerful selective tyrosine kinase inhibitors of c-Kit together with selective inhibitors of downstream pathways. 2002 3. but have mutations in a second tyrosine kinase receptor. et al: Efficacy and safety of imatinib mesylate in advanced gastrointestinal stromal tumors. A smaller subset are c-Kit negative. the platelet-derived growth factor receptor-alpha (PDGFRa). although en bloc resection is necessary in some cases because of dense adhesions. et al: Surgical management of advanced gastrointestinal stromal tumors after treatment with targeted systemic therapy using kinase inhibitors. Mudan SS. 2000 2. REFERENCES 1. Blanke CD. 2006 page 1085 . American College of Surgeons SESAP 13 Category 14 Item 8 Risk factors for adenocarcinoma of the small bowel include all of the following EXCEPT (A) familial adenomatous polyposis (B) Crohn’s disease (C) hereditary nonpolyposis colorectal cancer (D) ulcerative colitis (E) von Recklinghausen’s disease page 1086 . and alcohol consumption. familal adenomatous polyposis. and Crohn’s disease. WB Saunders. pp 453-456 page 1087 . von Recklinghausen’s disease. Wells SA Jr: Tumors of the small intestine. Limburg PJ.American College of Surgeons SESAP 13 Critique for Category 14 Item 8 Risk factors for adenocarcinoma of the small bowel include familial adenomatous polyposis syndromes (eg. Gardner’s syndrome. Other factors that have been suggested. to contribute to small bowel cancer include a high fat intake. cigarette smoking. celiac disease. REFERENCES 1. Population-based studies showed no increased risk of small bowel carcinoma in patients with ulcerative colitis. consumption of red meat or salt-cured or smoked food.Ashley SW. Philadelphia. Neczyporenko F. 2005 3.Delaunoit T. Erlichman C: Pathogenesis and risk factors of small bowel adenocarcinoma: a colorectal cancer sibling? Am J Gastroenterol 100:703-710. 2002. with a mean age of about 60 years for sporadic small bowel adenocarcinomas. and Turcot’s syndrome). Adenocarcinoma develops in 2% to 3% of patients with small intestinal Crohn’s disease.Evers BM: Adenocarcinoma of the small bowel. Most patients have had symptomatic disease for at least 5 years. Adenocarcinomas arising in association with Crohn’s disease tend to occur in a younger age group. hereditary nonpolyposis colorectal carcinoma. Semin Oncol 15:116-128. but not proven. 1988 2. in Bland KI (ed): The Practice of General Surgery. American College of Surgeons SESAP 13 Category 14 Item 9 Oncogenes and tumor suppressor genes involved in colorectal cancer include all of the following EXCEPT (A) Ras (B) adenomatosis polyposis coli (APC) (C) p53 (D) BRCA1 (E) deleted in colon cancer (DCC) page 1088 . Bapat B. suppressor genes. Oncogenes are dominantly acting genes. but chromosomal losses are frequent. called proto-oncogenes. Adv Surg 26:323-332. BRCA1 mutations have not been described in colorectal cancers. Swallow C. REFERENCES 1. both alleles of a tumor suppressor gene must be nonfunctional to initiate tumor formation.Gryfe R. 1997 2. Mutations in ras proto-oncogene occur in 40% to 50% of colon adenocarcinomas. 1993 page 1089 . Tumor suppressor genes identified as participating in colorectal carcinogenesis include the adenomatous familial polyposis coli (APC) gene. The end result is a growth advantage that contributes to the development of a neoplasm. Michelassi F: Oncogenes. Tumor suppressor genes normally produce proteins that inhibit tumor formation. Mutant ras genes result in a protein that is constitutively active and maintains the activity of the signal pathway at a constant level independent of the actual external signal. It soon followed that the normal genomes of animals and humans contained genes homologous to the v-oncogenes. Oncogenes were first discovered as part of the genome of viruses (Voncogenes). On the other hand. Westbrook CA. The protooncogenes are involved in cell growth regulation. et al: Molecular biology of colorectal cancer. An activating mutation produces a constitutively active protein. Wasylyshyn ML. Gene mutations may result in loss of function. For oncogenes. and allele losses in colon cancer.American College of Surgeons SESAP 13 Critique for Category 14 Item 9 Cancer-related genes known to play a role in colorectal cancer are oncogenes and tumor suppressor genes. Many of them encode receptors for growth factors or proteins involved in the cell signaling pathways. Their somatic mutations in sporadic breast cancers are rare. a change in one of the two copies of the gene is sufficient to produce the effect. which caused cancer in animals.Yaremko ML. p53. Germline mutations of BRCA1 and BRCA2 are responsible for familial breast cancers. resulting in the production of a protein product in increased quantity or with augmented function. and the mutant gene overrides. or is dominant to the normal allele. Redston M. Curr Probl Cancer 21:233-300. altered by activating mutations or changes in the DNA sequence. and deleted in colon cancer (DCC) gene. with resulting uninhibited cellular growth and formation of tumors. American College of Surgeons SESAP 13 Category 14 Item 10 Which of the following statements about the HER2/neu protein for breast cancer is NOT true? (A) Overexpression of the protein is associated with a poorer prognosis (B) A positive fluorescence in situ hybridization (FISH) test should be confirmed with an immunohistochemistry (IHC) test (C) Trastuzumab is indicated for patients with overexpression of the protein and metastatic disease (D) Amplification of HER2/neu protein product occurs during the in situ stage of tumor development (E) The HER2/neu protein product is in the family of epidermal growth factor receptors page 1090 . REFERENCES 1. et al: Prognostic and predictive value of c-erbB-2 overexpression in primary breast cancer. Clin Breast Cancer 6:240-246. Romond E. 1998 page 1091 . J Clin Oncol 16:462-469. some studies suggest that this prognostic factor may be stronger than estrogen receptor status. Cobleigh MA. and fluorescence in situ hybridization (FISH) assay.0001). The difference between combined and single therapy was not significant in patients with negative FISH tests. et al: Evaluation of clinical outcomes according to HER2 detection by fluorescence in situ hybridization in women with metastatic breast cancer treated with trastuzumab. Holmberg L. an agent that improves survival in such patients. 2005 2. objective responses occurred in 54% versus 30% with chemotherapy alone (P = 0.Paik S. Lindgren A.Mass RD. The HER2 oncogene is a type 1 growth factor receptor in the family of EGF receptors that eventually demonstrates its presence during the in situ phase of tumor development.Sjogren S. 2002 3. The two methods to detect HER2/neu overexpression are immunohistochemistry (IHC) staining of breast cancer specimens. J Natl Cancer Inst 94:852-854. appears to increase the response rate over chemotherapy alone in patients with positive FISH tests. however. Trastuzumab. Inganas M. several studies have demonstrated superior accuracy with the FISH assay. Tan-Chiu E. alone and in combination with other prognostic markers. Bryant J. Amplification of the HER2 oncogene has an inverse relationship to estrogen receptor positivity and thus a direct relationship to poorly differentiated tumor with poor prognosis. Furthermore. In one study. IHC was developed first and was more widely used. et al: Real-world performance of HER2 testing—National Surgical Adjuvant Breast and Bowel Project experience. and whether to use trastuzumab should be based on results of the FISH assay.American College of Surgeons SESAP 13 Critique for Category 14 Item 10 Overexpression of the HER2 growth receptor in breast cancer cells is an independent prognostic indicator of breast cancer survival. Press MF. Accurately identifying patients who overexpress this protein is even more important since the development of trastuzumab. Anderson S. a recombinant monoclonal antibody active against HER2. the most appropriate chemotherapy regimen to accompany radiation therapy would be fluorouracil (5-FU) plus (A) carboplatin (B) gemcitabine (C) leucovorin (D) mitomycin C (E) paclitaxel (Taxol) page 1092 .American College of Surgeons SESAP 13 Category 14 Item 11 For a 60-year-old woman with stage 3 epidermoid anal cancer. Stevens KR Jr. Deveney KE: The evolving treatment of anal cancer: how are we doing? Arch Surg 136:886-891. The first successful use of combined chemoradiotherapy as definitive treatment was reported by Nigro in 1974.Bartelink H. Surgical therapy alone results in a permanent colostomy and a high recurrence rate. basaloid. and of salvage chemoradiation in the definitive nonsurgical treatment of epidermoid carcinoma of the anal canal: results of a phase III randomized intergroup study. 1997 2. REFERENCES 1. cloacogenic. Eschwege F. Petrelli N. 1996 3. Carboplatin. Pajak TF. Rougier P. J Clin Oncol 15:2040-2049. and transitional cell tumors behave in similar fashion and are collectively termed epidermoid carcinomas. gemcitabine. and paclitaxel have no demonstrated efficacy in the treatment of epidermoid anal carcinoma. Roelofsen F. Cisplatin may be substituted for mitomycin C. et al: Concomitant radiotherapy and chemotherapy is superior to radiotherapy alone in the treatment of locally advanced anal cancer: results of a phase III randomized trial of the European Organization for Research and Treatment of Cancer Radiotherapy and Gastrointestinal Cooperative Groups. 2001 page 1093 .Flam M. et al: Role of mitomycin in combination with fluorouracil and radiotherapy.Whiteford MH. The standard combination of therapy has changed little since this initial report. Oh S. Typical radiation doses range from 45 to 50 Gy and are accompanied by fluorouracil (5-FU) plus mitomycin C. Squamous. Leucovorin is often used in combination with 5-FU and other agents in the treatment of adenocarcinoma of the rectum. J Clin Oncol 14:25272539. but is not included in regimens for epidermoid carcinoma of the anus. John M.American College of Surgeons SESAP 13 Critique of Category 14 Item 11 Epidermoid anal canal cancer is a locally aggressive malignancy that accounts for 1% to 6% of all anorectal neoplasms. the distal margin of the anal canal is defined as the (A) anal verge (B) anoderm (C) first valve of Houston (D) dentate line (E) intersphincteric groove page 1094 .American College of Surgeons SESAP 13 Category 14 Item 12 When evaluating a patient for anal cancer. and basaloid cell types. in National Comprehensive Cancer Network: Clinical Practice Guidelines in Oncology-v 2. they all have similar prognoses and treatment responses. It is devoid of secondary skin appendages such as sweat glands and hair follicles. Collectively termed epidermoid carcinoma. transitional. 2002. Fritz AG. Fleming ID. in Greene FL. Page DL. The dentate line is a true mucocutaneous junction within the anal region located in the mid portion of the anal canal. ed 6. Fleming ID. Fritz AG. et al: Anal cancer. The anal canal begins proximally where the rectum enters the puborectalis sling at the apex of the anal sphincter complex (anorectal ring) and extends distally to the anal verge.Tumors in this region are distinguished as either anal canal tumors or anal margin tumors. The anal verge is just distal to the palpable intersphincteric groove and is grossly identified as the demarcation between the hair-bearing perianal skin distally and the nonhairbearing anoderm proximally. The valves of Houston are the three congenital folds up in the rectum above the anal canal. New York.American College of Surgeons SESAP 13 Critique for Category 14 Item 12 Anal canal cancer usually consists of squamous. REFERENCES 1.2006 2. Springer-Verlag. Choti MA. Benson AB. pp 139144 page 1095 . et al (eds): AJCC Cancer Staging Handbook. Anal margin tumors are treated like skin cancer. et al: Anal canal. cloacogenic.Greene FL.Engstrom PF. The anoderm is a specialized squamous epithelium between the anal verge and the dentate line. Page DL. Chen YJ. American College of Surgeons SESAP 13 Category 14 Item 13 Genetic testing for hereditary nonpolyposis colorectal cancer (A) has shown an MLH1 gene abnormality in more than 50% of cases (B) seeks to identify the responsible oncogene (C) can predict for the risk of desmoid tumor development (D) is useful for at-risk family members when no mutation is detected in the proband (E) can be performed on peripheral WBCs page 1096 . The progression requires additional genetic mutations. REFERENCES 1. Genetic testing through DNA sequencing is available for both FAP and HNPCC syndromes and can assist in clinical management.Cohen J: Overview of conscious sedation for gastrointestinal endoscopy.Chung DC: The genetic basis of colorectal cancer: insights into critical pathways of tumorigenesis. Dovidio M. not in HNPCC patients. MSH2 and MLH1 mutations are the most common. yet together are identified in 40% or fewer cases. is also an autosomally dominant hereditary form of colon cancer. They can be performed on any nucleated cell in the body. 2000 2. The phenotype of these tumors is microsatellite instability. PMS2. ranging from classic FAP to attenuated FAP. Desmoid tumors are seen in FAP patients. two well-defined. If a mutation is identified in the proband. a form of familial adenomatous polyposis (FAP). resulting from the germline mutation in the adenomatous polyposis coli (APC) tumor suppressor gene. Hereditary nonpolyposis colorectal cancer (HNPCC). including easily available peripheral WBCs.Syngal S. MSH6). et al: Interpretation of genetic test results for hereditary nonpolyposis colorectal cancer: implications for clinical predisposition testing. including activations of the K-ras oncogene and subsequently inactivation of p53 and DCC tumor suppressor genes. Available at: www. The location of the genetic mutation in the APC gene can roughly predict the phenotypic spectrum of the disease. 1999 page 1097 . Of the five most common mismatch repair genes (MSH2. Fox EA. and the well-described adenoma to carcinoma sequence. looking for the same mutation. then atrisk family members can be tested. not an oncogene.uptodate. or Lynch syndrome. MLH1.com 3. PMS1. in which normally preserved short repeat segments of DNA are found to have disordered nucleotides. Loss of mismatch repair genes allows normal damage to DNA to go uncorrected. Li C. Loss of both copies of the APC gene plays a critical role in the early transition from normal mucosa into the adenoma to carcinoma sequence. JAMA 282:247-253. In addition. Gastroenterology 119:854-865. leading to a rapid accumulation of genetic mutations and ultimately cancer. These tests seek to identify germline mutations. UpToDate website. These discoveries were greatly facilitated by a readily available precursor lesion. Genetic testing always starts with a known affected individual (the proband). Gardner’s syndrome. If no abnormal mutation is found in the proband. the adenomatous polyp. then genetic testing of at-risk family members is not useful. is an autosomal dominant inherited disease caused by germline mutation in one of several mismatch repair genes. and even the risk of desmoid tumor development. This suspicion has subsequently been confirmed.American College of Surgeons SESAP 13 Critique of Category 14 Item 13 The genetic pathways for colorectal cancer have become well elucidated over the past decade. clinically distinct inherited colon cancer syndromes (hereditary nonpolyposis colorectal cancer and familial adenomatous polyposis) imply that two different genetic pathways were likely. the oral form of fluorouracil (5-FU).American College of Surgeons SESAP 13 Category 14 Item 14 Which of the following statements about systemic therapy for colorectal cancer is NOT true? (A) Prospective studies have demonstrated that chemotherapy prolongs survival and enhances quality of life in patients with metastatic disease (B) Capecitabine (Xeloda). bolus 5-FU. and 5-FU given as a continuous infusion have similar side effect profiles (C) Most patients with resected stage II (node-negative) colon cancers should receive adjuvant chemotherapy (D) The response rates achieved with the combination of oxaliplatin. and leucovorin (FOLFOX) are superior to those achieved with 5-FU and leucovorin alone in patients with metastatic colon cancer (E) Adjuvant chemotherapy has increased the likelihood of cure by 30% in patients with stage III colon cancer page 1098 . Recent studies have demonstrated the oral form of 5-FU (capecitabine. Zidani R. Prospective randomized studies in patients with metastatic colorectal cancer demonstrate improved survival and quality of life in patients treated with flourouracil (5-FU) and leucovorin. Exceptions to these recommendations are patients with node-negative tumors who present with perforation or obstruction. However. 2005 page 1099 . J Clin Oncol 18:136-147.American College of Surgeons SESAP 13 Critique for Category 14 Item 14 The efficacy of adjuvant chemotherapy for patients with colorectal cancer and chemotherapy for patients with metastatic colorectal cancer has improved dramatically over the last decade. Figer A. Le Bail N. et al: Leucovorin and fluorouracil with or without oxaliplatin as first-line treatment in advanced colorectal cancer. Addition of oxaliplatin to this regimen (FOLFOX) further improves response rates. Perpoint B. The convenience of daily oral medication has resulted in increased usage of capecitabine. current Medicare data suggest that many patients in the United States with node-negative tumors are receiving adjuvant chemotherapy . Mayer RJ: Systemic therapy for colorectal cancer. bevacizumab (Avastin) or cetuximab (Erbitux). et al: Phase III multicenter randomized trial of oxaliplatin added to chronomodulated fluorouracil-leucovorin as first-line treatment of metastatic colorectal cancer. Current guidelines recommend adjuvant chemotherapy only in patients with node-positive colorectal cancer.Meyerhardt JA. to FOLFOX in the adjuvant setting. REFERENCES 1. Despite these clear recommendations. N Engl J Med 352:476-487. Seymour M. eg. Homerin M. 2000 3.de Gramont A.Giacchetti S. or Xeloda) is as effective as infusional 5-FU. 2000 2. the expectation that toxicity would be less has not been realized. Current studies are assessing the value of adding biologic agents. J Clin Oncol 18:2938-2947. gram-negative rod bacteria.American College of Surgeons SESAP 13 Category 14 Item 15 A 67-year-old man with dyspepsia undergoes endoscopy showing a 3-cm submucosal mass in the body of the stomach. The most appropriate therapy for this patient would be (A) total gastrectomy (B) subtotal gastrectomy (C) oral antibiotics (D) chemotherapy (E) local excision page 1100 . Biopsy identifies mucosa-associated lymphoid tissue (MALT) and spiral-shaped. metronidazole. Lancet 345:1591-1594. Treatment for gastric MALT must be examined in light of the favorable prognosis for most patients with localized disease. Gastroenterology 105:1662-1671. Thiede C. Brousse N.American College of Surgeons SESAP 13 Critique for Category 14 Item 15 In 1983 two British pathologists made the observation that certain extranodal lymphomas were related to mucosa-associated lymphoid tissue (MALT) rather than to nodal lymphoid tissue. The gastrointestinal tract is the most commonly affected primary site and the stomach is the most frequent site in the gastrointestinal tract. amoxicillin. Delmer A. N Engl J Med 337:217-222. Canadian Collaborative Group on Endometriosis. 1995 2. Neubauer A. MALT Lymphoma Study Group.Ruskone-Fourmestraux A. Gastric MALT is linked to bacterial infection as an etiology. Berube S: Laparoscopic surgery in infertile women with minimal or mild endometriosis. Surgical resection of the stomach with focal MALT can control the disease.Marcoux S. Portlock CS. and common symptoms are bleeding. such as life-threatening hemorrhage. Those who relapse should be treated with radiotherapy. J Clin Oncol 16:1916-1921. MALT patients who are H pylori positive. and omeprazole can result in complete regression of MALT in 50% to 80% of cases. Radiation therapy is safe and effective for localized gastric MALT and leads to disease-control rates near 100%. Chemotherapy has not been successful for control of local disease and is associated with high rates (40%) of relapse. Aegerter P. however. as in this case. Maheux R. epigastric pain. Gastric MALT is usually localized to the body and antrum of the stomach on presentation.Bayerdorffer E. Constitutional symptoms are not usually present. 1997 3. Groupe d'Etude des Lymphomes Digestifs. REFERENCES 1. Eradication of H pylori using regimens such as clarithromycin. 1993 4. 1998 page 1101 . It is now widely accepted that MALT represents a distinct subset of B-cell malignancies that differs from other low-grade B-cell lymphomas in its morphologic and clinical characteristics. Rudolph B. et al: Regression of primary gastric lymphoma of mucosa-associated lymphoid tissue type after cure of Helicobacter pylori infection. Yahalom J: Treatment of mucosa-associated lymphoid tissue lymphoma of the stomach with radiation alone. leading to the theory that Helicobacter pylori acts as an antigenic stimulus in the formation of acquired gastric lymphoid tissue and MALT.Schechter NR. and dyspepsia. this should be reserved for the urgent setting in which radiation therapy cannot be delivered. should first be treated with antibiotics and closely followed-up for possible lymphoma relapse. et al: Primary digestive tract lymphoma: a prospective multicentric study of 91 patients. Which of the following offers the greatest benefit to this patient? (A) Staging laparoscopy (B) Splenectomy (C) Radiation therapy (D) Chemotherapy (E) Chemotherapy and autologous bone marrow transplant page 1102 . A bone marrow biopsy demonstrates marginal B-cell lymphoma. Chest and abdominal computed tomographic (CT) scan shows no lymphadenopathy. and thrombocytopenia. anemia.American College of Surgeons SESAP 13 Category 14 Item 16 A 68-year-old woman has worsening chronic abdominal complaints. an enlarged spleen. 1992 3. Autologous transplants have the advantage of a lower risk of graft rejection. et al: Splenic marginal-zone lymphoma: a distinct clinical and pathological entity. Laparoscopy is useful for staging of intra-abdominal malignancies. but these are usually incomplete and not durable.Chacon JI. Bone marrow involvement with lymphoma is a frequent finding. Callet-Bauchu E. Peripheral lymphadenopathy is uncommon. but its role in the treatment of low-grade lymphomas is controversial and it is not a described therapy for splenic marginal zone lymphoma. Blood 100:1648-1654. can benefit greatly from splenectomy. Am J Surg Pathol 16:455-466. 2002 2. Munoz E. Felman P. Symptoms are produced by the splenomegaly that accompanies this disease.Thieblemont C. This approach has become the treatment of choice for patients with relapsed chemosensitive intermediate grade lymphoma.American College of Surgeons SESAP 13 Critique for Category 14 Item 16 Splenic marginal zone B-cell lymphoma is a subset of indolent B-cell lymphomas. REFERENCES 1. These doses also destroy stem cells remaining in the body. Patients with symptomatic splenomegaly or evidence of significant hypersplenism. Fever. patients report fatigue and weakness related to mild to moderate anemia. et al: Splenic marginal zone lymphoma: clinical characteristics and prognostic factors in a series of 60 patients. but not for this patient who already has a tissue diagnosis and staging. infection. chills. Isaacson PG: Splenic marginal zone cell lymphoma. benefits of splenectomy are observed in 90% of patients over the subsequent 4 years and many achieve more durable results. Algara P. Traverse-Glehen A. Autologous bone marrow transplantation during high-dose chemotherapy involves isolation and storage of hematopoetic stem cells. This approach often eliminates all symptoms and corrects hematologic abnormalities. 2003 page 1103 . and night sweats (B lymphoma symptoms) occur in fewer than 5% of patients. Radiation therapy can be used for patients who are not candidates for splenectomy and chemotherapy can induce responses. Lancet Oncol 4:95-103.Schmid C. such as anemia and thrombocytopenia. and graft versus host disease than allogeneic (from another donor) transplantation. Moreover. Diss T. but the return of the patient's own stored stem cells can re-populate the bone marrow. Mollejo M. Additionally. Kirkham N. High-dose chemotherapy is then used to kill lymphoma cells. along with a moderate peripheral leukocytosis. Serum calcium is 11 mg/dL. There is no recent history of trauma. A representative image from recent magnetic resonance imaging (MRI) is shown.American College of Surgeons SESAP 13 Category 14 Item 17 A 76-year-old woman who has recently completed therapy for a T3N2M0 ductal carcinoma of the breast has the new onset of severe back pain and urinary incontinence. Results of other laboratory studies are normal. The most appropriate therapy for this patient would be (A) urgent chemotherapy (B) intravenous corticosteroids and urgent radiation therapy (C) emergency posterior decompressive laminectomy (D) intravenous pamidronate (E) epidural corticosteroid injection page 1104 . Bisphosphonates are a new class of agents that reduce the complications of bone metastases (pathologic fractures. 2000 2. page 1105 . Although laminectomy has been used historically for surgical decompression. is one of the most dreaded complications of metastatic cancer. American Society of Clinical Oncology Bisphosphonates Expert Panel.Maranzano E.Hillner BE. and should be obtained in any cancer patient presenting with new back pain. The decision to treat metastatic spinal tumors with operation versus radiation therapy remains controversial. as in this patient. Radiotherapy is the treatment of choice in most cases. Ingle JN. Perrucci E. Cord compression is an oncologic emergency and must be diagnosed and treated promptly to achieve best results and avoid progressive pain. and bladder and bowel incontinence. Prospective randomized studies found that using high-dose dexamethasone in combination with radiotherapy improves the odds that a patient will be ambulatory after therapy. spinal cord compression. paralysis. A transforaminal approach under image guidance is the usual route. Bisphosphonates should be given in patients with metastatic breast cancer who have imaging evidence of lytic lesions and are currently receiving other systemic therapy. Bisphosphonates have been shown to improve patient survival. Checcaglini F. regardless of findings on plain films or bone scans. Latini P. In general. Current surgical treatment is individualized and includes decompression (often using tumor resection) via an anterior or posterior approach combined with aggressive reconstruction and stabilization procedures. and hypercalcemia) associated with breast cancer. vertebral body collapse causes bony impingement on the cord or nerve roots. Magnetic resonance imaging (MRI) is the gold standard for diagnosis. Int J Radiat Oncol Biol Phys 24:301-306. operation is recommended for patients in whom radiation therapy is doselimited. or spine stabilization is necessary. Corticosteroids have been injected into the epidural space in an attempt to decrease abnormal concentrations of nociceptive and inflammatory mediators associated with neuropathic pain in spinal nerve roots. Metastatic spinal cord compression. This treatment has been used for chronic pain secondary to nerve impingement caused by herniated intravertebral discs and not compression secondary to spinal metastases.American College of Surgeons SESAP 13 Critique for Category 14 Item 17 Bone is the most common site of breast cancer metastasis. diagnosed in 3% to 7% of cancer patients. This patient’s MRI demonstrates tumor posterior to the cord without boney destruction or fracture of the vertebral body and does not meet these criteria. REFERENCES 1. Treatment before further loss of spinal cord function is imperative. et al: Radiation therapy of spinal cord compression caused by breast cancer: report of a prospective trial. because patients who are ambulatory at the time of diagnosis and treatment have a higher probability of obtaining a good response to radiation therapy and achieving a longer survival. sensory loss. but will not reverse complications of bony metastasis that have already occurred. Berenson JR. Janjan NA. and is the initial site of metastasis in 26% of cases. et al: American Society of Clinical Oncology guideline on the role of bisphosphonates in breast cancer. High-dose corticosteroids are recommended unless contraindicated or the patient is ambulatory or asymptomatic while receiving radiation therapy. J Clin Oncol 18:1378-1391. laminectomy alone achieves results no better than radiation therapy and has been abandoned. Hughes JE. Ricci S. et al: Treatment of neoplastic spinal cord compression: results of a prospective study.American College of Surgeons SESAP 13 1992 3. A prospective analysis of 105 consecutive patients. Latini P. Checcaglini F. Cancer 67:1311-1317. Cafferty M. Neurosurgery 29:645-650. et al: Radiation therapy in metastatic spinal cord compression.Maranzano E. Digiacinto GV. 1991 page 1106 .Sundaresan N. 1991 4. American College of Surgeons SESAP 13 Category 14 Item 18 A 39-year-old man with neurofibromatosis has a 6-month history of increasing constipation. The computed tomographic (CT) scan shown is obtained. Which of the following statements about management is TRUE? (A) Positron emission tomography (PET) has no role in this patient’s work-up (B) Resection should not include any major structures because this lesion is benign (C) A diagnosis of malignancy can be excluded preoperatively (D) Radiation therapy is curative (E) Local control can be obtained with a complete resection of this lesion page 1107 . Most are of the peripheral type (NF type I or von Recklinghausen’s disease). as was shown here. 1998 2. Most commonly these peripheral tumors are neurofibrosarcoma and present with the rapid progression of symptoms suggestive of malignant degeneration. Ann Nucl Med 19:55-58. VIIIth nerve masses (NF type II) in conjunction with peripheral nerve tumors. Central (NF type II) and segmental (isolated to a body region or limb) types also occur. Nishitani H: FDG-PET/CT findings of sarcomatous transformation in neurofibromatosis: a case report. optic nerve gliomas. Local control is best achieved with complete resection of the lesion. neurofibromas.American College of Surgeons SESAP 13 Critique for Category 14 Item 18 Neurofibromatosis (NF) is a neurocutaneous condition affecting 1 in 3000 persons. REFERENCES 1. Pathologic correlation is recommended in these cases. Graham MM. Patients with NF type I have an estimated 3% to 15% lifetime risk of developing malignant disease. Radiation therapy may be indicated in these patients for local control. long-term survival depends on extent of resection and the presence of clean margins on resection. axillary or inguinal freckling. Based on its location and rapid rate of growth. or. Key features include the presence of multiple café au lait nodules. as in the patient described here. and a family history. Magnetic resonance imaging (MRI) and computed tomography (CT) should be performed to define the anatomic extent because many of these tumors may be quite large or not apparent on physical examination. 2005 page 1108 . Half of the lesions treated with surgical resection are either completely excised without recurrence. but is not curative. Positron emission tomography (PET) with fluoro-deoxyglucose (FDG) is recommended preoperatively because a high FDG uptake is suggestive of malignancy. this patient’s tumor should be assumed to be a neurofibrosarcoma. As with all sarcomatous lesions. If complete resection is not possible. the remaining disease does not progress. In patients with NF type I. Radiation therapy has no role in the primary treatment of these lesions.Karnes PS: Neurofibromatosis: a common neurocutaneous disorder. since malignancy cannot always be excluded preoperatively. but small areas of sarcomatous degeneration may be missed on preoperative biopsy. Tumors of the plexiform type are irregular and may interdigitate with vital structures. symptomatic control of bowel symptoms can be achieved with diverting colostomy with far less morbidity than attempted partial resection. sphenoid wing dysplasia. which may require en bloc resection of associated structures. Some benign tumors may demonstrate increased FDG uptake. Kubo A. the most common tumor is a neurofibroma that may occur along the length of any nerve. if complete resection is not possible. iris hamartomas (Lisch nodules).Otsuka H. Mayo Clin Proc 73:1071-1076. making preoperative diagnosis difficult. it is otherwise asymptomatic and produces no discomfort. as shown. It has been present for “many years.” but recently has become ulcerated and weeping. The popliteal and inguinal nodal basins are free of palpable adenopathy. Histopathologic examination of the lesion demonstrates melanoma. Likewise.American College of Surgeons SESAP 13 Category 14 Item 19 A 62-year-old man presents with the lesion on his left heel. there is nothing to suggest metastasis in transit The next step should be (A) amputation (B) isolated limb perfusion (C) sentinel lymph node biopsy (D) local excision and skin grafting (E) excision with regional lymph node dissection page 1109 . Surprisingly. Nieweg OE. Surgical extirpation remains the mainstay of treatment of cutaneous melanoma. melanoma of the foot has a worse prognosis than lesions more proximal to the trunk. Amputation may have a role in selected patients. 2004 6. J La State Med Soc 151:373376. Fisher SG. eg. A positive sentinel lymph node would preclude amputation as a curative procedure and local control could be obtained with local excision and some form of wound coverage. Cancer 85:383-388. 2003 page 1110 . Women and those of Celtic heritage are more commonly affected. Surg Gynecol Obstet 169:1-6. 2005 2. split-thickness skin graft or local flap. J Foot Ankle Surg 42:193-198. et al: Isolated limb perfusion: what is the evidence for its use? Ann Surg Oncol 11:837-845. et al: Predictors of outcome after hyperthermic isolated limb perfusion: role of tumor response. a principle that should be applied prior to any type of definitive surgical therapy. Brennan MF: Major amputation for advanced malignant melanoma. Isolated limb perfusion chemotherapy with or without hyperthermia is generally reserved for locoregional recurrences or bulky disease not amenable to surgical resection.American College of Surgeons SESAP 13 Critique of Category 14 Item 19 For reasons that remain unclear. Morton DL: Survival of patients with melanoma of the lower extremity decreases with distance from the trunk. 1999 4. 1999 3. Coit DG. Arch Surg 140:1115-1120. Vrouenraets BC. including local excision.Aloia TA. Regional nodal disease and distant metastasis should be excluded before amputation is considered.Harwood AR: Radiotherapy of acral lentiginous melanoma of the foot.Jaques DP. REFERENCES 1.Hsueh EC. Qi K. Sage RA: Survival of patients with primary pedal melanoma. 1989 5.Walsh SM. Lymphatic mapping and sentinel lymph node biopsy should be the first step in the diagnostic evaluation of this patient.Noorda EM. Van Coevorden F. including curative operation in subungual melanoma and for in-transit metastasis developing after excision of the primary lesion. Lucci A. Onaitis M. Mosca PJ. Grubbs E. American College of Surgeons SESAP 13 Category 14 Item 20 Which of the following genetic mutations is NOT part of the pathogenesis of colon malignancy? (A) DCC (B) k-ras (C) APC (D) p53 (E) N-myc page 1111 . The human genome project has shown the way of the future. One example is familial adenomatous polyposis. a mutation in the adenomatous polyposis coli (APC) gene. leading to cellular morphologic alteration and to changes in cell behavior.Grady WM: Genomic instability and colon cancer. The concept of two colons-two cancers is based on the different genetic profiles of tumors from the right and left colon. Interestingly. These changes impart some survival advantage to these clone lines. Malignant degeneration can also occur when DNA is damaged over time or by exposure to carcinogenic agents.Bellacosa A: Genetic hits and mutation rate in colorectal tumorigenesis: versatility of Knudson’s theory and implications for cancer prevention. Hum Mutat 21:271-276. As yet it has not been associated with colon cancer. Further. and are more frequently identified in tumors below the splenic flexure. and immune modulation. Colon cancer results from a series of sequential events as genetic and epigenetic aberrations accumulate and are manifest in the transition from normal epithelium to malignant epithelium. 2002 page 1112 . K-ras is important in transmembrane signal transduction. Surg Clin North Am 82:891-904. p53 expression portends a worse survival for patients treated with chemotherapy.Jass JR: Pathogenesis of colorectal cancer. Genomic therapy shows promise through techniques involving viral oncolytic therapy. oncogene modulation. Mutations in the p53 tumor suppressor gene are found in 50% of all colon cancers. Br Med Bull 64:27-43. the development of suicide genes. These mutations are the ones most commonly identified in colon cancers. 2003 5. Bodmer WF: Genetics of colorectal cancer: hereditary aspects and overview of colorectal tumorigenesis. and nodal status—are crude at best and offer no insight into the tumor’s response to treatment regimens. Malignant degeneration can be genetically programmed. Wilding JL. Genes Chromosomes Cancer 38:382-388. although c-myc has. The APC gene is a tumor suppressor gene responsible for products related to cellular adhesion. 2002 3. 2003 2. N-myc is generally associated with pediatric neural and renal cancers and lung cancers. 2004 4. Microarray analysis has provided the opportunity to elucidate the central command and control center of individual tumors. Mutations in the k-ras gene may result in continuous stimulation of the ras with the effect of eliminating programmed apoptosis. Delete colon cancer gene (DCC) is also a tumor suppressor gene operative later in the colon polyp cancer transition. Our current prognostic indicators—size. and intracellular communication. apoptosis.Fearnhead NS. transfection with genes that render targeted cells more susceptible to standard treatment modalities such as radiation therapy or flurouracil through reactivation of p53 has shown early promise. Cancer Metastasis Rev 23:11-27. APC gene mutations are believed to occur early in the transition from normal mucosa to polyp formation. Loss of genetic stability is the key molecular step. depth of invasion. REFERENCES 1. This family of genes is believed to control cell proliferation and apoptosis.Iacopetta B: TP53 mutation in colorectal cancer.American College of Surgeons SESAP 13 Critique for Category 14 Item 20 Understanding the biologic basis of individual tumors is difficult because cancers of any particular organ system consist of a group of heterogenous diseases. Inhibition of some of these oncogenes or the repair and reactivation of others may allow development of targeted genomic therapy. Dunlop MG: Gene therapy for colon cancer. 2002 7. 1998 page 1113 . Lutz W: Regulation of N-myc expression in development and disease.Zwacka RM. Cancer Lett 180:107-119. Hematol Oncol Clin North Am 12:595-615.Strieder V.American College of Surgeons SESAP 13 6. The next step should be (A) chest computed tomography (CT) (B) right hemicolectomy (C) adjuvant chemotherapy (D) staging laparotomy with lymph node biopsies (E) no additional studies or interventions page 1114 . The final histopathologic report describes mucinous cystadenoma.American College of Surgeons SESAP 13 Category 14 Item 21 An otherwise healthy 62-year-old woman undergoes successful laparoscopic appendectomy for nonperforated appendicitis with no other pelvic pathology. II. Maddern GJ.Wolff M. They can be found in up to 5% of appendices. mucinous cystadenomas. Bertrand C. Right hemicolectomy and tumor staging are recommended for malignant lesions that are invasive or involve the proximal one third of the appendix.Deans GT. mucosal hyperplasia. 1976 page 1115 . Ahmed N: Epithelial neoplasms of the vermiform appendix (exclusive of carcinoid).Landen S. 1975 4. Herman D. Br J Surg 82:299-306. Their relationship to hyperplastic polyps. which is why some authors recommend colonoscopy when these neoplasms are found.Qizilbash AH: Mucoceles of the appendix. Benign lesions of the appendix in any location and malignant lesions confined to the distal two thirds of the appendix without invasion may be managed by appendectomy alone. This recommendation is based on a retrospective review of 42 cases and there is no further evidence to support or refute this. and adenomatous polyps of the appendix. papillary adenomas. Retention cysts. Malignant neoplasms will generally be recognized by cellular atypia or invasion of the cyst wall. Appendiceal cystadenomas are histologically similar to adenomas of the colon and elsewhere. 1992 3. They may be associated with synchronous or metachronous lesions in about 25% of cases. et al: Appendiceal mucoceles and pseudomyxoma peritonei. and cystadenoma are benign. whereas cystadenocarcinoma is malignant and has a 5-year survival rate of 55%. Cancer 37:25112522. Cystadenomas. with only half recognized at the time of operation. REFERENCES 1. and cystadenocarcinomas. Mucoceles are the most common neoplasm and are divided into 4 histologic types. Spence RA: Neoplastic lesions of the appendix. Arch Pathol 99:548-555. 1995 2.American College of Surgeons SESAP 13 Critique for Category 14 Item 21 Appendicitis is the most common presenting symptom of appendiceal neoplasms. Surg Gynecol Obstet 175:401-404. American College of Surgeons SESAP 13 Category 14 Item 22 For a patient with a T3N2 adenocarcinoma of the distal esophagus. neoadjuvant chemoradiotherapy (A) decreases perioperative length of stay (B) increases survival rate in pathologic complete responders (C) produces equivalent survival rates to surgical resection followed by chemoradiotherapy (D) is most effective when chemotherapy and radiation therapy are delivered sequentially (E) increases perioperative morbidity page 1116 . Wong RK. 2004 4. Zuraw L. Many patients with advanced disease never complete the regimen. and three-dimension conformal radiotherapy. neoadjuvant chemoradiotherapy has not been shown to affect perioperative morbidity or length of stay. Chung TD. those patients treated with multimodality therapy (neoadjuvant chemoradiotherapy followed by surgical resection) who achieve a complete pathologic response (no evidence of residual tumor in the resected specimen) have a significant improvement in 3-year survival compared with those who do not achieve a complete response. BMC Cancer 4:67. whether via a transhiatal or transthoracic approach. Surgical resection. 2004 5. J Surg Oncol 87:68-74. 2004 3. neoadjuvant chemotherapy.Malthaner RA. Despite the concern that preoperative treatment would increase the risk of surgical complications. 2005 2. A prolonged postoperative recovery significantly delays subsequent chemoradiotherapy. Wong RK. Although overall mortality from this aggressive disease remains high. Fitzgerald L. is associated with a spectrum of perioperative complications. In addition. Lancet Oncol 6:659-668. but randomized trials have clarified several aspects of multimodality therapy. concomitant delivery of chemotherapy and radiation has proven to be superior to sequential preoperative therapies.Malthaner RA.American College of Surgeons SESAP 13 Critique of Category 14 Item 22 Operation remains the preferred treatment of clinically localized esophageal carcinoma. Vasan H: A meta-analysis of randomized controlled trials that compared neoadjuvant chemoradiation and surgery to surgery alone for resectable esophageal cancer. Zuraw L. Rumble RB. Am J Surg 185:538-543. Kaplan BJ. REFERENCES 1.Urschel JD. Smithers BM.Frechette E. Buck DA. The recommended management of more advanced disease continues to evolve. et al: Neoadjuvant or adjuvant therapy for resectable esophageal cancer: a systematic review and meta-analysis. et al: Surgery alone versus chemoradiotherapy followed by surgery for resectable cancer of the oesophagus: a randomised controlled phase III trial. 2003 page 1117 .Burmeister BH. Rumble RB. Gebski V. BMC Med 2:35. et al: Esophageal cancer: outcomes of surgery. et al: Neoadjuvant or adjuvant therapy for resectable esophageal cancer: a clinical practice guideline. superior mesenteric vein. and portal vein are not involved. A fine-cut. dual phase computed tomographic (CT) scan with contrast and endoscopic ultrasound (EUS) both demonstrate a 2.5-cm solid mass in the head of the pancreas.American College of Surgeons SESAP 13 Category 14 Item 23 A 57-year-old man presents with painless jaundice and a 15-lb weight loss over the previous 3 months. and no evidence of extrapancreatic disease. The foregut arteries. The next step should be (A) angiography (B) endoscopic placement of a biliary stent and simultaneous brush cytology (C) intraoperative ultrasonography and enucleation (D) pancreaticoduodenectomy (E) percutaneous biopsy page 1118 . and if protocol-based chemotherapy is considered. instead measuring disease based on serum markers. Saunders.Alexakis N. not for potentially resectable lesions. Also. EUS can confirm and direct biopsies more safely than the percutaneous route. eg. Current chemoradiation protocols may not need biopsy material. Philadelphia.American College of Surgeons SESAP 13 Critique for Category 14 Item 23 This patient has a resectable pancreatic carcinoma. Although angiography is not contraindicated. Ghaneh P.Steer ML: Exocrine pancreas. 2004 2. but only for the specific protocol. Endoscopic ultrasonography (EUS) adds a substantial false positive rate to the consideration of unresectability by overcalling vascular invasion. Raraty M. a dilated common bile duct is easier to anastomose than a small. If unresectable by radiographic standards. and leads to higher rates of infectious complications. If likely to be unresectable by CT findings. Enucleation with ultrasonography is reserved for benign endocrine neoplasms of the pancreas. et al: Current standards of surgery for pancreatic cancer. and are likely to have a high false-negative rate. inflamed one harboring a contaminated stent. Stents should be reserved for palliation. Because biopsies of this lesion may lead to a pancreatic fistula or bleeding. The only hope for long-term survival is with complete resection. A biliary stent contaminates the biliary tract with bowel flora. CA 19-9 levels. Halloran C. REFERENCES 1. biopsy prior to attempted resection is unwarranted. it does not yield useful information above and beyond a high-quality CT scan. pp 1643-1678 page 1119 . Br J Surg 91:1410-1427. ed 17. in Townsend CM Jr (ed): Sabiston Textbook of Surgery: The Biological Basis of Modern Surgical Practice. A high-quality computed tomographic (CT) scan is the only imaging needed prior to resection in most cases. 2004. then a tissue biopsy may be useful. American College of Surgeons SESAP 13 Category 14 Item 24 Intraductal papillary mucinous neoplasms (IPMNs) of the pancreas (A) are always benign (B) are most commonly found in the tail of the pancreas (C) have a lower 5-year survival than adenocarcinoma of the pancreas (D) frequently recur (E) are preferentially treated by enucleation page 1120 . Hruban RH. but recurrence is also noted in up to 15% of patients believed to have noninvasive disease at initial resection.American College of Surgeons SESAP 13 Critique for Category 14 Item 24 Intraductal papillary mucinous neoplasms (IPMNs) are being found with increased frequency because of higher resolution computed tomographic (CT) scans. 2004 2. et al: Intraductal papillary mucinous neoplasms of the pancreas: an updated experience. The operative approach depends on the location.D’Angelica M. Five-year survival is up to 45% compared with 20% for adenocarcinoma of the pancreas. Given the 50% chance of cancer. Invasive cancer is present in 38% to 48% of resected IPMNs. enucleation is rarely indicated. REFERENCES 1. Careful postoperative surveillance is essential. Although the head and uncinate process are commonly involved. This incidence likely reflects incomplete resection of multifocal disease or a misclassification of these tumors at the time of operation. Yeo CJ. and finally invasive cancer. IPMNs are mucin-producing cystic neoplasms of the pancreas that generally involve the main pancreatic duct or major side branches. Klimstra D. Cameron JL. 12% to 24% of these neoplasms are confined to the distal half of the pancreas. 2004 page 1121 . Ann Surg 239:788-799. Ann Surg 239:400-408. Suriawinata AA. Brennan MF. IPMNs seem to progress from adenoma to dysplasia to carcinoma in situ. et al: Intraductal papillary mucinous neoplasms of the pancreas: an analysis of clinicopathologic features and outcome. and the favorable prognosis with complete resection. the high recurrence with incomplete resection. Recurrence rates are high in patients with invasive disease.Sohn TA. Like other tumors. 4 mg/dL. his systolic blood pressure is 95 mm Hg. and lactate. The next step in management should be (A) exploratory laparotomy (B) diagnostic laparoscopy (C) percutaneous cecostomy (D) colonoscopic decompression (E) neostigmine page 1122 . Two weeks after transplant. mild diffuse abdominal pain. he develops diarrhea. He is tachycardic. 5. The abdominal computed tomographic (CT) scan shown is obtained. Three days later.2. 1. and increased flatulence. 17 mEq/L. serum bicarbonate.American College of Surgeons SESAP 13 Category 14 Item 25 A 26-year-old man with long-standing nodular sclerosing Hodgkin’s disease has a bone marrow transplant from a sibling. His abdomen is distended with generalized tenderness but no guarding or rebound. Pertinent data include WBC count. and he has low urine output despite receiving 2 L of normal saline. he develops signs of graft versus host disease and is treated with corticosteroids. St Peter SD. Most patients are diagnosed after abdominal computed tomography (CT). Most patients should be made NPO initially and given broad-spectrum antibiotics if an underlying infectious process is suspected. Percutaneous cecostomy. These findings strongly suggest perforation of the gastrointestinal tract or ischemic bowel and mandate laparotomy. and its significance depends on the patient’s condition and the acuity of the primary event. 1999 2. Arch Surg 138:68-75. and expectant management along with treatment of the underlying condition could be attempted in stable patients. REFERENCES 1. pneumatosis has been reported to occur in immunosuppressed patients with graft versus host disease. Diagnostic laparoscopy would not contribute to this patient’s care and would be difficult given the degree of distention. The surgeon’s dilemma is determining whether operation will be needed. and gas-forming bacterial flora all contribute. but it seems likely that some combination of mucosal injury. colonic decompression. Also. Many patients with pneumatosis will have benign abdominal examinations. and neostigime would be considerations if the patient had acute colonic pseudo-obstruction (Ogilvie’s syndrome) rather than pneumatosis and free fluid. et al: Pneumatosis [correction of Pneumocystis] cystoides intestinalis with pneumoperitoneum and pneumoretroperitoneum in a patient with extensive chronic graft-versus-host disease. intraluminal pressure. although pneumatosis intestinalis can be seen on plain x-rays. Kelly KA: The spectrum of pneumatosis intestinalis. Abbas MA. Once diagnosed. As in this case. from a benign radiographic finding to intra-abdominal sepsis to multiple organ failure and death. Eisenhuber E. 2003 page 1123 . Pneumatosis represents a wide spectrum of disease. This patient is clearly developing sepsis and not responding well to resuscitation efforts. Bone Marrow Transplant 24:331-333. The exact mechanism of how gas forms or locates in the bowel wall continues to be debated.Schulenburg A. Herold C. Oberhuber G.American College of Surgeons SESAP 13 Critique for Category 14 Item 25 Pneumatosis intestinalis is the presence of gas in the wall of the bowel. a surgical consultation is usually requested. An underlying pathologic process is usually responsible for pneumatosis. the CT scan demonstrates free fluid in addition to pneumatosis intestinalis. 9-mm thick ulcerated melanoma with 1-cm clear margins.American College of Surgeons SESAP 13 Category 14 Item 26 A 55-year-old man presents with a pigmented lesion in the upper midline of his back. Excision reveals a 0. The next step in management should include (A) re-excision with 2-cm margins (B) levamisole and BCG (C) lymphoscintigraphy (D) adjuvant interferon therapy (E) vitamin A administration page 1124 . Physical examination is otherwise unremarkable. and sentinel lymph node biopsy is indicated. et al: Usefulness of preoperative lymphoscintigraphy for the identification of sentinel lymph nodes in melanoma.0 mm. In fact. Clin Exp Med 4:65-77. Stevens JS. The other agents listed have no standard role in the management of this patient’s melanoma. randomized data from the World Health Organization indicate that a 1-cm margin is adequate for T1 lesions. REFERENCES 1.Chung ES. Truncal melanomas are associated with an approximately 30% risk of ambiguous drainage (nodal drainage that cannot be predicted by anatomic location of the primary). Although wide excision should be done as well (and is often combined with sentinel node biopsy under the same anesthetic). it is ulcerated (stage T1b). Pommier RF. should be considered if the patient is node positive. 2004 2. 2001 page 1125 . and lymphoscintigraphy at the time of sentinel node mapping is essential to accurately identify the location of the sentinel node(s). on or off a clinical trial. Sondak VK: Current state of treatment for primary cutaneous melanoma.Morris KT. The presence of tumor ulceration significantly increases the risk of regional lymph node metastases. Adjuvant interferon therapy. sentinel lymph node status is the most important predictor of prognosis in this patient. Am J Surg 181:423426.American College of Surgeons SESAP 13 Critique for Category 14 Item 26 Although the lesion described has a Breslow thickness of less than 1. Fletcher WS. Sabel MS. American College of Surgeons SESAP 13 Category 14 Item 27 Which of the following statements about Mohs micrographic surgery is TRUE? (A) It produces the same cure rate for basal and squamous cell skin cancers as conventional excision (B) It is contraindicated for the treatment of melanoma in situ (C) It is contraindicated for the treatment of dermatofibrosarcoma protruberans (DFP) (D) It is a standard treatment for nonmelanoma skin cancers (E) The production of temporary painful open wounds is a barrier to widespread application page 1126 . Zitelli JA: Cutaneous head and neck melanoma treated with Mohs micrographic surgery. Although its use in these diseases is controversial.Bricca GM. 2005 2. REFERENCES 1. Leffell DJ: Mohs micrographic surgery: established uses and emerging trends. recent series show excellent results for in situ lesions. Oncology 19:1165-1175. Some consider it a treatment of choice for dermatofibrosarcoma protuberans (DFP). J Am Acad Dermatol 52:92-100.Pennington BE. 2005 page 1127 . Roenigk RK: A comparison between Mohs micrographic surgery and wide surgical excision for the treatment of dermatofibrosarcoma protuberans. Brodland DG. Other indications for Mohs surgery are emerging.Gloster HM Jr. Studies suggest that this technique maximizes tissue preservation while increasing cure rates for these lesions compared with standard excisions. Harris KR.American College of Surgeons SESAP 13 Critique for Category 14 Item 27 Mohs micrographic surgery is a staged skin resection technique involving frozen sections and mapping that is commonly applied for the treatment of nonmelanoma skin cancers (basal cell and squamous cell cancers). Ren D. 1996 3. J Am Acad Dermatol 35:8287. The resulting temporary open wounds are generally well tolerated and have not been a barrier to the use of this technique. This technique has also been used in both melanoma and melanoma in situ. as some series show lower recurrence rates compared with standard excision. the mass is hard and fixed. The most likely diagnosis is (A) malignant fibrous histiocytoma (B) recurrent testicular cancer (C) metastatic prostate cancer (D) metastatic adenocarcinoma (E) metastatic anal cancer page 1128 . there is no lymphadenopathy. On physical examination. he noticed a 4-cm mass in the right groin that has now grown to 12 cm. The computed tomographic (CT) scan shown is obtained.American College of Surgeons SESAP 13 Category 14 Item 28 A 62-year-old man had a right orchiectomy and radiation therapy for testicular cancer 30 years previously. Three years ago. The mass is not painful and he has no associated symptoms. or metastatic prostate cancer. REFERENCES 1. Eur J Cancer 29A:664-668. Adjuvant radiation therapy may be of value in the treatment of soft tissue sarcomas. Engelholm SA. 2002 4. fibrosarcoma. and osteosarcomas. Antonescu CR.Stefanovski PD. Bidoli E. Recurrent testicular cancer would be unlikely 30 years after initial treatment. Mellemgaard A. Maru S. Peralta EA. metastatic anal cancer. Chu DZ. angiosarcoma. The average time between exposure to ionizing radiation and the development of radiation-induced sarcoma is 10 years. the field has already been irradiated and it is not likely to tolerate additional doses sufficient to alter the tumor’s clinical course. Ann Surg 239:903-910. the view of the prostate appears normal on this computed tomographic (CT) image.Schwarz RE. The size of the lesion and the slow rate of growth argue against metastatic adenocarcinoma. Am J Clin Oncol 25:244-247. The optimal treatment of soft tissue sarcomas is wide local excision with surgical margins 2 cm or more from the gross tumor.Jacobsen GK. Moller H: Increased incidence of sarcoma in patients treated for testicular seminoma. Eur J Surg Oncol 28:153-164. but in the case presented. et al: Long-term survival after radical operations for cancer treatment-induced sarcomas: how two survivors invite reflection on oncologic treatment concepts. 2002 page 1129 . et al: Prognostic factors in soft tissue sarcomas: a study of 395 patients. Quan ML. Soft tissue sarcomas are relatively insensitive to chemotherapeutic agents. et al: Long-term results with resection of radiationinduced soft tissue sarcomas. Radiation-induced sarcomas account for 1% to 5% of all sarcomas. Hillebrand G.American College of Surgeons SESAP 13 Critique for Category 14 Item 28 The history and radiologic findings are most consistent with a radiation-induced sarcoma. 2004 2. Additionally. but can be as short as 1 year or as long as 50 years. Common histologic subtypes of radiation-induced sarcomas include malignant fibrous histiocytoma. Buonadonna A. De Paoli A.Cha C. 1993 3. The computed tomographic (CT) scan shown is obtained. The mass is not painful and he has no associated symptoms. On physical examination. the mass is hard and fixed. Three years ago.American College of Surgeons SESAP 13 Category 14 Item 29 A 62-year-old man had a right orchiectomy and radiation therapy for testicular cancer 30 years previously. there is no lymphadenopathy. he noticed a 4-cm mass in the right groin that has now grown to 12 cm. The best management would be (A) wide local excision (B) preoperative radiation therapy and wide local excision (C) chemotherapy (D) preoperative chemotherapy and wide local excision (E) chemotherapy and radiation therapy page 1130 . The optimal treatment of soft tissue sarcomas is wide local excision with surgical margins 2 cm or more from the gross tumor. Peralta EA. Hillebrand G.Jacobsen GK. De Paoli A. REFERENCES 1. Common histologic subtypes of radiation-induced sarcomas include malignant fibrous histiocytoma. Quan ML. Radiation-induced sarcomas account for 1% to 5% of all sarcomas. angiosarcoma. et al: Long-term results with resection of radiationinduced soft tissue sarcomas.Schwarz RE. Additionally. but in the case presented. 2002 4. The average time between exposure to ionizing radiation and the development of radiation-induced sarcoma is 10 years. Soft tissue sarcomas are relatively insensitive to chemotherapeutic agents. Adjuvant radiation therapy may be of value in the treatment of soft tissue sarcomas. Bidoli E. Mellemgaard A. 2002 page 1131 . or metastatic prostate cancer. et al: Long-term survival after radical operations for cancer treatment-induced sarcomas: how two survivors invite reflection on oncologic treatment concepts. the field has already been irradiated and it is not likely to tolerate additional doses sufficient to alter the tumor’s clinical course. Buonadonna A. Ann Surg 239:903-910. et al: Prognostic factors in soft tissue sarcomas: a study of 395 patients.Cha C. and osteosarcomas. Maru S. the view of the prostate appears normal on this computed tomographic (CT) image. Eur J Surg Oncol 28:153-164. The size of the lesion and the slow rate of growth argue against metastatic adenocarcinoma. Recurrent testicular cancer would be unlikely 30 years after initial treatment. Chu DZ.American College of Surgeons SESAP 13 Critique for Category 14 Item 29 The history and radiologic findings are most consistent with a radiation-induced sarcoma. fibrosarcoma. Moller H: Increased incidence of sarcoma in patients treated for testicular seminoma. 1993 3.Stefanovski PD. Engelholm SA. Eur J Cancer 29A:664-668. but can be as short as 1 year or as long as 50 years. 2004 2. Antonescu CR. metastatic anal cancer. Am J Clin Oncol 25:244-247. American College of Surgeons SESAP 13 Category 14 Item 30 Oral fluoropyrimidine (A) Fluorouracil (5-FU) (B) Capecitabine (C) Cetuximab (D) Irinotecan (E) Bevacizumab page 1132 . Venook A: Critical evaluation of current treatments in metastatic colorectal cancer. REFERENCES 1. Xeloda is associated with a significant incidence of severe hand-foot syndrome (palmar-plantar erythrodysesthesia).Allegra C. Its addition to 5-FU increases the response rate. and median duration of survival. multi-agent therapy has become standard treatment. being easier to administer. Cartwright T. The cisplatinum derivative oxaliplatin has become first-line treatment for patients with stage IV colorectal cancer when used in combination with 5-FU and leucovorin (FOLFOX regimen). Fehrenbacher L. In the US. Avastin is a monoclonal antibody directed against vascular endothelial growth factor (VEGF) and acts by inhibiting the growth of tumor blood vessels. More recently developed agents for treatment of stage IV colorectal cancer show great promise for decreasing risks or side effects of therapy. 2005 4. but blocking HER1-EGFR signaling is believed to render the tumor cells more vulnerable to cytotoxic chemotherapeutic agents. Although the optimal combination of dosing and sequencing of agents is not fully defined. et al: Bevacizumab plus irinotecan. N Engl J Med 352:2746-2748. or acting via a different mechanism. 2005 page 1133 . an inhibitor of topoisomerase I that was initially introduced as monotherapy for cancers resistant to 5-FU but has also demonstrated improved results in combination with 5-FU. 2005 2. Nowacki MP. randomized multi-institutional studies have confirmed its benefit for patients with stage III disease and even for those with stage II disease. fluorouracil. Two additional agents provide targeted molecular treatment for advanced colon cancer: bevacizumab (Avastin) and cetuximab (Erbitux). 5-FU is often combined with either oxaliplatin or irinotecan. Oncologist 10:250-261. Sargent DJ: Adjuvant therapy for colon cancer —the pace quickens. Erbitux is not sufficient by itself for tumor cytotoxicity. Although surgical resection cures 50% to 60% of patients with stage III disease. In more recent decades. Its oral form makes it easier to administer and its 45-minute half-life allows it to be given twice daily. N Engl J Med 352:2696-2704. et al: Capecitabine as adjuvant treatment for stage III colon cancer. Wong A. these combination agents have already improved median survival for patients with metastatic colorectal cancer from 12 months to 2 years.Hurwitz H. Erbitux is a monoclonal antibody that blocks the signaling by HER-1/EGFR (endothelial growth factor) receptors that stimulate tumor cell proliferation (migration. Abt M. up to 20% more can be cured if 5-FU is administered postoperatively. and angiogenesis and inhibit apoptosis).American College of Surgeons SESAP 13 Critique for Category 14 Item 30 Fluorouracil (5-FU) has been the standard cytotoxic chemotherapeutic agent for treatment of stage IV colon cancer for almost 50 years. N Engl J Med 350:2335-2342.Twelves C. adhesions. FOLFOX and FOLFIRI regimens are comparable in their overall efficacy and tolerability. a regimen known as FOLFIRI. and leucovorin for metastatic colorectal cancer. 2004 3. median time to disease progression. Novotny W. Controlled studies have demonstrated that its effectiveness is at least equal to that of 5-FU plus leucovorin with somewhat less neutropenia. Capecitabine (Xeloda) is an oral fluoropyrimadine prodrug that is converted through a multistep activation to fluorouracil at the cellular level. American College of Surgeons SESAP 13 Category 14 Item 31 Inhibitor of topoisomerase 1 (A) Fluorouracil (5-FU) (B) Capecitabine (C) Cetuximab (D) Irinotecan (E) Bevacizumab page 1134 . Abt M. N Engl J Med 352:2696-2704.Twelves C. Wong A. up to 20% more can be cured if 5-FU is administered postoperatively. Erbitux is not sufficient by itself for tumor cytotoxicity. Fehrenbacher L. being easier to administer. and median duration of survival. 2005 4. these combination agents have already improved median survival for patients with metastatic colorectal cancer from 12 months to 2 years.Venook A: Critical evaluation of current treatments in metastatic colorectal cancer. Cartwright T. a regimen known as FOLFIRI. FOLFOX and FOLFIRI regimens are comparable in their overall efficacy and tolerability. 5-FU is often combined with either oxaliplatin or irinotecan. N Engl J Med 350:2335-2342. median time to disease progression. Controlled studies have demonstrated that its effectiveness is at least equal to that of 5-FU plus leucovorin with somewhat less neutropenia. 2004 3. Its oral form makes it easier to administer and its 45-minute half-life allows it to be given twice daily. multi-agent therapy has become standard treatment. 2005 page 1135 . In the US. Its addition to 5-FU increases the response rate. an inhibitor of topoisomerase I that was initially introduced as monotherapy for cancers resistant to 5-FU but has also demonstrated improved results in combination with 5-FU. In more recent decades. N Engl J Med 352:2746-2748. Two additional agents provide targeted molecular treatment for advanced colon cancer: bevacizumab (Avastin) and cetuximab (Erbitux). randomized multi-institutional studies have confirmed its benefit for patients with stage III disease and even for those with stage II disease. or acting via a different mechanism.American College of Surgeons SESAP 13 Critique for Category 14 Item 31 Fluorouracil (5-FU) has been the standard cytotoxic chemotherapeutic agent for treatment of stage IV colon cancer for almost 50 years. Xeloda is associated with a significant incidence of severe hand-foot syndrome (palmar-plantar erythrodysesthesia). and leucovorin for metastatic colorectal cancer.Hurwitz H. but blocking HER1-EGFR signaling is believed to render the tumor cells more vulnerable to cytotoxic chemotherapeutic agents. More recently developed agents for treatment of stage IV colorectal cancer show great promise for decreasing risks or side effects of therapy. adhesions. Avastin is a monoclonal antibody directed against vascular endothelial growth factor (VEGF) and acts by inhibiting the growth of tumor blood vessels. Although the optimal combination of dosing and sequencing of agents is not fully defined.Allegra C. Although surgical resection cures 50% to 60% of patients with stage III disease. et al: Bevacizumab plus irinotecan. Oncologist 10:250-261. The cisplatinum derivative oxaliplatin has become first-line treatment for patients with stage IV colorectal cancer when used in combination with 5-FU and leucovorin (FOLFOX regimen). fluorouracil. Erbitux is a monoclonal antibody that blocks the signaling by HER-1/EGFR (endothelial growth factor) receptors that stimulate tumor cell proliferation (migration. and angiogenesis and inhibit apoptosis). et al: Capecitabine as adjuvant treatment for stage III colon cancer. 2005 2. Capecitabine (Xeloda) is an oral fluoropyrimadine prodrug that is converted through a multistep activation to fluorouracil at the cellular level. Novotny W. Nowacki MP. REFERENCES 1. Sargent DJ: Adjuvant therapy for colon cancer —the pace quickens. American College of Surgeons SESAP 13 Category 14 Item 32 Anti-angiogenic monoclonal antibody to vascular endothelial growth factor (VEGF) (A) Fluorouracil (5-FU) (B) Capecitabine (C) Cetuximab (D) Irinotecan (E) Bevacizumab page 1136 . median time to disease progression. REFERENCES 1. Abt M. and median duration of survival. a regimen known as FOLFIRI.Hurwitz H. Novotny W.Venook A: Critical evaluation of current treatments in metastatic colorectal cancer. More recently developed agents for treatment of stage IV colorectal cancer show great promise for decreasing risks or side effects of therapy. Fehrenbacher L. Sargent DJ: Adjuvant therapy for colon cancer —the pace quickens. Controlled studies have demonstrated that its effectiveness is at least equal to that of 5-FU plus leucovorin with somewhat less neutropenia. Xeloda is associated with a significant incidence of severe hand-foot syndrome (palmar-plantar erythrodysesthesia). 2005 4. Although surgical resection cures 50% to 60% of patients with stage III disease. being easier to administer. multi-agent therapy has become standard treatment. FOLFOX and FOLFIRI regimens are comparable in their overall efficacy and tolerability. or acting via a different mechanism. Erbitux is a monoclonal antibody that blocks the signaling by HER-1/EGFR (endothelial growth factor) receptors that stimulate tumor cell proliferation (migration.Twelves C. N Engl J Med 352:2746-2748. Cartwright T. Capecitabine (Xeloda) is an oral fluoropyrimadine prodrug that is converted through a multistep activation to fluorouracil at the cellular level. an inhibitor of topoisomerase I that was initially introduced as monotherapy for cancers resistant to 5-FU but has also demonstrated improved results in combination with 5-FU. The cisplatinum derivative oxaliplatin has become first-line treatment for patients with stage IV colorectal cancer when used in combination with 5-FU and leucovorin (FOLFOX regimen). N Engl J Med 352:2696-2704. and leucovorin for metastatic colorectal cancer. but blocking HER1-EGFR signaling is believed to render the tumor cells more vulnerable to cytotoxic chemotherapeutic agents. Avastin is a monoclonal antibody directed against vascular endothelial growth factor (VEGF) and acts by inhibiting the growth of tumor blood vessels.Allegra C. 2005 page 1137 . adhesions. Erbitux is not sufficient by itself for tumor cytotoxicity. et al: Capecitabine as adjuvant treatment for stage III colon cancer. up to 20% more can be cured if 5-FU is administered postoperatively. fluorouracil. 5-FU is often combined with either oxaliplatin or irinotecan. In more recent decades. Nowacki MP. these combination agents have already improved median survival for patients with metastatic colorectal cancer from 12 months to 2 years. N Engl J Med 350:2335-2342. Its addition to 5-FU increases the response rate. and angiogenesis and inhibit apoptosis). Oncologist 10:250-261. Wong A.American College of Surgeons SESAP 13 Critique for Category 14 Item 32 Fluorouracil (5-FU) has been the standard cytotoxic chemotherapeutic agent for treatment of stage IV colon cancer for almost 50 years. 2005 2. In the US. Its oral form makes it easier to administer and its 45-minute half-life allows it to be given twice daily. 2004 3. Two additional agents provide targeted molecular treatment for advanced colon cancer: bevacizumab (Avastin) and cetuximab (Erbitux). Although the optimal combination of dosing and sequencing of agents is not fully defined. randomized multi-institutional studies have confirmed its benefit for patients with stage III disease and even for those with stage II disease. et al: Bevacizumab plus irinotecan. American College of Surgeons SESAP 13 Category 14 Item 33 Standard adjuvant treatment of colon cancer for 50 years (A) Fluorouracil (5-FU) (B) Capecitabine (C) Cetuximab (D) Irinotecan (E) Bevacizumab page 1138 . multi-agent therapy has become standard treatment.Venook A: Critical evaluation of current treatments in metastatic colorectal cancer. Oncologist 10:250-261. REFERENCES 1. Nowacki MP. Fehrenbacher L. Erbitux is not sufficient by itself for tumor cytotoxicity. Capecitabine (Xeloda) is an oral fluoropyrimadine prodrug that is converted through a multistep activation to fluorouracil at the cellular level. but blocking HER1-EGFR signaling is believed to render the tumor cells more vulnerable to cytotoxic chemotherapeutic agents. Although surgical resection cures 50% to 60% of patients with stage III disease. a regimen known as FOLFIRI. FOLFOX and FOLFIRI regimens are comparable in their overall efficacy and tolerability. Its oral form makes it easier to administer and its 45-minute half-life allows it to be given twice daily. Avastin is a monoclonal antibody directed against vascular endothelial growth factor (VEGF) and acts by inhibiting the growth of tumor blood vessels.Twelves C. N Engl J Med 352:2746-2748. Abt M. an inhibitor of topoisomerase I that was initially introduced as monotherapy for cancers resistant to 5-FU but has also demonstrated improved results in combination with 5-FU. randomized multi-institutional studies have confirmed its benefit for patients with stage III disease and even for those with stage II disease. 2005 2. Two additional agents provide targeted molecular treatment for advanced colon cancer: bevacizumab (Avastin) and cetuximab (Erbitux). 2005 page 1139 . N Engl J Med 350:2335-2342. adhesions. Erbitux is a monoclonal antibody that blocks the signaling by HER-1/EGFR (endothelial growth factor) receptors that stimulate tumor cell proliferation (migration. Although the optimal combination of dosing and sequencing of agents is not fully defined. and median duration of survival. et al: Bevacizumab plus irinotecan. N Engl J Med 352:2696-2704. Wong A.Allegra C. being easier to administer. fluorouracil. 2004 3. In the US. Xeloda is associated with a significant incidence of severe hand-foot syndrome (palmar-plantar erythrodysesthesia). 5-FU is often combined with either oxaliplatin or irinotecan. up to 20% more can be cured if 5-FU is administered postoperatively. median time to disease progression. Novotny W. Sargent DJ: Adjuvant therapy for colon cancer —the pace quickens. In more recent decades. Its addition to 5-FU increases the response rate. Cartwright T. Controlled studies have demonstrated that its effectiveness is at least equal to that of 5-FU plus leucovorin with somewhat less neutropenia. 2005 4. or acting via a different mechanism. and leucovorin for metastatic colorectal cancer.American College of Surgeons SESAP 13 Critique for Category 14 Item 33 Fluorouracil (5-FU) has been the standard cytotoxic chemotherapeutic agent for treatment of stage IV colon cancer for almost 50 years. these combination agents have already improved median survival for patients with metastatic colorectal cancer from 12 months to 2 years. The cisplatinum derivative oxaliplatin has become first-line treatment for patients with stage IV colorectal cancer when used in combination with 5-FU and leucovorin (FOLFOX regimen). More recently developed agents for treatment of stage IV colorectal cancer show great promise for decreasing risks or side effects of therapy. et al: Capecitabine as adjuvant treatment for stage III colon cancer.Hurwitz H. and angiogenesis and inhibit apoptosis). American College of Surgeons SESAP 13 Category 14 Item 34 Capecitabine (A) Inhibits DNA synthesis (B) Anti-angiogenesis drug (C) Fluorouracil precursor (D) Anti-microtubule agent that blocks mitosis (E) Inhibits epidermal growth factor receptor page 1140 . Abt M. Wong A. Capecitabine (Xeloda) is an oral fluorouracil (5-FU) pro-drug precursor. leucovorin. N Engl J Med 352:2696-2704. The final stage of conversion to fluorouracil is catalyzed by thymidine phosphorylase. Capecitabine had the same efficacy profile and perhaps a better safety profile than 5-FU. Cetuximab (Erbitux) is a monoclonal antibody that blocks the epidermal growth factor receptor (EGFR). REFERENCES 1.American College of Surgeons SESAP 13 Critique for Category 14 Item 34 All the medications listed in these items have been used as chemotherapy in the treatment of stage III. et al: Capecitabine as adjuvant treatment for stage III colon cancer. It is often given in combination with conventional cytotoxic chemotherapy agents.com 2. 5-FU. Up To Date Website. Leucovorin: Drug information. eg. Nowacki MP. Capecitabine is an oral fluoropyrimidine. Bevacizumab (Avastin) is a monoclonal antibody that blocks angiogenesis and is indicated for the treatment of patients with metastatic colorectal cancer. which generates fluorouracil peripherally in tumor tissue by one step of a three-step enzymatic cascade. and oxaliplatin. node-positive colon cancer. and is indicated for the treatment of metastatic colorectal cancer that expresses EGFR.Twelves C. A recent trial of capecitabine for adjuvant (postoperative) treatment of colon cancer randomly assigned patients to receive capecitabine or bolus fluorouracil plus leucovorin over 24 weeks. which is more active in tumor than in healthy tissues. FOLFOX. or Avastin.utdol. Available at: www. in combination with irinotecan (Camptosar) or in patients who do not respond to irinotecan. 2005 page 1141 . American College of Surgeons SESAP 13 Category 14 Item 35 Bevacizumab (A) Inhibits DNA synthesis (B) Anti-angiogenesis drug (C) Fluorouracil precursor (D) Anti-microtubule agent that blocks mitosis (E) Inhibits epidermal growth factor receptor page 1142 . A recent trial of capecitabine for adjuvant (postoperative) treatment of colon cancer randomly assigned patients to receive capecitabine or bolus fluorouracil plus leucovorin over 24 weeks. Bevacizumab (Avastin) is a monoclonal antibody that blocks angiogenesis and is indicated for the treatment of patients with metastatic colorectal cancer. Capecitabine had the same efficacy profile and perhaps a better safety profile than 5-FU. 5-FU. Abt M. in combination with irinotecan (Camptosar) or in patients who do not respond to irinotecan. 2005 page 1143 . Capecitabine is an oral fluoropyrimidine. It is often given in combination with conventional cytotoxic chemotherapy agents. or Avastin. FOLFOX.com 2. Capecitabine (Xeloda) is an oral fluorouracil (5-FU) pro-drug precursor. Nowacki MP. node-positive colon cancer. Wong A. Available at: www. The final stage of conversion to fluorouracil is catalyzed by thymidine phosphorylase.Twelves C. N Engl J Med 352:2696-2704.utdol. and is indicated for the treatment of metastatic colorectal cancer that expresses EGFR. Up To Date Website. eg. which is more active in tumor than in healthy tissues. and oxaliplatin. Cetuximab (Erbitux) is a monoclonal antibody that blocks the epidermal growth factor receptor (EGFR). REFERENCES 1. Leucovorin: Drug information. which generates fluorouracil peripherally in tumor tissue by one step of a three-step enzymatic cascade.American College of Surgeons SESAP 13 Critique for Category 14 Item 35 All the medications listed in these items have been used as chemotherapy in the treatment of stage III. leucovorin. et al: Capecitabine as adjuvant treatment for stage III colon cancer. American College of Surgeons SESAP 13 Category 14 Item 36 Cetuximab (A) Inhibits DNA synthesis (B) Anti-angiogenesis drug (C) Fluorouracil precursor (D) Anti-microtubule agent that blocks mitosis (E) Inhibits epidermal growth factor receptor page 1144 . Capecitabine had the same efficacy profile and perhaps a better safety profile than 5-FU. Capecitabine (Xeloda) is an oral fluorouracil (5-FU) pro-drug precursor. in combination with irinotecan (Camptosar) or in patients who do not respond to irinotecan. Nowacki MP. REFERENCES 1. and oxaliplatin. Wong A.com 2. node-positive colon cancer. et al: Capecitabine as adjuvant treatment for stage III colon cancer. Leucovorin: Drug information. which is more active in tumor than in healthy tissues. or Avastin. and is indicated for the treatment of metastatic colorectal cancer that expresses EGFR. A recent trial of capecitabine for adjuvant (postoperative) treatment of colon cancer randomly assigned patients to receive capecitabine or bolus fluorouracil plus leucovorin over 24 weeks. N Engl J Med 352:2696-2704.utdol. 2005 page 1145 . Cetuximab (Erbitux) is a monoclonal antibody that blocks the epidermal growth factor receptor (EGFR). Available at: www. FOLFOX. It is often given in combination with conventional cytotoxic chemotherapy agents. 5-FU. leucovorin. Bevacizumab (Avastin) is a monoclonal antibody that blocks angiogenesis and is indicated for the treatment of patients with metastatic colorectal cancer.American College of Surgeons SESAP 13 Critique for Category 14 Item 36 All the medications listed in these items have been used as chemotherapy in the treatment of stage III. The final stage of conversion to fluorouracil is catalyzed by thymidine phosphorylase. Up To Date Website.Twelves C. Abt M. Capecitabine is an oral fluoropyrimidine. eg. which generates fluorouracil peripherally in tumor tissue by one step of a three-step enzymatic cascade. American College of Surgeons SESAP 13 Category 14 Item 37 Neonatal presacral mass (A) Wilms’ tumor (B) Neuroblastoma (C) Rhabdomyosarcoma (D) Teratoma (E) Ewing’s sarcoma page 1146 . At least 13% to 30% of patients present with metastatic disease at diagnosis.Kalapurakal JA. Ewing’s sarcoma is a malignant neoplasm of the skeleton and soft tissue. It typically presents with a palpable abdominal mass and may involve tumor thrombus in the renal vein and inferior vena cava. Complete surgical excision with coccygectomy is essential to reduce local recurrence rates.Meyer WH. and the extremities. Spunt SL: Soft tissue sarcomas of childhood. most commonly occurring in the second decade of life. smaller size primary tumors. Colombani PM. Malogolowkin M.to 5year-old children. and minimal or no residual tumor following pre-resection chemotherapy have improved event-free survival. mature teratomas composed of elements of the three embryonic germ layers.Skinner MA: Germ cell tumors. REFERENCES 1. Tashjian DB. Neuroblastomas arises from the neural crest cells. Botryoid and spindle cell histology are considered favorable. 637-648 page 1147 . and biliary tract. and these tumors occur in the adrenal medulla and in the paravertebral sympathetic chain in the chest and abdomen. Breuer CK: Neuroblastoma update. 2004 4. younger age. in Oldham KT. These are typically benign. Neuroblastoma is the most common pediatric abdominal malignancy. Perlman EJ. 2005. Lippincott Williams & Wilkins. 2004 3. Most children present with regional or distant metastases at the time of diagnosis. Curr Opin Oncol 17:19-23. In children under 12 months of age.Henry MC. Wilms’ tumor or nephroblastoma is a malignant renal tumor that occurs most frequently in 1. Skinner MA (eds): Principles and Practice of Pediatric Surgery ed 2. Lancet Oncol 5:37-46. Dome JS. genitourinary tract. genitourinary. 2005 2. Smaller tumor size and absence of metastatic disease are also associated with a more favorable prognosis. Cancer Treat Review 30:269-280. while alveolar histology is generally associated with a worse prognosis. Favorable prognosis is associated with age under 10 years. neuroblastoma may undergo spontaneous regression. Foglia RP. Preoperative imaging studies such as computed tomographic (CT) scanning and duplex ultrasonography are useful in identifying tumor thrombus in the renal vein and/or inferior vena cava. et al: Management of Wilms’ tumour: current practice and future goals. and tumors of the orbit and nonparameningeal head and neck. Philadelphia. Patients with tumors involving the distal extremities. The most common sites for childhood rhabdomyosarcoma are the head and neck.American College of Surgeons SESAP 13 Critique for Category 14 Item 37 Sacrococcygeal teratoma is the most common neonatal solid tumor occurring in the presacral region. American College of Surgeons SESAP 13 Category 14 Item 38 Alveolar histology associated with worse prognosis (A) Wilms’ tumor (B) Neuroblastoma (C) Rhabdomyosarcoma (D) Teratoma (E) Ewing’s sarcoma page 1148 . Perlman EJ. mature teratomas composed of elements of the three embryonic germ layers. and the extremities. Favorable prognosis is associated with age under 10 years. Complete surgical excision with coccygectomy is essential to reduce local recurrence rates. Lancet Oncol 5:37-46. Curr Opin Oncol 17:19-23. and tumors of the orbit and nonparameningeal head and neck. At least 13% to 30% of patients present with metastatic disease at diagnosis. Cancer Treat Review 30:269-280. Most children present with regional or distant metastases at the time of diagnosis. most commonly occurring in the second decade of life. It typically presents with a palpable abdominal mass and may involve tumor thrombus in the renal vein and inferior vena cava. in Oldham KT.Kalapurakal JA. Colombani PM. younger age. and biliary tract. Neuroblastomas arises from the neural crest cells. Botryoid and spindle cell histology are considered favorable. 2004 4. Skinner MA (eds): Principles and Practice of Pediatric Surgery ed 2. Spunt SL: Soft tissue sarcomas of childhood. Wilms’ tumor or nephroblastoma is a malignant renal tumor that occurs most frequently in 1. Dome JS. The most common sites for childhood rhabdomyosarcoma are the head and neck. et al: Management of Wilms’ tumour: current practice and future goals. 637-648 page 1149 . In children under 12 months of age. Foglia RP. smaller size primary tumors. Preoperative imaging studies such as computed tomographic (CT) scanning and duplex ultrasonography are useful in identifying tumor thrombus in the renal vein and/or inferior vena cava. and these tumors occur in the adrenal medulla and in the paravertebral sympathetic chain in the chest and abdomen. Ewing’s sarcoma is a malignant neoplasm of the skeleton and soft tissue. Neuroblastoma is the most common pediatric abdominal malignancy. Patients with tumors involving the distal extremities. 2005. These are typically benign. REFERENCES 1.Meyer WH. Lippincott Williams & Wilkins.American College of Surgeons SESAP 13 Critique for Category 14 Item 38 Sacrococcygeal teratoma is the most common neonatal solid tumor occurring in the presacral region. while alveolar histology is generally associated with a worse prognosis.Skinner MA: Germ cell tumors.Henry MC. Tashjian DB.to 5year-old children. neuroblastoma may undergo spontaneous regression. Smaller tumor size and absence of metastatic disease are also associated with a more favorable prognosis. Philadelphia. genitourinary tract. Malogolowkin M. 2005 2. 2004 3. and minimal or no residual tumor following pre-resection chemotherapy have improved event-free survival. genitourinary. Breuer CK: Neuroblastoma update. American College of Surgeons SESAP 13 Category 14 Item 39 Most common pediatric abdominal malignancy (A) Wilms’ tumor (B) Neuroblastoma (C) Rhabdomyosarcoma (D) Teratoma (E) Ewing’s sarcoma page 1150 . Henry MC.Meyer WH. Lippincott Williams & Wilkins. Preoperative imaging studies such as computed tomographic (CT) scanning and duplex ultrasonography are useful in identifying tumor thrombus in the renal vein and/or inferior vena cava. neuroblastoma may undergo spontaneous regression. Neuroblastomas arises from the neural crest cells. while alveolar histology is generally associated with a worse prognosis. It typically presents with a palpable abdominal mass and may involve tumor thrombus in the renal vein and inferior vena cava. Dome JS. Tashjian DB. The most common sites for childhood rhabdomyosarcoma are the head and neck. 2005. in Oldham KT. Lancet Oncol 5:37-46. and biliary tract. These are typically benign. younger age. Colombani PM. REFERENCES 1. Cancer Treat Review 30:269-280. In children under 12 months of age. Neuroblastoma is the most common pediatric abdominal malignancy. At least 13% to 30% of patients present with metastatic disease at diagnosis. and tumors of the orbit and nonparameningeal head and neck. and minimal or no residual tumor following pre-resection chemotherapy have improved event-free survival. Philadelphia. Smaller tumor size and absence of metastatic disease are also associated with a more favorable prognosis.to 5year-old children. 2004 4. Patients with tumors involving the distal extremities. et al: Management of Wilms’ tumour: current practice and future goals. Spunt SL: Soft tissue sarcomas of childhood. mature teratomas composed of elements of the three embryonic germ layers. 637-648 page 1151 . 2004 3.Kalapurakal JA. and the extremities. most commonly occurring in the second decade of life. 2005 2. Foglia RP. Breuer CK: Neuroblastoma update. Malogolowkin M. Ewing’s sarcoma is a malignant neoplasm of the skeleton and soft tissue. Botryoid and spindle cell histology are considered favorable.American College of Surgeons SESAP 13 Critique for Category 14 Item 39 Sacrococcygeal teratoma is the most common neonatal solid tumor occurring in the presacral region. Complete surgical excision with coccygectomy is essential to reduce local recurrence rates. and these tumors occur in the adrenal medulla and in the paravertebral sympathetic chain in the chest and abdomen. Favorable prognosis is associated with age under 10 years. Perlman EJ. genitourinary tract. genitourinary. Skinner MA (eds): Principles and Practice of Pediatric Surgery ed 2. smaller size primary tumors. Most children present with regional or distant metastases at the time of diagnosis. Curr Opin Oncol 17:19-23. Wilms’ tumor or nephroblastoma is a malignant renal tumor that occurs most frequently in 1.Skinner MA: Germ cell tumors. and oxaliplatin (A) Gastrointestinal stromal tumor (GIST) (B) Colon adenocarcinoma (C) Breast invasive ductal carcinoma (D) Esophageal adenocarcinoma (E) Gastric mucosa associated lymphoid tissue (MALT) page 1152 . leucovorin.American College of Surgeons SESAP 13 Category 14 Item 40 Fluorouracil (5-FU). and oxaliplatin (FOLFOX). depending on stage. et al: Efficacy and safety of imatinib mesylate in advanced gastrointestinal stromal tumors. For stage I or II disease. Mucosa-associated lymphoid tissue (MALT) lesions are related to the presence of Helicobacter pylori. ampicillin. and oxaliplatin combinations in patients with previously untreated metastatic colorectal cancer. Sargent DJ. Clin Adv Hematol Oncol 3:254-256. Suekane H. patient age. however. Nonrandomized trials have also demonstrated its effectiveness in down-staging tumors. and paclitaxel in the adjuvant setting is considered standard therapy. presence of estrogen receptors. J Clin Oncol 22:23-30. Morton RF. Felici A. Chemotherapeutic choices for unresectable tumors or those with distant metastatic disease were limited and ineffectual. The most commonly used regimen is omeprazole. 2005 2. et al: Taxanes with anthracyclines as first-line chemotherapy for metastatic breast carcinoma. and HER2/neu status.American College of Surgeons SESAP 13 Critique for Category 14 Item 40 Gastrointestinal stromal tumors (GIST) are treated with surgical resection. Cancer 104:532-540. REFERENCES 1.Goldberg RM. von Mehren M. Fuchs CS.Bria E. Peters WP. 2004 5. A variety of chemotherapeutic options are available for invasive ductal carcinoma of the breast. 2002 3. irinotecan. 2005 4. cyclophosphamide. For patients with node-positive disease.Nakamura S. these agents are not used in an adjuvant setting outside of a clinical trial. H pylori is also associated with an increased risk of gastric adenocarcinoma and gastric lymphoma. An American College of Surgeons trial is presently evaluating imatinib as adjuvant therapy after resection of GIST. et al: Long-term clinical outcome of Helicobacter pylori eradication for gastric mucosa-associated lymphoid tissue lymphoma with a reference to second-line treatment. In the adjuvant setting. Cancer 103:672-679. Randomized trials have shown that the tyrosine kinase inhibitor imatinib (Gleevec) is effective in metastatic and unresectable disease. It is also used for node-negative patients with ERpositive tumors larger than 2 cm and ER-negative tumors larger than 1 cm. Matsumoto T. Giannarelli D. Van den Abbeele AD. Nakamura S. this regimen has not been shown to produce a survival advantage.Forastiere A: The role of chemotherapy in the treatment of esophageal cancer. leucoverin. 2005 page 1153 . et al: A randomized controlled trial of fluorouracil plus leucovorin. Blanke CD. although some oncologists will consider their use in high-risk stage II disease (cancer associated with perforation or obstruction).Demetri GD. Randomized controlled trials have demonstrated a survival advantage for patients with stage III colon adenocarcinoma treated with a regimen of fluorouracial (5-FU). the combination of doxorubin. Eradication of the organism will lead to reversal of the MALT lesions. N Engl J Med 347:472-480. Cisplatin and 5-FU are used for treatment of esophageal adenocarcinoma in both the neoadjuvant and palliative setting. and clarithromycin. paclitaxel (A) Gastrointestinal stromal tumor (GIST) (B) Colon adenocarcinoma (C) Breast invasive ductal carcinoma (D) Esophageal adenocarcinoma (E) Gastric mucosa associated lymphoid tissue (MALT) page 1154 .American College of Surgeons SESAP 13 Category 14 Item 41 Doxorubin. cyclophosphamide. and oxaliplatin (FOLFOX). Felici A. N Engl J Med 347:472-480. 2005 2. et al: Efficacy and safety of imatinib mesylate in advanced gastrointestinal stromal tumors. these agents are not used in an adjuvant setting outside of a clinical trial. 2004 5. Matsumoto T. 2005 4.Nakamura S.Forastiere A: The role of chemotherapy in the treatment of esophageal cancer. In the adjuvant setting. Peters WP. Eradication of the organism will lead to reversal of the MALT lesions. Sargent DJ. For patients with node-positive disease. Nonrandomized trials have also demonstrated its effectiveness in down-staging tumors.Goldberg RM. 2002 3. Fuchs CS. 2005 page 1155 . this regimen has not been shown to produce a survival advantage. The most commonly used regimen is omeprazole. Nakamura S. leucoverin. presence of estrogen receptors. Mucosa-associated lymphoid tissue (MALT) lesions are related to the presence of Helicobacter pylori. Randomized trials have shown that the tyrosine kinase inhibitor imatinib (Gleevec) is effective in metastatic and unresectable disease. patient age. Van den Abbeele AD. ampicillin. Suekane H. Morton RF.American College of Surgeons SESAP 13 Critique of Category 14 Item 41 Gastrointestinal stromal tumors (GIST) are treated with surgical resection. von Mehren M. A variety of chemotherapeutic options are available for invasive ductal carcinoma of the breast. et al: Taxanes with anthracyclines as first-line chemotherapy for metastatic breast carcinoma. depending on stage. J Clin Oncol 22:23-30. and paclitaxel in the adjuvant setting is considered standard therapy. however. Cancer 104:532-540. It is also used for node-negative patients with ERpositive tumors larger than 2 cm and ER-negative tumors larger than 1 cm.Demetri GD. For stage I or II disease. Giannarelli D. Cisplatin and 5-FU are used for treatment of esophageal adenocarcinoma in both the neoadjuvant and palliative setting. An American College of Surgeons trial is presently evaluating imatinib as adjuvant therapy after resection of GIST.Bria E. Blanke CD. et al: A randomized controlled trial of fluorouracil plus leucovorin. Cancer 103:672-679. although some oncologists will consider their use in high-risk stage II disease (cancer associated with perforation or obstruction). cyclophosphamide. and HER2/neu status. irinotecan. et al: Long-term clinical outcome of Helicobacter pylori eradication for gastric mucosa-associated lymphoid tissue lymphoma with a reference to second-line treatment. and oxaliplatin combinations in patients with previously untreated metastatic colorectal cancer. Clin Adv Hematol Oncol 3:254-256. H pylori is also associated with an increased risk of gastric adenocarcinoma and gastric lymphoma. Randomized controlled trials have demonstrated a survival advantage for patients with stage III colon adenocarcinoma treated with a regimen of fluorouracial (5-FU). and clarithromycin. the combination of doxorubin. REFERENCES 1. Chemotherapeutic choices for unresectable tumors or those with distant metastatic disease were limited and ineffectual. ampicillin. clarithromycin (A) Gastrointestinal stromal tumor (GIST) (B) Colon adenocarcinoma (C) Breast invasive ductal carcinoma (D) Esophageal adenocarcinoma (E) Gastric mucosa associated lymphoid tissue (MALT) page 1156 .American College of Surgeons SESAP 13 Category 14 Item 42 Omeprazole. REFERENCES 1. this regimen has not been shown to produce a survival advantage. The most commonly used regimen is omeprazole. 2005 4. et al: Efficacy and safety of imatinib mesylate in advanced gastrointestinal stromal tumors. et al: A randomized controlled trial of fluorouracil plus leucovorin. It is also used for node-negative patients with ERpositive tumors larger than 2 cm and ER-negative tumors larger than 1 cm. et al: Long-term clinical outcome of Helicobacter pylori eradication for gastric mucosa-associated lymphoid tissue lymphoma with a reference to second-line treatment. Randomized controlled trials have demonstrated a survival advantage for patients with stage III colon adenocarcinoma treated with a regimen of fluorouracial (5-FU). von Mehren M. In the adjuvant setting. although some oncologists will consider their use in high-risk stage II disease (cancer associated with perforation or obstruction). ampicillin. J Clin Oncol 22:23-30.Goldberg RM. however. For patients with node-positive disease.Bria E. and oxaliplatin (FOLFOX). Van den Abbeele AD. 2002 3. Randomized trials have shown that the tyrosine kinase inhibitor imatinib (Gleevec) is effective in metastatic and unresectable disease.American College of Surgeons SESAP 13 Critique for Category 14 Item 42 Gastrointestinal stromal tumors (GIST) are treated with surgical resection. these agents are not used in an adjuvant setting outside of a clinical trial. and paclitaxel in the adjuvant setting is considered standard therapy. and clarithromycin. and oxaliplatin combinations in patients with previously untreated metastatic colorectal cancer. Nakamura S. H pylori is also associated with an increased risk of gastric adenocarcinoma and gastric lymphoma. 2005 page 1157 . 2005 2.Forastiere A: The role of chemotherapy in the treatment of esophageal cancer. leucoverin. et al: Taxanes with anthracyclines as first-line chemotherapy for metastatic breast carcinoma. Cisplatin and 5-FU are used for treatment of esophageal adenocarcinoma in both the neoadjuvant and palliative setting. Suekane H. N Engl J Med 347:472-480. patient age. Cancer 104:532-540. irinotecan. the combination of doxorubin. cyclophosphamide. and HER2/neu status.Demetri GD. Fuchs CS. Matsumoto T. Felici A. Cancer 103:672-679. depending on stage. Nonrandomized trials have also demonstrated its effectiveness in down-staging tumors. Sargent DJ. Blanke CD.Nakamura S. A variety of chemotherapeutic options are available for invasive ductal carcinoma of the breast. Eradication of the organism will lead to reversal of the MALT lesions. Chemotherapeutic choices for unresectable tumors or those with distant metastatic disease were limited and ineffectual. For stage I or II disease. 2004 5. Clin Adv Hematol Oncol 3:254-256. presence of estrogen receptors. An American College of Surgeons trial is presently evaluating imatinib as adjuvant therapy after resection of GIST. Peters WP. Morton RF. Giannarelli D. Mucosa-associated lymphoid tissue (MALT) lesions are related to the presence of Helicobacter pylori. American College of Surgeons SESAP 13 Category 14 Item 43 PTEN gene (A) Hereditary nonpolyposis colorectal cancer (HNPCC) (B) Familial adenomatous polyposis coli (FAP) (C) Peutz-Jeghers syndrome (D) Juvenile polyposis (E) Cowden syndrome page 1158 . Hereditary nonpolyposis colorectal cancer (HNPCC. Cowden syndrome features benign hamartomatous polyps of the skin and gastrointestinal tract. bile duct. FAP patients present in their 20s or 30s with hundreds to thousands of colonic polyps. may accompany the syndrome. Mutch MG: The genetic of other polyposis syndromes. page 1159 . 2004 2. the incidence of ovarian. formerly known as Lynch syndrome) is the most common type of hereditary colorectal cancer. but no identifiable pattern emerges in the rest of cases. and occur in younger patients (mean age at diagnosis is 46). A variety of extra-intestinal manifestations. which encodes for a phosphatase protein involved in apoptosis and the cell cycle. There are many confounding implications of obtaining and acting on genetic testing results. As knowledge about the impact of genetic mutations progresses. benign and malignant. brain. It is inherited in an autosomal dominant fashion in 20% to 50% of cases. and varies from 9% to 50%. Surgical intervention is recommended soon after diagnosis to prevent colon cancer. It is due to a mutation in the PTEN gene. or bowel obstruction due to intussusception. urinary tract. PJS patients develop polyps through their entire gastrointestinal tract. If left untreated. Juvenile polyposis is a rare heterogeneous syndrome characterized by multiple hamartomatous colonic polyps.Hunt S.American College of Surgeons SESAP 13 Critique for Category 14 Item 43 Genetics are increasingly useful for understanding the development of many benign and malignant conditions.3 and encodes for the serine-threonine kinase enzyme. Peutz-Jeghers syndrome (PJS) is a rare autosomal dominant disease caused by a mutation on the STK11 gene. clinicians must obtain a complete family history. gastric.Muldoon RL. The penetrance is almost 100%. which is located on chromosome 19p3. underscoring the importance of obtaining a complete family history and screening for associated malignancies. macrocephaly. The incidence of extra-intestinal cancers is increased in HNPCC. renal. and ethical perspective. The risk for gastrointestinal and extraintestinal malignancies is increased. Mutations in the SMAD4 and BMPR1A genes (both involved in TGF-β signaling pathways) have been identified in some kindreds. Patients and their families should understand the implications of harboring genetic mutations from a medical. Familial adenomatous polyposis coli (FAP) is an inherited autosomal dominant disease caused by a mutation on the adenomatous polyposis coli (APC) gene located on chromosome 5q21. Lowney JK: Familial adenomatous polyposis. The risk for gastrointestinal malignancy is increased. Semin Colon Rectal Surg 15:147-157. Any suggestive pattern should be the trigger for genetic counseling referral. Mucocutaneous lesions. essentially 100% of FAP patients will develop colon cancer by age 40. REFERENCES 1. Symptoms usually are due to occult bleeding. and thyroid abnormalities are frequently seen in Cowden syndrome. hMLH1. Colon cancers in HNPCC are more often right-sided (60% to 70%). The mutation may be identified in up to 70% of PJS patients. frequently synchronous (5% to10%). with a special predilection for the small bowel. HNPCC patients have a mutation in one or several of their mismatch repair genes: hMSH2. societal. and small bowel cancer is also significantly increased. and hMSH6 are the most frequently affected genes. The penetrance for this autosomal dominant disease is above 80%. Female HNPCC patients have a 60% risk of developing endometrial cancer. Semin Colon Rectal Surg 15:158-162. 2004 page 1160 . Semin Colon Rectal Surg 15:136146. Mutch MG: Hereditary nonpolyposis colorectal cancer.Wise PE.American College of Surgeons SESAP 13 2004 3. American College of Surgeons SESAP 13 Category 14 Item 44 hMSH2 gene (A) Hereditary nonpolyposis colorectal cancer (HNPCC) (B) Familial adenomatous polyposis coli (FAP) (C) Peutz-Jeghers syndrome (D) Juvenile polyposis (E) Cowden syndrome page 1161 . American College of Surgeons SESAP 13 Critique for Category 14 Item 44 Genetics are increasingly useful for understanding the development of many benign and malignant conditions. gastric. The penetrance is almost 100%. Symptoms usually are due to occult bleeding. or bowel obstruction due to intussusception. The mutation may be identified in up to 70% of PJS patients. may accompany the syndrome. renal. Lowney JK: Familial adenomatous polyposis. hMLH1. HNPCC patients have a mutation in one or several of their mismatch repair genes: hMSH2. which is located on chromosome 19p3. Mucocutaneous lesions. 2004 2. There are many confounding implications of obtaining and acting on genetic testing results. with a special predilection for the small bowel.Muldoon RL. and small bowel cancer is also significantly increased. brain. Semin Colon Rectal Surg 15:147-157. Hereditary nonpolyposis colorectal cancer (HNPCC. and ethical perspective. The risk for gastrointestinal and extraintestinal malignancies is increased. As knowledge about the impact of genetic mutations progresses.3 and encodes for the serine-threonine kinase enzyme. essentially 100% of FAP patients will develop colon cancer by age 40. Patients and their families should understand the implications of harboring genetic mutations from a medical. urinary tract. Female HNPCC patients have a 60% risk of developing endometrial cancer.Hunt S. and hMSH6 are the most frequently affected genes. Peutz-Jeghers syndrome (PJS) is a rare autosomal dominant disease caused by a mutation on the STK11 gene. FAP patients present in their 20s or 30s with hundreds to thousands of colonic polyps. The incidence of extra-intestinal cancers is increased in HNPCC. societal. page 1162 . which encodes for a phosphatase protein involved in apoptosis and the cell cycle. A variety of extra-intestinal manifestations. frequently synchronous (5% to10%). underscoring the importance of obtaining a complete family history and screening for associated malignancies. It is due to a mutation in the PTEN gene. benign and malignant. The risk for gastrointestinal malignancy is increased. Juvenile polyposis is a rare heterogeneous syndrome characterized by multiple hamartomatous colonic polyps. It is inherited in an autosomal dominant fashion in 20% to 50% of cases. bile duct. the incidence of ovarian. and thyroid abnormalities are frequently seen in Cowden syndrome. Mutch MG: The genetic of other polyposis syndromes. If left untreated. Semin Colon Rectal Surg 15:158-162. macrocephaly. The penetrance for this autosomal dominant disease is above 80%. formerly known as Lynch syndrome) is the most common type of hereditary colorectal cancer. Any suggestive pattern should be the trigger for genetic counseling referral. Surgical intervention is recommended soon after diagnosis to prevent colon cancer. Familial adenomatous polyposis coli (FAP) is an inherited autosomal dominant disease caused by a mutation on the adenomatous polyposis coli (APC) gene located on chromosome 5q21. REFERENCES 1. clinicians must obtain a complete family history. Mutations in the SMAD4 and BMPR1A genes (both involved in TGF-β signaling pathways) have been identified in some kindreds. and varies from 9% to 50%. but no identifiable pattern emerges in the rest of cases. PJS patients develop polyps through their entire gastrointestinal tract. Colon cancers in HNPCC are more often right-sided (60% to 70%). and occur in younger patients (mean age at diagnosis is 46). Cowden syndrome features benign hamartomatous polyps of the skin and gastrointestinal tract. American College of Surgeons SESAP 13 2004 3.Wise PE. Mutch MG: Hereditary nonpolyposis colorectal cancer. 2004 page 1163 . Semin Colon Rectal Surg 15:136146. American College of Surgeons SESAP 13 Category 14 Item 45 APC gene (A) Hereditary nonpolyposis colorectal cancer (HNPCC) (B) Familial adenomatous polyposis coli (FAP) (C) Peutz-Jeghers syndrome (D) Juvenile polyposis (E) Cowden syndrome page 1164 . Surgical intervention is recommended soon after diagnosis to prevent colon cancer. REFERENCES 1. Any suggestive pattern should be the trigger for genetic counseling referral. It is inherited in an autosomal dominant fashion in 20% to 50% of cases. Semin Colon Rectal Surg 15:158-162.3 and encodes for the serine-threonine kinase enzyme. and occur in younger patients (mean age at diagnosis is 46). Symptoms usually are due to occult bleeding. Mutch MG: The genetic of other polyposis syndromes. clinicians must obtain a complete family history. The penetrance for this autosomal dominant disease is above 80%. and varies from 9% to 50%. 2004 2. Mutations in the SMAD4 and BMPR1A genes (both involved in TGF-β signaling pathways) have been identified in some kindreds. Cowden syndrome features benign hamartomatous polyps of the skin and gastrointestinal tract. Female HNPCC patients have a 60% risk of developing endometrial cancer.Muldoon RL. It is due to a mutation in the PTEN gene. and small bowel cancer is also significantly increased. which is located on chromosome 19p3. If left untreated. FAP patients present in their 20s or 30s with hundreds to thousands of colonic polyps. The penetrance is almost 100%. brain. macrocephaly. formerly known as Lynch syndrome) is the most common type of hereditary colorectal cancer. Semin Colon Rectal Surg 15:147-157. and thyroid abnormalities are frequently seen in Cowden syndrome. Mucocutaneous lesions. and hMSH6 are the most frequently affected genes. Juvenile polyposis is a rare heterogeneous syndrome characterized by multiple hamartomatous colonic polyps. societal. HNPCC patients have a mutation in one or several of their mismatch repair genes: hMSH2. The incidence of extra-intestinal cancers is increased in HNPCC. Colon cancers in HNPCC are more often right-sided (60% to 70%). The mutation may be identified in up to 70% of PJS patients. underscoring the importance of obtaining a complete family history and screening for associated malignancies. may accompany the syndrome. Lowney JK: Familial adenomatous polyposis. urinary tract. essentially 100% of FAP patients will develop colon cancer by age 40. A variety of extra-intestinal manifestations. or bowel obstruction due to intussusception. The risk for gastrointestinal malignancy is increased. PJS patients develop polyps through their entire gastrointestinal tract. Peutz-Jeghers syndrome (PJS) is a rare autosomal dominant disease caused by a mutation on the STK11 gene. frequently synchronous (5% to10%). benign and malignant. the incidence of ovarian. bile duct. There are many confounding implications of obtaining and acting on genetic testing results. Hereditary nonpolyposis colorectal cancer (HNPCC. Patients and their families should understand the implications of harboring genetic mutations from a medical.American College of Surgeons SESAP 13 Critique for Category 14 Item 45 Genetics are increasingly useful for understanding the development of many benign and malignant conditions. hMLH1. The risk for gastrointestinal and extraintestinal malignancies is increased. with a special predilection for the small bowel. and ethical perspective. Familial adenomatous polyposis coli (FAP) is an inherited autosomal dominant disease caused by a mutation on the adenomatous polyposis coli (APC) gene located on chromosome 5q21. which encodes for a phosphatase protein involved in apoptosis and the cell cycle. As knowledge about the impact of genetic mutations progresses. renal. page 1165 . gastric.Hunt S. but no identifiable pattern emerges in the rest of cases. Wise PE. Mutch MG: Hereditary nonpolyposis colorectal cancer. 2004 page 1166 .American College of Surgeons SESAP 13 2004 3. Semin Colon Rectal Surg 15:136146. American College of Surgeons SESAP 13 Category 14 Item 46 Require(s) tissue diagnosis prior to operation (A) Figure A (B) Figure B (C) Both (D) Neither page 1167 . Philadelphia. REFERENCES 1. Generous resection of the extrahepatic biliary tree is required to ensure margin-free as well as tension-free anastomosis. A dilated intrahepatic biliary tree with a normal or collapsed gallbladder and extrahepatic biliary tree will be seen with a perihilar cholangiocarcinoma. which alone or in combination should give a high degree of suspicion for malignancy. the inability to obtain a tissue diagnosis should neither delay operation nor allow the surgeon to think that there is any other diagnosis. when imaging strongly suggests malignancy. ed 17. 2000 page 1168 . Chemotherapy does not improve survival either after resection or in the case of palliation for unresected cholangiocarcinoma. 2004. and margin-free resection is extremely important if improved survival is to be realized. in Townsend CM Jr (ed): Sabiston Textbook of Surgery: The Biological Basis of Modern Surgical Practice.American College of Surgeons SESAP 13 Critique for Category 14 Item 46 Cholangiocarcinoma of the extrahepatic biliary tract is an extremely rare but deadly tumor. Even those lying within the middle portions of the extrahepatic biliary tree will require Roux-en-Y hepaticojejunal anastomosis. The proximal or perihilar cholangiocarcinoma (image A. Distal cholangiocarcinomas (image B. Koike N. Initial imaging studies should include abdominal ultrasonography and computed tomographic (CT) scanning.Ahrendt SA. However. Tumors arising in the distal biliary tree will demonstrate significant dilatation of the gallbladder and both the intrahepatic and extrahepatic biliary tree. Neither distal nor proximal cholangiocarcinomas can be effectively treated with end-to-end ductal anastomosis. Much has been said about fine-needle aspiration. not including ampullary tumors) occur near or within the intrapancreatic portion of the bile duct. pp 1634-1638 2. also known as Klatskin tumor) can present an extremely difficult surgical challenge. Takahashi H. Kawamoto T. further evaluation with endoscopic retrograde cholangiopancreatography (ERCP) or MRCP. ERCP washings or brushings of the intraluminal lesion. Distal cholangiocarcinoma requires a pancreaticoduodenectomy (Whipple procedure) for the most effective results.Todoroki T. Most commonly it occurs at the hepatic duct bifurcation (approximately 70%). et al: Radical resection of hilar bile duct carcinoma and predictors of survival. or ductal lavage to obtain a tissue diagnosis preoperatively. The next step for diagnosis should include a thorough evaluation of the CT scan and if needed. Most patients have jaundice at the time of presentation. Perihilar or proximal cholangiocarcinoma most often requires a major hepatic lobar resection. It may occur anywhere along the intrahepatic or extrahepatic biliary tract. Br J Surg 87:306-313. Saunders. Pitt HA: Biliary tract. American College of Surgeons SESAP 13 Category 14 Item 47 May require pancreaticoduodenectomy (A) Figure A (B) Figure B (C) Both (D) Neither page 1169 . However. Even those lying within the middle portions of the extrahepatic biliary tree will require Roux-en-Y hepaticojejunal anastomosis. ed 17. further evaluation with endoscopic retrograde cholangiopancreatography (ERCP) or MRCP. Kawamoto T. or ductal lavage to obtain a tissue diagnosis preoperatively. Philadelphia. Perihilar or proximal cholangiocarcinoma most often requires a major hepatic lobar resection. pp 1634-1638 2. Pitt HA: Biliary tract. Much has been said about fine-needle aspiration.Ahrendt SA. Tumors arising in the distal biliary tree will demonstrate significant dilatation of the gallbladder and both the intrahepatic and extrahepatic biliary tree. when imaging strongly suggests malignancy. REFERENCES 1. Takahashi H. ERCP washings or brushings of the intraluminal lesion. Generous resection of the extrahepatic biliary tree is required to ensure margin-free as well as tension-free anastomosis. Most patients have jaundice at the time of presentation. 2004. Chemotherapy does not improve survival either after resection or in the case of palliation for unresected cholangiocarcinoma. Neither distal nor proximal cholangiocarcinomas can be effectively treated with end-to-end ductal anastomosis. Distal cholangiocarcinoma requires a pancreaticoduodenectomy (Whipple procedure) for the most effective results. A dilated intrahepatic biliary tree with a normal or collapsed gallbladder and extrahepatic biliary tree will be seen with a perihilar cholangiocarcinoma. in Townsend CM Jr (ed): Sabiston Textbook of Surgery: The Biological Basis of Modern Surgical Practice.Todoroki T. et al: Radical resection of hilar bile duct carcinoma and predictors of survival. also known as Klatskin tumor) can present an extremely difficult surgical challenge. It may occur anywhere along the intrahepatic or extrahepatic biliary tract. Saunders. Most commonly it occurs at the hepatic duct bifurcation (approximately 70%). The next step for diagnosis should include a thorough evaluation of the CT scan and if needed. Br J Surg 87:306-313. 2000 page 1170 . Koike N. Distal cholangiocarcinomas (image B. Initial imaging studies should include abdominal ultrasonography and computed tomographic (CT) scanning.American College of Surgeons SESAP 13 Critique for Category 14 Item 47 Cholangiocarcinoma of the extrahepatic biliary tract is an extremely rare but deadly tumor. and margin-free resection is extremely important if improved survival is to be realized. which alone or in combination should give a high degree of suspicion for malignancy. not including ampullary tumors) occur near or within the intrapancreatic portion of the bile duct. The proximal or perihilar cholangiocarcinoma (image A. the inability to obtain a tissue diagnosis should neither delay operation nor allow the surgeon to think that there is any other diagnosis. American College of Surgeons SESAP 13 Category 14 Item 48 May require major lobar hepatic resection (A) Figure A (B) Figure B (C) Both (D) Neither page 1171 . et al: Radical resection of hilar bile duct carcinoma and predictors of survival. Koike N. also known as Klatskin tumor) can present an extremely difficult surgical challenge. Generous resection of the extrahepatic biliary tree is required to ensure margin-free as well as tension-free anastomosis. Kawamoto T. Tumors arising in the distal biliary tree will demonstrate significant dilatation of the gallbladder and both the intrahepatic and extrahepatic biliary tree. Philadelphia. Much has been said about fine-needle aspiration. ed 17. Even those lying within the middle portions of the extrahepatic biliary tree will require Roux-en-Y hepaticojejunal anastomosis. ERCP washings or brushings of the intraluminal lesion. in Townsend CM Jr (ed): Sabiston Textbook of Surgery: The Biological Basis of Modern Surgical Practice. Initial imaging studies should include abdominal ultrasonography and computed tomographic (CT) scanning. Br J Surg 87:306-313. the inability to obtain a tissue diagnosis should neither delay operation nor allow the surgeon to think that there is any other diagnosis. or ductal lavage to obtain a tissue diagnosis preoperatively. Distal cholangiocarcinoma requires a pancreaticoduodenectomy (Whipple procedure) for the most effective results. pp 1634-1638 2. when imaging strongly suggests malignancy.Todoroki T. Distal cholangiocarcinomas (image B. and margin-free resection is extremely important if improved survival is to be realized. Perihilar or proximal cholangiocarcinoma most often requires a major hepatic lobar resection. A dilated intrahepatic biliary tree with a normal or collapsed gallbladder and extrahepatic biliary tree will be seen with a perihilar cholangiocarcinoma. not including ampullary tumors) occur near or within the intrapancreatic portion of the bile duct. However. Takahashi H. 2000 page 1172 .Ahrendt SA. The next step for diagnosis should include a thorough evaluation of the CT scan and if needed. which alone or in combination should give a high degree of suspicion for malignancy. Most commonly it occurs at the hepatic duct bifurcation (approximately 70%). Saunders. Chemotherapy does not improve survival either after resection or in the case of palliation for unresected cholangiocarcinoma. Most patients have jaundice at the time of presentation. 2004. It may occur anywhere along the intrahepatic or extrahepatic biliary tract. further evaluation with endoscopic retrograde cholangiopancreatography (ERCP) or MRCP. Pitt HA: Biliary tract. REFERENCES 1. The proximal or perihilar cholangiocarcinoma (image A.American College of Surgeons SESAP 13 Critique for Category 14 Item 48 Cholangiocarcinoma of the extrahepatic biliary tract is an extremely rare but deadly tumor. Neither distal nor proximal cholangiocarcinomas can be effectively treated with end-to-end ductal anastomosis. American College of Surgeons SESAP 13 Category 14 Item 49 Can be excised effectively with end-to-end biliary tract anastomosis if margins are negative (A) Figure A (B) Figure B (C) Both (D) Neither page 1173 . 2004. Koike N. ed 17. the inability to obtain a tissue diagnosis should neither delay operation nor allow the surgeon to think that there is any other diagnosis. Distal cholangiocarcinomas (image B. Takahashi H. The proximal or perihilar cholangiocarcinoma (image A. and margin-free resection is extremely important if improved survival is to be realized. Br J Surg 87:306-313. REFERENCES 1. Much has been said about fine-needle aspiration. or ductal lavage to obtain a tissue diagnosis preoperatively.Todoroki T. Saunders.American College of Surgeons SESAP 13 Critique for Category 14 Item 49 Cholangiocarcinoma of the extrahepatic biliary tract is an extremely rare but deadly tumor. However. Most commonly it occurs at the hepatic duct bifurcation (approximately 70%). Most patients have jaundice at the time of presentation. 2000 page 1174 . Neither distal nor proximal cholangiocarcinomas can be effectively treated with end-to-end ductal anastomosis. Kawamoto T. Perihilar or proximal cholangiocarcinoma most often requires a major hepatic lobar resection. which alone or in combination should give a high degree of suspicion for malignancy. when imaging strongly suggests malignancy. ERCP washings or brushings of the intraluminal lesion. further evaluation with endoscopic retrograde cholangiopancreatography (ERCP) or MRCP.Ahrendt SA. pp 1634-1638 2. in Townsend CM Jr (ed): Sabiston Textbook of Surgery: The Biological Basis of Modern Surgical Practice. Generous resection of the extrahepatic biliary tree is required to ensure margin-free as well as tension-free anastomosis. It may occur anywhere along the intrahepatic or extrahepatic biliary tract. et al: Radical resection of hilar bile duct carcinoma and predictors of survival. Even those lying within the middle portions of the extrahepatic biliary tree will require Roux-en-Y hepaticojejunal anastomosis. Tumors arising in the distal biliary tree will demonstrate significant dilatation of the gallbladder and both the intrahepatic and extrahepatic biliary tree. Initial imaging studies should include abdominal ultrasonography and computed tomographic (CT) scanning. A dilated intrahepatic biliary tree with a normal or collapsed gallbladder and extrahepatic biliary tree will be seen with a perihilar cholangiocarcinoma. also known as Klatskin tumor) can present an extremely difficult surgical challenge. not including ampullary tumors) occur near or within the intrapancreatic portion of the bile duct. Pitt HA: Biliary tract. The next step for diagnosis should include a thorough evaluation of the CT scan and if needed. Chemotherapy does not improve survival either after resection or in the case of palliation for unresected cholangiocarcinoma. Philadelphia. Distal cholangiocarcinoma requires a pancreaticoduodenectomy (Whipple procedure) for the most effective results. American College of Surgeons SESAP 13 Category 14 Item 50 Operation highly effective for stage I disease (A) Mesothelioma of the thorax (B) Bronchogenic carcinoma (C) Both (D) Neither page 1175 . and myasthenia-like myopathies are frequent complications of bronchogenic carcinoma. Luketich JD: Chest wall. but are rarely seen in mesothelioma. 2005 page 1176 . mesothelioma of the pleura has increased in incidence in the developed and emerging countries of the world because of increased exposure to airborne pollutants. mediastinum. Up to 74% of patients with mesothelioma present with radiographic evidence of pleural effusion compared with 5% to 15% of those with bronchogenic carcinoma. Both mesothelioma and bronchogenic carcinoma can cause constrictive pericarditis. it does not significantly improve survival. peripheral and central neuropathies. lung. carcinoid syndrome. Surgical resection is usually futile and treatment is usually directed towards palliation. stage I bronchogenic carcinoma has a 59% 5-year survival rate. et al (eds): Schwartz’s Principles of Surgery. Although radical surgical therapy consisting of extrapleural pneumonectomy or radical pleurectomy has been attempted. However. Lake RA: Advances in malignant mesothelioma. Five-year survival for mesothelioma is virtually zero compared with 13% for patients with all stages of bronchogenic carcinoma because of the high percentage of patients presenting with late-stage disease. Dunn DL. ed 8. Pericardiectomy can be used as a palliative intervention. and pleura. the most prominent of which is pain. Like bronchogenic carcinoma. McGraw-Hill.Maddaus MA. hypercalcemia. They are probably caused by hormone-like substances elaborated in the primary tumor REFERENCES 1. Andersen DK. particularly the pleura and peritoneum but occasionally also the pericardium and the tunica vaginalis. Billiar TR.American College of Surgeons SESAP 13 Critique for Category 14 Item 50 Mesothelioma affects serosal surfaces of the body cavity. Paraneoplastic syndromes such as Cushing’s syndrome. inappropriate ADH. pp 545-610 2. particularly asbestos. N Engl J Med 353:1591-1603. Mesothelioma usually progresses silently and presents as late stage disease 2 to 3 months after the onset of symptoms. usually by direct extension of the tumor. in Bruincardi FC.Robinson BW. New York. American College of Surgeons SESAP 13 Category 14 Item 51 Constrictive pericarditis often seen in advanced disease (A) Mesothelioma of the thorax (B) Bronchogenic carcinoma (C) Both (D) Neither page 1177 . Surgical resection is usually futile and treatment is usually directed towards palliation. mesothelioma of the pleura has increased in incidence in the developed and emerging countries of the world because of increased exposure to airborne pollutants. Five-year survival for mesothelioma is virtually zero compared with 13% for patients with all stages of bronchogenic carcinoma because of the high percentage of patients presenting with late-stage disease. Lake RA: Advances in malignant mesothelioma. Like bronchogenic carcinoma. Pericardiectomy can be used as a palliative intervention.American College of Surgeons SESAP 13 Critique for Category 14 Item 51 Mesothelioma affects serosal surfaces of the body cavity. McGraw-Hill.Robinson BW. lung. but are rarely seen in mesothelioma. stage I bronchogenic carcinoma has a 59% 5-year survival rate. pp 545-610 2. usually by direct extension of the tumor. the most prominent of which is pain. Both mesothelioma and bronchogenic carcinoma can cause constrictive pericarditis. Mesothelioma usually progresses silently and presents as late stage disease 2 to 3 months after the onset of symptoms. However. peripheral and central neuropathies. N Engl J Med 353:1591-1603. Although radical surgical therapy consisting of extrapleural pneumonectomy or radical pleurectomy has been attempted. and myasthenia-like myopathies are frequent complications of bronchogenic carcinoma. it does not significantly improve survival. New York. and pleura. hypercalcemia. Up to 74% of patients with mesothelioma present with radiographic evidence of pleural effusion compared with 5% to 15% of those with bronchogenic carcinoma. mediastinum. They are probably caused by hormone-like substances elaborated in the primary tumor REFERENCES 1. Dunn DL. inappropriate ADH. et al (eds): Schwartz’s Principles of Surgery. Paraneoplastic syndromes such as Cushing’s syndrome. ed 8. particularly asbestos. in Bruincardi FC. 2005 page 1178 . particularly the pleura and peritoneum but occasionally also the pericardium and the tunica vaginalis. carcinoid syndrome. Luketich JD: Chest wall.Maddaus MA. Billiar TR. Andersen DK. American College of Surgeons SESAP 13 Category 14 Item 52 Pleural effusion occurs in > 30% of presenting patients (A) Mesothelioma of the thorax (B) Bronchogenic carcinoma (C) Both (D) Neither page 1179 . stage I bronchogenic carcinoma has a 59% 5-year survival rate. mediastinum. but are rarely seen in mesothelioma. Like bronchogenic carcinoma. Paraneoplastic syndromes such as Cushing’s syndrome. Mesothelioma usually progresses silently and presents as late stage disease 2 to 3 months after the onset of symptoms. particularly the pleura and peritoneum but occasionally also the pericardium and the tunica vaginalis. et al (eds): Schwartz’s Principles of Surgery.Maddaus MA. Pericardiectomy can be used as a palliative intervention. hypercalcemia. carcinoid syndrome. inappropriate ADH. Surgical resection is usually futile and treatment is usually directed towards palliation. mesothelioma of the pleura has increased in incidence in the developed and emerging countries of the world because of increased exposure to airborne pollutants. Luketich JD: Chest wall. lung.Robinson BW. However. Five-year survival for mesothelioma is virtually zero compared with 13% for patients with all stages of bronchogenic carcinoma because of the high percentage of patients presenting with late-stage disease. and pleura. 2005 page 1180 .American College of Surgeons SESAP 13 Critique for Category 14 Item 52 Mesothelioma affects serosal surfaces of the body cavity. pp 545-610 2. peripheral and central neuropathies. Andersen DK. Up to 74% of patients with mesothelioma present with radiographic evidence of pleural effusion compared with 5% to 15% of those with bronchogenic carcinoma. McGraw-Hill. New York. ed 8. and myasthenia-like myopathies are frequent complications of bronchogenic carcinoma. it does not significantly improve survival. N Engl J Med 353:1591-1603. Dunn DL. Although radical surgical therapy consisting of extrapleural pneumonectomy or radical pleurectomy has been attempted. Lake RA: Advances in malignant mesothelioma. particularly asbestos. usually by direct extension of the tumor. the most prominent of which is pain. Billiar TR. in Bruincardi FC. Both mesothelioma and bronchogenic carcinoma can cause constrictive pericarditis. They are probably caused by hormone-like substances elaborated in the primary tumor REFERENCES 1. American College of Surgeons SESAP 13 Category 14 Item 53 Adjuvant chemotherapy is usually indicated (A) Extremity Ewing’s sarcoma (B) Retroperitoneal liposarcoma (C) Both (D) Neither page 1181 . Ewing sarcomas are radiosensitive. several studies have demonstrated good results with adjuvant chemoradiotherapy after resections with grossly negative but microscopically positive margins on critical structures. and there is little role for adjuvant radio. Judson I.American College of Surgeons SESAP 13 Critique for Category 14 Item 53 The management of extremity and retroperitoneal sarcomas is markedly different. eg. Most retroperitoneal liposarcomas are low-grade at presentation. Thomas JM: Soft-tissue sarcomas in adults. nerves. N Engl J Med 353:701711.or chemotherapy. Fisher C. Antonescu CR. 2005 2. Limb-sparing operation is possible in most patients. REFERENCES 1. Riedel E. Brennan MF: Histologic subtype and margin of resection predict pattern of recurrence and survival for retroperitoneal liposarcoma. 2003 page 1182 . On the other hand.Clark MA. Unlike retroperitoneal liposarcoma. most extremity Ewing sarcomas should receive adjuvant treatment. most retroperitoneal liposarcomas can be diagnosed with certainty on abdominal computed tomographic (CT) scan and do not require preoperative biopsies. which is notoriously radioresistant. The initial step in the management of most suspected extremity sarcomas should be an incisional biopsy. Whereas the key to the management of most retroperitoneal liposarcomas is en bloc resection with clear margins without adjuvant treatment. In efforts to achieve limb-sparing outcomes. Ann Surg 238:358-371.Singer S. American College of Surgeons SESAP 13 Category 14 Item 54 Primary chemoradiation therapy is curative (A) Extremity Ewing’s sarcoma (B) Retroperitoneal liposarcoma (C) Both (D) Neither page 1183 . eg. several studies have demonstrated good results with adjuvant chemoradiotherapy after resections with grossly negative but microscopically positive margins on critical structures. Whereas the key to the management of most retroperitoneal liposarcomas is en bloc resection with clear margins without adjuvant treatment. On the other hand. Fisher C. most extremity Ewing sarcomas should receive adjuvant treatment. Brennan MF: Histologic subtype and margin of resection predict pattern of recurrence and survival for retroperitoneal liposarcoma. Riedel E. Limb-sparing operation is possible in most patients. N Engl J Med 353:701711. Thomas JM: Soft-tissue sarcomas in adults. Antonescu CR. Ewing sarcomas are radiosensitive. and there is little role for adjuvant radio. REFERENCES 1. Unlike retroperitoneal liposarcoma. most retroperitoneal liposarcomas can be diagnosed with certainty on abdominal computed tomographic (CT) scan and do not require preoperative biopsies. which is notoriously radioresistant. In efforts to achieve limb-sparing outcomes.or chemotherapy.American College of Surgeons SESAP 13 Critique for Category 14 Item 54 The management of extremity and retroperitoneal sarcomas is markedly different. Ann Surg 238:358-371. The initial step in the management of most suspected extremity sarcomas should be an incisional biopsy. Judson I. Most retroperitoneal liposarcomas are low-grade at presentation.Clark MA. 2005 2. nerves.Singer S. 2003 page 1184 . American College of Surgeons SESAP 13 Category 14 Item 55 Adjuvant radiation therapy has little role in the treatment of primary tumors (A) Extremity Ewing’s sarcoma (B) Retroperitoneal liposarcoma (C) Both (D) Neither page 1185 . Antonescu CR. The initial step in the management of most suspected extremity sarcomas should be an incisional biopsy. Brennan MF: Histologic subtype and margin of resection predict pattern of recurrence and survival for retroperitoneal liposarcoma. Unlike retroperitoneal liposarcoma. Limb-sparing operation is possible in most patients. 2005 2. which is notoriously radioresistant. eg. N Engl J Med 353:701711.American College of Surgeons SESAP 13 Critique for Category 14 Item 55 The management of extremity and retroperitoneal sarcomas is markedly different. Ewing sarcomas are radiosensitive. REFERENCES 1. Ann Surg 238:358-371. several studies have demonstrated good results with adjuvant chemoradiotherapy after resections with grossly negative but microscopically positive margins on critical structures. In efforts to achieve limb-sparing outcomes. and there is little role for adjuvant radio. On the other hand. nerves. 2003 page 1186 . Whereas the key to the management of most retroperitoneal liposarcomas is en bloc resection with clear margins without adjuvant treatment. most extremity Ewing sarcomas should receive adjuvant treatment.Clark MA.Singer S.or chemotherapy. most retroperitoneal liposarcomas can be diagnosed with certainty on abdominal computed tomographic (CT) scan and do not require preoperative biopsies. Riedel E. Thomas JM: Soft-tissue sarcomas in adults. Most retroperitoneal liposarcomas are low-grade at presentation. Fisher C. Judson I. American College of Surgeons SESAP 13 Category 15 Item 1 Toxic epidermal necrolysis (TEN) is usually associated with (A) slough of both the dermis and epidermis (B) nondrug causes. most commonly herpes simplex infection (C) mortality equivalent to that associated with erythema multiforme (D) slough involving less than 10% of total body surface area (E) severe inflammation of the mouth page 1187 . Criteria vary. sore throat. the lesions may extend to the larynx. The more severe SJS is a blistering disorder characterized by erosions of multiple mucous membranes and small blisters developing on dusky or purpuric macules or atypical target lesions. bronchi. is usually self-limited . the skin of the face and extremities becomes tender and erythematous. Although the mortality of TEN generally ranges from 25% to 44%. Treatment is best undertaken in a major burn center. with 10% to 30% of body surface epidermal detachment. Wound management is also key. the wounds debrided. EM is characterized by distinctive target lesions. because the morbidity associated with the epidermal skin sloughing is comparable to that caused by second-degree burns. most commonly herpes simplex infection. Restoration of cardiopulmonary stability. urethra. Correia O. A prodrome usually consists of high fever.Auquier-Dunant A. Although it has been associated with sulfonamides. Naldi L. The skin changes are widespread. TEN is the most severe cutaneous exfoliative disorder and may be fatal. but the total percent of body surface area detachment is typically less than 10%. Blisters and bullae form. However. In general. The role of corticosteroids in the treatment of TEN remains unclear. the mildest of these disorders.cm target lesions. Stevens-Johnson syndrome. lesions then appear as large surfaces of red skin or 2. aminopenicillins. The eyes. is the first step in management. but small nasogastric or nasoenteric feeding tubes can usually be placed. Epidermal necrosis involves > 30% of body surface area. and allupurinol). Most authors advise against their use. if infected wounds are identified.American College of Surgeons SESAP 13 Critique for Category 15 Item 1 Toxic epidermal necrolysis (TEN) is the most extensive form of the severe exfoliative disorders. Prophylactic systemic antibiotics are not indicated in the absence of infection. and toxic epidermal page 1188 . uninfected disease. Overlap SJS/TEN shares characteristics of both SJS and TEN. patients who survive have an excellent prognosis. Nutrition is another major component of care. EM. penicillin. covered with Xeroform gauze or skin substitute. trachea. oxicam nonsteroidal anti-inflammatory agents. High fever and leukocytosis are usually present. One to two days later. and control of the febrile response. Mockenhaupt M. usually of the extremities. and gastrointestinal tract may also be involved. All of these syndromes are characterized by inflammation-induced dermal-epidermal separation. including aggressive pulmonary toilet and intubation if necessary. and malaise. EM most often has nondrug causes. and phenylbutazone. et al: Correlations between clinical patterns and causes of erythema multiforme majus. Drugs are the most frequent cause in adults (sulfonamides. Fever and malaise are common. then treated with topical antibiotics twice daily. REFERENCES 1. anticonvulsants. blisters should be debrided. The most characteristic feature of this disease is inflammation of the mouth. phenytoin. enteral nutrition is preferred over total parenteral nutrition. targeted systemic antibiotics should be initiated. which then slough. and esophagus. then covered with an outer gauze dressing that is changed twice daily. Patients may not be able to swallow and take adequate calories because of the oral lesions. intravascular volume replacement. which also include erythema multiforme (EM) and Stevens-Johnson syndrome (SJS).With early. and is often associated with sore throat and malaise. pp 1319-1332 4. 2002 page 1189 . sec 15. Kaiser LR. Arch Dermatol 138:1019-1024. 2005. in Kasper DL. pp 318-323 3.Palmieri TL. J Burn Care Rehabil 23:8796. et al: A multicenter review of toxic epidermal necrolysis treated in US burn centers at the end of the twentieth century. New York. Cutaneous drug reactions. Fink MP. McGraw-Hill. et al (eds): Harrison’s Principles of Internal Medicine. ed 16. Fauci AS. Greenhalgh DG. Stern RS. Jurkovich GJ.Heimbach DM. WebMD Inc. et al (eds): ACS Surgery: Principles and Practice 2006. Hauser Sl. 2002 2. Wintroub BU: Alterations in the skin. Spence RJ. chap 7. Trauma and Thermal Injury.Chosidow OM. Gibran NS: Miscellaneous burns and cold injuries. in Souba WW. New York.American College of Surgeons SESAP 13 necrolysis: results of an international prospective study. Braunwald E. Saffle JR. The next step should be (A) wound packing and return to the ICU (B) gram stain to determine further therapy (C) above-knee amputation (D) hyperbaric oxygen therapy (E) wide debridement of all involved tissues page 1190 . The area around the knee at incision is shown.American College of Surgeons SESAP 13 Category 15 Item 2 A 45-year-old alcoholic is admitted with septic arthritis. He develops progressive refractory cellulitis with fever and hypotension. Elliott DC. Staphylococcus. Burns 31:269-273. Black S. 2005 4. McMillan RW. Hyperbaric oxygen may be a useful adjunct to surgical therapy. but transfer to a facility for hyperbaric oxygen therapy should not be considered as a substitute for initial wide surgical debridement. de Virgilio C. 1996 3. Kufera JA. Zibari GB. Aultman DF. Cultures can be obtained at the time of operation as part of the surgical therapy. obtaining laboratory specimens. Myers RA: Necrotizing soft tissue infections. but urgent operation is indicated in this patient even if this sign is absent. 2005 5.Jallali N. Complete surgical debridement of affected tissue is most likely to improve this patient’s chance of survival. REFERENCES 1. Risk factors for mortality and strategies for management.Bilton BD. Broad-spectrum antimicrobial therapy should be initiated promptly. Klein SR: Objective criteria may assist in distinguishing necrotizing fasciitis from nonnecrotizing soft tissue infection. Examination of the fascia and the muscle below the subcutaneous tissues must be performed to allow complete debridement. The importance of urgent surgical intervention should not be underestimated: delayed operation increases mortality. Simply opening the wound. Supple KG. including gram-positive and gram-negative aerobes and anaerobes. Am J Surg 189:462-466.American College of Surgeons SESAP 13 Critique for Category 15 Item 2 The physical findings are most suggestive of a necrotizing soft tissue infection. Although further studies may support the diagnosis. or Clostridia) or polymicrobial infections. 2000 page 1191 . Am J Surg 179:17-21. Gamelli RL: The evolving characteristics and care of necrotizing soft-tissue infections. These infections can include monomicrobial processes (such as group A beta-hemolytic streptococcus. et al: Aggressive surgical management of necrotizing fasciitis serves to decrease mortality: a retrospective study. 1998 2. Computed tomography (CT) and magnetic resonance imaging (MRI) may note subcutaneous air. immediate operation and exploration of the involved area will provide both rapid diagnosis and therapeutic intervention. and packing would leave most of the diseased tissue in place. but is not the primary therapy. Am Surg 64:397-400. The initial operation should include radical debridement of all involved tissues and sampling for microbial analysis. Ann Surg 224:672-683. Amputation may be necessary depending on the depth of the infection and necrosis. Withey S. Butler PE: Hyperbaric oxygen as adjuvant therapy in the management of necrotizing fasciitis.Endorf FW. Further operative intervention may be necessary because the full extent of soft tissue injury may not be apparent at the time of the initial operation.Wall DB. represents a (A) keratoacanthoma (B) pyogenic granuloma (C) melanoma (D) basal cell carcinoma (E) portwine nevus page 1192 .American College of Surgeons SESAP 13 Category 15 Item 3 A 10-year-old boy presents with the chest lesion shown. which is rapidly growing and bleeding. This lesion. emedicine. Goldstein AO: Benign neoplasms of the skin. Pierson DM: Pyogenic granuloma (lobular capillary hemangioma). confluent. The mean age at presentation is 6. Available at: www.Goldstein BG. are not raised. The lesion is not infectious as the name suggests. Basal cell carcinomas are typically scaly.7 years. It arises over the course of several weeks.Pierson JC. and arise over an extended period. Excision is indicated for cosmetic reasons or because it may be bothersome and may bleed easily. It is relatively uncommon and represents about 0. and rarely bleed.com 2. eMedicine website. Port wine nevus or port wine stains are uncommon capillary malformations. but rather is associated with capillary proliferation.utod. They are large. It is often but not always associated with antecedent trauma. A pregnancy variant occurs in up to 5% of pregnancies. Up to Date website.American College of Surgeons SESAP 13 Critique for Category 15 Item 3 The lesion shown is a pyogenic granuloma. www. Keratoacanthoma is a rapidly growing nodule with a central keratin plug. but usually arise over a much longer period.5% of all skin nodules in children. do not bleed early.com page 1193 . Malignant melanoma can have varying presentations. 2006. REFERENCES 1. American College of Surgeons SESAP 13 Category 15 Item 4 An 80-yearold man admitted to a nursing home 2 months previously has the new sacral pressure sore shown in the photo. Bone can be felt when the wound is probed. The stage would be (A) IVa (B) IVb (C) III (D) II (E) I page 1194 . but is not exposed. Pressure ulcer formation within the first 3 months of admission to a long-term care facility is associated with a mortality > 90%. occur in geriatric patients. Pseudomonas aeruginosa. debridement of necrotic tissue. 2001 3. incontinence. 2001 page 1195 .Thomas DR: Issues and dilemmas in the prevention and treatment of pressure ulcers: a review. Dermatol Nurs 13:151-152. 2001 2. and Bacteroides fragilis. Among younger patients. fever. Up to 70% of pressure ulcers. and leukocytosis. malnutrition. Hofmann MT: Skin and soft tissues. Treatment of pressure ulcers includes pressure relief (air-fluidized bedding is preferred). tissue necrosis. Infections tend to be polymicrobial and common agents include Staphylococcus aureus. Signs of infection include increasing warmth. these ulcers tend to develop among the neurologically impaired. As such. purulent exudate.American College of Surgeons SESAP 13 Critique for Category 15 Item 4 Pressure ulcers display a bimodal distribution pattern. Management of four common infections in the nursing home patient. topical therapy using occlusive moist dressings. and depressed consciousness. and erythema. and antibiotic administration if infection is present. REFERENCES 1. Risk factors include immobility. edema. Intravenous antibiotics are indicated if patients require hospitalization for infected ulcers.O’Donnell JA. The decubitus ulcer pictured extends through the skin into the subcutaneous tissue. Enterobacteriaciae. Geriatrics 56:33-38. J Gerontol A Biol Sci Med Sci 56:M328-M340.Cuzzell J: Wound assessment and evaluation: documentation of pressure ulcers. it is a type III lesion. nutritional support. occurring in both the young and the elderly. foul odor. however. but does not involve the fascia overlying the sacrum. Classification is based on the depth of invasion (see table). American College of Surgeons SESAP 13 Category 15 Item 5 A 50-year-old man requires a forequarter amputation of his left arm for necrotizing fasciitis. Which of the following should be recommended after the patient recovers from the amputation? (A) Upper gastrointestinal endoscopy (B) Colonoscopy (C) Positron emission tomography (PET) (D) Cystoscopy (E) Tagged WBC scan page 1196 . Wound cultures grow Clostridium septicum. 1988 6.Larson CM. Breast. Musher DM. 1995 3. Ann Surg 193:361-364. et al: Malignancy. and colonoscopy is the most appropriate screening modality for this patient. Leukopenia and diabetes are considered two predisposing factors. These infections are often polymicrobial. et al (eds): ACS Surgery: Principles and Practice 2006. Clostridial involvement often leads to myonecrosis and produces the classic crepitance of gas gangrene. mortality. Eidus LB: Invasive Clostridium septicum infection in association with colorectal carcinoma. Eddy H. and medicosurgical management of Clostridium septicum infection. Bubrick MP. West MA. Derkac WM. cystoscopy. REFERENCES 1.Katlic MR. Rev Infect Dis 12:286-296. Jurkovich GJ. Skin. Rev Infect Dis 10:347-364. pp 224-237 5. Watson DA. Jacobs DM. Kaiser LR. McHenry CR: Soft tissue infection. harboring both aerobic and anaerobic bacteria. et al: Spontaneous. it is important to recognize when Clostridium septicum is present because C septicum is associated with an occult malignancy in 50% of cases.Lorimer JW. 1990 page 1197 . chap 3. or tagged WBC scan are not indicated once the patient recovers. WebMD Inc. Although Clostridium perfringens is the species commonly cultured from necrotizing infections. positron emission tomography (PET). Upper gastrointestinal endoscopy. amputation rates can reach 25%. In most cases. Surgery 118:592-598. Fink MP. New York. An occult colonic source should be ruled out in patients who present with C septicum infections. sec 2. Emergency radical debridement is required. and Soft Tissue. Up to 20 % of cases have no apparent patient predisposition or site of inoculation. Coleman WS: Clostridium septicum infection and malignancy. Only one organism is present in up to 30% of cases. 1981 2. 1994 4. the lesion is an adenocarcinoma of the colon.American College of Surgeons SESAP 13 Critique for Category 15 Item 5 Necrotizing fasciitis is a life-threatening soft tissue infection with high mortality. nontraumatic gangrene due to Clostridium septicum. in Souba WW.Panwalker AP: Unusual infections associated with colorectal cancer.Malangoni MA.Stevens DL. Can J Surg 37:245-249. American College of Surgeons SESAP 13 Category 15 Item 6 The detection of microscopic sulfur granules in a specimen of purulence is pathognomonic for infection with (A) actinomycosis (B) mucormycosis (C) Eikenella corrodons (D) Stenotrophomonas maltophilia (E) Nocardia asteroides page 1198 . previous antibiotic use. Abdominal actinomycosis usually causes pain in the ileocecal region. vomiting. Computed tomographic (CT) scanning may reveal an inflammatory mass that extends to involve bone. Filaments within a granule are often visible on Gram or methenamine-silver stain. Sinuses draining to the exterior may develop. et al: Central venous catheter and Stenotrophomonas maltophilia bacteremia in cancer patients. orbital. prolonged hospitalization. paranasal sinus. Stenotrophomonas maltophilia is a multidrugresistant. Major risk factors for the infection include neutropenia. Ten to 20 million units is given parenterally for 2 to 4 weeks. is an acute fungal disease with a high mortality rate. branching filamentous bacteria that may fragment into bacillary forms. grampositive. New York. Anaerobic culture is necessary to distinguish actinomycetes from nocardiae. Cancer 106:1967-1973. Lesions may develop in the gastrointestinal tract or lungs after ingestion or aspiration of the organism from its endogenous source in the mouth. The most common site of infection is the cervicofacial area (about 60% of cases). REFERENCES 1. gram-negative bacillus that has emerged as an important opportunistic pathogen. It may be confused with Crohn's disease or appendicitis. The granules are composed of an internal tangle of mycelial fragments and a rosette of peripheral clubs. Eikenella corrodens is a gram-negative. and weight loss. and medical devices such as prosthetic heart valves. primary cutaneous disease occurs in normal hosts with significant soil contact or as postoperative infections. causing a variety of soft-tissue infections particularly associated with human or animal bites. particularly in immunosuppressed and critically ill patients. corticosteroid use. Penicillin G is the drug of choice for actinomycosis. Ansari S. WebMD. rod-shaped bacterium. it can cause a wide spectrum of other infections as well. Nocardia species are ubiquitous soil-dwelling gram-positive bacteria responsible for a wide spectrum of disease in patients with both normal and abnormal immune systems. Although Nocardia may disseminate from the primary site of infection. Although infection commonly manifests as bacteremia. This bacterium can also be pathogenic. 2006 2. Therapy should be continued for weeks to months after clinical manifestations have disappeared to ensure cure. also known as zygomycosis and phycomycosis. blood cultures are rarely positive. They are often visible to the naked eye or by microscopy with low magnification. and indwelling central venous catheters. or cerebral regions. Federman DD (eds): ACP Medicine (2006). Bahna B. Mucormycosis. 2006. Infection typically follows extraction of a tooth or other pharyngeal trauma. facultative anaerobic. Patients generally present with signs and symptoms primarily located in the facial. found predominantly in subgingival plaque in patients with advanced periodontitis. however. Introduction through the respiratory tract is most common. Actinomycosis is characterized by the presence of sulfur granules within infected tissue. mechanical ventilatory support. The presence of a palpable abdominal mass may suggest the diagnosis of malignancy. Hanna H. though more calcified granules may be difficult to identify. pp 1610-1611 page 1199 .Chow AW: Anaerobic infections. spiking fever and chills. urinary catheters. Surgical procedures such as drainage and resection may be beneficial. Response to therapy is slow. in Dale DC. followed by oral penicillin (500 mg 4 times daily) or amoxicillin (750 mg twice daily). They are anaerobic.Boktour M.American College of Surgeons SESAP 13 Critique for Category 15 Item 6 Actinomyces occur in the normal flora of the mouth and tonsillar crypts. Billiar TR. New York. Young DM: Skin and subcutaneous tissue. Piro J: Disseminated rhinocerebral mucormycosis: a case report and review of the literature. George EB.American College of Surgeons SESAP 13 3. J Oral Maxillofac Surg 64:326-333. Mathes SJ. McGraw-Hill. Anderson DK. Dunn DR. Alessi AS. in Brunicardi FC. 2006 page 1200 .Hansen SL.O’Neill BM. 2005. ed 8. et al (eds): Schwartz’s Principles of Surgery. pp 429-452 4. American College of Surgeons SESAP 13 Category 15 Item 7 Which of the following statements about brown recluse spider (Loxosceles reclusa) bites is TRUE? (A) Dapsone is primary therapy (B) The area should be widely excised to prevent further cutaneous necrosis (C) Systemic features include hemolysis and renal failure (D) Commercial antivenom is available and should be given within 12 hours of bite (E) Systemic corticosteroids halt progression of the lesion page 1201 . Many cutaneous conditions. Envenomation can cause symptoms ranging from mild irritation to severe cutaneous necrosis. renal failure. but no evidence exists to support it. Nelson EE: Bites and stings. 2005 page 1202 . 2004. Auerbach PS. nausea and vomiting. Philadelphia.Norris RL.American College of Surgeons SESAP 13 Critique for Category 15 Item 7 The brown recluse spider (Loxosceles reclusa) is endemic to the central and southern Midwestern United States. and without long-term sequelae. disseminated intravascular coagulation. hemolysis. Although dapsone is commonly administered and its use supported by animal data. in Townsend CM Jr (ed): Sabiston Textbook of Surgery: The Biological Basis of Modern Surgical Practice. N Engl J Med 352:700-707. It is identifiable by its 6 paired eyes (most spiders have 8) and less reliably by a violinshaped pattern on the cephalothorax. malaise and arthralgias. pp 597-612 2. Surgical therapy should include only conservative debridement of obviously necrotic tissue because the eventual extent of necrosis is difficult to predict. and definitive diagnosis is virtually unknown outside of the spider’s endemic area. some far more dangerous. Most brown recluse spider bites are self-limited. Systemic symptoms such as headache. fever. REFERENCES 1. There is no antivenom commercially available in the United States. ed 17. and death can occur. Saunders. but are exceedingly rare. Diagnosis of cutaneous necrosis due to brown recluse spider envenomation is suspect unless the spider is provided and identified. self-healing. are commonly misdiagnosed as loxoscelism. Treatment involves supportive care. Definitive diagnosis of a brown recluse spider envenomation requires collection and identification of the spider responsible. Vetter RS: Bites of brown recluse spiders and suspected necrotic arachnidism. prospective trials in humans are lacking due to the difficulty of diagnosis and rarity of this condition. and signs such as thrombocytopenia.Swanson DL. Wide local excision of brown recluse spider bites is likely to be needlessly aggressive and disfiguring. Glucocorticoid therapy has also been recommended. and dry in the central portion of the wound. her wounds are red with pale areas centrally.American College of Surgeons SESAP 13 Category 15 Item 8 A 44-year-old woman with a history of seizure disorder pulls a pan of hot grease onto herself during a seizure. This patient has most likely sustained a (A) superficial burn (B) superficial partial-thickness burn (C) deep partial-thickness burn (D) full-thickness burn (E) first-degree burn page 1203 . chest. She has burns to her chin. and abdomen. On initial evaluation. slow to blanch. pp 357-373 2.Williams WG: Pathophysiology of the burn wound. World J Surg 16:10-15. These wounds may heal by re-epithelialization from the dermal appendages and wound periphery after 3 weeks and will most likely leave a hypertrophic scar if treated nonsurgically. pp 514-522 page 1204 .Coleman JJ. No dermal appendages are viable for re-epithelialization. A sunburn is a superficial or first-degree burn. These wounds may even be dark red in appearance as a result of dermal vascular stasis. ed 2. in Herndon DN (ed): Total Burn Care.Heimbach D.Garner WL: Thermal burns. These burn wounds are extremely painful and blanch easily when compressed. et al (eds): Plastic Surgery Indications. Marvin J: Burn depth: a review. A partial-thickness burn means that damage has occurred to the dermis. The superficial partial-thickness burn extends into the papillary dermis. Grube B. The skin appendages are damaged and the hair follicles usually are disrupted. and Outcomes. The damaged epidermis may exfoliate in approximately 7 to 10 days. These wounds have a leathery feel and are not supple like nonburned skin. These wounds contain necrotic tissue and the eschar is usually thick. Operations. 2002. The wounds have slow capillary refill because of damage to the dermal plexus and hemagglutination in subjacent capillaries. Deep partial-thickness burns extend into the reticular dermis. WB Saunders. in Achauer BM. Eriksson E. dry. The eschar is the product of coagulated proteins from the injured skin. This is characterized by blister formation with moist and pink wounds beneath. In a favorable environment. The skin appendages in the dermis are still viable and the hair follicles are difficult to remove. these wounds will re-epithelialize within 2 weeks by migration of the epithelial cells lining the hair follicles and sebaceous glands onto the surface of the dermis to replace the damaged and destroyed keratinocytes. REFERENCES 1. St. Louis. Engrav L. 1992 3.American College of Surgeons SESAP 13 Critique for Category 15 Item 8 The superficial or first-degree burn usually has a damaged but intact epidermis that is red in appearance. Mosby. but is usually healed underneath at the time of separation. These burns can be further subdivided into superficial or deep partial-thickness burns. London. 2000. Full-thickness burn injuries involve the entire thickness of the burn. These burns can be mottled and dry in appearance and will likely form an eschar. Vander Kolk C. Surgical intervention and excision and grafting are necessary unless the lesions are very small and can be left to separate and heal by constant contraction and epithelialization. but they do not blanch with pressure and are generally insensate. and white or charred in appearance. treatment should be (A) intravenous priscolene (B) intravenous calcium channel blockers (C) intra-arterial calcium gluconate (D) sublingual nitrate (E) intravenous magnesium sulfate page 1205 . The wounds appear to be superficial with minimal blistering.American College of Surgeons SESAP 13 Category 15 Item 9 A 33-year-old petroleum worker presents with a 4% total body surface area (TBSA) burn to his left hand and arm sustained when hydrofluoric acid spilled onto his clothing. The patient is writhing in pain that has not improved with intravenous morphine. In addition to further irrigation of the wound. His wounds were copiously irrigated with water at the scene. Acids cause a burn resulting in a coagulation necrosis of the skin. et al: Extensive hydrofluoric acid injuries: A serious problem. pp 475-480 4. the concentration or strength of the agent. particularly in the presence of severe pain. The initial treatment of chemical burns is to remove the patient’s clothing and copiously irrigate the wound with water. tissue necrosis can occur with subcutaneous and intra-arterial injections. acting as a metabolic poison. 2005 page 1206 . due to high milliequivalent concentrations of calcium in calcium chloride. J Trauma 28:642-647. and hyperkalemia. Hydrofluoric acid is commonly used in petroleum production and glass etching. these wounds often require debridement and grafting as well. once in contact with the cells. Pruitt BA Jr. in Herndon DN (ed): Total Burn Care. Local injections of a 10% solution of calcium gluconate with or without topical calcium gluconate gel are necessary in addition to irrigation of the wound. Alkalis produce a liquefaction necrosis and penetrate deeper into the tissues. If untreated. London. and the amount of tissue penetration of the agent.American College of Surgeons SESAP 13 Critique for Category 15 Item 9 Chemical burns vary in severity depending on the type of offending agent and its manner of reactivity.Kales SN. Catheter-directed intra-arterial calcium gluconate may be used in extremity burns.Wedler V. the fluoride ion. J Trauma 58:852-857. although solvents and hydrocarbons are also occasionally involved. Neutralizing agents should not be used because they can produce exothermic reactions when they come in contact with the offending agent. and additional therapy may be required. the duration of exposure. Moron M. nitrates. ed 2. priscolene. Irrigation should continue for at least 30 minutes or until the pH is 7. hypomagnesemia. combines with the positively charged ions of calcium and magnesium. These treatments should be used 4 to 6 times per day until the patient has relief from the significant pain that is the cardinal symptom. N Engl J Med 350:800-808. and calcium channel blockers are not effective. Herndon DN: Chemical burns. Ocular injuries should likewise be irrigated until the effluent pH is 7. the quantity of the agent in contact with the tissue. McManus WF. The water should be made to drain or flow away from the patient to dilute the concentration of the chemical and avoid further injury by the dissolved chemical. Christiani DC: Acute chemical emergencies. Guggenheim M. Until the fluoride ion is completely neutralized by calcium or magnesium. Vasodilators. Calcium chloride can also be injected intravenously for symptom relief. Kunzi W.Mozingo DW. Because of the attendant coagulation necrosis.Sanford AP. 2002. Smith AA. WB Saunders. the acid will continue to penetrate deeper into the tissues and can cause systemic hypocalcemia. Most chemicals involved in burns are either acids or alkalis. et al: Chemical burns. In addition to the coagulative necrosis caused by the low pH. 2004 2. the acid will continue to penetrate deeper into the tissues and cause further protein denaturation. However. REFERENCES 1. 1988 3. He has similar problems in the groins bilaterally.American College of Surgeons SESAP 13 Category 15 Item 10 A 52-year-old man has recurrent abscesses requiring repeated incision and drainage in both breasts. The best treatment for this patient would be (A) izoniazid (B) antibiotic therapy (C) bilateral simple mastectomy (D) synthetic retinoid (acitretin) therapy (E) excision of the nipple-areolar complex and any involved tissue of both breasts page 1207 . as shown. Yu CC. rather than apocrine glands. 1990 page 1208 . Excision of the nipple-areolar complex and any involved tissue of both breasts would be the best treatment.Li EN. Relapse is almost inevitable after withdrawal of medication. 2005 4.Shah N: Hidradenitis suppurativa: a treatment challenge. REFERENCES 1. especially in advanced disease such as this. Izoniazid is effective in tuberculosis. Retinoid therapy is of questionable benefit. a common disorder that results in recurrent abscesses and draining sinus tracts. However.Banerjee AK: Surgical treatment of hidradenitis suppurativa. Because the hair follicles and their associated abscesses are just below the dermis. Ann Plast Surg 52:220-223. excision of the involved skin with a thin layer of subcutaneous fat along with all abscesses should suffice. this option should be reserved for patients with deep abscesses involving all of the breast tissue. Br J Dermatol 122:763-769. is a far rarer site of hidradenitis. Am Fam Physician 72:1547-1552. Mofid MM. but there is no evidence that long-term antibiotic therapy alters the natural history of hidradenitis suppurativa. Goldberg NH. In females this should allow preservation of the breast contour. Incision and drainage including sinus and fistulous tracts unroofing may be done in the acute situation prior to more definitive operation. 2004 3. Silverman RP: Surgical management of hidradenitis suppurativa of the nipple-areolar complex. Breast biopsy is not indicated in patients with hidradenitis involving the breast tissue unless a mass is palpated or a malignant process suspected. Antibiotics may diminish odor and reduce pain. Br J Surg 79:863-866. The nipple-areolar complex. 1992 2. as in this patient. episodes of inflammation usually continue to occur. The sites most commonly involved include the axillae and inguinoperineal regions.American College of Surgeons SESAP 13 Critique for Category 15 Item 10 This patient has hidradenitis suppurativa. but not hidradenitis. Cook MG: Hidradenitis suppurativa: a disease of follicular epithelium. Although simple mastectomy has been described in the treatment of nipple-areolar hidradenitis. as in this patient. American College of Surgeons SESAP 13 Category 15 Item 11 Sentinel lymph node biopsy for melanoma (A) predicts prognosis (B) improves survival (C) is indicated for melanomas < 1 mm thick (D) is associated with a complication rate of 23% (E) should be followed by completion lymph node dissection in patients who have positive results page 1209 . Because many large retrospective studies have shown a strong negative correlation between the presence of metastatic melanoma in sentinel nodes and survival. 2005 3.5 mm. Elashoff RR. Tanabe KK. 2001 2. and 36% if it is 4. N Engl J Med 351:998-1012. et al: Detection of microscopic melanoma metastases in sentinel lymph nodes. Sober AJ: Management of cutaneous melanoma.Balch CM. et al: Sentinel node biopsy for early-stage melanoma: accuracy and morbidity in MSLT-I. Gadd MA.0 mm.8 to less than 1. Because patients with melanomas that are 1. sentinel lymph node biopsy (SLNB) is currently the most powerful staging and prognostic tool. Flotte TJ. 2004 4.5 to less than 4.0 mm or greater. J Clin Oncol 19:3635-3648. Ann Surg 242:302311. Atkins MB. Further studies are needed to determine the additional benefit of completion lymph node dissection in patients who have positive results on sentinel node biopsy. 8% if it is 0. 1999 page 1210 . REFERENCES 1. Atkins MB. Cochran AJ. SLNB is not commonly performed but could be considered if pathologic examination showed negative prognostic features such as ulceration or Clark level IV to V invasion. Cancer 86:617-27. Early results from 2120 patients in the Sunbelt Melanoma Trial estimate complication rates of 4% for SLNB and 23% for SLNB with completion lymph node dissection. Buzaid AC. Thompson JF.0 mm or less in thickness rarely have nodal disease.American College of Surgeons SESAP 13 Critique for Category 15 Item 11 The likelihood of detecting metastatic deposits in the sentinel lymph node depends on tumor thickness: approximately 1% if the thickness of the tumor is less than 0. et al: Final version of the American Joint Committee on Cancer staging system for cutaneous melanoma.Morton DL. 23% if it is 1. Performance of this minimally invasive procedure provides prognostic information and identifies candidates for systemic adjuvant treatment.8 mm. Soong SJ. an international multicenter trial.Tsao H.Yu LL. American College of Surgeons SESAP 13 Category 15 Item 12 Which of the following statements about the usage of antibiotics for the prevention of surgical site infection is NOT true? (A) Antibiotics should be administered within 60 minutes of incision (B) The therapeutic dose of the antibiotic should be administered intravenously (C) Adequate tissue concentrations should be maintained during operation by re-dosing as necessary (D) Antibiotics should be continued for 48 hours after operation (E) Vancomycin is the agent of choice for patients from long-term facilities undergoing hip replacement page 1211 . Mangram AJ. Infect Control Hosp Epidemiol 20:250-278. A second dose of the prophylactic antibiotic is warranted if the duration of the operation exceeds twice the half life of the antibiotic or if there is massive fluid infusion or blood loss intraoperatively. Infect Control Hosp Epidemiol 24:362-386. Silver LC. Houck PM. A single preoperative dose of the same strength as a full therapeutic dose is adequate in most instances. In patients at high risk for MRSA colonization such as institutionalized patients. vancomycin should be considered as an alternative to cefazolin in clean cases. Surgical Infection Prevention Guidelines Writers Workgroup.1999 3. American Academy of Orthopaedic Surgeons. 1999. Horan TC. and are now part of the Surgical Care Improvement Project (SCIP). The single dose should be given intravenously within 60 minutes before skin incision. Richet HM. and patients recently treated with antibiotics or recently hospitalized. 2003 page 1212 . Ostrowsky BE. Hospital Infection Control Practices Advisory Committee. and mounting data suggest that the preoperative dose is sufficient. et al: Guideline for prevention of surgical site infection. 2004 2. Pearson ML. No additional benefit has been demonstrated in continuing prophylaxis beyond 24 hours from the initial antibiotic dose.Bratzler DW. Clin Infect Dis 38:1706-1715. et al: SHEA guideline for preventing nosocomial transmission of multidrug-resistant strains of Staphylococcus aureus and enterococcus. REFERENCES 1. Vancomycin and fluoroquinolones should be infused over 60 minutes and can be given up to 2 hours prior to incision time. a Medicare Quality Improvement Project. dialysis patients.Muto CA. et al: Antimicrobial prophylaxis for surgery: an advisory statement from the National Surgical Infection Prevention Project. Jernigan JA.American College of Surgeons SESAP 13 Critique for Category 15 Item 12 The principles of antibiotic prophylaxis have been delineated by the Center for Disease Control and Prevention. Management should include (A) core biopsy of the breast mass (B) urgent surgical exploration of her right breast (C) aspiration and antibiotics (D) bilateral mammography (E) breast magnetic resonance imaging (MRI) page 1213 . she has a 2-cm fluctuant tender mass in the upper outer quadrant of her right breast.. She has no evidence of lymphadenopathy or other masses.American College of Surgeons SESAP 13 Category 15 Item 13 A patient who is 2 months postpartum has a tender mass in her right breast. On examination. Other imaging techniques including mammography are not helpful. ed 8. Repeated aspirations or continuous catheter drainage combined with oral antibiotics are also effective treatments. Epidemic mastitis occurs within 4 days of delivery. Milk leukocyte counts and quantitative bacterial cultures may be helpful. Incision and drainage should be reserved for larger abscesses. 2005. Nonepidemic (sporadic) mastitis usually occurs 2 or more weeks after delivery and rarely progresses to breast abscess. Ultrasonography for assessment of clinically suspected abscess in the acutely inflamed breast helps to prevent unnecessary surgical intervention. Inan A: Breast abscesses in lactating women. in Brunicardi FC. and is useful for following complete resolution of the abscess cavity. Schorr SJ: The diagnosis and management of breast problems during pregnancy and lactation. Am J Surg 170:401-405.Scott-Conner CE. 2003 3. Breast abscesses < 5 cm in diameter on physical examination can be treated with repeated aspirations with good cosmetic results. REFERENCES 1. Andersen DK. Dunn DL. epidemic and nonepidemic. et al (eds): Schwartz’s Principles of Surgery. There are two principal forms of mastitis. Daldal E: Management of lactational breast abscesses. McGraw-Hill. Core biopsies are indicated for lesions suspicious for malignancy.Dener C. Sahin M. 1995 page 1214 . with a spectrum ranging from localized mastitis to abscess formation.American College of Surgeons SESAP 13 Critique for Category 15 Item 13 Breast infection is a common problem in lactating women. Billiar TR. Until the last decade. surgical incision and drainage was the recommended treatment. Inadequate or delayed treatment can lead to abscess formation. World J Surg 27:130-133. Hakan Tekelioqlu M. Breast magnetic resonance imaging (MRI) is more expensive and does not provide any added benefit over ultrasound in the management of postpartum breast abscesses. Beenken SW. Most mastitis cases can be treated with antibiotics without interrupting breast-feeding. 2005 4. Copeland EM III: The breast.Eryilmaz R. Breast 14:375-379. New York. Puerperal mastitis affects 2% of lactating women.Bland KI. with a reported incidence of 5% to 11%. pp 453-500 2. Serum sodium is 129 mEq/L and WBC count is 21.8°C.American College of Surgeons SESAP 13 Category 15 Item 14 Twenty-four hours after sustaining an on-the-job puncture wound.000/mm3. The most appropriate treatment now would be (A) hyperbaric oxygen (B) immediate operative wide debridement (C) incision and drainage with packing and re-evaluation in 24 hours (D) irrigation with antibiotic solution and re-evaluation in 14 hours (E) intravenous broad-spectrum antibiotics page 1215 . The appearance of the wound is shown. a 33-year-old construction worker has a temperature of 39. 2005 3. Most clinical reviews report insufficient data to assess the potential benefits of hyperbaric oxygen treatment. Whereas clostridial infection involves only 10% of all patients with this disease. Sullivan SR. Approximately 50% of survivors require in-patient rehabilitation after acute care management. and significant health care resource utilization. creatinine level > 2 mg/dL. McMahon K.American College of Surgeons SESAP 13 Critique for Category 15 Item 14 Necrotizing soft tissue infections such as necrotizing fasciitis and Fournier’s gangrene are associated with mortality rates exceeding 20%. many patients with necrotizing fasciitis are too critically ill to undergo hyperbaric oxygen treatment and surgical debridement remains the mainstay of treatment. Whereas optimal tissue oxygenation may improve healing and reduce anaerobic infection. Delayed diagnosis and /or delayed surgical debridement of necrotizing soft tissue infections clearly increase mortality.000. Curr Opin Infect Dis 18:101-106.Wong CH. Independent predictors of limb loss included heart disease and shock (systolic blood pressure < 90 mm Hg) on admission. and heart disease at hospital admission. Supple KG. et al: Predictors of mortality and limb loss in necrotizing soft tissue infections. Independent predictors of mortality include WBC count > 30. Gamelli RL: The evolving characteristics and care of necrotizing soft-tissue infections.Endorf FW. high morbidity including amputation rates exceeding 25%. making early diagnosis difficult. Arch Surg 140:151-157. it is commonly associated with intravenous drug use and an elevated admission leukocytosis and is an independent predictor for limb loss and mortality. Specific cutaneous signs are often underwhelming. 2005 2. Patients with these diagnoses are increasingly being referred to trauma and burn centers for specialized wound and critical care. Over 50% of patients have polymicrobial infections. 2005 page 1216 . Burns 31:269-273. Nathens AB. Wang YS: The diagnosis of necrotizing fasciitis. REFERENCES 1.Anaya DA. Pulses are intact.American College of Surgeons SESAP 13 Category 15 Item 15 A 52-year-old man has had the ulcer pictured for 6 months. Examination of the wound reveals that it probes to bone. The test with the greatest accuracy for osteomyelitis is (A) plain x-rays (B) technetium bone scan (C) computed tomographic (CT) scan (D) isotope-labeled leukocyte scan (E) magnetic resonance imaging (MRI) page 1217 . J Foot Ankle Surg 39:S1-S60.Palestro CJ. and specificity of commonly used studies are listed in the table. but may not demonstrate any osseous changes for up to 14 days. Computed tomographic (CT) scan has not been shown to be useful for diagnosing osteomyelitis in the foot. Caprioli R. Nicholas GG. Grosser DM: Imaging modalities of the diabetic foot. Giurini J. The ability to probe bone with the use of a blunt. Wasser TE. 2003 5. Osborne MA. 2003 page 1218 . Rheum Dis Clin North Am 29:89-109. et al: Diabetic foot disorders: a clinical practice guideline. Edwards A. American College of Foot and Ankle Surgeons.Frykberg RG.Croll SD. sterile. REFERENCES 1. J Vasc Surg 24:266-270.Santiago Restrepo C. et al: Rapid diagnosis of pedal osteomyelitis in diabetics with a technetium-99m-labeled monoclonal antigranulocyte antibody. McCarthy K: Imaging of osteomyelitis and musculoskeletal soft tissue infections: current concepts. 2000 3. Gimenez CR. Plain x-rays should be the initial imaging study. Richardson HL. but this finding requires confirmation.Sella EJ. sensitivity. 1996 2. et al: Role of magnetic resonance imaging in the diagnosis of osteomyelitis in diabetic foot infections.American College of Surgeons SESAP 13 Critique for Category 15 Item 15 No consensus exists on the optimal criteria for diagnosing osteomyelitis in the diabetic foot. 2003 4. J Foot Ankle Surg 42:2-8. Clin Podiatr Med Surg 20:729-740. stainless steel probe had a positive predictive value (PPV) of 89% in one study. Armstrong DG. Love C. The accuracy. She has a history of intermittent diarrhea and abdominal pain. The wound is exquisitely painful and got larger when a diagnostic biopsy was obtained. The most likely diagnosis is (A) arterial insufficiency (B) sarcoidosis (C) calciphylaxis (D) venous insufficiency (E) pyoderma gangrenosum page 1219 .American College of Surgeons SESAP 13 Category 15 Item 16 A 55-year-old woman presents with a lower extremity wound. as shown. Fivenson DP: Leg ulcer diagnosis and management. They occur mostly in the gaiter region in the lower third of the leg. 1998 page 1220 . and postural color changes (reddish-bluish discoloration. Arterial insufficiency ulcers are usually below the level of the malleoli and have a punched out appearance. with large. plaques. 2006 4. and disuse atrophy. and hypomagnesemia. hypocalcemia. Pappas PJ: Current management of venous ulceration.Hoffman MD: Pyoderma gangrenosum. Piette WW: Cutaneous manifestations of systemic diseases. Minimal trauma. The typical signs and symptoms include intermittent claudication or rest pain. Labropoulos N. This cascade can predispose select patients to the progressive calcification of soft tissues and vessels known as calciphylaxis. The typical presentation is the appearance of a painful. REFERENCES 1. painful violaceous plaques quickly evolving to fullthickness necrosis and gangrene.Choucair MM. nodules. hypertriglyceridemia. End-stage renal failure can trigger a metabolic cascade characterized by hyperparathyroidism. The ulcers have a propensity to appear on the lower limbs or trunk. Su WP. and varicose veins. 2001 2. ulceration. The ulcerative form is the most common and is often associated with inflammatory bowel disease or arthritis. systolic bruits. Plast Reconstr Surg 117:254S-260S. The lesions contain sarcoid granulomas and include papules. Onset is often rapid. hyperphosphatemia. Perry HO: Pyoderma gangrenosum: classification and management. The surrounding skin has the stigmata of chronic venous insufficiency. or mottled duskiness of the skin). pigmentation due to hemosiderin deposition. are medial in 70% of patients. Dermatol Clin 19:659-678. pallor. rapidly developing ulceration with serpiginous. Venous ulcers comprise 80% to 90% of lower extremity ulcers. bluish. Wounds 11:2B-7B.American College of Surgeons SESAP 13 Critique for Category 15 Item 16 The only diagnosis consistent with the appearance and history of this wound is pyoderma gangrenosum.Robson KJ. Sarcoidosis may have cutaneous manifestation in about 25% of cases. pain with elevation. including edema. This phenomenon is known as pathergy and may be helpful in the diagnosis. delayed capillary refill. They do not present as ulceration.Patel NP. The edge of the ulcer is usually irregular and the base is pink with granulation tissue and is often covered with yellowish-green slough. gangrene. will cause ulcer expansion. and are relatively painless. J Am Acad Dermatol 34:395-409. Signs of occlusive arterial disease include diminished or absent pulses. Med Clin North Am 82:13591379. ischemic neuropathy. dermal fibrosis. 1999 3. undermined borders that developed from a pustule. including biopsies for histopathology. 1996 5. and scar infiltration.Powell FC. A probe is successfully passed through the dominant secondary opening into the rectum on a course that includes the internal and external anal sphincters. rectal mucosa is normal.American College of Surgeons SESAP 13 Category 15 Item 17 A 32-year-old man with Crohn’s disease presents with a complex perianal infection. At operation. There are 4 secondary openings on the perineum. He has not been treated for Crohn’s disease for several years. with a single primary opening in the posterior midline at the level of the dentate line. The optimal surgical management would be (A) fistulotomy (B) a tight cutting seton (C) a noncutting draining seton (D) fibrin glue injection (E) irrigation and packing page 1221 . These measures are not appropriate in the setting of active infection. drains the pus. definitive treatment may be appropriate. which put the patient at risk for fecal incontinence. Hyman N. the fistula is complex with sphincter involvement that precludes definitive fistulotomy or cutting seton. When the infection has been drained and inflammation minimized with prolonged seton drainage. Most patients should then be treated with long-term metranidazole or ciprofloxin. some patients may require further diagnostic imaging such as endoscopic ultrasound (EUS) or pelvic magnetic resonance imaging (MRI).Schwartz DA. longterm drug therapy. Dis Colon Rectum 48:1337-1342. The three essential elements of treatment are delineation and drainage. The anatomy must be delineated and drainage of infection optimized. and hastens resolution of active infection. antibiotics. Finally. In most cases. Buie WD. 2004 2. Herdman CR: Review article: The medical treatment of Crohn’s perianal fistulas. which usually requires formal evaluation under anesthesia. A vessel loop tied loosely with a suture is ideal. Kilkenny J III. Most patients should be treated with a noncutting or draining seton that keeps the secondary opening (buttock skin opening) open. Fibrin glue. Simple drainage and antibiotics are no longer considered adequate. an anal fisula plug. Aliment Pharmacol Ther 19:953-967. and evaluation of the status of intestinal Crohn’s disease. the status of intestinal Crohn’s disease should be evaluated in most patients. et al: Practice parameters for the treatment of perianal abscess and fistula-in-ano (revised). If the anatomy is unclear after surgical evaluation. and immunosuppression. and/or advancement flap may result in a healed fistula. REFERENCES 1.Whiteford MH. 2005 page 1222 . Aggressive treatment of intestinal Crohn’s likely helps with the management of perianal fistula disease.American College of Surgeons SESAP 13 Critique for Category 15 Item 17 Current recommendations for treatment of patients with Crohn’s disease and perianal infections include aggressive surgical evaluation and treatment. necrotic purple skin lesions on her proximal thigh that are very painful. A picture of one of her thigh lesions is shown.American College of Surgeons SESAP 13 Category 15 Item 18 A 42-year-old woman has mottled. She has insulin-dependent diabetes and is dialyzed through a brachiocephalic fistula for chronic renal failure. An initial evaluation for these skin lesions would include all of the following EXCEPT (A) skin punch biopsy (B) serum electrolyte levels (C) coagulation panel (D) angiogram of the affected extremity (E) wound cultures page 1223 . et al: Parathyroidectomy promotes wound healing and prolongs survival in patients with calciphylaxis from secondary hyperparathyroidism. anticoagulation is not effective. To make the diagnosis of calciphylaxis. Surgery 130:645-651. Nicol TL. It is an absolute indication for urgent parathyroidectomy.Duffy A. Chen H: The role of intraoperative parathyroid hormone testing in patients with tertiary hyperparathyroidism after renal transplantation. Vascular bypass procedures are not indicated. Most series have shown significant wound healing after parathyroidectomy. Warner T. 2001 3. An angiogram is not indicated in the evaluation of these patients. Schurr M. Mack E. Harmon JW.Girotto JA. When calciphylaxis is secondary to hyperparathyroidism. REFERENCES 1. Other causes of calciphylaxis include a hypercoagulable state. Ratner LE. Chen H: Long-term outcomes in patients with calciphylaxis from hyperparathyroidism. 2006 2. along with serum parathormone (PTH) levels to document the hyperparathyroidism. Starling JR. which prolongs survival in such patients. Surgery 138:1066-1071. Wound cultures may be helpful to determine whether suprainfection of the skin lesions has occurred. Ann Surg Oncol 13:96-102. The most appropriate treatment is subtotal or total parathyroidectomy to reduce the parathyroid hormone levels. Calciphylaxis is a potentially life-threatening condition that occurs in 2% to 4% of all patients with secondary hyperparathyroidism.Haustein SV. 2005 page 1224 . Excision and grafting is effective after but not before parathyroidectomy. a skin punch biopsy is required.American College of Surgeons SESAP 13 Critique for Category 15 Item 18 The patient in this scenario has calciphylaxis due to secondary hyperparathyroidism. The most appropriate therapy for the physician would be (A) zidovudine (B) zidovudine and lamivudine (C) zidovudine. lamivudine. and nelfinavir (D) indinavir and nelfinavir (E) stavudine page 1225 .American College of Surgeons SESAP 13 Category 15 Item 19 A physician punctures his finger with a suture needle while securing a drain tube in a patient with AIDS. Guidelines developed for healthcare workers after exposure include treatment of the exposed surface with soap and water. Animal studies have demonstrated that early initiation of PEP and small inoculum size correlate with successful prevention of transmission. REFERENCES 1. but can still be beneficial up to 2 weeks after exposure. there has been great concern about transmission of blood-borne pathogens to healthcare workers. Stavudine is a NRTI and indinavir is a proteas inhibitor that cannot be used as stand-alone therapies. MMWR Morb Mortal Wkly Rep 50:444-445. testing the source patient if the status is unknown.09%.Centers for Disease Control and Prevention (CDC): The 20th year of AIDS: a time to re-energize prevention.Bell DM: Occupational risk of human immunodeficiency virus infection in healthcare workers: an overview.Gerberding JL: Management of occupational exposures to blood-borne viruses. 2001 3. The CDC has developed guidelines for evaluating the risk of HIV transmission after occupational exposure. N Engl J Med 332:444-451. respectively. its use is not justified for exposures that pose a negligible risk for transmission. depending on the volume and duration of exposure. Nelfinavir is a protease inhibitor that prevents replication of HIV and should be used in combination with at least two other anti-HIV drugs to increase efficacy. Highest risk percutaneous exposures involve deep tissue penetration with hollow-bore needles after use in veins or arteries from the source patient. Because PEP is potentially toxic. Healthcare providers must strive to balance the risk for infection against the potential toxicity of the agent(s) used when selecting a drug regimen for HIV PEP.American College of Surgeons SESAP 13 Critique of Category 15 Item 19 Since the first report in 1984 of a healthcare worker developing HIV infection after a needlestick injury. In healthcare workers with high-risk exposures. Also. HCV. a 3-drug regimen such as zidovudine. Healthcare workers with moderate-risk exposures should receive 2 antiviral drugs. 1995 4. zidovudine and lamivudine).US Public Health Service: Updated US Public Health Service Guidelines for the Management of Occupational Exposures to HBV. PEP should be started immediately. The estimated risks for HIV transmission after injury through a needle contaminated with HIVinfected blood or after mucus membrane exposure are 0. and HIV and Recommendations for Postexposure Prophylaxis. and nelfinavir is recommended. The combination therapy is usually 2 nucleoside reverse transcriptase inhibitors (NRTIs) that prevent HIV from infecting uninfected cells in the body. Am J Med 102:9-15. Percutaneous exposures are considered moderate to high risk. preferably within 1 hour after exposure.3% and 0. 2001 page 1226 . insufficient evidence exists to support recommending a 3-drug regimen for all HIV exposures. This would be the recommendation for this case. Retrospective cohort studies among healthcare workers indicate that those who used oral zidovudine as PEP had an 81% reduction in HIV infection. 1997 2. lamivudine. MMWR Recomm Rep 50:1-52. Mucus membrane exposures or exposure to skin with open wounds is considered low to moderate risk. and post-exposure prophylaxis (PEP). but they do not help cells already infected with the virus (eg. This physician has experienced a moderate-risk exposure. documentation of the data. The ulcer bed is red and inflamed and it weeps a small amount of purulent fluid. Which of the following statements about this disease is TRUE? (A) Biopsies should be performed (B) Histologic findings are pathognomonic (C) Re-siting the stoma is the most effective treatment (D) It is best treated with both intravenous and topical antibiotics (E) It often responds to anti-TNF antibody therapy page 1227 . Ten months postoperatively she develops a painful ulceration near her ileostomy. The edges of the ulcer are purple in color and the overall appearance is consistent with pyoderma gangrenosum. Recovery is unremarkable and she manages the ileostomy without difficulty.American College of Surgeons SESAP 13 Category 15 Item 20 A 28-year-old woman has a total proctocolecotmy for Crohn’s colitis. Sapienza MS. et al: Infliximab for treatment of pyoderma gangrenosum associated with inflammatory bowel disease.Sheldon DG. the red punched-out ulceration with a purple or violaceous edge is diagnostic. Some authors have argued that biopsies should be avoided because the trauma from the biopsy may exacerbate the lesion. 2001 4. Sawchuk LL. Thirlby RC. but are of unclear benefit. 2003 2. Biopsies are not required. but re-siting should be considered if the tissue surrounding the stoma is so damaged that application of the stoma appliance is not feasible. Topical tacrolimus has some value in helping to heal these lesions. Re-siting the stoma should be undertaken only with caution. Even when biopsies are obtained. REFERENCES 1. Dig Dis Sci 49:1454-1457. Topical antibiotics can also be used. J Am Coll Surg 193:703. an anti-TNF antibody. Kozarek RA. Plevy S. Treatment options include meticulous application of the appliance to achieve a lasting seal. because pyoderma gangrenosum can recur rapidly at the new ostomy site. The current preferred treatment for a peristomal pyoderma gangrenosum.American College of Surgeons SESAP 13 Critique for Category 15 Item 20 Peristomal pyoderma gangrenosum is a rare debilitating complication. the histologic findings of pyoderma are nonspecific rather than diagnostic.Regueiro M. and topical corticosteroids. 2004 3. Intravenous antibiotics are of no value. Fleisher MR. is intravenous infliximab. and make obtaining a satisfactory seal on the stoma appliance very difficult. Valentine J. typically seen in Crohn’s patients with ileostomies or colostomies. Oral and intravenous cyclosporins are also effective at healing many of these lesions. however. Dimarino AJ: Treatment of pyoderma gangrenosum with infliximab in Crohn’s disease. Although the precise etiology of these lesions is not known. Pyoderma gangrenosum located around the ostomy site is often difficult to resolve because the irritation of the stoma appliance and the stoma effluent are a source of constant minor trauma to the area. Am J Gastroenterol 98:1821-1826. These lesions are often painful. Arch Surg 135:564-569. Diagnosis of peristomal pyoderma gangrenosum is primarily based on inspection of the lesion. For the experienced observer. Thirlby RC: Twenty cases of peristomal pyoderma gangrenosum: diagnostic implications and management.Sheldon D. it is a common observation that pyoderma gangrenosum can be initiated by minor skin trauma. 2000 page 1228 . Cohen S. Kozarek R: Peristomal pyoderma gangrenosum. American College of Surgeons SESAP 13 Category 15 Item 21 Hyperbaric oxygen (A) promotes regional vasodilatation. enhancing the microcirculation (B) has a direct toxic effect on Clostridia species (C) promotes a ten-fold increase in oxygen diffusion through tissue fluids (D) reduces the production of oxygen free radicals in ischemic tissue (E) is effective treatment of Fournier’s gangrene page 1229 . However. Cronier P.Bouachour G.Korhonen K: Hyperbaric oxygen therapy in acute necrotizing infections with a special reference to the effects on tissue gas tensions.Fry DE: The story of hyperbaric oxygen continues. 1994 page 1230 . The decreased transcapillary fluid exudation reduces interstitial pressure and tissue edema.American College of Surgeons SESAP 13 Critique for Category 15 Item 21 Hyperbaric oxygen (HBO). delivered at pressures of 2 to 3 atmospheres absolute (ATA). and capillary angiogenesis. despite increasing advocacy and hopeful clinical reports.Kindwall EP. REFERENCES 1. Microcirculation is improved. 1991 5. type I clinical data with appropriate statistical power demonstrating treatment efficacy in Fournier’s gangrene. leukocyte activation. et al: Hyperbaric oxygen therapy in the management of crush injuries: a randomized double-blind placebo-controlled clinical trial. It also enhances fibroblast differentiation and collagen synthesis. Some of these effects have been demonstrated in laboratory models and in clinical trials. Withey S. 1996 2. Gottlieb LJ. Plast Reconstr Surg 88:898-908. 2000 6. Am J Surg 189:467-468. or ischemic tissues. HBO induces production of toxic oxygen free radicals that have a direct lethal effect on Clostridia species. or any other disorder to which the technique has been applied. One would expect these demonstrated physiologic effects to translate into dramatic results with critically ill patients with crushed. improvements in the microcirculation. Toulemonde JL. direct antimicrobial effects (against anaerobes). infected. blood oxygenation is increased 10-fold. Meyer AA: Necrotizing soft-tissue infections. Ann Chir Gynaecol (Suppl) 214:7-36.Jallali N. Surg Clin North Am 74:591-607. Am J Surg 189:462-466. thereby mitigating tissue ischemia. has been proposed by many clinicians and investigators as an important adjunct to the treatment of a variety of soft tissue infections and injuries. 2005 4. 2005 3. are lacking. Theoretical beneficial effects of an enhanced oxygen environment include reversal of local ischemia. J Trauma 41:333-339. and diffusion of oxygen through tissue fluids is increased 3fold. Larson DL: Hyperbaric oxygen therapy in plastic surgery: a review article. At 2 ATA. reduction of endotoxin elaboration by microorganisms. Butler PE: Hyperbaric oxygen as adjuvant therapy in the management of necrotizing fasciitis. Gouello JP.Sutherland ME. American College of Surgeons SESAP 13 Category 15 Item 22 Treatment of wounds with negative-pressure wound dressings (VAC dressings) may result in all of following EXCEPT (A) increased apoptosis (B) increased granulation tissue (C) improved microvascular blood flow to wound edges (D) removal of excess fluid and debris (E) lower bacterial counts page 1231 . Therapy increases microvascular blood flow a few centimeters from the wound edge. The blood flow increase can be influenced by the amount of negative pressure applied and may be dependent on the type of tissue and the location in the wound. Increases in blood flow 40% to 50% above baseline are reported. decreasing interstitial edema. REFERENCES 1. The pressure can be applied constantly or intermittently. increasing granulation tissue. BMJ 332: 900-903. Harding KG: ABC of wound healing: Non-surgical and drug treatments. In addition to negative pressure. Negative pressure wound therapy clearly increases blood flow to the wound. Another mechanism promoting wound healing is related to the ability of the negative pressure therapy to exert micromechanical forces on cells. Stretching cells induces cell proliferation and division. Proposed mechanisms include removing excessive fluid and debris. The VAC dressing consists of foam applied over a wound and covered by an occlusive dressing. The device should be used with caution when wounds have exposed vessels or bowel. Intermittent pressure may be more beneficial than constant. Guidelines exist for using the device based on the type of wound and tissue type. and increasing capillary blood flow. available at www. while causing a hypoperfusion at the wound edges that is directly related to amount of negative pressure.No Authors Listed: VAC Therapy Clinical Guidelines: A reference source for clinicians (December 2006). The mechanism by which negative pressure wound therapy via the VAC dressing works is unknown.kci1. which is then connected to a pump that applies intermittent negative pressure across the wound. These dressing conform well and remain adherent. the dressing provides a humid environment facilitating wound healing. Application of the device is fairly easy and if a seal is maintained dressing changes are usually minimized when compared with other methods of wound care. Current devices come in many different sizes and are used for in-patient and out-patient therapy.pdf page 1232 . 2006 2. Grey JE. This effect is similar to the distraction osteogenesis associated with the Ilizarov device.com/2-B-128_Clin_Guidelines_Blue_Book_1-05. This differential is suspected to enhance the development of granulation tissue in the wound bed.Enoch S. decreasing bacterial counts. No evidence exists that this therapy increases apoptosis. The best effect appears to be associated with soft tissue defects.American College of Surgeons SESAP 13 Critique for Category 15 Item 22 Vacuum-assisted closure (VAC) dressings allow negative pressure to be applied to a wound. All of the following would be appropriate initial management EXCEPT (A) avoiding antiperspirants/deodorants (B) tetracycline (C) isotretinoin (Accutane) (D) triamcinolone injection (E) excision of the affected area page 1233 .American College of Surgeons SESAP 13 Category 15 Item 23 A 29-year-old man has the lesion shown in his axilla. Blackwell Publishing. Cox N. 2003 page 1234 . Br J Plast Surg 56:451-461. Oxford. Foreign substances that may occlude ductal structures in the affected region should be eliminated. Topical retinoids and intralesional corticosteroids have been used successfully to quell the inflammatory responses associated with HS. Adriaans BM: Bacterial infections. Griffiths C (eds): Rook’s Textbook of Dermatology.American College of Surgeons SESAP 13 Critique for Category 15 Item 23 The photo in this question shows hidradenitis suppurativa (HS). The pustules should be drained in addition to starting the patient on antibiotics.Hay RJ. medical management should be pursued initially. The patient should avoid using antiperspirant or deodorant. Management of HS is both medical and surgical. ed 7. 2004. Antibiotic therapy is given to control infection and anti-inflammatory medication is initiated. a poorly understood disease process affecting the apocrine sweat glands that ultimately leads to multiple infected cysts and draining sinuses in hair-bearing regions. REFERENCES 1. but are not curative.Slade DE. in Burns T. These therapies will suppress HS. Definitive cure will require surgical excision at some point in the disease process when active infection is not present. Because surgical therapy in the face of active infection is associated with significant complications. Tetracycline and clindamycin have both been used successfully to at least suppress and sometimes completely treat infections. Powell BW. Breathnach S. pp 82-85 2. Mortimer PS: Hidradenitis suppurativa: pathogenesis and management. American College of Surgeons SESAP 13 Category 15 Item 24 Impetigo (A) Staphylococcus aureus skin infection (B) Streptococcus pyogenes skin infection (C) Both (D) Neither page 1235 . but is particularly likely to begin in places with impaired lymphatic drainage such as the lymphedematous arm after axillary dissection. 1996 2. Erysipelas. localized warmth. surgical debridement or radical surgical excision becomes necessary. a deepseated. frequently associated with overlying bullae. the inflammation is often raised above the skin and clearly demarcated from adjacent normal tissues. Everett ED. Erysipelas may occur anywhere. is characterized by rapid onset. Clin Infect Dis 41:1373-1406. including aerobes with or without group A Streptococcus. Semin Dermatol 12:331-335. et al: Practice guidelines for the diagnosis and management of skin and soft-tissue infections.Feingold DS: Staphylococcal and streptococcal pyrodermas. but most cases are characterized by polymicrobial infections (type I necrotizing fasciitis). Because erysipelas involves superficial layers of the skin and its lymphatic drainage.Bisno AL. 2005 page 1236 . almost like a map. Stevens DL: Streptococcal infections of the skin and soft tissues. Impetigo consists of superficial ulcerations. fever.Stevens DL. erythema and pain. Bullous impetigo is almost always caused by S aureus that is responsive to mupirocin and often does not require systemic antibiotics. Streptococcus pyogenes may be the sole agent responsible for necrotizing fasciitis (type II). Chambers HF. but when local tissue necrosis or invasive necrosis with spread to adjacent normal tissues supervenes. and leukocytosis. The rapid onset of shock and organ failure may be related to streptococcal toxic shock syndrome. Streptococcal skin infection may cause acute glomerulonephritis. 1993 3. a rapidly progressing infection of the skin by group A beta-hemolytic Streptococcus. In most cases.American College of Surgeons SESAP 13 Critique for Category 15 Item 24 Both Staphylococcus aureus and Streptococcus pyogenes are capable of causing skin infections. antibiotic therapy and local wound care are adequate treatment. REFERENCES 1. rapidly progressing infection of fascia also involving subcutaneous fat. A nonbullous form of impetigo may be caused by either Staphylococcus or Streptococcus. Pharyngitis with group A Streptococcus is commonly associated with rheumatic fever. N Engl J Med 4:240245. Bisno AL. It is almost always responsive to penicillin G. not the deeper layers as seen in cellulitis. American College of Surgeons SESAP 13 Category 15 Item 25 Erysipelas (A) Staphylococcus aureus skin infection (B) Streptococcus pyogenes skin infection (C) Both (D) Neither page 1237 . localized warmth. It is almost always responsive to penicillin G. 1993 3. Because erysipelas involves superficial layers of the skin and its lymphatic drainage. Stevens DL: Streptococcal infections of the skin and soft tissues. but when local tissue necrosis or invasive necrosis with spread to adjacent normal tissues supervenes. antibiotic therapy and local wound care are adequate treatment. The rapid onset of shock and organ failure may be related to streptococcal toxic shock syndrome. et al: Practice guidelines for the diagnosis and management of skin and soft-tissue infections. Chambers HF.Stevens DL. not the deeper layers as seen in cellulitis. is characterized by rapid onset. frequently associated with overlying bullae. including aerobes with or without group A Streptococcus. Impetigo consists of superficial ulcerations. fever. the inflammation is often raised above the skin and clearly demarcated from adjacent normal tissues. and leukocytosis. A nonbullous form of impetigo may be caused by either Staphylococcus or Streptococcus. Bullous impetigo is almost always caused by S aureus that is responsive to mupirocin and often does not require systemic antibiotics. Streptococcus pyogenes may be the sole agent responsible for necrotizing fasciitis (type II). 1996 2. a deepseated.Feingold DS: Staphylococcal and streptococcal pyrodermas. Streptococcal skin infection may cause acute glomerulonephritis. surgical debridement or radical surgical excision becomes necessary.Bisno AL. erythema and pain. Pharyngitis with group A Streptococcus is commonly associated with rheumatic fever. 2005 page 1238 . Erysipelas may occur anywhere. a rapidly progressing infection of the skin by group A beta-hemolytic Streptococcus. Bisno AL.American College of Surgeons SESAP 13 Critique for Category 15 Item 25 Both Staphylococcus aureus and Streptococcus pyogenes are capable of causing skin infections. but is particularly likely to begin in places with impaired lymphatic drainage such as the lymphedematous arm after axillary dissection. REFERENCES 1. Semin Dermatol 12:331-335. but most cases are characterized by polymicrobial infections (type I necrotizing fasciitis). rapidly progressing infection of fascia also involving subcutaneous fat. Clin Infect Dis 41:1373-1406. N Engl J Med 4:240245. In most cases. almost like a map. Erysipelas. Everett ED. American College of Surgeons SESAP 13 Category 15 Item 26 Postmastectomy lymphangitis (A) Staphylococcus aureus skin infection (B) Streptococcus pyogenes skin infection (C) Both (D) Neither page 1239 . It is almost always responsive to penicillin G. A nonbullous form of impetigo may be caused by either Staphylococcus or Streptococcus. localized warmth. Pharyngitis with group A Streptococcus is commonly associated with rheumatic fever. not the deeper layers as seen in cellulitis. 1996 2. the inflammation is often raised above the skin and clearly demarcated from adjacent normal tissues. 2005 page 1240 . Stevens DL: Streptococcal infections of the skin and soft tissues. Impetigo consists of superficial ulcerations. Everett ED. but most cases are characterized by polymicrobial infections (type I necrotizing fasciitis). a deepseated. Streptococcus pyogenes may be the sole agent responsible for necrotizing fasciitis (type II). erythema and pain. REFERENCES 1. et al: Practice guidelines for the diagnosis and management of skin and soft-tissue infections. rapidly progressing infection of fascia also involving subcutaneous fat. including aerobes with or without group A Streptococcus. is characterized by rapid onset. Erysipelas may occur anywhere. fever. Clin Infect Dis 41:1373-1406.American College of Surgeons SESAP 13 Critique of Category 15 Item 26 Both Staphylococcus aureus and Streptococcus pyogenes are capable of causing skin infections. Bullous impetigo is almost always caused by S aureus that is responsive to mupirocin and often does not require systemic antibiotics. Bisno AL. The rapid onset of shock and organ failure may be related to streptococcal toxic shock syndrome. Semin Dermatol 12:331-335. 1993 3. frequently associated with overlying bullae. surgical debridement or radical surgical excision becomes necessary. N Engl J Med 4:240245. a rapidly progressing infection of the skin by group A beta-hemolytic Streptococcus. and leukocytosis. Chambers HF. but when local tissue necrosis or invasive necrosis with spread to adjacent normal tissues supervenes. Erysipelas.Stevens DL.Feingold DS: Staphylococcal and streptococcal pyrodermas. almost like a map. In most cases. Because erysipelas involves superficial layers of the skin and its lymphatic drainage. Streptococcal skin infection may cause acute glomerulonephritis.Bisno AL. antibiotic therapy and local wound care are adequate treatment. but is particularly likely to begin in places with impaired lymphatic drainage such as the lymphedematous arm after axillary dissection. American College of Surgeons SESAP 13 Category 15 Item 27 Postoperative adjuvant radiotherapy (A) Merkel cell carcinoma (B) Malignant melanoma (C) Both (D) Neither page 1241 . American College of Surgeons SESAP 13 Critique for Category 15 Item 27 Merkel cell carcinoma is an uncommon neuroendocrine carcinoma arising in the Merkel cells or mechanoreceptor cells of the base of the epidermis. These findings are used to recommend adjuvant radiotherapy of the primary site and lymph node basin for Merkel cell carcinoma. et al: Merkel cell carcinoma: treatment and outcomes. Merkel cell carcinomas are often aggressive lesions with a 5-year survival rate in the range of 50% compared with > 80% for malignant melanoma. et al: Current therapy of cutaneous melanoma. 1990 2. Morris CG. which arises in the melanocytes of the epidermis. Like malignant melanoma. it occurs most commonly in areas exposed to the sun. REFERENCES 1. Up to 25% of patients with Merkel cell carcinoma initially present with lymph node metastases compared with 10% of patients with malignant melanoma. Werning JW. 2000 page 1242 . It is uncommon in primary malignant melanoma. Although the primary therapy for both malignant melanoma and Merkel cell carcinoma is wide local excision. Coleman JJ. Gordon MS. which is rarely used in malignant melanoma. occurring more frequently in late stage primary disease or postsurgical recurrence. Mendenhall CM. Otolaryngol Clin North Am 23:845-864. Satellitosis is found in 11% of patients with Merkel cell carcinoma at initial diagnosis. Merkel cell lesions have a higher incidence of metastatic lymphadenopathy. Cancer 104:1761-1764.Wagner JD. Chuang TY. 2005 3. Plast Reconstr Surg 105:1774-1799.McAfee WJ.Hruza GJ: Mohs micrographic surgery. American College of Surgeons SESAP 13 Category 15 Item 28 Wide local excision (A) Merkel cell carcinoma (B) Malignant melanoma (C) Both (D) Neither page 1243 . Gordon MS. 2000 page 1244 . Chuang TY. it occurs most commonly in areas exposed to the sun. Merkel cell lesions have a higher incidence of metastatic lymphadenopathy. Morris CG.Hruza GJ: Mohs micrographic surgery. Werning JW. Like malignant melanoma. Cancer 104:1761-1764. 2005 3. REFERENCES 1. Mendenhall CM.McAfee WJ. Although the primary therapy for both malignant melanoma and Merkel cell carcinoma is wide local excision. Otolaryngol Clin North Am 23:845-864. et al: Current therapy of cutaneous melanoma. These findings are used to recommend adjuvant radiotherapy of the primary site and lymph node basin for Merkel cell carcinoma. Satellitosis is found in 11% of patients with Merkel cell carcinoma at initial diagnosis. Up to 25% of patients with Merkel cell carcinoma initially present with lymph node metastases compared with 10% of patients with malignant melanoma. which is rarely used in malignant melanoma. 1990 2.American College of Surgeons SESAP 13 Critique for Category 15 Item 28 Merkel cell carcinoma is an uncommon neuroendocrine carcinoma arising in the Merkel cells or mechanoreceptor cells of the base of the epidermis. Plast Reconstr Surg 105:1774-1799. Merkel cell carcinomas are often aggressive lesions with a 5-year survival rate in the range of 50% compared with > 80% for malignant melanoma. et al: Merkel cell carcinoma: treatment and outcomes. It is uncommon in primary malignant melanoma. which arises in the melanocytes of the epidermis. occurring more frequently in late stage primary disease or postsurgical recurrence. Coleman JJ.Wagner JD. American College of Surgeons SESAP 13 Category 15 Item 29 > 20% of patients present with regional lymph node metastases (A) Merkel cell carcinoma (B) Malignant melanoma (C) Both (D) Neither page 1245 . 2005 3. it occurs most commonly in areas exposed to the sun. Plast Reconstr Surg 105:1774-1799.Wagner JD. Although the primary therapy for both malignant melanoma and Merkel cell carcinoma is wide local excision. Mendenhall CM. Merkel cell lesions have a higher incidence of metastatic lymphadenopathy. which arises in the melanocytes of the epidermis. It is uncommon in primary malignant melanoma.Hruza GJ: Mohs micrographic surgery. occurring more frequently in late stage primary disease or postsurgical recurrence. Morris CG. 2000 page 1246 . Coleman JJ. Merkel cell carcinomas are often aggressive lesions with a 5-year survival rate in the range of 50% compared with > 80% for malignant melanoma.McAfee WJ. Up to 25% of patients with Merkel cell carcinoma initially present with lymph node metastases compared with 10% of patients with malignant melanoma. Chuang TY. Cancer 104:1761-1764. These findings are used to recommend adjuvant radiotherapy of the primary site and lymph node basin for Merkel cell carcinoma. which is rarely used in malignant melanoma. Werning JW. Gordon MS.American College of Surgeons SESAP 13 Critique for Category 15 Item 29 Merkel cell carcinoma is an uncommon neuroendocrine carcinoma arising in the Merkel cells or mechanoreceptor cells of the base of the epidermis. 1990 2. et al: Current therapy of cutaneous melanoma. REFERENCES 1. Satellitosis is found in 11% of patients with Merkel cell carcinoma at initial diagnosis. et al: Merkel cell carcinoma: treatment and outcomes. Like malignant melanoma. Otolaryngol Clin North Am 23:845-864. then select the best answer.American College of Surgeons SESAP 13 Category 15 Item 30 Please view the images. (A) Burn center (B) Community hospital (C) Both (D) Neither page 1247 . systemic disease. and patients who will require special social. other associated injuries or inhalation injury. or long-term rehabilitation A child with circumferential hand burns (item 30) is at risk for infection and functional impairment. WebMD Inc. Jurkovich GJ. the depth and severity of burn cannot be determined until the tar is removed. Discharge within 24 hours would be likely. REFERENCES 1.American College of Surgeons SESAP 13 Critique for Category 15 Item 30 In the ACS document Optimal Care of the Injured Patient. feet. and assuage guilt of the parents would be the best choice. third-degree burns in any age group. Because this is rarely known beforehand. He can easily be treated with ointment. He clearly meets the ABA/ACS burn center referral criteria. chap 7. and a light dressing. Kaiser LR.Gibran NS. burned children in hospitals without qualified personnel or equipment for the care of children. emotional. the guidelines for burn center referral include: partial thickness burns > 10%. Social services and perhaps child protective services may be involved. in Souba WW. In a patient who has a tar burn on the face (item 32). Fink MP. A healthy man with a shallow burn in a noncosmetic area (item 31) does not meet any of these criteria. not feeling guilty. A child who has a shallow scald burn in a nonfunctional area (item 33) could probably be treated as an outpatient if the family were functional. and a burn center is the optimal place to achieve the best result. Further. chemical burns. or perineum. sec 13. electrical burns > 220 volts with tissue loss. bathing. a circumferential scald is always suspect as to cause. vision and even his eye is at risk from poor healing. If the lids are burned deeply. New York. Trauma and Thermal Injury. face. and capable of providing good wound care. et al (eds): ACS Surgery: Principles and Practice 2006. He should be healed within a week or so. Heimbach DM: Management of the patient with thermal injuries. provide adequate pain management. a brief admission to the community hospital to teach wound care. pp 1295-1306 page 1248 . burns to hands. American College of Surgeons SESAP 13 Category 15 Item 31 Please view the image. (A) Burn center (B) Community hospital (C) Both (D) Neither page 1249 . then select the best answer. Trauma and Thermal Injury. vision and even his eye is at risk from poor healing. a circumferential scald is always suspect as to cause. Kaiser LR. face. electrical burns > 220 volts with tissue loss. New York. Jurkovich GJ. Discharge within 24 hours would be likely. a brief admission to the community hospital to teach wound care. Social services and perhaps child protective services may be involved. and a burn center is the optimal place to achieve the best result. et al (eds): ACS Surgery: Principles and Practice 2006. not feeling guilty. He can easily be treated with ointment. Further. bathing. chemical burns. or perineum. If the lids are burned deeply. and assuage guilt of the parents would be the best choice. the guidelines for burn center referral include: partial thickness burns > 10%. A child who has a shallow scald burn in a nonfunctional area (item 33) could probably be treated as an outpatient if the family were functional. A healthy man with a shallow burn in a noncosmetic area (item 31) does not meet any of these criteria. WebMD Inc. Heimbach DM: Management of the patient with thermal injuries. and a light dressing. REFERENCES 1.American College of Surgeons SESAP 13 Critique for Category 15 Item 31 In the ACS document Optimal Care of the Injured Patient. He should be healed within a week or so. and patients who will require special social. burned children in hospitals without qualified personnel or equipment for the care of children. in Souba WW. provide adequate pain management. systemic disease. feet. In a patient who has a tar burn on the face (item 32). or long-term rehabilitation A child with circumferential hand burns (item 30) is at risk for infection and functional impairment. He clearly meets the ABA/ACS burn center referral criteria. and capable of providing good wound care. emotional. chap 7. third-degree burns in any age group. Because this is rarely known beforehand.Gibran NS. sec 13. burns to hands. Fink MP. pp 1295-1306 page 1250 . other associated injuries or inhalation injury. the depth and severity of burn cannot be determined until the tar is removed. (A) Burn center (B) Community hospital (C) Both (D) Neither page 1251 . then select the best answer.American College of Surgeons SESAP 13 Category 15 Item 32 Please view the image. et al (eds): ACS Surgery: Principles and Practice 2006. Because this is rarely known beforehand. provide adequate pain management. Heimbach DM: Management of the patient with thermal injuries. Fink MP. feet. electrical burns > 220 volts with tissue loss. vision and even his eye is at risk from poor healing. third-degree burns in any age group. sec 13. and a burn center is the optimal place to achieve the best result. Discharge within 24 hours would be likely. Social services and perhaps child protective services may be involved. A healthy man with a shallow burn in a noncosmetic area (item 31) does not meet any of these criteria. WebMD Inc. or perineum. emotional. REFERENCES 1. Kaiser LR. He can easily be treated with ointment. in Souba WW. the guidelines for burn center referral include: partial thickness burns > 10%. Jurkovich GJ. He should be healed within a week or so.American College of Surgeons SESAP 13 Critique for Category 15 Item 32 In the ACS document Optimal Care of the Injured Patient. not feeling guilty. burned children in hospitals without qualified personnel or equipment for the care of children. burns to hands. Trauma and Thermal Injury. New York. and patients who will require special social. Further. chemical burns. or long-term rehabilitation A child with circumferential hand burns (item 30) is at risk for infection and functional impairment. the depth and severity of burn cannot be determined until the tar is removed. other associated injuries or inhalation injury. and capable of providing good wound care. face. a brief admission to the community hospital to teach wound care. In a patient who has a tar burn on the face (item 32).Gibran NS. bathing. and a light dressing. He clearly meets the ABA/ACS burn center referral criteria. A child who has a shallow scald burn in a nonfunctional area (item 33) could probably be treated as an outpatient if the family were functional. and assuage guilt of the parents would be the best choice. a circumferential scald is always suspect as to cause. If the lids are burned deeply. chap 7. systemic disease. pp 1295-1306 page 1252 . then select the best answer. (A) Burn center (B) Community hospital (C) Both (D) Neither page 1253 .American College of Surgeons SESAP 13 Category 15 Item 33 Please view the image. systemic disease. third-degree burns in any age group. a circumferential scald is always suspect as to cause. emotional. and a light dressing. vision and even his eye is at risk from poor healing. Fink MP. A healthy man with a shallow burn in a noncosmetic area (item 31) does not meet any of these criteria. Kaiser LR. He can easily be treated with ointment. chap 7. burned children in hospitals without qualified personnel or equipment for the care of children. sec 13. In a patient who has a tar burn on the face (item 32). Social services and perhaps child protective services may be involved. Jurkovich GJ. He should be healed within a week or so. pp 1295-1306 page 1254 . et al (eds): ACS Surgery: Principles and Practice 2006. electrical burns > 220 volts with tissue loss. provide adequate pain management.Gibran NS. the guidelines for burn center referral include: partial thickness burns > 10%. A child who has a shallow scald burn in a nonfunctional area (item 33) could probably be treated as an outpatient if the family were functional.American College of Surgeons SESAP 13 Critique for Category 15 Item 33 In the ACS document Optimal Care of the Injured Patient. Because this is rarely known beforehand. and capable of providing good wound care. burns to hands. in Souba WW. and assuage guilt of the parents would be the best choice. face. not feeling guilty. Further. Discharge within 24 hours would be likely. bathing. or perineum. or long-term rehabilitation A child with circumferential hand burns (item 30) is at risk for infection and functional impairment. WebMD Inc. He clearly meets the ABA/ACS burn center referral criteria. a brief admission to the community hospital to teach wound care. Heimbach DM: Management of the patient with thermal injuries. the depth and severity of burn cannot be determined until the tar is removed. and a burn center is the optimal place to achieve the best result. New York. other associated injuries or inhalation injury. REFERENCES 1. Trauma and Thermal Injury. If the lids are burned deeply. and patients who will require special social. feet. chemical burns. American College of Surgeons SESAP 13 Category 16 Item 1 According to the American College of Cardiology. which of the following is NOT a high cardiac risk surgical procedure? (A) Carotid endarterectomy (B) Exploratory laparotomy for perforated diverticulitis in an elderly patient (C) Aortobifemoral operation (D) Femoral-popliteal bypass (E) Repair of strangulated hernia with bowel obstruction page 1255 . Most vascular operations are higher risk because of the likelihood of associated cardiac disease. orthopaedic surgery. cataract. precordial auscultation.acc.American College of Surgeons SESAP 13 Critique for Category 16 Item 1 In 2002 the American College of Cardiology and the American Heart Association practice guidelines task force updated their 1996 perioperative cardiovascular evaluation for non-cardiac surgery guidelines. abdominal palpation. Petersen NJ. www. The history should also include a determination of the patient’s functional capacity and delineation of clinical predictors of increased risk (see table).Ashton CM. carotid pulse contour and bruits. and breast surgeries. and prostate surgery.org page 1256 . A complete cardiovascular examination can identify subtle disease that may not be forthcoming in the history. Ideally this examination should consist of vital signs (measuring blood pressure in both arms). Emergency major operations in the elderly or prolonged surgical procedures associated with large fluid shifts and/or blood loss place patients in the high-risk category. 1993 2. Wray NP. et al: The incidence of perioperative myocardial infarction in men undergoing noncardiac surgery. Those procedures that fall in the high-risk category infer a cardiac risk greater than 5% of fatal and nonfatal myocardial infarctions. intraperitoneal/intrathoracic surgery. Gibbons RJ. Those procedures that fall into the intermediate-risk category (cardiac risk less than 5%) include: carotid endarterectomy. Stratifying risk for various types of noncardiac surgical procedures has been reported and based on multiple studies. et al: ACC/AHA Guideline Update on peri-operative cardiovascular evaluation for non-cardiac surgery. Repair of a strangulated hernia with bowel obstruction will likely involve major fluid shifts. Ann Intern Med 118:504-510. Kiefe CI. 2002. REFERENCES 1. A careful history is the first step in identifying cardiac and/or co-morbid diseases that place the patient in a high surgical risk category.Eagle KA. A report of the American College of Cardiology/American Heart Association Task Force on Practice Guidelines. and examination of the extremities for edema and vascular perfusion. Antman EM. auscultation of the lungs. Low-risk procedures (less than 1% cardiac risk) include endoscopic and superficial procedures. jugular venous pressure and pulsations. The guidelines provide an evidenced-based framework for considering cardiac risk of noncardiac surgery in a variety of patient and surgical situations. Formal preoperative cardiac consultation is tailored to each patient. head/neck surgery. administering 0.5% bupivacaine without epinephrine into the area of the wound (including deep tissues) will (A) decrease the time to first rescue analgesia (B) increase the risk of wound complications (C) increase the use of nonsteroidal anti-inflammatory medications (D) decrease postoperative pain as measured by pain scales (E) increase narcotic analgesic consumption page 1257 .American College of Surgeons SESAP 13 Category 16 Item 2 Prior to making the incision for an emergency appendectomy. Lert-akyamanee N. Smith DS: New concepts in acute pain therapy: preemptive analgesia. Preemptive analgesia has markedly improved pain control for many ambulatory surgical procedures as well as procedures performed during hospital admission. pain scales were significantly lower both at rest and after ambulation in the patients who received preemptive analgesia. Although the effect on pain control did not decrease length of hospital stay. tonsillectomy. Given the relatively few complications. While time to first narcotic requirement was longer in the preemptive analgesia group. In a randomized controlled trial in which the study group received 10 mL of 0. REFERENCES 1. Am Family Physician 63:1979-1984. Jenkins BJ: The efficacy of preemptive analgesia for acute postoperative pain management: a meta-analysis. breast biopsy. 2004 3. retinal detachment. World J Surg 28:947-950. Rushatamukayanunt W: Efficacy of pre-incisional bupivacaine infiltration on postoperative pain relief after appendectomy: prospective double-blind randomized trial. The presumed mechanism of preemptive analgesia is prevention of nociceptive (painful) input from afferent stimulation of the central nervous system to prevent central nervous system hyperexcitability. preemptive analgesia may reasonably be used in these settings.Gottschalk A. this was not statistically different compared with the control group in this particular study. There was no increase in wound complication rate. laparoscopy. and cesarean section.5% bupivacaine without epinephrine injected into the skin and subcutaneous tissue along the proposed incision line after induction of general anesthesia along with another 10 mL of the local anesthetic infiltrated into the muscular layer after skin and subcutaneous incision. Anesth Analg 100:757-773.Lohsiriwat V.American College of Surgeons SESAP 13 Critique for Category 16 Item 2 Administering a local anesthetic prior to the skin incision is referred to as preemptive analgesia. Appendectomy is one of the most common emergency surgical procedures. 2005 page 1258 . patients subjectively reported improved pain control by this simple maneuver. Seymour RA. Preemptive analgesia has also been studied for hernia repair. Pain scores were variably affected. 2001 2. In some of these studies the reduction in time to first analgesic was statistically significant. Lirk P.Ong CK. arthroscopic knee surgery. a 21-year-old man develops the sudden onset of hypotension. cyanosis. The anesthesia gas canister is hot to the touch.American College of Surgeons SESAP 13 Category 16 Item 3 During an elective laparoscopic cholecystectomy. and flushed warm skin. Which of the following statements about his condition is NOT true? (A) Prompt identification and treatment substantially reduces mortality (B) The diagnosis is confirmed by an in vitro muscle test (C) Informing family members of this reaction is critical (D) Treatment includes administration of dantrolene sodium (E) Local anesthetics can also trigger the reaction page 1259 . tachycardia. Dantrolene side effects include muscle weakness and gastrointestinal upset. neuroleptic drugs. neurofibromatosis. Silverman DG: Optimizing postoperative outcomes with efficient preoperative assessment and management. a clean anesthesia machine is used. 2004 page 1260 . Juda R.American College of Surgeons SESAP 13 Critique for Category 16 Item 3 Malignant hyperthermia (MH) is an inherited muscle membrane disorder that can be precipitated by depolarizing muscle relaxants such as succinylcholine. and severe exercise in hot climates. Other early signs include a rise in end-tidal CO2 and increased heat production in the anesthesia canister. However. arrhythmias. Lancet 352:1131-1136. However. then pretreatment is unnecessary. Pretreatment with dantrolene in cases of suspected MH is one mechanism to reduce mortality. the mortality rate has decreased from 70% to 5% provided the diagnosis is made promptly and treatment instituted. REFERENCES 1. if a nontriggering anesthetic is administered. acidosis. muscle rigidity. This disorder may also be triggered by infectious diseases. Another possible presentation is unexplained cardiac arrest or arrhythmia shortly after induction.Denborough M: Malignant hyperthermia. and using this approach with spinal anesthesia if necessary is another way to avoid a crisis in the operating room. and dantrolene is readily available. The disease is confirmed with a muscle biopsy and positive caffeine-halothane contraction test. and strabismus. end-tidal CO2 is monitored. Because the disorder can be recognized preoperatively and can be treated with aggressive use of dantrolene sodium. In vitro muscle testing in patients who present with this sign alone reveals that 28% to 50% are susceptible to MH. 1998 2. There may be an increased likelihood of MH for patients with disorders such as muscular dystrophy. increased creatinine kinase. the strongest evidence of MH susceptibility is a history of an MH episode (with hypercarbia. and/or myoglobinuria) in the patient or a family member. Crit Care Med 32:S76-S86. One of the earliest clinical signs is masseter muscle spasm after induction of anesthesia with succinylcholine. MH is not precipitated by local anesthetics. hyperthermia.Halaszynski TM. Simultaneous contractions of skeletal muscles and/or volatile halogenated anesthesic agents alter the integrity of muscle cell membranes. American College of Surgeons SESAP 13 Category 16 Item 4 Propofol (Diprivan) (A) is not approved for use in obstetric anesthesia (B) is very allergenic (C) can precipitate a hypertensive crisis (D) can be safely used in patients with Parkinson’s disease (E) is associated with increased postoperative nausea and vomiting page 1261 . Recent consensus guidelines for managing PONV include the use of intravenous propofol as part of a multimodal strategy (other anti-emetics may also be necessary) to reduce this side effect in susceptible patients. et al: Hemodynamic effects of propofol: data from over 25. Hypotension (not hypertension) is a reported side effect of propofol that is dose related and occurs to a greater extent in elderly patients. Br J Anaesth 93:725-736. Propofol is well suited for use in the ambulatory setting. the overall incidence of hypotension was 15. Roizen MF. Because of its negative effect on neonatal outcome scores when compared with thiopental and inhalational agents.Sneyd JR: Recent advances in intravenous anaesthesia. 1993 2.American College of Surgeons SESAP 13 Critique for Category 16 Item 4 Propofol was introduced in 1986 and has rapidly become the most widely used sedative-hypnotic drug for induction and maintenance of general anesthesia. A limited number of reports suggest that Parkinson’s disease can be exacerbated by propofol by inducing myoclonic movements. REFERENCES 1.Hug CC Jr.000 patients. Bradycardia occurs less frequently at 4. The only absolute contraindication for use is allergy and this is rare.7% and this occurred during the first 10 minutes after injection. even for patients with egg allergies. In a large phase IV multicenter trial. Propofol decreases postoperative nausea and vomiting (PONV) compared with inhalational agents. Anesth Analg 77:S21-S29. Both cardiovascular effects are transient and are moderated by slower administration of the drug. it is not approved for use in obstetric anesthesia. Nahrwold ML. Propofol should be used very cautiously in patients with any movement disorders.2% and is more common when propofol is combined with opioids during anesthesia or with long-term preoperative use of beta-blockers. 2004 page 1262 . McLeskey CH. American College of Surgeons SESAP 13 Category 16 Item 5 Which of the following is a contraindication to the use of epidural anesthesia in critically ill patients? (A) Fever (B) Increased WBC count (C) Hypovolemia (D) Intractable angina (E) Peripheral vascular disease page 1263 . 2001 3. Indications for the use of epidural analgesia in critically ill patients include chest trauma. Crit Care Med 33:1400-1407. intractable angina. can be reduced or minimized. 2005 page 1264 . Thompson JS. Boezaart A. fever and an increased WBC count indicate the presence of a systemic inflammatory response syndrome rather than bacteremia. it can help achieve optimum patient comfort and reduce physiologic and psychological stress. do not provide a reliable diagnosis of bacteremia and are not contraindications to the placement of an epidural catheter. Sheldon DG. REFERENCES 1. and obstructive ileus. Contraindications to the use of epidural analgesia are equally important. including delirium.American College of Surgeons SESAP 13 Critique for Category 16 Item 5 Epidural analgesia can play a valuable role in the multimodal approach to pain management in critically ill patients. and peripheral vascular disease.Moraca RJ.Park WY. severe hypovolemia. Ann Surg 238:663-673. Fever and an increased WBC count alone. mental status changes. local infection at the puncture site. Lee KK: Effect of epidural anesthesia and analgesia on perioperative outcome: a randomized. Ann Surg 234:560-571. pancreatitis.. bacteremia and sepsis. hemodynamic instability. in the absence of positive blood cultures. Although epidural analgesia may not always improve mortality outcomes. By avoiding high systemic doses of narcotics. paralytic ileus.Schulz-Stubner S. several complications. In many ICU patients. Contraindications include coagulopathy. controlled Veterans Affairs cooperative study. Thirlby RC: The role of epidural anesthesia and analgesia in surgical practice. Hata JS: Regional analgesia in the critically ill. recent thoracic surgery. 2003 2. and gastrointestinal dysfunction. American College of Surgeons SESAP 13 Category 16 Item 6 Which of the following statements about the use of opiate analgesics in surgical patients is TRUE? (A) Meperidine is preferred to morphine sulfate in critical care patients with renal failure (B) Morphine is advantageous for patients with congestive heart failure due to its effect on venous capacitance (C) Morphine has a direct effect on myocardial contractility (D) Fentanyl should not be used in patients with reactive airway disease (E) Meperidine induces shivering in most patients and should be avoided in patients with hypothermia page 1265 . Feliciano DV.American College of Surgeons SESAP 13 Critique for Category 16 Item 6 Pain control is an essential component of the clinical care of surgical patients. Kress JP: Pain control. pp 165-180 2. In addition. including both arteriolar and venous dilation. Although morphine has no direct effects on the inotropic state of the heart. 2004. Schmidt GA.Mackersie RC. has proconvulsant activity. McGraw-Hill. Normeperidine accumulation is particularly troublesome in patients with renal failure. it has predictable hemodynamic effects. there is considerable uncertainty about the appropriate use of opiates in surgical patients. ed 3. sedation and use of muscle relaxants. Fentanyl does not release histamine and is the agent of choice in patients with bronchospasm. REFERENCES 1. New York. Meperidine is generally believed to be a poor analgesic choice in critically ill patients. pp 1213-1250 page 1266 . which may explain its utility in congestive heart failure. prolonged serum half-life. and a cumulative effect after repeated doses. ed 5. McGraw-Hill. Pittet JF.Gehlbach B. Normeperidine. Wood LDH (eds): Principles of Critical Care. and surgeons should have a working knowledge of the principles governing the appropriate use of analgesics. due to the potential for abuse. Dicker RA: Principles of critical care. 2005. New York. Morphine increases venous capacitance to a greater extent than arteriolar capacitance. Meperidine is occasionally selected for its ability to suppress shivering. a metabolite. in Hall JB. Mattox KL (eds): Trauma. in Moore EE. American College of Surgeons SESAP 13 Category 16 Item 7 Use of thoracic epidural analgesia rather than patient-controlled analgesia (PCA) for chest injury improves all of the following EXCEPT (A) tidal volume (B) functional residual capacity (C) maximal inspiratory force (D) visual analogue score for pain (E) incidence of pneumonia page 1267 . et al: Prospective evaluation of epidural versus intrapleural catheters for analgesia in chest wall trauma. Surgery 136:426-430. Flynn W. Edwards T. Crews J. 1994 3. Jurkovich GJ: Epidural analgesia improves outcome after multiple rib fractures. The pain from fractured ribs limits tidal volume. 1999 page 1268 .Moon MR. Kaiser R. The epidural route of analgesia also avoids the systemic effects of parenteral narcotics and allows patients to significantly improve tidal volume and maximal inspiratory force. None of the various routes of analgesia alter functional residual capacity.Bulger EM. maximal inspiratory force. and effective coughing. Gibson SW. et al: Prospective. Radafshar SM. epidural analgesia significantly reduces the incidence of pneumonia compared with parenteral analgesia. J Trauma 36:865-870. randomized comparison of epidural versus parenteral opioid analgesia in thoracic trauma. REFERENCES 1.American College of Surgeons SESAP 13 Critique for Category 16 Item 7 Rib fractures are associated with significant morbidity and mortality because of their impact on normal pulmonary function. Several prospective studies have demonstrated that epidural analgesia is superior to either patientcontrolled analgesia or intrapleural anesthetic administration for pain relief as measured by the visual analogue score. 2004 2. All are associated with risk for the subsequent development of pneumonia. Klotz P. Finally.Luchette FA. Luchette FA. Ann Surg 229:684692. Which of the following would be MOST effective in decreasing postoperative cardiac events in this patient? (A) Preoperative statin administration (B) Left anterior coronary artery stenting (C) Perioperative beta blockade (D) Intraoperative echocardiography (E) Perioperative calcium channel blocker therapy page 1269 . and normal valves. left ventricular ejection fraction of 40%.American College of Surgeons SESAP 13 Category 16 Item 8 A 65-year-old smoker is scheduled for an elective femoral-popliteal bypass. He has a history of myocardial infarction and is known to have stable angina. Preoperative stress echocardiogram shows reversible ischemia in the area supplied by the left anterior descending artery. Drugs that inhibit the enzyme HMG-CoA. A similar study of patients undergoing noncardiac elective surgery showed that perioperative atenolol significantly reduced cardiac deaths up to 2 years after operation. Eagle KA: Coronary revascularization before noncardiac surgery. 2003 2.McFalls EO. Bax JJ. and cardiopulmonary complications. 2004 3. or severe aortic stenosis. tissue-healing. Tonnesen H. There was no difference in the rate of postoperative myocardial infarction at 30 days. In contrast. a recent study (Coronary Artery Revascularization Prophylaxis trial. minor. N Engl J Med 341:1789-1794. et al: Coronary-artery revascularization before elective major vascular surgery. Beier-Holgersen R. or CARP) of patients undergoing elective vascular surgery compared coronary revascularization with no coronary revascularization. et al: Effects of intravenous fluid restriction on postoperative complications: comparison of two perioperative fluid regimens: a randomized assessor-blinded multicenter trial. Patients were excluded if they had at least 50% left main coronary artery stenosis. N Engl J Med 351:2795-2804. A randomized. REFERENCES 1. 2003 5. but most of the cardiopulmonary complications observed were related to fluid overload. et al: The effect of bisoprolol on perioperative mortality and myocardial infarction in high-risk patients undergoing vascular surgery.Brandstrup B. Thomson IR. Neither calcium channel-blocker administration nor intraoperative echocardiography has been shown to diminish perioperative cardiac morbidity or mortality.American College of Surgeons SESAP 13 Critique for Category 16 Item 8 Patients who undergo major vascular surgery have a 50% prevalence of coronary artery disease and a high incidence of perioperative cardiac complications and death.Poldermans D. A retrospective study comparing patients who died of cardiac causes after major vascular operation with matched case-controls suggested that statin administration was less common in those who died of cardiac causes than in controls. A randomized prospective multicenter trial compared the effects of a fluid-restrictive regimen with standard therapy for patients undergoing colorectal resection.7 years of follow-up.Moscucci M. Ward HB. Kertai MD. major.Poldermans D. Bax JJ. Moritz TE. Medical management was the same for both groups. 2004 4. N Engl J Med 351:2861-2863. Dutch Echocardiographic Cardiac Risk Evaluation Applying Stress Echocardiography Study Group. may have a direct stabilizing effect on coronary artery plaques. left ventricular ejection fraction < 20%. Boersma E. Circulation 107:1848-1851. Patients receiving the restrictive regimen had significantly less overall. et al: Statins are associated with a reduced incidence of perioperative mortality in patients undergoing major noncardiac vascular surgery. known as statins. Krenning B. 1999 page 1270 . Goldman S. and there were no perioperative myocardial infarcts reported in either group. prospective study published in 1999 showed that high-risk patients undergoing major vascular surgery treated perioperatively with the beta blocker bisoprolol were significantly less likely to suffer myocardial infarction than control patients receiving standard treatment (0% vs17%) and significantly less likely to die of cardiac causes (3% vs 17%) at 30 days after operation. and no difference in mortality between the two groups at 2. Hjortso E. Ann Surg 238:641-648. lidocaine can be given again in 1 hour (D) Lidocaine with epinephrine should not be used on fingers (E) Intravenous lidocaine may cause heart block page 1271 . Which of the following statements about local anesthetics is NOT true? (A) Maximum dose for lidocaine with epinephrine is 490 mg (B) Maximum dose for lidocaine without epinephrine is 300 mg (C) After maximum dose administration.American College of Surgeons SESAP 13 Category 16 Item 9 A 42-year-old man who weighs 70 kg is scheduled to have multiple lipomas removed in an ambulatory surgery center. utdol.Hsu DC. but can cause bradycardia. or ears. 2005. Inc: Lidocaine: Drug information. penis. REFERENCES 1. Lidocaine should not be re-administered within 2 hours of the maximum dosage. Select Drug Information from Lexi-Comp Online. so that a larger dose can be given (7 mg/kg). Up To Date website. Because localized vasoconstriction can lead to tissue ischemia. Dose is 3 to 5 mg/kg.American College of Surgeons SESAP 13 Critique for Category 16 Item 9 Lidocaine is used as an infiltrative anesthetic for minor procedures. atrioventricular block.Lexi-Comp. ventricular arrhythmia. and even cardiac arrest. nose. 1978present. typically as a 1% (10 mg/mL) solution. Intravenous lidocaine is useful for ventricular arrythmias.utdol. Available at: www. Up to Date Website. Infiltrative anesthetics.com 2. not to exceed 300 mg. lidocaine with epinephrine should not be infiltrated into digits.com page 1272 . The systemic absorption of lidocaine is also diminished. The addition of epinephrine causes vasoconstriction and decreases bleeding at the wound site. Available at: http://www. decreased contractility. American College of Surgeons SESAP 13 Category 16 Item 10 Which of the following sedatives has active metabolites that may accumulate in clinically significant amounts if it is used as a continuous drip? (A) Lorazepam (B) Midazolam (C) Diazepam (D) Dexmedetomidine (E) Propofol page 1273 . Shehabi Y. Midazolam has a rapid onset and few active metabolites. Maccioli GA: Dexmedetomidine. experience with infusion periods beyond 24 hours is limited. Propofol and dexmedetomidine both have extremely short half-lives. et al: Dexmedetomidine infusion for more than 24 hours in critically ill patients: sedative and cardiovascular effects. Prielipp RC: Benzodiazepines in the intensive care unit. 2001 page 1274 . but has the least prolonged effect due to active metabolites. Intensive Care Med 30:2188-2196. however. Crit Care Clin 17:843-862.American College of Surgeons SESAP 13 Critique for Category 16 Item 10 Diazepam. making it an ideal transition drug from longer-term sedation to preparation for extubation. with complete abrogation of their sedative effect moments after intravenous administration is discontinued. Coursin DB. 2004 3. Lorazepam is slower onset. Innes R. Ruettimann U. making it an excellent benzodiazepine when long-term sedation in the ICU is desired. Dexmedetomidine has very little suppression of respiratory drive. Adamson H. 2001 2.Coursin DB. the prototypical benzodiazepine. REFERENCES 1. has several active metabolites that contribute to its sedative and amnestic properties far beyond its half-life.Young CC. Curr Opin Crit Care 7:221—226. American College of Surgeons SESAP 13 Category 16 Item 11 The intravenous induction agent with the LEAST effect on the cardiovascular system is (A) propofol (B) ketamine (C) methohexital (D) etomidate (E) thiopental page 1275 . ed 6. Even in patients without cardiovascular disease. Glass PSA. An induction dose given to cardiac patients undergoing noncardiac operations results in almost no change in heart rate. Balser JR: General anesthetics.American College of Surgeons SESAP 13 Critique for Category 16 Item 11 When used in doses to induce anesthesia adequate for surgical procedures. REFERENCES 1. pp 317-378 page 1276 . methohexital and thiopental will produce significant cardiovascular effects. 2005. Lubarsky DA.Evers AS. Parker KL (eds): Goodman & Gilman's The Pharmacological Basis of Therapeutics. propofol. Lazo JS. in Brunton LL. ed 11. in Miller RD (ed): Miller’s Anesthesia. The most prominent effect of propofol. pulmonary artery wedge pressure. Etomidate has a minimal effect on cardiovascular function.Reves JG. Ketamine does have myocardial suppressant effects. pp 341-368 2. arterial blood pressure. McGraw-Hill. 2006. In normovolemic patients. with transiently decreased mean arterial pressures and cardiac outputs. induction doses of ketamine will result in decreased arterial blood pressure. and in the hypovolemic trauma patient with maximal adrenergic tone. appears to be due to peripheral vasodilatation and possibly myocardial depression. are also now well studied. The cardiovascular effects of the newer induction agent. induction doses of etomidate can still result in hypotension. which is associated with a decrease in cardiac output/index. Crowder CM. induction doses of ketamine may result in increases of arterial blood pressure by as much as 25% because of ketamine’s effect on the sympathetic nervous system. and cardiac index. a decrease in mean arterial pressure by as much as 30% during induction of anesthesia. induction doses produce a 25% to 40% reduction in systolic blood pressure. setting it apart from other rapid-onset induction agents. In hypovolemic patients. Churchill Livingstone. Philadelphia. New York. McEvoy MD: Intravenous nonopioid anesthetics. American College of Surgeons SESAP 13 Category 16 Item 12 The first step in the management of a patient with seizure due to an overdose of local anesthetic should be (A) intravenous midazolam (B) intravenous succinylcholine (C) intravenous diphenylhydantoin (D) supplemental oxygen (E) endotracheal intubation page 1277 . Endotracheal intubation is not necessary unless ventilation and oxygenation is judged to be inadequate. It does not affect CNS activity or metabolic demand. and respiratory and cardiovascular depression. The immediate step should be the establishment of a secure airway. and drugs. Circulation should be optimized with intravenous fluids. Kirby RR: Immediate reactions to local anesthetics.Covino BG. breathing. in Gravenstein N.5 to 1. pp 255-271 page 1278 . Stoelting RK (eds): Handbook of Clinical Anesthesia. coma. Philadelphia. Philadelphia. CNS depression and cessation of seizures can be accomplished with incremental intravenous doses of midazolam (1 to 5 mg). Lippincott Williams & Wilkins. ranging from tinnitus and lightheadedness to numbness of the tongue. with addition of atropine (0. thiopental (50 mg). Intravenous diphenylhydantoin is not indicated. Succinylcholine is not routinely necessary and should not be given unless the airway is secure. Lippincott-Raven. When these measures are in place. in Barash PG. assisted ventilation with oxygen is indicated if hypoventilation is suspected. or propofol (1 mg/kg) until seizure activity ceases.American College of Surgeons SESAP 13 Critique for Category 16 Item 12 Generalized central nervous system (CNS) toxicity of local anesthetics is dose dependent. 1996. circulation. then seizures. The treatment protocol for a patient exhibiting toxic effects of a local anesthetic such as lidocaine is largely supportive and follows standard resuscitation guidelines: airway. REFERENCES 1.[No Authors Listed]: Local anesthetics. Because respiratory acidosis and/or hypoxemia will enhance CNS drug levels and exacerbate the seizure threshold.0 mg in patients with bradycardia) and ephedrine (10 to 25 mg in those with hypotension). 2006. Kirby RR (eds): Complications in Anesthesiology. pp 541-563 2. Cullen BF. unconsciousness. diazepam (5 to 10 mg). American College of Surgeons SESAP 13 Category 16 Item 13 Which of the following statements about incisional wound infiltration with local anesthetics is NOT true? (A) Local infiltration increases the up-regulation of peripheral nocioceptors (B) It improves postoperative pain control (C) Preemptive use does not decrease postoperative pain after vertical midline incisions (D) The maximal dose of bupivacaine that can be infiltrated into the subcutaneous space is 2.5 mg/kg without epinephrine and 3.0 mg/kg with epinephrine (E) Lidocaine toxicity is seen with doses > 7 mg/kg page 1279 . Studies suggest that adding sodium bicarbonate to lidocaine significantly decreases burning sensations. Goroshina J. Generalized seizures and respiratory arrest can occur. Kundu S: Principles of office anesthesia: part I. tremors.Lepner U. Local anesthetic infiltration down-regulates rather than up-regulates nociocepters. The maximum dosage is 2. No studies have compared the efficacy of different concentrations of lidocaine. Intraperitoneal subphrenic placement of lidocaine does not seem to alter pain after laparoscopic cholecystectomy. Infiltration should be slow. Rushatamukayanunt W: Efficacy of pre-incisional bupivacaine infiltration on postoperative pain relief after appendectomy: prospective double-blind randomized trial. In vertical midline incisions. REFERENCES 1. Scand J Surg 92:121-124. 2004 4. tinnitus.0 hours. Symptoms include vasovagal responses. Manolitsas TP. Samarutel J: Postoperative pain relief after laparoscopic cholecystectomy: a randomised prospective double-blind clinical trial.5 mg/kg. the effectiveness of preemptive analgesia can vary with surgical procedures.American College of Surgeons SESAP 13 Critique for Category 16 Item 13 Preemptive analgesia is based on the hypothesis that it decreases nociceptive input from afferent stimuli to the central nervous system. or 7 mg/kg when mixed with epinephrine.Lohsiriwat V. and shivering. Cohn DE. ideally 25 to 30 gauge. the smallest gauge needle should be used. World J Surg 28:947-950. In general. Addition of epinephrine prolongs the duration of anesthetic and provides some degree of hemostasis.Achar S. Lidocaine toxicity can occur when dosed at levels greater than 7 mg/kg or when injected intravascularly. and preferentially into subcutaneous tissue rather than the dermis. 2003 page 1280 . Am J Obstet Gynecol 188:901-905. Lidocaine has an onset of less than 2 minutes and its effects last for 1.Updike GM. Lert-akyamanee N. confusion. This can increase to 3 mg/kg when epinephrine is added. Eaton LA. Bupivacaine (Marcaine) has an onset of 5 minutes and lasts for 2 to 4 hours. metallic tastes. leading to reduced hyperexcitability and subsequent decreases in postoperative pain. For unclear reasons. Infiltrative anesthesia. There is no conclusive study showing that mixing lidocaine and bupivacaine is beneficial.5 to 2. Utilization of local anesthetic in wounds following laparoscopic cholecystectomy and appendectomy significantly reduces abdominal postoperative pain and narcotic utilization. Am Fam Physician 66:91-94 2002 2. et al: Pre-emptive analgesia in gynecologic surgical procedures: preoperative wound infiltration with ropivacaine in patients who undergo laparotomy through a midline vertical incision. preoperative wound infiltration did not decrease postoperative analgesic utilization. 2003 3. when infiltrating skin in an awake patient. The maximum dose is 4 mg/kg. American College of Surgeons SESAP 13 Category 16 Item 14 Bupivacaine (Marcaine) injection for local analgesia may be associated with all of the following adverse effects EXCEPT (A) pruritus (B) seizure (C) hypotension (D) heart block (E) methemoglobinemia page 1281 . Pruritus at the injection site is a common but benign adverse effect associated with histamine reaction. REFERENCES 1. More significant effects include seizure when the toxic dose of bupivacaine has been exceeded. the intra-arterial injection of bupivacaine can have significant adverse effects. in Mosby’s Drug Consult 2007 online. and to provide postoperative pain relief in a procedure done with general anesthesia. Allanic L. Methemoglobinemia is not an associated complication of bupivacaine injection. They can be used to provide a field block for a procedure being done with just local anesthesia. Can J Anaesth 50:62-66. Myocardial depression is treated with inotropic support.Bupivacaine HCl. Unlike lidocaine. to supplement sedation for a procedure being performed without intubation. 2. 2003 page 1282 . 2007. Fassassi M. et al: Successful resuscitation of serious bupivacaine intoxication in a patient with pre-existing heart failure. Bupivacaine is commonly used because of its long duration of action.American College of Surgeons SESAP 13 Critique for Category 16 Item 14 Local anesthetic agents are used in many surgical situations. Elsevier Mosby. heart block. and myocardial depression.Favier JC. Da Conceicao M. Heart block needs to be aggressively treated with chronotropic agents and cardiac pacing. which has minimal effect when injected intra-arterially. American College of Surgeons SESAP 13 Category 16 Item 15 Adverse effects of prolonged (> 48 hours) propofol sedation include all of the following EXCEPT (A) rhabdomyolysis (B) renal failure (C) metabolic alkalosis (D) cardiac failure (E) polyneuropathy page 1283 . severe metabolic acidosis. Prolonged infusion of propofol. Candiani A. especially for patients with acute neurologic or inflammatory illnesses. 2003 page 1284 . particularly when combined with therapeutic catecholamines and/or corticosteroids. Intensive Care Med 29:1417-1425. and renal failure. Ann Pharmacother 36:1453-1456.Kang TM: Propofol infusion syndrome in critically ill patients. The syndrome was initially described in critically ill children. hyperkalemia. hepatomegaly with fatty infiltration. Rasulo F. Latronico N: The pathophysiology of propofol infusion syndrome: a simple name for a complex syndrome. Sudden or rapid onset of marked bradycardia resistant to standard treatments and progressing to asystole characterizes the cardiac failure. initiating a syndrome of cardiac failure associated with rhabdomyolysis. 2002 2. Current recommendations limit propofol infusions to 5 mg/kg/hour for less than 48 hours. Central nervous system activation of an intense catecholamine and glucocorticoid response as well as systemic cytokine activation are priming factors for cardiac and peripheral neuromuscular dysfunction. Patients with acute neurologic illnesses or inflammatory processes (severe infections or overt sepsis) appear to be particularly vulnerable. Diagnosis requires the presence of bradycardia and at least one of the other associated signs.Vasile B. REFERENCES 1. lipemia.American College of Surgeons SESAP 13 Critique for Category 16 Item 15 Propofol infusion syndrome is a rare but often fatal complication of prolonged high-dose propofol sedation. appears to act as a trigger. American College of Surgeons SESAP 13 Category 16 Item 16 Succinylcholine is safe to use in a patient with (A) coronary artery disease (B) glaucoma (C) intracranial hypertension (D) end-stage liver disease (E) paraplegia page 1285 . Lea & Febiger. ed 3. 2006.Vernick WJ. REFERENCES 1. Patients with heart disease have risks for anesthesia that are not related to succinylcholine administration. pp 938-1022 2. Lippincott Williams & Wilkins. and its use is dangerous in certain clinical situations. Rapid potassium fluctuations can immediately cause fatal cardiac arrythmias. Philadelphia. Hanson CW III: Anesthesia. El-Etr AA. head injury patients. et al: Relaxants—pharmacology and use. Transient increases in intracranial and intraocular pressures are known to occur. and a standard dose of succinylcholine can easily cause massive depolarization. Porrett PM. pp 111-121 page 1286 . The diffuse depolarization of muscle beds causes an immediate and transient rise in serum potassium levels of as much as 1 mEq within minutes of adminstration. in Collins VJ: Principles of Anesthesiology: General and Regional Anesthesia. Hameroff SR. in Atluri P. eg. Karakousis GC. Philadelphia. Patients with end-stage liver disease and those with renal insufficiency are particularly susceptible to rapid changes in potassium levels.Boswell MV. Kaiser LR (eds): The Surgical Review: An Integrated Basic and Clinical Science Study Guide. Paraor quadraplegic patients have up-regulated muscle endplate receptors. 1993. resulting in dangerous acute elevations of serum potassium. and succinylcholine is contraindicated in ophthalmic surgery cases and patients with intracranial hypertension. Calkins JM.American College of Surgeons SESAP 13 Critique for Category 16 Item 16 Succinylcholine paralyzes by depolarizing muscle tissue. American College of Surgeons SESAP 13 Category 16 Item 17 Tolerance (A) Psychic or physical state characterized by a compulsion to take a drug (B) Condition characterized by dissociation from the physical environment (C) Characteristic physiologic effect results from drug withdrawal (D) State of altered consciousness resulting from excessive consumption of a narcotic (E) Progressive decline in a drug’s potency with continuous use page 1287 . in Souba WW. the iatrogenic production of opioid addiction is very rare. ed 18. The possibility that the medical administration of opioids could result in addiction has generated much debate about the use of opioids. et al (eds): ACS Surgery: Principles and Practice 2006. chap 1.American College of Surgeons SESAP 13 Critique for Category 16 Item 17 Addiction is a compulsive and maladaptive dependence on a substance or behavior. sec 5. there were only four reasonably well-documented cases of subsequent addiction. 2006. Merck Research Laboratories. WebMD. REFERENCES 1. Basic Surgical and Perioperative Considerations. pp 75-89 2. in Beers MH. Tolerance refers to the need for increasing doses of a substance to achieve the same therapeutic endpoint. Jurkovich GJ. Dependence is indicated by the development of a characteristic set of adverse symptoms when the substance is withdrawn. Jones TV. Fink MP. Porter RS. Kaplan JL. In a prospective study of 12. et al (eds): The Merck Manual of Diagnosis and Therapy. Tolerance can occur without physical dependence.000 hospitalized patients receiving opioid for a protracted period.[No Authors Listed]: Psychiatric disorders. the converse does not appear to be true. and in none of these was there a history of previous substance abuse. New York. Kaiser LR. pp 1683-1700 page 1288 . Whitehouse Station.Kehlet H: Postoperative pain. Thus. It is important to distinguish addiction from tolerance and from physical dependence. American College of Surgeons SESAP 13 Category 16 Item 18 Addiction (A) Psychic or physical state characterized by a compulsion to take a drug (B) Condition characterized by dissociation from the physical environment (C) Characteristic physiologic effect results from drug withdrawal (D) State of altered consciousness resulting from excessive consumption of a narcotic (E) Progressive decline in a drug’s potency with continuous use page 1289 . ed 18. Fink MP. Thus. and in none of these was there a history of previous substance abuse. Kaplan JL.000 hospitalized patients receiving opioid for a protracted period. Tolerance can occur without physical dependence. Tolerance refers to the need for increasing doses of a substance to achieve the same therapeutic endpoint. 2006. the converse does not appear to be true. the iatrogenic production of opioid addiction is very rare. et al (eds): ACS Surgery: Principles and Practice 2006.American College of Surgeons SESAP 13 Critique for Category 16 Item 18 Addiction is a compulsive and maladaptive dependence on a substance or behavior.[No Authors Listed]: Psychiatric disorders. Jones TV. Dependence is indicated by the development of a characteristic set of adverse symptoms when the substance is withdrawn. It is important to distinguish addiction from tolerance and from physical dependence. there were only four reasonably well-documented cases of subsequent addiction. in Souba WW. REFERENCES 1. New York. In a prospective study of 12.Kehlet H: Postoperative pain. Merck Research Laboratories. Kaiser LR. chap 1. pp 1683-1700 page 1290 . et al (eds): The Merck Manual of Diagnosis and Therapy. The possibility that the medical administration of opioids could result in addiction has generated much debate about the use of opioids. sec 5. Basic Surgical and Perioperative Considerations. Porter RS. in Beers MH. WebMD. Whitehouse Station. Jurkovich GJ. pp 75-89 2. American College of Surgeons SESAP 13 Category 16 Item 19 Physical dependence (A) Psychic or physical state characterized by a compulsion to take a drug (B) Condition characterized by dissociation from the physical environment (C) Characteristic physiologic effect results from drug withdrawal (D) State of altered consciousness resulting from excessive consumption of a narcotic (E) Progressive decline in a drug’s potency with continuous use page 1291 . et al (eds): The Merck Manual of Diagnosis and Therapy. and in none of these was there a history of previous substance abuse. Kaiser LR. sec 5. Thus. It is important to distinguish addiction from tolerance and from physical dependence. Merck Research Laboratories. Tolerance refers to the need for increasing doses of a substance to achieve the same therapeutic endpoint. New York. The possibility that the medical administration of opioids could result in addiction has generated much debate about the use of opioids. REFERENCES 1. there were only four reasonably well-documented cases of subsequent addiction. Whitehouse Station. the iatrogenic production of opioid addiction is very rare. in Souba WW. 2006. Jurkovich GJ.000 hospitalized patients receiving opioid for a protracted period. the converse does not appear to be true. Jones TV. pp 75-89 2.American College of Surgeons SESAP 13 Critique for Category 16 Item 19 Addiction is a compulsive and maladaptive dependence on a substance or behavior. Tolerance can occur without physical dependence. In a prospective study of 12.Kehlet H: Postoperative pain. in Beers MH. Dependence is indicated by the development of a characteristic set of adverse symptoms when the substance is withdrawn. et al (eds): ACS Surgery: Principles and Practice 2006. Basic Surgical and Perioperative Considerations. ed 18. pp 1683-1700 page 1292 . WebMD.[No Authors Listed]: Psychiatric disorders. Fink MP. Porter RS. Kaplan JL. chap 1. American College of Surgeons SESAP 13 Category 16 Item 20 Hepatitis (A) Nitrous oxide (B) Halothane (C) Enflurane (D) Ketamine (E) Succinylcholine page 1293 . and conscious sedation. enflurane releases a potentially nephrotoxic fluoride ion. Williams CG. Nondepolarizing agents compete with acetylcholine by binding at another part of the receptor. Although ketamine does not significantly depress ventilatory drive. ed 2. Ketamine is an intravenous agent that produces a dissociative state of anesthesia. Saunders. Porrett PM. maintained with inhalational agents. pain management. pp 401-444 2. Inhalational agents are used in combination with oxygen and nitrous oxide. Nitrous oxide diffuses into closed spaces faster than nitrogen diffuses out and may induce elevated pressures in air-filled spaces. Lippincott Williams & Wilkins. Halothane has also been associated with a rare form of potentially fatal fulminant hepatitis. 2004. 2006. However. Kaiser LR (eds): The Surgical Review: An Integrated Basic and Clinical Science Study Guide. upper and lower motor neuron injuries. Depolarizing agents bind directly to the acetylcholine receptor of neuromuscular junctions. nitrous oxide is contraindicated in patients with closed gas spaces such as small bowel obstruction or pneumothorax. succinylcholine is associated with a number of side effects. in Townsend CM Jr (ed): Sabiston Textbook of Surgery: The Biological Basis of Modern Surgical Practice. it is associated with increased cerebral blood flow and oxygen consumption and is contraindicated in patients with head injury. Enflurane has toxic effects on the kidney and brain. Enflurane has also been associated with seizure activity at high doses. producing tachycardia. The most important side effect of nitrous oxide relates to its solubility. Succinylcholine may cause transient hyperkalemia that may be profound in patients with burns. and bronchodilation. and significant trauma. During metabolism. Prough DS: Anesthesiology principles. in Atluri P.American College of Surgeons SESAP 13 Critique for Category 16 Item 20 Modern anesthesia practice frequently uses multiple drugs. Karakousis GC. Philadelphia. but both have been replaced by newer agents that do not have such significant side effects. succinylcholine is the only depolarizing agent still used in modern anesthetic practice. pp 111-121 page 1294 .Vernick WJ. Hanson CW III: Anesthesia. REFERENCES 1. Two categories of neuromuscular blockers are currently in clinical use: depolarizing (noncompetitive) and nondepolarizing (competitive) agents. which will prolong the drug’s duration of effects. Hence. Halothane and enflurane were once popular inhalational anesthetics. causing fasciculations and eventual flaccid paralysis. and it sensitizes the myocardium to the effects of catecholamines. amnestics. it permits a lower dose of the second agent. increased blood pressure. It is a potent cardiac depressant. Although combining nitrous oxide with another inhalational agent provides only partial anesthetic effect. ed 17. General anesthesia is often initiated with intravenous induction agents. Its advantages include rapid onset and short duration of action. and supplemented by opiates.Sherwood E. Although many types of nondepolarizing agents are used clinically. and muscle relaxants. Philadelphia. Halothane has toxic effects on the heart and liver. Succinylcholine is also contraindicated in patients with pseudocholinesterase deficiency and those with liver disease because metabolism of succinylcholine is impaired in these conditions. It stimulates the sympathetic nervous system. This limits toxicity and facilitates induction and emergence from anesthesia. Halothane hepatitis tends to afflict middle-aged obese women and may have a familial predisposition. American College of Surgeons SESAP 13 Category 16 Item 21 Increased cerebral blood flow (A) Nitrous oxide (B) Halothane (C) Enflurane (D) Ketamine (E) Succinylcholine page 1295 . Karakousis GC. Halothane hepatitis tends to afflict middle-aged obese women and may have a familial predisposition. ed 17. Its advantages include rapid onset and short duration of action. pp 111-121 page 1296 . in Townsend CM Jr (ed): Sabiston Textbook of Surgery: The Biological Basis of Modern Surgical Practice. General anesthesia is often initiated with intravenous induction agents. 2004.American College of Surgeons SESAP 13 Critique of Category 16 Item 21 Modern anesthesia practice frequently uses multiple drugs.Sherwood E. upper and lower motor neuron injuries. Two categories of neuromuscular blockers are currently in clinical use: depolarizing (noncompetitive) and nondepolarizing (competitive) agents. Although many types of nondepolarizing agents are used clinically. The most important side effect of nitrous oxide relates to its solubility. and it sensitizes the myocardium to the effects of catecholamines. which will prolong the drug’s duration of effects. Enflurane has toxic effects on the kidney and brain. amnestics. Saunders. causing fasciculations and eventual flaccid paralysis. Inhalational agents are used in combination with oxygen and nitrous oxide. It stimulates the sympathetic nervous system. succinylcholine is the only depolarizing agent still used in modern anesthetic practice. This limits toxicity and facilitates induction and emergence from anesthesia. REFERENCES 1. It is a potent cardiac depressant. Depolarizing agents bind directly to the acetylcholine receptor of neuromuscular junctions. it permits a lower dose of the second agent. Lippincott Williams & Wilkins. Ketamine is an intravenous agent that produces a dissociative state of anesthesia. Porrett PM. pp 401-444 2. Kaiser LR (eds): The Surgical Review: An Integrated Basic and Clinical Science Study Guide. Halothane and enflurane were once popular inhalational anesthetics. and conscious sedation. and bronchodilation. Enflurane has also been associated with seizure activity at high doses. Halothane has also been associated with a rare form of potentially fatal fulminant hepatitis. Philadelphia. it is associated with increased cerebral blood flow and oxygen consumption and is contraindicated in patients with head injury. in Atluri P. and supplemented by opiates. enflurane releases a potentially nephrotoxic fluoride ion. However. Hence. producing tachycardia. Williams CG. Hanson CW III: Anesthesia. succinylcholine is associated with a number of side effects. Philadelphia. Nitrous oxide diffuses into closed spaces faster than nitrogen diffuses out and may induce elevated pressures in air-filled spaces. Although combining nitrous oxide with another inhalational agent provides only partial anesthetic effect. Halothane has toxic effects on the heart and liver. Prough DS: Anesthesiology principles. and muscle relaxants. During metabolism. nitrous oxide is contraindicated in patients with closed gas spaces such as small bowel obstruction or pneumothorax. 2006. pain management. ed 2. increased blood pressure. Although ketamine does not significantly depress ventilatory drive.Vernick WJ. but both have been replaced by newer agents that do not have such significant side effects. Succinylcholine may cause transient hyperkalemia that may be profound in patients with burns. Nondepolarizing agents compete with acetylcholine by binding at another part of the receptor. Succinylcholine is also contraindicated in patients with pseudocholinesterase deficiency and those with liver disease because metabolism of succinylcholine is impaired in these conditions. and significant trauma. maintained with inhalational agents. American College of Surgeons SESAP 13 Category 16 Item 22 Diffuses into closed air-containing spaces (A) Nitrous oxide (B) Halothane (C) Enflurane (D) Ketamine (E) Succinylcholine page 1297 . Williams CG. increased blood pressure. General anesthesia is often initiated with intravenous induction agents. Prough DS: Anesthesiology principles. succinylcholine is associated with a number of side effects. ed 2. Enflurane has also been associated with seizure activity at high doses. enflurane releases a potentially nephrotoxic fluoride ion. REFERENCES 1. This limits toxicity and facilitates induction and emergence from anesthesia. but both have been replaced by newer agents that do not have such significant side effects. Its advantages include rapid onset and short duration of action. pain management. Nondepolarizing agents compete with acetylcholine by binding at another part of the receptor. Depolarizing agents bind directly to the acetylcholine receptor of neuromuscular junctions. maintained with inhalational agents. Enflurane has toxic effects on the kidney and brain. It is a potent cardiac depressant. Although combining nitrous oxide with another inhalational agent provides only partial anesthetic effect. Succinylcholine is also contraindicated in patients with pseudocholinesterase deficiency and those with liver disease because metabolism of succinylcholine is impaired in these conditions. Halothane has also been associated with a rare form of potentially fatal fulminant hepatitis. and bronchodilation. Philadelphia. 2004. The most important side effect of nitrous oxide relates to its solubility. and significant trauma. Porrett PM. Inhalational agents are used in combination with oxygen and nitrous oxide. and supplemented by opiates.American College of Surgeons SESAP 13 Critique for Category 16 Item 22 Modern anesthesia practice frequently uses multiple drugs. It stimulates the sympathetic nervous system. which will prolong the drug’s duration of effects. it permits a lower dose of the second agent. Karakousis GC. Hanson CW III: Anesthesia. Succinylcholine may cause transient hyperkalemia that may be profound in patients with burns. Philadelphia. Kaiser LR (eds): The Surgical Review: An Integrated Basic and Clinical Science Study Guide. and conscious sedation. ed 17. Hence. Lippincott Williams & Wilkins. in Townsend CM Jr (ed): Sabiston Textbook of Surgery: The Biological Basis of Modern Surgical Practice. Although ketamine does not significantly depress ventilatory drive. Nitrous oxide diffuses into closed spaces faster than nitrogen diffuses out and may induce elevated pressures in air-filled spaces. amnestics. Halothane and enflurane were once popular inhalational anesthetics. nitrous oxide is contraindicated in patients with closed gas spaces such as small bowel obstruction or pneumothorax. in Atluri P.Sherwood E. pp 401-444 2.Vernick WJ. it is associated with increased cerebral blood flow and oxygen consumption and is contraindicated in patients with head injury. Halothane has toxic effects on the heart and liver. Ketamine is an intravenous agent that produces a dissociative state of anesthesia. Two categories of neuromuscular blockers are currently in clinical use: depolarizing (noncompetitive) and nondepolarizing (competitive) agents. pp 111-121 page 1298 . Halothane hepatitis tends to afflict middle-aged obese women and may have a familial predisposition. However. and it sensitizes the myocardium to the effects of catecholamines. succinylcholine is the only depolarizing agent still used in modern anesthetic practice. upper and lower motor neuron injuries. Saunders. causing fasciculations and eventual flaccid paralysis. During metabolism. producing tachycardia. 2006. and muscle relaxants. Although many types of nondepolarizing agents are used clinically. American College of Surgeons SESAP 13 Category 16 Item 23 Lowers seizure threshold (A) Droperidol (Inapsine) (B) Fentanyl (Sublimaze) (C) Meperidine (Demerol) (D) Midazolam (Versed) (E) Propofol (Diprivan) page 1299 . and reversal agents should always be on hand. and contraindication in patients receiving monoamine oxidase inhibitors has significantly diminished its use. particularly when injected through a small vein. Up to Date Website. than meperidine. and anti-emetic effects. REFERENCES 1. Meperidine Hydrochloride. anti-anxiety. Meperidine (Demerol) is also an opiate with sedative and analgesic properties. Medications are usually used in combination to achieve the desired balance of sedation. Fentanyl citrate (Sublimaze) is an opiate with both sedative and analgesic properties. risk of lowering seizure threshold.Drug Points System: Droperidol.Cohen J: Overview of conscious sedation for gastrointestinal endoscopy. Available at: http://utdol. Propofol (Diprivan) is a sedative-hypnotic agent with no analgesic properties. slower clearance.lhs. MICROMEDEX(R) Healthcare Series Vol. as well as a reduced incidence of nausea. Flumazenil (Romazicon) is used primarily to reverse the sedation and psychomotor impairment associated with benzodiazepines.do?topicKey=gi dis/20270&view=print page 1300 . Of the commonly used agents. and amnesia. Fentanyl Citrate. drug interactions. It is equally efficacious and less expensive than fentanyl. Physicians prescribing conscious sedation must be familiar with precautions. Most endoscopists favor midazolam over diazepam (Valium) because of its fast onset. but the combination of longer onset of action. Propofol. contraindications. Naloxone (Narcan) is an opioid antagonist used to reverse the sedation and respiratory depression caused by opioids such as fentanyl and meperidine. 126 Online Date November 9. and higher amnestic properties. Intravenous infusion of propofol often causes pain on injection. 2005 2. it has the fastest onset as well as clearance and is reversible within minutes of shutting off the infusion. analgesia. It should also not be used with other central nervous system depressants. short duration. and adverse effects.org/utd/content/topic. Droperidol (Inapsine) has neuroleptic. It is less effective in reversing respiratory depression. It is contraindicated in patients with known or suspected QT interval prolongation because it may enhance this effect. Midazolam (Versed) is a benzodiazepine with significant amnestic effects as well as sedation and antianxiety effects.American College of Surgeons SESAP 13 Critique for Category 16 Item 23 Advancements in intravenous pharmacology have dramatically improved the safety and efficacy of conscious sedation and permit greater flexibility and increased complexity of procedures such as flexible endoscopy. 2006. Conscious sedation has the inherent risk of oversedation. dosages. It has a more rapid onset of action and clearance. Midazolam Hydrochloride. American College of Surgeons SESAP 13 Category 16 Item 24 Contraindicated for patients receiving monoamine oxidase (MAO) inhibitors (A) Droperidol (Inapsine) (B) Fentanyl (Sublimaze) (C) Meperidine (Demerol) (D) Midazolam (Versed) (E) Propofol (Diprivan) page 1301 . as well as a reduced incidence of nausea. 2005 2. Most endoscopists favor midazolam over diazepam (Valium) because of its fast onset. Intravenous infusion of propofol often causes pain on injection. than meperidine. and contraindication in patients receiving monoamine oxidase inhibitors has significantly diminished its use. particularly when injected through a small vein. Available at: http://utdol. and adverse effects. Propofol. 126 Online Date November 9. Naloxone (Narcan) is an opioid antagonist used to reverse the sedation and respiratory depression caused by opioids such as fentanyl and meperidine. risk of lowering seizure threshold. REFERENCES 1. Midazolam Hydrochloride. analgesia. Up to Date Website. Conscious sedation has the inherent risk of oversedation. It should also not be used with other central nervous system depressants. contraindications. Physicians prescribing conscious sedation must be familiar with precautions. dosages. Meperidine (Demerol) is also an opiate with sedative and analgesic properties.lhs. Flumazenil (Romazicon) is used primarily to reverse the sedation and psychomotor impairment associated with benzodiazepines. Droperidol (Inapsine) has neuroleptic.Cohen J: Overview of conscious sedation for gastrointestinal endoscopy. It has a more rapid onset of action and clearance. Meperidine Hydrochloride.org/utd/content/topic. It is less effective in reversing respiratory depression. short duration. and higher amnestic properties. MICROMEDEX(R) Healthcare Series Vol. Fentanyl citrate (Sublimaze) is an opiate with both sedative and analgesic properties. drug interactions. Medications are usually used in combination to achieve the desired balance of sedation.American College of Surgeons SESAP 13 Critique for Category 16 Item 24 Advancements in intravenous pharmacology have dramatically improved the safety and efficacy of conscious sedation and permit greater flexibility and increased complexity of procedures such as flexible endoscopy. Of the commonly used agents. Propofol (Diprivan) is a sedative-hypnotic agent with no analgesic properties.do?topicKey=gi dis/20270&view=print page 1302 . but the combination of longer onset of action. and anti-emetic effects. 2006. slower clearance. It is equally efficacious and less expensive than fentanyl. anti-anxiety. and reversal agents should always be on hand. Fentanyl Citrate.Drug Points System: Droperidol. and amnesia. It is contraindicated in patients with known or suspected QT interval prolongation because it may enhance this effect. Midazolam (Versed) is a benzodiazepine with significant amnestic effects as well as sedation and antianxiety effects. it has the fastest onset as well as clearance and is reversible within minutes of shutting off the infusion. American College of Surgeons SESAP 13 Category 16 Item 25 Reversed by flumazenil (A) Droperidol (Inapsine) (B) Fentanyl (Sublimaze) (C) Meperidine (Demerol) (D) Midazolam (Versed) (E) Propofol (Diprivan) page 1303 . and anti-emetic effects. it has the fastest onset as well as clearance and is reversible within minutes of shutting off the infusion. Medications are usually used in combination to achieve the desired balance of sedation. Propofol. and amnesia. Fentanyl Citrate. and adverse effects. 126 Online Date November 9.American College of Surgeons SESAP 13 Critique for Category 16 Item 25 Advancements in intravenous pharmacology have dramatically improved the safety and efficacy of conscious sedation and permit greater flexibility and increased complexity of procedures such as flexible endoscopy. Of the commonly used agents. Midazolam (Versed) is a benzodiazepine with significant amnestic effects as well as sedation and antianxiety effects. particularly when injected through a small vein. Most endoscopists favor midazolam over diazepam (Valium) because of its fast onset.do?topicKey=gi dis/20270&view=print page 1304 . contraindications. MICROMEDEX(R) Healthcare Series Vol. and higher amnestic properties. risk of lowering seizure threshold. Naloxone (Narcan) is an opioid antagonist used to reverse the sedation and respiratory depression caused by opioids such as fentanyl and meperidine. dosages. Meperidine (Demerol) is also an opiate with sedative and analgesic properties.Drug Points System: Droperidol. slower clearance. Droperidol (Inapsine) has neuroleptic. Fentanyl citrate (Sublimaze) is an opiate with both sedative and analgesic properties. than meperidine. Conscious sedation has the inherent risk of oversedation. Midazolam Hydrochloride. Propofol (Diprivan) is a sedative-hypnotic agent with no analgesic properties. It should also not be used with other central nervous system depressants.Cohen J: Overview of conscious sedation for gastrointestinal endoscopy. 2006. Physicians prescribing conscious sedation must be familiar with precautions.org/utd/content/topic. Flumazenil (Romazicon) is used primarily to reverse the sedation and psychomotor impairment associated with benzodiazepines. Up to Date Website.lhs. It has a more rapid onset of action and clearance. anti-anxiety. Available at: http://utdol. REFERENCES 1. It is equally efficacious and less expensive than fentanyl. and reversal agents should always be on hand. 2005 2. analgesia. and contraindication in patients receiving monoamine oxidase inhibitors has significantly diminished its use. Intravenous infusion of propofol often causes pain on injection. It is contraindicated in patients with known or suspected QT interval prolongation because it may enhance this effect. It is less effective in reversing respiratory depression. Meperidine Hydrochloride. drug interactions. as well as a reduced incidence of nausea. short duration. but the combination of longer onset of action. American College of Surgeons SESAP 13 Category 16 Item 26 Most likely to cause injection site pain (A) Droperidol (Inapsine) (B) Fentanyl (Sublimaze) (C) Meperidine (Demerol) (D) Midazolam (Versed) (E) Propofol (Diprivan) page 1305 . short duration. and contraindication in patients receiving monoamine oxidase inhibitors has significantly diminished its use. and adverse effects.do?topicKey=gi dis/20270&view=print page 1306 . anti-anxiety. Conscious sedation has the inherent risk of oversedation. and reversal agents should always be on hand. It is contraindicated in patients with known or suspected QT interval prolongation because it may enhance this effect. MICROMEDEX(R) Healthcare Series Vol. it has the fastest onset as well as clearance and is reversible within minutes of shutting off the infusion. Medications are usually used in combination to achieve the desired balance of sedation. Droperidol (Inapsine) has neuroleptic. 2005 2. Naloxone (Narcan) is an opioid antagonist used to reverse the sedation and respiratory depression caused by opioids such as fentanyl and meperidine. drug interactions. and anti-emetic effects.org/utd/content/topic. but the combination of longer onset of action. Meperidine (Demerol) is also an opiate with sedative and analgesic properties. 126 Online Date November 9. particularly when injected through a small vein.American College of Surgeons SESAP 13 Critique for Category 16 Item 26 Advancements in intravenous pharmacology have dramatically improved the safety and efficacy of conscious sedation and permit greater flexibility and increased complexity of procedures such as flexible endoscopy. slower clearance. dosages. Intravenous infusion of propofol often causes pain on injection. as well as a reduced incidence of nausea. analgesia. and amnesia. It has a more rapid onset of action and clearance. Propofol (Diprivan) is a sedative-hypnotic agent with no analgesic properties. Available at: http://utdol. Up to Date Website. It is less effective in reversing respiratory depression. Physicians prescribing conscious sedation must be familiar with precautions.Cohen J: Overview of conscious sedation for gastrointestinal endoscopy. Midazolam Hydrochloride. It should also not be used with other central nervous system depressants. and higher amnestic properties. contraindications. than meperidine. 2006. Midazolam (Versed) is a benzodiazepine with significant amnestic effects as well as sedation and antianxiety effects. Most endoscopists favor midazolam over diazepam (Valium) because of its fast onset. Flumazenil (Romazicon) is used primarily to reverse the sedation and psychomotor impairment associated with benzodiazepines. It is equally efficacious and less expensive than fentanyl. risk of lowering seizure threshold.Drug Points System: Droperidol. Meperidine Hydrochloride. Fentanyl Citrate. REFERENCES 1. Propofol.lhs. Of the commonly used agents. Fentanyl citrate (Sublimaze) is an opiate with both sedative and analgesic properties. American College of Surgeons SESAP 13 Category 16 Item 27 Prolongs Q-T interval (A) Droperidol (Inapsine) (B) Fentanyl (Sublimaze) (C) Meperidine (Demerol) (D) Midazolam (Versed) (E) Propofol (Diprivan) page 1307 . Available at: http://utdol. MICROMEDEX(R) Healthcare Series Vol. short duration. analgesia. Medications are usually used in combination to achieve the desired balance of sedation. and reversal agents should always be on hand. Conscious sedation has the inherent risk of oversedation. Of the commonly used agents. It has a more rapid onset of action and clearance. Flumazenil (Romazicon) is used primarily to reverse the sedation and psychomotor impairment associated with benzodiazepines. dosages. Meperidine Hydrochloride. drug interactions. It is less effective in reversing respiratory depression. It is contraindicated in patients with known or suspected QT interval prolongation because it may enhance this effect. and amnesia. Fentanyl citrate (Sublimaze) is an opiate with both sedative and analgesic properties. slower clearance.Drug Points System: Droperidol. and contraindication in patients receiving monoamine oxidase inhibitors has significantly diminished its use. Fentanyl Citrate. Propofol. REFERENCES 1. risk of lowering seizure threshold. 2005 2.lhs.American College of Surgeons SESAP 13 Critique for Category 16 Item 27 Advancements in intravenous pharmacology have dramatically improved the safety and efficacy of conscious sedation and permit greater flexibility and increased complexity of procedures such as flexible endoscopy. contraindications. but the combination of longer onset of action. It should also not be used with other central nervous system depressants. than meperidine. Naloxone (Narcan) is an opioid antagonist used to reverse the sedation and respiratory depression caused by opioids such as fentanyl and meperidine. Up to Date Website. it has the fastest onset as well as clearance and is reversible within minutes of shutting off the infusion. Intravenous infusion of propofol often causes pain on injection. It is equally efficacious and less expensive than fentanyl. particularly when injected through a small vein.org/utd/content/topic.do?topicKey=gi dis/20270&view=print page 1308 . and higher amnestic properties. Physicians prescribing conscious sedation must be familiar with precautions. Most endoscopists favor midazolam over diazepam (Valium) because of its fast onset.Cohen J: Overview of conscious sedation for gastrointestinal endoscopy. Droperidol (Inapsine) has neuroleptic. Propofol (Diprivan) is a sedative-hypnotic agent with no analgesic properties. Meperidine (Demerol) is also an opiate with sedative and analgesic properties. 126 Online Date November 9. 2006. anti-anxiety. and anti-emetic effects. and adverse effects. Midazolam Hydrochloride. as well as a reduced incidence of nausea. Midazolam (Versed) is a benzodiazepine with significant amnestic effects as well as sedation and antianxiety effects. American College of Surgeons SESAP 13 Category 16 Item 28 A common drug that can trigger an episode of malignant hyperthermia (A) Propofol (B) Dantrolene (C) Ketamine (D) Succinylcholine (E) Neostigmine page 1309 . aggressive cooling measures.to 10-minute duration of action. hallucinations.Backman SB. decreasing intracellular calcium and thereby attenuating the crisis. 2006.[No Authors Listed]: Basic principles of pharmacology in anesthesia practice. ed 5. 2006. Stoelting RK (eds): Handbook of Clinical Anesthesia. 2006. Philadelphia. a dissociative anesthetic. Triggering agents cause a build-up of calcium in the myoplasm. ed 5. Cullen BF. Succinylcholine. potentially fatal hypermetabolic reaction to volatile anesthetics or depolarizing muscle relaxants.[No Authors Listed]: Muscle relaxants. Appropriate supportive measures will sustain life until the underlying defect has resolved. Philadelphia. Kaiser LR. Malignant hyperthermia is a rare. dantrolene appears to block release of calcium from the sarcoplasmic reticulum to the myoplasm. ed 5. to render the patient unconscious and facilitate intubation. et al (eds): ACS Surgery: Principles and Practice 2006. leading to transient sensory deprivation and resultant hallucinations. The principal goal of reversal of neuromuscular blockade is re-establishment of spontaneous respiration and the ability to protect the airway. Lippincott Williams & Wilkins. apnea. and administration of dantrolene. Stoelting RK (eds): Handbook of Clinical Anesthesia.American College of Surgeons SESAP 13 Critique for Category 16 Item 28 Propofol is a sedative drug commonly used for induction of general anesthesia. chap 1. Hallucinations and/or emergence delirium have limited its usefulness as an anesthetic agent. Lippincott Williams & Wilkins. Side effects include hypotension. Moore A. pp 235-254 4. pp 272-389 page 1310 . Cullen BF. New York. Affected patients have a genetic predisposition. Ketamine. Abnormal function of the calcium release channel in the sarcoplasmic reticulum of skeletal muscle may be the underlying cause. et al: Perioperative considerations for anesthesia. Basic Surgical and Perioperative Considerations. is used for induction of general anesthesia. Reversal involves the reduction of the effect of competitive blocking drugs by increasing the concentration of acetylcholine at the neuromuscular junction. in Souba WW. The resultant rise in acetylcholine at the synapse reverses the effect of nondepolarizing neuromuscular blockers at the completion of anesthesia. REFERENCES 1. Deschamps A. Bondy RM. Philadelphia. pp 120-271 3. Dissociative drugs block afferent sensory signals to the cerebral cortex. Fink MP. It is also used in the ICU for prolonged sedation of patients with elevated intracranial pressure or ventilator dependency. Stoelting RK (eds): Handbook of Clinical Anesthesia. in Barash PG. and seizures. It has a rapid onset (90 to 100 seconds ) and a 5. pp 46-59 2. It has been used in recent years as a recreational drug. Cullen BF. WebMD. in Barash PG. in Barash PG. Jurkovich GJ. sec 3. and a careful pre-anesthetic history identifying family members who have had similar reactions should alert the team and lead to appropriate testing. Lippincott Williams & Wilkins. Neostigmine is an anticholinesterase agent that blocks the breakdown of acetylcholine.[No Authors Listed]: Preparing for anesthesia. Therapy includes discontinuation of inciting agents. a depolarizing muscle relaxant. is well known as a triggering agent. American College of Surgeons SESAP 13 Category 16 Item 29 Emergence delirium (A) Propofol (B) Dantrolene (C) Ketamine (D) Succinylcholine (E) Neostigmine page 1311 . ed 5.[No Authors Listed]: Basic principles of pharmacology in anesthesia practice. hallucinations. Neostigmine is an anticholinesterase agent that blocks the breakdown of acetylcholine. 2006. is used for induction of general anesthesia. pp 235-254 4. It has been used in recent years as a recreational drug. Stoelting RK (eds): Handbook of Clinical Anesthesia. It has a rapid onset (90 to 100 seconds ) and a 5. 2006. Triggering agents cause a build-up of calcium in the myoplasm. 2006. chap 1.Backman SB. Hallucinations and/or emergence delirium have limited its usefulness as an anesthetic agent.[No Authors Listed]: Muscle relaxants. pp 120-271 3. Malignant hyperthermia is a rare. aggressive cooling measures. in Souba WW. Therapy includes discontinuation of inciting agents. The principal goal of reversal of neuromuscular blockade is re-establishment of spontaneous respiration and the ability to protect the airway. is well known as a triggering agent. and seizures. Basic Surgical and Perioperative Considerations. and administration of dantrolene. pp 46-59 2. Lippincott Williams & Wilkins. Deschamps A. Reversal involves the reduction of the effect of competitive blocking drugs by increasing the concentration of acetylcholine at the neuromuscular junction. et al (eds): ACS Surgery: Principles and Practice 2006. Cullen BF.American College of Surgeons SESAP 13 Critique for Category 16 Item 29 Propofol is a sedative drug commonly used for induction of general anesthesia. Stoelting RK (eds): Handbook of Clinical Anesthesia. Fink MP. in Barash PG. ed 5. WebMD.to 10-minute duration of action. Lippincott Williams & Wilkins. Abnormal function of the calcium release channel in the sarcoplasmic reticulum of skeletal muscle may be the underlying cause. pp 272-389 page 1312 . apnea. Side effects include hypotension. potentially fatal hypermetabolic reaction to volatile anesthetics or depolarizing muscle relaxants. leading to transient sensory deprivation and resultant hallucinations. to render the patient unconscious and facilitate intubation. decreasing intracellular calcium and thereby attenuating the crisis. REFERENCES 1. Succinylcholine. Cullen BF. Ketamine. ed 5. Philadelphia. Bondy RM. in Barash PG. Philadelphia. and a careful pre-anesthetic history identifying family members who have had similar reactions should alert the team and lead to appropriate testing. Lippincott Williams & Wilkins. Affected patients have a genetic predisposition. Appropriate supportive measures will sustain life until the underlying defect has resolved.[No Authors Listed]: Preparing for anesthesia. New York. Philadelphia. in Barash PG. et al: Perioperative considerations for anesthesia. Stoelting RK (eds): Handbook of Clinical Anesthesia. Kaiser LR. It is also used in the ICU for prolonged sedation of patients with elevated intracranial pressure or ventilator dependency. dantrolene appears to block release of calcium from the sarcoplasmic reticulum to the myoplasm. a depolarizing muscle relaxant. Dissociative drugs block afferent sensory signals to the cerebral cortex. Cullen BF. Moore A. a dissociative anesthetic. Jurkovich GJ. sec 3. The resultant rise in acetylcholine at the synapse reverses the effect of nondepolarizing neuromuscular blockers at the completion of anesthesia. American College of Surgeons SESAP 13 Category 16 Item 30 Can be given to patients to reverse muscle blockade from muscle relaxants (A) Propofol (B) Dantrolene (C) Ketamine (D) Succinylcholine (E) Neostigmine page 1313 . Cullen BF. Abnormal function of the calcium release channel in the sarcoplasmic reticulum of skeletal muscle may be the underlying cause. WebMD. Basic Surgical and Perioperative Considerations.to 10-minute duration of action. Affected patients have a genetic predisposition. decreasing intracellular calcium and thereby attenuating the crisis. is used for induction of general anesthesia. The principal goal of reversal of neuromuscular blockade is re-establishment of spontaneous respiration and the ability to protect the airway. Philadelphia.[No Authors Listed]: Basic principles of pharmacology in anesthesia practice. a depolarizing muscle relaxant. Lippincott Williams & Wilkins. in Souba WW. ed 5. and a careful pre-anesthetic history identifying family members who have had similar reactions should alert the team and lead to appropriate testing. a dissociative anesthetic. in Barash PG. chap 1.[No Authors Listed]: Preparing for anesthesia. to render the patient unconscious and facilitate intubation. ed 5. It has been used in recent years as a recreational drug. Side effects include hypotension. Cullen BF. is well known as a triggering agent. sec 3. potentially fatal hypermetabolic reaction to volatile anesthetics or depolarizing muscle relaxants. leading to transient sensory deprivation and resultant hallucinations. New York. Kaiser LR. hallucinations. REFERENCES 1. 2006. It has a rapid onset (90 to 100 seconds ) and a 5. Stoelting RK (eds): Handbook of Clinical Anesthesia. and seizures. ed 5. Stoelting RK (eds): Handbook of Clinical Anesthesia. et al (eds): ACS Surgery: Principles and Practice 2006. Jurkovich GJ. pp 46-59 2. Deschamps A. et al: Perioperative considerations for anesthesia. Ketamine. It is also used in the ICU for prolonged sedation of patients with elevated intracranial pressure or ventilator dependency. Lippincott Williams & Wilkins. Bondy RM. 2006. Moore A. Philadelphia.American College of Surgeons SESAP 13 Critique for Category 16 Item 30 Propofol is a sedative drug commonly used for induction of general anesthesia.Backman SB. Fink MP. pp 120-271 3. in Barash PG. Succinylcholine. Appropriate supportive measures will sustain life until the underlying defect has resolved. apnea. Stoelting RK (eds): Handbook of Clinical Anesthesia. The resultant rise in acetylcholine at the synapse reverses the effect of nondepolarizing neuromuscular blockers at the completion of anesthesia. Triggering agents cause a build-up of calcium in the myoplasm. Therapy includes discontinuation of inciting agents. Hallucinations and/or emergence delirium have limited its usefulness as an anesthetic agent. in Barash PG. Dissociative drugs block afferent sensory signals to the cerebral cortex. 2006. Philadelphia. dantrolene appears to block release of calcium from the sarcoplasmic reticulum to the myoplasm. Malignant hyperthermia is a rare. pp 235-254 4. Cullen BF. Lippincott Williams & Wilkins.[No Authors Listed]: Muscle relaxants. pp 272-389 page 1314 . and administration of dantrolene. Reversal involves the reduction of the effect of competitive blocking drugs by increasing the concentration of acetylcholine at the neuromuscular junction. aggressive cooling measures. Neostigmine is an anticholinesterase agent that blocks the breakdown of acetylcholine. and pneumatosis. bowel ischemia. but dialysis should not be contemplated page 1315 .American College of Surgeons SESAP 13 Category 17 Item 1 An 80-year-old woman with advanced Alzheimer’s disease has been ill for 4 days and is transferred from the nursing home with fever. Computed tomographic (CT) scan reveals a superior mesenteric artery thrombosis. She is acidotic and in acute renal failure. Surgical intervention would necessitate resection of a significant length of small and large bowel. Which of the following statements about this patient’s care is TRUE? (A) She is dying and palliative care is indicated (B) The decision for further careshould be left solely with the family (C) Operation is the only chance for cure of the bowel ischemia and should be performed (D) Aggressive ICU resuscitation is warranted and operation should be performed if the patient stabilizes (E) Operation should be performed. hypotension. and abdominal swelling. A second opinion may be appropriate if there is any level of concern on the part of the family. 1990 3. REFERENCES 1.HinshawDB.SchneidermanLJ. and how do we do it? Medical futility and withdrawal of care.YoungnerSJ: Who defines futility? JAMA 260:2094-2095. As many as 80% of dying patients are unable to speak for themselves and fewer than 25% provide their caregivers with advance directives. JonsenAR: Medical futility: its meaning and ethical implications. and can be qualitative or quantitative. The role of the surgeon in the management of this patient is to explain these issues to the family and assist them with understanding the medical issues of this process and why she/he is making the recommendation. Ann Intern Med 112:949-954. 1988 page 1316 . JeckerNS. PawlikT. including consideration of transfer of care to a surgeon who will provide the requested care. This patient is dying. Futility includes predictable failure. 2003 2. but the family is still concerned. the surgeon is responsible for ensuring care of the patient. et al: When do we stop. or treatment without a potentially beneficial outcome. MosenthalAC. Although this surgeon is not ethically obligated to operate on this patient in violation of his/her judgment. an ethics consultation should be obtained. J Am Coll Surg 196:621-651. If the second opinion concurs. Civetta JM.American College of Surgeons SESAP 13 Critique for Category 17 Item 1 Futile care is a potentially complicated subject that defies easy definition. mandatory reporting does not apply (D) Almost all victims of domestic violence seen in emergency departments support mandatory reporting (E) Using specific screening questions in the emergency department does not increase the detection of domestic violence page 1317 .American College of Surgeons SESAP 13 Category 17 Item 2 Which of the following statements about mandatory reporting in cases of domestic violence is TRUE? (A) Physicians in all states must report cases of domestic violence to the police (B) Mandatory reporting does not deter victims of domestic violence from seeking medical attention (C) Because domestic violence is not a crime. but the details of mandatory reporting vary greatly from state to state. 50% of patients were aware of mandatory reporting laws. there is no diagnostic profile that identifies victims. Ann Emerg Med 34:336-341. Abbott J: Mandatory reporting laws do not deter patients from seeking medical care. Sachs C: Preventive care in the emergency department: screening for domestic violence in the emergency department. punched or otherwise hurt by someone in the past year. pp 275-282 2. Interestingly.American College of Surgeons SESAP 13 Critique for Category 17 Item 2 Domestic violence has been defined as a pattern of coercive control consisting of physical. Acad Emerg Med 10:1118-1127. The Partner Violence Screen consists of three questions: have you been kicked. et al: Violence in America: a public health crisis—domestic violence. 2003 3. et al: Accuracy of 3 brief screening questions for detecting partner violence in the emergency department. The incidence is probably underestimated because physicians may not ask the right questions to identify victims.Sisley A. and several screening tools have been studied to better identify these patients. Feldhaus K. most states have laws that require health care providers to report injuries inflicted by a weapon. do you feel safe in your current relationship. JCAHO requires that emergency departments have formal policies. Because of heightened awareness. Jacobs LM. and/or psychological assault against former or current intimate partners. Mandatory reporting laws may be seen as barriers to disclosure for some women. Koziol-McLain J. Women rarely volunteer information. Most cases involve women as victims of domestic violence by male perpetrators. and domestic violence is reported to occur fairly uniformly across demographic categories with prevalence varying little from public hospitals to private clinics. Identification of victims and reporting crime are just two areas of importance in the area of domestic violence. Increasing the number of specialized health services such as immediate counseling by personnel specifically trained in domestic violence remains a need in many hospitals.American College of Surgeons: Trauma in women. ed 7. and education in place specifically aimed at domestic violence. 2004. especially when seen in the acute care setting.Anglin D. Norton IM. but only 12% said that they would be less likely to seek medical care for a domestic violence related injury because of these laws. Advanced Trauma Life Support Program for Doctors. 1997 4. Poole G. American College of Surgeons. 1999 page 1318 . Amsbury HL. JAMA 277:1357-1361. in ATLS. REFERENCES 1. and is there a partner from a previous relationship who is making you feel unsafe now? Using this tool identifies 65% to 70% of women who have been victims of domestic violence. In a survey of patients presenting to the emergency department or to primary care clinics. J Trauma 46:1105-1112. hit. Thorson A.Feldhaus KM. Campbell S. sexual. procedures. Chicago.Houry D. 1999 5. the surgeon performs a final time-out as he begins incising the skin for the herniorrhaphy. Both of them verify with him his identity and the procedure to be performed. A mark is placed on his right foot to indicate the operative side. In the operating room. After registering at the outpatient desk. he reports to preoperative holding where he is seen by both the circulating nurse and the anesthesiologist. In this scenario.American College of Surgeons SESAP 13 Category 17 Item 3 A 15-year-old boy is scheduled for an elective open right inguinal hernia repair. Wrong Procedure. Wrong Person Surgery were properly implemented? (A) None (B) One (C) Two (D) Three (E) Four page 1319 . how many components of the JCAHO’s Universal Protocol For Preventing Wrong Site. The JCAHO believes that any sentinel event should trigger a root cause analysis (RCA) designed to understand both the latent (organizational) and immediate conditions leading to the event in order to prevent further occurrences. Active communication between members of the operative team verifying the procedure. is an attempt to introduce a national. only the preoperative verification process was properly implemented. Wrong site surgery is included in this data base. In an attempt to eliminate these devastating events. and the time out was performed after the procedure had begun. The JCAHO maintains a data base of voluntarily reportable sentinel events. Other important root causes in descending order included improper training. The time out should be performed just prior to starting the procedure.Joint Commission on Accreditation of Healthcare Organizations: A follow-up review of wrong site surgery.org/PatientSafety/UniversalProtocol/ 3. and poor leadership. REFERENCES 1. Wrong SurgeryTM.jointcommission. in collaboration with other healthcare organizations. In 2004 alone.jointcommission. approximately 70 cases of wrong site surgery were reported. This document. endorsed by over 50 healthcare associations. The preoperative verification is an ongoing process to obtain all pertinent information before the procedure and ensure that it is consistent with the patient’s and team members’ expectations. and a final time out.org/SentinelEvents/SentinelEventAlert/sea_24. Wrong Procedure. Wrong Person Surgery.American College of Surgeons SESAP 13 Critique for Category 17 Item 3 The Joint Commission on Accreditation of Healthcare Organizations (JCAHO) defines a sentinel event as an unexpected occurrence resulting in actual or potential death or serious injury. the JCAHO. Issue 24. improper patient assessment. RCA of the 370 incidents of wrong site surgery reported to the JCAHO from 1995 to 2004 indicates that poor communication was a root cause in almost 80% of cases. WebMD. http://www. http://www. In the case described. and patient is required.jointcommission. The marking must be made so that it is visible after the patient is prepped and draped.Joint Commission on Accreditation of Healthcare Organizations: Joint Commission sentinel event statistics.Rhodes RS: Patient safety in surgical care: a systems approach. limited availability of information. The three major components include a preoperative verification process. 2001. 2004. developed The Universal Protocol For Preventing Wrong Site. The site marking on the foot would not be visible after patient draping. Failure to provide adequate documentation of its use during a site visit by the JCAHO could jeopardize a hospital’s accreditation. operative site marking. reproducible protocol for preventing these errors. Wrong Procedure. procedural noncompliance. in ACS Surgery Principles and Practice CDROM.org/SentinelEvents/Statistics/ 4. Available at: http://www. Operative site marking is required in cases involving right/left sided or multiple level procedures. Sentinel Event Alert.Joint Commission on Accreditation of Healthcare Organizations: Universal Protocol for Preventing Wrong Site. Use of the protocol has been mandatory since July 1. Such harm can be physical or psychological. 2004 page 1320 .htm 2. site. American College of Surgeons SESAP 13 Category 17 Item 4 Which of the following practices would help promote a culture of safety in the operating room environment? (A) Maintaining strict accountability for mistakes (B) Emphasizing timely completion of procedures (C) Addressing everyone by formal title (D) Reporting problems face to face (E) Stressing role flexibility in crisis situations page 1321 . Rhodes RS: Patient safety in surgical care: a systems approach. but typically reflects an accumulation of design flaws within the complex. and flexibility in adapting to new situations or problems. Maintaining strict accountability for mistakes would be detrimental to reporting errors and near misses. 2004 page 1322 . Stressing flexibility with roles in a crisis situation is a key element of a culture of safety. in ACS Surgery Principles and Practice CDROM.Corrigan J. Although accountability is less important than learning.American College of Surgeons SESAP 13 Critique for Category 17 Item 4 It has been estimated that medical errors annually lead to as many as 98. Using first names fosters a flattened hierarchy. REFERENCES 1. Fostering a culture of learning requires open. Face-to-face reporting of problems might prevent some from speaking up. a just system delineating appropriate from inappropriate behavior should be established. The use of formal titles creates a rigid hierarchical structure. and rigid hierarchical structures should be replaced with models emphasizing the practice of authority with participation and encouraging assertiveness with respect. Because of the active role of surgeons in each of the in-hospital environments at most risk for error development. WebMD. Washington DC. with potentially more serious outcomes.000 preventable deaths and contribute to nearly $29 billion in additional total costs. J Leg Med 24:7-27. ICUs. A culture of safety within healthcare at all levels of care is needed to address this problem. 2005 3. 2003 2. Key characteristics include a confidential system of reporting errors or near misses.Clarke JR: Making surgery safer. an emphasis on learning over accountability. An emphasis on the timely completion of procedures would create a latent condition within the system in which safety might be compromised for productivity. An organization or group committed to a culture of safety places it above all other concerns.Barach P: The end of the beginning: lessons learned from the patient safety movement. honest interaction between all members of the group. Anonymous systems ensure confidentiality. and interdependent healthcare delivery system that can result in sequential breakdowns (the Swiss cheese paradigm). Kohn LT. editors: To Err is Human: Building a Safer Health System. National Academy Press. dynamic. Medical error is not the result of an isolated action of a certain person or persons (the bad apple paradigm). 2000 4. the surgeon’s leadership in promoting such a culture is imperative. Donaldson MS. and emergency departments are all environments at high risk for error occurrence. J Am Coll Surg 200:229-235. Operating rooms. Doe. and I am in charge (B) We’ve got a lot to do.American College of Surgeons SESAP 13 Category 17 Item 5 Which of the following statements is an example of communication promoting improved team functioning in the operating room? (A) Hello. my name is Dr. so let’s be sure we’re all on the same page (C) That is totally wrong! No wonder you’re a resident (D) Quiet! I’ve got a problem here! (E) Are you sure about that? page 1323 . sccm. In another study. Bellows C. Answer E implies concern regarding an action. communication breakdown was responsible for 43% of all surgical adverse events in three teaching hospitals. it is often inadequate. An important characteristic of a highly reliable team is the use of flexible and open communication that allows it to adapt to dynamic changes in the environment and foster a shared mental model. Available at: http://sccmwww.Gottlieb Daimler and Karl Benz Foundation.asp1n1 page 1324 . In the scenario here.Awad SS. Unfortunately. and D reinforce a rigid hierarchical system in which open communication is discouraged. stating concerns in a direct. The use of the first person plural in the team environment fosters a team perspective and increases as team members become more familiar with one another.org/publications/eNewsletters_Archive/03_03_2005. 2004 3.Rhodes RS: Patient safety in surgical care: a systems approach. 2004 4. proper communication within and between the numerous teams of healthcare personnel taking care of a patient is essential. unambiguous manner. REFERENCES 1.American College of Surgeons SESAP 13 Critique of Category 17 Item 5 In a healthcare delivery system that has become increasingly complex and dynamic.The Society of Critical Care Medicine: Safety tip 15 — just say we/let’s. SCCM Website. March 3 2005. Fagan SP. thinking that the communication among the team is good at the same time other members perceive it as poor. the safer the world: golden rules of group interaction in high risk environments: evidenced based suggestions for improving performance. At one institution. the introduction of a standardized preoperative briefing designed to enhance communication improved the rates of proper perioperative deep vein thrombosis and antibiotic prophylaxis. Am J Surg 190:770-774. Its use also correlates with better team performance and fewer team errors. in ACS Surgery Principles and Practice CDROM. Answer B fosters a team perspective by using the first person plural and helps promote a shared mental model by requesting everyone to be on the same page. Junior members of the team should be explicit in their communication. such communication should be simple and use standard phrases in order to direct as much mental attention to the task at hand. C. Swiss Re Centre for Global Dialogue: The better the team. Use language to foster team perspective. WebMD. Ladenburg and Ruschlikon. In a high workload situation. communication events between members of the operating room team resulted in communication failure 30% of the time. Surgeons are often unaware of this situation. Improved team communication among operating room personnel improves the care given patients. but is not explicit regarding the concern. et al: Bridging the communication gap in the operating room with medical team training. In one study. 2005 2. Albo D. answers A. He now requires hospitalization for pain relief. In the 4 weeks since his diagnosis.American College of Surgeons SESAP 13 Category 17 Item 6 A 66-year-old man with nonresectable pancreatic cancer has significant abdominal pain. his pain has steadily increased despite increasing oral dosages of narcotics. most likely caused by the tumor. He is a smoker and has mild chronic obstructive pulmonary disease (COPD). Which of the following statements is TRUE? (A) Adding serotonin antagonists to oral narcotics should be tried prior to intravenous narcotics (B) Initiation of intravenous narcotics raises serious ethical concerns (C) Intravenous narcotics can be given in increasing dosages without respiratory side effects (D) Celiac block will offer better pain relief at this time (E) A change in opioid type will usually enhance pain control page 1325 . All physicians are aware of the dangerous side effects of opioids.Thompson AR. New York. J Am Coll Surg 196:321-324. 2003 page 1326 . particularly with pain relief medication. The tolerance effects will minimize the possibility of untoward respiratory side effects. Schuster JL. opioids at any dose do not cause significant visceral organ damage. 2001 2. treatment for a patient suffering enough to warrant such treatment may be initiated if the intent is to relieve suffering. and the ongoing tolerance then is associated with a lowered incidence of significant respiratory depression. Schuster JL. An increasing literature describes a number of unique pharmacologic properties of opioid use and development of tolerance.Lee KF. Unlike nonsteroidal anti-inflammatory drugs (NSAIDs). particularly respiratory depression. pp 59-72 3. Many physicians are concerned about prescribing increasing doses of opioids or narcotics in terminally ill patients. Under this principle. is not as completely understood. Dunn GP: Chronic pain management and the surgeon: barriers and opportunities. However. and none of the other possible answers are applicable or relevant at this time. and acetaminophen. vomiting. Because there is no plateau on the dose response curve. His mild chronic obstructive pulmonary disease (COPD) is not a contraindication. 2000. A tolerance to these effects develops. intravenous narcotics can be instituted with increasing doses. even for long-term use. which can cause irreversible renal damage. and mental aberration. the safety of opioids. In this patient. REFERENCES 1.American College of Surgeons SESAP 13 Critique of Category 17 Item 6 This scenario deals with the use of increasing doses of narcotics for patients with unresectable malignancies. Ray JB: The importance of opioid tolerance: a therapeutic paradox. Increasing opioid levels can cause nausea. Kabcenell A (eds): Improving Care for the End of Life: A Sourcebook for Health Care Managers and Clinicians. Oxford University Press. J Am Coll Surg 193:689-701. Opioids also do not have a plateau dose beyond which increasing doses are ineffective. but these effects are all reversible with dose reduction and no permanent organ damage has been documented. there is no ceiling for opioid effect: increased opioid doses will continue to have pain relief effect. which can produce hepatic necrosis. Concern about inherent dangers often leads to under-prescription and ineffective pain relief. The double effect doctrine is used to assess whether narcotic prescription or any other treatment with some risk of causing death is justifiable in terminally ill patients. None of the other options are principles of pain management in palliative care. Kabcenell A: Managing dyspnea and ventilator withdrawal. Increasing the opioid dose allows continued and effective pain relief.Lynn J. This is unique. Ray JB. in Lynn J. The next day. he has an advance directive stating that feedings and intravenous fluids should be discontinued if his condition worsens. Which of the following statements is TRUE? (A) Federal law requires that intravenous hydration be given (B) A metabolic work-up is necessary and will likely discover the cause of the patient’s coma (C) Discontinuing intravenous hydration may help with pain control and comfort measures (D) Central venous access can be performed for certain comfort measures (E) The patient’s son may overrule the patient’s wishes and have the tube feedings increased page 1327 .American College of Surgeons SESAP 13 Category 17 Item 7 An 86-year-old man with colon cancer and widespread hepatic metastasis is admitted for volume depletion. He has been receiving feedings through a previously placed jejunal tube. On admission. he is comatose. nor should intravenous fluids be given if he is later unable to make his own decisions.Easson AM. It is also fairly clear that cachexia in this clinical situation has resulted in profound metabolic changes that cannot and will not be reversed by adding or increasing ongoing short-term nutritional supplementation. The patient has clearly and directly stated that parenteral feeding should not be increased. 2002 page 1328 . In the terminally ill patient. no federal directives govern the physician’s role in placing access for hydration or nutrition in a patient who has clearly professed that he does not desire this. starvation and dehydration might result in the increased production of natural endorphins. 2002 2. This is associated with a possible natural anesthetic through resultant euphoria. J Am Coll Surg 194:225-228. there is little to suggest that he would now desire placement of a central line. In this scenario. REFERENCES 1. nor does the son have the right to overrule his father’s wishes so directly. Both of these controversial areas can lead to agonizing choices if and when the patient cannot speak for himself. J Am Coll Surg 194:835-839. Hinshaw DB.American College of Surgeons SESAP 13 Critique for Category 17 Item 7 This clinical scenario describes the terminal care of an intelligent 86-year-old able to make his own decisions. possibly resulting in pain alleviation and general unawareness. but its role in the patient with severe cachexia due to overwhelming cancer suggests that other considerations will need to be made.Huffman JL. Dunn GP: The paradox of hydration in advanced terminal illness. Enteral nutrition has produced advances for patients with poor nutrition. making (C) the best answer for this question. When the patient’s directives were clear and clearly stated. Johnson DL: The role of tube feeding and total parenteral nutrition in advanced illness. methods for data collection. statistical methods. and protections for human subjects (E) Verbally inform the head of the IRB that the procedure is scheduled without obtaining written consent page 1329 . risks. and alternatives (C) Obtain a generalized consent for surgical treatment using a standard consent form without discussion of specific risks and benefits (D) Submit a formal research study to the IRB describing the procedure. stressing the uncertainties. including the surgeon’s previous experience.American College of Surgeons SESAP 13 Category 17 Item 8 Which of the following statements most accurately reflects a surgeon’s legal and ethical responsibilities when planning a novel surgical procedure? (A) Obtain informed consent in the usual manner and then a waiver for experimental surgery from the institutional review board (IRB) (B) Completely inform the patient of the surgical plan. Answers (A) and (E) are incorrect because the IRB has no jurisdiction over the practice of medicine. Answer (B) is correct because it respects the principles of complete informed consent. Complete informed consent requires a thorough discussion of the planned procedures. 2004 page 1330 . the completeness of consent is the most important ethical consideration. Richman BW: Ethics of surgical innovation to treat rare diseases. The practice of surgery includes procedures that are designed solely to enhance the well-being of an individual patient. and the uncertainty associated with long-term outcomes. and report generalized conclusions.American College of Surgeons SESAP 13 Critique for Category 17 Item 8 The Belmont Report in 1979 established the ethical principles governing the protection of human subjects of biomedical research. Answer (D) is incorrect because there is no plan to undertake formal medical research. the surgeon has no obligation to obtain approval from the institutional review board (IRB). In the decision to perform a novel surgical procedure. REFERENCES 1.Jones JW. Br J Surg 88:897-898. McCullough LB. When the purpose of a novel operation is the treatment of a single patient with a unique clinical problem rather than biomedical research.Gillett G: Ethics of surgical innovation. then approval to perform biomedical research should be sought from the appropriate governing bodies. 2001 2. gather and analyze data. detailed description of unique risks and benefits associated with the new procedure. the alternative treatments if any. J Vasc Surg 39:918-919. If the surgeon intends to treat a series of patients in a similar fashion. Answer (C) is incorrect because adequate informed consent should always include specific discussion of appropriate risks and benefits of the proposed treatment. the surgeon’s personal experience with the procedure and qualifications for performing a novel surgical technique. American College of Surgeons SESAP 13 Category 17 Item 9 A 92-year-old woman with chronic renal insufficiency has the sudden onset of abdominal pain and is rushed to the hospital by her family. Ultrasonography reveals a ruptured abdominal aortic aneurysm. Which of the following is the best option? (A) Attempt repair (B) Honor the patient’s advance directive (C) Transfer to a hospital with endovascular capabilities (D) Perform slow code in the operating room (E) Call for an ethics committee consultation page 1331 . but she has an advance directive (living will) declining heroic measures. En route she loses consciousness. The facility does not have endovascular surgery capabilities. The patient’s family insists that operative repair be attempted. In the emergency department the patient suffers a cardiac arrest but is resuscitated at the request of her family. Medical power of attorney is a way for a patient to designate who will be making medical decisions should the patient become unable to make decisions.Crane MK.5 are all factors predictive of death. or creatinine > 1. Cohen JR: Factors that affect the survival rate of patients with ruptured abdominal aortic aneurysms. Because the patient in this scenario has determined that she would not want heroic measures taken. Futile care can be considered care that either will not succeed in achieving the specific objective being treated or will not succeed in improving the patient’s situation. Wittink M. Am Fam Physician 72:1263-1268. a history of loss of consciousness. A living will is recognized in all states to be legally binding. Cardiopulmonary resuscitation.American College of Surgeons SESAP 13 Critique for Category 17 Item 9 Patients with ruptured abdominal aortic aneurysm have mortality rates as high as 80%. Philadelphia. 2004. Doukas DJ: Respecting end-of-life treatment preferences.Carson RA: Ethics in surgery. Saunders. it determines nothing about what that person will decide. the most appropriate and only legal action is to honor her living will and forego operation. REFERENCES 1. an advance directive can consist of a verbal agreement between the patient and his family that can help the family understand how the patient feels about similar issues. 1997 page 1332 . hemoglobin < 10 mg/dL. ed 17. Although this designates a decision-maker. J Vasc Surg 26:939-948. blood pressure < 90 mm Hg. Advance directives are a set of techniques for patients to communicate their wishes regarding medical care. which is a set of written instructions given by a patient regarding how he or she would like to be cared for. and her clinical situation is futile. Kline RG. An ethics committee consultation is not needed and performing a slow code in the operating room is inappropriate. 2005 3. in Townsend CM Jr (ed): Sabiston Textbook of Surgery: The Biological Basis of Modern Surgical Practice. D'Angelo AJ. At the very least. An advance directive can consist of a living will. particularly at the end of life or in the event that the patient cannot make or understand decisions. pp 21-25 2.Halpern VJ. and oliguric. The parents and child are Jehovah’s Witnesses and refuse all blood products. and hematocrit is 18%. the patient becomes hypotensive. tachycardic.American College of Surgeons SESAP 13 Category 17 Item 10 A 12-year-old child undergoes spinal fixation for severe scoliosis. Postoperatively. The next step should be to (A) withhold all blood products (B) transfuse packed red blood cells (C) administer erythropoietin (D) transfuse human hemoglobin substitute (PolyHeme) (E) obtain court decision for blood transfusion page 1333 . Courts throughout the Western world recognize parental rights.Hickey KS.American College of Surgeons SESAP 13 Critique for Category 17 Item 10 Jehovah’s Witnesses fundamentally refuse blood products. and parents do not have an absolute right to refuse medical treatment for their children based on their religious beliefs. the power to give or withhold consent to medical treatment on their behalf traditionally lies with those who have parental responsibility.Prince v Massachusetts 321 US 158 (1944) 4. But it does not follow that they are free.Carson JL. explaining that the right to practice religion freely does not include liberty to expose the child to ill health or death. 2005 page 1334 . and faithbased healing. Used frequently when parental religious beliefs preclude specific treatments. the law. even when such a refusal may result in death.Woolley S: Children of Jehovah’s Witnesses and adolescent Jehovah’s Witnesses: what are their rights? Arch Dis Child 90:715-719. Three main points have been emphasized in all prior such cases: the child’s interests and those of the state outweigh parental rights to refuse medical treatment. Gould SA: Mortality and morbidity in patients with very low postoperative Hb levels who decline blood transfusion. REFERENCES 1. but these rights are not absolute. Noveck H. This principle applies whether or not the child is in imminent danger. Theor Med Bioeth 25:265-276. parental consent is legally necessary to perform any medical procedure on a child. Erythropoietin will not acutely treat the anemia. PolyHeme is not an approved treatment. Prince v Massachusetts set out the reigning legal principle:“Parents may be free to become martyrs themselves. 2002 2. parental rights do not give parents life and death authority over their children. The courts are therefore asked to exercise their power under the doctrine of parens patriae. Except in an emergency. the court can justify compulsory medical treatment based on the avoidance of physical harm. which allows state interference to protect a child’s welfare. Lyckholm L: Child welfare versus parental autonomy: medical ethics. to make martyrs of their children…” The Supreme Court was clear in upholding the decision in Prince. and well-being. in identical circumstances. When parental refusal is based on religious beliefs. When the postoperative hemoglobin falls to the range of 5 to 6 g/dL. Parental rights to raise children are qualified by a duty to ensure their health. safety. This child’s hematocrit of 18% is a critical anemia and can be associated with increased mortality when further bleeding is likely. 2004 3. Parents cannot make decisions that may permanently harm or otherwise impair the child’s healthy development. Transfusion 42:812-818. None of the other transfusion options are available without legal consent. Berlin JA. For young children. morbidity and mortality increase sharply in Jehovah’s Witnesses who undergo elective surgery. as parents are always required to make decisions in the child’s best interests. American College of Surgeons SESAP 13 Category 17 Item 11 All of the following are recommended qualifications for physicians who act as expert witnesses EXCEPT (A) a current. unrestricted license to practice medicine (B) diplomate of or have status with a specialty board recognized by the American Board of Medical Specialties (C) evidence of continuing medical education relevant to the specialty or the subject matter of the case (D) application of current standards of care to the case in question (E) willingness to disclose the fees or compensation received page 1335 . The American College of Surgeons: Statement on the physician acting as an expert witness. Bull Am Coll Surg 89:22-23. and should be a diplomate of or have status with a specialty board recognized by the American Board of Medical Specialties. The physician expert witness should be able to demonstrate evidence of continuing medical education relevant to the specialty or the subject matter of the case. Available at: www. and these standards should be used to distinguish between actual negligence (ie. a qualified medical expert is needed to establish that the defendant physician has or has not deviated from established standards of medical care that resulted in injury or death to the plaintiff. physicians acting as expert witnesses should have comparable education. The physician expert witness who provides testimony for a plaintiff or a defendant in a case involving a specific surgical procedure (or procedures) should hold current privileges to perform those same procedures in a hospital that is accredited by the Joint Commission on Accreditation of Healthcare Organizations (JCAHO) or the American Osteopathic Association (AOA). substandard medical care that results in harm) and unfortunate medical outcome (ie. In response to the need to define the qualifications and behavior of the expert witness. complications resulting from factors beyond medical negligence). the American Medical Association. The specialty of the physician expert witness should be appropriate to the subject matter in the case. and unrestricted license to practice medicine in the state in which he or she practices. Physician expert witnesses should review the accepted practices prevailing at the time nd place of the alleged occurrence.American College of Surgeons SESAP 13 Critique for Category 17 Item 11 The physician who acts as an expert witness is a central figure in malpractice litigation. the Council of Medical Specialty Societies. Finally. In these cases.994 Expert witness testimony. In general. and other medical groups have developed recommended guidelines. the physician expert witness should be prepared to document the percentage of time that is involved in serving as an expert witness. valid. The physician expert witness should be familiar with the standard of care provided at the time of the alleged occurrence and should be actively involved in the clinical practice of the specialty or the subject matter of the case during the time the testimony or opinion is provided. training.The American Medical Association. as well as be qualified by experience or demonstrated competence in the subject of the case. Current practice standards may not be applicable to the case in question. and occupational experience in the defendant’s field. REFERENCES 1. 2004 2. and should be willing to disclose the amount of fees or compensation obtained for such activities and the total number of times he or she has testified for the plaintiff or defendant.org/apps page 1336 . H-265. Guidelines issued by the American College of Surgeons state that the physician expert witness must have a current.amaassn. American College of Surgeons SESAP 13 Category 17 Item 12 Which of the following statements about elder protection is TRUE? (A) The US has well-developed policies to protect elders (B) The federal government is charged with the identification and management of cases of elder abuse (C) Lawsuits against nursing homes are most commonly related to fractures and pressure ulcers (D) Physicians are the primary reporters of elder abuse (E) The Centers for Medicare and Medicaid Services (CMS) is directly responsible for ensuring safety standards in nursing homes page 1337 . Although a large portion of Medicare and Medicaid dollars is used each year to cover nursing home care and services for the elderly and disabled. and home health aides. Information about Laws Related to Elder Abuse. but understudied and underreported.cfm 4. the statutes vary widely in terms of the definition of abuse. whether reporting is mandatory or voluntary. neglect.Department of Health and Human Services: Nursing Homes: About Nursing Home Inspections. 2006. National Center on Elder Abuse Website. APS laws relate only to persons who reside in the community and separate laws address incidents of elder abuse occurring in long-term care facilities.elderabusecenter. The Centers for Medicare and Medicaid Services (CMS) contracts with each state to conduct on-site inspections that determine whether its nursing homes meet the minimum Medicare and Medicaid quality.American College of Surgeons SESAP 13 Critique for Category 17 Item 12 Elder abuse is common. Health Aff 22:219-229. Medicare Website.gov/Nursing/AboutInspections.Stevenson DG. Available at: http://www. Most cases are reported by family members. and exploitation that are covered. 2002 2.National Center on Elder Abuse: Elder abuse law background information.Ahmad M. 2003 page 1338 . The federal Older Americans Act provides definitions of elder abuse and authorizes the use of federal funds for the National Center on Elder Abuse. Falls and pressure ulcers are the most common factors leading to lawsuits. and safety standards. and procedures and remedies for abuse. In some states. investigation responsibility.org/default. Lachs MS: Elder abuse and neglect: what physicians can and should do. 2007. but the individual states and the District of Columbia are responsible for adult protective services (APS) laws. Texas and Florida have the highest rates of litigation against nursing homes. hospital staff.medicare.cfm?p=backgrounder. Available at: http://www. the state governments oversee the licensing of nursing homes. despite mandatory reporting laws in most states. types of abuse. Studdert DM: The rise of nursing home litigation: findings from a national survey of attorneys. Cleve Clin J Med 69:801-808.asp 3. Physicians report only 2% of all reported cases of elder abuse. Although APS laws establish a system for the reporting and investigation of elder abuse. REFERENCES 1. performance. American College of Surgeons SESAP 13 Category 17 Item 13 A 78-year-old patient in the ICU is DNR/DNI by his own living will. His adult son and daughter believe that their father would not want the endotracheal tube replaced if he were not confused. PCO2 is 65 torr. WBC count is 14. and 4 hours later his arterial blood gases are slightly worse. and he should not be intubated (B) The patient is confused. Two days after extubation.000/mm3 . Which of the following statements is the most ethically sound? (A) The patient’s original living will should be followed. Face mask ventilation (BiPAP) produces minimal initial improvement.30. which was confirmed on hospital admission. and his closest living relatives are automatically granted medical guardianship (C) The patient’s confusion is not sufficient to preclude his own decision-making and an endotracheal tube should be inserted (D) If the patient’s children stand to benefit from his death. Endotracheal intubation is deemed necessary to prevent death from acute pneumonia. pH is 7. He develops a cough productive of copious yellowbrown sputum. The patient is mildly confused (date and time) but able to communicate and is otherwise oriented. A chest xray shows a lobar infiltrate in the right lower lobe. He had a colon resection for diverticulitis. his breathing becomes labored with a respiratory rate of 35. they should not be allowed to take part in the decision (E) Psychiatric consultation should be obtained to determine the patient’s ability to make his own decisions page 1339 . The DNI order was suspended for the perioperative period to allow for intubation during surgery and subsequent recovery. PO2 is 53 torr. and ask that he not be re-intubated. and HCO3 is 26. and consents to replacement of the endotracheal tube. As defined by the English philosopher WD Ross. The system was designed to be a simple. the contributions of four prima facie principles– autonomy. a previously written living will should not preclude him the autonomy to change his mind. and living wills may not provide situationally specific guidance to patient circumstances. The wishes of the relatives in such a situation could be considered moot. if a patient is considered competent enough to make his own decision. as their decision-making should only be in proxy for a patient who is no longer able to make his own decisions. and respect for morally acceptable laws (legal justice). and whether or not his mild confusion disallows him changing his mind. Prominent national cases (Terry Schiavo. Justice has three main categories in medical ethics: fair distribution of scarce resources (distributive justice). and offers a common. when they are in conflict with each other good decisions must provide a net benefit to the patient. to provide the maximum benefit for the patient while causing no harm. Even if the patient’s children are biased because of financial concerns. this construct acknowledges the important context of the patient’s legal rights and obligations within a society. Certainly. a prima facie principle is binding unless it conflicts with another moral principle. Karen Ann Quinlan) demonstrate that the legal issues of such cases are far from providing firm guidance – if they even should. beneficence. eg. Although it does seem to stray philosophically from the four principles as being culturally neutral. REFERENCES 1. as well as the right to change one’s mind. the lynchpin would appear to be the patient’s autonomy. It may be simpler to consider justice a societal construct – the patient’s rights and obligations within a society. While these two principles are complimentary. nonmaleficence. Autonomy means respect for the patient’s right to self-determination. basic moral analytical framework and language. accessible. BMJ 309:184-188. respect for people's rights (rights-based justice). and the law are currently a matter of debate and no small amount of emotional contention. and culturally neutral approach to solving ethical dilemmas. They certainly could be challenged on the basis of justice if there was a suspicion that they were not acting in the patient’s best interests (nonmaleficence). and does not constrain the system from being applied in the society of which the patient is a part.American College of Surgeons SESAP 13 Critique for Category 17 Item 13 End-of-life decisions are often difficult. medical professionals.Gillon R: Medical ethics: four principles plus attention to scope. justice can actually encompass a seemingly heterogeneous array of concerns. and justice – are considered and their relative contribution bound by the subjective scope of each. In this case. 2004 page 1340 . In this system. Beneficence and nonmaleficence involve a re-stating of the Hippocratic oath. and does not appear to inhibit the patient’s ability to make a decision regarding a potentially life-saving treatment. beneficence greater than harm. In medical ethics discussions. Regarding specific guiding principles. The relative roles of families. a reasonably popular and well-regarded approach is the four principles + scope system pioneered in the US by Beauchamp and Childress. Basic examples include respecting a patient’s right to make up his own mind about his or her health care. In the clinical scenario here. but may not be feasible in a case where urgent action is required. they are not precluded from participating in medical decision-making for an incompetent patient. the confusion is mild. Psychiatric and ethics consultations are valuable adjuncts to difficult decisions in patient care where the patient’s competency may be in doubt. J Med Ethics 29:275-280. 2003 page 1341 .Macklin R: Applying the four principles.American College of Surgeons SESAP 13 2. American College of Surgeons SESAP 13 Category 17 Item 14 Which of the following philosophical factors has predominantly contributed to the challenge of routine circumcision by medical ethicists? (A) Personal autonomy (B) Future humiliation (C) Lack of analgesia during the procedure (D) Disfigurement (E) Non-physician circumcision page 1342 . Task Force on Circumcision. Because devices. some believe it would be a desirable option for the whole world. this undermines the personal autonomy of the child. Plastibell device. age-appropriate adolescents and men could choose for themselves whether to have the procedure performed. risk of AIDS and penile carcinoma). Although the risk of complications is very low in newborn circumcision (< 1%).to 10-year-old boys may be held down on the kitchen table. there is some evidence that newborn circumcision decreases the subsequent risk of urinary tract infection from 7:1000 to < 2:1000. technically no complication should be tolerated. this age might be ideal. This is physically cruel. 2004 4. The risk of complications after circumcision is low.2%). J Med Ethics 30:238-240. and undoubtably leaves major psychological scars. because the foreskin becomes too thick to adequately render the foreskin ischemic when the ligature is tied. potentially dangerous. 2004 3. Circumcision in some parts of the world would come close to the legal definition of assault in this country. while a family member cuts off the foreskin. The Plastibell device is not as easy to use in late childhood or adolescence. A primary ethical issue is that routine circumcision in babies lacks an absolute medical indication. male circumcision has become an issue for medical-ethical debate.Hutson JM: Circumcision: a surgeon’s perspective. Five. who may later regret that this procedure was done.Mussell R: The development of professional guidelines on the law and ethics of male circumcision. but can include accidental amputation of the end of the penis. Benatar D: Between prophylaxis and child abuse: the ethics of neonatal male circumcision. the numbers are small.American College of Surgeons SESAP 13 Critique for Category 17 Item 14 Nontherapeutic male circumcision has been practiced worldwide for centuries and is deeply embedded in the family life of some populations. and the cost is low. excessive removal of penile shaft skin leading to secondary deformity. eg. Despite this. AM J Bioeth 3:35-48. Because most of the medical risk factors that have been associated with lack of circumcision occur later in life (transmission of ulcerative sexually transmitted diseases. rather than being preempted in their decisions. the Gomco clamp. 1999 page 1343 . As the medical evidence has mounted in favor of male circumcision for the prevention of widespread public health problems such as the AIDS pandemic and cervical carcinoma. because the risks of the procedure are not being balanced against the significant risk of a disease. J Med Ethics 30:254-258. or diathermy burns leading to isolated or complete necrosis. However. American Academy of Pediatrics. The principle of personal autonomy has predominantly contributed to the challenge of routine circumcision by medical ethicists. Although the more severe complications are individual catastrophes. Although parents often wish their boys to be circumcised. and lays emphasis on individuals choosing options for themselves wherever possible. used in newborns have low complication rates (on the order of 0. Some would argue that neonatal circumcision done without anesthesia is not far removed from this practice. 2003 2.[No Authors Listed]: Circumcision policy statement. Pediatrics 103:686-693.Benatar M. REFERENCES 1. four quadrants are packed. and the new staff completes the operation. The spleen is at first sutured. Abdominal closure is difficult because of bowel edema. but bleeding continues during the rest of the exploration and the spleen is removed. The operation occurs during shift change. The LEAST important risk factor for a retained sponge or instrument in this patient is (A) obesity (B) emergency operation (C) change in operative plan (D) new operative nursing team (E) towel used to help closure page 1344 . Sponge and instrument counts are correct.American College of Surgeons SESAP 13 Category 17 Item 15 A man who weighs 300 lb has a gunshot wound to the abdomen. At exploration. A towel is used to protect the intestines during fascial suturing. Legislation does not require that sponges be counted. Contemp Surg 61:398-400. Sponges are particularly likely to be left behind in obese patients. Studdert DM. but did not retain individual significance. N Engl J Med 348:229—235. Asian J Surg 28:109-115.Camazine B: The persistent problem of the retained foreign body. Multivariate analysis has confirmed three variables that pose a significantly higher risk of a retained object: emergency procedure. REFERENCES 1. AORN guidelines mandate that sponges be counted on permanent relief of either the scrub nurse or the circulator. in cases where a sponge is left. and body mass index of the patient. Failure to perform a count of sponges and instruments had a strong association with emergency operation. emergency operations. the counts were most often correct. 2003 page 1345 .Gawande AA. and are most commonly surgical sponges. Brennan TA. Under the doctrine of res ipsa loquitur . Yaghan RJ: Retained surgical sponges (gossypiboma). Gharaibeh KA . et al: Risk factors for retained instruments and sponges after surgery. 2005 2. 2005 3. and is often assumed by juries in resulting malpractice cases. Nonetheless. only that they not be negligently left in patients. Orav EJ.Bani-Hani KE . the presence of a retained foreign body implies a breach of the standard of care. unplanned change in procedure.American College of Surgeons SESAP 13 Critique for Category 17 Item 15 Retained foreign bodies after operation are associated with up to 50% morbidity and 10% mortality. and after an unexpected change in operative procedure. who has been relying on the patient’s pension. The medical team believes that dialysis would be futile. his renal function has deteriorated due to sepsis. His advanced directive in the medical record that states that he does not desire extraordinary measures to prolong life. has his durable power of attorney and wants dialysis to be initiated. However. his wife. and renal dialysis will be required within the next few days. What is the next step in management? (A) Acknowledge the wife’s power of attorney and follow her wishes (B) Discontinue the patient’s sedation to determine his wishes (C) Consult the hospital attorney for support in refusing to initiate dialysis (D) Honor his wishes and provide comfort care only (E) Review the patient’s advance directive with the wife to clarify his wishes page 1346 . a 68-year-old man is sedated and continues to receive inotropic and mechanical ventilatory support.American College of Surgeons SESAP 13 Category 17 Item 16 Four weeks after coronary artery bypass complicated by a sternal dehiscence and mediastinitis. In the last week. Haheim LL. a sufficiently clear mental status is unlikely to be achievable with simple sedation reversal and will increase this patient’s suffering.Angelos P: Ethical guidelines in surgical patient care. Krouse RS. Pawlik T. if a negotiated treatment plan cannot be achieved and the POA is perceived to be conflicted. Angelos P: Ethical considerations and barriers to research in surgical palliative care. 2005 4. REFERENCES 1. J Am Coll Surg 188:55-58. Hinshaw DB.Hofmann B. However. Br J Surg 92:802-809. et al: Clinical palliative care for surgeons: part 1. The wife. 2004 6. J Am Coll Surg 198:477-491. Hinshaw DB. 2003 3. et al: Clinical palliative care for surgeons: part 2. J Am Coll Surg 196:621-651.Hinshaw DB. legal counsel may be necessary to resolve the conflict. The identification of a legal surrogate with power of attorney (POA) for health issues offers an additional. 2003 5. Consultation with legal counsel at this stage may fuel animosity in a situation that requires tact and compassion. An advanced directive outlining any limits to aggressive interventions assists healthcare professionals when a patient can no longer actively participate in decisions.Tonelli MR: Waking the dying: must we always attempt to involve critically ill patients in end-of-life decisions? Chest 127:637-642. Purcell GP. et al: When do we stop. 2005 page 1347 . Soreide JA: Ethics of palliative surgery in patients with cancer. review of the patient’s advanced directive coupled with a clear discussion about the patient’s prognosis is an appropriate first step to reconciling differences. 2004 7. Easson AM. Purcell GP. 1999 2. Mosenthal AC. who is his legal guardian. Although legally she has the authority to continue care. and how do we do it? Medical futility and withdrawal of care. These personal and legal directives may occasionally be in conflict. Given the scenario of sepsis and renal failure. Civetta JM. J Am Coll Surg 198:303-319. has a potential conflict of interest: honoring her husband’s wishes versus loss of her financial support. Johnson DL. as in this example. J Am Coll Surg 196:469-474. clear line of authority for such decisions.Lee KF. Clarification without confrontation is often sufficient to negotiate a treatment plan that meets the needs of the patient as well as his spouse.American College of Surgeons SESAP 13 Critique for Category 17 Item 16 The ethical management of the critically ill centers on acknowledgement of an individual patient’s desires for his/her health care.Lee KF. A POA’s responsibility is to make decisions in line with the patient’s best interests and as outlined in any advanced directive.Krouse RS. and lacks decision-making capacity. He is dependent in all activities of daily living. but relieves symptoms (D) does not prolong life. is mostly nonverbal.American College of Surgeons SESAP 13 Category 17 Item 17 An 89-year-old man has been in a nursing home for 2 years with a diagnosis of dementia. and does not relieve symptoms (E) reduces the incidence of aspiration pneumonia page 1348 . the surgeon is best guided by recent evidence that tube feeding (A) prolongs life but does not relieve symptoms (B) prolongs life and relieves symptoms (C) does not prolong life. In considering the placement of a gastrostomy tube. He was recently hospitalized for pneumonia believed to be secondary to aspiration. These findings persist after adjustments for age. Nonrandomized retrospective observations of nursing home residents have found no survival advantage with tube feeding.American College of Surgeons SESAP 13 Critique for Category 17 Item 17 Although the use of feeding tubes in patients with dementia was previously believed to prolong life. and no data show that it can reduce the risk from regurgitated gastric contents. a review of the existing literature reveals that it has in fact been remarkably difficult to demonstrate any difference in longevity between patients treated with or without feeding tubes. presence of swallowing disorder. N Engl J Med 342:206-210. 1999 2. Travis K: Tube feeding in patients with advanced dementia: a review of the evidence. and cognitive status. 2000 page 1349 . Tube feeding has not been found to be associated with healing of pre-existing pressure sores. functional status.Gillick MR: Rethinking the role of tube feeding in patients with advanced dementia. Tube feeding cannot be expected to prevent aspiration of oral secretions. resuscitation wishes.Finucane TE. the use of tube feeding in nursing home residents has been reported to significantly increase 1-year mortality and is associated with potential morbidity. or improvement in quality of life. Further. REFERENCES 1. decubitus ulcer. improved functional status. JAMA 282:1365-1370. history of pulmonary aspiration or stroke. Christmas C. Which pair of drug classes should be used? (A) Opioid and benzodiazepine (B) Antihistamine and benzodiazepine (C) Opioid and antihistamine (D) Antimuscarinic and antihistamine (E) Antimuscarinic and H2 blocker page 1350 .American College of Surgeons SESAP 13 Category 17 Item 18 A family has asked to have mechanical ventilatory support removed from a patient who is receiving comfort care only. reduction in oxygen consumption. data from these centers suggest that the clinical side effects of opioids in this setting are relatively small and that death has not been hastened by drug administration. Smedira NG. JAMA 267:949-953. and improved cardiovascular function. 2002 2. reduction in anxiety associated with dyspnea. Higgins JP. which cross the blood brain barrier and can cause a number of CNS side effects REFERENCES 1. Meta-analysis of existing data has demonstrated a statistically significant positive effect of opioids on the sensation of breathlessness. reduction in sensitivity to hypercapnia. Opioids in this setting are typically combined with sedative drugs such as benzodiazepines.Wilson WC. The other options listed are suboptimal in comparison. McDowell JA. et al: A systemic review of the use of opioids in the management of dyspnoea.Jennings AL. especially those including antimuscarinics. given either orally or parenterally. 1992 page 1351 . are the drugs of choice for treatment of dyspnea in patients with advanced disease. Gibbs JS. Thorax 57:939-944.American College of Surgeons SESAP 13 Critique for Category 17 Item 18 Opioids. Fink C. Davies AN. Further. Studies from large trauma centers reveal that at least 75% of patients for whom life support was withdrawn or withheld have received this combination of drugs with good provider satisfaction. et al: Ordering and administration of sedatives and analgesics during the withholding and withdrawal of life support from critically ill patients. The possible mechanisms of action of opioids in this setting include reduction in the central perception of dyspnea (similar to the reduction in the central perception of pain). American College of Surgeons SESAP 13 Category 17 Item 19 Duty to intervene on the patient’s behalf to increase comfort. and well-being (A) Nonmaleficence (B) Autonomy (C) Euthanasia (D) Beneficence (E) Justice page 1352 . health. Beneficence is the duty to directly intervene. REFERENCES 1. Because this is often not possible in absolute terms. p 588 page 1353 . and wishing to take action to help the other. Clin J Pain 18:S99-S107. nonmaleficence. to increase the patient’s comfort. ie. where possible. distribution according to merit. It requires that a patient is truly free to choose a treatment. Autonomy refers to the independence of the patient to make her or his own decisions. 1994. ed 30. beneficence.American College of Surgeons SESAP 13 Critique for Category 17 Item 19 Principle-based medical ethics involves four key tenets that are the bases for evaluation of the appropriateness of clinical actions: autonomy. Beneficence is closely related to compassion. Euthanasia means deliberately ending the life of a person who has an incurable or painful disease. The tenet of justice suggests that treatments should be made available in equitable fashion to all patients who need them. and has the mental capacity to evaluate options and choose. distribution according to most urgent need. and distribution according to the effort the patient will apply to the treatment/healing process. Nonmaleficence is the obligation to avoid deliberate and unintentional harm to a patient through carelessness or abandonment. principles to govern distribution of limited treatment resources have been developed. Examples of such guidelines for just distribution include care delivery in equal share to persons in need. and current and future well-being. primum no nocere.Cohen MJ.Dorland NW (ed): Dorlands Illustrated Medical Dictionary. Philadelphia. Margolis CG: Ethical perspectives: opioid treatment of chronic pain in the context of addiction. has the information required to make a choice. Herron PD. 2002 2. caring about another. and justice. Jasser S. or do no harm. health. WB Saunders Company. American College of Surgeons SESAP 13 Category 17 Item 20 Obligation to avoid deliberate and unintentional harm through carelessness or abandonment (A) Nonmaleficence (B) Autonomy (C) Euthanasia (D) Beneficence (E) Justice page 1354 . Cohen MJ. distribution according to most urgent need. where possible. Jasser S. p 588 page 1355 . and wishing to take action to help the other. Nonmaleficence is the obligation to avoid deliberate and unintentional harm to a patient through carelessness or abandonment. caring about another. has the information required to make a choice. Philadelphia.American College of Surgeons SESAP 13 Critique for Category 17 Item 20 Principle-based medical ethics involves four key tenets that are the bases for evaluation of the appropriateness of clinical actions: autonomy. Beneficence is the duty to directly intervene. beneficence. nonmaleficence. primum no nocere. It requires that a patient is truly free to choose a treatment. REFERENCES 1. 2002 2.Dorland NW (ed): Dorlands Illustrated Medical Dictionary. Clin J Pain 18:S99-S107. ie. Autonomy refers to the independence of the patient to make her or his own decisions. to increase the patient’s comfort. 1994. Because this is often not possible in absolute terms. Herron PD. health. Margolis CG: Ethical perspectives: opioid treatment of chronic pain in the context of addiction. The tenet of justice suggests that treatments should be made available in equitable fashion to all patients who need them. distribution according to merit. and has the mental capacity to evaluate options and choose. Examples of such guidelines for just distribution include care delivery in equal share to persons in need. WB Saunders Company. principles to govern distribution of limited treatment resources have been developed. Beneficence is closely related to compassion. and distribution according to the effort the patient will apply to the treatment/healing process. ed 30. Euthanasia means deliberately ending the life of a person who has an incurable or painful disease. and justice. and current and future well-being. or do no harm. American College of Surgeons SESAP 13 Category 17 Item 21 Equitable availability of treatments to those who may benefit (A) Nonmaleficence (B) Autonomy (C) Euthanasia (D) Beneficence (E) Justice page 1356 . to increase the patient’s comfort. Beneficence is closely related to compassion. Examples of such guidelines for just distribution include care delivery in equal share to persons in need. has the information required to make a choice. Herron PD. and justice.American College of Surgeons SESAP 13 Critique for Category 17 Item 21 Principle-based medical ethics involves four key tenets that are the bases for evaluation of the appropriateness of clinical actions: autonomy. beneficence. and has the mental capacity to evaluate options and choose. Beneficence is the duty to directly intervene. and wishing to take action to help the other. where possible. Jasser S. 1994. ed 30. nonmaleficence. WB Saunders Company. and distribution according to the effort the patient will apply to the treatment/healing process. health.Dorland NW (ed): Dorlands Illustrated Medical Dictionary. 2002 2. Philadelphia. ie. principles to govern distribution of limited treatment resources have been developed. The tenet of justice suggests that treatments should be made available in equitable fashion to all patients who need them. p 588 page 1357 . distribution according to most urgent need. Clin J Pain 18:S99-S107. primum no nocere. and current and future well-being. Euthanasia means deliberately ending the life of a person who has an incurable or painful disease. distribution according to merit. Autonomy refers to the independence of the patient to make her or his own decisions.Cohen MJ. Nonmaleficence is the obligation to avoid deliberate and unintentional harm to a patient through carelessness or abandonment. Margolis CG: Ethical perspectives: opioid treatment of chronic pain in the context of addiction. It requires that a patient is truly free to choose a treatment. Because this is often not possible in absolute terms. caring about another. or do no harm. REFERENCES 1. able to make decisions unaided by others. free. and self-directing (A) Nonmaleficence (B) Autonomy (C) Euthanasia (D) Beneficence (E) Justice page 1358 . the quality or state of being independent.American College of Surgeons SESAP 13 Category 17 Item 22 Right of self-determination in medical decision-making. distribution according to merit. Clin J Pain 18:S99-S107. Nonmaleficence is the obligation to avoid deliberate and unintentional harm to a patient through carelessness or abandonment.Cohen MJ. and distribution according to the effort the patient will apply to the treatment/healing process. where possible. has the information required to make a choice. WB Saunders Company. nonmaleficence. Because this is often not possible in absolute terms. Beneficence is the duty to directly intervene. principles to govern distribution of limited treatment resources have been developed. caring about another. REFERENCES 1. and justice. Euthanasia means deliberately ending the life of a person who has an incurable or painful disease. Autonomy refers to the independence of the patient to make her or his own decisions. distribution according to most urgent need. 1994. Herron PD. It requires that a patient is truly free to choose a treatment. to increase the patient’s comfort. and current and future well-being. health. Margolis CG: Ethical perspectives: opioid treatment of chronic pain in the context of addiction. Examples of such guidelines for just distribution include care delivery in equal share to persons in need. or do no harm. primum no nocere. Beneficence is closely related to compassion.American College of Surgeons SESAP 13 Critique for Category 17 Item 22 Principle-based medical ethics involves four key tenets that are the bases for evaluation of the appropriateness of clinical actions: autonomy. and wishing to take action to help the other.Dorland NW (ed): Dorlands Illustrated Medical Dictionary. Philadelphia. and has the mental capacity to evaluate options and choose. The tenet of justice suggests that treatments should be made available in equitable fashion to all patients who need them. 2002 2. Jasser S. p 588 page 1359 . ed 30. beneficence. ie. American College of Surgeons SESAP 13 Category 17 Item 23 One-way analysis of variance (A) Two independent samples of continuous data (B) Two independent samples of ordinal data (C) Two groups of nominal data (D) Multiple groups of continuous data (E) Multiple groups of noncontinuous data page 1360 . Samples are independent when they have no connection with each other. And finally. 42% of patients were male.Velanovich V: A review of data analysis of surgical research.Riffenburgh RH (ed): Statistics in Medicine. Matching the appropriate mode of analysis with the data set is necessary to develop a useful conclusion. ed 2. multiple groups of ordinal data should be analyzed with Friedman’s ANOVA. eg. Identifying the type of data is critical to choosing the appropriate mode of analysis. eg. the authors must use statistical analysis to convince us that the differences between data groups are real. Two groups of nominal data can be analyzed with Fisher’s exact test or a chi score test. but the space between is discontinuous.American College of Surgeons SESAP 13 Critique for Category 17 Item 23 Every article published in the surgical literature attempts to convince us that its conclusions are true. Continuous data can be arranged on a continuum. MA. Multiple groups of continuous data should be analyzed with the one-way analysis of variance (ANOVA). This is usually done by constructing an argument using data generated in the study as premises leading to conclusions. As most data are comparative. hemoglobin level. Burlington. eg. blood levels in the same patient before and after an intervention. Ordinal data are data that can be arranged in a hierarchy. Nominal data is a determination of the frequency of subjects in a certain group. cancer stage. Two independent samples of ordinal data should be analyzed with the Mann-Whitney U-test. The authors try to demonstrate the “truth” of their premises by convincing us of the truth of their data. 2006 2. eg. Two independent samples of continuous data should be analyzed with Students’ t-test. Academic Press. 1990 page 1361 . REFERENCES 1. Curr Surg 47:410-418. and are paired when they do. American College of Surgeons SESAP 13 Category 17 Item 24 Fisher’s exact test (A) Two independent samples of continuous data (B) Two independent samples of ordinal data (C) Two groups of nominal data (D) Multiple groups of continuous data (E) Multiple groups of noncontinuous data page 1362 . eg. And finally. eg. REFERENCES 1. Continuous data can be arranged on a continuum. MA.Riffenburgh RH (ed): Statistics in Medicine. ed 2. Multiple groups of continuous data should be analyzed with the one-way analysis of variance (ANOVA). Two groups of nominal data can be analyzed with Fisher’s exact test or a chi score test. Samples are independent when they have no connection with each other. Nominal data is a determination of the frequency of subjects in a certain group. Two independent samples of continuous data should be analyzed with Students’ t-test.Velanovich V: A review of data analysis of surgical research. the authors must use statistical analysis to convince us that the differences between data groups are real. eg. hemoglobin level. As most data are comparative.American College of Surgeons SESAP 13 Critique for Category 17 Item 24 Every article published in the surgical literature attempts to convince us that its conclusions are true. 2006 2. blood levels in the same patient before and after an intervention. Burlington. multiple groups of ordinal data should be analyzed with Friedman’s ANOVA. Two independent samples of ordinal data should be analyzed with the Mann-Whitney U-test. eg. Academic Press. 1990 page 1363 . 42% of patients were male. This is usually done by constructing an argument using data generated in the study as premises leading to conclusions. Identifying the type of data is critical to choosing the appropriate mode of analysis. Ordinal data are data that can be arranged in a hierarchy. Matching the appropriate mode of analysis with the data set is necessary to develop a useful conclusion. but the space between is discontinuous. cancer stage. The authors try to demonstrate the “truth” of their premises by convincing us of the truth of their data. Curr Surg 47:410-418. and are paired when they do. American College of Surgeons SESAP 13 Category 17 Item 25 Mann-Whitney U-test (A) Two independent samples of continuous data (B) Two independent samples of ordinal data (C) Two groups of nominal data (D) Multiple groups of continuous data (E) Multiple groups of noncontinuous data page 1364 . American College of Surgeons SESAP 13 Critique for Category 17 Item 25 Every article published in the surgical literature attempts to convince us that its conclusions are true. As most data are comparative. eg. Two independent samples of continuous data should be analyzed with Students’ t-test. and are paired when they do. eg. And finally. 42% of patients were male. eg. 2006 2. Academic Press. ed 2. Multiple groups of continuous data should be analyzed with the one-way analysis of variance (ANOVA).Velanovich V: A review of data analysis of surgical research. Burlington. hemoglobin level. Two groups of nominal data can be analyzed with Fisher’s exact test or a chi score test. Matching the appropriate mode of analysis with the data set is necessary to develop a useful conclusion. REFERENCES 1. eg. This is usually done by constructing an argument using data generated in the study as premises leading to conclusions. Samples are independent when they have no connection with each other. Two independent samples of ordinal data should be analyzed with the Mann-Whitney U-test.Riffenburgh RH (ed): Statistics in Medicine. the authors must use statistical analysis to convince us that the differences between data groups are real. Nominal data is a determination of the frequency of subjects in a certain group. cancer stage. Ordinal data are data that can be arranged in a hierarchy. multiple groups of ordinal data should be analyzed with Friedman’s ANOVA. Curr Surg 47:410-418. Identifying the type of data is critical to choosing the appropriate mode of analysis. 1990 page 1365 . Continuous data can be arranged on a continuum. MA. but the space between is discontinuous. blood levels in the same patient before and after an intervention. The authors try to demonstrate the “truth” of their premises by convincing us of the truth of their data. American College of Surgeons SESAP 13 Category 17 Item 26 Used to measure quality of life (A) Time trade-off technique (B) SF-36 Health Status Survey (C) Both (D) Neither page 1366 . 1999 page 1367 . They are complex to administer and to interpret. It is a functional outcome measure. Measuring quality of life can be challenging. There are multiple instruments available based on various models. Church JM. and length of stay. Adv Surg 33:293309. These instruments allow patients to choose between two options for health care. ed 2. Outcomes of traditional interest to surgeons include postoperative complication rates. Alternatively. requiring significant time and effort from patients and researchers. survival rates (overall and disease-specific). The SF-36 Health Status Survey is one of the many validated instruments used to measure quality of life. In the utilitarian measurement model. pp 567-570 2. For example. diseases. the psychometric approach to measuring quality of life uses questionnaires to measure patients’attitudes. Delaney CP (eds): Current Therapy in Colon and Rectal Surgery.American College of Surgeons SESAP 13 Critique for Category 17 Item 26 Surgical outcome measurements allow for comparison of surgeons. Both the SF-36 instrument and the time trade-off technique have proved to be valid and reliable in measuring different aspects of quality of life. patients. Philadelphia. Outcomes of interest to patients more frequently include postoperative quality of life measurements. 2005.Church JM: Outcomes analysis and measurement of quality of life. Various instruments exist to measure outcomes in surgery. REFERENCES 1. and therapies. or perceptions about various aspects of their life. in a time-tradeoff technique the patient is asked to choose between receiving chemotherapy with multiple side effects versus no chemotherapy and shorter lifespan. Elsevier Mosby. in Fazio VW. feelings. mortality rates. specialties. the patient is asked to rate a preference for a particular outcome under various conditions.McLeod RS: Quality-of-life measurement in the assessment of surgical outcome. American College of Surgeons SESAP 13 Category 17 Item 27 Utility rating scale (A) Time trade-off technique (B) SF-36 Health Status Survey (C) Both (D) Neither page 1368 . Alternatively. 2005. Outcomes of interest to patients more frequently include postoperative quality of life measurements. For example. and length of stay. diseases.American College of Surgeons SESAP 13 Critique for Category 17 Item 27 Surgical outcome measurements allow for comparison of surgeons. Delaney CP (eds): Current Therapy in Colon and Rectal Surgery. feelings. It is a functional outcome measure. specialties. In the utilitarian measurement model. Outcomes of traditional interest to surgeons include postoperative complication rates. The SF-36 Health Status Survey is one of the many validated instruments used to measure quality of life. ed 2. the patient is asked to rate a preference for a particular outcome under various conditions. Both the SF-36 instrument and the time trade-off technique have proved to be valid and reliable in measuring different aspects of quality of life. 1999 page 1369 . Adv Surg 33:293309. mortality rates. Various instruments exist to measure outcomes in surgery. They are complex to administer and to interpret. There are multiple instruments available based on various models. pp 567-570 2. patients. survival rates (overall and disease-specific). and therapies.McLeod RS: Quality-of-life measurement in the assessment of surgical outcome. Measuring quality of life can be challenging. REFERENCES 1.Church JM: Outcomes analysis and measurement of quality of life. These instruments allow patients to choose between two options for health care. in a time-tradeoff technique the patient is asked to choose between receiving chemotherapy with multiple side effects versus no chemotherapy and shorter lifespan. the psychometric approach to measuring quality of life uses questionnaires to measure patients’attitudes. requiring significant time and effort from patients and researchers. or perceptions about various aspects of their life. Philadelphia. Elsevier Mosby. Church JM. in Fazio VW. American College of Surgeons SESAP 13 Category 17 Item 28 Validated and reliable (A) Time trade-off technique (B) SF-36 Health Status Survey (C) Both (D) Neither page 1370 . feelings. The SF-36 Health Status Survey is one of the many validated instruments used to measure quality of life. mortality rates. the patient is asked to rate a preference for a particular outcome under various conditions. requiring significant time and effort from patients and researchers.McLeod RS: Quality-of-life measurement in the assessment of surgical outcome. Alternatively. REFERENCES 1. Outcomes of interest to patients more frequently include postoperative quality of life measurements. Church JM. 2005.Church JM: Outcomes analysis and measurement of quality of life.American College of Surgeons SESAP 13 Critique for Category 17 Item 28 Surgical outcome measurements allow for comparison of surgeons. 1999 page 1371 . In the utilitarian measurement model. in a time-tradeoff technique the patient is asked to choose between receiving chemotherapy with multiple side effects versus no chemotherapy and shorter lifespan. There are multiple instruments available based on various models. in Fazio VW. diseases. and therapies. ed 2. Adv Surg 33:293309. survival rates (overall and disease-specific). and length of stay. pp 567-570 2. Both the SF-36 instrument and the time trade-off technique have proved to be valid and reliable in measuring different aspects of quality of life. Delaney CP (eds): Current Therapy in Colon and Rectal Surgery. For example. They are complex to administer and to interpret. These instruments allow patients to choose between two options for health care. Measuring quality of life can be challenging. Various instruments exist to measure outcomes in surgery. specialties. Elsevier Mosby. It is a functional outcome measure. the psychometric approach to measuring quality of life uses questionnaires to measure patients’attitudes. Outcomes of traditional interest to surgeons include postoperative complication rates. Philadelphia. or perceptions about various aspects of their life. patients.
Copyright © 2024 DOKUMEN.SITE Inc.